Text
                    Д. Г. МАКСИМОВ
КУРС
ЭЛЕКТРОТЕХНИКИ
ВОЕННОЕ ИЗДАТЕЛЬСТВО
МИНИСТЕРСТВА ОБОРОНЫ СОЮЗА ССР
МОСКВА-1В6В

Д. Г. МАКСИМОВ КУРС ЭЛЕКТРОТЕХНИКИ ИЗДАНИЕ ТРЕТЬЕ ( переработанное ) ВОЕННОЕ ИЗДАТЕЛЬСТВО МИНИСТЕРСТВА ОБОРОНЫ СОЮЗА ССР МОСКВА —1958
Д. Г. Максимов. Курс электротехники В книге изложены физические основы электротех- ники, теория постоянного и переменного токов, даны основные сведения о химических источниках электри- ческой энергии, электрических машинах и электриче- ских измерениях. Все теоретические положения и рас- четные формулы сопровождаются большим количеством практических примеров с решениями.
ПРЕДИСЛОВИЕ Настоящая книга, выходящая третьим изданием, существенно переработана по сравнению с ее вторым изданием. Книга рассчитана на читателей, знакомых с математикой и физикой в объеме средней школы, и соответствует программам средних специальных школ, где электротехника является одной из основных общетехнических дисциплин. При подготовке третьего издания книги автором были учтены отзывы и замечания рецензентов и обучающихся. Автор выражает глубокую благодарность всем лицам, критика которых способствовала выявлению недостатков предыдущего издания книги и, следовательно, улучшению настоящего ее издания. Отзывы и пожелания просьба направлять по адресу: Москва, Тверской бульвар, 18, Военное издательство. Автор

БУКВЕННЫЕ ОБОЗНАЧЕНИЯ НЕКОТОРЫХ ЭЛЕКТРИЧЕСКИХ ВЕЛИЧИН (по ГОСТ 1494—49) Наименование величины К Обозначения главные запасные Емкость электрическая Заряд электрона Индуктивность; коэффициент самоиндукций Индуктивность взаимная; коэффициент взаим- ной индукции Индукция магнитная Количество электричества; заряд электриче- ский Коэффициент связи Коэффициент электрического сопротивления температурный Мощность активная Мощность кажущаяся Мощность мгновенная Мощность реактивная Намагниченность Напряжение электрическое Напряженность магнитного поля Напряженность электрического поля Плотность тока Поток магнитный Потокосцепление магнитное; магнитный поток полный Проводимость активная электрическая Проводимость полная электрическая Проводимость реактивная электрическая Проводимость удельная электрическая Проводимость в комплексном выражении пол- ная электрическая Проницаемость диэлектрическая Проницаемость магнитная Разность фаз тока и напряжения Сила намагничивающая (сила магнито-движу- щая) Сила электродвижущая Сопротивление активное электрическое Сопротивление реактивное электрическое Сопротивление полное электрическое Сопротивление удельное электрическое Сопротивление в комплексном выражении пол- ное электрическое Ток (величина тока) Фаза начальная Частота Частота угловая Число витков обмотки С е L М В k а Р S р Q J и. и н Е Ф Ф g У ’ b Y Y 8 Р' <Р F Е, е г X 2 Р Z /, i Ф f W я рл Рь Рк Рг е j “> Р п 5
УСЛОВНЫЕ ГРАФИЧЕСКИЕ ИЗОБРАЖЕНИЯ, ПРИМЕНЯЕМЫЕ В ЭЛЕКТРИЧЕСКИХ СХЕМАХ Наименование Условные графические изображения Провод соединительный Провода пересекающиеся, электрически не со- единенные ——4—— Провода пересекающиеся, электрически со- единенные | Ответвление провода —г- Постоянный ток 1 1 Переменный ток Переменный ток трехфазной\ети Постоянный и переменный ток Выключатель однополюсный Выключатель двухполюсный Переключатель однополюсный на два поло- г жения Переключатель двухполюсный на два поло- М, жения Сопротивление нерегулируемое, общее обозна- ? ? чение (активное, омическое) Сопротивление регулируемое, общее обозна- чение 4 6-
Продолжение Наименование Сопротивление полное Зажимы Заземление х Соединение провода с корпусом Предохранитель плавкий Высокое напряжение Соединение обмоток „звездой1* . Соединение обмоток „треугольником1* Катушка индуктивности Дроссель низкой частоты Трансформатор низкой частоты Электромагнит Конденсатор постоянной емкости. Конденсатор переменной емкости Условные графические изображения 77777777 $1 7
Продолжение Наименование Условные графические изображения Осветительная лампа Неоновая лампа Нагревательный элемент Сухой выпрямитель Термоэлемент с прямым подогревом Вольтметр Амперметр Химический источник электрической энергии Батарея химических источников электриче- ской энергии Генератор электрической энергии Илллл|-* © *^||||||||=* 0—0
ВВЕДЕНИЕ L Выдающиеся деятели электротехники XIX века Электротехника — наука, изучающая вопросы, связанные с производством (генерированием), передачей на расстояние и потреблением электрической энергии. Электрическая энергия нашла самое широкое применение в нашей жизни. Это объясняется тем, что ее можно относи- тельно легко- получить в результате преобразования других ви- дов энергии, например, механической, тепловой, химической, весьма быстро и экономично передавать на большие расстояния и рационально использовать. Начало развития электротехники относится ко второй поло- вине XVIII и первой половине XIX веков. Весьма важную роль в деле развития электротехники сыграли научные труды гениального русского ученого М. В. Ло- моносова (1711—1765). Работы М. В. Ломоносова в области исследования атмосфер- ного электричества на сотню лет опередили научную мысль того времени. В своем знаменитом трактате «Слово о явлениях воз- душных, от электрической силы происходящих» он впервые объ- яснил электрическую сущность грозовых явлений. Ломоносову принадлежат также работы по физике, химии, астрономии, географии, минералогии, метеорологии. М. В. Ло- моносов открыл один из важнейших законов природы — закон сохранения массы и движения. В своих фундаментальных работах по естествознанию М. В. Ломоносов развивал и отстаивал учение о материальности мира и его познаваемости. Материалистический подход к объ- яснению физических явлений природы, и в частности электри- ческих, оказал плодотворное влияние на развитие научной мысли XIX века. Будучи пламенным патриотом, М. В. Ломоносов всемерно стремился развивать русскую науку и культуру и неустанно бо- ролся с «неприятелями наук российских». Наш народ по праву назвал гениального ученого М. В. Ло- моносова отцом русской науки. 9
Большая заслуга в исследовании электрических явлений при- надлежит также работавшему вместе с М. В. Ломоносовым рус- скому академику Г. В. Рихману (1711—1753). Рихман создал первый электрический прибор — «электрический указатель», при помощи которого можно было производить количественные изме- рения электричества. Этот прибор, напоминавший современный электроскоп, использовался Ломоносовым и Рихманом при изуче- нии атмосферного электричества. Русский ученый академик Ф. У. Эпинус (1724—1802) изобрел конденсатор и простейший прибор для получения электричества, названный «электрофором». Заслуга Эпинуса заключается также в том, что он впервые высказал мысль о связи между электриче- скими и магнитными явлениями. Итальянский физик А. Вольта (1745—1827) изобрел в 1800 г. первый источник гальванического тока и этим положил начало внедрению в электротехнику химических источников электриче- ской энергии. Выдающийся русский изобретатель академик В. В. Петров (1761 —1836) сконструировал первую мощную по тому времени батарею химических источников электрической энергии и с по- мощью ее проделал ряд важнейших опытов по электролитиче- скому, тепловому, световому и физиологическому действию элек- трического тока. В 1803 г. В. В. Петров открыл явление электриче- ской дуги и предсказал возможность использования ее для электрического освещения, электрической сварки, получения металлов из их окислов и т. д. Ряд важнейших электротехнических проблем, выдвинутых Петровым, не мог быть практически применен при жизни, уче- ного, так как в то время еще нельзя было получить достаточного количества электроэнергии. В. В. Петров впервые применил на практике изолированные провода, что сыграло существенную роль в развитии электро- техники. Наш народ по праву называет академика В. В. Петрова пер- вым русским электротехником. Немецкий физик Г. С. Ом (1787—1854) открыл в 1827 г. один из важнейших законов электротехники, названный з а ко- ном О м а. Замечательный английский физик М. Фарадей (1791—1867) открыл в 1831 г. явление электромагнитной ин- дукции, сыгравшее важную роль в развитии электротехники. Талантливый русский изобретатель П. Л. Шиллинг (1786— 1837) первый в мире применил электрическую энергию в тех- нике связи. В 1832 ,г. он сконструировал электромагнит- ный телеграфный аппарат и осуществил первую теле- графную линию связи. 7 10
Большой вклад в электротехнику внес знаменитый русский ученый академик Э. X. Ленц (1804—1865), открывший два важ- нейших закона электротехники. Первый закон, открытый Э. X. Ленцем в 1833 г. и вошедший в электротехнику под названием правила Ленца, устанав- ливает связь между электрическими и магнитными явлениями как двумя сторонами единого электромагнитного явления. При- менив этот закон, Ленц в 1838 г. установил одно из важнейших положений практической электротехники — принцип обра- тимости электрических машин. Второй закон, открытый Ленцем в 1844 г., устанавливает за- висимость между количеством тепла, выделяемого током в про- воднике, и величиной этого тока. Этот закон вошел в электро- технику под названием закона Ленца — Джоуля, так как одновременно с Ленцем он был открыт английским ученым Джоулем. Большие заслуги в области развития электротехники имеет замечательный русский изобретатель академик Б. С. Якоби (1801—1874). В 1834 г. он сконструировал первый электро- двигатель постоянного тока и успешно применил его на практике. В 1837 г. Якоби изобрел гальванопластику и этим положил начало развитию новой весьма важной отрасли прак- тической электротехники. Немецкий физик Г. Р. Кирхгоф (1824—1887) открыл в 1845 г. законы протекания тока в электрических цепях (з а к о и ы Кирхгофа). Выдающийся английский ,ученый Д. К. Максвелл (1831 — 1879) впервые изложил основы теории электромагнитного поля. В 1873 г. он опубликовал «Трактат об электричестве и магне- тизме», где изложил свою электромагнитную теорию света. Плодотворную работу в области развития Теоретической и практической электротехники провел известный русский ученый, профессор Московского университета А. Г. Столетов (1839— 1896). В 1872 г. он установил зависимость между намагничиванием ферромагнитного вещества и намагничивающей силой, а в 1889 г. открыл я вл е- ние фотоэффекта и сконструировал прибор, послуживший прототипом современных фотоэлементов. Замечательный русский изобретатель П. Н. Яблочков (1847— 1894) изобрел в 1875 г. первый практически пригодный электро- осветительный прибор, получивший название свечи Яблоч- кова. Работая над проблемой питания электрических свечей пере- менным током, Яблочков впервые в мире выдвинул и осуществил идею трансформирования переменного тока. Его индукционная катушка, применяемая для «дробления све- 11
та», — прототип современных трансформаторов переменного тока. Одновременно с П. Н. Яблочковым вопросами электрического освещения занимался русский изобретатель А. Н. Лодыгин (1847—1923). После многочисленных опытов он в 1874 г. создал первую в мире электрическую лампочку накалива- ния — прототип современных электрических ламп. Русский изобретатель лаборант Московского университета И. Ф. Усагин (1855—1919) сконструировал в 1882 г. первый мощный трансформатор переменного тока. Изобре- тение Усагина способствовало широкому внедрению в электро- технику переменного тока и разрешению проблемы передачи электрической энергии на большие расстояния. Известный русский изобретатель М. О. Доливо-Доброволь- ский (1862—1919) сконструировал впервые в электротехнике в 1891 г. трехфазную систему переменного тока, которая до сих пор является стандартной не только в СССР, но й во многих других странах. Одновременно с этим Доливо-Добровольский изобрел трех- фазный асинхронный электродвигатель перемен- ного тока, наиболее распространенный в современной электро- технике. . Немецкий физик Г. Р. Герц (1857—1894) экспериментально доказал существование электромагнитных волн и этим подтвердил теоретические выводы, сделанные Максвеллом. Выдающийся русский ученый А. С. Попов (1859—1906) в 1896 г. в п е р в ы е в мире осуществил радиосвязь и этим положил начало развитию радиотехники. Советский на- род гордится тем, что наша страна является родиной радио — этого замечательного изобретения, играющего огромную роль в жизни человеческого общества. Плеяду прославленных русских электротехников XIX века можно дополнить именами многих других изобретателей и уче- ных. Например, А. Т. Болотов (1738—1833) положил начало применению электричества в медицине (электротерапия). Ф. А. Пироцкий (1845—1898) впервые претворил в жизнь идею передачи электроэнергии на расстояние. Д. А. Лачинов (1842— 1902) дал теоретическое обоснование и основы расчета линий электропередачи и внес ряд ценных предложений в области практической электротехники. В. Н. Чиколев (1845—1898) изо- брел дуговую лампу с дифференциальным регулятором, Н. Г. Славянов (1854—1897) положил начало промышленной электротермии, Н. А. Умов (1846—1915) написал ряд научных работ, содействовавших развитию учения об электромагнетизме, П. Н. Лебедев (1866—1912) впервые установил явление давле- ния света и этим способствовал окончательному утверждению электромагнитной теории света. ? 12
Русская наука XVIII и XIX веков развивалась в очень тяже- лых условиях. Господствующие классы царской России не ве- рили в творческие силы русского народа и раболепно преклоня- лись перед Западом. В правительственных и культурно-просвети- тельных учреждениях было засилье иностранцев, которые часто сознательно тормозили развитие русской науки и куль- туры. Однако, невзирая на все эти трудности, русская наука XVIII и XIX веков не только не отставала от зарубежной, но и шла во многом впереди нее. Великий русский народ выдвинул из своей среды славную плеяду ученых и изобретателей, которые внесли неоценимый вклад в сокровищницу науки. В частности, русские ученые XVIII и XIX веков сыграли большую роль в развитии электротехники, обогатив ее рядом важнейших открытий и изобретений. II. Советская электротехника Великая Октябрьская социалистическая революция создала все условия для мощного развития производительных сил на- шего народа. В первые же годы советской власти В. И. Ленин обратил особое внимание на проблему электрификации страны. По ука- занию В. И. Лепина в 1920 г. была создана Государственная комиссия по электрификации России, которая разработала пер- спективный план развития электрификации страны (план ГОЭ Л РО). Только в условиях советского общественного строя смог за- родиться и осуществиться такой смелый по замыслу, научно обоснованный план снабжения страны электрической энергией. Ленинские слова: «Коммунизм — это есть Советская власть плюс электрификация всей страны» — были с энтузиазмом вос- приняты советским народом. Уже к концу 1935 г. план ГОЭЛРО был перевыполнен в два с половиной раза. К этому времени в СССР были сооружены мощные гидроэлектрические станции на Волхове, Свири, Днепре и ряд тепловых электрических станций большой мощности, на- пример, Шатурская, Каширская, Зуевская и др. Густая сеть линий электропередач высокого напряжения по- крыла к этому времени все промышленные районы нашей стра- ны. С каждым днем все глубже и глубже внедрялась электриче- ская энергия во все отрасли бурно развивающейся социалисти- ческой экономики. В результате успешного выполнения довоенных пятилеток наша страна была превращена в могучую индустриально-колхоз- ную державу По многим техническим показателям СССР вы- шел на первое место в мире. 13
Наглядным показателем индустриализации СССР за годы довоенных пятилеток может служить рост потребления электро- энергии. Если в 1913 г. выработка электроэнергии составляла 1,9 миллиарда киловатт-часов, то в 1928 г. было выработано 5, в 1932 г.— 13,5, в 1937 г. — 36,2 и в 1940 г. — 48,3 миллиарда киловатт-часов, т. е. она возросла примерно в 25 раз по сравне- нию с 1913 г. — годом наиболее высоких технических показате- лей дореволюционной России. В годы Великой Отечественной войны был нанесен большой урон экономике Советского Союза и в особенности его электро- энергетике. Было разрушено 50 крупных электростанций, в том числе Днепровская и Свирская, и более 10 тысяч зданий, при- надлежащих электрическим станциям и подстанциям, уничто- жено около 10 тысяч километров высоковольтных линий электро- передач и много другого электрооборудования. Враги Советского Союза надеялись на то, что советскому на- роду потребуется минимум 20 лет, чтобы восстановить экономику страны. Однако они жестоко цросчитались, так как не учли преимуществ советского общественного и государственного строя перед капиталистическим. В ходе самой войны уже шла интенсивная работа на востоке страны по созданию новой энергетической базы. За годы войны мощность электростанций Урала выросла более чем в 2 раза, Кузбасса — в 1,7 раза, Караганды — в 4,1 раза, Узбекистана — в 1,9 раза. В послевоенные же годы советский народ под руководством Коммунистической партии сумел не только быстро ликвидиро- вать тяжелые последствия войны, но и шагнуть далеко вперед по пути дальнейшего подъема социалистической экономики. К 1950 г. было закончено восстановление электростанций, разрушенных во время войны, и создан ряд новых крупных электростанций. Вы- работка электроэнергии в СССР превысила уровень 1940 г. на 87%'. По производству электроэнергии СССР вышел на первое место в Европе и на второе в мире. В дальнейшем производство электроэнергии в СССР бурно возрастало с каждым годом. Так, например, в 1951 г. было выработано 104, в 1952 * г.— 117, в 1953 г.— 133, в 1954 г.— 148, в 1955 г.— 166, в 1956 г.— 192 и в 1957 г.— 210 миллиардов киловатт-часов энергии. В 1954 г. в СССР была пущена в эксплуатацию первая в мире атомная электростанция мощностью в 5000 кет. Таким образом, атомная энергия впервые в истории человечества была применена в промышленных целях в СССР и стала служить не войне, не смерти, не уничтожению культурных ценностей, а делу мирного созидательного труда. XX съезд Коммунистической партии Советского Союза в своих директивах по пятилетнему плану развития СССР на 1956— 1960 гг. наметил'план дальнейшего, еще более мощного подъема 14
социалистической экономики и, в частности, развития электрифи- кации страны. Директивами XX съезда партии предусмотрено увеличение общей мощности турбинных электростанций к концу шестой пя- тилетки примерно в 2,2 раза, гидроэлектрических станций — в 2,7 раза, увеличение мощности атомных электростанций до 2—2,5 миллионов киловаТт и выработки электроэнергии до 320 миллиардов киловатт-часов. Задачи, поставленные партией и правительством СССР перед советским народом, успешно им выполняются. В 1957 г. уже вступила в строй пущенная на полную мощность Куйбышевская ГЭС, которая в настоящее время является самой мощной элек- тростанцией в мире. Мощность всех ее двадцати гидрогенерато- ров составляет 2,1 миллиона киловатт. В средний по водности год эта станция будет вырабатывать примерно 10 миллиардов киловатт-часов энергии. В конце шестой пятилетки вступит в строй строящаяся ныне Сталинградская ГЭС. Она будет развивать полную мощность, равную 2,31 миллиона киловатт, и вырабатывать в год несколько более 10 миллиардов киловатт-часов энергии. Одновременно с волжскими гидроэлектростанциями развер- нуто строительство ряда других мощных гидроэлектростанций, которые будут введены в строй в шестой пятилетке. Среди них особое внимание привлекает к себе Братская гидроэлектрическая станция, строящаяся ныне в Сибири. Полная мощность этой гидроэлектростанции будет доведена до 3 миллионов 200 тысяч киловатт. В шестой пятилетке предусмотрено сооружение единой энер- гетической системы с линией электропередачи напряжением 400 киловольт путем объединения Куйбышевской и Сталинград- ской гидроэлектростанций с Центральной, Южной и Уральской энергосистемами. Благодаря широкому внедрению электрической энергии в про- мышленность, сельское хозяйство, транспорт и домашний быт многие районы СССР стали районами сплошной электрификации. Сельскохозяйственный труд в нашей стране постепенно превра- щается в разновидность индустриального труда, и таким образом постепенно стирается противоположность между городом и де- ревней. В настоящее время СССР занимает второе место в мире по выработке электроэнергии. Перспективным планом развития электрификации СССР намечено в ближайшие десять — пятна- дцать лет довести выработку электроэнергии в год примерно до 800—900 миллиардов киловатт-часов. И нет никакого сомнения в том, что в ближайшие годы СССР догонит и перегонит США в выработке электроэнергии в год и прочно завоюет первое ме- сто в мире. 15
Передовая советская наука достигла в наши дни такого вы- сокого уровня развития, что по многим вопросам значительно опередила науку самых развитых капиталистических стран. Удачный запуск искусственных спутников земли, постройка в кратчайший срок самой мощной в мире Куйбышевской гидро- электростанции, постройка первой в мире атомной электростан- ции и т. д.— все это является наглядным доказательством роста советской науки и техники. В настоящее время по уровню развития науки и техники, по количеству высококвалифицированных кадров специалистов, по оснащенности народного хозяйства самой разнообразной техни- кой Советский Союз занимает одно из первых мест в мире. Бла- годаря этому самые сложные производственно-технические проб- лемы могут решаться теперь нашими учеными и техниками, нашей промышленностью самостоятельно, своими собственными силами. Впереди еще много нерешенных задач. Внимание Коммуни- стической партии, Советского правительства, всего советского на- рода направлено на то, чтобы как можно успешнее решить эти задачи/Партия и народ, полные глубокой веры в свои созида- тельные силы, уверенно идут по пути дальнейшего развития и упрочения социалистического хозяйства, по пути строительства коммунизма.
ЧАСТЬ ПЕРВАЯ ФИЗИЧЕСКИЕ ОСНОВЫ ЭЛЕКТРОТЕХНИКИ ГЛАВА I ЭЛЕКТРИЧЕСКИЕ СВОЙСТВА ВЕЩЕСТВА § 1. МАТЕРИЯ И ЕЕ ДВИЖЕНИЕ Марксистский философский материализм исходит из того, что мир по своей природе материален, т. е. природа во всем ее бес- конечном разнообразии есть проявление единой, вечно суще- ствующей и движущейся материи. В. И. Ленин дает такое философское определение материи: «Материя есть философская категория для обозначения объек- тивной реальности, которая, дана человеку в ощущениях его, ко- торая копируется, фотографируется, отображается нашими ощу- щениями, существуя независимо* от них» (В. И. Ленин. Соч., т. 14, стр. 117). Следовательно, материя в философском смысле этого слова есть понятие наиболее общее, «охватывающее весь мир в целом, во всём его бесконечном многообразии». Философское понятие материи не следует отождествлять с общеупотребительным в естествознании понятием вещества. Современная наука, исходя из диалектического материализ- ма, установила, что вещество не является единственно возмож- ной формой материи и что наряду с веществом существуют иные виды материи — физические поля. К последним относятся: электромагнитное поле, поле тяготения (гравита- ционное поле) и ядер п ое поле, имеющееся в ядрах атомов. В частности, свет является особым видом материи, и он не менее материален, чем любые вещественные тела, состоящие из атомов, молекул и т. д. Материя обладает бесконечным разнообразием свойств, не- которых основным является ее движение. Материя без движения так же немыслима, как и движение без материи. Движение материи осуществляется в самых разнообразных формах, начиная от простейших — механического движения фи- зических тел — и кончая ее сложнейшими формами — органиче- £—1377* 17
ской жизнью природы и сознательной деятельностью человека. Следовательно, под движением материи в философском смысле этого слова надо понимать не только простое механическое пере* мещение физических тел, но изменяемость материи вообще, пере* ход ее от одних форм существования к другим. Так как материя не исчезает и не создается вновь, то и дви- жение материи, как внутренне присущий ей способ существова- ния, вечно и неуничтожимо. Движущаяся материя существует в Пространстве и вре- мени. Было время, когда в науке господствовало ньютоновское представление о пространстве и времени, согласно которому под пространством понималась абсолютная пустота, заполненная физическими телами, а время считалось чем-то таким, что течет само по себе независимо от физических процессов в природе. Диалектический материализм отверг идеалистическое пред- ставление Ньютона о пространстве и времени, указав, что про- стра нство и время — объективные формы суще- ствования материи, проявляющиеся в протяженности материальных тел и длительности физических процессов. Всякий материальный объект, будь то физическое веществен- ное тело или такой вид материи, как электромагнитное поле, об- ладает массой. О величине массы всякого материального объекта судят по его инертности, которую он проявляет при своем движении. Этой инертностью обладает и световой поток как некоторое ма- териальное образование, а поэтому свет, как и всякое веществен- ное тело, обладает массой. Установлено, что относительно большие по своим размерам физические тела, называемые обычно макротелами, имеют массу, независимую от скорости движения их в пространстве. Что же касается мельчайших материальных частиц, называемых обычно микрочастицами, то у них масса зависит от скорости их движения. В отличие от вещественных микрочастиц «элементарная ча- стица» электромагнитного поля, например света — фотон, не об- ладает «массой покоя», и присущая ему масса — это «масса движения». Это значит, что свет как вид материи не существует в состоянии покоя. Всякое прекращение движения света связано с исчезновением его, с качественным превращением его в иные виды материи и иные формы материального движения, напри- мер атомно-молекулярное (тепловое) движение вещества. По- следнее явление мы можем наблюдать, например, в случае преграждения движения света с помощью веществ, интенсивно поглощающих световые лучи. Способность материи к самодвижению, к видоизменению форм своего движения ха-
растеризуется энергией, присущей каждой форме материального движения. Энергии в чистом виде, т. е. независимой от материи и ее движения, в природе нет и быть не может, так как энергия есть мера движения материи. О том, как изменяется качественно и количественно данная форма материального движения, мы -можем судить по измене- ниям энергии, присущей данному движению. Так как в природе существуют качественно различные формы движения материи, то, естественно, должны существовать и ка- чественно различные виды энергии. Например, при механи- ческом движении материальных тел проявляется механиче- ская энергия, при химической реакции — химическая энергия, при молекулярно-атомном движении — тепловая энергия, при ядерных реакциях внутри атомов — атомная энергия, при электромагнитной форме движения материи — электромагнитная энергия и т. д. Важнейший закон природы — закон сохранения энергии •— свидетельствует о том, что при всяких физико-химических про- цессах, сопровождаемых переходом одниЯ видов энергии в дру- гие, общее количество энергии сохраняется неизменным. А так как энергия есть мера движения материи, то этот закон является частным случаем более общего закона природы — закона сохра- нения материи и ее движения. Современная наука установила, что между массой матери- альных тел и энергией, присущей им, существует строго опреде- ленная связь, а именно — всякое изменение массы материального тела связано с изменением его энергии. Если энергия тела убывает, то в соответствии с этим убывает масса тела, и, наоборот, с ростом массы тела растет его энергия. Необходимо особо подчеркнуть, что убыль массы материаль- ной частицы при отдаче ею энергии нельзя рассматривать как превращение материи в энергию, что безуспешно пытаются дока- зать некоторые физики-идеалисты. Надо помнить, что при всякого рода превращениях одной формы движения материи в другую сохраняется неизменным общее количество как массы матери- альных объектов, так и присущей им энергии. Может изменяться только- вид массы'и в соответствии с этим и вид энергии. Среди разнообразнейших форм движения материи особый ин- терес для электротехники представляет электромагнитная фор- ма движения материи, называемая электромагнитным явле- нием. Мерой всякого электромагнитного движения является прису- щая ему электромагнитная энергия. Носителем этой энергии яв- ляется электромагнитное поле как некоторый вид материи. Хотя электромагнитное поле как некоторый вид материи ка- чественно отличается от вещества, однако, как мы увидим далее, 2* 19
Ойо может преобразовываться в вещество и, наоборот, возникать за счет вещества. Взаимопревращаемость вещества в поле и обратно подтверж- дает однощз основных положений диалектического материализ- ма — о всеобщей связи вещей и явлений природы. Связь элек- тромагнитного поля и вещества, свидетельствует об общности свойств этих двух видов материи. Установлено, что свет (электромагнитное поле) как некото- рый вид материн обладает двоякой, противоречивой при- родой. В одних случаях он обнаруживает непрерывность, проявляю- щуюся в волновом характере движения, а в других — прерыв- ность (дискретность), проявляющуюся в потоке движущихся «элементарных» частиц электромагнитного поля — фотонов, об- ладающих квантами (порциями) энергии. Волновые свойства света наиболее резко проявляются при его распространении, а дискретность (прерывность) — при излучении и поглощении его разного рода веществами'. Изучение глубокой взаимосвязи между светом и веществом позволило установить, что и вещественные микрочастицы пред- ставляют по своей природе единство противоречивых свойств — прерывности и непрерывности. Согласно представлениям совре- менной физики всякая вещественная микрочастица, помимо своих прерывных свойств, обладает еще явно выраженными волновыми свойствами. Например, поток таких вещественных ча- стиц, как электроны, обладает волновыми свойствами, аналогич- ными волновым свойствам светового потока. На использовании волновых свойств электронного потока основано устройство элек- тронных микроскопов, широко применяемых в науке и технике. Электромагнитное поле и вещество как две разновидности материи находятся в тесном взаимодействии друг с другом. Под действием электромагнитного поля на вещество в последнем возникают разнообразнейшие формы материального движения, например тепловое движение, химическое, механическое и др. Электромаг- нитное поле, действуя на электропроводящие вещества, напри- мер металлы, заставляет перемещаться находящиеся в них элек- трические заряды. Явление направленного, упорядоченного движения электриче- ских зарядов в проводящих веществах, возникающее под дей- ствием на них электромагнитного поля, называется электриче- ским током. Когда в электропроводящих веществах возникает электриче- ский ток, то одновременно там происходят и иные физические процессы, например тепловые, химические и т. д. Следовательно, в общем случае явление электрического тока — это сложный комплекс разнообразнейших физических процессов. Однако под электрическим током в узком смысле этого слова'" подразуме- 20
вается направленное и упорядоченное движение электрических зарядов в электропроводящих веществах, происходящее под влиянием сил электромагнитного поля. В этом смысле мы и бу- дем в дальнейшем применять термин «электрический ток». Чтобы понять природу электрического тока как некоторой своеобразной формы материального движения, необходимо озна- комиться с теми элементарными вещественными частицами, ко- торые служат носителями электрических зарядов. Для этого рас- смотрим основные положения современной электронной теории строения вещества. § 2. ХИМИЧЕСКИЕ ЭЛЕМЕНТЫ Все физические тела природы построены из разновидности материи, называемой веществом. Вещества подразделяются на две основные группы — веще- ства простые и сложные. Сложными веществами называются такие вещества, которые путем химических реакций могут быть разложены на другие, более простые вещества. В отличие от сложных простыми веществами назы- ваются такие, которые химическим путем не могут быть разло- жены на еще более простые вещества. Примером сложного вещества может служить вода, которая путем химической реакции может быть разложена на два дру- гих, более простых вещества — водород и кислород. Что же касается последних двух, то они химическим путем уже не могут быть разложены на более простые вещества, а поэтому являются простыми веществами, или, иначе, химическими элементами. В первой половине XIX века в науке существовало предпо- ложение, что химические элементы являются неизменными веще- ствами, не имеющими общей связи друг с другом. Однако вы- дающийся русский ученый Д. И. Менделеев (1834—1907) впер- вые в 1869 г* выявил диалектическую связь химических элемен- тов, показав, что качественная характеристика каждогоизнихнаходится в зависимостиот его количественной характеристики — атомного веса. Изучая свойства химических элементов, Д. И. Менделеев подметил, что свойства их периодически повторяются в зависи мости от их атомного веса. Эту периодичность он отобразил в форме таблицы, вошедшей в науку под названием «Периоди- ческая система элементов Менделеева». Своим открытием периодического закона химических элемен- тов Д. И. Менделеев положил начало новой эпохе в развитии естествознания. Ниже приведена современная периодическая таблица химиче- ских элементов Менделеева, содержащая сто один элемент. Три- надцать из них, в том числе девять элементов так называемой 21
Периодическая таблица ГРУППЫ I И 1 I П1 IV V 2 Е( О S Ош ш с 1 1 н Водород 1,008 2 L1 3 Литий 6,9 Be 4 Бериллий 9 5 В, Бор 10,8 8 с Углерод 12 7 N Азот 14 3 Na ’I Натрий 23 Mg 12 Магний 24,3 13 А1 Алюминий 27 I4 S1 Кремний 28,1 15 р Фосфор 31 4 К 18 Калий 39,1 Са 29 Кальций 40,1 SC 21 Скандий 45 22 Титан 47,9 у 23 Ванадий 51 29 Си Медь 63,5 39 Zn Цинк 65,4 31 Ga Галлий 69,7 32 Ge Германий 72,6 33 As Мышьяк 74,9 ;5 ВЬ 37 Рубидий 85,6 8Г 38 Стронций 87,6 Y 39 Иттрий 88,9 Zr 4° . Цирконий 91,2 Nb 4i Ниобий 92,9 47 Ag Серебро 107,9 48 Cd Кадмий 112,4 49 In Индий 114,8 59 Sn Олово 118,7 5i Sb Сурьма 121,8 6 CS 55 Цезий 132,9 Ва 99 Барий 137,4 La 87* Лантан 138,9 Hf 72 Гафний 178,6 Та 73 Тантал 180,9 79 Аи Золото 197,0 89 Hg Ртуть 200,6 81 Ц Таллий 204,4 82 .. РЬ Свинец 207,2 83 В1 Висмут 209 7 JY 87 Франций (223) Ва 88 Радий 226 Ас 89** Актиний (227) Th 99 Торий 232,1 Ра 91 Протактиний 231 ♦РЕДКОЗЕМЕЛЬНЫЕ Се 59 Церий 140,1 рг 59 Празеодим 140,9 Nd 89 Неодим 144,3 Pm 81 Прометий (145) Sm 82 Самарий 150,4 ТЬ 15 Тербий 159 Dy 88 Диспрозий 162,5 Но 87 Гольмий 164,9 Ег 68 Эрбий 167,2 Ти 89 Тулий 169 ** ТРАНСУРАНОВЫЕ Np 83 Нептуний (237) Ри 94 Плутоний (242) Ат 95 Америций (243) Ст 98 Кюрий (245) Вк 97 Беркелий (249) 22
Таблица 1 элементов Менделеева ЭЛЕМЕНТО В VI VII VIII 0 ♦ Не 2 Гелий 4 » О Кислород 16 9 F Фтор 19 Ne 1° Неон 20,2 16 S Сера 32,1 1’ С1 Хлор 35,5 Аг 18 Аргон 39,9 Сг 24 Xjjom Мп 25 Марганец 54,9 Ге 28 Железо 55,9 Со 27 Кобальт 58,9 N1 28 Никель 58,7 34 Se Селен 79 35 ВГ Бром 79,9 Кг Зв Криптон 83,8 Мо 42 Молибден 98 Тс 43 Технеций (99)? Ви 44 Рутений 101,7 Bh 45 Родий 102,9 Pd 48 Палладий 106,7 б.2 Те. Теллур 127,6 53 J Иод 126,9 Хе 54 Ксенон 131,3 W 74 Вольфрам 183,9 Be 75 Рений 186,3 08 76 Осмий 190,2 1г 77 Иридий 192,2 Pt 78 Платина 195,2 84 р0 Полоний (209) ”5 At Астатин (210) Вп 88 Радон (222) 92** Уран 238,1 ЭЛЕМЕНТЫ Ей 63 Европий 152 Gd 84 Гадолиний 156,9 Yb ’о Иттербий 173 Lu 71 Лютеций 175 ЭЛЕМЕНТЫ Cf 88 Калифорний (249) Еп 88 Эйнштейний 253 Гт i°° Фермий 255 Mv in Менделевий 256 23
заурановой группы (элементы, расположенные в периодической таблице за ураном), были созданы искусственно, '§ 3. АТОМЫ Согласно современным представлениям науки каждый хими* ческий элемент состоит из совокупности мельчайших материаль- ных (вещественных) частиц, называемых атомами. Атомом называется самая малая доля химического элемента, которая уже не может быть разложена химическим путем на другие, более мелкие и простые материальные частицы. Атомы различных по своей природе химических элементов* отличаются друг от друга своими физико-химическими свой* ствами, структурой, размерами, массой, атомным весом, соб* ственнойг энергией и некоторыми иными свойствами. Например, атом водорода резко отличается по своим свойствам и структуре от атома кислорода, а последний — от атома урана и т. д. Установлено, что атомы химических элементов чрезвычайно малы по своим размерам. Если условно принять, что атомы имеют шарообразную форму, то поперечники их должны быть равны стомиллионным долям сантиметра. Например, поперечник атома водорода — самого маленького атома в природе — равен одной стомиллионной доле сантиметра (10""8 см), а поперечники самых больших атомов, например атома урана, не превышают трех стомиллионных долей сантиметра (3 • 10~8 см). Следова- тельно, атом водорода во столько раз меньше шарика радиусом в один сантиметр, во сколько последний меньше земного шара. В соответствии с весьма малыми размерами атомов их масса также очень мала. Например, масса атома водорода равна т = 1,67- 10~24г. Это значит, что в одном грамме водорода со- держится примерно 6 • 1023 атомов. ‘За условную единицу измерения атомных весов химических элементов принята часть веса атома кислорода. В соответ- ствии с этим атомным весом химического элемен- та называют отвлеченное число, показываю- щее, во сколько раз вес данного химического элемента больше части веса атома кисло- 10 рода. В периодической таблице элементов Д. И. -Менделеева при- ведены атомные веса всех химических элементов (см. число, по- мещенное под названием элемента*). Из этой таблицы мы видим, что наиболее легким атомом является атом водорода, имеющий атомный вес 1,008. Атомный вес углерода равен 12,,кислорода — 16 и т. д. Что же касается более тяжелых химических элементов, то их атомный вес превышает атомный вес водорода более чем в двести раз. Так, атомный вес ртути равен 200,6, радия,— 220 24
и т. д. Чем выше порядок номера, занимаемого химическим эле- ментом в периодической системе элементов, тем больше его атомный вес. Большая часть атомных весов химических элементов выра- жается дробными числами. Это в известною мере объясняется тем, что такие химические элементы состоят из совокупности не- скольких сортов атомов, обладающих различными атомными весами, но одинаковыми химическими свойствами. Химические элементы, занимающие одинако- вый номер в периодической системе элементов, а следовательно, обладающие одинаковыми хи- мическими свойствами, но различными атом- ными весами, называются изотопами. Изотопы найдены у большинства химических элементов. Хлор имеет два изотопа, кальций — четыре, цинк — пять, олово — одиннадцать и т. д. Многие изотопы получены искусственным путем, среди них некоторые имеют большое практическое зна- чение. § 4. ЭЛЕМЕНТАРНЫЕ ЧАСТИЦЫ ВЕЩЕСТВА Долгое время считалось, что атомы химических элементов являются пределом делимости вещества, т. е. как бы элементар- ными «кирпичиками» мироздания. Современная наука отвергла эту гипотезу, устЗновив, что атом любого химического элемента представляет собой совокупность еще более мелких материаль- ных частиц, чем сам атом. Согласно электронной теории строения вещества атом любого химического элемента представляет собой систему, состоящую из центрального ядра, вокруг которого вращаются «элементар- ные» вещественные частицы, называемые электронами. Ядра атомов, согласно общепринятым взглядам, впервые вы- сказанным советским ученым Д. Д. Иваненко, состоят из сово- купности «элементарных» вещественных частиц — протонов и нейтронов. Чтобы понять строение атомов и физико-химические процессы в них, необходимо хотя бы вкратце ознакомиться с основными характеристиками элементарных частиц, входящих в состав атомов. Установлено, что электрон — это вещественная частица, обладающа ясам ы м малым наблюдае- мым в природе отрицательным электрическим зарядом. В начале XX века существовало убеждение в том, что элек- трон — это частица, имеющая строго определенную и неизмен- ную форму, объем, массу. Однако уже в 1908 г. В. И. Ленин в своей книге «Материа- лизм и эмпириокритицизм» с гениальной прозорливостью пред- 25
сказал, что электрон не так прост, как это казалось физикам XX века. «Электрон,— писал Ленин,— так же неисчерпаем, как и атом, природа бесконечна...» «...исчезает тот предел, до кото- рого мы знали материю до сих пор, наше знание идет глубже...» (В. И. Ленин. Материализм и эмпириокритицизм. Соч., т, 14, изд. 4, стр. 247 и 249). Современная наука полностью подтвердила высказанную Лениным мысль о неисчерпаемости электрона, раскрыв богатое физическое содержание этой «элементарной» частицы материи. Если условно считать, что электрон как частица имеет шаро- образную форму, то поперечник электрона должен быть равным 4-10-18 см, т. е. он меньше поперечника любого атома в десятки тысяч раз. Электрон, как и всякая иная вещественная частица, обладает массой. «Масса покоя» .электрона, т. е. та масса, кото- рой он обладает в состоянии относительного покоя, равна то = 9,1 • 10“28 г. Исключительно малая «масса покоя» электрона свидетель- ствует о том, что инертные свойства электрона проявляются исключительно слабо, а это значит, что электрон под влиянием переменной электрической силы может колебаться в простран- стве с частотой во много миллиардов периодов в секунду. Масса электрона настолько мала, что для получения одного грамма электронов их потребовалось бы взять 1027 единиц. Чтобы иметь хотя бы некоторое физическое представление об этом колоссально большом числе, приведем такой пример. Если бы можно было один грамм электронов расположить на прямой линии вплотную друг к другу, то они образовали бы цепочку длиной в четыре миллиарда километров. Масса электрона, как и всякой иной вещест- венной микрочастицы, зависит от скорости его. движения. Электрон, находясь в состоянии относительного покоя, обладает «массой покоя», имеющей механическую приро- ду, как и масса всякого физического тела. Что же касается «массы движения» электрона, увеличивающейся с ростом скорости его движения, то она электромагнитного происхожде- ния. Она обусловлена наличием у движуще- гося электрона электромагнитного поля как некоторого вида материи, обладающего мас- сой и электромагнитной энергией. Чем быстрее движется электрон, тем больше проявляются инерционные свой- ства его электромагнитного поля, тем, следовательно, больше масса последнего и соответственно электромагнитная энергия его. Так как электрон со своим электромагнитным полем составляет единую, органически связанную материальную систему, то есте- ственно, что массу движения электромагнитного поля электрона можно непосредственно приписать самому электрону. Зависимость величины массы электрона, как и всякой иной 26
микрочастицы, от скорости его движения определяется следую- щей формулой: У 1 с2 где mv—масса движущегося электрона в граммах; т0—«масса покоя» электрона в граммах; . v—скорость движения электрона в сантиметрах в секунду; с — 3 • 1010 см!сек — скорость света. Пример 1. Электрон имеет «массу покоя» то = 9,1 • 10“28г. Определить, какова будет масса электрона, если скорость его относительного движения достигнет 0,865 доли скорости света. Решение. Применяя формулу (1), находим (0,865-3. Ю10)2 (3-10^)2 9,1 - IO"28 = 18,2-10-28 г, т. е. масса электрона в этом случае возросла в 2 раза. Как было сказано выше, электрон, помимо свойств частицы, обладает и волновыми свойствами. Опытом установлено, что по- ток электронов, подобно световому' потоку, распространяется в форме волнообразного движения. Характер волнового движения электронного потока в пространстве подтверждается явлениями интерференции и дифракции электронных волн. Интерферен- ция электронов — это явление наложения электронных волн друг на друга, а дифракция электронов — это яв- ление огибания электронными волнами краев узкой щели, сквозь которую проходит электронный поток. Следовательно, электрон — это не просто частица, а «части- ца-волна», длина которой зависит от массы и скорости движе- ния электрона. Установлено, что электрон, помимо своего поступательного движения, совершает еще и вращательное движение вокруг своей оси. Этот вид движения электрона получил название «спин а» (от английского слова «с пин» — веретено). В ре- зультате такого движения электрон, кроме электрических свойств, обусловленных электрическим зарядом, приобретает еще и маг- нитные свойства, напоминая в этом отношении элементарный магнитик. В заключение отметим, что о строении электрона, о форме су: ществования материи в нем современной науке пока еще ничего неизвестно. Однако нет сомнения в том, что наука будущего су- меет распознать его внутреннее содержание, так как в природе нет непознаваемых вещей, а есть только вещи, еще непознанные, которые впоследствии будут раскрыт^ и познаны силами науки И техники. 27
Как было упомянуто выше, в состав ядер атомов входят «элементарные» вещественные частицы — протоны и нейтроны. Протон — это вещественная частица, обла- дающая положительным электрическим заря- дом, равным по абсолютной величине электри- ческому заряду электрона. Масса протона равна 1,67-10 “24 г, т. е. она примерно в 1840 раз больше «массы покоя» электрона. В отличие от электрона и протона, нейтрон не обладает электрическим зарядом, т. е. он является э л е к т р о н е й- тральной «элементарной» частицей вещества. Масса нейтрона практически равна массе протона. Электроны, протоны и нейтроны, находясь в составе атомов, взаимодействуют друг с другом. В частности, электроны и про- тоны взаимно притягиваются друг к другу как частицы, обла- дающие разноименными электрическими зарядами. Одновременно с этим электрон от, электрона и протон от протона отталкиваются как частицы, обладающие одноименными электрическими заря- дами. Взаимодействие всех этих электрически заряженных ча- стиц происходит через их электрические поля. Эти поля представляют собой особый вид материи, состоящей из совокуп- ности элементарных материальных частиц, называемых фото- нами. Каждый фотон обладает строго определенным присущим ему количеством энергии (квантом э н(е р г и и). Взаимодей- ствие электрически заряженных материальных вещественных ча- стиц осуществляется путем обмена их друг с другом фотонами. Сила взаимодействия электрически заряженных частиц обычно называется электрической силой. Нейтроны и протоны, находящиеся в ядрах атомов, также взаимодействуют друг с другом. Однако это взаимодействие их осуществляется уже не через электрическое поле, так как нейт- рон — электронейтральная частица вещества, а через так назы- ваемое ядерное поле. Это поле также представляет собой особый вид материи, со- стоящей из совокупности элементарных материальных частиц, называемых мезонами. Взаимодействие нейтронов и прото- нов осуществляется путем обмена их друг с другом мезонами. Сила взаимодействия нейтронов и протонов друг с другом назы- вается ядерно й силой. Установлено, что ядерные силы действуют в ядрах атомов в пределах исключительно малых расстояний — примерно 10"13 см. Ядерные силы значительно превосходят по своей величине электрические силы взаимного отталкивания протонов в ядре атома. Это приводит к тому, что они в состоянии не только преодолеть внутри ядер атомов силы взаимного отталкивания про< тонов, но и создать из совокупности протонов и нейтронов весьма прочные системы ядер. Устойчивость ядра каждого 28
атома зависит от соотношения двух противо- речивых сил — ядерных (взаимное притяжение протонов и нейтронов)' и электрических (взаимное отталкивание протонов). Мощные ядерные силы, действующие в ядрах атомов, способ- ствуют превращению нейтронов и протонов друг в друга. Эти взаимопревращения нейтронов и протонов осуществляются в ре- зультате выделения или поглощения ими более легких элемен- тарных частиц, например мезонов. Мезоны имеют несколько разновидностей. К ним относятся: ji-мезоны (мю-мезоны), имеющие отрицательный электри- ческий заряд, равный заряду электрона, а массу больше «массы покоя»'электрона в’ 200 раз, ^-мезоны (пи-мезоны), имею- щие положительный электрический заряд, а массу больше «мас- сы покоя» электрона в 280 раз. Существуют мезоны, являющие- ся электронейтральными частицами, мезоны, имеющие массу в 1200 раз больше «массы покоя» электрона, и ряд других мезо- нов. Опытом установлено, что мезоны способны к качественным превращениям. Например, ji-мезон, просуществовав в свобод- ном состоянии две миллионные доли секунды, превращается в элементарные материальные частицы: позитрон и два ней- трино. Эти частицы не содержались в р,-мезоне в готовом виде, они возникли в результате перехода одной формы сущест- вования материи в другую, качественно иную, форму. ' Позитрон — это вещественная частица, обладающая поло- жительным электрическим зарядом, равным по абсолютной вели- чине заряду электрона. Масса же позитрона равна массе элек- трона. Нейтрино — вещественная частица, обладающая массой, значительно меньшей, чем «масса покоя» электрона, и не имею- щая электрического заряда. В науке имеются неопровержимые доказательства существования этой материальной частицы в при* роде. Однако вследствие своей электронейтральности и исключи- тельно малой массы она до сих пор еще в опыте непосредствен- но не была наблюдаема. Опытом установлено, что при радиоактивных явлениях наб- людается испускание радиоактивными элементами так называе- мых гамма-лучей. Характерной особенностью этих лучей является то, что их фотоны обладают относительно большим ко- личеством, (квантом) энергии. Так как гамма-фотон не имеет электрического заряда, то он может относительно свободно по- дойти к ядру атома. Здесь он попадает в сферу влияния мощ- ных ядерных сил, под действием которых переходит в пару ве- щественных частиц — электрон и позитрон, прекращая свое су- ществование. Таким образом, невещественная материальная ча- стица— гамма-фотон превратилась под влиянием ядерных сил в пару вещественных частиц. Вновь «рожденный» электрон про- 29
должает свое существование, как и всякий обычный электрон. Что же касается позитрона, то продолжительности его самосто- ятельного существования весьма невелика. Спустя незначитель- ную долю секунды после возникновения он соединяется с пер- вым же встречным электроном, образуя совместно с ним неве- щественные материальные частицы — фотоны (кванты света). Здесь мы не можем привести полный перечень всех «элемен- тарных» частиц, открытых современной физикой. Однако выше- изложенные краткие сведения о некоторых из них вполне доста- точны для того, чтобы составить простейшее представление о строении вещества. В заключение отметим, что рассмотренные нами частицы на- званы элементарными потому, что они не состоят из совокупно- сти других, более простых частиц материи. Но в то же время не надо забывать, что они способньгпревращаться друг в друга, возникать за счет друг друга. Таким образом, эти частицы яв- ляются некоторыми сложными образованиями, т. е. их элемен- тарность условна. § 5. СТРОЕНИЕ АТОМОВ Простейшим по своему устройству атомом является атом во- дорода. Он состоит из совокупности только двух элементарных частиц — протона и электрона. Протон в системе атома водорода играет роль центрального ядра, вокруг которого по некоторой орбите вращается электрон. На рис. 1 схематически показана модель атома водорода. Эта Рис. 2. Атом водорода (движение электрона вокруг ядра в среднем за относительно большой* промежуток времени) Рис. 1. Схема строения атома водорода модель — только грубое приближение к действительности. Дело в том, что электрон как «частица-волна» не обладает резко от- граниченным от внешней среды объемом, А это значит, что еде- 30
Электрон Квант лучиста! . энергии Рис. 3. Электрон при пере- ходе с далекой орбиты на более близкую к ядру атома излучает квант лучи- стой энергии дует говорить не о некоторой точной линейной орбите электрона, а о своеобразном электронном облачке (рис. 2). При этом электрон чаще всего занимает некоторую среднюю линию в об- лачке, являющуюся одной из возмож- ных орбит его в атоме. Надо сказать, что и сама орбита электрона не является строго неизмен- ной и неподвижной в атоме — она то- же в силу изменения массы электрона совершает некоторое вращательное движение. Следовательно, движение электрона в атоме носит относительно сложный характер. Так как ядро атома водорода (про- тон) и вращающийся вокруг него элек- трон обладают разноименными элек- трическими зарядами, то они взаимно притягиваются. Одновременно с этим электрон, вращаясь вокруг ядра атома, развивает центробежную силу, стремящуюся удалить его от ядра. Следовательно, электрическая сила взаимного при- тяжения ядра атома и электрона и центробеж- ная сила, действующая на электрон,— силыпро- тцворечивые. При равновесии их электрон занимает отно- сительно устойчивое положение на некоторой орбите в атоме. Но так как масса электрона очень мала, то для уравновешивания силы притяжения к ядру атома он должен вращаться с громад- ной скоростью, равной примерно 6 • 1015 оборотам в секунду. Это значит, что электрон в системе атома водорода, как и во вся- ком ином атоме, движется по своей орбите с линейной ско- ростью, превышающей тысячу километров в секунду. В нормальных условиях электрон вращается в атоме водо- рода по наиболее близко расположенной к ядру орбите. При этом он обладает минимальным возможным количеством энер- гии. Если же по тем или иным причинам, например под воздей- ствием каких-либо иных материальных частиц, вторгнувшихся в систему, атома, электрон перейдет на более удаленную от ядра атома орбиту, то он уже будет обладать несколько большим ко; личеством энергии. Однако на этой новой орбите электрон обыч- но пребывает ничтожно малое время, после чего он снова воз- вращается на ближайшую к ядру атома орбиту. При этом пере- ходе он отдает излишек своей энергии в виде кванта электро- магнитного излучения — лучистой энергии (рис. 3). Чем больше получает извне энергии электрон, тем на более удаленную от ядра атома орбиту он переходит и тем большее количество электромагнитной энергии он излучает, когда воз- вращается на ближайшую к ядру орбиту. Измеряя количество энергии, излучаемой электроном при переходе с различных ор- 31
бит на ближайшую к ядру атома, удалось установить,. что эле- ктрон в системе атома водорода, как и в системе любого иного атома, может переходить не на любую произвольную орбиту, а на строго определенную в соответствии с той энергией, которую он получает под действием внешней силы. Орбиты, которые мо- жет занимать электрон в атоме, называются дозволенны- миорбитами. Так как положительный заряд ядра атома водорода (заряд протона) и отрицательный заряд электрона численно равны, то суммарный их заряд равен нулю. Это значит, что атом водоро- да, находясь в нормальном состоянии, является электроней- тральной частицей. Как мы увидим далее, это справедливо для атомов всех химических элементов: атом любого химиче- ского элемента, находящийся в нормальном со- стоянии, является электро нейтральной части- цей из-за численного равенства его положи- тельных и отрицательных зарядов. Поскольку в состав ядра атома водорода входит только од- на «элементарная» частица — протон, то так называемое массо- вое число этого ядра равно единице. Массовым числом ядра атома любого химического элемента на- зывается общее число протонов и нейтронов, входящихвсоставэтогоядра. Природный водород в основном состоит из совокупности ато- мов с массовым числом, равным единице. Однако в составе его имеется и другой сорт атомов водорода, с массовым числом, равным двум. Ядра атомов этого тяжелого водорода, называемые дейтонами, состоят из двух частиц — протона и, нейтрона. Этот изотоп водорода называется дейтерием. В природном водороде дейтерия содержится весьма незначи- тельное количество. На каждые шесть тысяч атомов легкого во- дорода (массовое число равно единице) приходится только один атом дейтерия (тяжелого водорода). За последние годы удалось получить искусственным путем еще один изотоп водорода — сверхтяжелый водород, получивший название тритий. В ядрах атома этого изотопа водорода имеются три частицы: протон и два нейтрона, связан- ных друг с другом ядерными силами. Массовое число ядра ато- ма трития равно трем, т. е. атом трития в три раза тяжелей атома легкого водорода. Хотя атомы изотопов водорода и имеют различные массы, но все же они обладают одинаковыми химическими свойствами. Например, легкий водород, вступая в химическое взаимодейст- вие с кислородом, образует с ним сложное вещество — воду. Аналогично этому изотоп водорода — дейтерий, соединяясь с кислородом, образует воду, которая в отличие от обычной воды называется тяжелой водой. Тяжелая вода находит большое 32
применение в процессе производства ядерной (атомной) энер- гии. Следовательно, химические свойства атомов зависят не от массы их ядер, а От каких-то иных количественных характери- стик. В дальнейшем мы увидим, что химические свойст- ва атомов зависят только от строения элек- тронной оболочки атома. Поскольку в атомах легкого водорода, дейтерия и трития имеется одинаковое количество электронов (по одному на каждый атом), эти изотопы имеют, одинаковые химические свойства. Химический элемент водород не случайно занимает первый номер в периодической системе элементов. Дело в том, что меж- ду номером любого элемента в периодической системе элемен- тов и величиной заряда ядра атома этого элемента существует некоторая связь. Ее можно сформулировать так: порядковый номер всякого химического элемента в периодической системе элементов численно равен положительному заряду ядра этого элемента, а следовательно, и числу вращающихся вокруг него электронов. Так как водород занимает первый номер в периодической системе элементов, то это значив, что положительный заряд яд- ра его атома равен единице и что вокруг ядра вращается один электрон. Химический элемент гелий занимает второй номер в перио- дической бистеме элементов. Это значит, что он имеет положи- тельный электрический заряд ядра, равный двум единицам, т. е. в составе его ядра должно быть два протона, а в электронной оболочке атома — два электрона. Природный гелий состоит из двух изотопов — тяжелого и легкого гелия. Массовое число тя- желого гелия равно четырем. Это значит, что в состав ядра ато- ма тяжелого гелия, помимо вышеупомянутых двух протонов, должны входить еще два нейтрона. Что же касается легкого ге- лия, то его массовое число равно трем, т. е. в состав, его ядра, помимо двух протонов, должен входить еще один нейтрон. Уста- новлено, что в природном гелии число,атомов легкого гелия со- ставляет примерно одну миллионную долю атомов тяжелого ге- лия. На рис. 4 показана схемати- чески модель атома гелия. Дальнейшее усложнение строе- ния атомов химических элементов идет за счет увеличения количества протонов и нейтронов в ядрах этих атомов и одновременно за счет уве- личения количества электронов, вра- щающихся вокруг ядер (рис. 5). Пользуясь периодической систе- мой элементов, легко определить число электронов, протонов и ней- Рис. 4. Схема строения атома гелия 3—1377 33
• Протон О Нейтрон Рис. 5. Схемы строения ядер атомов: I — гелий; 2 — углерод; 3 — кис- лород тронов, входящих в состав различных атомов. Как было упомя- нуто выше, порядковый номер химического элемента равен числу протонов, находящихся в ядре атома, и одновременно с этим числу электронов, вращающихся вокруг ядра. Что же касается атомного веса, то он приближенно равен массовому числу атома, т. е. числу вместе взятых протонов и нейтронов в ядре. Следовательно, вы- читая из атомного веса элемента число, равное порядковому номеру элемента, можно определить, какое количество нейтронов содержится в данном ядре: л = Д — 7V, где п — число нейтронов; N — порядковый номер элемента; А — атомный вес элемента. Установлено, что ядра легких химических элементов, имею- щих в своем составе поровну протонов и нейтронов, отличаются весьма большой прочностью, так как ядерные силы в них отно- сительно велики. Например, ядр'о атома тяжелого гелия отли- чается исключительно большой прочностью, так как оно состав- лено из двух протонов и двух нейтронов, связанных друг с дру- гом мощными ядерными силами. Ядра атомов более тяжелых химических элементов содержат в своем составе уже неодинаковое количество протонов и ней- тронов, поэтому их связь в ядре слабее, чем в ядрах легких химических элементов. Ядра этих элементов могут быть'относи- тельно легко расщеплены «при бомбардировке их атомными «снарядами» (нейтронами, ядрами атома гелия и т. д.). Что же касается наиболее тяжелых химических элементов, в частности радиоактивных, то их ядра отличаются настолько малой проч- ностью, что они самопроизвольно распадаются на составные ча- сти. К примеру, атомы радиоактивного элемента радия, состоя- iHgro из совокупности 88 протонов и 138 нейтронов, самопроиз- вольно распадаются, превращаясь в атомы радиоактивного эле- мента радона. Атомы же последнего в свою очередь распадают- ся на составные части, переходя в атомы других элементов. Ознакомившись вкратце с составными частями ядер атомов химических элементов, рассмотрим строение электронных оболо- чек атомов. Как известно, электроны могут вращаться вокруг ядер ато- мов только,по строго определенным орбитам. При этом они так сгруппированы в электронной оболочке каждого атома, что можно различить отдельные слои электронов. В каждом слое может находиться количество электронов, не превышающее, строго определенного числа. Так, например, в первом, ближай- шем к ядру атома электронном слое может находиться макси- 34
Рис. 6. Схема строе- ния атома неона мум два электрона, во втором — не более восьми электронов и т. д. Те атомы, у которых внешние электронные слои целиком заполнены, имеют наи б о лее устойчивую электронную оболочку. Это значит, что данный атом прочно держит все свои электроны и не нуждается в получении извне добавочного количества их. Напри- мер, атом гелия имеет два электрона, цели- ком заполняющих первый электронный слой, а атом неона имеет десять электронов, из которых первые два целиком заполняют первый электронный слой, а остальные — второй (рис. 6). Следовательно, атомы гелия и неона имеют вполне устойчивые электронные оболочки, не стремятся их как- нибудь видоизменить количественно. Такие элементы химически инертны, т’ е. не вступают в химическое взаимодействие с другими элементами. Однако большинство химических элементов имеет такие ато- мы, в которых внешние электронные слои не целиком заполне- ны электронами. Например, атом калия имеет девятнадцать электронов, из которых восемнадцать целиком заполняют пер- вые три'слоя, а девятнадцатый электрон один находится в сле- дующем, незаполненном электронном слое. Слабое заполнение электронами четвертого электронного слоя приводит к тому, что ядро атома весьма слабо удерживает самый внешний — девят- надцатый электрон, а поэтому последний может быть легко вы- рван из атома. Или, например, атом кислорода имеет восемь электронов, из которых два целиком заполняют первый слой, а остальные шесть размещены во втором слое. Таким образом, для полного завершения построения второго электронного слоя в ато- ме кислорода ему не хватает только двух электронов. Поэтому атом кислорода не только прочно удерживает свои шесть элек- тронов во втором слое, но и обладает возможностью притянуть к себе два недостающих ему электрона для заполнения своего второго электронного слоя. Этого он достигает путем химиче- ского соединения с атомами таких элементов, у которых внеш- ние электроны слабо связаны со своими ядрами. Химические элементы, атомы которых не имеют целиком за- полненных электронами внешних электронных слоев, как прави- ло, химически активны, т. е. охотно вступают в химическое вза- имодействие. Итак, электроны в атомах химичеЬких элемен- тов располагаются в строго определенном по- рядке и всякое изменение их пространственно- го расположения или количества в эле'ктрон- 3* 35
ной оболочке атома приводит к изменению фи- зико-химических свойств последнего. Равенство числа электронов и протонов в системе атома яв- ляется причиной того, что суммарный электрический заряд его равен нулю. Если равенство числа электронов и протонов в си- стеме атома нарушается, то атом становится, электрически за- ряженной системой. Атом, в системе которого нарушено равновесие разноимен- ных электрических зарядов вследствие того, что он потерял часть своих электронов или, наоборот, приобрел лишнее количе- ство их, называется ионом. Если атом теряет часть своих электронов, то он превращается в положительный ион, т. е. ион с положительным зарядом. При потере одного электрона атом превращается в однозарядный положительный ион; если он теряет два электрона, то превращается в двухзарядный по- ложительный, и т. д. Например, атом водорода, потеряв свой единственный электрон, превращается в однозарядный положи- тельный ион водорода Н+; атом цинка, потерявший два электро- на, превращается в двухзарядный положительный ион цинка Zn++ и т. д. Наоборот, если атом приобретает некоторое лишнее количество электронов, то он стано- вится отрицательным ионом. Например, атом хлора, получивший один лишний электрон, превращается В однозаряд- ный отрицательный ион хлора С1~; атом кислорода, получивший лишних два электрона, превращается в двухзарядный отрица- тельны^ ион кислорода О и т. д. Атом, превратившийся в ион, становится по отношению к внешней среде электрически заряженной системой. А это зна- чит, что атом стал обладать электрическим полем, вместе с ко- торым он составляет единую материальную систему и через это поле осуществляет электрическое взаимодействие с другими электрически заряженными частицами вещества — ионами, элек- тронами, положительно заряженными ядрами атомов и т. д. Способность разноименных ионов взаимно притягиваться друг к другу является причиной того, что они химически соеди- няются, образуя более сложные частицы вещества — моле- кулы. В заключение следует отметить, что размеры атома очень ве- лики по сравнению с размерами тех вещественных частиц, из которых они состоят. Ядро самого сложного атома вместе со всеми электронами занимает миллиардную долю объема атома. Простой подсчет показывает, что если бы удалось один кубиче- ский метр платины сжать так крепко, чтобы исчезли внутри- атомные и междуатомные пространства, то получился бы объем, равный примерно одному кубическому миллиметру. -36
§ 6. ЭНЕРГИЯ АТОМНОГО ЯДРА Согласно закону взаимосвязи массы и энергии между мас- сой и энергией любого материального тела существует строго определенная зависимость, выражаемая формулой W==mc\ (2) где W—энергия материального тела в эргах; т—масса материального тела в граммах; с = 3* 1010см/сек,— скорость света в сантиметрах в секунду. Следовательно, всякое выделение энергии материальным те- лом связано с уменьшением массы этого тела и, наоборот, вся- кое поглощение энергии материальным телом связано с увели- чением его массы. Напомним, что изменение массы материаль- ного тела" при выделении им энергии нельзя рассматривать как явление перехода массы тела (вещества) в энергию. При всяко- го рода излучениях энергии сохраняется как общее количество массы, так и общее количество энергии. Например, излучение (свет), появившееся в результате преобразования массы веще- ства, обладает массой, равной массе вещества, израсходованно- го на его образование. Рассмотрим несколько примеров, иллюстрирующих взаимо- связь массы и энергии. Как,известно, в результате многих химических реакций, на- пример горения (соединения тел с кислородом), выделяется не- которое количество тепловой энергии (тепла). А если это так, то при горении тел должна уменьшаться масса вещества соот- ветственно выделяемой тепловой энергии. Например, при пол- ном сгорании 12 г углерода расходуется 32 г кислорода. Каза- лось бы, что в результате этой химической реакции должно бы- ло бы получиться 44 г углекислого газа. Однако в действитель- ности его получается несколько меньше. Дело в том, что при полном сгорании 12 г углерода выдели- лось 96 больших калорий тепла и в соответствии с этим умень- шилась масса веществ, принимавших участие в химической ре- акции (горении углерода). Подсчитаем, на какую же величину Дли изменилась масса веществ (углерода и кислорода) при сго- рании 12 г углерода. Из формулы (2) следует, что = (3) где Дот— уменьшение величины массы в граммах; Ц7=96 больших калорий — 4 • 1012 эрг, так как 1 большая калория = 4,18 • 1010 эрг. Следовательно, 4. 1012 „Дот — -ff.fQiop — 4)4 • 10-’ грамма, 37
т. е. в результате выделения 96 больших калорий тепла при полном сгорании 12 г углерода общая масса веществ (углеро- да и кислорода) уменьшилась на 4,4 миллиардных доли грам- ма. Значит, при данной'химической реакции выделилось не 44 г углекислоты, а на 4,4 миллиардных доли грамма меньше. Надо признать, что убыль массы в данном случае оказалась настоль- ко малой, что ею практически можно было бы пренебречь. Во- обще говоря, убыль массы вещества при разного рода химиче- ских реакциях относительно мала и энергия, выделяемая при этом, также относительно невелика. / Иная картина наблюдается при так называемых ядерных ре- акциях, происходящих в ядрах атомов химических элементов. Ядерная реакция — это такой физический про- цесс, в результате которого ядра одних атомов превращаются в ядра других, качественно иных атомов. Яйерное вещество, принимающее участие в этих реакциях, например при взрыве атомной бомбы, теряет уже практически ощутимую величину массы, измеряемую грам- мами. В соответствии с этим выделяется весьма большое коли- чество ядерной (атомной) энергии. Если, например, при атом- ном взрыве масса «ядерного горючего вещества» преобразова- лась в массу иного вида материи, равную 10 а, то величина энергии, выделившейся при этом, равна W = тс2 = 10 (3 • 1010)2 = 9 • 1021 эрг = = 216-109 больших калорий. Примечание. 1 эрг = 0,24• 10“10больших калорий. Следовательно, в результате преобразования 10 г массы «ядерного горючего вещества» выделилось количество энергии, эквивалентное двести шестнадцати миллиардам больших кало- рий. Для выделения такого количества тепла необходимо было бы затратить примерно 30 000 тонн высокосортного каменного угля. Интересно отметить, что солнце в результате выделения ог> ромнейшего количества энергии ежегодно теряет массу, равную 1200 миллионам тонн. Энергия, излучаемая солнцем, получает- ся на нем в результате разного рода ядерных преобразований. Установлено, что выделение энергии атомных ядер можно осуществить двояким образом. В одном случае ее можно полу- чить в результате расщепления сложных атомных ядер на ’более простые, например расщепления ядер урана или плутония. В другом случае, наоборот, ее получают в результате образова- ния сложных ядер атомов за счет воссоединения более простых, например образования ядер атомов гелия за счет воссоединения ядер атомов тяжелого и сверхтяжелого водорода. Рассмотрим реакцию расщепления ядер атомов урана, поло- женную в основу действия атомной бомбы. 38-
Природный уран содержит ё своем составе три изотопа, из которых один имеет атомный вес 238, другой — 235 и третий — 234. Основным изотопом природного урана является наиболее тяжелый изотоп — уран 238, составляющий примерно 99,3% все- го количества природного урана. Изотопа урана 235 содержится в природном уране 0,7% и, наконец,. изотопа урана 234 — 0,006%. Доля изотопа урана 234 в природном уране настолько мала, что им практически можно пренебречь. Ядро урана 238 отличается большей прочностью по сравне- нию с ядром урана 235. Чтобы расщепить ядро урана 238, не- обходимо бомбардировать его быстрым нейтроном, летящим со скоростью (0—15 тысяч километров в секунду. Если же на ядро урана 238 воздействовать медленным нейтроном, скорость дви- жения которого измеряется десятками километров в секунду, то оно не расщепится, а просто поглотит этот нейтрон. В резуль- тате этого он превратится в ядро нептуния, а затем плутония, занимающих соответственно номера 93 и 94 в периодической таблице элементов, т. е. по соседству с ураном, занимающим но- мер 92. Наоборот, ядра атомов урана 235 малоустойчивы. На них достаточно воздействовать медленными нейтронами, чтобы их можно было расщепить. Но, как известно, в природном уране изотоп урана 235 содержится в очень малом количестве, а по- этому для осуществления бурной ядерной реакции в уране необхо- дим!) иметь уран 235 в чистом виде и в достаточном коли- честве. Ядерная реакция в уране 235 происходит следую- щим образом. Допустим, что какой-нибудь блуждающий ней- трон проник в одно из ядер атомов урана 235. Под действием этого нейтрона ядро атома расщепляется на два осколка (ба- рий 56 и криптон 36 или лантан 57 и бром 35), разлетающихся в разные стороны с громадными скоростями. Одновременно с этим из распавшегося ядра атома урана 235 вылетают три быстрых нейтрона (вторичные нейтроны), каждый из которых бомбардирует встретившееся ему на пути ядро атома. В резуль- тате происходит расщепление трех новых ядер атомов ура- на 235, которые выделяют все вместе уже девять новых быст- рых нейтронов. Последние же бомбардируют девять новых ядер атомов, после расщепления которых выделяется двадцать семь быстрых нейтронов. Они бомбардируют двадцать семь ядер ато- мов, расщепляют их, при этом выделяется восемьдесят один ней- трон и т. д. Эта так называемая цепная реакция расщеп* ления ядер атомов урана 235 (рис. 7) протекает весьма быстро, в форме атомного взрыва, и при этом выделяется громадней- шее количество ядерной (атомной) энергии. Сила атомного взрыва, при котором был израсходован 1 кг «ядерного горючего вещества», эквивалентна силе взрыва шестна- дцати миллионов килограммов сильного, взрывчатого вещества. 39
Значительно больший эффект выделения ядерной энергий дают так называемые термоядерные реакции. К ним относятся -ядерные реакции образования более тяжелых ядер из совокупности легких при температурах, измеряемых миллиона- ми градусов. Энергия, выделяемая при. термоядерных реакциях, Нейтроны Рис. 7. Схематическое изображение цепной реакции рас- щепления ядер атомов урана 235 во много раз превышает энергию, получаемую при ядерных ре- акциях расщепления тяжелых ядер. Примером такой термоядер- ной реакции может служить образование ядер атомов тяжелого гелия из совокупности ядер тяжелого водорода (дейтерия) и ядер сверхтяжелого водорода (трития). Ядро дейтерия состоит из одного протона и одного нейтрона, а ядро трития — из одно- го протона и двух нейтронов. При весьма высокой температуре (в несколько миллионов градусов), которую практически полу- чают при атомном взрыве, ядра дейтерия соединяются с ядрами трития и при этом образуются ядра тяжелого гелия, состояще- го из совокупности двух нейтронов и двух протонов. Что же ка- 40
сается одного лишнего нейтрона, то он с громадной скоростью выделяется в окружающее пространство. Ядерная (атомная) энергия может быть использована и для гфомышленных целей. В данном случае в качестве исходного материала используется уран 238. Ядра урана 238, как было сказано выше, не расщеп- ляются в результате бомбардировки их медленными нейтрона- ми. В этом случае дело ограничивается тем, что ядра погло- щают медленные нейтроны и превращаются в ядра соседнего элемента — нептуния, а последние в свою очередь превращают- ся в ядра следующего соседнего элемента — плутония. Получившийся в результате ядерной реакции плутоний яв- ляется радиоактивным веществом. Выбрасывая из себя аль- фа-частицу (ядро атома тяжелого гелия), ядро атома плу- тония превращается в ядро изотопа урана 235, являющегося, как известно, весьма активным «ядерным горючим». Ядра плутония — весьма ценное «ядерное горючее». Во-пер- вых, ядро плутония может быть легко расщеплено медленными нейтронами и, во-вторых, отличаясь значительно по своим хими- ческим свойствам от урана, оно может быть легко отделено от последнего с помощью обычных химических реакций, в то время как отделение изотопа урана 235 от изотопов урана 238 затруд- нительно и требует относительно большой затраты энергии. Устройство, с помощью которого получается радиоактивный плутоний за счет ядерной реак- ции урана 238, называется атом- ным реактором (котлом). В атомный реактор заклады- вается природный уран, состоя- ний из изотопов — урана 238 и урана 235. Под действием мед- ленных нейтронов ядра урана 235 расщепляются, превращаясь в ядра-осколки более легких хими- ческих элементов. При этом из каждого ядра урана 235 выле- тают’ по три быстрых нейтрона. На своем пути они встречают спе- циально введенные в атомный реактор вещества, называемые замедлителями, например графит (рис. 8). Назначение за- медлителей —- отбирать у быстрых нейтронов часть их кинетической энергии и превращать их в мед^ ленные нейтроны. Это необходимо для того, чтобы быстрые ней- троны, выбрасываемые расщец- нейтрон U-235 быстрые Осколон после нейтроны усколон после деления J | у деления | Замедлитель | (J-238 Превращается б плутоний Медленные нейтроны | Замедлитель 1 О енол о к после деления .Может \ быть утерян Псколон после деления Медленные нейтроны вызывают дальнейшее деление Рис. 8. Ядерная реакция в уране с замедлителем U-235 41
ленными ядрами урана 235, не смогли расщепить ядра атомов урана 238. Одна часть замедленных нейтронов продолжает рас- щеплять ядра атомов урана 235, так что цепная реакция проис- ходит непрерывно в атомном реакторе. Но она протекает не бурно, не сопровождается взрывом, а идет медленно, так как ядер атомов урана 235 содержится гораздо меньше, чем ядер атомов урана 238? Другая и более значительная часть замедленных нейтронов поглощается ядрами атомов урана 238, что в ко- нечном итоге приводит к превращению их в ядра атомов плу- тония. Таким образом, характерной особенностью работы атомного реактора является то, что в нем одновременно с расходом «ядерного горючего» — урана 235 вырабатывается и накапли- вается не менее ценное «ядерное горючее» — плутоний. Если в атомном реакторе за год расходуется одна тонна «ядерного го- рючего» — урана 235, то одновременно с этим за счет урана 238 получается тонна нового «ядерного горючего» — плутония. Ис- пользовав эту тонну плутония в атомном реакторе в качестве «ядерного горючего», можно через год получить новую тонну плутония за счет расхода урана 238 и т. д. Следовательно, ос- новным расходным материалом атомного реактора является уран 238, который, как известно,, составляет 99,3% всего коли- чества природного урана. * Чем больше медленных нейтронов имеется в атомном реак- торе, тем быстрее протекает в нем ядерная реакция и, наоборот, чем меньше их там, тем менее интенсивной становится она. В целях регулирования скорости ядерИой реакции в атомном реакторе в последний вводятся специальные поглотители медленных нейтронов, например кадмий. При увеличе- нии количества кадмия в атомном реакторе ядерная реакция урана становится все более и более замедленной и может быть вообще прекращена. Чтобы медленные нейтроны не улетучивались из атомного реактора, последний, как правило, облицовывается специальной оболочкой, отражающей нейтроны внутрь реактора. Работа с атомным реактором требует большой осторожно- сти, так как при ядерных реакциях происходит интенсивное ра- диоизлучение в виде потока гамма-лучей, нейтронов'- и' т. д., что может угрожать жизни и здоровью обслуживающего персонала. Для предупреждения этого атомные реакторы обычно изоли- руют от внешнего пространства рядом веществ (например, кад- мий, свинец и некоторые другие), сильно поглощающих радио- активное излучение. И в довершение всего атомный реактор по- крывается извне слоем бетонной оболочки толщиной в несколь- ко метров. При работе атомного реактора выделяется большое количе- ство тепловой энергии, которая может быть целесообразно ис- пользована в промышленных целях, 42
На рис. 9 показано схематически устройство атомного реак- тора, предназначенного для обслуживания электрической стан- ции. Здесь мы видим: урановые стержни, являющиеся «ядерным горючим»; графит, играющий роль замедлителя ядерной реак- ции; отражатель, имеющий назначение удерживать нейтроны в атомном реакторе, и защитную бетонную оболочку толщиной в несколько метров, предохраняющую от проникновения из атомного реактора во внешнюю среду нейтронов и гамма-лучей. Вода или какой-либо жидкий металл, например калий, натрий, Рис. 9. Схема устройства атомного реактора с за- медлителем из графита свинец, прогоняются насосом из атомного реактора в теплооб- менник, где они отдают свое тепло воде, циркулирующей в зме- евике теплообменника, а затем возвращаются обратно в атом- ный реактор. Вода же, нагретая в змеевике теплообменника, превращается в пар высокой температуры и давления и по па- ропроводу направляется в паровую турбину, приводящую во вращение генератор электрической энергии. Преимущества атомной электростанции по сравнению с обычной теплоэлектростанцией очень велики. Например, если атомная электростанция израсходует в год две тонны ядерного горючего, то она даст энергии более десяти миллиардов кило- ватт-часов, т. е. больше, чем даст строящаяся ныне самая мощ- ная в мире Сталинградская гидроэлектростанция. Для получе- ния такого .количества энергии от обычной тепловой электро- станции потребовалось бы сжечь много миллионов тонн высоко- качественного каменного угля. § 7. МОЛЕКУЛЫ И ФИЗИЧЕСКИЕ ТЕЛА Атомы, соединяясь друг с другом, образуют более сложные по своему строению частицы вещества, называемые молеку- лами. 43
Молекулой называют наименьшее количество сложного ве- щества, сохраняющее индивидуальные химические свойства это- го вещества. Способность различных атомов присоединять к себе различ- ное количество других атомов носит название валентности. За единицу валентности атома принимается способность атома присоединять к себе толь- ко один атом водорода или за.мещать его в ка- ком-либо сложном веществе. Например, атомы хлора, натрия, водорода одновалентны, кислорода и серы — двухвалентны, азота и фосфора — трехва- лентны, углерода и кремния — четырехвалентны и т. д. При образовании молекулы сложного вещества атомы обыч- но соединяются так, что на каждую единицу валентности одно- го атома приходится одна единица валентности другого атома. Например, атом кислорода О двухвалентен, а атом водорода Н одновалентен. Следовательно, для того чтобы полностью исчерпать свою двухвалентность, атом кислорода должен всту- пать в химическое взаимодействие с двумя одновалентными ато- мами водорода. В результате этого соединения образуется мо- лекула воды Н2О. Молекулы весьма малы по размерам, а поэтому для образо- вания физических тел требуется колоссально большое число мо- лекул. Например, для образования капли воды требуется при- мерно 1500 миллиардов миллиардов молекул (15 - 1020 мо- лекул). Способность атомов вступать в химическое взаимодействие и образовывать физические тела обусловлена строением их электронных оболочек и, в частности, количеством- электронов, содержащихся во внешнем электронном слое атома. Существуют различные виды связи атомов друг с другом, в результате которых из совокупности их создаются молекулы и физические тела. Вначале ознакомимся с так называемой ионной (полярной) связью атомов, рассмотрев в каче- стве примера образование кристаллов хлористого натрия (пова- ренной соли) NaCl из совокупности положительных ионов нат- рия Na+ и отрицательных ионов хлора С1~. ( Атом хлора имеет в своей электронной оболочке 17 электро- нов, из которых* во внешнем (третьем) электрическом слое на- ходится семь электронов. Поэтому атом хлора стремится ото- брать от атомов других химических элементов недостающий ему восьмой электрон для сформирования устойчивого восьмиэлек- тронного внешнего слоя. Атом натрия имеет в своей электронной оболочке 11 элек- тронов, из которых во внешнем (третьем) электронном слое на- ходится всего один электрон. Поэтому внешний электронный слой (третий слой) в атоме натрия весьма неустойчив. Единст- венный электрон этого слоя слабо притягивается ядром атома, 44
а потому относительно легко может покинуть пределы данного атома. Естественно, что атом хлора при соприкосновении с атомом натрия не замедлит отобрать у него внешний электрон (элек- трон третьего слоя) / и в результате этого он превратится в од- нозарядный отрицательный ион хлора. Что же касается атома натрия, то он, потеряв свой внеш- ний электрон, превращается в положительный ион 4 натрия. Вновь образовавшиеся* отрица- тельный ион хлора и положитель- ный ион~ натрия, будучи разно- именно заряженными электриче- ством частицами, притягиваются друг к другу. В результате этого образуется так называемая кри- сталлическая решетка поваренной соли (рис. 10). Здесь каждый положительный ион на- трия (темные шарики) окружен Рис» 10. Кристаллическая решетка поваренной соли симметрично со всех сторон шестью отрицательными ионами хлора (светдые шарики) и, наоборот, каждый ион хлора сим- метрично окружен шестью ионами натрия. Надо сказать, что в кристалле поваренной соли все десять электронов положи- тельного иона натрия вращаются вокруг его ядра и не покида- ют пределов иона. В отрицательном ионе хлора все восемна- дцать электронов также вращаются вокруг ядра и не покидают пределов этого иона. Даже электрон, отобранный атомом хлора от атома натрия, прочно удерживается ионом хлора в своей си* стеме. Следовательно, в кристаллах поваренной соли практиче- ски нет таких электронов, которые перемещались бы в между- ионном пространстве и переходили бы от одного иона к друго- му. Это положение весьма характерно для всех кристалличе- ских веществ, образованных в результате ионной связи, т. е. взаимного притяжения разноименных ионов. Отличительной осот бенностью тел подобного рода является то, что они хрупки, как поваренная соль, не ковки, более или менее прозрачны, плохо проводят тепло и почти не проводят электричества. Другой вид связи, с помощью которой образуются более сложные частицы вещества,— это так называемая го м ео no- ji я р н а я связь. При ней для образования молекулы объеди- няются однородные атомы, например два атома водорода, два атома кислорода и т. д.^Так как в этом случае объединяются два однородных атома’, то невозможно, чтобы один из них стал положительным ионом, а другой — отрицательным путем пере- хода электронов из одного атома в другой. Здесь связь атомов осуществляется несколько иначе. Оба однородных атома выде- 45
ляют по одному внешнему электрону й йрй этом становятся по- ложительными ионами. Что же касается выделенных атомами электронов, то они располагаются в междуионном пространстве как бы посредине между двумя положительными ионами и сво- им электрическим взаимодействием стягивают их в единую си- стему — молекулу. Например, два атома кислорода О выделяют из своего состава по одному электрону, в результате чего поло- жительные ионы атомов* кислорода, связанные этими двумя об- щими электронами, образуют молекулу кислорода О2. Третий вид связи — это металлическая связь, при которой из совокупности ионов металла и электронов образует- ся кристаллическая решетка металлов. В этом случае атомы ме- таллов соединяются друг с другом в результате электрического взаимодействия положительных ионов и электронов. При образо- вании металлического кристалла каждый атом металла в зави- симости от его валентности отдает один — два электрона «в об- щее пользование» всех частиц металла, превращаясь в положи- тельный ион. Поэтому кристаллическая решетка металла содер- жит только положительные ионы, размещенные в ней строго симметрично в определенном порядке. Выделенные же атомами металла электроны свободно перемещаются в междуионном пространстве, образуя нечто вроде электронного газа. Эти «об- обществленные» электроны и являются тем «цементирующим ве- ществом», с помощью которого ионы металла образуют моно- литную крепкую кристаллическую решетку. «Обобществленные» электроны, перемещающиеся в между- ионном пространстве кристаллов металла, называются сво- бодными электронами. Они названы так потому, что каждый из них не привязан к какому-нибудь определенному иону металла, а принадлежит всем ионам, а поэтому может сво- бодно перемещаться в междуионном пространстве. Физические тела, возникшие в результате металлической связи, являются металлами. Характерная особенность их — ков- кость, непрозрачность, хорошая проводимость тепла и электри- чества. В заключение следует отметить, что нельзя резко разграни- чивать тела по видам их атомной связи. В некоторых физиче- ских телах могут одновременно проявляться различные виды связи, например ионная и металлическая. В силу этого многие вещества наряду с металлическими свойствами могут обладать в той или иной степени свойствами неметаллических веществ (например, некоторые сплавы металлов). § 8. ЭЛЕКТРИЧЕСКИЕ ПРОВОДНИКИ ПЕРВОГО РОДА В предыдущем параграфе было выяснено, что кристалличе- ская решетка металла состоит из совокупности положительных ионов, между которыми по всевозможным направлениям переме- 46
щаются свободные электроны, осуществляющие металлическую связь ионов. Силы взаимного притяжения положительных ионов металла и свободных электронов не только преодолевают взаим- ное отталкивание положительных ионов металла, но и связы- вают их в прочную систему — кристаллическую решетку. Количество свободных электронов в металлах чрезвычайно велико. Если, например, каждый атом серебра в силу его одно- валентности выделяет для общего пользования один свободный электрон, то это значит, что число свободных электронов, содер- жащихся в одном кубическом сантиметре серебра, равно при- мерно шестьдесят тысячам миллиардов миллиардов (6‘К)22) электронов. Движение свободных электронов в междуионном простран- стве металла можно уподобить тепловому движению молекул газа. Поэтому всю совокупность свободных электронов металла иногда называют «электронным газом», а движение электронов в нем называют хаотическим, или беспорядочным. Однако движение свободных электронов в междуионном про- странстве металлов подчинено особым законам, согласно кото- рым к а ж д ы й с в о б о д н ы й электрон занимает в лю- бой данный момент времени строго определен- ное энергетическое состояние, которое уже не может занимать в этот же момент другой сво- бодный электрон. Общее расположение всех свободных электронов в металле в нормальных условиях таково, что образуется система с наи- меньшей энергией электронов, г!ри этом их заряды полностью компенсируют заряды положительных ионов металла по отно- шению к внешней среде. Это значит, что в нормальных услови- ях металл электрически нейтрален. Следовательно, для того чтобы зарядить металл электричеством, надо как-то изъять из него часть свободных электронов. Тогда равновесие электриче- ских зарядов ионов металла и отрицательных зарядов свобод- ных электронов будет нарушено и металл в целом по отноше- нию к внешней среде зарядится положительным электричеством. Если в металлическом теле, например в медной проволоке, электрического поля нет, то свободные электроны перемещаются хаотически (беспорядочно) в самых разнообразных направле- ниях. (рис. 11). При этом сколько свободных электронов дви- жется ъ направлении оси проволоки справа налево, столько же в данный момент времени их движется слева направо. Следова- тельно, направленного движения потока электронов в проволо- ке в данном случае не наблюдается. Теперь допустим, что в медной проволоке возникло электри- ческое поле. Свободные электроны, очутившись в нем, начи- нают испытывать на себе действие сил этого поля. Те свобод- ные электроны, направление движения которых совпадает с на- правлением действия сил поля, получают некоторую добавоч-
ную скорость, а электроны, движение ^которых совершается на- встречу действию сил поля, замедляют скорость своего движе- ния, некоторая часть их даже изменяет направление движения на противоположное. Короче говоря, движение каждого свобод- ного электрона металлической проволоки, /находящейся в элек- трическом поле, представляет собой совокупность двух его дви- жений: беспорядочного (теплового) и направленного, совпадаю- щего с направлением действия сил электрического поля. Рис. 11. Беспорядочное движение свободных элек- тронов в металле Направленное движение потока свободных электронов вдоль металлической проволоки можно уподобить движению роя насе- комых, когда он поступательно движется вперед и в то же вре- мя внутри самого роя каждое отдельное насекомое движется беспорядочно с некоторой относительно большой скоростью. Рис. 12. Свободные электроны металла в электри- ческом поле движутся направленно со скоростью v9 Направленное (упорядоченное) движение потока свободных электронов внутри металлического проводника называется элек- трическим током проводимости. На рис. 12 схематически изображена картина электрического тока проводимости. Здесь показано, что скорость движения электронов имеет две составляющие: одна из них, v9i обуслов- ливает направленное движение всего потока электродов, т. ё. ток проводимости, а другая, и, различная для каждого отдель- ного электрона, обусловливает его тепловое движение. &
Следовательно, свободные электроны металла являются теми вещественными элементарными частицами, благодаря которым в металлах возникает электрический ток проводимости. '' Если скорость беспорядочного движения свободных электро- нов в металлах в среднем измеряется тысячью километров в се- кунду, то скорость-направленного движения потока свободных электронов при электрическом токе проводимости не превышает долей сантиметра в секунду. Силы электрического поля, перемещая свободные электроны вдоль металлической проволоки, совершают некоторую работу. Эта работа идет на повышение энергии свободных электронов, которые в свою очередь отдают ее ионам металла в виде тепло- вой энергии. ' * Физические тела, обладающие свободными электронами и в соответствии с этим электронной проводимостью, называются электрическими проводниками первого рода. К ним относятся ме- таллы и сплавы металлов. Проводники первого рода играют исключительно большую роль в электротехнике. К числу металлов, широко применяемых в электротехнике для изготовления проводников, относятся медь, алюминий и сталь, § 9. ЭЛЕКТРИЧЕСКИЕ ПРОВОДНИКИ ВТОРОГО РОДА •Большая часть жидких веществ состоит в основном из сово- купности электрически нейтральных молекул, так что практиче- ски в них очень мало свободных электронов иЛи каких-либо иных электрически заряженных вещественных частиц. Поэтому такие жидкие вещества весьма плохо проводят электричество. К по- добного рода жидкостям относятся химически чистая вода, ке- росин, минеральные масла, водные растворы сахара, глице- рина и т. д. Однако наряду с жидкостями, плохо проводящими электриче- ство, имеются и такие, которые обладают большой электропрово- димостью. К ним относятся водные растворы солей, кислот, щелочей, например раствор серной кислоты H2SO4, медного купо- роса CuSO4, едкого натра NaOH и т. д. Такого рода электропро- водящие жидкие вещества называют электролитами или провод- никами второго рода. ч Опытом установлено, что во всяком электролите происходит непрерывный процесс диссоциации молекул, т. е. распада их на составные части — разноименные ионы. Например, в водном рас- творе хлористого натрия NaCl молекула распадается ца две самостоятельные части: положительный однозарядный ион нат- рия Na+ и отрицательный однозарядный ион хлора С1“. Диссоциацию молекулы поваренной соли NaCi можно запи* сать следующей формулой;, NaCl±;Na++4Cr. 4—1377 49
Процесс диссоциации молекул в растворах солей и кислот протекает очень интенсивно, так что в каждом кубическом сан- тиметре раствора содержится громадное количество ионов обоих знаков; эти ионы принимают участие в тепловом движении мо- лекул, беспорядочно перемещаясь в растворе. Одновременно с явлением диссоциации молекул в электроли- тах происходит и обратный процесс — молизация молекул, т. е. процесс образования нейтральных молекул в результате воссое- динения разноименных ионов. Таким образом, в нормальных условиях в электролитах про- исходит непрерывный процесс диссоциации и молизации молекул. При этом электролит в целом электрически нейтрален, так как в нем разноименные ионы в каждый данный момент размещены так, что электрически взаимно нейтрализуются. Степень диссоциации молекул растворенного вещества зави- сит от его природы и от концентрации раствора. В слабых рас- творах солей и кислот практически все молекулы диссоциированы. Количество ионов в каждом кубическом сантиметре электролита измеряется миллиардами миллиардов ионов. Даже в чистой воде Н2О, которая считается слабо диссоциированной, на один куби- ческий Сантиметр приходится около шести миллиардов положи- тельных ионов водорода Н+ и столько же отрицательных одно- зарядных ионов гидроксила ОН". Установлено, что при диссоциации молекул элек- тролитов металлы, и водород всякий раз вы- деляются из молекул в виде положительных ионов, а остальная часть молекулы образует отрицательны йион. Если воздействовать на ионы электролита силами электриче- ского поля, то положительные ионы приобретают добавочную скорость в направлении поЛя, а отрицательные — в противополож- ном направлении. В результате этого, помимо беспорядочного теплового движения ионов, в элек- тролите будет ’ наблюдаться на- правленное, упорядоченное движе- ние двух потоков разноименных ионов в прямо противоположных направлениях (рис. 13). Упорядоченное и йаправленное движение потока ионов в элек- тролите, происходящее под влия- нием сил электрического поля, на- зывается ионным электрическим током. Следовательно, основное отли- Рис. 13. Схематическое изобра- жение ионного тока в электролите 50
чие ионного электрического тока от тока проводимости заклю- чается в том, что ток проводимости создается движением свобод- ных электронов, а ионный электрический ток — движением ионов, т. е. зарядов, размещенных на частицах распавшихся молекул. Скорость направленного движения потоков разноименных ионов в электролитах весьма мала — в среднем она измеряется долями сантиметра в секунду. Проводники второго рода (электролиты) играют важную роль в электротехнике, например при работе химическйх источ- ников электрической энергии (аккумуляторов и гальванических элементов), при электролизе и в ряде иных случаев. * К числу электролитов, широко применяемых в электротехнике, относятся: водный раствор серной кислоты H2SO4 для кислотных аккумуляторов, водный раствор едкого кали КОН для щелочных аккумуляторов, водный раствор едкого натра NaOH для щелоч- ных аккумуляторов, водный раствор хлористого аммония (на- шатыря) NH4C1 для гальванических элементов марганцовой си- стемы. § 10. полупроводники Наряду с проводниками электричества в природе существует много веществ, обладающих значительно меньшей электропрово- димостью, чем металлические проводники. Вещества подобного рода называются полупроводниками. К полупроводникам относятся: некоторые' химические эле- менты, например селен Se, кремний Si и германий Ge; сернистые соединения, например сернистый таллий T1S, сернистый кадмий CdS, сернистое серебро Ag2S; карбиды, например карборунд SiC, и ряд других веществ. Природа электрической проводимости полупроводника зави- сит от рода примесей, имеющихся в основном материале полу- проводника, и от технологий изготовления его составных частей. Различают две основные разновидности электрической проводимости^ полупроводников — электронную и «д ы- р о ч н у ю». Полупроводник обладает электронной проводимостью, если в атомах его примеси внешние электроны относительно слабо связаны с ядрами этих атомов. Если в подобного рода полупро- воднике создать электрическое поле, то под влиянием сил этого поля внешние электроны атомов примеси полупроводника поки- нут пределы своих атомов и превратятся в свободные электроны. Свободные электроны создадут в полупроводнике электрический ток проводимости под влиянием сил электрического поля. Следо- вательно, природа электрического тока в полу- проводниках с'электронной проводимостью та Же, что и в металлических проводниках. Но так как свободных электронов в единице объема полупроводника во много раз меньше, чем в единице объема металлического провод- 4* 51
Рис. 14, Схематическое изображение тока з полупроводнике с „дырочной1* проводимостью ника, то естественно, что при всех прочих одинаковых условиях ток в полупроводнике будет во много раз меньше, чем в металли- ческом проводнике. Полупроводник обладает «дырочной» проводимостью, если атомы его примеси не только не отдают своих внешних электро- нов, но, наоборот, стремятся захватить электроны атомов основ- ного вещества полупроводника. Если атом примеси отберет электрон у атома основного ве- щества, то в последнем образуется нечто вроде свободного места для электрона — «дырка». Атом полупроводника, потерявший электрон, называют «электронной дыркой», или просто «дыркой». Если «дырка» заполняется электроном, перешедшим с соседнего атома, то она ликвидируется и атом становится нейтральным в электрическом отношении, а «дырка» смещается на соседний атом, потерявший электрон. Следовательно, если на полупровод- ник, обладающий «дырочной» проводимостью, воздействовать электрическим полем, то «электронные дырки» будут смещаться в направлении этого поля. На рис. 14 показана схематически картина электрического тока в полупроводнике с «дырочной» проводимостью. Здесь мы видим, что электрическое поле направлено слева направо вдоль полупроводника. Переход электронов с одного атома на другой происходит навстречу электрическому полю, а «электронные дырки» смещаются в направлении поля. Смещение «элек- тронных дырок» в направлении действия элек- трического поля аналогично перемещению по- ложительных электрических зарядов в поле 52
и, следовательно, представляет собой явление электрического тока в п о л у п р о во д н'и ке. Полупроводники нельзя строго разграничивать по механизму их электрической проводимости, так как наряду с «дырочной» проводимостью данный полупроводник может в той или иной степени обладать и электронной проводимостью. В современной технике полупроводники нашли самое широ- кое применение4. Ниже приведено несколько примеров практиче- ского использования полупроводников в современной технике. П о л у проводниковые фотоэлементы. Фотоэле- ментом называется прибор, дающий возможность непосред- ственно преобразовывать лучистую (световую) энергию в элек- трическую. . ‘ На рис. 15. схематически показано устройство полупроводни- кового вентильного фотоэлементач Он состоит из металлической пластинки /, на которую нанесен слой полупроводника 2, а на последний в свою очередь нанесен тончайший полупрозрачный слой серебра или золота 4, Слой полупроводника 2, например кремния, предварительно подвергается специальной обработке, в результате которой на одной стороне его, примыкающей к металлической пластинке /, образуется слой, обладающий «дырочной» проводимостью, а на противоположной стороне, примыкающей к полупрозрачному слою металла 4,— слой, обладающий электронной проводи- мостью. Так как полупроводник 2 имеет в разных своих слоях различную по природе проводимость (электронную и «дыроч- ную»), то на границе этих слоев образуется так называемый за- пирающий слой 3. Последний обладает вентильным свойством, т. е. способностью пропускать электроны только в одном направ- лении — от слоя полупроводника с электронной проводимостью к слою с «дырочной» проводимостью. В обратном же направле- нии электроны через запирающий слой проникнуть не могут. Если на металлический полупрозрачный слой 4 па- дают лучи света, то фотоны света частично отражаются от него, частично поглощают- ся им, а некоторая их часть проникает в ту область полу- проводника, где он обладает электронной проводимостью. Здесь фотоны света вступают во взаимодействие с внеш- ними электронами- атомов полупроводника, заставляют их покинуть пределы своих атомов и стать свободными электронами. Последние же, Рис* 15. Схематическое изображение полупроводникового вентильного фото- элемента: 1 — металлический электрод; 2 — полупроводник с яиырочной“ 4- и электронной — проводимостя- ми; 3 — запирающий слой; 4 — полупрозрачный металлический слой 53
покинув атомы, относительно легко переходят через запираю- щий слой 3 и проникают в ту область полупроводника, где' они имеют «дырочную» проводимость. В результате нижняя ме- таллическая пластинка 1 зарядится отрицательным электриче- ством (избыток электронов), а металлический слой 4 — поло- жительным (недостаток электронов). Это значит, что между ме- таллической пластиной 1 и полупрозрачным металлическим слоем 4 возникает некоторая разность потенциалов (напряжение). Если же электроды соединены проводниками, то в данной цепи будет протекать электрический ток. Этот ток будет продолжаться до тех пор, пока не прекратится поступление фотонов света (элек- тромагнитного поля) в полупроводник. Следовательно, в рас- смотренном нами полупроводниковом фотоэлементе происходит преобразование энергии фотонов света, или, иначе сказать, энер- гии электромагнитного поля, в энергию электрическую. В современной ^технике полупроводниковые фотоэлементы на- ходят весьма широкое применение. В настоящее время выпу- скаются весьма совершенные по своей конструкции и качествам полупроводниковые фотоэлементы, например селеновые, герма- ниевые, кремниевые, сернисто-серебряные и т. д. Таким образом, проблема преобразования в широких масштабах световой энер- гии солнечных лучей в энергию электрическую стала вполне раз- решимой. Известно, например, что в ясный солнечный день кре- мниевые фотоэлементы могут снимать с 1 квадратного метра поверхности, освещаемой солнцем, электрическую энергию мощ- ностью примерно в 50 ватт. Наряду с полупроводниковыми, фотоэлементами широкое при- менение получили полупроводниковые термоэле- менты, например, при радиофикации населенных пунктов, не снабжаемых электрической энергией. Удобство применения тер- моэлемента заключается в том, что в нем электроэнергия полу- чается непосредственно за счет тепловой энергии. Например, по- лупроводниковый термоэлемент типа ТГК-3, изготовляемый оте- чественной электропромышленностью, мо5кет полностью обеспе- чить электроэнергией многоламповый радиоприемник ^Родина» при подогреве его керосиновой лампой. Свойство полупроводников резко изменять свою электропро- водимость с изменением температуры дало возможность использовать их в качестве термосопротивлений. Полу- проводниковые термометры, изготовленные из термосопротивле- ний, дают возможность измерять температуру в весьма широких пределах с точностью до тысячных долей градуса Цельсия. Полупроводники находят теперь весьма широкое применение в радиотехнике. Например, кристалл и ч еский герма- ниевый усилитель, имеющий размер не более спичечной головки й в то же время весьма простой по своему устройству и надежный в эксплуатации, с успехом заменяет усилительную электронную лампу, 54
Благодаря применению полупроводниковых приборов удается значительно уменьшить вес и габариты радиотехнических уст- ройств. К примеру, ультракоротковолновый радиопередатчик, по- строенный из полупроводниковых материалов, свободно разме- щается в футляре для карманных часов. Весьма интересен тот факт, что электрическая батарея эле- ментов, построенная из полупроводников совместно с радиоак- тивным элементом стронцием, может давать без подзаряда непрерывно электрический ток на протяжении примерно двух де- сятков лет. Применяя полупроводниковый материал люминофор, удалось построить «лампы дневного’ света», весьма экономич- ные и дающие неизнурительное для глаз освещение. Не будет ошибочным утверждать, что полупроводниковые приборы в ближайшем будущем окажут весьма сильное влияние на'технический прогресс во многих отраслях техники, и в част- ности в технике связи, автоматике, вычислительных и управляю- щих машинами т. д. § 11. ДИЭЛЕКТРИКИ Диэлектриками, или, иначе, изоляторами, на- зываются такие вещества, ч которые практи- чески не проводят электричества. К диэлектрикам относятся кристаллы солей, каучук, эбонит, стекло, фарфор, слюда, минеральные масла, газы и ряд других веществ. Диэлектрики кристаллического строения, например хлори- стый натрий (поваренная соль) NaCl, состоят из совокупности разноименных ионов, которые строго симметрично расположены в пространственной кристаллической решетке (см. рис. 10). Здесь электроны как положительных, так и отрицательных ионов кристалла крепко связаны с ядрами, так что при нормальном состоянии кристалла они не могут покинуть пределы этих ионов. Вследствие отсутствия свободных электронов или каких-либо иных электрически заряженных частиц в междуионном простран- стве кристалла электропроводимость кристалла практически равна нулю. Если на кристаллический диэлектрик воздейство- вать силами электрического поля, то в нем ток возникнуть не может. Однако все же под действием сил поля положительные ионы кристалла сместятся на весьма незначительное расстояние в направлении поля, а отрицательные ионы — навстречу полю. В результате этого диэлектрик поляризуется, т. е. на его противоположных сторонах появятся так называемые связанные электрические заряды. Эти заряды в отличие от обычных зарядов, могущих перемещаться на любые расстояния, связаны прочно с кристаллической решеткой диэлек- трика и могут только смещаться на ничтожно малые расстояния. Следовательно, связанные электрические заряды нельзя снять с диэлектрика, например отвести в землю. Однако, эти заряды 55
создают свое электрическое поле, которое изменяет силу и кон- фигурацию поля внешнего, осуществившего поляризацию диэлек- трика. z ; В отличие от диэлектриков с кристаллическим строением все прочие диэлектрики, как твердые, жидкие, так и газообразные, состоят из совокупности отдельных молекул. Отсутствие свобод- ных электронов или каких-либо иных вещественных частиц, заря- женных электричеством, в междумолекулярном пространстве ди- электрика обеспечивает ему хорошие изоляционные свойства. Существуют такие диэлектрики, молекулы которых в нор- мальных условиях нейтральны в электрическом отношении, так как их положительные заряды полностью компенсируются от- рицательными. К подобного рода диэлектрикам относятся, на- пример, газы: азот N2, водород Н2, углекислый газ СО2 и др. Рис. 16. Схематическое изображение поляризованного диэлектрика Если на такой диэлектрик воздействовать силами электрического поля, то «центры тяжести» положительных и отрицательных электрических зарядов молекул несколько сместятся. В резуль- тате этого молекула превратится в так называемый электри- ческий диполь, т. е. систему двух разноименных электри- ческих зарядов, размещенных по некоторой оси на весьма малом - расстоянии друг от друга. Под влиянием сил электрического поля в диэлектрике образуется множество электрических дипо- лей (рис. 16), ориентированных в направлении действия этих сил. Диэлектрик с ориентированными в нем электрическими ди- полями становится поляризованным. Его диполи создают свое собственное электрическое поле, которое накладывается на поле внешнее, осуществившее поляризацию диэлектрика. Чем больше сила электрического поля, действующего в диэлектрике, тем бо- лее энергично осуществляется его поляризация и тем больше связанных зарядов возникает на его поверхностях* В природе существуют и такие диэлектрики, которые уже в готовом виде содержат в себе электрические диполи (жесткие дипольные молекулы). Поляризация подобных диэлектриков осуществляется в результате поворота электрических диполей, т. е. их ориентировки в направлении действия поля. К диэлек- 56
трикам подобного рода относятся такие жидкости, как вода, ни- тробензол, эфиры, органические кислоты, и такие газы, как сернистый ангидрид SO2, аммиак NH3 и др. Диэлектрики играют весьма большую роль в электротехнике. От них обычно требуются хорошие изоляционные свойства, т. е. способность не пропускать электрические токи, и большая элек- трическая прочность, т. е. способность выдерживать высокое электрическое напряжение./ К числу диэлектриков, широко применяемых в электротех- нике, относятся: слюда для изготовления диэлектрика конденса- торов, изоляции в электронагревательных приборах, коллектор- ных пластин в генераторах и т. д.; фарфор для изготовления изоляторов линий передач, линий связи и т. д.; радиофарфор и ультрафарфор для изоляции в радиотехнике; резина для изоля- ции проводов; эбонит для устройства электроизолирующих дета- лей; мрамор для изготовления распределительных щитов и панелей; керамика для изготовления диэлектриков конденсато- ров; шеллак для склеивания и изоляции витков катушек и др.; электрокартон дЛя изготовления каркасов трансформаторов и ка- тушек и т. д. § 12. ЭЛЕКТРОПРОВОДИМОСТЬ ГАЗОВ Газы в нормальном состоянии являются хорошими диэлек- триками (например, чистый, не ионизированный воздух). Однако, если газы содержат в себе влагу с примесью органиче- ских и неорганических частиц и при этом они ионизированы, то они проводят электричество. Электропроводимость газов зависит главным образом от степени их ионизации, которая может быть осуществлена различными способами. В основном 'иони- зация газов осуществляется в результате от- щепления электронов от нейтральной моле- кулы газа. Выделившийся й молекулы газа электрон пере- мещается в междумолекулярном пространстве газа, и здесь в зависимости от рода газа он может сохранить относительно долго «самостоятельность» своего движения (например, в таких тазах, как водород Н2, азот N2) или, наоборот, быстро проник- нуть в нейтральную молекулу, превратив ее в отрицательный ион (например, в кислороде). Наибольший эффект ионизации га- зов достигается путем облучения их рентгеновыми, катодными лучами или лучами, испускаемыми радиоактивными веществами. Атмосферный воздух летом весьма интенсивно ионизируется под влиянием солнечных лучей. Влага, находящаяся в воздухе, конденсируется на его ионах, образуя мельчайшие капельки воды, заряженные электричеством. В конечном итоге из отдельных электрически заряженных капелек воды образуются грозовые тучи, сопровождаемые молниями,, т. е. электрическими разря- дами атмосферного электричества.
ГЛАВА II ЭЛЕКТРИЧЕСКИЙ ЗАРЯД И ЕДИНИЦЫ ЕГО ИЗМЕРЕНИЯ § 13. СИСТЕМЫ ЕДИНИЦ ИЗМЕРЕНИЯ ЭЛЕКТРИЧЕСКИХ И МАГНИТНЫХ ВЕЛИЧИН При изучении электротехники приходится иметь дело с элек- трическими, магнитными и механическими величинами и измерять эти величины. Измерить электрическую, магнитную или какую-либо иную величину — это значит сравнить ее с другой однородной величи- ной, принятой за единицу. В электротехнике применяется ряд систем единиц, с помощью которых измеряются электрические и магнитные величины. В на- стоящей книге мы будем пользоваться абсолютной практической электромагнитной системой единиц МКСА, получившей за по- следнее время наиболее широкое применение в электротехнике. Одновременно с этим в книге будут приведены и некоторые иные системы единиц, которые еще до сих пор встречаются в электро- технической литературе. В абсолютной практической» электромагнитной системе еди- ниц МКСА за единицу длины принимается 1 метр (1 м), за еди- ницу массы— масса 1 килограмма (1 кг), за единицу времени— 1 секунда (1 сек.) и за единицу электрического тока— 1 ампер (1 а). В соответствии с этим наименование абсолютной практи- ческой электромагнитной системы единиц сокращенно обозна- чается символом МКСА, где М обозначает метр, К — килограмм, С — секунда, А — ампер. Все прочие единицы измерения электрических, магнитных и меу нических величин — производные от вышеуказанных четы- рех основных единиц. В табл. 2 приведены основные и механические единицы си- стемы единиц МКСА. Система единиц МКСА' может быть рационализированной и нерационализированной. Рационализированная система единиц МКСА — это такая система, при которой возможна рационали- 58
Таблица 2 Наименования и определения основных и механических единиц системы единиц МКСА Наименование Сокращенное обозначение Определение русским алфавитом латинским (греческим) алфавитом 1. Основные единйцы Единица длины Метр М Ш Единица длины, определяемая платино-иридиевой мерой, носящей знак 28 и являющейся государствен- ным эталоном длины в СССР. Единица массы Килограмм * кг kg Единица массы, определяемая платино-иридиевой мерой, носящей знак 12 и являющейся государствен- ным эталоном массы в СССР. Единица времени Секунда сек. или с. sec или s Единица времени -(среднего сол- нечного), равная ста’ шестнадцати миллионам пятидесяти семи тысячам шестьсот двадцати восьми десяти- биллионным (0,0000116057628) сред- них звездных суток. Единица электриче- ского тока Ампер а А Неизменяющийся электрический ток, который, протекая по каждому из двух бесконечно длинных парал- лельных прямолинейных проводни- ков ничтожно малого кругового се- чения, расположенных в безвоздуш- ном пространстве на расстоянии 2 метров друг от друга, создает между этими проводниками на каж- дый метр их длины силу, равную од- ному ньютону. 1L Механические единицы Единица скорости Метр в секунду м/сек m/sec Скорость равномерно и прямоли- нейно движущейся точки, переме- щяющейся на 1 метр в 1 секунду. Единица ускорения Метр на секунду в квадрате м/сек? m/sec2 Ускорение равномерно-ускоренно и прямолинейно движущейся точки, скорость которой изменяется на 1 метр в секунду за 1 секунду. Единица силы Ньютон н n Сила, сообщающая массе в 1 ки- лограмм ускорение в 1 метр на се- кунду в квадрате. 69
Продолжение •• Сокращенное обозначение Наименование Определение русским алфавитом латинским (греческим) алфавитом Единица работы Джоуль или ватт- секунда дж вт-с J W-s Работа, производимая силой в 1 ньютон при перемещении точки приложения этой силы на 1 метр по ее направлению. , Единица мощности Ватт вт W Мощность, при которой в течение 1 секунды равномерно производится работа в 1 джоуль. Примечание. Сила в 1 ньютон = 105 дин = 0,102 килограмма (силы). Работа в 1 джоуль = 1 ньютон X 1 метр = 10б дин X Ю2 сантиметров = = 107 эрг. зация записи некоторых исходных уравнений электротехники пу- тем введения в них множителя 4^. Благодаря этому наиболее часто применяемые расчетные формулы электротехники избав- ляются от коэффициента 4к и становятся более удобными* для применения и запоминания. В нера цион а л и з и р о в а иной же системе единиц МКСА множитель 4т: отсутствует в мало применяемых для расчета формулах и, наоборот, имеется в, широко применяемых. Преимущество рационализированной системы перед нерационализированной заключается в том, что при рациональной форме записи уравнений электромагнитного поля уравнения электрического и магнитного полей аналогичны по построению. Поэтому мы будем в настоящей книге применять систему единиц МКСА при рациональной записи уравнений элек- тротехники. Помимо системы единиц МКСА, в научной и технической ли- тературе широко применяются так называемые абсолютная электростатическая (СГСЭ) и абсолютная электромагнитная (СГСМ) системы единиц. В абсолютной электростатической системе единиц (СГСЭ) за единицу длины принимается 1 сантиметр (1 см), за единицу массы — масса 1 грамма (1 г), за единицу времени—1 секунда (1 сек.); одновременно с этим диэлектрическая проницаемость «пустоты» s0 принимается равной отвлеченной (безразмерной) единице. В соответствии с этим абсолютная электростатическая система единиц сокращенно обозначается символом СГСЭ, где С — сантиметр, Г — грамм, С — секунда, Э — электростатиче- ская. В абсолютной электромагнитной системе единиц (СГСМ) за единицу длин# принимается 1 сантиметр (1 см)^ за единицу «О
массы — Масса 1 грамма (1 г), за единицу времени—1 секунда (1 сек.); одновременно с этим магнитная проницаемость «пу- стоты» до принимается равной отвлеченной (безразмерной) еди- нице. В соответствии с этим абсолютная ’электромагнитная си- стема единиц сокращенно обозначается символом СГСМ (С — сантиметр, Г—грамм, G — секунда, М — магнитная). В конце книги приведена .сводная таблица (приложение, разд. II), в которой даны соотношения между единицами МКСА, СГСЭ и СГСМ систем, единиц. Воспользовавшись основными единицами системы единиц МКСА, мы можем дать определения всем прочим единицам из- мерения электрических и магнитных величин, с которыми будем встречаться при дальнейшем изложении настоящего Курса элек- тротехники. § 14. ЭЛЕКТРИЧЕСКИЙ ЗАРЯД ФИЗИЧЕСКОГО ТЕЛА В предыдущей главе мы выяснили, что всякое вещество со- стоит из мельчайших частиц, называемых атомами. Каждый атом представляет собой систему, состоящую из' заряженного положительным электричеством центрального ядра и вращаю- щихся вокруг него электронов, обладающих отрицательным электричеством. Если в атоме имеется поровну электронов и протонов и они размещены так, что электрически взаимно компенсируют друг друга, то такой атом по отношению к внешней среде электриче- ски нейтрален. Это значит, что всякий электрический заряд, расположенный вне данного атома, не будет испытывать ца себе действие его электрических зарядов (электронов и про- тонов). Если атом теряет хотя бы один из своих электронов, то он становится положительным ионом. Разобщенные электрические заряды — электрон, покинувший пределы атома, и образовав- шийся положительный ион — уже не в состоянии уравновесить друг друга. Они стремятся вновь соединиться и образовать ней- тральный в электрическом отношении атом. Атом, потерявший один электрон, превращается в однозаряд- ный положительный ион; атом, потерявший два электрона,— в двухзарядный положительный ион и т. д. Наоборот, атом, получивший один лишний электрон, превра- щается в однозарядный отрицательный ион, а получивший два лишних электрона — в двухзарядный отрицательный ион и т. д. Следовательно, величина и знак электрического заряда иона зависят от величины избытка или недостатка электронов в атоме, из которого образовался данный ион. Теперь допустим, что в каком-нибудь физическом теле, на- пример а стекле, удалось изъять из значительного числа атомов 61
часть их электронов. Это значит, что стекло, потеряв часть своих электронов, окажется заряженным положительным элек- тричеством, так как в нем положительные заряды получили пе- ревес над отрицательными. Изъятые из стекла электроны исчезнуть не могут и должны быть где-то размещены. Допустим, что после того как электроны были изъяты из стекла, они оказались размещенными на метал- лическом шарике. Тогда очевидно, что металлический шарик, получивший лишние электроны, зарядился отрицательным элек- тричеством, так как вч нем отрицательные заряды получили пере- вес над положительными. Наэлектризовать физическое тело — значит создать в нем избыток или недостаток электронов, т. е. нарушить в нем равно- весие двух противоположностей, а именно положительных и от- рицательных 'зарядов. Наэлектризовать два физических тела одновременно и со- вместно разноименными электрическими зарядами — значит изъять из одного тела электроны и передать их другому телу. Если где-либо в природе образовался положительный элек- трический заряд, то одновременно с ним неизбежно должен воз- никнуть такой же по абсолютной величине отрицательный заряд, так как всякий избыток электронов в любом физическом теле возникает за счет недостатка их в каком-нибудь другом физиче- ском теле. Разноименные электрические заряды вы- ступают в электрических явлениях как неиз- менно сопутствующие друг другу противопо- ложности, единство и взаимодействие которых составляет внутреннее содержание электри- ческих явлений в веществах. Указание на то, что положительное электричество противопо- ложно отрицательному, мы находим у Ленина в «Философских тетрадях». Перечисляя важнейшие противоречия в природе и обществе, Ленин пишет: «В математике + и —. Дифференциал и интеграл. » механике действие и противодействие. » физике положительное и отрицательное электричество. » химии соединение и диссоциация атомов. ’ » общественной науке классовая борьба». /В. И. Ленин. Философские тетради. Изд. ОГИЗ, 1947 г., стр. 327.) Способов электризации физических тел достаточно много. Здесь мы ограничимся только кратким описанием некоторых из них. 1. Если цинковую пластину погрузить в раствор серной кис- лоты H2SO4, то она частично в нем растворится. Часть атомов цинковой пластины, оставив по^ава своих электрона на цинковой 62
пластине, перейдет в раствор серной кислоты в виде двухзаряд- ных положительных ионов цинка Zn+Л В результате цинковая пластина зарядится отрицательным электричеством (избыток электронов), а раствор серной кислоты — положительным (избы- ток положительных ионов цинка). Это явление электризации цинка в растворе серной кислоты использовано в гальваническом элементе как основной процесс возникновения электрической энергии. 2. Если на поверхности таких металлов, как цинк, цезий и некоторые другие, падают лучи света, то с этих поверхностей вы- деляются свободные электроны в окружающую среду. В резуль- тате металл заряжается положительным электричеством, а окру- жающее его пространство — отрицательным. Испускание элек- тронов освещенными поверхностями некоторых металлов назы- вается фотоэффектом, нашедшим <&бе применение в фото- элементах. Явление фотоэффекта было открыто русским ученым А. Г. Столетовым. Им был Построен первый фотоэлемент, послу- живший прототипом современных фотоэлементов. 3. Если металлическое тело нагреть до состояния белого ка- ления, то с его поверхности будут вылетать свободные элек- троны в окружающее пространство. В результате этого металл, потерявший электроны, зарядится положительным ,электриче- ством, а окружающая среда — отрицательным. Термоэлек- тронная эмиссия металла используется в электронных лампах. 4. Если спаять концы двух разнородных проволок, например висмутовой и медной, и место их спая нагреть, то свободные электроны частично перейдут из медной проволоки на висмуто- вую. В результате медная проволока зарядится положительным электричеством, а висмутовая — отрицательным. Явление элек- тризации двух физических тел при поглощении ими тепловой энергии используется в термоэлементах. 5. Опытом установлено, что при трении разнородных тел друг о друга они заряжаются равными по абсолютной величине, но различными по знаку электрическими зарядами. Если, например, стеклянную палочку потереть куском шел- ковой материи, то стекло зарядится положительным электриче- ством, а шелковая материя — отрицательным. Это значит, что со стекла часть электронов перешла на шелк- и в первом из них получился недостаток электронов, а во втором — избыток их. Подобный же опыт можно провести и с рядом других тел; например, янтарь, сургуч, эбонит при трении их о мех заря- жаются отрицательным электричеством, а мех — положи- тельным. Знак электрического заряда, получающегося при трении тел, зависит от природы этих тел, от состояния их поверхностей и от ряда других причин. Поэтому не исключена возможность, что 63
одно и то же физическое тело может в одном случае зарядиться положительным, а в другом — отрицательным электричеством; например, металлы при трении их о стекло и шерсть электри- зуются отрицательно, а при трении о каучук — положительно. Независимо от того, по какому принципу работает тот или иной источник электрической энергии, в каждом из них происхо- дит процесс электризации физических тел, т. е. разделение элек- трических зарядов, имеющихся в источнике электрической энер- гии, и сосредоточение их на определенных местах, например на электродах или зажимах источника электрической энергии. В результате этого процесса на одном из зажимов источника электрической энергии (катоде) получается избыток отрицатель- ных зарядов (электронов), а на другом зажиме (аноде) —недо- статок электронов, т. е. первый из них, заряжается отрицатель- ным, а второй — положительным электричеством. $ 15. ЕДИНИЦЫ ИЗМЕРЕНИЯ ЭЛЕКТРИЧЕСКОГО ЗАРЯДА Электрическим зарядом, или, иначе, количеством электриче- ства, данного физического тела называется величина избытка или недостатка электронов в этом теле. Измерить электрический заряд — это значит сравнить его с другими зарядами, принятыми за единицу. Элементарным отрицательным электрическим зарядом яв- ляется электрический заряд электрона, а элементарным положи- тельным зарядом — заряд позитрона (протона). Казалось бы, что в качестве единиц измерения отрицательных и положитель- ных электрических зарядов можно было взять соответственно электрические заряды электрона и позитрона. Однако- числен- ное значение этих зарядов очень мало и пользоваться ими для измерения электрических зарядов в практической электротехнике крайне неудобно. Например, через нить электрической лампочки средней мощности ежесекундно проходит несколько биллионов (миллиардов миллиардов) электронов. В абсолютной практической электромагнитной системе еди- ниц МКСА за едницу измерения величины электрического заряда (количества электричества) принят 1 кулон («, С). Между кулоном и зарядом электрона существует следующее численное соотношение: 1 кулон = 6,29 • 1018 зарядов электрона, или один кулон электричества численно равен. 6,29 биллионам зарядов электрона. В табл. 3 приведены единицы измерения количества электри- чества, их условные обозначения и соотношения их с основной единицей. 64
Таблица 3 Единицы измерения количества электричества в системе единиц МКСА Наименование величины и ее обозначение Название единицы Обозначение Соотношение с основной единицей русское междуна- родное кулон К С — Количество электричества q • милликулон МК шС 10-8 К микрокулон мкк иС 10-6 к Между одной из основных величин системы единиц МКСА— током 1 ампер — и электрическим зарядом 1 кулон существует простое соотношение: > > 1 ампер 1 кулон 1 секунда ’ т. е. если через поперечное сечение проводника ежесекундно про- ходит один кулон электричества, то ток в проводнике равен одному амперу. Примечание. В абсолютной электростатической системе единиц СГСЭ за единицу измерения электрического заряда принимается 1 абсо- лютная электростатическая единица заряда (количество электричества), которая специального названия не имеет. Между 1 кулоном и 1 СГСЭ единицей электрического заряда суще- ствует такое соотношение: 1 кулон = 3 • 109 СГСЭ единиц электрического заряда.
ГЛАВА III ЭЛЕКТРИЧЕСКОЕ ПОЛЕ ЭЛЕКТРИЧЕСКИХ ЗАРЯДОВ § 16. ПОНЯТИЕ ОБ ЭЛЕКТРИЧЕСКОМ ПОЛЕ ЭЛЕКТРИЧЕСКИХ ЗАРЯДОВ Всякая совокупность электрических зарядов обладает элек- трическим полем и совместно с ним образует единую материаль- ную систему. Следовательно, всякое изменение взаимного расположения электрических зарядов в данной системе, равно как и изменение величин этих зарядов во времени, неизбежно связано с измене- нием их электрического поля. Электрическое поле — это такой вид материи, через который осуществляется физический процесс взаимодействия электриче- ских зарядов — взаимное притяжение разноименных электриче- ских зарядов и отталкивание одноименных. Если взаимодействие электрических зарядов осуществляется через их собственное электрическое поле, значит, поле действует непосредственно на эти заряды. В природе имеется бесконечное многообразие электрических полей, отличающихся друг от друга по своей конфигурации, по силе, по электрической энергии, запасенной в них, и т. д. Для исследования электрических полей обычно применяют пробный электрический заряд, т. е. единичный по- ложительный заряд, настолько малый по величине, что его при- сутствие в исследуемом электрическом поле практически не влияет ни на силу, ни на энергию, ни на конфигурацию этого поля. Если пробный электрический заряд внести в электрическое поле, то на него будет действовать сила поля, или, иначе говоря, электрическая сила, стремящаяся перемещать его в строго опре- деленном направлении. Траектория, которую опишет пробный электрический заряд при своем перемещении под действием сил поля, называется электрической силовой линией. Направление электрической силовой линии совпадает с на- правлением движения вдоль нее пробного электрического заряда. 66
Следовательно, электрическая силовая линия — это линия геометрическая, в любой точке которой сила поля F9 (электрическая сила), дей- ствующая на пробный электрический заряд?, совпадает с напра в лениемк а с ательной к этой линии (рис. 17). Электрические силовые линии начинаются на положительных электрических зарядах и оканчиваются на , отрицательных. Это понятно, так как пробный электрический заряд под действием поля всякий раз будет перемещаться от положительного заряда Рис. 18. Силовые линии электриче- ского поля направлены от положи- тельных зарядов к отрицательным Рис. 17. Электрическая силовая линия к отрицательному. На рис. 18 изображены силовые линии элек- трического поля системы двух разноименных электрических за- рядов. Здесь мы видим, что электрические силовые линии имеют направление от положительного заряда к отрицательному, что и отмечено на этих линиях условными стрелками. Графическое изображение электрического поля с помощью электрических силовых линий облегчает изучение этого поля. Во-первых, направление электрических силовых линий дает пред- ставление о направлении поля и, следовательно, о направлении действия электрических сил в каждой точке поля; во-вторых, гу- стота расположения электрических силовых линий, как это мы увидим далее, характеризует величину силы поля. Всякое электрическое поле, как некоторый вид материи, об- ладает энергией. Величина энергии электрического поля заря- дов зависит от количества электрических зарядов, создающих это поле, от их взаимного расположения, от физических свойств той среды, где существует это поле. Следовательно, при всяком изменении взаимного расположения электрических зарядов в поле энергия этого поля как-то должна изменяться по своей величине. 5* 67
Если электрические заряды перемещаются под влиянием сил поля, например разноименные элек- трические заряды сближаются или одноименные удаляются, то электрическая энергия этого поля убывает, так как в этом случае она преобразуется в иные виды энергии. Наоборот, если какие-либо внешние силы, называемые обычно сторонними силами FCTf будут удалять друг от друга разноименные электрические заряды или сближать одноимен- ные, то они, преодолевая сопротивление этому движению со сто- роны сил поля зарядов F3, будут совершать некоторую работу за счет расхода какой-нибудь иной энергии, например химиче- ской, механической и т. д. В результате этого энергия электриче- ского поля зарядов возрастет. Силы по^ля зарядов F3 и внешние (сторон- ние) силы FCT в рассматриваемом нами явлении высту- пают как силы противоречивые, стремящиеся перемещать электрические заряды в прямо противоположных направлениях. От соотношения величин этих сил зависят характер и направление движения электрических зарядов в поле. Всякое электрическое поле зарядов занимает некоторую об- ласть пространства. Однако оно не является резко ограниченным материальным объектом. Если говорят, что электрическое поле зарядов занимает некоторую область пространства, то под этим подразумевают, что во всех прочих областях пространства оно настолько слабо себя проявляет, что действием сил его там можно практически пренебречь. О наличии электрического поля зарядов в данной области пространства можно судить по силам поля, которые действуют на электрические заряды, имеющиеся в этом поле. § 17. ЭЛЕКТРОСТАТИЧЕСКОЕ ПОЛЕ В природе все движется, все изменяется. В ней нет и не мо- жет быть застывших форм. Это в полной мере относится и к лю- бой совокупности взаимодействующих электрических зарядов, связанных электрическим полем в единую материальную си- стему. Вся совокупность электрических зарядов вместе с их электрическим полем всегда перемещается в пространстве. Одно- временно с этим и внутри самой этой системы заряды как-то перемещаются относительно друг друга, изменяются с течением времени по величине, например в результате взаимной нейтрали- зации разноименных зарядов. В соответствии с этим и электри- ческое поле зарядов является в общем случае переменным по- лем, изменяющимся в пространстве и во времени. Однако в пре- делах этой формы движения материи, как и всякой иной, воз- можны случаи относительного покоя, т. е. в рассматриваемый отрезок времени изменяемость взаимного расположения электри- 68
ческих зарядов в поле и их величин настолько мала, чтб ею можно практически пренебречь. В этом случае с достаточной точностью можно считать, что рассматриваемая система электри- ческих зарядов является статической, а ее электрическое поле — электростатическим. Следовательно, электростатическим полем называется такое электрическое поле, которое создано относительно неподвижными в пространстве и практически неизменными во времени электри- ческими зарядами. Изучить электростатическое поле — это значит определить величины, характеризующие это поле в каждой его точке. Та- кими величинами, как это мы увидим далее, являются напря- женность и потенциал электрического поля. Зная их, можно определить силы, действующие на электрические заряды, поме- щенные в различные точки поляг работу сил поля при переносе зарядов из одной точки поля в другую, энергию, запасенную в электрическом поле, и ряд других физических величин. Изучение электростатических полей — весьма важная за- дача электротехники, так как на практике очень часто приходится иметь дело с подобными полями. В настоящей главе мы рассмо- трим основные величины, характеризующие электростатическое поле. § 18. ПОТЕНЦИАЛ ЭЛЕКТРОСТАТИЧЕСКОГО ПОЛЯ Допустим, что в некоторой области пространства имеется электростатическое поле, создаваемое системой относительно Рис. 19. Сторонняя сила Лст, перемещая электрический заряд q, вносит его в элек- трическое поле неподвижных в пространстве и неизменных во времени электри- ческих зарядов, а за пределами этого поля находится некоторый посторонний для этой системы электрический заряд q (рис. 19). Приложим к заряду q стороннюю силу FCT, которая внесет его в заданное электростатическое поле и будет перемещать его 69
там, преодолевая сопротивление силы электростатического поля Р9. Следовательно, сторонняя сила, перемещая электриче- ский заряд q в поле, будет совершать некоторую работу, а энер- гия электростатического поля в соответствии с этим будет не- прерывно возрастать. Если сторонняя сила, переместив заряд q в точку 1 электро- статического поля, совершила работу то соответственно энер- гия электростатического поля возросла на величину Wb равную этой работе. Теперь допустим, что в точку 1 электростатического поля внесен единичный положительный заряд. В соответствии с этим приращение энергии электростатического поля изменится в q раз по сравнению с предыдущим случаем, когда в точку I поля был внесен заряд q. Приращение энергии электростатического поля, отнесенное к единичному положительному заряду, в этом случае можно выразить следующей формулой: = (4) где — приращение энергии электростатического поля, отне- сенное к единичному положительному заряду; — приращение энергии электростатического поля, связан- ное с помещением в точку 1 поля электрического за- ряда q. Приращение энергии электростатического поля, приходящееся на единичный положительный заряд, помещенный в исследуе- мую точку поля, называется электрическим потенциалом этой точки поля. Следовательно, если положительный электрический заряд q своим присутствием в некоторой произвольной точке N поля уве- личивает энергию этого поля на величину 11^, то потенциал этой точки поля равен WN ^=7' (5) Так как приращение энергии электростатического поля чис- ленно равно работе сторонней силы, затраченной при переносе электрического заряда из-за пределов поля в данную точку его, то можно сказать, что потенциал данной точки электростатиче- ского поля численно равен той работе, которую должна совер- шать сторонняя сила при переносе единичного положительного заряда из-за пределов поля в данную его точку. Исходя из формулы (5), можно определить размерность по- тенциала 70
т. е. размерность потенциала в системе единиц МКСА выра- жается в джоулях на кулон или, иначе, в вольтах, так как джоуль на кулон иначе называется вольтом. В соответствии с этим за единицу измерения потенциала в си- стеме единиц МКСА принят 1 вольт (1 в , 1 V). Если в формуле (5) положить = 1 джоуль и q= 1 кулон, то получим . 1 джоуль 1 ВОЛЬТ = -i----—, 1 кулон ’ т. е. потенциал данной точки равен одному вольту, если при переносе в нее из-за пределов поля электрического заряда, рав- ного одному кулону, сторонняя сила совершила работу, равную одному джоулю. Пример 2. Определить потенциал точки 1 электростатического поля, если известно, что при переносе из-за пределов поля в эту точку электрического заряда </ = 2‘10~8 к сторонняя сила совершила работу /11 =4* 10“6 дж. Решение. Так как электрический заряд q перемещался в поле сторон- ней силой, то энергия поля увеличилась на величину, равную = Л = 4.10-5 дж. В соответствии с этим потенциал исследуемой точки поля будет равен Wi 4-10-5 ?> = ^ = 1По^ = 2ОООв- В табл. 4 приведены единицы измерения потенциала электро- статического поля и их соотношение с основной единицей. Таблица 4 Единицы измерения потенциала электростатического поля в системе единиц МКСА Наименование величины и ее обозначение Название единицы Обозначение Соотношение с основной единицей русское междуна- родное вольт в V — Потенциал <? киловольт Кв kV 103 в милливольт Мв mV IO-3 в микровольт мкв pV 10-6 в Если электростатическое поле создано совокупностью только положительных электрических зарядов, то потенциалы всех его точек положительны, так как при переносе единичного положи- тельного заряда из-за пределов поля в любую точку его сторон- 71
няя сила будет совершать работу, которая идет на приращение потенциальной энергии поля. Если, наоборот, электростатическое поле создается совокуп- ностью отрицательных электрических зарядов, то потенциалы всех его точек отрицательны, так как при переносе единичного положительного заряда из-за пределов поля в любую точку его будет работать не сторонняя сила, а сила электростатического поля за счет расхода энергии этого поля. Следовательно, работа сил поля в данном случае отрицательна, поскольку она связана с убылью энергии электростатического поля. А это значит, что потенциалы всех точек электростатического поля, созданного со- вокупностью отрицательных зарядов,— величины отрицательные. Пример 3. Требуется определить потенциал точки 1 электростатического поля, если известно, что при переносе из-за пределов поля в данную точку его 1 положительного заряда q = 4 • 10~8 к сила поля совершила работу Ai = 6 • 10~6 дж. Решение. Так как в рассматриваемом случае работала сила поля, то работа ее связана с убылью энергии электростатического поля, т. е. она яв- ляется отрицательной величиной: = — 6.10-6 дж. В соответствии с этим потенциал точки 1 поля равен Wt —6-10-6 q “ 4-10—8 = —150 в. Итак, мы видим, что потенциал электростатического поля может быть как положительной, так и отрицательной величи- ной, т.. е. это величина алгебраическая. Естественно, возникает вопрос, какие же точки электростатического поля имеют потен- циалы, равные нулю. Условно принято считать, что потенциал точек поверхности земли равен нулю. Следовательно, всякий металлический про- водник, соединенный с землей (заземленный проводник), имеет потенциал, равный нулю. Всякий положительный потенциал больше потенциала земли и отрицательного потенциала, а потен- циал земли в свою очередь больше отрицательного потенциала. Потенциал электростатического поля можно сравнить с высо- той подъема физического тела. Подобно тому как высоту подъема физического тела, например высоту вершины горы, можно сравнивать с уровнем поверхности воды в океане, который условно принят равным нулю, так и потенциалы любых точек электростатического поля можно сравнивать с потенциалом земли, условно принятым равным нулю. Геометрическое место точек электростатического поля, обла- дающих одинаковыми по величине потенциалами, называется равнопотенциальной (эквипотенциальной) по- верхностью. Примером равнопотенциальной поверхности может служить поверхность всякого статически заряженного металличе- ского проводника. 72
§ 19. НАПРЯЖЕНИЕ В ЭЛЕКТРОСТАТИЧЕСКОМ ПОЛЕ Сила, электрического поля перемещает по- ложительные электрические заряды всегда от точек с более высокими потенциалами к точкам с меньшими потенциалами, т. е., иначе, в направлении падения (убывания) потенциала; наоборот, от- рицательные электрические заряды она пере- мещает от точек с менее высокими потенциа- лами к точкам с более высокими потенциалами. Как в первом, так и во втором случае перемещение электриче- ских зарядов связано с расходом энергии электростатического поля. Чтобы подсчитать работу чсилы поля при переносе электриче- ских зарядов из одних точек поля в другие, необходимо знать разность потенциалов, или напряжение между данными точками поля. Напряжением между двумя любыми точками электростатиче- ского поля называется алгебраическая разность потенциалов этих точек поля Ц,2 = <Р1 — ?2, (6) где —напряжение (разность потенциалов) между точками 1 и 2 электростатического поля в вольтах; epi—потенциал точки 1 электростатического поля в вольтах; ?2 — потенциал точки 2 электростатического поля в воль- тах. Формуле (6) можно придать такой вид: =^’. Р) где Ц,2 — напряжение между точками 1 и 2 электростатического поля; q— электрический заряд, переносимый силой поля из точки 1 в точку 2 поля; А,2 — работа, совершенная силой поля при переносе заря- да q из точки 1 в точку 2 поля. На основании формулы (7) ♦ можно сделать такой вывод: напряжение между любыми двумя точками электростатического поля численно равно той работе, которую совершает сила поля при переносе единичного положительного заряда из одной точки поля в другую. Если в формуле (7) положить, что Лъ2 = А — = 1 дж и q = 1 к, то получим 1 1 джоуль 1 вольт = я-----— , 1 кулон ’ т. е. напряжение между двумя точками поля равно одному вольту, если при переносе одного кулона электричества из одной 73
точки поля в другую сила поля совершает работу, равную одному джоулю. Пример 4. Требуется определить напряжение между двумя точками элек- тростатического поля, если известно, что сила поля, перенося электрический заряд q = 5 • 10~8я из одной данной точки поля в другую, совершила работу Л1>2 = 8- 10“б дж. Решение. Применяя формулу (7), находим A,s 8-Ю”6 ^ = —= ЗЛо^= 1600 9- § 20. РАБОТА СИЛЫ ЭЛЕКТРОСТАТИЧЕСКОГО ПОЛЯ Если известно напряжение Ц>2 между двумя любыми точками электростатического поля, то нетрудно подсчитать величину ра- боты силы поля при переносе электрического заряда q из одной заданной точки поля в другую, так как эта работа будет в q раз больше, чем при переносе единичного заряда, т. е. А,2 = ^Л,2 Я = (?i ?г) Яу (8) где Л1>2 — работа силы поля при переносе электрического за- ряда q из точки 1 электростатического поля в точку 2 в джоулях; £7Ь2 — напряжение между точками 1 и 2 электростатического поля в вольтах; q—электрический заряд в кулонах. Пример 5. Определить, какую работу совершила сила электростатиче- ского поля, если ею был перемещен заряд <? = 4- 10~6 к из одной точки поля в другую, между которыми напряжение равно Uit2 = 3000 в. Решение. Применяя формулу (8), находим А,2 = £ЛЛ = 3-103-4-10~8= 1,2-10—2 дж. Из формулы (8) следует, что работа силы электростатиче- ского поля не зависит от конфигурации пройденного зарядом в поле пути, а зависит от величины этого заряда q, от разности потенциалов между точками начального и конечного положений Рис. 20. Работа силы поля не за- висит от пути, вдоль которого перемещается заряд электрического заряда q в поле. Следовательно, по какому бы из путей, /, II или III (рис. 20), ни перемещался электрический за- ряд q из точки М в точку N поля, величина работы силы поля остается одинаковой: Amn — (?Л4 — ?лг) #• Если бы мы пожелали элек- трический заряд q возвратить из 74
точки М в точку М электростатического поля (рис. 20), то вы- нуждены были бы приложить к нему стороннюю силу. Эта сила, перемещая электрический заряд q из точки N в точку Л4, будет совершать некоторую работу, так как на пути движения заряда она должна будет преодолеть сопротивление силы поля. Вели- чина работы сторонней силы в этом случае будет равна Ann. = (?2v — ?м) Я = — (?Л1 — ?zv) Я — — Аилг, т. е. она численно равна работе силы поля AMN, но противопо- ложна ей по знаку, так как работа силы поля связана с умень- шением энергии электростатического поля, а работа сторонней силы — с увеличением энергии этого поля. Следовательно, суммарная работа силы поля и сторонней силы при переносе заряда дчиз точки М в точку N электроста- тического поля и обратно равна нулю: + ^NM = (?Л1 'P/v) Я + (?2V — <7=0. Отсюда вытекает весьма важный вывод, что работа силы электростатического поля при переносе электрического заряда по любому замкнутому контуру в этом поле тождественно равна нулю. Поля, работа сил которых по любому замкнутому контуру тождественно равна нулю, называются потенциальными полями. Следовательно, электростатическое поле — потенциальное. Его можно сравнивать с потенциальным гравитационным полем (по- лем сил тяжести), в котором, как известно, работа сил при пере- носе физических тел зависит не от конфигурации пути, по кото- рому перемещается физическое тело, а от разности уровней между конечным и начальным положениями тела. В соответ- ствии с этим работа сил тяжести по любому замкнутому контуру тождественно равна нулю. § 21. НАПРЯЖЕННОСТЬ ЭЛЕКТРОСТАТИЧЕСКОГО ПОЛЯ Всякий электрический заряд, находящийся в электростатиче- ском поле, испытывает на себе действие силы этого поля. Если известна величина силы поля, действующей на единич- ный положительный заряд, помещенный в данную точку поля, то легко определить силу поля F3y действующую на любой электри- ческий заряд ?, помещенный в эту точку поля, так как величина силы F3 пропорциональна величине заряда q\ F3 = Eq, (9) где F3 — сила поля, действующая на электрический заряд, поме- щенный в данную точку поля; Е—сила поля, отнесенная к единичному положительному заряду, называемая напряженностью электростатиче- ского поля. 75
Из формулы (9) следует, что Е = (Ю) т. е. напряженность электростатического поля численно равна силе поля, действующей на единичный положительный заряд, поме- щенный в исследуемую точку поля. Исходя из формулы (10), определим размерность напряжен- ности электростатического поля: гр-. Г F9 П ньютон джоуль I J L <7 J кулон метр-кулон __ вольт•кулон вольт метр-кулон метр ’ т. е. напряженность электростатического поля в системе единиц МКСА выражается в вольтах на метр. В соответствии с этим за единицу измерения напряженности - вольт А в 1 V \ электростатического поля принимается I метр (^l—, 1— Если в формуле (10) положить, что = 1 ньютон и q = 1 кулон, то будем иметь Г 1 ньютон . вольт £ = п-------= 1 ------, 1 кулон метр * т. е. напряженность электростатического поля в данной его точке равна одному вольту на метр, если в этой точке поля на элек- трический заряд, равный одному кулону, действует сила поля, равная одному ньютону. В табл. 5 приведены единицы измерения напряженности элек- тростатического поля и их соотношения с основной единицей. Таблица 5 Единицы измерения напряженности электростатического поля в системе единиц МКСА Наименование величины и ее обозначение Название единицы Обозначение Соотношение с основной единицей русское междуна- родное Напряженность поля Е ... . 4 вольт на метр вольт на сантиметр в М в СМ V m V cm в 102 зг Пример 6. Определить напряженность электростатического поля в данной точке, если в ней на электрический заряд <7 = 5’10—8 к, действует сила поля F& = 2 • 10“4 ньютона. 76
Решение. Применяя формулу (10), находим Л Q 2-10—4 5.10—8 = 4000 — . м Зная величину напряженности электростатического' поля, можно по формуле (9) определить величину силы поля, дей- ствующей на любой электрический заряд, помещенный в дан- ную точку поля. Пример 7. Требуется определить силу поля, действующую на электриче- ский заряд # = 4’10“7я, помещенный в точку электростатического поля, где напряженность поля Е = 15 000 Решение. Исходя из формулы (9), находим Лэ = Eq = 15-103 • 4 -10“7 = 6« 10“3 ньютона. Так как напряженность электрического поля есть сила, отне- сенная к единице заряда, то она является величиной векторной, т. е. характеризующейся не только численным значением, но и направлением в пространстве. Направление вектора напряженности электрического поля в любой точке последнего совпадает с направлением действия электрической силы на единичный положительный заряд, поме- щенный в данную точку поля. Следовательно, вектор напряженности электри- ческого поля Е в любой точке совпадаете ка- сательной к силовой линии, проходящей че- рез эту точку (рис. 21). В соответствии с этим электриче- скую силовую линию называют также линией вектора напряженности электрического поля. Электростатическое поле в общем случае неоднородно, т. е. его напряженность изменяется по величине и направлению при переходе из одной точки в другую. Примером неоднородного электростатического поля может служить поле двух точечных разноименных электрических зарядов (см. рис. 18). Здесь напря- женность поля изменяется при переходе из одной точки поля в другую по величине и направлению. Там, где напряженность поля чис- ленно больше, электрические сило- вые линии расположены более гу- сто. При графическом изображении электростатического поля с помощью силовых линий плотность этих линий обычно подбирают такой, чтобы она в любом данном месте поля была пропорциональна напряженности по- ля в этом месте. Однородным электро- статическим полем назы- вается такое поле, в котором напря- Рис. 21. Линии вектора Е 77
Ё= const Рис. 22. Однород- ное электростати- ческое поле женность всюду одинакова по величине и на- правлению. В соответствии с этим электриче- ские силовые линии в однородном электроста- тическом поле направлены параллельно друг другу и расположены всюду с одинаковой плот- ностью. На рис. 22 графически изображено однородное электростатическое поле. § 22. ЗАКОН КУЛОНА Допустим, что задана совокупность двух неподвижных отно- сительно друг друга взаимодействующих электрических зарядов. Предположим также, что данные заряды точечные, т. е. они размещены на таких физических телах, линейные размеры кото- рых весьма малы по сравнению с расстояниями между этими телами. Так как рассматриваемые точечные заряды неподвижны от- носительно друг друга, то их совместное электрическое поле является электростатическим. Это поле действует на размещен- ные в нем электрические заряды, и через него осуществляется взаимодействие зарядов. Чем больше величина этих зарядов, тем при всех прочих равных условиях больше напряженность их электростатического поля и тем с большей силой оно действует на заряды. Следовательно, сила вза- имодействия электрических заря- дов находится в зависимости от величин этих зарядов. А это зна- чит, что величину этой силы. ^,1 1,2 ____о------------------------- Рис. 23. Сила взаимодействия двух одноименных электрических зарядов можно математически выразить через величины зарядов. Зависимость силы взаимодействия точеч- ных зарядов от их величин впервые была сформулирована фран- цузским ученым Ш. О. Кулоном (1736—1806) в 1775 г. Она по- лучила в науке название закона Кулона. Сила взаимодействия двух точечных зарядов пропорцио- нальна произведению величин этих зарядов и обратно пропор- циональна квадрату расстояния между ними. Закон Кулона в абсолютной практической рационализирован- ной системе единиц МКСА выражается следующей формулой: 4лег2 \ (И) где F — сила взаимодействия электрических зарядов в ньюто- нах (рис. 23); г — расстояние между зарядами в метрах; qb Яг — величины электрических зарядов в кулонах; е — диэлектрическая проницаемость среды. 78
Диэлектрическая проницаемость среды е— величина, характеризующая влияние этой среды на силу взаимодействия помещенных в нее электрических зарядов. Из формулы (11) сле- дует, что чем больше диэлектрическая проницаемость среды, тем при прочих равных условиях меньше сила взаимодействия элек- трических зарядов. Исходя из формулы (11), определим размерность диэлектри- ческой проницаемости е в системе единиц МКСА: Г qxq^ “I_ кулон-кулон ____ кулон2 |_4к/т2Д ньютон-метр2 джоуль 2 метр МеТР __ кулон2 _ _ кулон вольт-кулой-метр вольт-метр * Но величина в^льт' называется фарадой, а поэтому размер- ность диэлектрической проницаемости в системе единиц МКСА выражается в фарадах на метр Ч кулон ________фарада -I вольт-метр метр ‘ В соответствии с этим за единицу измерения диэлектрической проницаемости в системе единиц МКСА принята одна фарада 1л Ф 1 на метр (I-, 1-J. Диэлектрическую проницаемость е обычно выражают в виде произведения двух сомножителей: е = егео, (12) где в0 — диэлектрическая проницаемость вакуума, которая в ра- ционализированной МКСА системе единиц принимается равной ____ 1 фарада е° 4z-9-109 метр ’ ег— относительная диэлектрическая прони- цаемость среды — число отвлеченное, показы- вающее, во сколько раз диэлектрическая про- ницаемость данной среды больше, чем диэлек- трическая проницаемость вакуума. В табл. 6 приведены величины относительной диэлектриче- ской проницаемости наиболее употребляемых на практике ве- ществе 79
Таблица 6 Относительная диэлектрическая проницаемость некоторых веществ Наименование материала Относительная диэлектриче- ская прони- цаемость Наименование материала Относительная диэлектриче- ская прони- цаемость Асфальт естественный 2—4 Мрамор . 8—10 Базальт 10—12 Оргстекло 3,2—3,6 Бакелит 3,8—5,0 Парафин 2,2—2,3 Бумага кабельная су- Резина листовая . . . 2,6—3,5 хая 2,3—3,5 Слюда мусковит .... 6—7,5 Бумага, пропитанная Слюда флогонит .... 4—5,5 маслом 3,4—3,7 Стеатит 5,5—6,5 Воздух 1,0 Стекло 5,5—10 Воск пчелиный .... 2,8—2,9 Тиконд 60—80 Гетинакс, сорт А и Б 7—8 Фарфор 5—7,5 Дуб парафинированный 4,5—5 Фибра 3,5 Карболит 3—5 Шифер 4—16 Лакоткань 3,5—5 Церезин 2,1—2,3 Масло трансформатор- Эбонит 2,4—4,5 ное 2—2,5 Электрокартон ЭМ . . 2,5—4 Миканит . 4,6—6 Тибар 6000—8000 Пример 8. Определить силу взаимодействия двух точечных зарядов, qi = 4 • 10“б к и q% = 25 • Ю~6 к, помещенных в среду с относительной ди- электрической проницаемостью ег = 2 на расстоянии г = 0,1 м друг от друга. Решение. Применяя формулу (11), находим 1 4-10—6-25.10—6 :--------------- = 45 ньютонов. 4в.2. 1 пр 4я-9-109и ’ Исходя из формулы (И), можно определить напряженность электростатического поля, создаваемого уединенным точечным зарядом: 03) т. е. напряженность электростатического поля, создаваемого уединенным точечным зарядом, изменяется обратно пропорционально ква- драту расстояния от точки расположения за- данного точечного заряда. Пример 9. Определить напряженность электростатического поля в воз- духе (er = 1), создаваемого точечным уединенным зарядом q = 2 • Ю~8 к на расстоянии г = 1 м от него. Решение. Применяя формулу (13), находим р— ft _ 2-10-» _ion вольт £ ~ 4^ “ 1 16U метр * 4«-910» §0
Если электростатическое поле создано совокупностью точеч- ных электрических зарядов, то напряженность резуль- тирующего поля равняется геометрической Рис. 24. Напряженность электростатического поля Е равна геометрической сумме напря- женностей полей точечных зарядов сумме напряженностей’полей всех заданных точечных зарядов (рис. 24): ^рез—-^1 4" Е2 + ... + Еп. (14)
ГЛАВА IV ПРОВОДНИКИ и диэлектрики В ЭЛЕКТРОСТАТИЧЕСКОМ ПОЛЕ § 23. ПРОВОДНИКИ ПЕРВОГО РОДА В ЭЛЕКТРОСТАТИЧЕСКОМ ПОЛЕ I Допустим, что некоторый проводник А (рис. 25) заряжен от- рицательным электричеством и в электростатическое поле этого проводника вносится другой, незаряженный металлический про- водник Б. Назовем электростатическое поле проводника А по отношению к проводнику Б внешним полем и напряжен- ность его обозначим через £внеш- Это внешнее поле при внесе- нии в него проводника Б образуется не только вне последнего, но и внутри него. Свободные электроны проводника Б, очутив- шись во внешнем поле, будут испытывать на себе действие сил этого поля и начнут перемещаться вдоль металлического проводника от точек с меньшими потенциалами к точкам с боль- шими потенциалами, т. е. навстречу полю, с некоторой средней скоростью v3. Причиной возникновения электрического тока в металлическом проводнике явилось электрическое поле, под влиянием сил которого свобод- Рис. 25. Вблизи заряжен- ного проводника А на про- воднике Б появляется элек- тричество двух знаков Рис. 26. Внутреннее и внешнее поля в металлическом проводнике направлен^ навстречу друг другу
ные электроны получили спо- собность направленно и упо- рядоченно перемещаться в проводнике. По мере того как под дей- ствием сил внешнего поля поток свободных электронов движется вдоль проводни- ка, на одном из концов его появляется избыток электро- нов, а на другом конце на- блюдается их недостаток. Рис. 27. Внутреннее и внешнее поля взаимно уравновешивают друг друга А это значит, что в проводнике произошло перераспределение электрических зарядов, возникли два противоположных по знаку, но равных по величине электрических заряда. Проводник перестал быть электрически нейтральным, так как он зарядился двумя разноименными электрическими зарядами, размещенными на его концах, и сам стал источником электрического поля. Внутри про- водника это поле, как и всякое электрическое поле вообще, на- правлено от положительных зарядов к отрицательным. Назовем это поле в отличие от внешнего поля внутренним полем и напряженность его обозначим через Евпут. Нетрудно видеть, что внутреннее поле проводника, возникшее в нем в результате разделения его разноименных электрических зарядов, направлено навстречу внешнему полю, осуществившему это разделение зарядов. Из рис. 26 мы видим, что вектор напря- женности внешнего электрического поля ^неш направлен на- встречу вектору напряженности внутреннего поля EBliyT. Интен- сивность процесса электризации проводника зависит от соотно- шения двух противоречивых сил — сил внешнего и внутреннего электростатических полей. По мере того как электроны все более и более накапливаются на одном из концов проводника, а на другом конце увеличи- вается положительный заряд, растет напряженность внутреннего поля £внут. Если сила внешнего поля с течением времени не изменится, то противодействие сил внутреннего поля силам внешнего поля будет все более возрастать, и, наконец, наступит такой момент, когда эти силы сравняются, уравновесят друг друга, т. е. напряженности этих полей по величине станут рав- ными, но противоположно направленными (рис. 27):' I ^внут I 1 ^*внеш I • Тогда результирующая сила, действующая на свободные электроны в проводнике, будет равна нулю и направленное дви- жение потока свободных электронов вдоль проводника прекра- тится, т. е. исчезнет электрический ток проводимости. Хотя в проводнике вновь установилось состояние равновесия зарядов, но оно резко отличается по качеству от предыдущего 6* 83
состояния их равновесия. Теперь в нем взаимодействуют две противоположности: силы внешнего и внутреннего полей, направ- ленные навстречу друг другу, и на концах проводника сосредо- точены разноименные электрические заряды. Установившееся равновесие сил внешнего и внутреннего по- лей в проводнике будет наблюдаться до тех пор, пока силы внешнего поля сохранятся неизменными. Теперь допустим, что заряженный проводник А (см. рис. 25) начнет удаляться и вследствие этого внешнее поле внутри про- водника Б будет исчезать. В этом случае силы внутреннего поля в проводнике Б получат перевес над силами внешнего поля и поток свободных электронов вновь начнет направленное дви- жение. Но в отличие от предыдущего случая поток электронов будет теперь перемещаться с конца проводника, заряженного отрицательным электричеством, к концу проводника, заряженного положительным электричеством. Свободные электроны, переместившись на противоположный конец проводника, нейтрализуют имеющиеся там положительные заряды. Когда исчезнет внешнее поле, а вместе с ним и поле внутреннее, прекратится направленное движение потока свобод- ных электронов внутри проводника, т. е. электрический ток про- водимости. В проводнике вновь возникнет статическое состояние, но с совершенно новым качеством по сравнению с предыдущим. Теперь исчезло внешнее поле, а с ним вместе и внутреннее поле, а свободные электроны равномерно распределились по всей длине проводника. Рассмотренный процесс электрического тока проводимости в проводнике будет длиться весьма незначительное время, изме- ряемое долями секунды, так как свободные электроны очень быстро распределятся в проводнике и поле внешнее уравнове- сится полем внутренним. Для того чтобы электрический ток проводимости в провод- нике существовал продолжительное время, необходимо создать такие условия, чтобы электроны в проводнике в процессе их движения нигде не накапливались и внутри проводника по- стоянно действовало внешнее электрическое поле. Для этого необходимо выполнить особые условия, с которыми мы по- дробно ознакомимся впоследствии. Свободные электроны при своем беспорядочном (тепловом) движении в проводнике обладают относительно большими ско- ростями движения. Скорость же направленного движения потока свободных электронов в металлах, создающего электрический ток проводимости, относительно мала: она измеряется долями миллиметра в секунду. Однако электрическое поле распростра- няется вдоль проводников со скоростью, близкой к скорости рас- пространения света (300 000 км!сек). А это значит, что, каковы бы ни были размеры проводника, все его свободные электроны 84
практически одновременно придут в движение под влиянием возникшего внешнего поля. Явление наведения двух разноименных и равных по абсо- лютной величине электрических зарядов в проводнике в присут- ствии другого электрически заряженного проводника называется электростатической индукцией. Это явление легко продемонстрировать при помощи разъемного проводника 5, соединенного по концам с электроскопами (рис. 28). Если обе половины проводника Б соединены, то при поднесе- нии к нему проводника Л, заряженного отрицательным электри- чеством, листочки электроскопа разойдутся, что говорит о нали- чии зарядов на концах проводника Б. При удалении провод- ника А листочки электроскопа опадут, что говорит об исчезнове- нии зарядов с концов проводника Б. Рис. 28. На разъемном про- Рис. 29. Электроны с проводника воднике Б индуктировано Б ушли в землю . электричество двух знаков Если, не удаляя заряженного проводника А, разъединить обе половины проводника J5, то листочки электроскопа не опадут даже в том случае, если проводник А удалить. Значит, разъеди- ненные электрические заряды сохранились на изолированных друг от друга половинах проводника Б. Знаки зарядов на разноименно заряженных половинах про- водника Б можно определить, прикасаясь к ним телами, наэлек- тризованными положительно или отрицательно. При соприкосновении разноименно заряженных половин про- водника Б листочки электроскопов тотчас же сходятся, так как заряды на них, будучи противоположными по знаку и равными по абсолютной величине, взаимно нейтрализуются. Если к проводнику Б поднести проводник А, заряженный от- рицательным электричеством, и, не удаляя последний, соединить проводник Б с землей (рис. 29), то с проводника Б электроны уйдут в землю и на нем останется только положительный заряд. Последний связан с отрицательным зарядом проводником А, а поэтому нейтрализоваться зарядами земли он не может. После отсоединения проводника Б от земли и удаления заряженного проводника: А положительный заряд проводника Б сохраняется. 85
Рис. 30. Прибор А экранирован от внешнего электростатического поля Количество наведенного (ин- дуктированного) электричества на проводнике Б будет мень- ше, чем количество индукти- рующего электричества, имею- щегося на проводнике А. В част- ном случае, если проводник А окружить проводником Б, то на последнем индуктируют- ся разноименные электрические заряды, порознь равные по аб- солютным значениям индукти- рующему электрическому за- ' ряду. При электростатической индукции разделение электрических зарядов в проводнике происходит до тех пор, пока поле внеш- нее не будет полностью уравновешено полем внутренним. Отсут- ствие поля внутри проводника, находящегося в электростатиче- ском поле, может быть использовано для экранирования. Если необходимо освободить прибор от влияния посторонних электро- статических полей, то его обычно окружают металлическим про- водником, или, иначе, экраном (рис. 30). Электрические силовые линии внешнего поля в данном слу- чае не смогут проникнуть внутрь прибора. § 24. ДИЭЛЕКТРИКИ В ЭЛЕКТРОСТАТИЧЕСКОМ ПОЛЕ Допустим, что две металлические пластины, расположенные своими плоскостями параллельно друг другу, заряжены рав- ными по абсолютной величине, но противоположными по знаку электрическими зарядами +? и —q. Электростатическое поле, со- Рис. 31. Диэлектрик в электростатическом поле . 86
здаваемое этими зарядами, однородно. Внесем в это поле кусок плоского диэлектрика, боковые грани которого расположены па- раллельно заряженным металлическим пластинам (рис. 31). Под влиянием сил электростатического поля, создаваемого зарядами -\-q и —q металлических пластин, диэлектрик поляризуется. Это значит, что нейтральные в электрическом отношении молекулы диэлектрика превратились в электрические диполи или находя- щиеся в диэлектрике ранее в готовом виде диполи повернулись своими осями в направлении действия сил поля. Напомним, что электрическим диполем назы- вается 'совокупность двух равных по абсо- лютной велич и*не, но различных по знаку элек- трических зарядов, расположенных на очень малом расстоянии друг от друга по сравне- нию с расстоянием от диполя до точек на- блюдения. На рис. 32 показаны поле электрического диполя и поле внешнее, под влиянием _ сил которого данный диполь 1 \ образовался из нейтральной Г | / молекулы. \ I /\ / / Электрические диполи, ори- . \\\/ Xх"Т\ I / / ентировавшись своими осями в ^внеш Х\ направлении действия сил внеш- ** // него поля, образовали на боко- // /\ \ вых поверхностях диэлектрика 1 / \\. 7 / \ электрические заряды. При f 1 / I этом на левой стороне диэлек- ^в„еш \ • трика (см. рис. 31) образовал- ' ся поверхностный отрицатель- Рис. 32. Поле электрического диполя ный заряд, а на правой — по- ложительный. Эти заряды, в отличие от зарядов металлических пластин, называемых свободными,— связанные, так как их нельзя ни отделить друг от друга, ни отвести с диэлектрика. Это объясняется тем, что связанные заряды принадлежат электриче- ским диполям. Связанные электрические заряды диэлектрика образовали в нем свое собственное электростатическое поле, направленное навстречу полю внешнему. Это значит' что результирующее электростатическое поле в диэлектрике слабее, чем поле внеш- нее. Следовательно, диэлектрик с помощью своих связанных электрических зарядов ослабляет внешнее электростатическое поле. Картина этого ослабления поля внутри диэлектрика изобра- жена на рис. 31 с помощью силовых линий. Мы видим, что гу- стота их в пространстве между металлическими заряженными пластинами и диэлектриком больше, чем внутри диэлектрика, тай как часть силовых линий внешнего поля, заканчивая свой путь на связанных электрических зарядах диэлектрика, не про- никает в глубину последнего. Следовательно, напряженность 87
Рис. 33. Электростатическое поле точечного заряда, окруженного ша- ровым слоем диэлектрика электростатического поля в пространстве между металли- ческими пластинами и диэлек- триком будет больше, чем внутри этого диэлектрика. Чем больше диэлектриче- ская проницаемость диэлек- трика, тем больше (при всех прочих равных условиях) ве- личина его связанных элек- трических зарядов и тем, значит, слабее в нем резуль- тирующее электростатическое поле. Напряженность и потен- циал электростатического по- ля в диэлектрике (при всех прочих равных условиях) во столько раз меньше соответ- ственно напряженности и по- тенциала поля в вакууме, во сколько раз диэлектрическая прони- цаемость этого диэлектрика больше, чем диэлектрическая прони- цаемость вакуума. Пример 10. Точечный заряд q — 4 • 10 9 к помещен в среду с относитель- ной диэлектрической проницаемостью еГ1 = 1 и окружен шаровым слоем ди- электрика, имеющим относительную диэлектрическую проницаемость еГ2 = 5 и радиусы внутренней и внешней поверхностей, соответственно равные = м и /?2 = 0,15 м (рис. 33). Шаровой слой расположен симметрично относительно заряда. Определить напряженность электростатического поля, создаваемого данным зарядом в точках, удаленных от него на расстояния П = 0,05 м, г2 = 0,12 м и гз = 0,2 м. Решение. По формуле (13) находим: Е-----q — Л 2 4лег eorf Е — - q — 3 4’ter,e»'3 ---------= 14400^; ь^Н0-05)2 -- =500Л; -yOZ9.,. =900 — . 5- Е = 2 Так как источником электростатического поля являются как свободные, так и связанные электрические заряды, то электриче- ские силовые линии (линии вектора напряженности электроста- тического поля) могут иметь начало как на свободных, так и на связанных положительных зарядах, а конец — на свободных и связанных отрицательных зарядах. Это значит, что при пере- ходе потока электрических силовых линий (общего числа сило- 88
вых линий) через пло- I - скость раздела двух ди- 1 ^2 электриков с различными С2 На диэлектрическими прони- ! цаемостями часть линий а заканчивается на этой пло- скости, так как на ней имеются связанные элек- et трические заряды поляри- / 1 зованных диэлектриков. / | Но, как показывает опыт, при этом изменяется не только величина потока Рис. 34. Преломление электрической сило- силовых линий, но и об- вой линии щее направление его. Если силовая линия падает на плоскость раздела аЬ двух диэлектриков под углом ои к ее нормали (рис. 34), то по другую сторону плоскости раздела она выходит уже под углом аг к нор- мали. Установлено, что силовые линии испытывают преломление при переходе плоскости раздела двух сред с различными диэлек- трическими проницаемостями согласно следующему закону: tg = £i tg а2 е2 ' т. е. тангенс угла падения (угла ai) силовой линии так относится к тангенсу угла преломления (угла аг), как относятся диэлектри- ческие проницаемости соответствующих сред. Пример 11. Электрическая силовая линия падает из воздуха (sfi = 1) на плоскость диэлектрика (ef = 5) под углом ai = 45° (рис. 34) к ее нор- мали. Требуется определить угол преломления а2 этой силовой линии. Решение. Применяя формулу (15), находим 5 1 . вл 1 4тс • 9 • 10* . л to г л t tga2 = T--tgai =------i-----tg45° = 5-l =5. 61 1--------------- 4те-9-10° По тригонометрической таблице тангенсов для tg a2 = 5 находим, что Я2 = 78о50'. Из формулы (15) следует, что силовые линии при переходе из среды с большей диэлектрической проницаемостью в среду с меньшей диэлектрической проницаемостью отклоняются в сто- рону нормали к поверхности раздела сред.
ГЛАВА V ЭЛЕКТРИЧЕСКАЯ ЕМКОСТЬ § 25. ЭЛЕКТРИЧЕСКАЯ ЕМКОСТЬ ПРОВОДНИКА И ЕДИНИЦЫ ЕЕ ИЗМЕРЕНИЯ Опытом установлено, что между зарядом и потенциалом лю- бого уединенного металлического проводника существует прямая пропорциональная зависимость, т. е. во сколько раз увеличится или уменьшится заряд данного проводника, во столько раз соот- ветственно увеличится или уменьшится потенциал этого провод- ника. Эта пропорциональная зависимость между зарядом и по- тенциалом уединенного проводника выражается следующей фор- мулой: <7 = С<Р, (16) где q — электрический заряд проводника; ср — потенциал проводника; С — коэффициент пропорциональности, называемой электри- ческой емкостью или просто емкостью проводника. Из формулы (16) следует, что C = f. (17) т. е. емкость проводника равна отношению его заряда к потеш циалу. Это значит, что емкость проводника численно равна за- ряду, приходящемуся на единицу потенциала этого проводника. Следовательно, чем больше емкость проводника, тем большее необходимо при всех прочих одинаковых условиях сообщить ему количество электричества, чтобы поднять величину его потен- циала на единицу. Если один проводник имеет емкость в п раз большую, чем другой проводник, то при заряде их до одинакового потенциала потребуется сообщить первому из них количество электричества в п раз большее, чем второму. Следовательно, можно сказать, что емкость проводника есть величина, характеризующая способ- ность проводника накапливать в себе электрические заряды. 90
Исходя из формулы (17), определим размерность емкости: [С] = [у] = ^ = фарада’ т. е. емкость в системе единиц МКСА имеет размерность фа- рады. В соответствии с этим за единицу измерения емкости в си- стеме единиц МКСА принята 1 фарада (1 ф, 1 F). Если в фцрмуле (17) положить, что q = 1 кулон и <р = 1 вольт, то получим с==^ = _1кулон_ <р 1 ВОЛЬТ > т. е. проводник обладает емкостью в одну фараду, если при со- общении ему одного кулона электричества его потенциал изме- няется на один вольт. В табл. 7 приведены единицы измерения емкости и соотноше- ние между ними. Таблица 7 Единицы измерения емкости в системе единиц МКСА Наименование величины и ее обозначение Название единицы Обозначение Соотношение с основной единицей русское междуна- родное фарада ф F — Емкость С . . . . микрофарада мкф |xF пикофарада пф pF IO” 12 ф Пример 12. Требуется определить емкость проводника, если известно, что при сообщении ему электрического заряда q = 4 • 10“6 к потенциал его изменился на величину <р = 2« 103 в. Решение. Применяя формулу (17), находим С = — = = 2 • Ю-8^ = 0,02 мкф. <р 2 •IO* zr > J- Емкость проводника не зависит от его материала. Например, медный, свинцовый, серебряный и другие шарики одинакового радиуса обладают совершенно одинаковыми емкостями. Емкость проводника не зависит от его массы. Два провод- ника, имеющие одинаковые массы, но различные конфигурации, могут обладать различными емкостями. Емкость проводника зависит от величины его поверхности. Это и понятно, так как электрические заряды на проводнике в статическом состоянии размещаются только по его поверх- 91
ности, а поэтому чем больше поверхность проводника, тем легче разместиться на нем электрическим зарядам. На емкость проводника влияет среда, в которой размещен проводник. С ростом диэлектрической проницаемости среды емкость проводника растет в прямой пропорциональной зависи- мости. Если, например, проводник из воздуха перенести в керо- син, то его емкость увеличится примерно в два раза. На емкость проводника влияет соседство других проводников. Чем ближе размещать проводники один к другому, тем большей становится их емкость. В этом случае обычно говорят не о емкости одного проводника, а о емкости системы проводников. Емкость системы двух проводников численно равна тому за- ряду, который надо сообщить одному из этих проводников, чтобы разность потенциалов (напряжение) между ними изменилась на единицу. Емкость системы двух проводников равна одной фараде, если при сообщении одному из проводников одного кулона электри- чества разность потенциалов этих проводников изменится на один вольт. с= —q—= ± — ?2 С/ Если создать систему из двух проводников с большой поверх- ностью, расположив их на малом расстоянии друг от друга в среде с большой диэлектрической проницаемостью, то можно достигнуть значительной емкости этой системы. Приборы, осно- ванные на этом принципе, называются конденсаторами. Они по- лучили большое распространение в электротехнике. В ближай- ших параграфах мы остановимся на описании некоторых кон- струкций конденсаторов и на расчете их емкостей. Пример 13. Определить емкость С системы двух проводников, если при сообщении одному из них заряда q = 2 • 10“6 к разность потенциалов между ними возросла на U = 400 в. Решение. По формуле (18) находим С = -Sj = =0,5-10-8# = 0,005 мкф. § 26. ПЛОСКИЙ КОНДЕНСАТОР Плоским конденсатором называется прибор, состоящий из двух плоских металлических пластин А и Б (рис. 35), располо- женных параллельно друг другу и разделенных слоем диэлек- трика. Если плоский конденсатор присоединить к зажимам источ- ника постоянного тока (рис. 36), то пластины (обкладки) кон- денсатора зарядятся равными по абсолютной величине, но про- тивоположными по знаку электрическими зарядами. При этом обкладка конденсатора, подключенная к положительному за- 92
жиму источника, зарядится положительным электрическим заря- дом а обкладка, подключенная к отрицательному зажиму,— отрицательным зарядом —q. В соответствии с этим между об- кладками конденсатора, в его диэлектрике, возникнет однород- ное электростатическое поле, напряженность которого будет равна с d - (19) где Е — напряженность электростатического поля в диэлектрике конденсатора в вольтах на метр; U—напряжение на обкладках конденсатора в вольтах; d—расстояние между обкладками конденсатора в метрах. Рис. 36. Электростатическое поле плоского конденсатора Пример 14. К плоскому конденсатору приложено напряжение U = 250 в. Требуется определить напряженность электростатического поля в диэлектрике конденсатора, если расстояние между обкладками конденсатора d = 2 мм. Решение. Применяя формулу (19), находим ^ = 4 = ^ = 125000-1. Емкость плоского конденсатора зависит от площади обкладок, от расстояния между об- кладками и от диэлектрической проницае- мости диэлектрика: С—(20) где С — емкость плоского конденсатора в фарадах; е — диэлектрическая проницаемость диэлектрика в фарадах на метр; " 5 — площадь каждой из обкладок конденсатора в квадрат- ных метрах; d — расстояние между обкладками в метрах. 93
Пример 15. Определить емкость плоского конденсатора, у которого пло- щадь каждой из обкладок S = 75 см2, расстояние между обкладками d — = 2 мм и относительная диэлектрическая проницаемость диэлектрика е = 6. Решение. Применяя формулу (20), находим с с 6’"л—Л то"75-10~~4 С = ------= 2 • Ю'10^ = 200 пф. Cl U л' 1U Для увеличения емкости плоских конденсаторов их делают многопластинчатыми (рис. 37). Рис. 37. Плоский многопластинчатый конден- сатор (вид в разрезе) Емкость многопластинчатого плоского конденсатора опреде- ляется по формуле (21) где п — число всех обкладок конденсатора. Пример 16а. Требуется определить емкость плоского многопластинча- того конденсатора, имеющего п = 500 обкладок, если площадь каждой об- кладки S = 31,4 см2, расстояние между обкладками d = 0,005 см и относи- тельная диэлектрическая проницаемость диэлектрика = 5. Решение. Применяя формулу (21), находим С = (л-1).4=(л-1).^- = 5~. q ,п9 -31,4-10-* = (500 — 1)--- 5 ю-5---------= 1,38 • 1О-6д6 = 1,38 мкф- § 27. КОНДЕНСАТОРЫ А. ОСНОВНЫЕ ТИПЫ КОНДЕНСАТОРОВ И ИХ ПАРАМЕТРЫ В современной практике применяются самые разнообразные типы конденсаторов. С точки зрения конструктивного оформления конденсаторов они подразделяются на два основных типа: конденсаторы по- стоянной и переменной емкости. По типу диэлектрика конденсаторы подразделяются на бу- мажные, слюдяные, керамические, воздушные и электролитиче- ские. 94
Основными параметрами конденсатора являются номиналь- ная емкость, класс точности, электрическая прочность, сопротив- ление изоляции, потеря энергии. Номинальной емкостью конденсатора называется емкость, на которую рассчитан данный конденсатор при его изготовлении на заводе. Она согласно общесоюзному стандарту (ГОСТ 2519—49) обычно указывается в паспорте конденсатора. Конденсаторы, вы- пускаемые отечественной промышленностью, имеют номиналь- ную емкость от 1 пикофарады до 2000 микрофарад. Класс точности конденсатора определяется отношением пре- дела допустимого отклонения фактической емкости конденса- тора к номинальной емкости его, выраженной в процентах. К ну- левому классу точности относят конденсаторы, у которых допу- стимые отклонения фактической емкости от номинальной колеб- лются в пределах +2%, к первому классу точности — в пре- делах +5%, ко второму классу точности — в пределах + 10% и к третьему классу точности — в пределах +20 %. Электрической прочностью конденсатора называют то макси- мальное напряжение, при котором конденсатор . может работать продолжительное время (практически не менее десяти тысяч часов). Сопротивлением изоляции конденсатора называют величину сопротивления диэлектрика конденсатора прохождению через него постоянного тока (тока утечки). Чтобы ток утечки конденсатора был по возможности мал (практически равен нулю), сопротивление изоляции конденса- тора стремятся делать как можно большим (порядка сотен или тысяч мегомов). Потеря электрической энергии в конденсаторе обусловлена переменной поляризацией диэлектрика, в результате которой часть электрической энергии в диэлектрике преобразуется в тепло. Энергия, поглощаемая ежесекундно диэлектриком кон- денсатора, называется мощностью потерь конденсатора. Она от- носительно мала и в среднем составляет доли процента мощ- ности, развиваемой генератором электрической энергии в самохм конденсаторе. Б. КОНДЕНСАТОРЫ ПОСТОЯННОЙ ЕМКОСТИ Конденсаторы постоянной емкости не допускают изменения их емкости или ее регулировки, т. е. емкость их практически должна быть постоянной. Однако в процессе работы конденсаторов она все же может изменяться по ряду причин, и в частности от изме- нения температуры конденсаторов. В зависимости от рода диэлектрика конденсаторы постоян- ной емкости подразделяются на бумажные, слюдяные, керами- ческие, воздушные и электролитические. Бумажные конденсаторы. Бумажные конденсаторы имеют в качестве диэлектрика тонкую бумагу, пропитанную парафином. Материалом обкладок являются станиолевые ленты. ‘95
Способ изготовления бумажных конденсаторов в основном сводится к следующему. Две длинные станиолевые ленты изоли- руют друг от друга лентами тонкой папиросной, пропитанной па- рафином, бумаги. Затем эти ленты сворачивают в рулон, укла- дывают в металлический корпус и заливают парафином. Концы станиолевых лент присоединяют к зажимам конденсатора. На рис. 38 приведено несколько типов бумажных конденсаторов. Один из них показан в развернутом виде, так что в нем видны все основные детали. Широкое применение получили бумажные конденсаторы типа КБГ (конденсатор бумажный герметизированный). Корпуса для этого типа конденсаторов большей частью делаются металличе- скими, герметизация достигается путем тщательной пропайки Рис. 38. Различные типы бумажных конденсаторов швов корпуса и устройства выводов через стекловидную массу, делающую конденсатор непроницаемым для внешнего воздуха. Для бумажных конденсаторов малых емкостей иногда приме- няют корпуса цилиндрической формы из керамики. В табл. 8 приведены некоторые данные бумажных герметизи- рованных конденсаторов. На практике получили также широкое применение конденса- торы типа МБГ (металло-бумажные герметизированные конден- саторы). Отличительная особенность этих конденсаторов—та, что в них обкладки наносятся непосредственно на бумажный диэлектрик путем вакуумного испарения. В менее ответственных установках применяются бумажные конденсаторы типа КБ (конденсаторы бумажные негерметизиро- ванные). Бумажные конденсаторы изготовляются различных емкостей, от сотен пикофарад до нескольких десятков микрофарад. Рабочие напряжения, на которые рассчитаны бумаж- ные конденсаторы, находятся в пределах 200— 1500 в. При этом испытательное напряжение конденсаторов превышает их рабо- 96
Таблица 8 1377 Некоторые данные бумажных герметизированных конденсаторов Тип конденсатора Рабочее напряжение, в Конструктивное оформление 200 400 600 I 1000 ] | 1500 Емкость, мкф КБГ-И 4,7.10-4-4-0,1 4,7. ю-4-4-0,05 4,7-IO”4—-о,025 — — В цилиндрическом кор- пусе из керамики КБГ-М 0,025-4-0,25 0,01-4-0,25 0,01-4-0,2 — — В цилиндрическом ме- таллическом корпусе КБГ-МЛ 0,025-4-2,0 0,1-4-1,0 0,01-4-1,0 0,01—е—0,5 0,01-4-0,25 В металлическом кор- пусе плоской формы КБГ-МН 1,0-4-10,0 0,5-4-8,0 0,5Н-6,0 0,25-?-4,0 0,5-4-2,0 В металлическом кор- пусе нормальной прямо- угольной формы
чие напряжения примерно на 200—300%. Допустимые отклоне- ния от номинальной емкости для некоторых типов бумажных конденсаторов могут достигать 20%. Бумажные конденсаторы обладают рядом существенных не- достатков. Например, при токах высокой частоты в них стано- вятся относительно высокими потери энергии, индуктивное со- противление также сильно возрастает. Поэтому бумажные кон- денсаторы в цепях переменного тока высокой частоты, как пра- вило, не применяются. Слюдяные конденсаторы. Слюдяные конденсаторы соби- раются из отдельных полосок металлической фольги, между ко- торыми, прокладываются тонкие пластинки слюды, играющие роль диэлектрика. Для предохранения слюдяных конденсаторов от влияния внешних условий их герметизируют металлической обжимкой или опрессовывают пластмассой. В отличие от бумажных конденсаторов слюдяные конденса- торы имеют большое сопротивление изоляции (слюда) и благо- даря этому не имеют утечки тока. Они обладают большой элек- относительно малые потери энергии в диэлектрике. В силу этого слюдяные конденсаторы нашли достаточно широкое при- менение в радиотехнике, где, как известно, требуется высоко- качественная и стабильная ра- бота конденсаторов. Слюдяные конденсаторы из- готовляются различных емко- стей, от десятков до десятков тысяч пикофарад. Рабочее на- пряжение их колеблется от со- тен до нескольких тысяч вольт. Допустимое отклонение емко- сти от номинальной — от +9 % до ±20%. Наибольшее применение на практике получили следующие КСО (конденсаторы слюдяные опрессованные), СОМ (конденсаторы слюдяные опрессованные малогабаритные, КСГ (конденсаторы слюдяные герметизирован- ные) и СГМ (конденсаторы слюдяные герметизированные мало- габаритные). В табл. 9 приведены данные некоторых слюдяных конденса- торов.. На рис. 39 показан внешний вид нескольких типов слюдяных конденсаторов, применяемых в технике связи. трическои прочностью и имеют Рис. 39. Слюдяные конденсаторы типы слюдяных конденсаторов: 98
Таблица 9 Данные некоторых слюдяных конденсаторов Тип конденсатора Емкость, пф Рабочее напряжение, в Размеры, мм КСО-1 51—220 250 13X7X4,5 КСО-2 100—1000 Z 500 18X11X5,5 КСО-3 470-330 1 3600—6800 J 7500-10000 *500 250 20X20X9 СОМ-1 240-750 25 13,7X4,5 СОМ-2 750—2400 500 18ХПХ5,5 СОМ-3 2700-6200 6800-10000 / 500 I 250 J 20X20X6,5 КСГ-1 470—20000 500 26X24X14 КСГ-2 20000—100000 500 45X33X23 СГМ-1 100—560 250 13X9X5 СГМ-2 620—1200 250 13x9,6x6,6 СГМ-3 100—4300 500 18X13X7 СГМ-4 5800—40000 4700—6200 250 500 21,3X18X8,3 Керамические конденсаторы. В качестве диэлектрика в кера- мических конденсаторах применяется высококачественная кера- мика, например ультрафарфор, тиконд, ультрастеатит и др. Обкладки керамических конденсаторов выполняются в виде слоя серебра, наносимого на поверхность керамики. Керамические конденсаторы подразделяются на два основных типа: КДК (конденсатор дисковый керамический) и КТК (кон- денсатор трубчатый керамический). В табл. 10 приведены данные неко- торых наиболее употребительных типов керамических конденсаторов. а Керамические конденсаторы обла- дают более высокими качествами по сравнению со слюдяными конденсато- «= рами. Они имеют * большое сопротив- ление изоляции, малые потери и отно- сительно высокую стабильность, т. е. постоянство емкостей. Рис. 40. Керамические кон- денсаторы: дисковый (а) и трубчатый (б) 99
Таблица 10 Данные некоторых типов керамических конденсаторов Тип конденса- тора Емкость, пф Рабо- чее напря- жение, в Размеры, мм Примечание диаметр трубки длина трубки диаметр диска КТК 2-750 500 4 11—50 — Конденсатор труб- чатый керамический КДК 1-100 500 — — 8—16 Конденсатор диско- вый керамический кгк 5-750 500 6,5 16—55 — Конденсатор герме- тизированный кера- мический КЭТ 180—620 500 6,2 11—20 — Конденсатор эмали- рованный трубчатый кэд 30-360 250 — — 4—12 Конденсатор эмали- рованный дисковый код 30—200 250 — — 8-13,5 Конденсатор кера- мический опрессован- ный дисковый Керамические конденсаторы благодаря относительно боль- шой толщине их диэлектриков (керамики) имеют относительно небольшую емкость, от нескольких пикофарад до 1000 пико- фарад. Керамические конденсаторы широко применяются в радио- технике, и в частности в цепях аппаратуры ультракоротких волн. На рис. 40 показан внешний вид конденсаторов типа КДК (кон- денсатор дисковый керамический) и типа КТК (конденсатор трубчатый керамический). Воздушные конденсаторы. Воздушные конденсаторы имеют обкладки из алюминиевых пластин, разделенных воздушными прослойками, играющими роль диэлектрика. Воздушные конденсаторы по своим качествам выше керами- ческих. В них потери энергии меньше, стабильность выше и со- противление диэлектрика (воздуха) достаточно велико. Недостаток воздушных конденсаторов — их громоздкость, по- этому они применяются сравнительно редко. В. КОНДЕНСАТОРЫ ПЕРЕМЕННОЙ ЕМКОСТИ Во многих случаях практики, например в радиотехнических цепях, возникает необходимость плавного изменения емкости. Для этого применяются конденсаторы переменной емкости. 100
Конденсаторы переменной емкости состоят из групп подвиж- ных и неподвижных пластин (рис. 41). Подвижные пластины (ротор) при помощи ручки приводятся во вращение и могут за- нимать различное положение относительно неподвижных пластин (статор). Если подвижные пластины полностью перекрывают неподвижные, то емкость конденсатора максимальна, и, наобо- рот, если подвижные пластины полностью выведены из системы неподвижных пластин, то емкость конденсатора минимальна. На практике широко применяются полупеременные < конден- саторы, емкость которых можно изменять в незначительных пре- делах. Применяются они главным образом в радиотехнических контурах как подстроечные. Рис. 42. Керамические под- строечные конденсаторы: а — типа КПК; б — пружинный кон- денсатор Рис. 41. Конденсатор переменной ём- кости: 1 — подвижные пластины (ротор); 2 — неподвиж- ные пластины (статор) Существует несколько разновидностей полупеременных кон-к денсаторов. Из них лучшие — конденсаторы типа КПК (кера- мические подстроечные конденсаторы), изображенные на рис. 42, а. В них неподвижные основания (статоры) сделаны из керамики, на поверхность которой нанесен тонкий слой серебра. Роторами конденсаторов служат керамические диски, на верх- нюю поверхность которых нанесен слой серебра в виде полукруга. Таким образом, керамический полупеременный конденсатор имеет в качестве обкладок два слоя серебра, а диэлектриком — керамику. Подстройка контура с помощью подстроечного кера- мического кондейсатора осуществляется путем поворота ротора конденсатора. В табл. 11 приведены данные некоторых применяемых в ра- диотехнике керамических подстроечных конденсаторов. 101
Таблица 11 Данные некоторых керамических подстроечных конденсаторов Тип кон- денсатора Минимальная и максимальная емкость каждого конденсатора, пф Рабочее напряже- ние, а КПК-1 2—7 4-15 6—25 8—30 — — — — 500 КПК-2 6—60 10-100 25—150 75—200 125—250 200—325)275—375 350—450 500 В менее ответственных установках применяются пружинные подстроечные керамические конденсаторы (рис. 42,6). Каждый из них представляет собой совокупность двух металлических об- кладок (пластин), смонтированных на керамической колодочке так, что одна находится над другой. Верхняя обкладка обладает свойством плоской пружины и с помощью регулировочного винта может приближаться ,к нижней обкладке и удаляться от нее, чем и достигается изменение емкости конденсатора. Г. ЭЛЕКТРОЛИТИЧЕСКИЕ КОНДЕНСАТОРЫ Весьма широкое применение в радиотехнике получили элек- тролитические конденсаторы. Электролитический конденсатор состоит из двух алюминиевых пластин, погруженных в электролит, состав которого бывает раз- личным. В процессе изготовления электролитического конденса- тора через него пропускают постоянный электрический ток, при- соединив одну из алюминиевых пластин его (анод) к положи- тельному зажиму источника тока, а другую (катод) — к отрица- тельному. При этом в электролите происходит электролиз, т. е. разложение током электролита на составные части. В результате электролиза анод покрывается тонким слоем окисла, а катод не окисляется. Тонкая пленка окисла, образовавшаяся на аноде, является диэлектриком, а поэтому по мере возрастания толщины слоя диэлектрика электрический ток, протекающий через конден- сатор, уменьшается и в конце концов совершенно прекращается. Получившийся таким образом конденсатор имеет в качестве об- кладок алюминиевую пластину (анод) и электролит, а в каче- стве диэлектрика — слой окисла на аноде. Чтобы соединить вто- рую обкладку конденсатора (электролит) с отрицательным зажи- мом источника тока, в него и погружают вторую алюминиевую пластину (катод). Следует отметить, что если к аноду и катоду конденсатора приложить напряжение постоянно/о тока обратной полярности, то слой окисла на аноде будет исчезать. В резуль- тате этого конденсатор может быть электрически пробит и вый- дет из строя. 102
Следовательно, электролитические конденсато- ры обладают строго определенной поляр- ностью, т. е. они выполняют роль конденсаторов только в том случае, когда анод (алюминиевая пластина, покрытая пленкой окисла) присоединен к положительному полюсу источника по- стоянного тока. Это значит, что электролитический конденсатор может работать только в цепях постоянного или пульсирующего тока. В последнем случае требуется, чтобы переменная состав- ляющая пульсирующего тока не превышала, примерно десяти процентов постоянной составляющей пульсирующего ток^а. Так как пленка окисла на аноде электролитического конден- сатора очень тонка, т. е. расстояние между обкладками конден- сатора (анод-электролит) очень мало, то емкость этого конден- сатора относительно велика. Обычно она даже при относительно малых размерах конденсатора измеряется десятками, сотнями или даже тысячами микрофарад. В этом — существенное пре- имущество электролитических конденсаторов перед другими ти- пами их. Однако электролитические конденсаторы имеют и существен- ные недостатки. Во-первых, их нельзя применять в цепях пере- менного тока (изменяющегося по направлению), так как они об- ладают строго определенной полярностью. Во-вторых, в них имеется большая утечка тока из-за относительно большой прово- димости диэлектрика (слоя окисла). В-третьих, емкость этих конденсаторов не стабильна, так как она в значительной степени зависит от температуры (например, с уменьшением температуры она уменьшается). Емкость их зависит также от приложенного напряжения: чем оно выше, тем толще становится слой диэлек- трика (слой окисла) и тем меньше емкость конденсатора. Электролитические i дующим образом. Бе- рут две длинные узкие ленты из алюминие- вой фольги, между ко- торыми прокладывают пропитанную электро- Литом материю или фильтрованную бума- гу. Затем эти ленты сворачивают в ру- лон, который поме- щают чаще всего в алюминиевый корпус и сверху прикрывают изоляцией. Анод кон- денсатора обычно под- Рис. 43. Электролитические конденсаторы (а) и их условное обозначение в схемах (б) ключается к зажи- му, находящемуся на 103
крышке конденсатора, а катод соединен с корпусом конден- сатора. Нашей электропромышленностью выпускаются электролити- ческие конденсаторы типа КЭ (конденсатор электролитический): КЭ-1, КЭ-2 и КЭ-3. Конденсаторы типа КЭ подразделяются на высоковольтные и низковольтные. Первые рассчитаны на рабочее напряжение до 500 в и емкость до нескольких десятков микрофарад, вторые — на рабочее напряжение до 4 в и емкость до 2000 мкф. На рис. 43 показан внешний вид нескольких типов электро- литических конденсаторов. § 28. ПАРАЛЛЕЛЬНОЕ СОЕДИНЕНИЕ КОНДЕНСАТОРОВ Совокупность нескольких конденсаторов, соединенных между собой параллельно, последовательно или смешанно, называется батареей (магазином) конденсаторов. Вначале рассмотрим па- п раллельное соединение кон- г денсаторов, применяемое в Чз том случае, когда желают увеличить емкость в цепи. На рис. 44 приведена схема батареи из трех па- Рис. 44. Параллельное соединение раллельно соединенных КОН- конденсаторов денсаторов, имеющих емко- ч сти Ci, С2 и С3. Так как конденсаторы соединены параллельно, то напря- жения на зажимах отдельных конденсаторов равны друг другу и напряжению на зажимах источника электрической энергии, т. е. Ц = ^2=ж=С/8 = ^ ^(22) где U — напряжение на зажимах источника электрической энергии; Ц, ^8— напряжения на зажимах соответственно первого, второго и третьего конденсаторов. Определим общую емкость рассматриваемой батареи. Если йа ее зажимах напряжение равно U, то каждый из конденсато- ров зарядится некоторым количеством электричества, которое можно определить по формуле (18): 91 = G U; q2 ~ C2U- q3 = C3U, (22') где ?i, 92 и <7з — электрические заряды первого, второго и третьего конденсаторов. 104
Из формул (22') следует, что -&-3, (23) Яз Я» Сз Яг t-i т. е. электрические заряды параллельно сое- диненных конденсаторов пропорциональны емкостям этих конденсаторов. Общий заряд q всей батареи конденсаторов равен сумме за- рядов отдельных конденсаторов: • Я = Яг + Яг + Яъ (24) или, принимая во внимание формулы (22'), q = CJJ + C2U + CZU = (Q + С2 + С3) £7, откуда находим, что 'lf=zCl + С2 + С3. Но величина-^-,т. е. отношение полного заряда q всей бата- реи конденсаторов к напряжению U на ее зажимах, определяет собой величину общей емкости Со всей батареи, т. е. С —-3- следовательно, Со ™ Cj 4- С2 + С3, (25) т. е. общая (эквивалентная) емкость батареи параллельно соеди- ненных конденсаторов равна сумме емкостей отдельных конден- саторов, входящих в батарею. Пример 166. Определить эквивалентную емкость батареи из трех паоал* лельно соединенных конденсаторов, имеющих емкости Ci = 2 мкф, С2 «0,5 мкф и С3==0,1 мкф. Решение. По формуле (25) находим Со - Сх + С2 + С3 - 2 + 0,5 + 0,1 » 2,6 мкф. Если конденсаторы, соединенные параллельно, обладают оди- наковыми емкостями С, то общая их емкость равна произведе- нию емкости С отдельного конденсатора на их число п в бата- рее, т. е. Со = Си. (26) Пример 17. Определить, сколько конденсаторов необходимо взять для составления батареи емкостью Со = 2 мкф, если имеются конденсаторы емкостью С = 0,25 мкф. 105
Решение. Число конденсаторе® в батарее определим, исходя из формулы (26): __ Сл __ 2 ___о п = = 8 конденсаторов. § 29. ПОСЛЕДОВАТЕЛЬНОЕ СОЕДИНЕНИЕ КОНДЕНСАТОРОВ Рис. 45. Последовательное соедине- ние конденсаторов На рис. 45 изображена батарея конденсаторов, состоящая из трех последовательно соединенных конденсаторов, имеющих ем- кости, соответственно равные Ci, С2 и Сз. Последовательное соединение конденсаторов применяется главным образом в том случае, когда желают предохранить конденсаторы от возможности электрического пробоя их ди- электриков напряжением. Дело в том, что при последователь- ном соединении конденсаторов общее напряжение, приложен- ное к батарее, распределяется по отдельным конденсаторам таким образом, что на зажи- мах каждого конденсатора на- пряжение всегда меньше, чем напряжение, поданное на всю ба- тарею в целом. В тех случаях, когда желают уменьшить емкость цепи, также часто прибегают к составлению батареи последовательно соеди- ненных конденсаторов, так как общая емкость батареи конден- саторов всегда меньше наименьшей емкости, входящей в батарею. Процесс заряда батареи последовательно соединенных кон- денсаторов в основном сводится к следующему. Электроны от отрицательного зажима источника электрической энергии пере- мещаются на левую обкладку первого конденсатора и спустя не- значительное время (доли секунды) заряжают ее отрицательным электричеством, величина заряда которого равна —q. Вследствие явления электростатической индукции правая обкладка первого конденсатора заряжается положительным электричеством (+<?), так как с нее свободные электроны уходят на левую обкладку второго конденсатора, заряжая ее отрицательным электричеством (—q). Аналогично правая обкладка второго конденсатора полу- чает заряд, равный +<?, левая обкладка третьего конденсатора — заряд — q и, наконец, правая обкладка третьего конденсатора — заряд так как с нее свободные электроны уходят к положи* тельному зажиму источника электрической энергии. Следовательно, в результате заряда батареи последовательно соединенных конденсаторов она получает электрический заряд, численно равный q, и одновременно с этцм каждый из конденса- 106
торов, входящих в батарею, в результате явления электростати- ческой индукции получает также электрический заряд, численно равный q: Ях = Я2=Яз = Я, (27) где ?i, <?2, <?з — электрические заряды соответственно первого, второго и третьего конденсаторов; q — электрический заряд батареи последовательно соединен- ных конденсаторов. Общее напряжение U, приложенное ко всей батарее, равно сумме напряжений Ui, U2 и т. д., приложенных к отдельным кон- денсаторам: и = их + иг + и^. , (28) Согласно формуле (22') = Cjt/i; Я2~^2^2'< Яъ — С&и%. А так как qi = <72 = <7з = Я, то Cjt/j = Сги2 = CJJt = q, откуда находим Подставив полученные выражения для 1Л, U2 и U3 в фор- мулу (28), получим С, + с,+ с3> или, после сокращения на q, _L==JL + _L+J_ д С, С3 С8 • Но U__ 1 <7 “ Со’ где Со — общая емкость всей батареи конденсаторов. Следовательно, <29> т. е. величина, обратная общей емкости батареи последовательно соединенных конденсаторов, равна сумме величин, обратных ем- костям отдельных конденсаторов, входящих в эту батарею. Пример 18. Определить общую емкость батареи из трех последовательно соединенных конденсаторов, обладающих емкостями Ci = 3 мкф, С2 = 4 мкф и С3 = 6 мкф.
Решение. По формуле (29) находим • =_!_ . _L 4._L > + _L 4. 1 = э 77 Cl + Сг+ С3 Т + 4 + Т 17 ’ / откуда Со = 4 = 1,33 мкф. В частном случае, если имеется только два конденсатора, соединенных последовательно, получим 1 _ 1 । 1 ________ 1 „G + G ✓-» I х-» , ИЛИ х-» л"* л* » *-»0 C»i С>2 C«q откуда найдем, что с» = <£%, <3°) т. е. общая емкость батареи из двух последовательно соединен* ных конденсаторов равна произведению их емкостей, разделен- ному на сумму этих емкостей. Пример 19. Определить общую емкость двух последовательно соединен- ных конденсаторов, обладающих емкостями Cj — 0,6 мкф и Cg = 0,3 мкф. Решение. По формуле (30) находим (2 CjCj 0,6-0,3 л п МК/А С° Cl 4 С, 0,6 4 0,3 U’2 Если все п конденсаторов, соединенных последовательно, имеют одинаковые емкости, т. е. Cj == С2 = С8 = . . . = С„ = С, то 1 =_!_ 4Х4 j__L= " 77 С -I-77--г ...-t- с -Q-, откуда находим, что общая емкость <з1> т. е. общая емкость п последовательно соединенных конденсато- ров, имеющих одинаковые емкости, меньше, чем емкость отдель- ного конденсатора, в п раз. Пример 20. Соединены последовательно три конденсатора, из которых каждый обладает емкостью С« 1,2 мкф. Определить общую емкость всей батареи конденсаторов. Решение. По формуле (31) находим сов 4“т“0'4 мкФ- 108
Теперь посмотрим, как распределяется напряжение между отдельными последовательно соединенными конденсаторами. Выше мы выяснили, что Cjt/je = = Q, Отсюда находим; что Ui С*> . U, С3 t U3 Ci /оп\ ~ сх > и3~ с2> Ui~ с3> т. е. напряжение между последовательно соединенными конден- саторами распределяется обратно пропорционально их емкостям. Пример 21. Определить, как распределится напряжение U = 120 в мъжру тремя последовательно соединенными конденсаторами, имеющими емкости Ci = 0,3 мкф, С2 = 0,2 мкф и С3 = 0,12 мкф, а также определить общую емкость и заряд всей батареи. Решение. Определим общую емкость всех трех конденсаторов по формуле (29) 1 __ 1 I 1 । 1 1 , 1 , 1 _ 100 Со — С, *" С, + С, = 0J + 0,2 ' 0,12 ~ 6 » откуда Со = О,О6 мкф. Далее определим заряд всей батареи: <7 = Со£/ = 0,06 • 10"® • 120 = 7,2 • 10"® к. Напряжение на отдельных конденсаторах U - Я _ 7.2-Ю-e _ 471 ~ ТГ “ о,з-ю-о “24 ’ 7,2-10-е _ 0,2-10~в "36 ’ 7.2-Ю-e _сп 0,12-10-е -60 Л U — — Ua~ С, _ и — — и> ~ С, “ Для проверки правильности решения сложим полученные напряжения: ^ = Ц + Ц + г/3 = 24 + 36 + 60 = 120 вг. т. е. получим общее напряжение, равное заданному. § 30. СМЕШАННОЕ СОЕДИНЕНИЕ КОНДЕНСАТОРОВ Батарея смешанно соединенных конденсаторов состоит из совокупности последовательного и параллельного соединения их. На рис. 46 изображена схема смешанно соединенных четырех конденсаторов, из которых составлены две параллельно соеди- ненные группы, имеющие по два последовательно соединенных конденсатора. Рассчитаем общую емкость данной батареи. 109
Вначале определим об- щие емкости отдельных групп, в которых конден- саторы соединены после- довательно. По формуле (30) находим р ___ , р ___________ ’ .п — = ^3^4 С3 4- ’ Рис. 46. Смешанное соединение конден- ГД6 С} и Сп общие ем- • саторов кости отдельных групп конденсаторов. Группы соединены параллельно, а поэтому общая емкость всей батареи конденсаторов определится по формуле Со — • Пример 22. Для некоторой электротехнической установки требуется конденсатор емкостью С = 2 мкф, рассчитанный на рабочее напряжение U = 500 в. Имеются конденсаторы емкостью С = 2 мкф, но рассчитанные на рабочее напряжение Г/= 250 в, а поэтому их использовать в данной установке, казалось бы, нельзя из-за возможности их пробоя при напряже- нии 500 в. Для использования этих конденсаторов в данной установке необходимо составить из них батарею, обладающую требуемыми стойкостью и напряжением. Решение. Составим батарею из четырех конденсаторов согласно схеме, изображенной на рис. 46. Емкости отдельных параллельных групп конденсаторов будут равны = Ci + С3 = 2 + 2 = 1 МК$' СИ= Сз + Ct = 2 + 2 ='1 MK<$' Общая емкость всей батареи Со = Cj 4е Сц = 1 4- 1== 2 мкф. Напряжение распределится по отдельным конденсаторам поровну, так как емкости последних равны. Следовательно, = -^ = 250 в; 673 = Ц=-^2. = 25О в. ПО
Рис. 47. Зависимость напряжения на обкладках конденсатора от величины заряда конденсатора § 31. ЭНЕРГИЯ ЭЛЕКТРОСТАТИЧЕСКОГО ПОЛЯ & Всякий конденсатор при заряде поглощает некоторое коли- чество энергии от источника электрической энергии, заряжаю- щего его. Эту энергию он нака- пливает в электрическом поле, имеющемся в его^ диэлектрике. Определим величину этой энер- гии для случая, когда заряжается плоский двухпластиночный кон- денсатор. По мере заряда конден- сатора растет его заряд q, полу- чаемый от источника электриче- ской энергии, и в соответствии с*.этим растет напряжение Uc на его обкладках. Как известно, между зарядом q конденсатора и напряжением Uc на его обкладках существует прямо пропор- циональная зависимость: q = CUc. Эта зависимость показана на рис. 47 в виде прямой линии ОБ. Допустим, что по окончании заряда конденсатор получил от источника электрической энергии заряд Q и в соответствии с этим напряжение на его зажимах стало равным U. Работа, которую совершил источник электрической энергии по переносу зарядов на конденсаторе при заряде последнего, чис- ленно равна площади треугольника OQE (рис. 47), у которого основанием является Q — полный заряд конденсатора и высо- той U — напряжение на конденсаторе при окончании его заряда. Площадь треугольника, как известно, равна половине произведе- ния основания на высоту, а поэтому Л — UQ zqq\ (33) где Q— электрический заряд в кулонах; U — напряжение в вольтах; А — работа в джоулях. Заменив в формуле (33) величину Q выражением Q = Ct7, получим (34) где С — емкость в фарадах; U — напряжение в вольтах; W — энергия в джоулях. 111
Пример 23. Конденсатор при напряжении на зажимах U = 500 в имеет заряд q = 0,0001 к. Определить энергию поля этого конденсатора. Решение. По формуле (33) находим W—— 522^2221«. 0,025 дж. Пример 24. Определить энергию электростатического поля конденсатора, если известно, что его емкость С = 2 мкф, а напряжение на его обкладках U = 500 в. Решение. Применяя формулу (34), находим 2-ю-в.500г=0>25 дж. Энергия электростатического поля распределена во всем объеме этого поля с некоторой конечной плотностью, величина которой пропорциональна квадрату напряженности поля, т. е. ^0 = -^, , (35) где Wq — объемная плотность энергии поля в джоулях на ку- бический метр; Е — напряженность электростатического поля в вольтах на метр; 1 (Ф\ е0 = ~4д';'9.109' () — диэлектрическая проницаемость ва- куума; — относительная проницаемость среды, где распреде- лено электростатическое поле. Если известно распределение объемной плотности энергии поля, то можно определить энергию всего данного электростати- ческого поля. Эта задача в общем случае решается с помощью приемов высшей математики, поэтому не может быть здесь рас- смотрена. Мы ограничимся более простым, частным случаем, когда электростатическое поле однородно, т. е. в каждой его точке величина и направление вектора напряженности одина- ковы. В этом случае объемная плотность энергии поля всюду одинакова, а поэтому общая энергия всего поля определяется произведением объемной плотности энергии на объем простран- ства, занимаемого всем полем: Г=1Г0У, (36) где W — энергия электростатического поля в джоулях; 1Г0 — объемная плоскость энергии в джоулях на кубиче- ский метр; V —объем в кубических метрах. Пример 25. Требуется определить энергию, запасенную в электростатиче- ском поле плоского двухпластинчатого конденсатора, если площадь каждой 112
из его обкладок S « 0,004 кв. м, расстояние между обкладками d » 0,001 м, относительная диэлектрическая проницаемость диэлектрика ег «5 и напря- жение на обкладках U 120 в. Р е ш е Ви е. Так как электростатическое поле плоского конденсатора в данном случае можно считать однородным, то напряженность этого поля будет равна 120000 Объемная плотность энергии в поле равна ^ = ^=4^0^=0,318 дж/м\ Объем, занимаемый электростатическим полем, равен у = 5^ = 0,004-0,001 = 4-10“6 лЛ Энергия, запасенная во всем объеме электростатического поля, равна Ц7= Ц70У = 0,318-4-10”6 = 1,272-10-6 дж. § 32. ЭЛЕКТРИЧЕСКАЯ ПРОЧНОСТЬ ДИЭЛЕКТРИКОВ Если напряжение на зажимах конденсатора увеличивается, то в соответствии с этим увеличивается и напряженность элек- тростатического поля в его диэлектрике. Опыт показывает, что при достижении некоторой, определенной для каждого диэлек- трика, величины напряженности электростатического поля ди- электрик теряет свои изолирующие свойства и в нем происходит электрический пробой. Физический процесс электрического пробоя газовых и твер- дых диэлектриков протекает различно. В каждом газе при нормальном его состоянии всегда име- ется некоторое количество свободных электронов, а также поло- жительных и отрицательных ионов. Но так как их мало, то при относительно слабых электрических полях ток проводимости в газе настолько мал, что практически им можно пренебречь. Однако электрическое состояние газового диэлектрика заметно изменяется по мере увеличения напряженности электрического поля в нем. Скорость движения свободных электрических заря- дов при этом растет, а следовательно, увеличивается и кинети- ческая энергия зарядов. Когда напряженность электрического поля достигает некоторой величины, кинетическая энергия сво- бодных зарядов возрастает настолько, что при столкновении их с молекулами возникает явление ударной ионизации этих моле- кул. Этот процесс развивается в газах весьма 'интенсивно, нося лавинообразный характер, так как всякий возникший в процессе ионизации свободный заряд сам принимает непосредственное участие в дальнейшей ионизации газа. В конечном итоге количе- 8—1377 113
ственные изменения свободных зарядов в газе приводят к рез- кому качественному изменению его состояния, в результате чего возникает электрический пробой газа — через диэлектрик про- ходит относительно большой ток проводимости в виде электри- ческой искры. Пример такого мощногр|электрического разряда в газах — грозовой разряд в воздухе (молния), сопровождаемый громом. Процесс пробоя твердых диэлектриков протекает несколько иначе по сравнению с пробоем в газах. Так как длина свободного пробега электронов между атомами и молекулами твердого диэлектрика относительно мала, то они не могут разви- вать скорость движения до такого предела, при котором могла бы возникнуть ударная ионизация. Пробой твердого диэлектрика зависит главным образом от тепловых процессов, которые развиваются в нем при продолжи- тельном действии электрического поля. Если твердый диэлектрик имеет относительно большую толщину, то потери энергии в нем могут вызвать значительный нагрев его. С ростом же темпера- туры диэлектрика его сопротивление падает из-за отрицательного температурного коэффициента. В результате начинает быстро расти ток проводимости в диэлектрике, что еще более способ- ствует повышению его температуры. Наконец, температура твер- дого диэлектрика может достигнуть такой величины, при которой начинается качественное изменение самого диэлектрика, приво- дящее к уменьшению его электрической прочности, а следова- тельно, и к пробою. Пробой твердых диэлектриков, возникший в результате теплового действия, носит название теплового пробоя. Таблица 12 Электрическая прочность материалов Наименование диэлектрика Электрическая проч- ность, KBfCM Бумага кабельная сухая............... Бумага, пропитанная маслом............ Воздух............................... Масло трансформаторное............... Миканит.............................. Мрамор............................... Парафин ............................. Электрокартон сухой ................. Электрокартон, пропитанный маслом . . Слюда мусковитая .................... Слюда флогопит....................... Стекло .............................. Фибра ............................... Фарфор............................... Шифер................................ Эбонит .............................. 60—90 100—250 30 50—180 150—300 35—55 150—300 80—100 120—170 1200—2000 600—1250 100—400 40—110 180—250 15—30 80—100 114
Если толщина твердого диэлектрика в конденсаторе мала, то возможность теплового пробоя диэлектрика уменьшается благо- даря лучшей теп^эотдаче конденсатора. Однако в тонкослойном твердом диэлектрике не исключена возможность ионизацион- ного пробоя. Величина напряженности электрического поля, при которой наступает пробой диэлектрика, называется электрической проч- ностью (крепостью) диэлектрика. В табл. 12 приведены данные электрической прочности наи- более употребительных в электротехнике диэлектриков.
ГЛАВА VI ЭЛЕКТРИЧЕСКИЙ ТОК § 33. НАПРАВЛЕНИЕ И ВЕЛИЧИНА ЭЛЕКТРИЧЕСКОГО ТОКА Электрическим током называется направленное, упорядочен- ное движение электрических зарядов в электрическом поле. При изучении законов электрического тока перед нами неиз- бежно встанет вопрос, какое же имеет направление ток в про- водниках, так как потоки разноименных электрических зарядов в электрическом поле обычно протекают в диаметрально проти- воположных направлениях. Рис. 48. Направление электрического поля Е, тока I и движения положи- тельных и отрицательных ионов в электролите Рис. 49. Направление поля Е, тока Z и движения свободных электронов Условно принято считать, что электрический ток в электриче- ском поле имеет направление от точек с большими потенциалами к точкам с меньшими потенциалами. Это значит, что направле- ние электрического тока всегда совпадает с направлением дви- жения положительных электрических зарядов, например с на- правлением положительных ионов в электролитах и газах (рис. 48). Там же, где электрический ток создается движением 116
только потока отрицательно заряженных частиц, например потока свободных электронов в металлах, за направление электриче- ского тока принимают направление, противоположное движению электронов (рис. 49). Этого общепринятого условного направле- ния движения электрического тока мы и будем придерживаться в дальнейшем. Чем больше напряженность электрического поля в провод- нике, тем больше сила, действующая на электрические заряды в нем, и тем с большей скоростью они перемещаются направ- ленно в этом поле. А это значит, что с ростом напряженности поля в проводнике будет увеличиваться количество электриче- ства, протекающего через поперечное сечение проводника в еди- ницу времени. Мерой интенсивности движения электрических зарядов в про- водниках является величина тока, или просто ток (7,г). Вели- чина тока — это количество электрических зарядов (электриче- ства), протекающих через поперечное сечение проводника в еди- ницу времени. Вместо термина «ток» («величина тока») часто применяется термин «сила тока». Однако последний нельзя назвать удачным, так как сила тока не есть какая-либо сила в буквальном смысле этого слова, а только интенсивность движения электрических за- рядов в проводнике, количество электричества, проходящего 'за единицу времени через площадь поперечного сечения провод- ника. Если при равномерном движении электрических зарядов по проводнику за время t протекло количество электричества 7, то ток в проводнике можно определить по формуле / = (37) В системе единиц МКСА за единицу тока принимается один ампер (а, А). В проводнике ток равен одному амперу, если через площадь поперечного сечения его за одну секунду протекает один кулон электричества: . 1 кулон 1 ампер = Г'еУу---. В табл. 13 приведены единицы измерения тока с указанием их обозначений и соотношений с основной единицей. Следовательно, если в формуле (37) количество электриче- ства выразить в кулонах, время t — в секундах, то ток / будет выражен в амперах. 117
а Таблица 13 Единицы измерения тока в системе единиц МКСА Наименование величины и ее обозначение Название единицы Обозначение Соотношение с основной единицей русское междуна- родное ампер а А — Ток / миллиампер ма mA 10-’а микроампер мка |лА 10-в а Пример 26. Через потребитель при неизменном токе в течение t — 5 мин. протекло количество электричества q = 4,5 к. Определить ток / в потре- бителе. Решение. По формуле (37) находим / = ^“s^ = °.°15 “ Зная ток / в проводнике, можно определить количество элек- тричества q, прошедшего через поперечное сечение проводника за время t. Из формулы (37) непосредственно следует, что ? = (38) где q — количество электричества в кулонах; / — ток в амперах; t — время в секундах. Пример 27. Какое количество электричества прошло через электриче- скую лампу за время / = 0,5 часа, если ток в лампе равен-1 = 0,6 а. Решение. По формуле (38) находим </ = // = 0,6-0,5-60-60= 1080 к. В тех случаях когда приходится иметь дело с большими то- ками, количество электричества измеряется более крупной еди- ницей, называемой ампер-часом (а-ч, Ah). 1 ампер-час = 3600 кулонам. В электротехнике приходится иметь дело с токами от микро- ампера до десятков тысяч ампер. Для питания цепей аппаратуры связи применяются токи, измеряемые миллиамперами или в не- которых случаях микроамперами, а поэтому электротехнику связи относят к технике слабых токов. Электротехнические устройства, в которых токи измеряются единицами, десятками, сотнями и т. д. ампер, относятся к тех- нике сильных токов. Электрические станции, электросиловые установки, электрическое освещение и т. д. относятся к технике сильных токов. 118
Для измерения тока в электрической цепи служат приборы, называемые амперметрами (рис. 50). Амперметр включается своими зажимами в цепь так, чтобы через него проходил весь измеряемый ток. На рис. 51 показана Рис. 50. Внешний вид переносного ам- перметра электрическая цепь, состоя- щая из батареи гальваниче- ских элементов /, ампермет- ра А и некоторого потреби- теля электрической энер- гии 2. С принципом дейст- вия и устройством ампер- метра мы познакомимся по- дробно в главе об измери- тельных приборах. 1 2 Рис. 51. Схема включения ампер- метра в электрическую цепь § 34. ПЛОТНОСТЬ ТОКА В электротехнике.очень часто бывает важно знать не только величину тока в проводнике, но и плотность тока, так как плот- ность тока является мерой допустимой электрической нагрузки проводов. Плотностью тока (/, 3) называется ток, приходящийся на еди- ницу площади поперечного сечения проводника: /=4> <39) где /— плотность тока в проводнике; I — ток в проводнике; 5*—площадь поперечного сечения проводника. Если в формуле (39) .ток I выразить в амперах, а пло- щадь поперечного сечения^ S — в квадратных метрах, то плот- ность тока j будет выражена в амперах на квадратный метр / а А \ \ м2 ’ ш2 / В тех случаях когда ток протекает по линейному проводнику, поперечное сечение которого обычно измеряется в квадратных 119
миллиметрах, плотность тока, измеренная в нем, будет в амперах на квадратный миллиметр -^5-). В табл. 14 приведены единицы измерения плотности тока, их условные обозначения и соотношение их с основной единицей. Таблица 14 Единицы измерения плотности тока в системе единиц МКСА Наименование вели- чины и ее обозначение Название единицы Обозначение Соотношение с основной единицей русское международ- ное Плотность тока у, 8 ампер на квадрат- ный метр ампер на квадрат- ный сантиметр ампер на квадрат- ный миллиметр а а см2 а мм2 А ш2 А ст2 А шт2 м2 м2 Пример 28. Через катушку, на которую намотан цилиндрический провод диаметром d = 2 мм, проходит ток /=12,56-а. Определить плотность тока в проводе катушки. Толщиной изоляции провода на катушке можно пре- небречь. Решение. Вначале определим площадь S поперечного сечения про- вода на катушке. Согласно известной геометрической формуле с nd2 3,14-22 9 5 = — = -д-д— = 3,14 мм2. Далее по формуле (39) находим плотность тока в проводе: . / _ 12,56 _ . а ПЦ4~ ~ 4 мл<2 • § 35. ПОСТОЯННЫЙ ТОК И СТАЦИОНАРНОЕ ЭЛЕКТРИЧЕСКОЕ ПОЛЕ Постоянным электрическим током называется такой ток, ко- торый с течением времени не изменяется по своей величине. На рис. 52 изображен график постоянного тока. Здесь по горизонтальной оси отложен масштаб времени /, а по вертикаль- ной — масштаб тока /. Как видно из рис. 52, график постоянного тока представляет Собой прямую линию, параллельную горизон- тальной оси (оси времени). Величина постоянного тока / для любого момента времени сохраняется неизменной, и в соответ- ствии с этим все перпендикуляры, опущенные из любой точки графика тока / на горизонтальную ось (ось времени), равны. При постоянном токе через каждое поперечное сечение про- водника в единицу времени протекает одинаковое количество электричества (электрических зарядов). Это значит, что в каж- 120
дой точке проводника, по ко- торому протекает постоян- ный ток, одни элементарные электрические заряды непре- рывно сменяются другими, совершенно одинаковыми электрическими зарядами. Следовательно, несмотря на непрерывное перемещение электрических зарядов вдоль проводника, общее простран- ственное их расположение внутри проводника как бы Рис. 52. График постоянного тока остается неизменным во времени, или стационарным. Электрическое поле, с помощью которого создается и поддер- живается постоянный ток в проводнике и в соответствии с этим стационарное распределение в нем электрических зарядов, на- зывается стационарным (неизменным во времени) электриче- ским полем. Стационарное электрическое поле существенно отличается от поля электростатического. Если металлический проводник нахо- дится во внешнем электростатическом поле (£Внеш), то электри- ческие заряды распределены в нем так (рис. 53), что создавае- мое ими собственное внутреннее поле_ (^внут) полностью ком- пенсирует внешнее поле (£ВНеш = — £Внут)- В силу этого на- пряженность результирующего электрического поля внутри про- водника равна нулю (£’рез=0)- Но если это так, то разность потенциалов двух любых точек внутри проводника, а также и на поверхности его тождественно равна нулю (^ — Это значит, что поверхность проводника, находящегося в электроста- тическом поле, — равнопотенциальная поверхность. Перераспре- деление зарядов в проводнике, находящемся в электростатиче- ском поле, возможно только в том случае, если начнет изме- няться внешнее электростатическое поле, т. е. если оно превра- тится в переменное во времени электрическое поле. Совершенно иная картина наблюдается внутри металличе- ского проводника, если в нем существует стационарное электри- Рис. 53. Статическое распределение зарядов в металлическом проводнике Рис. 54. Стационарное распреде- ление электрических зарядов в металлическом проводнике 121
веское поле (рис. 54). В этом случае напряженность электриче- ского поля в любой точке проводника отлична от нуля (£ =# 0); поверхность проводника уже не является равнопотенциальной (<рдг =# потенциалы поля внутри проводника и на поверх- ности его убывают в направлении тока I (в направлении поля); свободные электроны непрерывным и равномерным потоком дви- жутся навстречу полю, но в то же время пространственное рас-' пределение их внутри проводника стационарно (неизменно во времени). Электрические заряды в стационарном электрическом поле нигде не накапливаются и нигде не исчезают, так как при всяком пространственном перераспределении зарядов неизбежно должно было бы измениться само стационарное поле и соответ- ственно то$ перестал бы быть постоянным во времени. Но если для стационарности поля и тока требуется, чтобы электрические заряды нигде не накапливались и нигде не теря- лись, а перемещались непрерывным и равномерным потоком вдоль проводников, необходимо, чтобы эти проводники совме- стно образовали замкнутый на себя контур. В этом случае будет достигнуто непрерывное круговое равномерное движение электрических зарядов вдоль всего замкнутого контура. Итак, постоянный электрический ток может существовать только в замкнутом на себя контуре, состоящем из совокупности про- водников электричества, в котором действует стационарное элек- трическое поле. § 36. СТОРОННИЕ ЭЛЕКТРИЧЕСКИЕ ПОЛЯ Сравнивая электростатическое и стационарное электрические поля, необходимо указать на еще одну существенную разницу между ними. Для поддержания постоянства электростатического поля нет необходимости расходовать энергию, так как в этом случае элек- трические заряды находятся в относительном покое. Наоборот, для поддержания постоянства ста- ционарного электрического поля необходима непрерывная затрата энергии. Дело в том, что ста- ционарное поле существует только при постоянном токе в про- водниках, т. е. при условии, что вдоль этих проводников непре- рывным потоком движутся электрические заряды. Силы стацио- нарного поля, перемещая эти заряды, совершают работу, которая в конечном итоге преобразуется в другие виды энергии, напри- мер тепловую. Следовательно, стационарное электрическое поле, поддерживая постоянный электрический ток в проводниках, непрерывно расходует свою энергию, преобразуя ее с помощью тока в иные виды энергии. Естественно, для поддержания стационарного электрического поля необходимо непрерывное возмещение потерь его энергии за счет каких-либо иных видов энергии, например механической, 122
химической и т. д. Это осуществляется с помощью источников электрической энергии, в которых механическая (в электриче- ских машинах) или химическая (в аккумуляторах и гальваниче- ских элементах) энергия преобразуется в энергию электриче- ского поля. Электрические поля, создаваемые источниками электрической энергии, существенно отличаются от электростатического поля. Они обычно называются сторонними электрическими полями. Термин «сторонние» здесь подчеркивает то, что эти поля по своей природе отличны от поля электростатического. Сторонние электрические поля могут быть созданы в резуль- тате самых разнообразных физических явлений. Если, например, Рис. 55. В металличе- ском проводнике, пере- секающем при своем движении магнитные линии, возникает сто- роннее электрическое поле Рис. 56. На границе со- прикосновения цинка с раствором серной кислоты возникает стороннее элек- трическое поле металлическую проволоку перемещать в магнитном поле так, чтобы она пересекала магнитные линии, то внутри этой прово- локи возникнет стороннее электрическое поле (рис. 55). На этом принципе преобразования механической энергии движения про-, водника в магнитном поле в электрическую энергию стороннего электрического поля основано действие электрических машин. Стороннее электрическое поле возникает также в месте со- прикосновения металлов с растворами солей и кислот,, например цинка и раствора серной кислоты (рис. 56). В этом случае сто- роннее поле получает энергию в результате преобразования хи- мической энергии растворяемого в электролите (растворе серной кислоты) цинка. На принципе преобразования химической энер- гии в электрическую энергию стороннего электрического поля основано действие гальванического элемента. Стороннее поле получается при подогреве места спая двух разнородных металлов, например меди и висмута, В этом случае 123
стороннее поле получает энергию в результате преобразования тепловой энергии. На принципе преобразования тепловой энергии в энергию стороннего поля основана работа термоэлементов. Здесь мы не имеем возможности подробно останавливаться на описании физической сущности преобразования различных видов энергии в энергию сторонних электрических полей. Это будет сделано впоследствии, при описании принципа действия источников электрической энергии. Силы, с помощью которых осуществляется действие сил сто- роннего поля на электрические заряды, помещенные в нем, на- зываются сторонними силами. Здесь термин «сторонние» подчер- кивает то, что эти силы не электростатического происхождения. Природа их во многих случаях еще не вполне выяснена. Всякое стороннее электрическое поле, как и поле электро- статическое, характеризуется напряженностью в каждой его точке. Напряженность стороннего электрического поля в данной его точке численно равна силе, действующей на единичный положи- тельный заряд, помещенный в эту точку, и совпадает с направ- лением действия этой силы: р _____ ^стор ^стор q ’ где £Стор — напряженность стороннего электрического поля в вольтах на метр; ^стор — механическая сила в ньютонах, действующая на электрический заряд q, помещенный в данную точку поля; q— заряд в кулонах. Итак, для существования постоянного тока в системе про- водников и соответственно стационарного электрического поля, создающего этот ток, необходимо, чтобы эта система образовала замкнутый на себя контур. Одним из необходимых звеньев этого контура должен быть источник электрической энергии, в котором механическая или химическая энергия преобразуется в энергию их стороннего электрического поля. Стороннее же электрическое поле передает свою энергию стационарному электрическому полю, которое в свою очередь посредством электрического тока преобразует ее в какие-либо иные виды энергии. В дальнейшем для краткости записи будем называть сторон- ние электрические поля просто электрическими полями, оставляя за электрическим полем относительно неподвижных электриче- ских зарядов название электростатическое поле.
ГЛАВА VII ЭЛЕКТРОДВИЖУЩАЯ СИЛА И НАПРЯЖЕНИЕ ИСТОЧНИКА ЭЛЕКТРИЧЕСКОЙ ЭНЕРГИИ § 37. ЭЛЕКТРОДВИЖУЩАЯ СИЛА ИСТОЧНИКА ЭЛЕКТРИЧЕСКОЙ ЭНЕРГИИ Источники электрической энергии постоянного тока подразде- ляются на две основные группы: 1) электрические машины (генераторы) постоянного тока, преобразующие механическую энергию в электрическую, и 2) химические источники электриче- ской энергии (аккумуляторы и гальванические элементы), пре- образующие химическую энергию в электрическую. Независимо от принципа действия источника электрической энергии постоянного тока каждый из них создает стороннее элек- трическое поле. В электрических машинах постоянного тока это поле создается в металлических проводниках якоря, вращающе- гося в магнитном поле, а в аккумуляторах и гальванических элементах — в месте соприкосновения электродов с электроли- том (растворами солей или кислот) при их химическом взаимо- действии. Стороннее электрическое поле, имеющееся в источнике элек- трической энергии постоянного тока, непрерывно действует на электрические заряды проводников, образующих вместе с ним 'замкнутую цепь, и создает в ней постоянный электрический ток. Перемещая электрические заряды по замкнутой цепи, силы стороннего электрического поля преодолевают сопротивление противодействующих сил, например вещественных частиц про- водников. Это приводит к тому, что силы стороннего электриче- ского поля совершают работу за счет энергии этого поля. По мере расхода энергии стороннее электрическое поле пополняет ее за счет механической или химической энергии. В результате работы сил стороннего электрического поля энергия этого поля переходит в цепи в какие-либо иные виды энергии, например тепловую в металлических проводниках, теп- ловую и химическую в электролитах, тепловую и световую в элек- трических лампочках и т. д. 125
Выражение «работа сил стороннего электрического поля» источника электрической энергии ради краткости обычно заме- няют выражением «работа источника электриче- ской э н е р г и и». Если известна работа, совершаемая источником электриче- ской энергии при перемещении единичного электрического заря- да по всей замкнутой электрической цепи, то легко определить работу, совершаемую им при переносе некоторого электриче- ского заряда q по этой цепи, так как величина работы пропор- циональна величине заряда. Величина, численно равная работе, совершаемой источником электрической энергии при переносе единицы положительного заряда по всей замкнутой цепи, называется электродвижущей силой Е. Следовательно, если источник электрической энергии при переносе заряда q по всей замкнутой цепи совершил работу Л, то его электродвижущая сила Е равна Е = (40) В системе единиц МКСА за единицу измере- ния электродвижущей силы принимается один вольт (в, V). Электродвижущая сила источника электрической энергии равна одному вольту, если при перемещении одного кулона элек- тричества по всей замкнутой цепи им была совершена работа, равная одному джоулю: 1 вольт = (41) 1 кулон ' > Если, например, электродвижущая сила какого-либо источ- ника электрическрй энергии Е = 125 в, то это надо понимать так, что источник электрической энергии, перемещая один кулон электричества по всей замкнутой цепи, совершит работу А = 125 джоулям, так как Е = — = >25 ДЖ°Улей = 125 вольт q 1 кулон Таким образом, зная электродвижущую силу Е источника электрической энергии и количество электричества q, перенесен- ное им по замкнутой цепи, можно определить работу, совершен- ную источником. Из формулы (40) следует, что А = Eq, (42) т. е. работа источника электрической энергии при переносе элек- трического заряда по всей замкнутой цепи равна произведению величины электродвижущей силы Е его на величину переноси- мого электрического заряда q. 126
Пример 2^. Электродвижущая сила гальванического элемента £ = 1 в. Определить, какую работу совершит этот элемент, перенося количество электричества q = 8 к по всей замкнутой цепи. Решение. По формуле (42) находим A = Eq = 1-8=8 дж. § 38. НАПРЯЖЕНИЕ НА УЧАСТКАХ ЭЛЕКТРИЧЕСКОЙ ЦЕПИ Если источник электрической энергии переносит электриче- ский заряд q по всей замкнутой цепи, то он совершает некото- рую работу А. Часть этой работы До он совершает при пере- носе заряда q по внутреннему участку цепи (участок внутри са- мого источника электрической энергии), а другую часть А — при переносе заряда q по внешнему участку цепи (вне источника). Следовательно, Д = Д0 + Л, (43) т. е. работа Л, совершаемая источником элек- трической энергии при переносе электриче- ского заряда q по всей замкнутой цепи, равна сумме работ, совершаемых им при переносе этого заряда по внутреннему и внешнему уча- сткам этой цепи. Разделив левую и правые части равенства (43) на величину заряда q, получим работу источника электрической энергии, от- несенную к единичному заряду: А = А + Д. (44) Но, как известно, работа источника электрической энергии, совершаемая им при переносе единичного заряда по всей замк- нутой цепи, численно равна его электродвижущей силе, т. е. где Е — электродвижущая сила источника электрической энер- гии. Величина -у-, численно равная работе, совершаемой источ- ником электрической энергии при переносе единичного заряда по внутреннему участку цепи, называется падением напря- жения (напряжением) на внутреннем участке ц е п и, т. е. А <1 где Uo — падение напряжения на внутреннем участке цепи. 127
Величина -у-, численно равная работе, совершаемой источ- ником электрической энергии при переносе единичного заряда по внешнему участку цепи, называется падением напря- жения (напряжением) на внешнем участке цеп и, т. е. !Л = ~г, где 171 — падение напряжения на внешнем участке цепи. Следовательно, равенству (44) можно придать такой вид: / E^U. + Ub (45) т. е. электродвижущая сила источника электрической энергии, создающего ток в электрической цепи, равняется сумме падений напряжения на внутреннем и внешнем участках этой цепи. Из равенства (45) следует, что т. е. падение напряжения на внешнем участке цепи меньше электродвижущей силы источ- ника электрической энергии на величину паде- ния напряжения на внутреннем участке цепи. Следовательно, чем больше падение напряжения внутри источника электрической энергии, тем меньше при всех прочих равных условиях падение напряжения на внешнем участке цепи, т. е. напряжение на зажимах источника электрической энергии. Так как падение напряжения имеет одинаковую размерность с электродвижущей силой, т. еч. выражается в джоулях на ку- лон, или, иначе, в вольтах, то за единицу измерения падения на- пряжения (напряжения) принят один вольт. Напряжение на зажимах источника электрической энергии (падение напряжения во внешнем участке цепи) равно одному вольту, если источник электрической энергии совершает работу, равную одному джоулю, при переносе электрического заряда в один кулон по внешнему участку цепи. Пример 30. Источник электрической энергии, перенося электрический заряд q«5 к по замкнутой цепи, совершает работу, равную А =* 120 дж, а при переносе по внутреннему участку — работу Ло = 10 дж. Требуется определить электродвижущую силу источника электрической энергии, паде- ние напряжения на внутреннем и внешнем участках цепи и работу, совер- шаемую источником электрической энергии во внешнем участке цепи. Решение. Электродвижущая сила источника электрической энергии равна £ = Т=™=24в’ 128
Падение напряжения внутри источника электрической энергии ^о = ^ = -Г = 2в- Падение напряжения во внешнем участке цепи —<70 = 24 —2 = 22 в. Работа, совершаемая источником электрической энергии во внешнем участке цепи, равна 4 = 4 — 4 = 120—10 = 110 дж. Если цепь, состоящую из источника электрической энергии и какого-либо внешнего сопротивления, разомкнуть, то ток в цепи станет равным нулю. В этом случае источник электрической энергии, как говорят, находится в режиме холостого хода. Но если при режиме холостого хода источника электри- ческой энергии ток в цепи равен нулю, то внутри источника электрической энергии падение напряжения также равно нулю (С7о = 0). Следовательно, в этом случае напряжение на зажимах источника электрической энергии будет*равно t/x.x=£-t70=^ (46) т. е. напряжение на зажимах источника элек- трической энергии при холостом ходе равно его электродвижущей силе Е. Если зажимы источника электрической энергии замкнуть та- ким проводником, сопротивление которого практически равно нулю, то работа источника электрической энергии при переносе электрических зарядов по внешнему участку цепи будет равна нулю. Такой режим называется режимом короткого за- мыкания. Следовательно, падение напряжения на внешнем участке цепи при коротком замыкании источника электрической энергии равно нулю: где t7K,3 — напряжение короткого замыкания* В соответствии с этим Е = ио + и^ио + О = ио, т. е. при коротком замыкании источника элек- трической энергии его электродвижущая сила Е целиком расходуется во внутреннем участке цепи. ©—1377 129
Для измерения напряжения на участках электрической цепи применяется прибор, называемый вольтметром. На рис. 57 пока- зан внешний вид одного из переносных вольтметров, а на рис. 58 показана схема включения вольтметра для? измерения напряже- ния на участке цепи с сопротивлением /?. Надо помнить, что Рис, 57. Внешний вид перенос- ного вольтметра Рис. 58. Схема включения вольт- метра в электрическую цепь вольтметр своими зажимами всегда присоединяется к тем точ- кам цепи, между которыми он должен измерить падение напря- жения.
ГЛАВА VIII ЭЛЕКТРИЧЕСКОЕ СОПРОТИВЛЕНИЕ § 39. ЭЛЕКТРИЧЕСКОЕ СОПРОТИВЛЕНИЕ ПРОВОДНИКОВ Если в проводнике проходит электрический ток, то это зна- чит, что в нем существует электрическое поле и направленное движение потока свободных электронов (в металлических ftpo- водниках) или потока ионов (в электролитах). Свободные электроны и ионы, перемещаясь направленно (упорядоченно) вдоль проводников, на пути своего движения «сталкиваются» с атомами, молекулами вещества и при этом от- дают им часть своей энергии. Эти «столкновения» электронов и ионов с атомами и молекулами приводят к тому, что первые из них испытывают как бы некоторое противодействие (сопротивле- ние) своему движению. Чем больше это противодействие, тем меньше скорость направленного, упорядоченного движения элек- тронов и ионов в проводниках и тем, следовательно, меньше будет ток в проводниках при прочих равных условиях. Противодействие, которое оказывает проводник проходящему через него электрическому току, называется электрическим со- противлением проводника или просто сопротивлением (/?, г). Опытом установлено, что сопротивление проводника зависит от его геометрических размеров, от материала и ряда физиче- ских условий. В системе единиц МКСА за единицу сопротивления прини- мается один ом (ом, 2)- Проводник обладает сопротивлением в один ом, если при на- пряжении на его концах, равном одному вольту, в нем устанав- ливается ток, равный одному амперу: 1 I ВОЛЬТ z Л'тч 1 ОМ==ТЪ^- (47) В табл. 15 приведены единицы измерения сопротивлений с указанием их условных обозначений. 9* 131
Таблица 15 Единицы измерения сопротивлений проводников в,системе единиц МКСА Наименование величины и ее обозначение * Название единицы Обозн русское ачение междуна- родное Соотношение с основной единицей Сопротивление R, г { ОМ мегом ОМ Мом Q MQ 106 ОМ В качестве эталона, обладающего сопротив- лением в один ом, принимают сопротивление столбика ртути высотой 106,3 см и площадью по- перечного сечения 1 мм2 при 0°Ц, что соответ- ствует массе ртути в 14,45 г. § 40. ЗАВИСИМОСТЬ СОПРОТИВЛЕНИЯ проводников ОТ ИХ РАЗМЕРОВ И МАТЕРИАЛА Сопротивление проводника прямо пропорционально его длине и обратно пропорционально площади поперечного сечения. Математически эта зависимость выражается следующей фор- мулой; = (48) где R — сопротивление проводника; I — длина проводника; S — площадь поперечного сечения проводника; р — коэффициент пропорциональности, называемый удель- ным сопротивлением проводника. Исходя из формулы (48), определим размерность удельного сопротивления р: Р“ — ом-м* м RS I (ом-метр), (49) т. е. размерность удельного сопротивления в системе единиц МКСА выражается в ом-метрах. В соответствии с этим за единицу измерения удельного со- противления в системе единиц МКСА принимается 1 ом-метр (1 ом-м, 2m). Проводник обладает удельным сопротивлением в 1 ом-метр, если при длине его, равной одному метру, и площади попереч- ного сечения в один квадратный метр он имеет сопротивление один ом: 132
В тех случаях когда длину проводника берут в сантиметрах, а площадь поперечного сечения — в квадратных сантиметрах, удельное сопротивление выражают в ом-сантиметрах. Удельное сопротивление проводника равно одному ом-санти- метру, если при длине его, равной одному сантиметру, и площади поперечного, сечения в один квадратный сантиметр сопротивле- ние его равно одному ому: 1 ом-см 1 ом-\ см2 1 см При расчете сопротивлений проводов обычно длину их изме- ряют в метрах, а площадь поперечного сечения — в квадратных миллиметрах. В соответствии с этим удельное сопротивление ом • мм2 проводов выражают в —. Л 7 1 ОМ- мм2 Удельное сопротивление провода равно I —-—, если при длине провода в один метр и площади поперечного сечения в один квадратный миллиметр сопротивление его равно одному ому. В табл. 16 приведены единицы измерения удельных сопро- тивлений с указанием их обозначений и соотношений с основ- ной единицей. Таблица 16 Единицы измерения удельного сопротивления в системе единиц МКСА Наименование величины и ее обозначение Название единицы Обозначение Соотношение с основной единицей русское междуна- родное Удельное ом-метр ом-сантиметр ОМ-М ОМ'СМ Qcm 10”2 ОМ-М сопротивление р ом-миллиметр квадратный на метр ОМ • ММ2 м Qmm2 m 10"6 ОМ-М В табл. 17 приведены удельные сопротивления некоторых наиболее употребительных проводников первого рода при 20° Ц. В табл. 18 приведены удельные сопротивления электролитов при 10° Ц. Зная удельное сопротивление и размеры проводника, можно, пользуясь формулой (48), определить его сопротивление. Пример 31. Определить сопротивление медного провода длиной I = 300 м, площадью поперечного сечения 5 = 6 мм2 и удельным сопротив- ХЛ17К ОМ'ММ2 лением р= 0,0175 ------ г * «у 133
Таблица 17 Удельные сопротивления важнейших проводников первого рода Наименование материала Удельное сопроти- вление при 20° Ц ом ♦ мм2 м Состав сплава, % Серебро Медь . Алюминий Вольфрам Цинк Никель Сталь Свинец Ртуть Висмут Латунь Нейзильбер . 0,0147—0,0175 0,0154—0,0175 0,0262—0,0278 0,049—0,0612 0,059—0,062 0,07—0,138 0,0987—0,14 0,206—0,221 0,948—0,9569 1,1428—1,39 0,02—0,06 0,3 Си 60,16%; Zn 25,37о/о; Ni 14,03%; Никелин 0,42 Fe 0,3% Си 61,63%; Zn 19,67%; Ni 18,46%; Манганин 0,374 Fe 0,24% Си 87,24%; Мп 10,26%; Ni 1,77%; Константан 0,49 Fe 0,52% Си 60%; Ni 40% Чугун Нихром 0,4—0,5 1,0—1,11 Cr 15%; Ni 62%; Fe 23% Меганир 1,4 Fe 65%; Cr 30%; Al 5% Фехраль 1,2 Fe 82%; Cr 13,5%; Al 4,5% Хромаль 1,35 Fe 65,8%; Cr 30%; Al 4,2% Таблица 18 Удельные сопротивления электролитов при 10° Ц в ом см Наименование электролита Удельное сопротивление при 10° Ц, ом см при концентрации водного раствора 5% 10% 15% | 20% | 25% 30% Раствор нашатыря NH4C1 10,9 5,6 3,9 3,0 2,5 — Раствор хлористого натрия NaCl 14,9 8,3 6,1 5,1 4,7 — Раствор цинкового купороса ZnSO4 52,4 31,2 24,1 21,3 20,8 22,7 Раствор медного купороса CuSO4 52,9 31,3 23,8 — — — Раствор едкого кали КОН 5,8 3,2 2,4 2,0 1,9 1,8 Раствор едкого натра NaOH 5,1 3,2 2,9 3,0 3,7 4,8 Раствор серной кислоты H2SO4 4,8 2,6 ч , 1,5 1,35 134
Решение. По формуле (48) находим /? = 4 =0,0175--^=0,875 ом. Пользуясь формулой (48), можно найти любую из величин, входящих в нее, если известны остальные три величины. Длина провода определится формулой / = (50) Пример 32. Сколько метров проволоки диаметром d = 0,5 мм необхо- димо взять для изготовления катушки сопротивлением R = 6 ом, если проволока изготовлена из нейзильбера с удельным сопротивлением п о ом • мм2 Р = U,о •-----. г М Решение. Вначале определим площадь поперечного сечения проволоки с itd2 3,14 • 0,52 п шг 9 S = — = 4 0,196 л/л/3. По формуле (50) находим / = ^ = ±^ = 3,92 м. р 0,3 ’ Из формулы (48) можно определить площадь поперечного сечения S проводника: 5 = 4. . (51) Пример 33. В электролит с удельным сопротивлением р = 2,6 ом • см погружены два электрода одинакового размера, расположенных параллельно друг другу на расстоянии /=12 см. Определить площади этих электродов, если сопротивление столба электролита между электродами равно R — 1,3 ом. Решение. По формуле (51) находим 5 = 4 = ^т^ = 24 см2. К 1,0 Пример 34. Определить удельное сопротивление проволоки, имеющей длину /=12,56 м, диаметр d = 0,2 мм и сопротивление /?=193 ом. Решение. Определим площадь поперечного сечения провода о ltd2 3,14 0,22 9 5 = —7-= . =0,0314 мм2. 4 4 ’ По формуле (49) найдем удельное сопротивление проволоки /?£ 193-0,0314 ало ом-мм2 * = — = 12,56 = 0)48-м---- § 41. ПРОВОДИМОСТЬ ПРОВОДНИКОВ Иногда для удобства расчета электрических цепей вместо сопротивления проводника вводят величину, обратную сопроти- влению, называемую проводимостью проводника (G. g)- 135
В противоположность сопротивлению проводимость провод- ника характеризует его способность проводить через себя элек- трический ток. Чем больше проводимость проводника, тем мень- ше его сопротивление и тем лучше он проводит электрический ток. Проводимость G есть величина, обратная сопротивлению R, т. е. О = Ц-. (52) Единица измерения проводимости в системе единиц МКСА специального названия не имеет. Из формулы (52) следует, что проводимость имеет размер- ность ~ . ом Проводник обладает проводимостью, равной единице, если его сопротивление равно одному ому. Единица проводимости в системе единиц МКСА = 1 -±-. ОМ Пример 35. Определить проводимость проводника, если его сопротивле- ние R — 0,25 ол«. Решение. По формуле (52) находим 0=j_=_l_=4 а. w К 0,25 ом* Удельной проводимостью называется величина, об- ратная удельному сопротивлению: (53) 1 где 7 — удельная проводимость в — ; р — удельное сопротивление в ом • м. ~ ом-мм2 Если же удельное сопротивление выражено в —-—, то соответственно этому удельная проводимость будет выражена м ОМ'ММ2 ’ В табл. 19 приведены удельные проводимости наиболее упо- требительных материалов. Таблица 19 Удельные проводимости важнейших материалов Материал Удельная проводимость м Материал Удельная проводимость м ом • мм* ом • мм* Серебро 61,0 Латунь 15,0—50,0 Медь 57,0 Никелин 3,0 Алюминий 34,0 Константан 2,04 Сталь 10,0 Нейзильбер 2—2,2 Свинец 5,0 Нихром ....... 0,9—1,06 136
§ 42. ЗАВИСИМОСТЬ СОПРОТИВЛЕНИЯ ПРОВОДНИКОВ ОТ ТЕМПЕРАТУРЫ С изменением температуры сопротивление проводников изме- няется. Металлические проводники, за очень небольшим исклю- чением, с ростом температуры увеличивают свое сопротивление. Между приростом сопротивления проводника и температурой его в пределах до 100° Ц существует прямая пропорциональная за- висимость. Для определения сопротивления проводника в зави- симости от температуры пользуются следующей формулой: =/?! + «/?! (^-ф, (54) где —сопротивление проводника .при температуре /х°Ц; /?2 — сопротивление проводника при температуре /20 Ц; а — коэффициент пропорциональности, называемый тем- пературным коэффициентом. Температурным коэффициентом называется прирост сопро- тивления проводника, приходящийся на каждый ом его сопро- тивления при изменении температуры на 1° Ц. В табл. 20 приведены средние значения температурных коэф- фициентов наиболее употребительных проводников. s Таблица 20 Температурные коэффициенты проводников Наименование материала Температурный коэффициент Наименование материала Температурный коэффициент Серебро 0,004 Нихром 0,00011 Медь 0,00445 Латунь 0,002 Алюминий 0,00423 Нейзильбер 0,00036 ' Вольфрам О,Ь'О464 Никелин 0,0003 Сталь 0,00625 Манганин 0,000015 Свинец 0,00411 Константан —0,00003 Ртуть 0,00027 Уголь —0,0005 Существуют и такие проводники, сопротивление которых с ростом температуры уменьшается, например уголь, сплавы не- которых металлов и электролиты. Для константана температурный коэффициент а = —0,00003, т. е. сопротивление его очень мало уменьшается с повышением температуры. Для растворов солей и кислот температурный ко- эффициент в среднем равен а = —0,025. Пример 36. Телеграфная линия при температуре =—20° Ц обладает сопротивлением /?1 = 800 ом. Определить сопротивление линии при темпе- ратуре = +30° Ц, если температурный коэффициент проволоки равен а = 0,005. Решение. По формуле (54) находим /?2 = /?! + а /?! — /j) = 800 + 0,005 • 800 [30 — (—20)] = 800 + 200 = 1000 ом, Г. е. сопротивление линии изменилось на 25%. 137
§ 43. РЕОСТАТЫ И МАГАЗИНЫ СОПРОТИВЛЕНИЙ Реостатом называется устройство, дающее возможность из- менять сопротивление электрической цепи и тем самым регули- ровать в ней величину тока. Реостаты по своей конструкции подразделяются на проволоч- ные и непроволочные. В проволочном реостате токоведущей частью является проволока, а в непроволочном — токопроводя- щий слой металла, нанесенный на основание из изоляционного материала. Проволочные реостаты в свою очередь подразделяются на два основных типа: реостаты со скользящим контактом и ступен- чатые реостаты. Рис. 59. Реостат со скользящим контактом: 1 — стойки; 2 — направляющий металлический стержень; 3 — ползун; 4 — прижимные контакты; 5, 6 и 7 — зажимы Реостаты со скользящим контактом. Реостаты со скользящим контактом дают возможность плавно изменять сопротивление электрической цепи. На рис. 59 показан один из существующих на практике типов реостата со скользящим контактом. На керамиковую трубу его навита проволока из константана или какого-либо иного сплава, применяемого для изготовления реостатной проволоки. Витки этой проволоки уложены на керамиковой трубе плотно друг к другу, так что при скольжении по ним ползуна они не могут быть сдвинуты с места. К стойкам 1 реостата прикреплен на- правляющий металлический стержень 2, по которому переме- щают ползун 3. Последний с помощью своих прижимных кон- тактов 4 плотно прижимается к виткам реостатной проволоки и этим обеспечивает надежный контакт проволоки с ползуном. Реостат имеет три зажима, из которых два (5 и 6) смонтиро- ваны на стойках /, по одному на каждой. К этим зажимам при- соединены концы проволоки реостата. Третий зажим (7) присо- единен к направляющему стержню 2 реостата. На рис. 60 приведена схема включения в цепь реостата со скользящим контактом для регулирования величины тока в цепи. 138
Реостат подключен к цепи зажимами Н и /7, из которых первый соединен с началом реостатной обмотки, а второй — с ползуном. Зажим К, соединенный с концом реостатной обмотки, оставлен свободным — не присоединенным к цепи. Перемещая скользящий контакт ползуна вдоль витков реостатной проволоки, можно плавно изменять величину вводимого в цепь сопротивления реостата. При крайнем левом положении скользящего контакта ползуна, т. е. когда он установлен непосредственно у зажима Н, введен- ное в цепь сопротивление реостата становится минимальным — практически равным нулю. Когда же скользящий контакт пол- зуна установлен у зажима К, то введенное в цепь сопротивление реостата становится максимальным. Рис. 60. Включение реостата со скользящим контактом для регулирования тока в цепи Для устройства реостатов применяют реостатную проволоку, изготовленную из различных сплавов металлов, например нике- линовую, константановую, нейзильберовую и т. п., или же из чистых металлов, например из железа или никеля. Реостатная проволока должна иметь высокое удельное сопро- тивление, малый температурный коэффициент и устойчиво вы- держивать продолжительный нагрев током до нескольких сот градусов Цельсия. Такие материалы, как нейзильбер, никелин и реотан, дешевы, легко обрабатываются, но не допускают нагревания больше чем до 200° Ц. Что же касается константана и других медноникеле- вых сплавов, то они выдерживают продолжительный нагрев до температуры 500° Ц. В табл. 21 приведены данные некоторых сортов реостатной проволоки. Реостаты со скользящим контактом весьма разнообразны как по конструктивному выполнению, так и по электрическим дан- ным. В качестве примера можно привести реостаты типа РП (рео- стат ползунковый), изготовляемые нашей электропромышлен- ностью: реостат типа РП-3, рассчитанный на сопротивления 500-н 1000 ом и соответственно на предельные токи 0,6 ~ 0,4 а; 139
Таблица 21 Реостатная проволока и ее данные Наименование материала Состав сплава Удельное сопротивление ом • мм2 м Температура плавления, °Ц Максимальная рабочая темпе- ратура, °Ц Константан 60% меди, 40% никеля 0,50 1206 500 Манганин 84% меди, 12% никеля, 4% марганца 0,43 910 100 Нейзильбер 60,6% меди, 25,4% цинка, 14% никеля 0,30 1100 200 Никелин 62% меди, 18% никеля, 20% цинка 0,42 1100 200 Нихром 62% никеля, 12% хрома, 26% железа 1,1 1550 1000 Реотан 53,4% меди, 25,3% ни- келя, 16,9% цинка, 4,4% железа 0,47 1100 200 реостат типа РП-4 — на сопротивления 1000-?- 2000 ом и соот- ветственно на токи 0,4 -т- 0,2 а и реостат типа РП-5 (в защит- ном металлическом кожухе) — на сопротивления 18 -4- 200 ом и соответственно на токи 4 -4- 1 а. На рис. 61 показан внешний вид одного из типов проволоч- ных реостатов со скользящим контактом, получивших широкое применение в радиотехнике. Ступенчатые реостаты. Наряду с реостатами, имеющими скользящие контакты, широкое применение нашли ступенчатые реостаты. BililB SgoaO [jSs Рис. 61. Реостат со сколь- зящим контактом, применяе- мый в радиотехнике ES“.‘ I Й J fe Рис. 62. Ступенчатый реостат: К — контакты, Ш — штурвал 140
На рис. 62 показан один из широко приме- няемых в электротех- нике типов ступенча- тых реостатов. Он со- стоит из ряда секций реостатной проволоки, присоединенных к со- ответствующим контак- там К на крышке рео- стата. Изменение со- противления реостата, введенного в электри- ческую цепь, дости- гается путем поворота штурвала Ш. При этом ползун, жестко связан- Рис. 63. Непроволочные реостаты ный с ним, скользит по контактам /С При переходе ползуна с одного контакта на дру- гой, соседний с ним контакт введенное в цепь сопротивление реостата изменяется скачком на величину сопротивления введен- ной и выведенной секций реостатной проволоки. Ступенчатые реостаты применяются главным образом в це- пях электрических машин, например, в качестве пусковых рео- статов для электродвигателей и т. д. Непроволочные реостаты: Весьма широкое применение в ра- диотехнике нашли так называемые непроволочные реостаты. В отличие от проволочных реостатов сопротивление их выпол- нено в виде пластинки кольцевой формы с нанесенным на нее тончайшим слоем токопроводящего материала. Изменение со- противления реостата, введенного в электрическую цепь, осуще- ствляется путем поворота ручки реостата. При этом ползун, жестко связанный с ручкой реостата, скользит по токопроводя- щему слою пластинки реостата и тем самым выводит или вводит часть его сопротивления. Непроволочные реостаты изготовляются на различные сопро- тивления, от килоомов до нескольких мегомов. На рис. 63 показан внешний вид непроволочных реостатов, применяемых в радиотехнике. Магазин сопротивлений. Магазином сопротивлений назы- вается прибор, дающий возможность установить с достаточно высокой степенью точности необходимое сопротивление. На рис. 64 показан штепсельный магазин сопротивлений, со- стоящий из набора катушек с проволокой различных сопротив- лений. Концы проволок катушек присоединены к соответствую- щим колодочкам, смонтированным на крышке прибора. Если в гнездо между двумя соседними колодочками вставить 141
Рис. 64. Внешний вид и схема магазина сопротивлений штепсель, то сопротивление ме- жду колодочками будет близко к нулю; если же штепсель изъять из гнезда, то между ко- лодочками будет введено со- противление той катушки, кон- цы которой присоединены к этим колодочкам. У каждого гнезда на крышке магазина по- мечено число, показывающее, какое сопротивление дает мага- зин, если из данного гнезда вы- нуть штепсель. х Проволока для магазинов сопротивлений применяется обычно манганиновая или кон- стантановая, как обладающая очень малым температурным коэффициентом. Следует пом- нить, что перегружать магази- ны сопротивлений током опасно из-за возможности их порчи от перегрева. § 44. СОПРОТИВЛЕНИЯ ПОСТОЯННОЙ ВЕЛИЧИНЫ Сопротивления постоянной величины, применяемые в радио- технике, подразделяются на проволочные и непроволочные. Сопротивления постоянной величины изготовляются на раз- личные номинальные сопротивления, от нескольких омов до не- скольких мегомов. Номинальная мощность сопротивлений постоянной величины, т. е. мощность, развиваемая в них током (мощность рассеяния), не должна превышать 3—5 ватт, иначе сопротивле- ние может быть повреждено из-за перегрева его током. Обычно номинальная мощность устанавливается такой, чтобы темпера- тура сопротивления при продолжительном нахождении его под нормальным током превышала температуру окружающей среды не больше чем на 50° Ц. Отклонения истинных величин сопротивления сопротивлений постоянной величины колеблются в пределах +5 — +20%. Электрической прочностью сопротивления постоян- ной величины называется предельное рабочее напряжение на нем, при котором мощность, развиваемая током в этом сопро- тивлении, не превышает номинальной и сопротивление может работать под током продолжительное время — несколько тысяч часов. 142
Электрическая прочность высокоомных сопротивлений по- стоянной величины достигает обычно нескольких сотен вольт. Проволочные сопротивления постоянной величины. В технике связи нашли широкое применение постоянные проволочные со- противления. На рис. 65 приведены типы постоянных проволоч- ных сопротивлений. Они состоят из керамической трубки, на которую намотана проволока из сплава металлов высокого удель- ного сопротивления (нихром, константан и др.). На концах трубки закреплены колпачки или хомутики, к которым припаяны концы проволоки. При помощи колпачков проволочное сопро- тивление подключается к схеме. В некоторых постоянных про- волочных сопротивлениях концы проволочной намотки при- паиваются к гибким жгутам из красной меди. Проволочная об- мотка сверху покрыта стекловидной эмалью, предохраняющей проволоку от окисления, механических повреждений и загрязне- ния и служащей электроизолирующим слоем. Постоянные про- волочные сопротивления изготовляются в пределах от единиц омов до нескольких десятков тысяч омов. Основное достоинство проволочных сопротивлений постоянной величины — их способность выдерживать относительно большую перегрузку током и стабильность величины их сопротивлений. Недостаток их — относительно большая величина собственных индуктивности и емкости, что приводит к сильной зависимости величины их сопротивления от частоты-тока. Чтобы снизить соб- ственные индуктивность и емкость сопротивления, применяют специальные способы намотки проволоки. Непроволочные сопротивления постоянной величины. В не- проволочных сопротивлениях токопроводящей деталью является полупроводник, например, из углерода или тонкого слоя метал- лизации. Рис. 65. Постоянные проволоч- ные сопротивления Рис. 66. Постоянные непро- волочные сопротивления 143
Наибольшее применение в радиотехнике получили непрово- лочные сопротивления типов ВС и МЛТ. Устройство их неслож- но. На фарфоровый стержень нанесен слой углерода или тон- чайший (молекулярный) слой металлизации. Эти полупроводя- щие слои покрыты сверху специальным лаком, что предохраняет их от влияния внешней среды и механического повреждения. На концах этих сопротивлений установлены металлические контакты, к которым припаиваются проводники радиотехнической схемы. На рис. 66 приведены некоторые основные типы постоянных непроволочных сопротивлений. § 45. СОПРОТИВЛЕНИЕ ПОЛУПРОВОДНИКОВ И ДИЭЛЕКТРИКОВ Идеальных диэлектриков в природе нет, а поэтому нельзя проводить резкую грань между диэлектриками и полупроводни- ками. Всякий диэлектрик в той или иной степени обладает про- водимостью, величина которой зависит от состояния диэлектрика и внешних физических условий. В табл. 22 приведены средние значения удельных сопротив- лений различных диэлектриков и полупроводников. Таблица 22 Удельные сопротивления диэлектриков и полупроводников Материал Удельное сопротивление, ом* см Материал Удельное сопротивление, ом>см Бакелит Бумага кабель- 1011—Ю14 Пахотная земля Песок 3,5-IO3—10-Ю3 10-103—100-Ю3 ная Глинистые поро- 1012—1013 Электрокартон Слюда Юн—Ю12 1012—1017 ды Дерево парафи- 1,5-103—10-10» Стекло Растительный пе- 1011—1015 нированное . . 10!2 регной .... Ю3—10-103 Каучук 1014—Ю16 Фибра 109—10Ю Камень песчаный Кварц плавле- 1011 Фарфор Чернозем .... 1014—1015 8-Ю3—20-Ю3 ный 1(Я Шеллак 1015—1016 Масло трансфор- Шифер 107—10*0 маторное . . . 1012—Ю13 Целлулоид . . . 2-Ю10 Миканит .... 101В Эбонит Ю17 Мрамор . . . , . Парафин • • • . 109—10П 1016—1О!6 Эксцельсиор . . . 1012—1Q13 Пример 37. Требуется определить сопротивление эбонитовой пластинки, имеющей площадь поперечного сечения S = 10 кв. см и толщину I = 0,5 см. Удельное сопротивление эбонита р = 1017 ом • см. Решение. Применяя формулу (48), находим 7? = р4- = Ю174к-=5-1015 ом, О 1U т. е. данная эбонитовая пластинка обладает сопротивлением в пять мил- лионов миллиардов ом.
ГЛАВА IX ПРОСТАЯ ЦЕПЬ ПОСТОЯННОГО ТОКА § 46. ЗАКОН ОМА ДЛЯ УЧАСТКА ЦЕПИ Опытом установлено, что между током 7, протекающим по участку цепи, напряжением I/, приложенным к этому участку цепи, и сопротивлением /? последнего существует строго опреде- ленная зависимость (рис. 67). Чем больше напряжение (7, приложенное к участку цепи, обладающему неизменным со- противлением /?, тем больше ток I в этом участке, и, наобо- рот, чем меньше это напряже- ние, тем меньше ток. Это по- нятно, так как чем больше на- пряжение на участке цепи, тем сильнее электрическое поле вну- три проводника этого участка, Рис. 67. К участку цепи с сопро- тем С большей СИЛОЙ дейст- тивлением R приложено напряжение вуют электрические силы этого 110 цепи ПРОХ°ДИТ ток 1 поля на электрические за- ряды проводника и тем, следовательно, больше ток в нем. Наоборот, чем меньше напряжение, тем слабее электрическое поле в проводнике, тем, следовательно, меньше ток в нем. Если величину напряжения U на данном участке цепи под- держивать постоянной и в то же время изменять сопротивле- ние /?, то и в этом случае ток на участке цепи будет изменяться. В частности, если сопротивление R будет возрастать, то ток, про- текающий по участку, будет уменьшаться. Это объясняется тем, что с ростом сопротивления участка цепи увеличивается проти- водействие вещества проводника этого участка электрическим зарядам. А так как сила поля в проводниках участка цепи сохра- няется неизменной из-за постоянства приложенного к нему на- пряжения U, то, естественно, скорость направленного движения зарядов в проводнике изменится и, следовательно, уменьшится 10—1377 145
ток в нем. Наоборот, если сопротивление проводника участка цепи уменьшится, то скорость направленного движения электри- ческих зарядов в проводнике при заданном неизменном напря- жении U увеличится, т. е. ток в проводнике возрастет. Зависимость между током /, напряжением U и сопротивле- нием R на участке цепи была впервые установлена в 1827 г. не- мецким физиком Г. С. Омом и вошла в электротехнику под названием закона Ома. Для участка цепи закон Ома формулируется следующим об- разом: величина тока I на участке цепи прямо пропорциональна напряжению (7, приложенному к этому участку, и обратно про- порциональна его сопротивлению 7?. Математически закон Ома для участка цепи выражается фор- мулой <55) где /—ток в амперах; R—сопротивление в омах; U — напряжение в вольтах. Пример 38. Определить ток / в потребителе, обладающем •сопротивле- нием /? = 250 ом, если напряжение, приложенное к нему, U = 125 в. Решение. По формуле (55) находим т U 125 Л - R ~ 250 —°>5 а‘ Из формулы (55) следует, что U = IR, (56) т. е. напряжение, приложенное к участку цепи, равно произве- дению величины тока I на величину сопротивления R. Пример 39. Определить, какое напряжение надо подать на зажимы элек- тронной лампы для питания нити накала ее, если известно, что при сопро- тивлении лампы R = 24 ом через ее нить должен протекать ток / = 150 ма. Решение. По формуле (56) находим = = 0,15-24 = 3,6 в. Из формулы (55) также следует, что Rr=-T> (57) т. е. сопротивление участка цепи численно равно отношению на- пряжения U к величине тока I на данном участке цепи. Пример 40. Определить сопротивление -4L проволоки катушки, если из* вестно, что при напряжении £7=125 в ток в ней равен 7 = 0,4 а. Решение. По формуле (57) находим я = -^ = -£1 = 312,5 ом. 146
§ 47. ЗАКОН ОМА ДЛЯ ВСЕЙ ЦЕПИ Закон Ома для всей цепи устанавливает зависимость между электродвижущей силой Е источника электрической энергии, ве- личиной тока I в цепи и полным сопротивлением R последней. Пусть задана простая электрическая цепь (рис. 68), состоя- щая из источника электрической энергии, обладающего электро- движущей силой Е и внутренним сопротивлением /?о, и внешнего сопротивления Если эту цепь замкнуть, то в ней установится некоторый ток /. Напряжение C7i, приложенное к внешнему сопротивлению /?1, со- гласно формуле (56) равно Аналогично этому во внутрен- нем участке цепи напряжение U» = IR,. Электродвижущая сила Е ис- точника электрической энергии, Рис. 68. Схема простой цепи по- стоянного тока как известно, равна сумме напряжений на внутреннем и внеш- нем участках цепи, т. е. или, иначе, (58) сопро- E = IRq + IRi = I(R, + RJ. Отсюда находим, что г Е Е Rq + Ri ~ R ' R = Rq+ Ry — полное сопротивление (эквивалентное j тивление) всей цепи в омах; Rq — внутреннее сопротивление источника электри- ческой энергии в омах; Rx — внешнее сопротивление цепи в омах; Е — электродвижущая сила источника электриче- ской энергии в вольтах; / — ток в цепи в амперах. Формула (58) является математическим выражением з а- кона Ома для всей простой цепи. Закон Ома для всей цепи формулируется следующим обра- зом: величина тока I в цепи прямо пропорциональна электро- 10* 147
движущей силе Е источника электрической энергии и обратно пропорциональна полному сопротивлению К всей цепи. Под полным сопротивлением цепи надо понимать сумму со- противлений внутреннего (/?0) и внешнего (/?i) участков цепи. Пример 41. К источнику электрической энергии с электродвижущей си- лой Е = 3 в и внутренним сопротивлением /?о = 1 ом подключен микрофон телефонного аппарата, имеющий в спокойном состоянии сопротивление, рав- ное /?1 = 47 ом. Определить ток / в цепи микрофона, напряжение СЛ, при- ложенное к микрофону, напряжение Uq на внутреннем участке цепи. Решение. Полное сопротивление R всей цепи R = /?о -|- R± = 1 4- 47 = 48 ом. Ток в цепи / = 4 = 4 = 0,0625 а. К ЧО Напряжение ГД на микрофоне Ц = = 0,0625 • 47 = 2,9375 в. Напряжение Uq на внутреннем участке цепи Ц = /#0 = 0,0625 • 1 = 0,0625 в. § 48. ЦЕПЬ ПОСТОЯННОГО ТОКА С ПОСЛЕДОВАТЕЛЬНО СОЕДИНЕННЫМИ ПОТРЕБИТЕЛЯМИ Если потребители включены в электрическую цепь один за другим в ряд, т. е. последовательно, так что электриче- ский ток нигде не ответвляется, то такая цепь называется последовательной. На рис. 69 изображена схема последовательной це- пи, состоящей из источника электрической энергии с элек- тродвижущей силой Е и вну- тренним сопротивлением Rq и трех последовательно со- единенных потребителей, об- ладающих сопротивлениями Ri, R2 и /?з. Так как в последовательной цепи электрический ток нигде не ответвляется и электрические заряды на пути своего движения нигде не накапливаются, то количество электрических зарядов, проходящих через любое поперечное сечение проводников цепи в единицу времени, всюду одинаково. Отсюда следует, что величина тока во всех участках последо- вательной цепи независимо от величин их сопротивлений уста- навливается одинаковой: /1 = /2 = /3 = /, (59) где /1, /2 и /3 — токи в отдельных участках цепи. 148
Справедливость этого положения легко проверить, включая амперметр в различные участки последовательной цепи. Напряжения на отдельных участках цепи равны Ц = //?1; i/2 = //?2; tZ3 = Z/?3. (60) Так как ток во всех участках цепи одинаков, а сопротивления их в общем случае различны, то и напряжения на отдельных участках последовательной цепи должны быть в общем случае различными. Разделив поочередно равенства (60) друг на друга, найдем соотношение напряжений на участках последовательной цепи: . ^2 __ ^2 . ^3__#3 /ГМЧ и2 — R, ’ U3 — R3 ’ т. е. напряжение в последовательной цепи распределяется по от- дельным участкам ее прямо пропорционально их сопротивле- ниям. Следовательно, чем больше сопротивление одного участка цепи по сравнению с сопротивлением другого, тем больше на- пряжение на зажимах первого из них по сравнению с напряже- нием на зажимах второго. Общее напряжение U, приложенное ко всей внешней после- довательной цепи, равно сумме напряжений, приложенных к от- дельным участкам цепи, т. е. t7=£71 + f72 + £73. (62) В справедливости формул (61) и (62) легко убедиться, из- меряя вольтметром напряжения на зажимах всей цепи и на от- дельных ее участках. Если все члены формулы (62) разделить на величину тока I, которая по условию постоянна, то получим । 14 । и* • 1 " i / • Исходя из формулы закона Ома для участка цепи, вместо по- следнего равенства можно написать R = Ri 4- Т?2 + /?з, (63) где R — полное (эквивалентное) сопротивление всей цепи; Rb R2 и Rs — сопротивления отдельных участков цепи (потре- бителей) . Из формулы (63) следует, что полное (эквивалентное) сопро- тивление последовательной цепи равно сумме сопротивлений от- дельных ее участков. 149
Закон Ома для внешнего участка последовательной цепи вы- разится формулой , U U R Ri + Ra + Ra ’ (64) т. е. величина тока I на внешнем участке последовательной цепи прямо пропорциональна приложенному к нему напряжению U и обратно пропорциональна его полному сопротивлению 2?. Внутренний и внешний участки всей цепи соединены между собой последовательно, а поэтому формула закона Ома для всей последовательной цепи принимает вид , Е — Е 1 “ Яо + + R2 + Яз ” R > т. е. величина тока 1 во всей последовательной цепи прямо про- порциональна электродвижущей силе Е источника электрической Рис. 70. Схема простейшей теле- графной цепи Решение. Сопротивление /?з энергии, действующего в цепи, и обратно пропорциональна полно- му сопротивлению Е всей цепи. Пример 42. Два телеграфных аппа- рата М-44, имеющие сопротивление = 340 ом каждый, соединены после- довательно медным проводом телеграф- ной линии протяженностью 35 км (рис. 70). Определить ток I в цепи и распределение напряжения по отдель- ным ее участкам, если сопротивление линии составляет 6 ом на километр, со- противление заземлений (земля яв- ляется «обратным проводом») /?2 = = 108 ом, электродвижущая сила источ- ника электрической энергии, питающего аппараты М-44, Е — 20 в и его внутрен- нее сопротивление Ro = 2 ом. всего медного провода /^3 = 35-6 = 210 ом. Полное сопротивление всей цепи /? = 2Т?! +/?2+/?з +/?о = 2-340 + 108 + 210 + 2 = 1000 ом. Ток в цепи т Е 20 л ла 1 ~ ~R ~ 1000 — 0,02 а' Напряжение на каждом из аппаратов М-44 Ц = = 0,02 • 340 = 6,8 в. Напряжение, теряемое в заземлениях, U2 = 1R2 = 0,02-108 = 2,16 в. 150
Напряжение, теряемое в медном проводе, t/3 = //^3 = 0,02-210 = 4,2 в. Напряжение, теряемое внутри источника электрической энергии, UQ = IRq = 0,02-2 = 0,04 в. Проверка правильности решения задачи: Е = U, + t/2 + U3 + UQ = 2 • 6,8 + 2,16 + 4,2 + 0,04 = 20 в. § 49. РАСПРЕДЕЛЕНИЕ ПОТЕНЦИАЛА В ЭЛЕКТРИЧЕСКОЙ ЦЕПИ Если в электрической цепи течет постоянный ток, значит внутри проводников этой цепи имеется стационарное электриче- ское поле. Потенциал его изменяется по величине при переходе от одной точки поля в^ другую в направлении тока. Наибольшим потенциалом во внешнем участке цепи обладает положительный зажим источника электрической энергии, к ко- торому подключен данный внешний участок цепи, а наименьшим потенциалом — отрицательный зажим. Падение потенциала происходите проводниках цепи в направлении течения постоянного тока. Наобо- рот, прирост потенциала можно наблюдать в противоположном направлении. Если известен потенциал какой-либо точки цепи, то, зная b -0 - а + 0- направление и величину тока, а также сопротивления отдельных участков цепи, можно определить потенциалы любых точек этой цепи. Допустим, что на некото- ром участке цепи ab (рис. 71) течет постоянный ток /, а сопротивление этого участ- ка равно гаЬ. Если известен потенциал точки а, то мож- но определить потенциал точки Ь. Так как ток I в участке цепи течет от точки а к точке &, то потенциал <ра первой из них больше, чем потенциал второй, т. е. I Рис. 71. На участке цепи ab падение потенциала происходит в направлении от точки а к точке Ь fa > fb- Падение напряжения на участке ab цепи равно произведению тока I на сопротивление гаЬ этого участка, т. е. иаь = 1гаь. Но напряжение Uab равно разности потенциалов точек а и Ь, т. е. 4a — fb = Irar 151
Из последнего равенства получаем ЧЬ = Ча — ^аЬ, т. е. потенциал конечной точки b участка цепи ab равняется потенциалу <?а начальной точки а этого участка минус падение напря- жения в сопротивлении гаЬ. Наоборот, если бы необходимо было по заданному потен- циалу точки Ь определить потенциал точки а, то мы при- менили бы формулу = + т. е. потенциал уа начальной точки а участка цепи ab равен потенциалу <рд конечной точки b этого участка плюс падение потенциала на сопротивлении гаЬ. На внутреннем участке цепи также происходит падение по- тенциала в направлении тока, т. е. от минус-зажима источника электрической энергии к плюсу. Однако наряду с этим на вну- треннем участке цепи наблюдается скачок потенциала на вели- чину, равную электродвижущей силе Е источника электрической энергии. Этот скачок потенциала будет положительным, если перемещаться по источнику электрической энергии в направле- нии действия электродвижущей силы, и отрицательным, если пе- ремещаться в противоположном направлении. Потенциал — величина относительная. Численное значение потенциала зависит от величины потенциала некоторой исходной точки цепи. Условно принято считать, что потен- циал земли равен нулю и, следовательно, вся- кая заземленная точка имеет нулевой по- тенциал. Если перемещаться по электрической цепи от точки с нулевым потенциалом в направлении тока, то будет на- блюдаться падение потенциала, т. е. последний будет приобре- тать отрицательные значения. Наоборот, если перемещаться от точки цепи с нулевым потенциалом навстречу току, то будет на- блюдаться прирост потенциала, т. е. последний будет приобре- тать положительные значения. Графическое изображение изменения потенциала вдоль всей замкнутой электрической цепи называется потенциальной диаграммой. В примере 43 показан расчет потенциалов электрической цепи и метод построения потенциальной диаграммы длящее. Пример 43. Требуется построить потенциальную диаграмму для цепи (рис. 72), если электродвижущие силы источников электрической энергии £1 = 16 в и £2 = 4 в, а их внутренние сопротивления соответственно равны г01 = 1 ом, гQ2 = 1 ом\ остальные сопротивления цепи равны: и = 2 ом, r2 = 1,5 ом и г3 = 0,5 ом, 152
Решение. Вначале определим направление и величину тока в цепи. Электродвижущие силы Ei и Ё2 направлены навстречу друг другу. Следова- тельно, результирующая электродвижущая сила будет равна разности Ei—Е2, а ток будет иметь направление, совпадающее с направлением дей- ствия электродвижущей силы Е\, так как Е\ > Е2. Значит, ток / во внешней цепи имеет направление от плюс-зажима источника электрической энергии с электродвижущей силой Е\ к минус-зажк?’у, как обозначено на рио 72 стрелкой. Величину тока I найдем, исходя из формулы закона Ома для последо- вательной цепи г ________Ei Еъ___________________16 4_________ Q /м + б + Лю + ^ + ^з 1+2 + 1 4-1,5 + 0,5 Потенциал точки а равен нулю (?а = 0), так как эта точка заземлена. Потенциал точки b ниже потенциала точки а на величину падения на* пряжения в сопротивлении гог. ъ = —//'01=0 —2 = —2 в. Потенциал точки с больше потенциала точки b на величину, равную электродвижущей силе Е\: Чс = + Е1 = + 16 = 14 в- Потенциал точки d меньше потенциала точки с на величину падения напряжения в сопротивлении и: <fa = <рг — 1гх = 14 — 2 • 2 = 10 в. Потенциал точки е меньше потенциала точки d на величину электродви- жущей силы E2i так как эта электродвижущая сила направлена навстречу току /: —Ъ — Е2 = Ю — 4 = 6 в. Потенциал точки f меньше потенциала точки е на величину падения напряжения в сопротивлении г'о2« ?/ = ?«. —^02 = 6 —2-1 =4 в. 153
Потенциал точки g меньше потенциала точки f на величину падения напряжения в сопротивлении г2: = — /г2 = 4 — 2 -1,5 = 1 в. Потенциал точки а меньше потенциала точки g на величину падения напряжения в сопротивлении г3: <?а = /гз = 1 — 2-0,5 = 0. Для построения потенциальной диаграммы нанесем на плоскость чер- тежа две взаимно перпендикулярные оси. По горизонтальной оси отложим в некотором масштабе сопротивления, а по вертикальной — потенциалы (рис. 73). W) Изменение потенциала между точками а и b показано в виде наклонной прямой aby между точками b нс— в виде вертикальной прямой Ьс, между точками с и d — в виде наклонной прямой cd, между точками d и е — в виде вертикальной прямой de, между точками е и f — в виде наклонной пря- мой ef, между точками f и g — в виде прямой fg и между точками g и а — в виде наклонной прямой ga.
ГЛАВА X РАБОТА И МОЩНОСТЬ ЭЛЕКТРИЧЕСКОГО ТОКА § 50. РАБОТА ЭЛЕКТРИЧЕСКОГО ТОКА Электрический ток, проходя по проводникам, совершает ра- боту, превращая электрическую энергию в какую-либо иную энергию: тепловую, световую, механическую, химическую и т. д. В дальнейшем мы ознакомимся более подробно с превращением электрической энергии в другие виды энергии, а пока рассмотрим основные формулы, по которым можно подсчитать расход элек- трической энергии. Если к потребителю электрической энергии приложено на- пряжение один вольт, то это значит, что источник электрической энергии, перенося один кулон электричества через потребитель, расходует в нем один джоуль электрической энергии. Если же к потребителю приложено напряжение U вольт и при этом через него проходит q кулонов электричества, то источ- ник электрической энергии отдает потребителю электрическую энергию, равную W=Uq. (65) Электрический ток превращает эту энергию в какой-либо иной вид энергии, а поэтому принято говорить, что электриче- ский ток, проходя через потребитель, совершает работу. Вели- чина этой работы А равна величине электрической энергии, из- расходованной источником, т. е. (66) Пример 44. Напряжение, приложенное к электрической лампочке, (7=127 в. Определить энергию, израсходованную в лампочке источником электрической энергии, если через лампочку прошло количество электри- чества, равное q = 200 к. Решение. По формуле (66) находим W = Uq = 127-200 = 25 400 дж. 155
Как известно, количество электричества qt прошедшее через потребитель за время /, равно произведению тока / на время т. е, q = It. , Подставив вместо q это его выражение в формулу (66), полу- чим вторую разновидность формулы по определению величины работы электрического тока: Д=Ж (67) где А—работа в джоулях; U— напряжение в вольтах; I—ток в амперах; t— время в секундах. Пример 45. Электрический чайник подключен к сети с напряжением [7 = 220 в. Определить энергию, израсходованную в чайнике за /=12 мин., если ток в нагревательном элементе чайника был равен I = 2,5 а. Решение. По формуле (67) находим 117 = ^ = 220-2,5-12-60 = 396000 дж. Формулу (67) можно преобразовать, принимая во внимание, что U=IR, тогда A = PRt. (68) Пример 46. Через нить накала электронной лампы проходил ток 7=150 ма в течение времени ^ = 2 часа. Определить энергию, израсходо- ванную источником электрической энергии в нити лампы, если сопротивле- ние ее R = 24 ом. Решение. По формуле (68) находим w = PRt = 0,152 • 24 •2 • 60 • 60 = 3888 дж. Формулу (68) также можно преобразовать, принимая во вни- мание, что тогда A = -^Rt^t. (69) Пример 47. Напряжение U, приложенное к катушке, равно 12 в. Опре- делить работу электрического тока в катушке за время t = 10 минут, если сопротивление катушки R = 300 ом. Г56
Решение. По формуле (69) находим Д = ^-.10-60 = 288 дж. i\ uUU В электротехнике сильных токов для измерения работы элек- трического тока принимается единица, называемая киловатт- час (квт-ч, kWh). 1 киловатт-час = 3 600 000 джоулям. В табл. 23 приведены единицы измерения работы электриче- ского тока с их условными обозначениями и соотношения с основной единицей. Т а б л и ц а .23 Единицы измерения работы электрического тока в системе единиц МКСА Наименование величины и ее обозначение Название единицы Обозначение Соотношение с основной единицей русское между- народное Работа А | джоуль гектоватт-час киловатт-час дж гвт-ч квт-ч J hWh kWh 3,6-105 дж 3,6-106 дж Примечание. За единицу измерения работы электрического тока в абсолютной электростатической системе единиц СГСЭ принимается 1 эрг. 1 джоуль = 107 эргов. § 51. МОЩНОСТЬ ЭЛЕКТРИЧЕСКОГО ТОКА Мощностью электрического тока назы- вается его работа, отнесенная к единице времени: р=4> <7°) где Р— мощность электрического тока; А— работа электрического тока, совершенная за время t. В системе единиц МКСА за единицу измерения мощности принимается один ватт (er, W). Электрический ток развивает мощность в один ватт, если он ежесекундно совершает работу, равную одному джоулю: 1 ПЛТт =1джоуль Г7П 1 секунда ’ ' 157
В табл. 24 приведены единицы измерения мощности с указа- нием их условных обозначений и соотношений с основной еди- ницей. Таблица 24 Единицы измерения мощности в системе единиц МКСА Наименование величины и ее обозначение Название единицы Обозначение Соотношение с основной единицей русское между- народное ватт вт W __ киловатт кет kW 103 вт Мощность Р гектоватт гвт hW 102 вт милливатт мет mW 10“3 вт микроватт мквт pW 10“6 вт Пример 48. Электрический ток в течение t = 8 мин. совершил работу А = 1200 джоулей. Определить мощность, развиваемую током. Решение. По формуле (70) находим п А 1200 п - Р==7-=8^о =2>5 вт- Подставив в формулу (70) вместо А выражения из фор- мул (67), (68) и (69), получим еще три разновидности формул для определения мощности электрического тока: P = ^ = UI, (72) т. е. мощность электрического тока равна произведению вели- чины тока I на величину напряжения U‘t P==J1BL==PP) (73) т. е. мощность, развиваемая током в неизменном сопротивле- нии J?9 прямо пропорциональна квадрату величины тока; т. е. мощность электрического тока, развиваемая им в неизмен- ном сопротивлении R, пропорциональна квадрату напряжения, приложенного к этому сопротивлению. Пример 49. Напряжение, приложенное к реостату, U = 40 в. Определить мощность, развиваемую в реостате, если ток в последнем I = 25 а. Решение. По формуле (72) находим р = 67 = 40-25 = 1000 вт = \ квпг. Пример 50. Каким сопротивлением обладает электрическая лампа при горении, если при токе I = 0,5 а она потребляет мощность Р = 50 вт. 158
Решение. Из формулы (73) находим ^ = Т = |г=200^ Пример 51. Под каким напряжением находится электрический нагрева- тельный прибор, имеющий сопротивление R = 24 ом, если он потребляет мощность Р = 600 вт. Решение. Из формулы (74) находим U = VPR = V 600-24 = 120 в. § 52. ИССЛЕДОВАНИЕ РЕЖИМА В ЦЕПИ С ИСТОЧНИКОМ ЭЛЕКТРИЧЕСКОЙ ЭНЕРГИИ, ИМЕЮЩИМ ПОСТОЯННУЮ ЭЛЕКТРОДВИЖУЩУЮ СИЛУ И ПОСТОЯННОЕ ВНУТРЕННЕЕ СОПРОТИВЛЕНИЕ Ознакомившись с расчетными формулами и единицами из- мерения мощности и работы электрического тока, исследуем ре- жим в цепи с источником электрической энергии, обладающим постоянной электродвижущей сопротивлением. Допустим, что задана электрическая цепь постоян- ного тока (рис. 74), состоя- щая из источника электриче- ской энергии, обладающего постоянной электродвижущей силой Е и постоянным вну- тренним сопротивлением Ro, и потребителя, сопротивле- ние которого Ri можно из- менять от нуля до беско- нечно большой величины. Исследуем для заданной це- пи зависимость напряже- ния U на зажимах источника силой и постоянным внутренним Рис. 74. Схема электрической цепи с Е = const, = const и переменным сопротивлением Rx электрической энергии, полной мощности Pq, полезной мощно- сти Р и электрического коэффициента полезного действия т] от нагрузки (тока) I в цепи, начиная от режима холостого хода до режима короткого замыкания источника электрической энергии. При режиме холостого хода источника электрической энер- гии ток в цепи равен нулю, так как сопротивление внешнего участка цепи бесконечно велико (внешняя цепь отключена от источника электрической энергии), т. е. А.Х ’0, R1X.X где /ХфХ — ток холостого хода; /?1Х,Х — внешнее сопротивление при холостом ходе. 159
При режиме короткого замыкания источника электрической энергии ток в цепи достигает максимальной величины, так как сопротивление внешнего участка цепи равно нулю, т. е. 3 == И /?1К, 3 = О, где 1К, з — ток короткого замыкания источника электрической энергии; RiK.3 — внешнее сопротивление при коротком замыкании. 1. Напряжение на зажимах источника электрической энергии, как известно, равно разности между электродвижущей силой источника электрической энергии и падением напряжения во внутренней цепи последнего, т. е. U = E-IRQ, (75) где U — напряжение источника электрической энергии в воль- тах; Е — электродвижущая сила в вольтах; 7?0 — внутреннее сопротивление в омах; I — нагрузка источхчика электрической энергии в амперах. Так как электродвижущая сила источника электрической энергии по условию постоянна, т. е. Е = const, и внутреннее со- противление его также постоянно, т. е. /?0 = const, то, как это видно из формулы, между напряжением U и током I существует линейная зависимость. Это значит, что график зависимости на- пряжения U от тока I должен изображаться прямой линией. Для построения этого графика необходимо определить какие-либо две его точки, так как по двум точкам всегда можнсТ построить пря- мую линию. В данном случае для определения этих двух точек графика мы воспользуемся режимами холостого хода и корот- кого замыкания. а) При холостом ходе 4. х — 0. Подставив величину тока холостого хода /х. х в формулу (75), получим ^х. X = х^?о = 0 • /?0 = где С7Х.Х — напряжение на зажимах источника электрической энергии при холостом ходе. Следовательно, напряжение на зажимах источ- ника электрической энергии при холостом ходе равно электродвижущей силе этого источника. б) При коротком замыкании I = ~ Ro * 160
Подставив величину дока короткого замыкания 4. 3 в фор- мулу (75), получим Us. з = Е — 4.3/?0 = Е — Ro = Е — Е == О, где £7К. 3 — напряжение на зажимах источника электрической энергии при коротком замыкании. Следовательно, напряжение на зажимах источ- ника электрической энергии при коротком замыкании его равно нулю. На рис. 75 показан график зависимости напряжения U ис- точника электрической энер- гии, обладающего постоянной электродвижущей силой Е и постоянным внутренним сопро- тивлением /?о, от нагрузки I. Здесь угол р характеризует сте- пень наклона прямой (гра- фика) к горизонтальной оси, т. е. быстроту падения напря- жения U с ростом нагрузки /. Величину угла р можно опре- делить из прямоугольного треугольника OUXt 3 по его тан- генсу 'к. 3 и Ro Следовательно, чем больше внутреннее сопротивление /?0, тем больше tg р и угол р, т. е. тем быстрее уменьшается напряже- ние U с ростом нагрузки /. Отсюда можно сделать практиче- ский вывод, что чем меньше внутреннее сопротивление Ео источ- ника электрической энергии, тем меньше зависимость напряже- ния (7 на зажимах источника электрической энергии от на- грузки /. При весьма малом внутреннем сопротивлении Ео источника, например аккумулятора, сопротивление которого из- меряется сотыми долями ома, напряжение на его зажимах в пределах допустимой нагрузки можно считать практически по- стоянным по величине. 2. Полной мощностью источника электрической энергии на- зывают мощность, которую он развивает во всей цепи, т. е. как во внутренней, так и во внешней. Рассмотрим зависимость полной мощности, развиваемой источником электрической энергии, от нагрузки» 11—1377 161
Полная мощность, развиваемая источником электрической энергии в цепи, определяется следующей формулой: Р. = ЕЦ ’ (76) где Ро — полная мощность, развиваемая источником электриче- ской энергии, в ваттах; Е — электродвижущая сила в вольтах; / — нагрузка в амперах. Электродвижущая сила источника тока постоянна по вели- чине, а поэтому, как следует из формулы (76), между полной мощностью Ро и током I существует прямая пропорциональная (линейная) зависимость. Следовательно, для построения графика зависимости полной мощности Ро от нагрузки I необходимо определить две точки графика. Для этой цели опять воспользуемся режимами холо- стого "хода и короткого замыкания источника электрической энергии. а) При холостом ходе Л.х = 0. Подставив величину тока холостого хода Zx. х в формулу (76), получим полную мощность РОх. х при холостом ходе: РОх. х = ^4. х = Е -0 = 0, т. е. полная мощность источника электрической энергии при холостом ходе равна нулю. б) При коротком замыкании т JL /кэ== Ро ’ Подставив величину тока короткого замыкания /к. 3 в фор- мулу (76), получим полную мощность РОк. з при коротком за- мыкании: т. е. при коротком замыкании полная мощ- ность, развиваемая источником электриче- ской энергии, максимальна. На рис. 76 показан график зависимости полной мощности от нагрузки, развиваемой источником электрической энергии с по- стоянной электродвижущей силой и постоянным внутренним со- противлением. 3. Полезной мощностью источника электрической энергии на- зывается мощность, развиваемая им во внешнем сопротивлении. 162
Полезная мощность источника электрической энергии опре- деляется формулой P=UI, (77) где Р—полезная мощность в ваттах; U — напряжение на зажимах источника электрической энер- гии в вольтах; / — ток в амперах. Но, как известно, U = E — IR0, а поэтому P = (E-IR9)I=EI-FR0. (78) Из полученной формулы следует, что полезная мощность Р равна разности между полной мощностью PQ = EI и мощностью потерь внутри источника электрической энергии Рвнут =/2/?0. Как видно из формулы (78), зависимость полезной мощно- сти Р от нагрузки сложная и вы- ражается она графически кривой, называемой параболой. Опреде- лим три характерные точки этой кривой, а именно точки, соответ- ствующие режиму холостого хода, короткого замыкания и макси- мальной полезной мощности. а) При холостом ходе 4.х = о. Рис. 76. График зависимости полной мощности Ро от тока / Подставляя величину тока холостого хода /х, х в фор- мулу (78), находим Px.x=FZx.x-Z2XtX7?o = £-O-O/?o = O, т. е. полезная мощность источника электриче- ской энергии при холостом ходе равна нулю. б) При коротком замыкании I =-£- /?0 • Подставляя величину тока короткого замыкания 1К, 3 в фор- мулу (78), находим 73». э= з 3 Ro = Е Ro ~ О» 11* 163
т. е. полезная мощность источника электриче- ской энергии при коротком замыкании его равнануЛю. в) Основываясь на методах высшей математики, которыми мы здесь воспользоваться не имеем возможности, можно было бы доказать, что источник электрической энергии развивает максимальную полезную мощность в том случае, когда внешнее сопротивление Pi цепи равно внутреннему сопротивлению Ро, т. е. Pi — Ro. Величина тока в цепи для этого случая будет равна . Е _______ Е /к . э 1 ~ Ro + Rx ~~ 1Р? — ’ т. е. источник электрической энергии разви- в а ет м а к С и м а л ь н у ю п о л е з н у ю мощность в том случае, когда его нагрузка равна половине тока короткого замыкания. На рис. .77 показан график зависимости полезной мощности Р от нагрузки I. 4. Электрическим коэффициентом полезного действия назы- вается отношение полезной мощности к полной мощности источ- ника электрической энергии, т. е. (79) * о где т) — электрический коэффициент полезного действия; Р—полезная мощность; Ро — полная мощность. Выясним зависимость электрического коэффициента полез- ного действия к; от нагрузки I источника электрической энергии. Для этого преобразуем формулу (79), подставив вместо Р и Ро соответствующие им выражения: _ Р _ Ul _ Е— IR0 _ . //?0 _ 1 I 71 — Ро El ~ Е ~1 Е ~1 Е ’ или, иначе, 71 = 1--Д-. (80) *К. 3 Из формулы (80) следует, что зависимость электрического коэффициента полезного действия к] от нагрузки I линейная. Следовательно, график зависимости выражается прямой линией. Для построения этого графика определим величину т; при хо- лостом ходе и при коротком замыкании. а) При холостом ходе /х. ж = 0. 164
Подставляя величину тока холостого хода /х., в форму- лу (80), находим, что т. е. электрический коэффициент полезного действия при холостом ходе равен единице. Рис. 77. График зависимости полезной мощности Р от тока I Рис. 78. График зависимости электрического коэффициента полезного действия щ от тока / Величину т] — 1 при холостом ходе источника электрической энергии надо рассматривать как тот предел, к которому стре- Р мится отношение -р- по мере уменьшения нагрузки источника. В пределе, когда Р-> 0 и Ро -*• 0, их отношение стремится к еди- нице, т. е. т] -> 1. б) При коротком замыкании I = ~ к-3 Ro * Подставив величину тока короткого замыкания /к,3 в фор- мулу (80), получим = = =0, 'к. э т. е. электрический коэффициент полезного действия при коротком замыкании источника электрической энергии равен нулю. На рис. 78 показан график зависимости электрического коэф- фициента полезного действия т) источника электрической энер- гии от нагрузки 7. При максимальной полезной мощности, развиваемой источни- ком электрической энергии, ток в цепи I = . Следовательно, А 165
электрический коэффициент полезного действия в этом случае будет равен Лс. э т] = 1—= 1 — 0,5 = 0,5, или 50%, ‘К. 3 т. е. при максимальной полезной мощности, раз- виваемой источником электрической энергии, половину энергии, вырабатываемой им, он поглощает сам. Напомним, что рассмотренные зависимости были исследованы для источника электрической энергии с постоянной электродви- жущей силой и постоянным внутренним сопротивлением. К та- ким источникам можно отнести химические источники электриче- ской энергии, например гальванические элементы и аккумуля- торы. Для электрических машин эти соотношения иные, более сложные; о них будет сказано ниже, в разделе электрических машин.
ГЛАВА Xl ЦЕПЬ ПОСТОЯННОГО ТОКА С ПАРАЛЛЕЛЬНО И СМЕШАННО СОЕДИНЕННЫМИ СОПРОТИВЛЕНИЯМИ § 53. ПЕРВЫЙ ЗАКОН КИРХГОФА В отличие от последовательной цепи в параллельной цепи электрический ток распределяется по нескольким направ- лениям. На рис. 79 изображена схема параллельной цепи, состоящей из трех рараллельно соединенных сопротивлений /?ь /?2 и /?3. Точки а и b этой цепи, к ко- торым присоединены свои- ми концами сопротивле- ния, называются узловы- ми точками схемы. Так как электрические заряды при своем движе- нии по проводникам цепи нигде не накапливаются и нигде не теряются, то ко- личество электричества. Рис. 79. Схема параллельной цепи с включенными в нее амперметрами притекающего ежесекунд- но к любой узловой точке цепи, равно количеству электричества, утекающего от нее по параллельно соединенным сопротивлениям (ветвям). А это значит, что величина тока в про- воде до узловой точки цепи должна равняться сумме величин то- ков, имеющихся в параллельных ветвях, т. е. для схемы, изобра- женной на рис. 79, можно написать: \ 4 = 4 + 4 + 4* (81) В справедливости последней формулы легко убедиться, изме- рив амперметром токи во всех участках параллельной цепи. Формула (81) является математическим выражением первого закона Кирхгофа, который для общего случая формулируется так. 167
Сумма токов, притекающих к любой узловой точке парал- лельной цепи, равна сумме токов, утекающих от нее. Если токи, притекающие к узловой точке цепи, условно счи- тать положительными, а утекающие от нее — отрицательными, то первый закон Кирхгофа можно сформулировать так. Алгебраическая сумма токов, притекающих к точке развет- вления, равна нулю: Л+4+4+. ..=0, (82) или, иначе, EZ = O, где I/ — алгебраическая сумма токов. Пример 52. К узловой точке цепи притекают токи Zi = 3 а, /2 = 5 а и утекают от нее токи /3=1,5 а, Ц = 4,5 а и /5. Требуется определить ток /5. Решение. По закону Кирхгофа Л + А = 4 + Л + Л, или 3 + 5 = 1,54-4,5 + 4, откуда находим Z5 = 2 я. Если эту задачу решать, исходя из формулы (82), то мы должны поло- жить, что /1 = 3 а\ 12—5 а\ /3 =—1,5 а и Ц = —4,5 а и" тогда £7 = /1 + /2 + /3 + /4 + /5 = 0, или 3 + 5-1,5 — 4,5 + Z5 = 0, откуда Л== 2 я, v т. е. ток /5 утекает от точки разветвления. § 54. НАПРЯЖЕНИЯ НА ЗАЖИМАХ ПАРАЛЛЕЛЬНО СОЕДИНЕННЫХ СОПРОТИВЛЕНИЙ Если сопротивлениями проводов, идущих от источника элек- трической энергии к узловым точкам параллельной цепи, можно пренебречь, то напряжения на зажимах параллельно соединен- ных сопротивлений должны быть равны друг другу и равны на- пряжению на зажимах источника электрической энергии, т. е. = = = (83) 168
где Ub U2 и т. д.— напряжения на отдельных параллельно со- единенных сопротивлениях (потребителях); U — напряжение на зажимах источника электри- ческой энергии. В справедливости формулы (83) можно убедиться при по- мощи вольтметров, включенных в цепь согласно рис. 80. Если же сопротивлением проводов, идущих от источника электрической энергии к параллельно соединенным потребите- Рис. 80. Схема параллельной цепи с включенными в нее вольтметрами Рис. 81. Схема параллельной цепи, в которой неразветвленный участок обладает некоторым сопротивлением лям, пренебречь нельзя (рис. 81), то напряжение на зажимах параллельно соединенных потребителей будет меньше, чем на зажимах источника электрической энергии, на величину падения напряжения в проводах, т. е. Ц = £/2 = Ц=£7-//?пр) (84) где /—ток в соединительных (подводящих) проводах; /?пр — сопротивление соединительных проводов. Пример 53. Генератор, обладающий электродвижущей силой Е = 130 в и внутренним сопротивлением /?о = 0,3 ом, питает током три параллельно соединенных потребителя, потребляющих токи, соответственно patBHhie Л = 2 а, 12 = 3 а и /3 = 5 а. Определить напряжение на зажимах генера- тора и напряжения на зажимах потребителей, если сопротивление подводя- щих проводов 7?пр = 0,2 ом. Решение. Ток / в подводящих проводах согласно первому закону Кирхгофа равен / = Д 4~/2 + /з = 2 + 34-5=10 а. Падение напряжения внутри генератора = = 10-0,3 = 3 в. 169
Напряжение на зажимах генератора U = E—Uq = 130 — 3 = 127 5. Падение напряжения в подводящих проводах С7пр = //?„₽ = 10-0,2 = 2 в. Напряжение на приемниках -2 = 125 в. § 55. РАСПРЕДЕЛЕНИЕ ТОКОВ В ПАРАЛЛЕЛЬНО СОЕДИНЕННЫХ СОПРОТИВЛЕНИЯХ Напряжения на зажимах параллельно соединенных сопро- тивлений (потребителей) равны друг другу, если они присоеди- нены к одним и тем же узловым точкам схемы: t71==t/2 = t/3 = t7. Отсюда следует, что если сопротивления параллельно пенных потребителей различны, то токи в них согласно Ома должны быть также различными: г — и т _ и т _ и J'~ Ri ’ У2 — R;s И ;3 — R3- соеди- закону Разделив последние равенства поочередно друг на друга, получим 4 __ ^2 . 4 __ „ 4 _____ 4 — Ri ’ 4 R2 4 ~ R* т. е. токи в параллельно соединенных потребителях распреде- ляются обратно пропорционально величинам их сопротивлений. Это значит, что во сколько раз первое сопротивление больше второго, во столько раз ток во втором сопротивлении больше, чем в первом, и т. д. В справедливости формулы (85) легко убедиться при помощи амперметров, включенных в параллельные ветви. Пример 54. Цепь составлена из двух параллельно соединенных сопро- тивлений = 12 ом и /?2 == 5 ом. Определить ток в первом из них, если во втором ток 4 == 24 а. Решение. По формуле (85) находим _4__^ 4 Ri ’ откуда г 4/?2 1 = "RT 24-5 12 = 10 а. 170
Допустим, что ток /0 разветвляется по двум параллельно со- единенным сопротивлениям и R2. Определим токи 7j и I2 в этих сопротивлениях, выразив их через общий ток /0. Согласно формуле (85) •» Л —А 4 “ • Прибавив к правой и левой частям этой пропорции по еди- нице, получим - 2k । 1 = 2^1 । 1 4 + Ri + Ъ или, иначе, Л + 4___ Ri + Rt 4 ~ Ki ‘ Но согласно первому закону Кирхгофа Л + 4 = а поэтому /0 “Ь ^2 4 Ri 1 откуда находим (86) Аналогично для тока 71 получим / — I Ji Rt + Rt '°' § 56. ЭКВИВАЛЕНТНОЕ СОПРОТИВЛЕНИЕ НЕСКОЛЬКИХ ПАРАЛЛЕЛЬНО СОЕДИНЕННЫХ ПОТРЕБИТЕЛЕЙ Пусть задана цепь постоянного тока, состоящая из трех па- раллельно соединенных потребителей, обладающих сопротивле- ниями Ri, R2 и Rs. Определим общее сопротивление этих 'по- требителей. Если пренебречь падением напряжения в подводящих прово- дах, то напряжения на зажимах всех потребителей будут равны друг другу и напряжению на зажимах всей цепи, т. е. и1 = и2=и3=и. Токи во всех параллельных ветвях можно определить по за- кону Ома: / -JL. i = JL 1 — Rt ’ 2 R, ’ 3 R3 • 171
Применяя первый закон Кирхгофа к любой из узловых точек цепи, можем написать, что [ Т I / I I _У_ _L _1_ jZ. /0 — /] -f- /2 /3 — -f- R* -f- R* , где Io — общий ток (ток в неразветвленном участке цепи). Если эквивалентное сопротивление всей цепи равно R, то согласно закону Ома I =^~ Jo R и вместо последнего равенства можем написать U _ U U U R ~ Rt + R, R3' Разделив все члены последнего равенства на U, получим Но, как известно, = (?1 — проводимость первой ветви; = С2 — проводимость второй ветви; -4- = О3 — проводимость третьей ветви; -R- = & — эквивалентная проводимость всей цепи. Принимая во внимание последние выражения, вместо фор- мулы (87) можем написать + + (88) т. е. эквивалентная проводимость нескольких параллельно со- единенных потребителей равна сумме их проводимостей. После того как по формуле (88) найдена эквивалентная про- водимость G, нетрудно определить эквивалентное сопротивление по формуле = (89) Пример 55. Определить эквивалентное сопротивление четырех парал- лельно соединенных потребителей, обладающих сопротивлениями = 2 ом, #2 = 6 ом, #з = 4 ом, #4 = 3 ом. Решение. По формуле (87) находим # “ #i “Т" #2 "Г #3 * #4 “ 2 6 “Г 4 -г 3 “ 4 ’ 172
откуда определяем эквивалентное сопротивление /? = 4- = -4- = 0,8 ом. 5 5 ’ т В частном случае, если заданы только два параллельно со- единенных потребителя, обладающих сопротивлениями 7?i и /?2, 1—111 __£l±A R ~ Ri ф R2 ~ RiRi » откуда <9°) т. е. эквивалентное сопротивление двух параллельно соединенных потребителей равно произведению их сопротивлений, разделен- ному на их сумму. Пример 56. Соединены параллельно два потребителя, имеющих сопро- тивления /?1 = 0,6 ом, Rz = 0,3 ом. Определить эквивалентное сопротивле- ние этих потребителей. Решение. По формуле (90) находим Если несколько параллельно соединенных потребителей имеют одинаковые сопротивления, т. е. /?! = /?2 = ^3 == ••• = Rn> то согласно формуле (87) —=— +—+ +—=— R Ri Ri •“ Rx Rx’ где п — число всех параллельно соединенных потребителей. Из последнего равенства получим, что /? = -§-> <91) т. е. эквивалентное сопротивление нескольких одинаковых парал- лельно соединенных потребителей равно сопротивлению одного из потребителей, деленному на число их. Пример 57. Параллельно соединено п = 5 ламп по R\ — 400 ом каждая. Определить эквивалентное сопротивление всех ламп. Решение. По формуле (91) находим /? = = 4г = 80 ом. п о 173
Следует отметить, что при параллельном соединении несколь- ких потребителей их эквивалентное сопротивление всегда будет меньше наименьшего из сопротивлений, имеющегося в данном соединении. § 57. ПОТЕНЦИОМЕТР В тех случаях когда для потребителей необходимо взять от источника электрической энергии определенную часть, напряже- ния, применяются приборы, называемые потенциометрами. ' В простейшем случае в качестве потенциометра может быть использован обычный ползунковый реостат. На рис. 82 показана схема включения потенциометра в цепь. Проволока потенцио- метра через зажимы (точки а и b на схеме) подключается к за- жимам источника электрической энергии (под полное напряже- ние). К потребителю же от потенциометра подаются два про- вода, из которых один присоединен к одному из концов прово- Рис» 82. Схема включения потенциометра в цепь Рис. 83. Внешний вид потенциометра, приме- няемого в радиотехнике локи потенциометра (точка а на схеме), а другой — к подвиж- ному контакту с последнего. Тогда, если на концах проволоки по- тенциометра имеется полное напряжение источника электриче- ской энергии, на концах проводов, подающих напряжение к по- требителю, будет только часть этого* напряжения. Величина по- следнего зависит от того, в каком положении установлен подвиж- ный контакт с потенциометра. Если он находится в точке 6, то потребитель получит максимальное возможное напряжение, если же он находится в точке а, то напряжение у потребителя будет равно нулю. На рис. 83 показан внешний вид потенциометра, применяе- мого в радиотехнике. § 58. ЦЕПИ ПОСТОЯННОГО ТОКА СО СМЕШАННО СОЕДИНЕННЫМИ СОПРОТИВЛЕНИЯМИ И ОДНИМ источником ЭЛЕКТРИЧЕСКОЙ ЭНЕРГИИ Электрическая цепь, в которой имеются участки с последова- тельным и параллельным соединением сопротивлений, назы- вается цепью со смешанно соединенными сопротивлениями или 174
просто смешанной цепью. На рис. 84 изображена смешанная цепь, состоящая из участков ab, Ьс и de, соединенных последова- тельно друг с другом; одновременно с этим участки Ьс и de со- стоят каждый в отдельности из совокупности нескольких парал- лельно соединенных сопротивлений. Смешанная цепь, питаемая одним источником электрической энергии, обычно рассчитывается методом последова- тельного упрощения схемы, т. е. данная смешанная цепь преобразуется в более простую, эквивалентную ей схему. Отметим, что эквивалентной схемой по отношению Рис. 84. Схема смешанно соединенных сопротивле- ний к данной реальной схеме называется такая схема, в которой источник электрической энергии создает такой же ток и разви- вает такую же мощность, что и в данной реальной схеме. После преобразования смешанной цепи в более простую экви- валентную схему определение режима в цепи значительно упро- щается и обычно сводится к применению закона Ома для после- довательной цепи. Рассмотрим применение метода последовательного упроще- ния схемы к расчету смешанной цепи, схема которой представ- лена на рис. 84. Допустим, что в заданной смешанной цепи нам известны напряжение U, приложенное ко всей цепи, и все со- противления, имеющиеся в этой цепи. Требуется определить токи: /о, Л, h, h, h и /5. Решение. В данной смешанной цепи три участка: ab, Ьс и de, соединенные между собой последовательно. Определим эквивалентные сопротивления участков ab, Ьс и de смешанной цепи, применяя уже известные правила расчета последовательной и параллельной цепей. ' 175
Для участка цепи ab Rab = ri + r2. Для участка цепи Ьс Решив последнее уравнение относительно RbCi найдем экви- валентное сопротивление участка цепи Ьс. Для участка цепи de _L = J_ + _L_ +_________!___. Rde Р, + ^7 + + Решив это уравнение относительно Rdei определим эквива- лентное сопротивление участка цепи de. Рис. 85. Схема, эквивалентная цепи, изобра- женной на рис. 84 Теперь мы можем перейти к эквивалентной цепи, состоящей из совокупности трех последовательно соединенных сопротивле- ний: Rab, Rbc и Rde (рис. 85). Применив формулу закона Ома для последовательной цепи, определим ток /о в ней, а следовательно, и ток /о в неразвет- вленном участке данной смешанной цепи, так как по условию эквивалентности схем эти токи должны быть одинаковыми по величине. Итак, I и ° Rab + Rbc + Rde Определив ток /0, найдем падение напряжения на участках ab, Ьс и de эквивалентной схемы: U аъ UЬс ъ^Ьс* de ^oRde* 176
Но в силу эквивалентности рассматриваемых нами схем на- пряжение Uab в эквивалентной схеме равно напряжению. Uab в смешанной цепи и т. д. Зная напряжения на участках смешанной цепи, легко опре- делить токи в параллельных ветвях этих участков: / Ubc . J Ubc . 1 Гз ’ 2 Г4 ’ / __ . Г __ Ude . т __ Ude 3 ГЬ ’ 4 Г6 + Г7 ’ 5 ^8 + бй + 6о Определив искомые токи, можно проверить правильность ре- шения задач, применив первый закон Кирхгофа. Для рассматри- ваемой смешанной цепи должно удовлетворяться следующее соотношение; Рис. 86. Схема смешанного соединения сопротивлений Пример 58. Определить токи во всех участках смешанной цепи (рис. 86) и общую мощность, развиваемую током в цепи, если напряжение источника электрической энергии U — 30 в, сопротивление и = 1 ом, г2 = 1 ом, г3 = 5 ом, г4 = 6 ом, Г5 = 3 ом, гб = Ю ом. Решение. Рассматриваемая цепь состоит, из двух последовательно со- единенных участков цепи ag и gh. В свою очередь участок цепи ag состоит из двух параллельных ветвей ef и bd. Будем решать задачу методом последовательного упрощения схемы. Определим эквивалентное сопротивление участка ag смешанной цепи. Для этой цели вначале определим эквивалентные сопротивления его парал- лельных ветвей ef и bd: RgJ = /*2 4“ ^*3 = 1 4“ 5 = 6 ом\ R“-^+'i--6T3+,l>=12"' Теперь мы можем определить эквивалентное сопротивление участка ag смешанной цепи: р — 6-12 =4 ом ^-R^ + Rm 6+12 12—1377 177
Эквивалентное сопротивление всей смешанной цепи равно = Rag 4- Rgh = 44-1=5 ом. Определяем ток 7о в неразветвленном участке смешанной цепи: г __ U _ 30 _ й Rah 5 6 • Определяем напряжение Uag: Uag = U— /ог1=ЗО — 6-1 = 24 в. Определяем ток /1, протекающий через сопротивления Г2 и г3: Ц = а. 1 Ref *» Определяем ток /2, протекающий через сопротивление Гб’ Uai, 24 4 = -^ = тг = 2 а. ttbd 12 Определяем напряжение Ubc: Ubc = ^6 = 24-2-10 = 4 в. Определяем ток /3, протекающий через сопротивление г4: = а. Определяем ток /4, Протекающий через сопротивление Г5’ а- гъ ° Общая мощность, развиваемая током в смешанной цепи, равна pQ = UIQ = 30-6 = 180 вт.
ГЛАВА XII СЛОЖНЫЕ ЦЕПИ ПОСТОЯННОГО ТОКА § 59. ВТОРОЙ ЗАКОН КИРХГОФА Сложной цепью .постоянного тока называется такая электри- ческая цепь, которая состоит из совокупности замкнутых на себя контуров, электрически связанных друг с другом. На участках каждого отдельного замкнутого контура сложной цепи в общем случае протекают разные по величине токи и имеются те или иные электродвижущие силы. Рис. 87. Замкнутый контур abcda сложной цепи На рис. 87 показан замкнутый контур abcda, выделенный из некоторой сложной цепи постоянного тока. Этот контур состоит из четырех участков (ab, be, cd и da), в каждом из которых имеются сопротивления, а в некоторых из них — и электродви- жущие силы. Допустим, в данном контуре токи протекают в направлениях, отмеченных стрелками, т. е. ток Д течет от точки а к точке Ь, ток Д — от точки b к точке с и т. д. Полярность источников тока обозначена здесь, как и всегда, знаками плюс и минус, где знак плюс отмечает положительный полюс источника тока, а знак минус — отрицательный. «г U* 179
Проследим sa тем, как изменяется потенциал в данном кон- туре, если мы будем -обходить этот контур, начиная с точки а и кончая ею же. Будем совершать свой обход в направлении движения часовой стрелки, т. е. в направлении изогнутой стрел- ки, показанной внутри контура. Обходя участок ab, мы должны будем поочередно пройти по сопротивлению ri и источнику элек- трической энергии с электродвижущей силой Е± и внутренним сопротивлением Гоь Проходя по сопротивлению п, мы отметим падение потенциала в нем на величину Лгх, так как здесь обход совершается в направлении тока, а ток, как известно, течет в на- правлении падения потенциала. Далее, проходя через источник электрической энергии, мы отметим падение потенциала в его внутреннем сопротивлении rOi на величину Дгог и, кроме того, возрастание потенциала на величину, равную электродвижущей силе Ei, так как мы проходим внутри источника в направлении его действия, т. е. от минус-зажима к плюс-зажиму его. Следо- вательно, потенциал в точке b будет равен ?* = ?«- Vi - Vol + Ёь (А) где сра — потенциал точки а\ — потенциал точки 6. Далее, при переходе от узловой точки b к узловой точке с на участке Ьс отметим падение потенциала в сопротивлении Г2' на величину /2Г2, падение потенциала на внутреннем сопротив- лении Го2 источника на величину /2Го2 и, наконец, падение потен- циала внутри источника на величину, равную его электродвижу- щей силе £2, так как здесь мы проходим внутри источника от плюс-зажима к минус-зажиму его, т. е. навстречу действию его электродвижущей силы £2. Следовательно, потенциал в точке с будет равен <fc= ЛГ2 ЛГ02 ^2‘ (Б) При переходе от узловой точки с к узловой точке d отметим падение потенциала в сопротивлении Гз на величину /з^з, так как здесь мы идем через сопротивленце г3 в направлении тока /3, т. е. в направлении падения потенциала. Следовательно, потен- циал в точке d будет равен Vs- (В) И, наконец, при переходе от узловой точки d к исходной узловой точке а отметим повышение потенциала в сопротивле- нии г4 на величину Л^4, так как здесь мы идем навстречу току Z4, т. е. в направлении возрастания потенциала. Следовательно, потенциал в точке а будет равен = + (Г) 180
Сложив уравнения (А), (Б), (В) и (Г), получим: = Та — V1 — ЛГ01 +" Е\ + + Ъ> ~ ^2 — 4Л)2 — Е2 + — 73Г3 + + Л>4, или, после несложных преобразований: ^*1 Е2 = 1 + Л^о1 “1“ Д^о2 4" До- полученное нами уравнение выражает математически связь между электродвижущими силами и падением напряжения на отдельных участках замкнутого контура (на отдельных сопро- тивлениях) . Если токи и электродвижущие силы, совпадающие с направ- лением обхода по контуру, считать положительными, а направ- ленные навстречу обходу — отрицательными, то последнее урав- нение можно записать так: Е\ + (—£*2) = ЛГ1 + ЛГ01 + ДГ2 + ДГ02 + + (—ЛГ4) или в сокращенной записи S£ = EZr. *(92) Полученное нами уравнение является математическим выра- жением одного из важнейших законов электротехники, называе- мого вторым законом Кирхгофа. Второй закон Кирхгофа может быть сформулирован так: алгебраическая сумма электродвижущих сил; имеющихся в зам- кнутом контуре, равна алгебраической сумме падения напряже- ния во всех сопротивлениях, входящих в этот контур. Второй заксй - Кирхгофа справедлив для любого замкнутого контура и, в частности, для контура, не содержащего в себе электродвижущих сил. В этом случае уравнение согласно вто- рому закону Кирхгофа запишем так: LZr = 0, т. е. алгебраическая сумма падения напряжения в данном замк- нутом контуре равна нулю. § 60. ПРИМЕНЕНИЕ ЗАКОНОВ КИРХГОФА К РАСЧЕТУ СЛОЖНЫХ ЦЕПЕЙ Рассчитать сложную цепь — это значит определить режим в этой цепи, т. е. по заданным электродвижущим силам и сопро- тивлениям ее отыскать величины и направления токов в отдель- ных участках цепи. Может быть поставлена и обратная задача: по заданным одному — двум токам и сопротивлениям сложной цепи определить некоторые электродвижущие силы и падения напряжения в цепи и т. д. 181
В настоящем параграфе мы рассмотрим применение метода уравнений Кирхгофа к расчету сложной цепи. Допустим, что задана сложная цепь (рис. 88), состоящая из двух источников электрической энергии, обладающих электро- движущими силами и Е2 и соответственно внутренними со- противлениями i\ и г2. К этим источникам подключен по- требитель» обладающий со- противлением г3. Требуется определить режим в данцой цепи, т. е. найти величины и направления токов во всех ее-участках. Для решения данной за- дачи применим метод урав- нений Кирхгофа, т. е. уравнений, составленных со- гласно первому и второму законам Кирхгофа. Для этой уравнений, сколько в данной цели необходимо составить столько сложной цепи имеется неизвестных токов. Так как неизвестных токов в данной цепи мы имеем три: ток h в участке цепи с ис- точником электрической энергии, обладающим электродвижущей силой Elf ток /2 в участке цепи с источником, обладающим элек- тродвижущей силой £2, и ток /з в потребителе с сопротивле- нием Гз,— то необходимо составить систему из трех уравнений. Вообще для любой сложной схемы и, в частности, для заданной можно составить число уравнений больше числа неизвестных то- ков. Однако некоторые из этих уравнений, как правило, не будут независимыми, т. е. они будут следствием остальных уравнений. Для того чтобы все уравнения были независимыми, рекомен- дуется при их составлении придерживаться следующих правил: 1. Если сложная цепь имеет п узловых точек, то можно со- ставить согласно первому закону Кирхгофа (п—1) независи- мых уравнений, так как уравнение для остающейся n-й узловой точки уже будет следствием первых (п— 1) уравнений, т. е. оно не будет независимым. 2. Недостающее число уравнений составляется согласно вто- рому закону Кирхгофа. Чтобы все этй уравнения были незави- симыми, т. е. каждое из них не было следствием других, необ- ходимо выбирать такие контуры для составления уравнений, чтобы каждый из них отличался от других по меньшей мере одним новым участком цепи (одной ветвью). При составлении уравнений согласно законам Кирхгофа не- обходимо знать направления токов. Но направления этих токов, а также и их величины нам неизвестны. Однако это затруднение отпадает, если принять во внимание, что ток является 182
алгебраической величиной, т. е. если ток одного на- правления на данном участке цепи положителен, то ток противо- положного направления на этом участке отрицателен. Следовательно, прежде чем приступить к решению данной задачи, необходимо задаться положительными токами на всех участках сложной цепи, указав на схеме направления положи- тельных токов стрелками, например пунктирными, как это пока- зано на схеме (рис. 88). Тогда, если в результате решения дан- ной задачи некоторый ток Л окажется положительной величиной (А > 0), то это значит, что он течет в ту сторону, куда показы- вает пунктирная стрелка. Если же, наоборот, ток А окажется отрицательной величиной (А<0), значит, он течет в противо- положном направлении. Составляя уравнения согласно второму закону Кирхгофа, мы должны иметь дело с электродвижущими силами и падением напряжения на отдельных участках цепи как величинами алгеб- раическими. Когда же электродвижущие силы и падение напря- жения считать положительными и когда отрицательными? Прежде всего алгебраический знак падения напряжения на любом участке цепи зависит от положительного направления тока на этом участке. Если, обходя по контуру, мы будем проходить по данному участку цепи в на- правлении, совпадающем с положительным направлением тока (в направлении пунктирной стрелки), то падение напряжения на этом участке будем принимать за положительную величину. Наобо- рот, мы его будем принимать за отрицательную величину, если будем проходить по данному участку цепи в направлении, противополож- ном направлению положительного тока (на- встречу пунктирной стрелки). Что касается знака электродвижу- щей силы, то он зависит от того, в каком направлении мы будем при обходе по данному контуру проходить через источник электрической энергии. Если при обходе по контуру мы будем входить в источник электрической энергии со стороны его отрицательного по- люса и выходить со стороны положительного полюса, то его электродвижущую силу будем считать положительной. Наоборот, если при о б- ходе, по контуру мы будем в х о д и т ь в и с т о ч н и к электродвижущей энергии со стороны его положительного полюса, а выходить со сто- роны отрицательного, то его электродвижу- щую силу будем считать отрицательной. В заключение отметим, что направление обхода по контуру можно выбирать произвольно. Например, по одному из контуров можно совершать обход в направлении вращения часовой стрел- ки, а по другому — в противоположном направлении. В каком бы 183
направлении мы ни обходили контур, по движению часовой стрелки или против него, в любом случае получим тождественные уравнения. Условившись относительно того, когда токи, падения напря- жений и электродвижущие силы считать положительными и когда отрицательными, можно приступить к составлению урав- нений согласно законам Кирхгофа. В заданной схеме сложной цепи (рис. 88) имеются две узло- вые точки, Ь и е. Следовательно, мы можем составить только одно уравнение согласно первому закону Кирхгофа, например, для узловой точки Ь. Остальные же два уравнения необходимо составить согласно второму закону' Кирхгофа. Для этой цели выбираем два контура, 'dbefa и bcdeb. Они дадут возможность составить два независимых уравнения, так как эти контуры от- личаются друг от друга некоторыми своими участками: напри- мер, в контуре abefa имеется сопротивление п, а в контуре bcdeb его нет. Задавшись положительными направлениями токов, для узло- вой точки b составим уравнение согласно первому закону Кирх- гофа: /1'+/2=/8. (I) Обходя контур abefa в направлении движения часовой стрел- ки, составим уравнение согласно второму закону Кирхгофа: + (II) Обходя контур bcdeb в направлении против движения часо- вой стрелки, составим уравнение согласно второму закону Кирх- гофа: £2 = 12г2 4- I3r3. (III) Итак, мы имеем систему из трех уравнений первой степени с тремя неизвестными (тремя токами: Л, Z2 и Z3), которую мож- но решать приемами элементарной алгебры. Решив эти уравнения, т. е. выяснив величины и направления реальных (истинных) токов в схеме, обозначим эти направления стрелками (в нашем примере сплошными в отличие от пунктир- ных, которыми были обозначены положительные направления токов). Проверить правильность решения задачи можно путем под- становки полученных значений токов Л, /2 и Z3 в уравнения, со- ставленные согласно законам Кирхгофа. Если полученные реше- ния удовлетворяют этим уравнениям, то задача решена пра- вильно. Пример 59. Задана сложная электрическая цепь (рис. 88), в которой один источник электрической энергии имеет электродвижущую силу Е\ = 24 в, а другой £2=18 в. В участке цепи с электродвижущей силой Е\ имеется 184
Сопротивление ri = 4 ом, а в участках цепи с электродвижущей силой £2 сопротивление г2 = 6 ом. Требуется определить режим в этой цепи для трех вариантов: 1) для случая, когда потребитель имеет сопротивление г3 = 4 ом; 2) для г3 = 12 ом и 3) для г3 = 16 ом. Решение. Так как нам необходимо решить данную задачу в несколь- ких вариантах, то целесообразно ее решить вначале в общем виде. Составляем систему трех уравнений для определения трех неизвестных токов: /1, /2 и /3, применяя для этой цели законы Кирхгофа. Для узловой точки b Л 4- /2 = Л- (I) Для контура abefa Ап 4-Ла (П) Для контура bcdeb £2 = Цг2 + /3г3. (III) Подставив выражение для тока /3 из уравнения (I) в уравнения (II) и (III), получим следующие два уравнения: А = АЛ 4- (Л 4- 4) гй = A (г, + г3) + /2г3; (И') Е2 = Лг2 + (Л 4- /2) гз = //з 4- Л (г2 4- г3). (ИГ) Умножим левую и правую части уравнения (IT) на (r2-f-r3), а левую и правую части уравнения (IIIх) — на г3 и после этого вычтем из первого уравнения второе: _ ^1(''2 4-г3)=А(Л 4-Гз) 4-Г8) + Л/з ('2 4-Г3) __________________ЕЛ = ^1 + 12(г2 + г3)г3______________________ Ei (г2 4- г3) — Е2г3 = Л (Г1 4- г8) (г2 + гз) — Ai или Е1 (г2 4- г3) — Е2г3 = Ц (rxr2 + г2г3 4- r3rY 4- 4 — ф, откуда находим, что Г __ £j (^*2 4~ Г3) £2^*3 1 Г1Г2 + Г2Г3 + Л/1 Аналогично этому, умножив уравнение (IF) на г3, а уравнение на (fl + Гз) и вычтя из первого второе, получим уравнение, из найдем, что т = Е2 (П 4- Гз) — £^з 2 Г1Г2 + Г/3 + Г3ГХ И наконец, подставив выражения для токов Л и /2 в уравнение (I), найдем величину тока 13. = Ei (r2 -F г3) — £2<з + £2(п + г3) — £1ГЭ г\г2 + ГгГз + г3гх ’ (IV) (III') - которого Л — Л 4-Л 185
или J — + ^2^1 3 V2 + Vs 4- Vi ’ (VI) Для первого варианта задачи Ei = 24 в, Е2 = 18 в, п = 4 ом, г2 = 6 ом и /'з = 4 ом. Подставив в формулы (IV), (V) и (VI) значения данных величин, по- лучим: т (М~ ^з) ^2гз 24 (6 4- 4) 18» 4 q г» л г п. 1 V2"bV3 4-V*i 4-6+ 6-4 4-4-4 ’ ’ т _ ^(П + Гэ)-^ _ 1% (4 4- 4) — 24-4 - 2 гхг2 4- г2г3 4- г3г\ 4-6 4- 6-4 4- 4-4 ’ ’ т ___ Е^2 + Е2/\ __ 24-6 4- 18-4 ___n «yr 3 V2 + V3 + Vi 4-6 4-6-4 4-4-4 ’ Все токи получились положительными, а это значит, что они текут в на- правлениях, указанных пунктирными стрелками. На основании этого можно сделать вывод, что оба источника электрической энергии работают в данном случае <в режиме генераторов, питая током /3 = 3,375 а потребитель с со- противлением гз- При этом на долю первого источника приходится ток /1 = 2,625 а, а на долю второго — ток /2 = 0,75 а, поскольку Ц + /2 = 2,625 + 0,75 = 3,375 а = 73Г Правильность решения задачи можно проверить путем подстановки по- лученных значений токов в исходные уравнения, например в уравнение (II): = 1\Г\ + /3г3, или, иначе, 24 = 2,625 • 4 4“ 3,375 • 4 = 24, т. е. задача решена правильно. Для второго варианта задачи Е\ = 24 в, Е2 =18 в, Г\ = 4 ом, г2 = 6 ом и г3 = 12 ом. Применяя формулы (IV), (V) и (VI), получим: г _ (G + гз) — Е2г3 __ 24 (6 4- 12) — 18-12 « дп 1 V2 4" V*3 4“ Vi 4-6 4- 6-12 4- 12-4 ’ ’ . _ E2(r14-r3)-E1r3 _ 18 (4 -Ь 12) — 24-12 _ 2~ V2 4-Vs 4-Vi 4-6 4- 6-12 4- 12-4 U’ т __ E^r2 + Е2г\ ____ 24-6 4- 18-4 _____1 4Q л '3~V24-V3 +Vi~ 4-6 4- 6-12 4- 12-4 . На основании полученных значений токов можно сделать следующий вы- вод. Первый источник электрической энергии работает в режиме генератора, посылая ток потребителю с сопротивлением г3=12 ом. Второй источник на- ходится в режиме холостого хода, т. е. его ток 12 = 0. Это значит, что он, невзирая на то, что подключен к цепи, не отдает своей энергии потребителю и не получает ее от другого источника. Для третьего варианта задачи £1 = 24 в, Е2= 18 в, г\ = 4 ом, г2 = 6 ом и Гз = 16 ом. Применив формулы (IV), (V) и (VI), получим: т __ Ej (г2 4- ^з) — Е2г3 _24 (6 4- 16) — 18-16 _1 Q л. 1 V2 + V3 + V1 4-6 4- 6-16 4" 16-4 ’ ’ 186
/ = ^2 (Л + r3) — gtr3 18(4+16)-24» 16 п 2 nr2 + r2r3 + г3Г1 4-6+ 6-16+16-4 v,io а, 7 ___ ^1^2 + ^2^1 24»6 + 18-4 __117/7 8 V2 4“ "Ь /*3/*1 4«6 + 6’16+16.4 ’ На основании полученных решений для токов Л, /2 и 1з можно сделать следующий вывод. Первый источник электрической энергии работает в ре- жиме генератора, посылая ток во второй источник и в потребитель с со- противлением г3. Второй источник имеет ток отрицательный (/2 = — 0,13 а). Это значит, что ток поступает в него со стороны его положительного по- люса и этот источник работает в режиме потребителя — аккумулятор заря- жается со стороны первого источника. В этом случае /]+/2 = /э’ или, иначе, 1,3+ (—0,13) = 1,17 =/3. Примечание, При решении задач по расчету сложных цепей по- стоянного тока методом применения законов Кирхгофа не обязательно ре- шать задачи в общем виде до конца. Последнее необходимо в том случае. Рис. 89. Схема сложной цепи когда из полученного решения мы желаем сделать какие-нибудь общие выводы. Например, из формулы (IV) мы можем сделать следующие полез- ные выводы. Если соотношение параметров цепи таково, что Е\ (г2 + г3) > > Е2г3, то ток /1 > 0. Следовательно, источник электрической энергии с элек- тродвижущей силой Е\ в этом случае работает в режиме генератора. Если же Е\ (г2 + 'з) < E2r3, то ток /1 < 0. Это значит, что источник работает в режиме потребителя. И, наконец, если Ei (г2 + г3) = Е2г3, то ток /1 = 0, т. е. источник находится в режиме холостого хода. Аналогичный анализ можно провести на основании формулы (V), даю- щей возможность определить ток /2 второго источника электрической энергии. Если мы не задаемся целью получить решение задачи в общем виде, то бывает полезно сразу же подставлять в исходные уравнения Кирхгофа вместо букв численные значения и этим несколько облегчить решение урав- нений. Пример 60. Задана сложная цепь (рис. 89), в которой один источник электрической энергии имеет электродвижущую силу Ei — 120 в, а другой £*2=110 в. Сопротивления в цепи равны: = 1 ом, r2 = 1 ом, г3=10 ом, г4 = 2 ом и Гб = 2 ом. Требуется определить величины и направления токов во всех участках заданной цепи. 187
Решение. В данной задаче имеется пять неизвестных токов: Л, 7г, /з> и 7б, положительные направления которых отмечены на схеме (рис. 89) пунктирными стрелками. Для определения этих токов необходимо составить пять независимых уравнений согласно первому и второму законам Кирхгофа. Для составления уравнений по первому закону Кирхгофа используем узловые точки b и с схемы, а для составления остальных трех уравнений используем кон- туры abgha, cdefc и bcfgb. Для узловой точки b 11=!з+16. (I) Для узловой точки с Ц = + (И) Для контура abgha = M +М> или 120=4 + 104. (III) Для контура cdefc Ег=1гГг + Ц1\, или 110 = 4 + 24. (IV) Для контура bcfgb Цг3+Цг4=13Г3, или 2/5 + 2/4 = 10/8; после сокращения на два получаем 4 + /4 = 5/3. (V) Подставим в уравнение (III) вместо 1\ его выражение из уравнения (I): 120=4+4 + 104, или 4 + 114 = 120. (ПГ) Подставим в уравнения (IV) и (V) вместо /4 его выражение из урав- нения (II): 110=4 + 24 + 24, или 34+24 = 110, (IV') 188
и 2Z5+2Z2 + 2Z5 = 10Z3, ИЛИ 2Z2 + 4Z5 = 10Z3, или же Z2 + 2Z5 = 5Z3. (V) Из уравнения (V') получаем Z2 = 5Z3-2Z5. (V") Подставляем в уравнение (IV') вместо 72 его выражение из уравне- ния (V"): 3(5Z3-2Z5)+2Z5 = 110, ИЛИ 15ZS- 6Z5 + 2Z3 = 110, или, иначе, 15Z3 —4Z3 = ПО. (IV") Решим совместно уравнения (ПГ) и (IV"). Для этого помножим урав- нение (ПГ) на 4 и сложим его с уравнением (IV"): 4/5 + 44/3 = 480 + — 4ZS + 15Z3 = 110 59Z3 = 590’ ИЛИ , 590 1 п Z3 — -jg- — 10 а. Из уравнения (IV") находим Л: г 1573 — НО 15-10-110 /5 =-----J----=-------------= 10 а. Из уравнения (I) определяем ток 1\*. /1=/3 + /б = 10 + 10 = 20 а. Из уравнения (V") находим ток 72: /2 = 5/3 —2Z5 = 5-10 —2-10 = 30 а. Из уравнения (II) находим ток Ц- Ц 4- Z5 = 30 + 10 = 40 а. 189
Все токи получились положительными, а поэтому они текут в цепи в на- правлении пунктирных стрелок, показывающих положительное направление тока. Правильность решения задачи можно проверить путем подстановки зна- чений полученных токов в исходные уравнения, например в уравнение (III): 120 = 4+10/3; 120 = 20+ 10-10 = 120 или в уравнение (V): 4 + 4 = 5/3; 10 + 40 = 5-10 = 50. Следовательно, задача решена правильно. § 61. МЕТОД контурных токов При применении законов Кирхгофа к расчету сложных цепей постоянного тока каждый раз приходится составлять столько уравнений, сколько неизвестных. Если этих неизвестных относи- тельно много, что весьма часто бывает в практике, то примене- ние метода уравнений Кирхгофа становится затруднительным из-за необходимости решать систему с большим числом уравне- ний. В этих случаях обычно используют иные методы, в частно- сти метод контурных токов, дающий возможность зна- чительно сократить число уравнений для определения искомых токов. Конкретно, этот метод дает возможность избавиться от необходимости составления уравнений по первому закону Кирх- гофа и ограничиться решением системы уравнений, составлен- ных только по второму закону Кирхгофа. 190
Допустим, что задана сложная цепь (рис. 90), в которой из- вестны все электродвижущие силы и сопротивления и требуется определить величины токов во всех участках этой цепи. Если бы мы эту задачу стали решать, применяя метод урав- нений Кирхгофа, то нам понадобилось бы составить систему из шести уравнений с шестью неизвестными токами (А, /2, ..., /в). Так как в заданной схеме имеются четыре узловые точки, то мы составили бы три уравнения по первому закону Кирхгофа, а остальные три уравнения — по второму закону. Метод контур- ных токов дает возможность решить эту задачу, составив систе- му из трех уравнений по второму закону Кирхгофа. Допустим, что в контурах abhga, ehgfe и cbhedc протекают некоторые условные контурные токи. Например, во всех участ- ках контура abfcga проходит контурный ток /р во всех участ- ках контура ehgfe — ток /п и во всех участках контура cbhedc— ток 1т. Контурные токи являются величинами алге- браическими, а поэтому их положительные направления, условно задаваемые нами, обычно отмечаются кривыми стрелка- ми в контурах (рис. 90):' В нашем случае положительное на- правление контурного тока Zj совпадает с направлением враще- ния часовой стрелки, а контурный ток /п направлен против ча- совой стрелки, так же как и контурный ток /ш. Если будут определены контурные токи во всех контурах за- данной сложной цепи, то уже не представит затруднений вычис- лить реальные токи во всех участках этой цепи. В тех участках цепи, где одновременно имеется несколько контурных токов, реальный ток в них равен алгебраической сум- ме контурных токов. Например, в участке цепи с сопротив- лением г6 (рис. 90) имеются два контурных тока/j и /п, причем их положительные направления совпадают. Следовательно, реаль- ный ток /6, протекающий по сопротивлению Гб, равен алгебраи- ческой сумме контурных токов Zj и Zir 4 = А + А? Наоборот, в сопротивлении г5 контурные токи /п и /ш имеют направления противоположные. Следовательно, реальный ток, протекающий по сопротивлению г5, будет равен h = Ai Air Если в результате решения задачи ток /5 окажется положи- тельной величиной, то он имеет направление, совпадающее с по- ложительным направлением контурного тока /ц, а если он полу- чится отрицательным, то он имеет направление, совпадающее с положительным направлением контурного тока /ш. 191
Если в данном участке цепи имеется только один контурный ток, то реальный ток в этом участке цепи равен по величине данному контурному току и совпадает с ним по направлению. Для определения контурных токов /,, /п и /ш составим три уравнения, аналогичные уравнениям второго закона Кирхгофа. Однако в отличие от последних в этих уравнениях падения на- пряжения на участках цепи будут приниматься равными алге- браической сумме падений напряжений от каждого из контур- ных токов. Например, в сопротивлении г4 (рис. 90) падение на- пряжения будет равно + А1/4, падение напряжения в сопротивлении г5 будет равно ^5 = Л]Г5 ~ Апг5 и т. д. Если мы будем обходить контур abhga в направлении, совпадающем с положительным направлением контурного тока Zr, то уравнение второго закона Кирхгофа в соответствии с вы- шеизложенным запишем в следующем виде: = 4~ 4* 4~ Л1/4 4“ или =Л(Г1 4" г4 4-/*б) 4-/пг6 4-/шг4- (I) Обходя контур ehgfe в направлении, совпадающем с поло- жительным направлением контурного тока /п, запишем уравне- ние второго закона Кирхгофа в следующем виде: £*2 “ ^11^2 4“ 4“ или Е2 = //б + Л] (г2 + + г6) — Лпг5- (П) И, наконец, обходя контур cbhedc в направлении, совпадаю- щем с положительным направлением контурного тока /ш, запи- шем уравнение второго закона Кирхгофа в следующем виде: £*3 = /ц/з 4" ^111^4 4“ ^П1Г5 4” /|^4 Л/б» или Ez = 7Л — Vs + Ап (гз + (III) Систему уравнений (I), (II) и (III) решаем относительно неизвестных контурных токов приемами, применяемыми в эле- ментарной алгебре. 192
Найдя величины контурных токов /р /п и /ш, Определим ре- альные токи Л, /2, /з, Л и /5, исходя из следующих соотношений: А = А» Л = Ар А == Ап*» А = А + Ап; А = Ai Ап и А = А -Ь А* Определив величины и направления реальных токов, обозна- чим прямыми короткими стрелками их направления на всех уча- стках заданной сложной цепи, Правильность решения задачи можно проверить, написав обычное уравнение второго закона Кирхгофа с учетом реальных токов для какого-либо контура. Например, для контура abhga оно принимает следующий вид: Если в результате подстановки в это уравнение численных значений сопротивлений и, г4, г6, токов Л, /4 и 1$ и электродви^ жущей силы Е\ получим тождество, то задача решена пра^- ВИЛЬНО. Пример 61. Определить токи во всех участках сложной цепи (рис. 90)* если известно, что Е{ = 130 в, Е2 = 40 в, Е3 = 100 в, п = 1 ом, г2 = 5 ом, г3 = 2 ом, = 4 ом, г5 = 10 ом и г6 — 5 ом. Решение. Задаемся положительным!! направлениями контурных то- ков 7Р /п и /ш так, как это показано изогнутыми стрелками на схеме (рис. 90). Для контура abhga El = А (п + >4 + Л>) + 4 + /шг4, или 130 = 10/I + 5/n + 4/lu. (I) Для контура ehgfe Е2 = + Al (г2 + г5 + г6) — /шгб, или 40 = 5/j + 20/п— 10/ш> сокращая на 5, получим: z 8 = ZI + 4/„-2/in. (II) Для контура cbhedc Е3 = — /н + /ш (гз + + г6), 13-1377 193
или 100 = 4/j—10/п+16/ш, или же 2Zj — 5/п + 8/ш = 50. (III) Из уравнения (II) находим /1 = 8-4/„ + 2/ш. (IF) Подставляем полученное выражение для 7j в уравнение (I): 10 (8 - 4/„ + 2/ш) + 5/„ + 4/ш = 130, ИЛИ 80 — 40/п + 207ш + 5/п + 47Ц1 = 130, или, иначе, —35/ц + 24/ш = 50. (Г) Подставляем выражение для 7j в уравнение (III): 2 (8 - 4/п + 2/ш) - 5/и + 8/ш = 50, ИЛИ 16 - 8/„ 4- 471Ц — 57п + 8/ш = 50, или, иначе, -13/„ + 12/ш = 34. (ПГ) Умножим левую и правую части уравнения (ПГ) на 2 и вычтем из него уравнение (Г): —26/п + 24/ш = 68 357п + 24/ш = 50 9Гп =18 Отсюда находим, что /ц = 4=2а. Из уравнения (ПГ) находим: 34 + 13Л. 34 + 13-2 „ Au 12 = 12 - 5 Из уравнения (1Г) получаем. ZI = 8 — 4/u + 2Zn) = 8 —4-2 + 2-5 = 10 а. 194
Определив контурные токи, найдем реальные токи в заданной цепи: Л =/, = 10 а; л = 10 + 5 = 15 а; А = Ai = 2 dj /5= Ап Ai= 2 = 3 сг, А = Ап = 5 а> /6 = А + Ai =10 + 2 =12 Направление тока Л совпадает с положительным направлением контур, ного тока /1П. Проверка правильности решения: Для контура abhga Ei = hr\+ Цг4 + 1бг6, или 130 = 10-1 + 15-4 + 12-5 = 130, т. е. решение, полученное нами, правильно.
ГЛАВА XIII ТЕПЛОВОЕ ДЕЙСТВИЕ ЭЛЕКТРИЧЕСКОГО ТОКА § 62. ТЕРМИЧЕСКИЙ ЭКВИВАЛЕНТ Электрический ток всегда сопровождается выделением тепла в проводнике, по которому он протекает. Чем больше ток в проводнике, тем быстрее движутся элек- трические заряды в нем, тем чаще они «сталкиваются» с ато- мами, встречающимися на пути их движения. В результате этих «столкновений» заряды отдают часть своей электрической энер- гии атомам, заставляя их более интенсивно колебаться. А это значит, что электрическая энергия, отдаваемая зарядами ато- мам, превращается в тепловую энергию. Получение тепла за счет электрической энергии является ко- нечной целью работы некоторых потребителей электрической энергии, например электрических нагревательных приборов» Но в большинстве случаев неизбежное превращение части электри- ческой энергии в теплоту представляет собой непроизводительный расход энергии, снижающий к. п. д. установки (например, выделе- ние тепла в соединительных проводах, в обмотках машины и т. д.). Для того чтобы подсчитать, какое количество тепла выделяет ток в проводнике, надо установить эквивалентное соотношение между электрической и тепловой энергией: 1 джоуль = кгм. Но, как известно, 1 кгм эквивалентен большой калории 1000 г 1 тепла, или малым калориям тепла. Следовательно, 1 джоуль 1000 л эквивалентен g81.j27 = 0,24 малой калории тепла: 1 дж^0,24 м. кал. 196
Величину С -0,24 малая калория джоуль (93) называют термическим эквивалентом. § 63. ЗАКОН ЛЕНЦА-ДЖОУЛЯ Изучая тепловое действие тока, русский ученый Э. X. Ленц открыл закон, устанавливающий зависимость между количе- ством тепла, выделяемого током, и величиной этого тока. Этот закон вошел в электротехнику под названием закона Ленца — Джоуля, так как одновременно с Ленцем он был открыт и ан- глийским физиком Джоулем. Э. X. Ленц известен своими фунда- ментальными трудами в области теоретической электротехники. Кроме вышеуказанного закона, он сформулировал так называе- мое «правило Ленца», с которым мы подробно ознакомимся впо- следствии, а также обосновал принцип обратимости электриче- ских машин и ряд других теоретических положений из области электромагнетизма. Научная работа Э. X. Ленца протекала на заре развития электротехники, и его открытия в области теорети- ческой электротехники исключительно важны. Рассмотрим, в чем же заключается сущность закона Ленца — Джоуля и каково его практическое значение. Как известно, работа электрического тока может быть под- считана по формуле A = I2Rt, где А — работа электрического тока в джоулях; / — ток в амперах; — сопротивление в омах; t — время в секундах. Один джоуль эквивалентен 0,24 м. кал. Следовательно, для того чтобы подсчитать, какое количество тепла выделится в про- воднике за счет электрической энергии, надо число затраченных джоулей помножить на термический эквивалент: q __ р малая калория ’ джоуль • В соответствии с этим Q = СД = 0,24/2/?/, (94) где Q — количество тепла в малых калориях. Формула (94) является математическим выражением закона Ленца — Джоуля, который можно сформулировать следующим образом: количество тепла, выделяемого током в проводнике при 197
неизменном сопротивлении R последнего, прямо пропорцио- нально квадрату величины тока и времени, в течение которого этот ток протекал по проводнику. Пример 62. Определить, какое количество тепла выделилось в провод- нике, обладающем сопротивлением # = 25 ом, в течение /=15 минут, если ток в нем был равен 1 = 5 а. Решение. По формуле (94) находим Q = 0,24/2/?/= 0,24-52-25-15-60 = 135000 м. кал. § 64. ПЛАВКИЕ ПРЕДОХРАНИТЕЛИ При значительной перегрузке электрических приборов токами или при коротком замыкании их выделяется такое большое ко- личество тепловой энергии, что возникает угроза сохранности установки и даже опасность пожара. Для предохранения проводов и аппаратов от чрезмерно боль- ших токов применяют ряд предохранительных приспособлений, Рис. 91. Пробочный плавкий предохранитель из которых наибольшей простотой конструкции отличаются плавкие предохранители. Принцип действия плавкого предохранителя заключается в следующем. Последовательно с потребителем электрической энергии вклю- чается легкоплавкая проволока, которая при.определенном токе перегорает и, разрывая цепь, отключает потребитель от генера- тора электрической энергии. Обычно на каждом плавком предохранителе указывается но- минальный ток /ном, т. е. предельный допустимый рабочий ток, который продолжительное время может протекать через плавкую вставку предохранителя. Если ток превышает номинальный на 150—5—200%, то плавкая вставка должна расплавиться. Согласно стандартным нормам плавкие вставки предохрани- телей изготовляются на номинальные токи: 4, 6, 10, 15, 20, 25 а и т. д. Запрещается использовать вместо плавких предохранителей суррогаты, например металлическую проволоку и т. п., так как 19S
При таком «Предохранителе» может возникнуть аварийный ре- жим в цепи* и вспыхнуть пожар. Предохранители, используемые в электротехнике, бывают раз- личных конструкций. Рассмотрим некоторые из наиболее употре- бительных типов плавких предохранителей. 1. На рис. 91 показан широко применяемый тип пробочного предохранителя. Пробка ввинчивается в соответствующий патрон и этим замыкает цепь через легкоплавкую нить, смонтированную внутри пробки. На каждой пробке указана та максимальная ве- личина тока, на которую она рассчитана. 2. На рис. 92 показан широко применяемый в технике связи трубочный предохранитель, рассчитанный на относительно малые токи. Патрон в предохранителе состоит из стеклянного баллона /, в который помещена тонкая проволочка 2 из легкоплавкого материала, припаянная концами к металлическим колпачкам 5, посаженным по концам трубочки. При прохождении тока свыше, например, 0,25 а проволочка в патроне перегорает. Патрон пре- дохранителя вставляется в пружинные стойки, смонтированные на цоколе из изолирующего материала. Рис. 92. Трубочный плав- кий предохранитель: 1 — стеклянный баллон; 2 — про- волочка из легкоплавкого мате- риала; 3 — металлические колпачки Рис. 93. Схема термической катушки: I — стержень; 2 — катушка; 3 — втулка; 4 основа- ние; 5 — штифт; 6 — головка штифта Предохранитель устанавливается на каждый линейный про- вод.- связи при вводе последнего в помещение телеграфной станции. 3. В телефонных установках для предохранения аппаратуры от случайного попадания больших токов в линию применяются термические катушки (рис. 93). На латунный стержень 1 надета катушка 2, которая одним концом прикреплена к основанию 4 катушки, а другим — к стержню. Стержень ] одним концом заделан в эбонитовую втулку 3. Металлический штифт 5, имею- щий на одном конце головку 6, другим концом закреплен в стержне 1 легкоплавким материалом. Если в цепи будут токи более 0,5 а, то легкоплавкий состав под действием тепла, выде- ляющегося в катушке 2, расплавится и штифт 5 при помощи пружины, воздействующей на головку 6 штифта, будет извлечен из стержня 1 и разорвет цепь. 199
§ 65. ЭЛЕКТРИЧЕСКИЕ ЛАМПЫ Рис. 94. Газона- полненная элек- трическая лампа Электрический ток, проходя по металлическим проводникам, выделяет в них тепло, повышая их температуру. Если нагреть проводник до белого каления, то он начнет испускать ослепи- тельно яркий поток световой энергии. На этом основана работа электрических ламп, применяемых для освещения. Приоритет изобретения электрической лампы накаливания принадлежит известному русскому изобретателю А. Н. Лодыгину. Первая лампа Лодыгина послужила прообразом всех современ- ных электрических ламп. На рис. 94 показана газонаполненная элек- трическая лампа. Она состоит из стеклянного баллона, в котором воздух заменен инертным газом (например, азотом или аргоном), не под- держивающим *горение. В баллоне лампы по- мещена металлическая нить, изготовленная из тугоплавких материалов, например вольфрама, осмия, тантала или их сплавов. Через цоколь лампы наружу выведены концы ламповой нити, из которых один припаян к пяточке цо- коля, а другой — к его винтовой нарезке. При прохождении электрического тока че- рез нить лампы она накаливается и излучает световой поток. Чем больше величина тока в нити лампы, тем сильнее она накаливается и тем ярче светит. В современных газонаполнен- ных лампах температуру вольфрамовой нити доводят примерно до 3000° Ц (температура плавления вольфрама 3370° Ц). Коэффициент полезного действия газонаполненных ламп не превышает 5%, так как значительная часть электрической энер- гии в них превращается в тепло. Электрическая лампа является самым несовершенным с точки зрения к. п. д. прибором в ряду других потребителей электрической энергии. Наполнение баллона лампы инертным газом преследует цель создать в баллоне давление на нить и тем самым предотвратить сильное распыление нити лампы, приводящее к ее перегоранию. В настоящее время широкое распространение получили «лампы дневного света». Это стеклянные трубки, внутренняя по- верхность которых покрыта особым светосоставом. Трубка .на- полнена газом, например парами ртути, под небольшим давле- нием. Газ при прохождении через него ионного электрического тока излучает ультрафиолетовый свет. Ультрафиолетовые лучи, попадая на светосостав, нанесенный на внутреннюю поверхность трубки, заставляют его светиться. При этом свет, излучаемый лампой, мало чем отличается, от обычного дневного естествен- ного света. «Лампы дневного света» весьма экономичны в экс- плуатации в силу малых тепловых потерь энергии. Они дают 200
приятный для глаза свет, не утомляющий зрения. Следует отме- тить, что в деле создания «ламп дневного света» плодотворную работу проделали советские ученые и инженеры под руковод- ством академика С. И. Вавилова (1891—1951), автора ряда научных трудов в области световых явлений. § 66. ЭЛЕКТРИЧЕСКАЯ ДУГА Если два угольных стержня (рис. 95) подключить к зажимам генератора постоянного тока и затем довести их до соприкоснове- ния друг с другом, то в образовавшейся замкнутой цепи возник- нет постоянный ток. Вследствие относительно большого переход- ного сопротивления контакта угольных электродов в нем выде- ляется значительное количество тепла, которое при соответствую- щей величине тока нагревает кон- цы угольных электродов до бе- лого каления. Если начать посте- пенно раздвигать угольные элек- троды, то электрический ток в це- пи не прекратится и тогда, когда .между ними образуется некото- рый зазор. Под влиянием слиш- ком высокой температуры концов угольных электродов между ними образуется раскаленный газ, об- ладающий ионной проводимостью, так что фактически цепь электри- ческого тока остается замкнутой. Рис. 95. Схема включения элек- трической дуги в электрическую цепь Раскаленный, ярко светящийся газ между угольными электро- дами перекинут с одногд электрода на другой в виде мостика, напоминающего дугу. 4 * Явление прохождения электрического тока в промежутке между двумя электродами, сопровождаемое очень высокой тем- пературой концов^ электродов и излучением мощного светового потока, называется электрической дугой. Явление электрической дуги было открыто в 1803 г. русским ученым В. В. Петровым. Физические процессы, наблюдающиеся в электрической дуге, в основном сводятся к следующему. С раскаленного добела катода, т. е. электрода, соединенного с отрицательным полюсом источника электрической энергии, вы- деляется мощный поток свободных электронов, которые под влиянием сил электрического поля мчатся с большйми скоро- стями по направлению к аноду — угольному электроду, соеди- ненному с положительным зажимом источника электрической энергии. Эти электроны, пролетая через раскаленный газ, стал- 201
киваются на пути' своего движения с молекулами газами выби- вают из них электроны, т. е. ионизируют» их. Электроны, достиг- шие анода, бомбардируют его, отдавая ему свою кинетическую энергию (энергию движения), и этим способствуют возникнове- нию на аноде очень высокой температуры. Появившиеся в рас- каленном газе положительные ионы под влиянием сил поля мчатся по направлению к катоду и, от- давая ему свою кинетическую энергию, поддерживают его высокую темпера- туру. Температура катода в электриче- ской дуге с угольными электродами в среднем достигает 3000° Ц, а темпе- ратура анода — 4000° Ц. При такой вы- сокой температуре уголь испаряется и на конце анода возникает нечто вроде кратера (впадины), излучающего осле- пительно яркий поток световых лучей. Электрическая дуга может существовать рактеристика электриче- ской дугц только в том случае, если катод имеет температуру^белого каления, так как только при этом условии он может излучать поток электронов. Раскаленные частицы угля, испаряющиеся с поверхности анода, переносятся на катод и ча- стично осаждаются на нем. В результате катод принимает не- сколько заостренную форму. По этим характерным очертаниям концов электродов легко отличить анод от катода (см. рис. 95). • С ростом тока, питающего электрическую дугу, усиливается ионизация, увеличивающая проводимость дуги. Поэтому падение напряжения на электрической дуге уменьшается с ростом тока. Следовательно, в электрической дуге закон Ома теряет свою силу, так как нет прямой пропорциональной зависимости между напряжением U на дуге и величиной тока /. На рис. 96 показан график зависимости между током / и на- пряжением U в электрической дуге, называемый характери- стикой дуги. Из графика мьг видим, что с ростом тока I напряжение U падает и при некоторой величине тока резко, скачком, уменьшается до определенной величины, а затем уже мало изменяется с ро- стом тока. При нормальных значениях тока (левая часть характеристики дуги) дуга горит нормально, устойчиво. Но при больших значениях тока, в особенности после скачкообразного падения напряжения, дуга горит неустойчиво, анод обгорает не- ровно, горение сопровождается шипением. В этом режиме элек- трическую дугу использовать нельзя. Чрезвычайно высокая температура анода электрической дуги сопровождается мощным излучением светового потока, который используется для освещения. Электрическая дуга успешно при- меняется в прожекторах, а также для электросварки металличе- ских деталей и проводов. 202
Работай над усовершенствованием приборов для рёгулирова* ния электрической дуги, русский ученый П. И. Яблочков изобрел в 1876 г. осветительный прибор, который в электротехнику во- шел под названием свечи Яблочкова. Эта свеча состояла из двух вертикально и параллельно расположенных ретортных углей, отделенных друг от друга слоем изолирующего вещества. Элек- трическая дуга1 зажигалась между концами палочек ретортного угля, и по мере сгорания их изолирующая прослойка между ними испарялась. Свеча Яблочкова в свое время сыграла исклю- чительную роль как первое практическое применение электриче- ского освещения в широком масштабе.
ГЛАВА XIV МАГНИТНОЕ ПОЛЕ * § 67. ПОНЯТИЕ О МАГНИТНОМ ПОЛЕ Всякая система электрических токов обладает магнитным полем. Магнитное поле представляет собой особый вид материи, че- рез который осуществляется физический процесс взаимодействия электрических токов — взаимное притяжение проводов с токами одного направления и отталкивание проводов с токами разного направления. Всякое изменение взаимного расположения проводов с то- ками в пространстве, а также изменение величины этих токов во времени связано с изменением их общего магнитного поля. Изменяемость магнитного поля в пространстве и во времени представляет собой своеобразную форму материального движе- ния в природе, при котором наблюдается переход магнитного поля в электрическое и соответственно этому преобразование энергии магнитного поля в энергию электрического поля. Если взаимодействие проводников с электрическими токами осуществляется через их общее магнитное поле, значит, это поле действует непосредственно на проводники с токами. Всякое магнитное поле, как некоторый вид материи, обладает энергией. Величина энергии магнитного поля зависит от вели- чины электрических токов, создающих это поле, от их конфигу- рации и от физических свойств той среды, где это поле суще- ствует. Если величина тока, создающего данное магнитное поле, ра- стет, то вместе с этим растет энергия и масса его магйитного поля. Приращение энергии магнитного поля в данном случае осуществляется за счет работы источника электрической энергии, создающего ток. Наоборот, если величина тока уменьшается, то магнитное поле также становится более слабым. Его энергия в этом случае исчезает, преобразуясь в другие виды энергии, например энергию электрического поля или энергию тепловую. О наличии магнитного поля в данной области пространства 204
можно судить по тем механическим силам, с которыми данное поле действует на проводники с токами, магнитные стрелки и т. д., помещенные в этом поле. Действие сил в магнитном поле можно обнаружить различ- ными способами. Например, если под прямолинейный проводник Рйс. 97. Магнитная стрелка Рис. 98. Магнитная стрелка устанавливаете^ перпендику- лярно к направлению тока с током поместить магнитную стрелку (рис. 97), могущую вра- щаться вокруг вертикальной оси, то она под влиянием сил маг- нитного поля будет устанавливаться перпендикулярно направле- нию тока (рис. 98). Если направление тока в проводе изменится, то магнитная стрелка под влиянием сил магнитного поля повернется на угол 180° вокруг своей вертикальной оси (см. рис. 98). Значит, на- правление сил магнитного поля зависит от направления электри- ческого тока в проводе. Если вертикально расположенный прямолинейный проводник с током про- пустить через плоский картон и на по- следний насыпать мелких железных опилок, то, постукивая слегка по кар- тону, можно заставить железные опил- ки расположиться по замкнутым кри- вым линиям вокруг проводника с током (рис. 99). Это значит,, что магнитные силы действуют на железо и что направ- ление их действия совпадает с напра- влениями кривых линий, охватываю- щих проводник с током. Эти линии яв- ляются концентрическими окружностями Рис. 99. Железные опилки вокруг прямолинейного провода с током распола- гаются по замкнутым кри- вым линиям с центрами на оси про- водника. Если поместить на картоне маленькие магнитные стрелочки на вертикальных осях, то они расположатся вокруг проводника с током по замкнутым концентрическим окружностям. 205
Известно, что магнитная стрелка, например стрелка компаса, в нормальных условиях устанавливается в направлении с юга на север, т. е. примерно в направлении географического меридиана. Тот конец магнитной стрелки, который поворачивается на север, является северным магнитным полюсом, а противо- положный ему конец — южным магнитным полюсом. Опыт показал, что в пространстве, окружающем земной шар, существует магнитное поле,, в котором направление действия магнитных сил примерно совпадает с направлением географиче- ского меридиана. Это магнитное поле называется полем зем- ного магнетизма, и создается оно электрическими токами. Земной шар окружен атмосферой, заряженной электричеством (положительными и отрицательными ионами). Эти атмосферные заряды, вращаясь вместе с земным шаром вокруг его оси, со- здают круговые токи, сопровождаемые магнитным полем. Кроме того, в недрах земли есть большое количество магнитной желез- ной руды, которая также создает магнитное поле земного магне- тизма. Но, как это мы увидим впоследствии, магнитная' руда создает -свое магнитное поле также за счет круговых токов, имеющихся в ее элементарных частицах. Пользуясь магнитной стрелкой, можно установить направле- ние действия магнитных сил, или, иначе говоря, направление магнитного поля. Условно принято считать, что направление магнитной силы в данной точке поля совпадает с тем направлением, которое показывает северный полюс магнитной стрел- ки, ориентированной в данной точке поля.’ На основании этого можно сказать, что магнитное поле земного маг- нетизма на поверхности земли направлено с юга на север и что магнитная стрелка своим южным полюсом всегда обращена навстречу силам поля. Для графического изображения электрического поля мы пользовались электрическими силовыми линиями. Аналогично этому для графического изображения магнитного поля исполь- зуют магнитные силовые линии. Магнитной силовой линией называют такую геометрическую линию, в любой точке которой магнитная сила является каса- _______ тельной к ней и направлена в ту сторону, Рис. 100. Северный полюс магнитной стрелки показывает направление магнитного поля куда показывает своим северным полю- сом ориентированная в этом поле магнит- ная стрелка (рис. 100). Согласно этому определению магнитные силовые линии в поле земного магнетизма направлены с юга на север. В общем случае на практике прихо- дится иметь дело с такими магнитными полями, в которых магнитные линии имеют самую разнообразную конфигурацию и 2С6
Рис. 101. Магнитное поле стерж- невого магнита Рис. 102. Однород- ное магнитное поле располагаются в различных областях поля с различной плотностью. Магнитные поля подобного рода называются неоднород- ными. Примером неоднородного магнитного тюля может слу- жить поле стержневого магнита, изображенное на рис. 101. В противоположность неоднородному магнитному полю в однородном магнитном поле магнитные линии прямоли- нейны, расположены параллельно друг другу и с одинаковой плотностью. Примером однородного магнитного поля может служить поле между разноименными магнитными полюсами двух постоянных магнитов, имеющих плоские торцовые поверх- ности, расположенные параллельно друг другу и на относительно небольшом расстоянии друг от друга (рис. 102). § 68. МАГНИТНАЯ ИНДУКЦИЯ Каждое магнитное поле характеризуется не только направлен нием в пространстве, но и своим магнитным состоянием. Меха- ническая сила, действующая на один и тот же проводник с то- ком, в одном магнитном поле может быть больше или меньше, чем в другом; энергия, запасенная в единице объема одного маг- нитного поля, может быть также больше или меньше, чем в дру- гом поле, и т. д. Мерой магнитного состояния магнитного поля является маг- нитная индукция. Чем больше магнитная индукция В поля, тем при всех прочих одинаковых условиях больше механическая сила, действующая на проводник с током, помещенный в это поле, тем больше энергия, приходящаяся на единицу объема этого поля, и т. д. Магнитная индукция — величина векторная. Это значит, что она характеризуется в каждой точке магнитного поля не только численным значением, но ц направлением в про- странстве.
Вектор магнитной индукции В имеет в каждой точке магнит- ного поля направление, совпадающее с направлением касатель- ной к магнитной линии, проходящей через эту точку (рис. 103). Следовательно, вектор магнитной индукции в каждой точке маг- нитного поля показывает направление этого поля. В соответ- ствии с этим магнитные линии иначе называют линиями вектора магнитной индукции, так как по сути дела это одни и те же геометрические линии. Чтобы иметь представление о численном значении вектора магнитной индукции В, необходимо рассмо- треть какое-либо проявление магнитного поля, например дей- ствие механической силы на проводник с током, помещенный в это поле. Рис. 103. Вектор магнитной ин- дукции В совпадает с направле- нием касательной к магнитной линии Рис. 104. Про- водник с током в магнитном поле Допустим, что в однородное магнитное поле (рис. 104) внесен прямолинейный проводник длиной Z, по которому протекает по- стоянный ток /. Этот проводник с током I расположен в магнит- ном поле перпендикулярно магнитным линиям, и одновременно с этим он расположен .перпендикулярно плоскости рисунка. Кру- жок, изображенный на рис. 104, представляет собой площадь по- перечного сечения данного проводника, а крестик, изображенный в кружке, показывает, что постоянный ток в проводнике направ- лен от нас за плоскость рисунка. Если бы этот ток был направ- лен из-за плоскости рисунка к нам, то в сечении проводника это направление тока было бы обозначено точкой. Опытом установлено, что на проводник с током, помещенный в магнитное поле, действует механическая сила В, которая в рас- сматриваемом нами случае направлена перпендикулярно провод- нику с токрм I и магнитным линиям (рис. 104). Величина меха- нической силы F, действующей на рассматриваемый нами про- водник с током, пропорциональна величине магнитной индук- ции В, величине тока /, текущего по проводнику, и длине / этого проводника; в соответствии с этим она определяется по формуле F=BIl, (95) 208
где F — механическая сила; / — ток в проводе; I — длина провоДа; В — магнитная индукция. Из формулы (95) получаем: В~~ и ’ (96) т. е. магнитная индукция численно равна ме- ханической силе, действующей на единицу длины провода, по которому протекает ток, равныйединице. Воспользовавшись формулой (96), выясним, какова размер- ность магнитной индукции в системе единиц МКСА г™___Г У7"!__ ньютон ___ джоуль _ L •* [_ Il J ампер-метр ампер-кв. метр __ вольт-ампер-сек. _ вольт-секунда ампер-кв. метр____кв. метр ’ т. е. размерность магнитной индукции выражается в вольт-се- кундах на квадратный метр. В соответствии с этим за единйцу измерения магнитной ин- дукции в системе единиц МКСА принята - вольт-секунда Л в-сек, 1 V-sec X кв. метр \ л/2 ’ m2 J Если в формуле (96) положить, что сила F = 1 ньютон, ток I= 1 ампер, длина провода I = 1 метр, то магнитная индук- ция В будет равна I —: о____ F 1 ___. в-сек ° II — 1 • 1 “ 1 Следовательно, одна вольт-секунда на квадратный метр есть магнитная индукция такого однородного магнитного поля, в ко- тором на прямолинейный провод длиной в один метр с током в один ампер, расположенный перпендикулярно магнитным линиям поля, действует сила, равная одному ньютону. Пример 63. Определить магнитную индукцию однородного магнитного поля, если прямолинейный проводник длиной I = 2 м с током I — 20 а, на- правленный перпендикулярно к магнитным линиям поля, испытывает на себе действие механической силы F = 32 ньютона со стороны этого поля. 14-1377 209
Решение. Применяя формулу (96), находим о_________________ F __ 32 ____~~ в-сек & ~~ ~ТГ "20^2" “"и’6 • Примечание. В абсолютной электромагнитной системе единиц СГСМ за единицу измерения магнитной индукции принят 1 гаусс (1 ec, Gs): 1 1 - д ё'СЗК I гаусс = 10 4 2—. § 69. МАГНИТНАЯ ПРОНИЦАЕМОСТЬ Всякое вещество независимо от его природы способно в той или иной степени намагничиваться. Такие вещества, как воздух, вода, уголь, дерево, бумага, фарфор, стекло, медь, алюминий, цинк, свинец и т. д., намагничиваются весьма слабо, так что практически их намагничиванием можно пренебречь. Наоборот, такие вещества, как железо, никель, кобальт и некоторые сплавы, намагничиваются очень сильно и создают достаточно мощное собственное магнитное поле. Величина, характеризующая способность тел намагничиваться, называется магнитной проницаемостью (ji). Опытом установлено, что чем больше магнитная проницае- мость ’[1 среды, тем при прочих равных условиях в ней больше магнитная индукция. Выясним размерность магнитной проницаемости р. и единицу измерения ее в системе единиц МКСА. Исследования показали, что постоянный ток /, протекающий по бесконечно длинному прямолинейному проводу относи- тельно малого сечения, создает магнитное поле, магнитная ин- дукция которого пропорциональна величине тока /, протекаю- щего по этому проводу, и обратно пропорциональна расстоя- нию г от оси провода до исследуемой точки поля. В соответствии с этим величина магнитной индукции В магнитного поля, созда- ваемого током /, протекающим по бесконечно длинному проводу, определяется по формуле = (97) где н—магнитная проницаемость среды, в которой находится данное магнитное поле; 2ц — 2 • 3,14 = 6,28 — постоянное число. Из формулы (97) следует, что 2тсГВ !х==_—Т-' Воспользовавшись формулой (98), выясним размерность маг- нитной проницаемости р в системе единиц МКСА: вольт-секунда метр---т--------- г 1 Г 2яг2? Ч г 1св. метр Н=[——J =-------------, 210
или г -J ом-секунда генри * метр метр * т. е. магнитная проницаемость р. в системе единиц МКСА имеет размерность ом-секунда на метр или, иначе, генри на метр, так как 1 генри равен 1 ом-секунда. В соответствии с этим за единицу измерения магнитной про- ницаемости р в системе единиц МКСА принят 1 генри на метр — 1 генРи Л гн J в \ метр \ м ’ m / Магнитная проницаемость 1 — весьма большая величина. Лучшие сорта магнитных сплавов обладают магнитной прони- цаемостью, в сотые доли генри на метр. Магнитная проницаемость вакуума равна __ 4п гн Ио — -Jot' ~ • В природе существуют вещества, которые имеют магнитную проницаемость большую, чем магнитная проницаемость вакуума, и меньшую. Отвлеченное число, показывающее, во сколько раз магнитная проницаемость дан- ного вещества больше магнитной проницае- мости вакуума или какую ее долю она со- ставляет, называется относительной магни т- ной проницаемостью Относительная проницаемость меди равна 0,999991, воз- духа— 1,00000036, марганца— 1,0037 и‘т. д. Так называемые ферромагнитные вещества и некоторые специальные сплавы, имеют относительную магнитную проницаемость, измеряемую сотнями, тысячами и десятками тысяч единиц. Об этих веще- ствах будет подробно сказано в главе XVI «Магнитная цепь со сталью». Если известна относительная магнитная проницаемость веще- ства то его магнитную проницаемость можно вычислить по формуле И = РгНо> (99) где р— магнитная проницаемость вещества в генри на метр; Ру—относительная магнитная проницаемость вещества; 4п гн Ро = -цр- —— магнитная проницаемость вакуума. 14* 211
Пример 64. Отожженное железо имеет относительную магнитную прони- цаемость = 7000. Определить магнитную проницаемость этого железа. Решение. Применяя формулу (99), находим ' = И,Ио = 7000- -g- = 8,8- Ю-з . Примечание. Магнитная проницаемость вакуума ро в системе единиц СГСМ принята равной единице и является безразмерной величиной (ро=1). § 70. МАГНИТНЫЙ ПОТОК Магнитным потоком называется общее чи ело магнитных линий, пронизывающих данную поверхность. Допустим, что задано однородное магнитное поле, в котором, как известно, магнитные линии являются прямыми линиями, расположенными всюду параллельно друг другу и с одина- ковой плотностью. Определим магнитный поток, пронизываю- щий плоскость, расположенную перпендикулярно магнитным ли- ниям (рис. 105). Так как магнитная индукция численно равна магнитному потоку, приходящемуся на единицу плоскости, то весь магнитный поток равен произведению магнитной индукции на величину всей рассматриваемой плоскости, т. е, Ф = 55, (100) где Ф— магнитный поток; В—магнитная индукция; 5—величина плоскости. Воспользовавшись формулой (100), определим размерность магнитного потока в системе единиц МКСА: вольт-секунда-кв. метр кв. метр ’ ИЛИ [Ф] = вольт-секунда. Следовательно, размерность магнитного потока в системе единиц МКСА выражается в вольт-секундах.' В соответствии с этим за единицу измерения магнитного по- тока в системе единиц МКСА принята 1 вольт-секунда (1 в-сек; 1 V-sec). Если* в формуле (100) положить, что В = 1^^ и S = 1 кв. м, то получим Ф = 1.1 = 1 в-сек, т. е. одна вольт-секунда есть такой магнитный поток, который пронизывает плоскость в один квадратный метр, расположенную 212
перпендикулярно магнитным линиям однородного магнитного поля, обладающего магнитной индукцией, равной одной вольт- секунде на квадратный меФр. Примечание. В абсолютной электромагнитной системе единицСГСМ за единицу измерения магнитного потока принят 1 максвелл (1 мкс; 1 Мх): 1 максвелл =10“8 вольт-секунд. Если магнитные линии однородного на плоскость под углом а (рис. 106), магнитного поля падают то величину магнитного Рис. 105. Магнитные линии пере- секают плоскость под прямым углом Рис. 106. Магнит- ные линии пересе- кают плоскость ПОД углом а потока, пронизывающего данную плоскость, можно определить по формуле Ф = BS-cos а, (101) где Ф— магнитный поток в вольт-секундах; В— магнитная индукция в вольт-секундах на квадратный метр; а—угол, составленный магнитной линией с перпендикуля- ром (нормалью) к плоскости S; 5 — площадь в квадратных метрах. Пример 65. Магнитные линии магнитного поля падают на плоскость под углом а = 60° (рис. 106). Определить величину магнитного потока, прони- зывающего эту плоскость, если ее односторонняя поверхность S = 0,004 \м2, „ л « в'сек а магнитная индукция В => 0,2 —. Решение. Применяя формулу (101), находим Ф = BS-cos а = 0,2-0,004-cos 60° «= 4-10-4 в-сек. Магнитные линии, или, иначе сказать, линии век- тора магнитной индукции В, всегда непрерыв- ны, т. е. з а м к н у т ы на себя. Следовательно, магнитный поток при переходе из одной среды в другую, имеющую иную магнитную проницаемость, не 213
Рис. 107. Магнитная линия при переходе через плоскость раз- дела двух сред с различными магнитными проницаемостями преломляется должен изменяться по своей величине, магнитные линии при пе- реходе через границу раздела двух сред никогда не разрываются. Однако, как показывает опыт, магнитный поток при переходе через границу раздела двух сред с разными магнитными прони- цаемостями меняет свое направле- ние. Его магнитные линии прелом- ляются, и чем больше разница в ве- личинах магнитной проницаемости этих сред, тем большее претерпевают преломление магнитные линии. Допустим, что две однородные и неограниченные среды с магнитны- ми проницаемостями, соответственно равными ры и р-2, имеют границей своего раздела плоскость. На рис. 107 показана прямая линия ab, являю- щаяся линией пересечения плоско- сти рисунка с плоскостью раздела двух заданных сред. Предположим, что в среде с магнитной проницае- мостью [11 магнитные линии (линии вектора магнитной индукции) падают на плоскость раздела двух заданных сред под углом оц. Это значит, что вектор магнитной индукции Bi у поверхности раздела двух сред составляет угол оц с перпендикуляром к этой поверхности. Угол ai называется углом падения магнитной линии. При переходе через плоскость раздела двух заданных сред магнитные линии как-то преломляются, и вектор магнитной ин- дукции В2 во второй среде (в среде с магнитной проницаемо- стью |i2) составляет с перпендикуляром к плоскости раздела двух сред уже угол а2. Угол а2 называется углом прелом- лениямагнитнойлинии. -- Между углом падения и углом преломления магнитных линий существует строго определенная зависимость: (Ю2) tg«2 РГ2 т.е. тангенс угла падения ai магнитной линии так относится к тангенсу угла преломления а2 ее, как относительная магнитная проницае- мость рг1 первой среды относится к относи- тельной магнитной проницаемости |хг2 второй среды? -Пример 66. Линии магнитной индукции в железе составляют угол ai = 45° (угол падения) с перпендикуляром к плоскости раздела железа и воздуха. Требуется определить угол а2 (угол преломления), составленный линией маг- нитной индукции в воздухе с перпендикуляром к плоскости раздела, если известно, что относительная магнитная проницаемость железа равна Ртжел = 500, а относительная магнитная проницаемость воздуха рго = L к 214
Решение. Применяя формулу (102), находим tg 45* _ 500 tg«2 1 ’ откуда '8«! = -!%5Г = Й = '>.002- По тригонометрическим таблицам находим, что для tg а2 = 0,002 угол а2 = 0°10', т. е. линии магнитной индукции по выходе из железа в воздух направлены практически, перпендикулярно плоскости железа. § 71. НАПРЯЖЕННОСТЬ МАГНИТНОГО ПОЛЯ Наряду с магнитной индукцией для характеристики магнит- ных полей применяется вспомогательная расчетная величина, называемая напряженностью магнитного поля. Напряженность магнитного поля — величина векторная. Вектор напряженности магнитного поля Н имеет в однородной неограниченной среде одинаковое направление с вектором магнитной индукции В. Соотношение численных значений векторов магнитной индук- ции В и напряженности магнитного поля Н определяется по формуле <103> t* где Н— напряженность магнитного поля; В— магнитная индукция; р.— магнитная проницаемость среды. Воспользовавшись формулой (103), выясним размерность на- пряженности магнитного поля в системе единиц МКСА; вольт-секунда кв. метр __ вольт-секунда генри ом-секунда-метр ’ метр р у ампер J метр ’ т. е. размерность напряженности магнитного поля выражается в системе единиц МКСА в амперах на метр. В соответствии с этим за единицу измерения магнитной ин- дукции в системе единиц МКСА принят 1 ампер на метр (1~, или 21§
Как известно, магнитная проницаемость вакуума |±0 = — • Следовательно, если в данной точке магнитного поля магнитная индукция в вакууме равна Во, то напряженность магнитного поля Но в этой точке поля равна тт ___ Во _ Д) Ю7 D ю7 Если в последней формуле положить, что магнитная индук- ция в вакууме равна BQ = , то напряженность магнит- ного поля в соответствующей точке вакуума будет равна одному амперу на метр: J_f _ Ю7 D _ 107 4п _ , а По—-4Г^— 4тс ’ лГ’ Следовательно, данная точка магнитного поля в вакууме (практически в воздухе) имеет напряженность 1 ампер на метр, если магнитная индукция в этой точке > Н, поля равна /р ______________________________ 4я в-се к “ То7’ * J? f yr Примечание. В абсолютной электромаг- / // нитной системе единиц СГСМ за единицу напря- / / женности магнитного поля принимается 1 эрстед / ¥ (1 э, 1 Ое): / / 1 1000 а оп а / 1 ЭРСТеД = -^—= 80—. Рис. 108. Линии век- Геометрическая линия, в любой точке тора напряженности которой вектор напряженности магнитного магнитного поля г — r г поля Н совпадает с касательной к этой линии (рис. 108), называется линией вектора напря- женности магнитного поля или просто силовой линией магнитного поля. В однородных средах сило- вые линии магнитного поля совпадают по направлению с маг- нитными линиями, т. е. линиями вектора магнитной индукции В.
ГЛАВА XV МАГНИТНОЕ ПОЛЕ ПОСТОЯННОГО ТОКА В ОДНОРОДНОЙ СРЕДЕ § 72. МАГНИТНОЕ ПОЛЕ ПОСТОЯННОГО ТОКА, ПРОТЕКАЮЩЕГО ПО ПРЯМОЛИНЕЙНОМУ ПРОВОДУ Допустим, что в некоторую однородную среду помещен прямо- линейный провод, по которому протекает постоянный ток I. Вы- ясним, какова конфигурация и напряженность магнитного поля этого тока. Опытом установлено, что магнитное поле относительно длинного прямолиней- ного провода с током аксиально симме- трично, т. е. во всех точках поля, удален- ных на одинаковые расстояния от оси про- вода, численное значение напряженности магнитного поля одинаково. Магнитные линии этого поля в любой плоскости, рас- положенной перпендикулярно проводу, представляют собой семейство концентри- Рис. 109. Магнитное поле ческих окружностей с центром на оси про- прямолинейного провода вода (рис. 109). с током Направление магнитного поля прямо- линейного провода с током определяется правилом буравчика: если поступательное движение буравчика правовинтовой системы совпадает с направлением тока в проводе, то вращательное дви- жение буравчика совпадает с направлением магнитных линий (рис. ПО). Напряженцость магнитного поля вне прямолинейного про- вода конечных размеров (рис. 111), по которому протекает по- стоянный ток /, определяется по формуле Н== 4^7 <sin а1 + sin (Ю4) где Н — напряженность магнитного поля в амперах на метр; I — ток в амперах; 217
Рис. 110. Поступа- тельное движение буравчика совпадает с направлением тока в проводе, вращатель- ное— с направлением магнитных линий г— расстояние от оси провода до исследуемой тоцки поля в мет- рах; ах и а2—углы, составленные прямыми линиями, проведенными из ис- следуемой точки поля к кон- цевым точкам прямолинейного участка провода, и перпенди- куляром, опущенным на ось провода из точки М. Пример 67. Задан прямолинейный провод с то- ком / = 6,28 а. Определить напряженность маг- нитного поля вне провода. на расстоянии Рис. 111. Вектор напряженно- сти магнитного поля в точке М направлен из-за плоскости чертежа на нас и перпенди- кулярен ей г = 0,02 м от его оси, если положить, что углы на рис. 111 равны друг другу: ai = а2 = 30°. Решение. По формуле (104) находим н = ^7 <si” ". +sin “>) “ тадада (°,5+о,5) = 25 -5-. Если длина провода с током очень велика по сравнению с расстоянием от оси провода до исследуемой точки поля, то углы ai и аг будут близки к 90° и в соответствии с этим sin ctj = sin а2« sin 90° — 1. Следовательно, для длинного прямолинейного провода с то- ком / напряженность магнитного поля Н в точках, расположен- ных вблизи провода и вне его, определяется по формуле «=2^- О'») Пример 68. Задан очень длинный прямой провод, по которому проте- кает ток / = 25,12 а; определить напряженность магнитного поля на рас- стоянии г = 0,2 м от оси провода. 218
Решение. По формуле (105) находим Н = — = —25,12— = 20 — 2w 2-3,14-0,2 м • Если в формуле (105) положить 1= 1 а и = то по- лучим Т_Т _ I ___ 1 __ 1 Л ~ 2кг о 1 м • 2те-2к На основании последнего соотношения можно так определить единицу напряженности магнитного поля в системе МКСА: если в бесконечно длинном прямолинейном проводе ничтожно малого кругового сечения, помещенном в однородную и изотропную среду, протекает ток в 1 ампер, то в точках магнитного поля, 1 удаленных от оси провода на расстояние метра, напряжен- ность магнитного поля равна 1 амперу на метр. Примечание. Однородной и изотропной средой называется такая среда, которая по всем направлениям обладает одинаковыми физическими свойствами. Рис. 112. Магнитное поле внутри провода круглого сечения радиусом R с то- ком Теперь посмотрим, каково магнитное поле внутри прямоли- нейного провода с постоянным током /. Магнитное поле внутри прямолинейного провода с током, как и вне его, аксиально симметрично. Магнитные линии этого поля в любой плоскости, расположенной перпендикулярно проводу с током, представляют собой семейство концентрических окруж- ностей с центром на оси провода (рис. 112). Напряженность магнитного поля внутри провода одинакова для всех точек, равноудаленных от оси провода. Однако в отличие от магнитного поля, расположенного вне прямолинейного провода с током, где напряженность поля изме- няется обратно пропорционально расстоянию от оси провода, в магнитном поле, расположенном внутри провода, напряженность поля изменяется прямо пропорционально расстоянию от оси провода. Величина напряженности магнитного поля внутри прямолинейного провода с током определяется по формуле и = (106) где Н— напряженность магнитного поля внутри прямолинейного провода в амперах на метр; I—ток в проводе в амперах; 219
/?— радиус поперечного сечения провода в метрах; г— расстояние от оси провода до исследуемой точки про- вода в метрах. Пример 69. По прямолинейному проводу радиусом R = 4 мм протекает постоянный ток I ~ 15 а. Требуется определить напряженность магнитного поля внутри провода на расстоянии г = 2 мм от оси провода. Решение. Применяя формулу (106), находим и _ 1г _ 15-2-10~3 _а 2^’“ 2-3,14 (4-10-3)2 м • На рис. 113 показан график зависимости напряженности маг- нитного поля тока 7, протекающего по прямолинейному проводу радиусом /?, от расстояния г, отсчитываемого от оси провода. Здесь прямолинейный участок графика характеризует зависи- Рис. 113. График зависимости напря- женности магнитного поля Н от рас- стояния г от провода мость Н от г для поля внутри провода и криволинейный уча- сток — зависимость Н от г для поля вне провода. В заключение отметим, что для определения магнитной ин- дукции рассмотренного магнит- ного поля прямолинейного про- вода с током необходимо на- пряженность магнитного поля умножить на магнитную прони- цаемость среды, так как В = рЛ, § 73. МАГНИТНОЕ ПОЛЕ КОЛЬЦЕВОГО ТОКА Допустим, что кольцо круглого сечения, по которому проте- кает постоянный ток 7, расположено в однородной среде. На рис. 114 это кольцо показано в разрезе плоскости чертежа. Здесь мы видим, что в верхнем сечении кольца ток уходит от нас за плоскость рисунка (обозначено крестиком), а в нижнем сечении —к нам из-за плоскости рисунка (обозначено точкой). Магнитные линии такого тока представляют собой замкнутые на себя кривые линии. Они выходят с левой стороны кольца, за- тем расходятся по внешнему пространству, сходятся на другой стороне кольца и замыкаются на себя внутри кольца. Для определения направления магнитных линий внутри коль- цевого тока применяется правило буравчика: если вращательное движение буравчика совпадает с направлением тока в кольце, то поступательное движение буравчика совпадает с направле- нием магнитного поля внутри кольца. Установлено, что напряженность магнитного поля в любой точке оси кольцевого тока (рис. 115) определяется по формуле H-^sin’a, (107) 220
веденной деляется где Н — напряженность магнитного поля в амперах на метр; I — ток в амперах; R — радиус кольца в метрах; а — угол, составленный осью кольца с* прямой линией, про- к средней линии кольца из точки, где опре- величина напряженности магнитного поля. Пример 70. Определить напряженность магнит- ного поля на оси кольцевого тока (рис. 115), если известно, что ток в кольце I = 20 а, радиус кольца R = 0,05 м и а = 30°. Решение. Применяя формулу (107), находим Н = — sin3 а = 2/? ' = sin3 30° = 200 • 0,5s = 25 4 • Z • U,UO M Рис. 114. Магнитное поле кольцевого тока н Рис. 115. Вектор напряженно- сти магнитного поля Н в точ- ке Мнаправлен по оси вправо В центре кольца на его оси угол а = 90°, а поэтому напря- женность магнитного поля в центре кольца может быть опреде- лена по формуле H = 4r- (108) Пример 71. Определить напряженность магнитного поля в центре кольца на его оси, если известно, что радиус кольца R = 0,1 м, а ток в кольце 7 = 25 а. Решение. Применяя формулу (108), находим § 74. МАГНИТНОЕ ПОЛЕ ОДНОСЛОЙНОЙ ЦИЛИНДРИЧЕСКОЙ КАТУШКИ Допустим, что цилиндрическая катушка с однослойной об- моткой, по которой протекает постоянный ток /, расположена в однородной среде. На рис. 116 катушка показана в разрезе плоскости чертежа. Ток в верхних сечениях витков катушки ухо- дит от нас за плоскость рисунка, а в нижних сечениях витков течет к нам из-за плоскости рисунка. 221
Рис. 116. Однослойная цилиндрическая катушка и ее магнитное поле Магнитная стрелка, поме- щенная внутри катушки, по- казывает своим северным по- люсом направление магнит- ного поля внутри катушки. Магнитные линии магнитно- го поля катушки в рассма- триваемом случае выходят с левой торцовой стороны катушки, затем, распространяясь по внешнему пространству, схо- дятся с другой торцовой стороны катушки, входят в ее внутрен- нюю плоскость и замыкаются на себя. Та сторона катушки, из которой выходят магнитные линии, имеет северный магнитный полюс N, а противоположный конец катушки, куда стекаются магнитные линии,— южный магнитный полюс S. Полярность магнитного поля катушки можно определить, применяя правило правой руки: если ладонь правой руки нало- жить на витки катушки так, чтобы вытянутые четыре пальца указывали направление тока в витках катушки, то отставленный большой палец укажет северный магнитный полюс катушки. Установлено, что напряженность магнитного поля в произ- вольной точке М на оси рассматриваемой катушки (рис. 117) определяется по формуле 7V=-^- (cos а, — cosa2), (109) где Н— напряженность магнитного поля в амперах на метр; /—ток в амперах; w—число витков обмотки катушки; I—длина катушки в метрах; а— углы, составленные осью катушки с прямыми линиями, проведенными к конечному и начальному виткам ка- тушки из точки, где определяется величина напряженно- сти магнитного поля. Рис. 117. Напряженность магнитного поля Н в точке М на оси соленоида Пример 72. Определить напряженность магнитного поля однослойной цилиндрической катушки в некоторой точке М на оси ее (рис. 117), если известно, что ток в витках катушки /= 1,2 а, число витков катушки w = 200, длина катушки I = 0,5 м и углы ой = 60° и аг = 90°. Решение. Применяя фор- мулу (109), находим Н = (cos — cos а2) — (cos 60° — cos 90°)= = 240 (0,5-0) = 120^-. С током 222
Если однослойная цилиндрическая катушка имеет значи- тельно большую длину по сравнению с радиусом ее поперечного сечения (радиусом ее витков), то напряженность магнитного поля внутри катушки, в точках на оси ее, может быть опреде- лена по формуле (110) Пример 73. Определить напряженность магнитного поля внутри относи- тельно длинной цилиндрической катушки, если известно, что число витков катушки w = 200, длина катушки .1 == 0,4 м и ток <в катушке I = 0,25 Л. Решение. Применяя формулу (110), находим и wl 200-0,25 1ПС а //=“Г = “ол~ = 125 V- Произведение числа витков катушки на величину тока, протекающего по ним, назы- вается ампер-витками катушки. Из формулы (ПО) следует, что напряженность магнитного поля катушки с током равна числу ампер-витков катушки, при- ходящихся на единицу длины катушки. В соответствии с этим единицу измерения напряженности магнитного поля называют также 1 ампер-виток на метр и при этом считают, что 1 ампер на метр= I ампер-виток на метр (I — = 1 , Число ампер-витков на метр называют удельными ампер-витками и обозначают условно символом aw, т. е. H==aw
ГЛАВА XVI МАГНИТНАЯ ЦЕПЬ СО СТАЛЬЮ § 75. НАМАГНИЧИВАНИЕ ПАРАМАГНИТНЫХ И ДИАМАГНИТНЫХ ВЕЩЕСТВ Всякий электрический ток, будь то ток проводимости, ионный ток, ток смещения или молекулярные токи в веществе, создает магнитное поле. Рассмотрим, какое влияние оказывают молекулярные токи вещества на процесс намагничивания этого вещества. Известно, что в каждом атоме имеются электроны, которые вращаются вокруг ядра. Эти круговые движения электронов в атомах можно уподобить элементарным кольцевым токам с присущими им магнитными свойствами. Каждый такой эле- ментарный кольцевой ток, или, иначе, молекуляр- ный ток, создает свое собственное магнитное поле с явно выраженными магнитными полюсами. Если все молекулярные токи в веществе расположены так, что их результирующее поле равно нулю, то такое вещество не намагничено (рис. 118, а). Если же на молекулярные токи вещества воздействовать внешним магнитным полем и тем самым заставить их опре- деленным образом ориентиро- ваться в этом поле (рис. 118, б), то такое вещество намагнитит- ся, т. е. станет само источником магнитного поля. Помимо вращательного дви- жения электрона вокруг ядра атома, наблюдается также его вращательное движение вокруг собственной оси — так назы- ваемый спин электрона. В результате этого вращатель- Рис. 118. Расположение элемен- тарных кольцевых токов в железе: а — при их собственном результирующем магнитном поле, равном нулю; б — при воздействии внешнего магнитного поля 224
ного движения электрон создает также магнитное поле. В данном случае электрон со своим магнитным полем напоминает элемен- тарный магнитик с явно выраженными магнитными полюсами. На рис. 119 схематически показан вращающийся вокруг соб- ственной оси электрон (спин электрона) с явно выраженными магнитными полюсами. Следовательно, спин электрона можно рассматривать как некоторую разновидность молекулярного тока в атомах вещества. Установлено, что орбиты, по которым движутся электроны в атомах, непрерывно изменяют положение своих плоскостей в пространстве. Если на вещество воздействовать внешним маг- нитным полем, то оси электронных орбит будут совершать вра- щательное движение вокруг направления этого поля, т. е. воз- никнет так называемое прецессионное движение электронных орбит внутри атома (рис. 120). Это до- полнительное движение электрона в атоме, связанное с прецес- сионным движением его орбиты, создает также магнитное поле, которое направлено навстречу внешнему магнитному полю. Сле- довательно, прецессионное движение орбит электронов в атоме тоже можно рассматривать как своеобразные молекулярные токи в атоме. Таким образом, молекулярные токи в веществе могут быть обусловлены различными причинами. При этом характер на- магничения вещества зависит от того, какая разновидность моле- кулярных токов и в какой степени преобладает в элементарных частицах вещества при его намагничивании. Если молекулярные токи в веществе ориентируются так, что создаваемое ими магнитное поле усиливает внешнее магнитное н Рис. 119. Спин электрона Рис. 120. Прецессионное движение электронной ор- биты 15-1377 225
поле, намагничивающее вещество, то такое явление называется парамагнетизмом, а вещества, в которых это явление наблюдается, называются ' парамагнитными. К парамагнитным веществам относятся вольфрам, марганец, платина и ряд других веществ. Если молекулярные токи в веществе ориентируются в основ- ном так, что создаваемое ими магнитное поле ослабляет внешнее магнитное поле-, намагничивающее вещество, то такое явление называется диамагнетизмом, а вещества, в которых это явление наблюдается, называются диамагнитными. К диамаг- нитным веществам относятся медь, серебро, углерод, кадмий, висмут и ряд других веществ. Итак, при расчете магнитных полей в веществе приходится учитывать не только внешние магнитные поля, осуществляющие намагничивание веществ, но и магнитные поля, создаваемые мо- лекулярными токами вещества. Выясним, каково ^удет результирующее магнитное поле в ве- ществе, если в нем создать внешнее магнитное поле. Допустим, что в некотором однородном веществе возникло внешнее магнитное поле, напряженность которого равна Н. Под влиянием этого поля вещество намагнитилось, т. е. его молеку- лярные токи как-то ориентировались во внешнем магнитном поле и создали свое собственное магнитное поле, которое назовем внутренним магнитным полем. Последнее, наложившись на внешнее магнитное поле, создает совместно с ним некоторое ре- зультирующее магнитное поле. Величина магнитной индукции результирующего магнитного поля в веществе может быть определена по формуле В = + 1 (111) где В— вектор магнитной индукции результирующего магнит- __ ного поля в веществе; \^Н—вектор магнитной индукции внешнего магнитного поля в веществе; Рр—магнитная проницаемость вакуума; Н — вектор напряженности внешнего магнитного поля; 7 (читать «йот») — магнитная индукция, создаваемая молекуляр- ными токами вещества, называемая намагниченностью вещества. Из формулы (111) следует, что вектор магнитной индук- ции В результирующего магнитного поля в веществе^ равен геометрической сумме вектора магнитной индукции внеш- него магнитного поля, имеющегося в веществе, и вектора магнит ной индукции J (намагниченности) магнитного поля, создавае- мого молекулярными токами вещества. 226
Если векторы В, раН и J совпадают по направлению, что чаще всего наблюдается в веществах, то равенство (111) можно записать не в векторной форме: B = ^QH + J. (112) Но, как известно, магнитная индукция В в любой среде равна произведению напряженности Н магнитного поля в этой среде на магнитную проницаемость р. этой среды, т. е. В = ?Н. Подставив данное выражение в формулу (112), получим V-H = р0Л/ + J. Отсюда находим, что И = Но + 4’ (ИЗ) т. е. магнитная проницаемость вещества равняется сумме маг- нитной проницаемости вакуума р,0 и некоторой добавочной маг- нитной проницаемости, численно равной отношению намагничен- ности J к напряженности Н внешнего магнитного поля. Так как магнитная проницаемость вакуума равна |л0, то, исходя из формулы (113), получим Го = Но + тт*, или 7/ = 0 и JQ = О, т. е. намагниченность Jo вакуума равна нулю. Это понятно, так как в вакууме нет собственных молекулярных токов. Для парамагнитных и диамагнитных ве- ществ намагниченность J пропорциональна напряженности Н внешнего магнитного поля: J = x/7, (114) где х (читать «каппа») — коэффициент пропорциональности, называемый магнитной восприимчивостью вещества. Подставив вместо J его выражение из формулы (114) в фор- мулу (113), получим , ъН = + или И— Ho + Z- 015) Для парамагнитных веществ магнитная восприимчивость х>0, и в соответствии с этим магнитная проницаемость их больше магнитной проницаемости вакуума (г > го)* Наоборот, 15* 227
для диамагнитных веществ магнитная восприимчивость % < О, и в соответствии с этим магнитная проницаемость диамагнитных веществ меньше магнитной проницаемости вакуума (р.<ро). Если равенство (115) разделить почленно на ро, то получим -^ = 1 + -, Но Но ИЛИ г, = 1 + ^, (116) где нг—относительная магнитная проницаемость вещества; --— относительная магнитная восприимчивость вещества. Для парамагнитных веществ относитель- ная магнитная проницаемость больше еди- ницы, а для диамагнитных веществ — меньше единицы. В табл. 25 приведены относительные магнитные про- ницаемости некоторых парамагнитных и диамагнитных веществ. Таблица 25 Относительные магнитные проницаемости некоторых сред Парамагнитное вещество Магнитная проницаемость Диамагнитное вещество Магнитная проницаемость Воздух 1,00000036 ' Висмут 0,999825 Олово 1,000004 Графит 0,999895 Алюминий 1,000023 Сурьма 0,999937 Иридий 1,000063 Ртуть 0,999975 Платина 1,000364 Серебро 0,999981 Палладий 1,00069 Цинк 0,999989 Марганец 1,0037 Медь 0,999991 § 76. НАМАГНИЧИВАНИЕ ФЕРРОМАГНИТНЫХ ВЕЩЕСТВ Особый интерес для практики представляют так называемые ферромагнитные вещества, так как намагниченность их весьма значительно превышает намагниченность обычных парамагнит- ных веществ. К ферромагнитным веществам относятся железо, сталь, чугун, никель, кобальт и некоторые сплавы. Если в ферромагнитном веществе создать внешнее магнит- ное поле, то это вещество намагнитится, т. е. образует собствен- ное магнитное поле за счет действия своих молекулярных токов. Магнитная индукция результирующего магнитного поля в этом случае будет равна В = + Л (И7) 228
где В— вектор магнитной индукции результирующего магнит- ного поля; р0/7— магнитная индукция внешнего магнитного поля; J—вектор намагниченности (магнитная индукция, созда- ваемая молекулярными токами вещества). Вотличие от парамагнитных веществ намаг- ниченность J ферромагнитных веществ не про- порциональна напряженности внешнего маг- нитного поля. На рис. 121 показана кривая Оа\ЬхС\ зависимости намагни- ченности J ферромагнитного вещества от напряженности Н внеш- него магнитного поля в веществе. Эта кривая показывает, что магнитной индукции В ферромагнитного вещества от напряженности внешнего магнитного поля Н при относительно малых значениях напряженности И внешнего магнитного поля намагниченность J вещества растет весьма ин- тенсивно и пропорционально напряженности И внешнего магнит- ного поля (участок кривой Осц). Это объясняется тем, что моле- кулярные токи еще слабо ориентированы во внешнем магнитном поле и в соответствии с этим еще слабо противодействуют силам внешнего магнитного поля, осуществляющего поворот этих то- ков. Затем по мере намагничивания вещества скорость нараста- ния его намагниченности J начинает спадать (участок кри- вой Я1&1). Это объясняется тем, что уже значительная часть мо- лекулярных токов ориентирована соответствующим образом во внешнем магнитном поле и дальнейший поворот этих токов со- пряжен с большим противодействием их силам внешнего магнит- ного поля. И, наконец, рост намагниченности J настолько сни- жается, что кривая J переходит в прямую линию, имеющую от- 229
носительно малый наклон по отношению к горизонтальной оси ОН. Это объясняется тем, что в ферромагнитном веществе наступает состояние магнитного насыщения, при котором вели- чина намагниченности J постепенно приближается к своему пре- дельному значению Jmax, изображенному на рис. 121 прямой пунктирной линией тп. Что же касается составляющей магнитной индукции роН внешнего магнитного поля, то она изменяется пропорционально напряженности Н этого поля. На рис. 121 эта зависимость пока- зана прямой линией Ok. Чтобы получить кривую зависимости магнитной индукции В результирующего магнитного поля от напряженности Н внеш- него магнитного поля, необходимо сложить величины ^Н и J и затем по полученным точкам построить кривую зависимости В от Н. На рис. 121 эта зависимость изображена кривой Оа2&2^2, называемой первоначальной кривой намагничи- вания. Эта кривая напоминает кривую зависимости намагни- ченности J от Н. Здесь также имеется начальный участок Оа2 кривой, где магнитная индукция растет весьма быстро вместе с ростом Н и изменяется почти пропорционально последней. За- тем рост магнитной индукции В замедляется (участок кри- вой а2Ь2). И, наконец, когда наступает магнитное насыщение, магнитная индукция растет еще медленнее и изменяется почти пропорционально Н. Однако в отличие от намагниченности J магнитная индукция В растет до тех пор, пока растет вели- чина Я. Это понятно, так как при прекращении роста намагни- ченности J величина рю# продолжает расти вместе с ростом ве- личины Н. На рис. 122 приведена кривая первоначального намагничива- ния электротехнической стали, широко применяемой в современ- ной практической электротехнике. 230
Ниже приведена табл. 26 характеристик намагничивания не- которых ферромагнитных материалов, применяемых в электро- технической промышленности. Таблица 26 Характеристики намагничивания материалов Магнитная индукция В в-сек м2 тт г> а Напряженность магнитного поля Н — электротехниче- ская сталь . марки Э42 электротехниче- ская сталь марки Э11 мягкая кованая сталь марки Ст2 чугун 0,1 80 600 0,2 21 87 160 900 0,3 29 113 240 1220 0,4 37 140 320 1640 0,5 48 171 400 2200 0,6 63 211 488 2940 0,7 84 261 584 3920 0,8 110 318 682 5400 0,9 140 397 798 7360 1,0 185 502 924 10100 1,1 260 647 1090 14000 1,2 380 843 1290 19200 1,3 680 1140 1590 26200 1,4 1450 1580 2090 34800 1,5 3100 2500 2890 47800 1,6 5600 4370 4100 — 1,7 9500 7780 — — 1,8 14600 12800 — — 1,9 23000 19700 — — 2,0 41000 31000 — — Из табл. 26 видно, как значительно различаются характери- стики намагничивания приведенных в ней ферромагнитных ве- ществ. Так, например, чтобы иметь магнитную индукцию в стали марки Э42, равную 1,0-^-^-, необходимо создать в ней на- пряженность внешнего магнитного поля, равную 185—, а для того чтобы получить такую же магнитную индукцию в чугуне, необходимо создать в нем напряженность внешнего магнитного поля, равную 10100— . Выяснив характер изменения магнитной индукции в ферро- магнитном веществе в зависимости от напряженности внешнего магнитного поля, перейдем к рассмотрению зависимости магнит- ной проницаемости р, ферромагнитных веществ от напряжен- ности Н внешнего магнитного поля. Как известно, магнитная проницаемость любого вещества равна В н— н 231
Но так как согласно формуле (117) В = 4- 7, то можно написать, что , J Р = Ро + 77 (118) Из формулы (118) следует, что магнитная проницаемостьр ферромагнитного вещества зависит от отношения намагничен- ности J к напряженности Н внешнего магнитного поля в нем. Вначале, когда Н = 0 и 7 = 0, магнитная проницаемость ферро- магнитного вещества равна -магнитной проницаемости вакуума (р = ро). Затем по мере роста Н намагниченность J растет весьма быстро, и в соответствии с этим также быстро растет отношение а вместе с ним и магнитная проницаемость р. iz 7 Когда отношение становится максимальным, достигает ма- ксимального значения и • магнитная проницаемость р. После этого рост отношения замедляется и при Н-> оо это отно- шение стремится к нулю, поскольку как некоторой ко- нечной величине. Магнитная проницаемость в этом случае стремится к вели- чине, равной магнитной проницаемости вакуума (р = р0). Если разделить левую и правую части равенства на р,0, по- лучим = — = 1 +-77, (119) г Ро Рс/7 ’ . v 7 проницаемость вещества. где р,— относительная магнитная Из формулы (119) следует, что Рис. 123. Кривая зависимости магнит- ной индукции В и магнитной прони- цаемости р стали от напряженности внешнего магнитного поля Н в совершенно размагничен- ном ферромагнитном веще- стве относительная магнит- ная проницаемость его pr = 1. Затем в соответствии с ро- стом отношения она ра- стет, достигает некоторого максимума, а затем спадает, стремясь при магнитном на- сыщении к единице (рг -> 1). На рис. 123 приведен график зависимости относительной магнитной проницаемости р стали от напряженности Н внешнего магнитного поля. 232
В табл. 27 приведены предельные значения магнитных прони- цаемостей некоторых ферромагнитных веществ. Таблица 27 Магнитная проницаемость ферромагнитных веществ Ферромагнитное вещество Нт Примечание Кобальт .................. Чугун неотожженный . . . . Чугун отожженный.......... Никель.................... Мягкая сталь.............. Железо отожженное......... Трансформаторная листовая сталь .................... Электролитическое железо, плавленное в вакууме . . . Сплав железа с никелем . . Пермаллой „С“ ............ 174 240 620 1 120 2 180 7 000 7 500 12 950 60 000 115 000 Fe 20%, Ni 74%, Си 5%, Мп 1% Ni 78%, Fe 18%, Mo 3%, Мп 0,5% Сложная зависимость магнитной проницаемости ферромагнит- ного вещества от напряженности внешнего магнитного поля и из- менение ее в весьма широких пределах значительно затрудняет расчеты магнитных полей в ферромагнитных веществах. Поэтому при расчете магнитных полей в ферромагнитных веществах при- бегают к кривым намагничивания веществ, т. е. к кривым зави- симости магнитной индукции В в веществе от напряженности Н внешнего магнитного поля. Кривые намагничивания железа впервые были построены рус- ским ученым А. Г. Столетовым. В своей докторской диссертации «Исследование функции намагничения мягкого железа» он впервые установил, что по мере увеличения намагничивающей силы магнитная проницаемость железа сначала растет, дости- гает максимума и потом вновь убывает. Столетов писал: «...и зучение функции намагничивания железа может иметь практическую важность при устройстве и употреблении как электромаг- нитных двигателей, так и тех магнитоэлектри- ческих машин нового рода(т. е. динамомашин), в которых временное на-магничивание железа играет главную роль». Научные работы А. Г. Столетова в области исследования на- магничивания железа имели громадное значение для развития электротехники. § 77. ЦИКЛИЧЕСКОЕ ПЕРЕМАГНИЧИВАНИЕ Если изменять напряженность магнитного поля в стали, то одновременно с этим будет изменяться магнитная проницаемость 233
и магнитная индукция в ней. Проследим, как будет изменяться магнитная индукция в стали, если напряженность магнитного поля изменяется по величине и направлению. Допустим, что имеется тороидальная катушка (рис. 124), сер- дечником которой является стальной тороид — стальное кольцо. Напряженность магнитного поля, создаваемая такой катушкой, как и очень длинного цилиндрического соленоида, определяется формулой ^=4- <12Q) / где I — средняя длина силовой линии в тороиде в метрах (по- казана на рис. 124 пунктиром). Магнитная индукция в тороиде будет равна В = ^Н = ^-. (121) Если изменять величину тока в обмотке тороидальной ка- тушки, то в соответствии с этим будет изменяться напряженность магнитного поля Н и магнитная индукция В сердечника. Вели- чину тока можно измерить амперметром, а напряженность маг- нитного поля подсчитать по формуле (120). Магнитную индук- цию В снимают обычно специальными приборами. Если сталь довести до состояния насыщения, например, маг- нитную индукцию довести до величины Вт (точка А на кривой рис. 125), а затем начать размагничивать сталь, уменьшая ток/ и, следовательно, напряженность магнитного поля //, то и вели- чина магнитной индукции В начнет также уменьшаться. Но па- дение величины магнитной индукции будет совершаться не по кривой первоначального намагничивания, а по некоторой кри- вой АС, расположенной выше нее. 234
Когда ток и соответственно с ним напряженность магнитного поля уменьшатся до нуля, магнитная индукция еще не достиг- нет нулевого значения, а сохранит некоторую величину, опреде- ляемую отрезком ОС. Величиной этого отрезка определяется так называемая остаточная магнитная индукция Вг Чтобы сталь раз- магнитить полностью, необходимо в обмотку катушки пропустить ток противоположного направления по сравнению с предыдущим. При некотором значении напряженности магнитного поля, опре- деляемой отрезком OD, вновь образовавшееся магнитное поле полностью скомпенсирует остаточный магнетизм, и сталь сердеч- ника будет совершенно размагничена. Чем больше отрезок OD, тем больше надо затратить энергии, чтобы освободить сталь от остаточного магнетизма. Отрезок OD характеризует собой со- противляемость стали размагничиванию и называется задержи- вающей силой (коэрцитивной силой) намагниченного вещества. Увеличивая напряженность Н в новом направлении, заставим магнитную индукцию возрасти, добиваясь состояния насыщения (ДЛ соответствующего точке Е. Затем, размагничивая сталь, при Н = 0 получим остаточную магнитную индукцию, определяе- мую отрезком OF. Изменяя направление тока и увеличивая его, добьемся того, что сталь размагнитится при напряженности магнитного поля, соответствующей отрезку OG, и при дальнейшем увеличении Н достигнет вновь точки А на первоначальной кривой намагничи- вания. При дальнейшем повторении процесса перемагничивания стали изменение магнитной индукции повторится по замкнутой кривой ACDEFGA, называемой циклической кривой перемагни- чивания. Рис. 125. Циклическая кри- вая перемагничивания Рис. 126. Семейство циклических кривых перемагничивания 235
В рассмотренном процессе перемагничивания материала про- исходило запаздывание намагничивания материала (магнитной индукции) от намагничивающей силы (напряженности магнит- ного поля). Это явление называется гистерезисом. В соот- ветствии с этим циклическую кривую перемагничивания стали часто называют петлей гистерезиса. Если ограничить намагничивание и размагничивание стали другими пределами напряженности магнитного поля (рис. 126), то получится серия кривых перемагничивания для данного мате- риала. Нетрудно видеть, что здесь одному и тому же значению Н стали может соответствовать много различных значений магнит- ной индукции, так как последняя зависит не только от сорта стали, но и от предыдущего ее Магнитного состояния. Эта много- значная зависимость В от Н заставляет прибегать к графиче- скому методу определения ее при помощи кривых намагничи- вания. Кривая линия О А (рис. 125), являющаяся геометрическим местом вершин циклических кривых перемагничивания (петель гистерезиса), называется основной кривой намагни- чивания. Перемагничивание ферромагнитных веществ связано с поте- рей энергии генератора электрической энергии, током которого осуществляется перемагничивание вещества. Эта энергия преоб- разуется в ферромагнитном веществе в тепловую, доказательством чего служит его нагревание при перемагничивании. Установлено, что количество энергии, израсходованной гене- ратором электрической энергии на необратимый процесс (преоб- разование электрической энергии в тепловую) за. полный цикл перемагничивания, пропорционально площади, ограниченной циклической кривой перемагничивания,— площади петли гистере- зиса (рис. 125). Следовательно, во избежание непроизводитель- ных затрат на перемагничивание стали необходимо выбирать ферромагнитные вещества с относительно малой площадью петли гистерезиса. § 78. МАГНИТНЫЕ МАТЕРИАЛЫ Ферромагнитные вещества, обладающие высокими магнит- ными качествами, нашли самое широкое применение в электро- технике. Магнитные материалы, применяемые в электротехнике, под- разделяются на две основные группы: магнитно-мягкие и маг- нитно-твердые материалы. 1. Магнитно-мягкие материалы. Отличительная особенность магнитно-мягких материалов — высокая магнитная проницае- мость р и малая задерживающая (коэрцитивная) сила /7С. Вследствие этого из таких материалов обычно изготовляются сердечники электромагнитов, полюсные наконечники и т. п. 236
К магнитно-мягким материалам предъявляется требование высокой магнитной проницаемости: в них при относительно ма- лых значениях напряженности Н магнитного поля должна быть относительно большая величина магнитной индукции В. Однако при больших значениях переменной магнитной индукции в сер- дечниках электромагнитов довольно велики потери энергии на вихревые токи и на перемагничивание. Для уменьшения их сердечники электромагнитов, трансформаторов и т. д. выполняют не из сплошного ферромагнитного материала, а из отдельных тонких изолированных друг от друга листов и, кроме того, до- бавляют в сталь несколько процентов кремния, значительно уве- личивая этим ее* удельное сопротивление. В современной электротехнике широко применяется магнитно- мягкая легированная кремнием листовая сталь, называемая электротехнической сталью. В табл. 28 приведены некоторые основные магнитные харак- теристики электротехнической стали. Таблица 28 Основные характеристики некоторых типов электротехнической стали Марка стали Содержание крем- ния, % Магнитная индукция В в-сек м2 е сопроти- ом мм2 а? 'w ~ 008 = 7/ ибн при Н = 2000 — м при Н = 4000 — м при Н = 8000 — м при н = 16000 2- м о £ S Ч ж £ * 5 м ЭИ 0,25 0,8—1,8 1,5 1,62 1,75 1,97 Э12 0,25 0,8—1,8 — 1,49 1,61 1,74 1,96 Э21 0,40 1,8—2,8 — 1,48 1,59 1,73 1,94 Э31 0,50 2,8—4,0 — 1,46 1,57 1,7 1,9 Э41 0,57 4,0—4,8 1,3 1,45 1,56 1,68 1,88 Э42 0,57 4,0—4,8 1,29 1,44 1,55 1,67 1,87 Э43 0,57 4,0—4,8 1,28 1,43 1,54 1,66 1,87 Э310 0,50 2,8-4,0 1,57 1,7 1,8 1,9 1,98 Э320 0,50 2,8—4,0 1,65 1,8 1,87 1,92 2,0 ЭЗЗО 0,50 2,8-4,0 1,7 1,85 1,9 1,95 2,0 . Среди магнитно-мягких материалов, применяемых в технике связи, особое значение приобрели так называемые магнитоди- электрики. Они очень хорошо проводят магнитные потоки, но в то же время обладают малой электропроводностью, что значи- тельно снижает в них потери энергии при переменных токах вы- сокой частоты. Магнитодиэлектрики изготовляются из мелкого железного по- рошка, частицы которого связаны электроизолирующим лаком. На практике существует много различных магнитодиэлектри- ков. Из них наибольшее распространение получили карбонильное 237
Основные характеристики некоторых Наименование материала Начальная относи- тельная магнитная проницаемость Максимальная отно- сительная магнитная проницаемость тах Электролитическое железо 500—600 15 000 Карбонильное железо 2000—3000 20 000—21 500 Армко-железо (после отжига 900° Ц) 250—350 3000—7000 Пермаллой (78,5% Ni, 21,5% Fe) -> 10000—20000 100 000 Пермаллой хромистый (78,5% Ni, 3,8% Сг и др.) 12 000 62 000—100 000 Пермаллой молибденовый (78,5% Ni, 3,8% Мо и др.) 8000—15 000 70 000—140 000 Гайперник 2000—3500 70 000—90 000 Альсифер (84,9% Fe, 9,5% Si, 5,6% Al) 10 000-35 000 110 000 Белый чугун — 186 Серый чугун — 180 238
Таблица 29 магнитно-мягких материалов Остаточная k магнитная индукция Bf в-сек Магнитная индукция насыщения ах в-сек мг Зад 'ржиигю- шля (коэрци- тивная) сила Нс а м Область применения 0,5 — 16-28,6 Сердечники высокой частоты 0,6 2,03 6,4—8,0 Сердечники большой чувствитель- ности 1,2 - 1,8-Т-2,0 56— 136 Сердечники реле и трансформаторов 0,8—1,2 1,1 24—56 Сердечники реле, экраны и т. д. С,8 0,8 2,0 Сердечники катушек индуктивности и магнитные экраны 0,85 — — То же 0,73 1,6 3,2—5,6 Сердечники реле 0,3 1,2—Н 1,5 2,2 / Корпуса приборов, магнитные экраны 0,475 1,23 960- 1200 Корпуса электрических машин и ап- паратов 0,5 1,39 1200—1430 То же 239
железо, порошкообразный пермаллой, альсифер, магнетит. На- пример, альсифер обладает начальной относительной магнитной проницаемостью ^= 10000-4-35000, максимальной относитель- ной магнитной проницаемостью рГтах = 110000, магнитной ин- дукцией насыщения Втяч = 1,2н-1,5 . В табл. 29. приведены основные характеристики некоторых наиболее употребительных магнитно-мягких материалов. Помимо магнитодиэлектриков, широкое применение получили магнитно-мягкие материалы ферриты, представляющие собой со- единение окислов железа с металлами — никелем, цинком и др. Ферриты благодаря относительно большому удельному сопротив- лению являются полупроводниками. Потери в них на вихревые токи при высоких частотах относительно малы. Ферриты разра- ботаны в Советском Союзе профессором Г. А. Займовским. Из основных марок ферритов, применяемых в Советском Союзе, можно отметить следующие: НЦ-40 (никелево-цинковый с на- чальной относительной магнитной проницаемостью = 40), НЦ-90, НЦ-200, НЦ-250, НЦ-500 и НЦ-1000. Феррит, называе- мый магнетитом, не содержит в себе никаких иных метал- лов, кроме железа, и состоит из закиси — окиси железа Fe3O4. 2. Магнитно-твердые материалы. Отличительная особенность магнитно-твердых материалов — их небольшая магнитная про- ницаемость р,, высокая остаточная магнитная индукция Вг и большая задерживающая (коэрцитивная) сила /Ус. Вследствие этого обычно из таких материалов изготовляются постоянные магниты, применяемые в технике связи, электроизмерительной технике и т. п. На практике существует большое разнообразие типов маг- нитно-твердых материалов. Из них наиболее употребительны вольфрамовые, хромовые и кобальтовые стали, обладающие от- носительно большими величинами задерживающей силы и оста- точного магнетизма. В последние годы советские ученые разработали ряд железо- никелево-алюминиевых сплавов, обладающих весьма высокими магнитными качествами. К ним относятся такие сплавы, как альни, альниси и альнико. Профессор А. С. Займовский разработал магнитно-твердый материал под названием «м а г- н и к о», превосходящий по своим магнитным качествам выше- указанные магнитно-твердые материалы. Железо-никелево-алюминиевые сплавы — нековкие мате- риалы, а поэтому магниты получаются из них путем отливки или спекания их. К новым магнитно-твердым материалам, обладающим весьма высокими магнитными качествами, относятся сплавы платины с железом или кобальтом. . В табл. 30 приведены основные характеристики магнитно- твердых материалов. 240
Таблица 30 Основные характеристики некоторых магнитно-твердых материалов Наименование материала и процентное содержание его примесей Задерживающая сила Н с а м Остаточная магнитная индукция в-сек мг Вольфрамовая сталь ЭЕВА (0,7% С, 6% W) Хромовая сталь ЭЕХЗА (1,0% С, 4 800 1,0 3% Сг) Кобальтовая сталь ЭЕК5 (1,0% С, 4 800 0,9 5% Сг, 5% Со) * . . Кобальтовая сталь ЭЕК10 (1,0% С, 7 200 0,9 5% Сг, 1,5% W, 10% Со) Кобальтовая сталь ЭЕК15 (1,0% С, 10 400 0,9 8% Сг, 5% W, 15% Со) Кобальтовая сталь ЭЕКЗО (1,0% С, 14 400 0,8 5% Сг, 5% W, 30% Со) 17 600 0,9 Альни (40,и Си, 25% Ni, 14% А1) . . 44 000 0,55 Альниси (1% Si, 34% Ni, 14% Al) . . Альнико (6% Си, 12% Со, 17% Ni, 64 000 0,4 10% Al) Магнико (3% Си, 24°/0 Со, 13% Ni, 40 000 0,7 8% Al) - Платино-железный сплав (Pt 78%, 44 000 1,25 Fe 22%) Платино-кобальтовый сплав 90 000—120 000 0,4—0,6 (Pt 76,7%, Со 23,3%) 150 000—240 000 0,3—0,5 Постоянные магниты, изготовленные из железо-никелево-алю- миниевых, платино-кобальтового и платино-железного сплавов, обладают относительно большой энергией. Например, в одном кубическом сантиметре постоянного магнита, изготовленного из магнико, содержится магнитной энергии примерно в пятнадцать раз больше, чем в постоянном магните из вольфрамовой стали. Поэтому постоянные магниты из сплавов альнико, магнико и др., значительно меньше по габаритам, чем обычные постоянные маг- ниты из вольфрамовой, кобальтовой стали и т. д., что исключи- тельно важно при конструировании электромагнитных механиз- мов. § 79. ЗАКОНЫ ДЛЯ МАГНИТНЫХ ЦЕПЕЙ Закон Ома для магнитной цепи. Магнитной цепью называется совокупность физических тел, по которым замыкается магнитный поток, создаваемый токами в этой цепи. Магнитная цепь в об- щем случае^ состоит из самых разнородных физических тел, на- пример металлов (сталь, железо, медь, алюминий и др.), разного рода изоляционных прослоек, воздушных зазоров и т. д. Магнитная цепь, в которой магнитный поток нигде не развет- вляется, называется простой магнитной цепью. При наличии же 16-1377 241
Рис. 127. Простая магнит- ная цепь со стальным сер- дечником где Н— напряженность ответвлений магнитного потока магнитная цепь называется раз- ветвленной. Рассмотрим магнитный режим в простой магнитной цепи, состоящей из стального круглого коль- ца, на который равномерно нане- сена обмотка, состоящая из w вит- ков проволоки (рис. 127). Площадь поперечного сечения стального сер- дечника заданной кольцевой катушки равна S, а средний диаметр кольца ра- вен rfcp. Если по обмотке данной катушки пропустить постоянный ток, то внутри стального сердечника катушки возник- нет однородное магнитное поле, напря- женность которого равна магнитного поля в амперах на метр; I—ток, протекающий по обмотке катушки в амперах; /ср—длина средней линии кольца в метрах (показана на рис. 127 пунктиром); — число витков обмотки катушки. Магнитная индукция .в сердечнике катушки равна 4ср где В— магнитная индукция в вольт-секундах на квадратный метр; р.— магнитная проницаемость сердечника катушки в генри на метр. Так как внутри данного стального сердечника магнитное поле однородно, то величина магнитного потока, проходящего через любое поперечное сечение кольцевого сердечника, равна Ф = В5 = -^5, (122) *ср где Ф— магнитный поток в вольт-секундах; 5— площадь поперечного сечения стального сердечника в квадратных метрах. , Преобразуем формулу (122), придав ей Следующий вид: Ф = -^. (123) {15 242
Величина wl, стоящая в числителе формулы, называется намагничивающей силой (магнитодвижущей силой) катушки F = wl, (124) где F—намагничивающая сила катушки; w—число витков обмотки катушки; /—ток, протекающий по обмотке катушки. Из формулы (124) следует, что намагничивающая сила F катушки равна произведению числа витков w катушки на величину тока / в этих витках. Пользуясь формулой (124), определим размерность намагни- чивающей силы: [F] = [w/] = ампер, т. е. размерность намагничивающей силы в системе единиц МКСА выражается в амперах. В соответствии в этим за единицу измерения намагничиваю- щей силы в системе единиц МКСА принят 1 ампер (1 а, 1 А). Если в формуле (124) положить число витков ш = 1 и ток I = 1 ампер, то получим F = wl = 1 • 1 — 1 а, т. е. намагничивающая сила равна одному амперу, если через контур, состоящий из одного витка, протекает ток, равный одному амперу. Пример 74. Определить намагничивающую силу катушки с током, если она имеет число витков w = 200, по которым протекает ток / = 5 а. Решение. Применяя формулу (124), находим F= wl = 200 • 5 = 1000 а. Намагничивающая сила катушки с током равна произведе- нию числа витков w катушки на величину тока I в этих витках. Это произведение называется ампер-витками катушки. В соответствии с этим единицу измерения намагничивающей силы называют также 1 ампер-виток, так что 1 ампер-виток = 1 ампер. Намагничивающая сила (ампер-витки) катушки является основной причиной создания магнитного потока в магнитных цепях. Выражение -^г, стоящее в знаменателе формулы (123), называется магнитным сопротивлением магнитной цепи: = (125) где Rm — магнитное сопротивление магнитной цепи: 16* 243
Пользуясь формулой (125), выясним размерность магнитного сопротивления в системе единиц МКСА: гп 1 = Г /сР ~1 =____метр_________ 1 = 1 l/bwJ LuS J ом-секунда ом-сек генри’ ------- кв. метр г метр---------------------------г т. е. размерность магнитного сопротивления в системе единиц МКСА выражается величиной, обратной генри. Из формулы (125) следует, что магнитное сопротивление маг- нитной цепи прямо пропорционально длине /ср магнитопровода, обратно пропорционально площади сечения S его и зависит от магнитной проницаемости р. материала сердечника катушки. Чем больше р., тем меньше при всех прочих одинаковых условиях магнитное сопротивление катушки. Теперь мы сможем записать формулу (123) в сокращенном виде; Ф = #. (126) Формула (126) является математическим выражением основ- ного закона для* магнитной цепи. Основной закон для магнитной цепи формулируется следую- щим образом: магнитный поток Ф в магнитной цепи прямо пропорционален намагничивающей силе F и обратно пропор- ционален магнитному сопротивлению Rm этой цепи. Нетрудно видеть, что основной закон для магнитной цепи аналогичен закону Ома для электрической цепи. Здесь анало- гом намагничивающей силы F является электродвижущая сила Е, аналогом магнитного сопротивления Rm — электриче- ское сопротивление /?, а аналогом магнитной проницаемости р. — удельная проводимость 7 проводника. В соответствии с этим формулу (126) называют по аналогии с формулой закона Ома для электрической цепи формулой закона Ома для магнитной цепи. Второй закон Кирхгофа для магнитной цепи. Допустим, что на замкнутом стальном сердечнике (рис. 128) помещены три ка- тушки с числами витков, равными соответственно w2 и w$. Постоянный ток, протекающий по первой катушке, равен Д, по второй —12 и по третьей — /3. Направление токов в обмотках катушек отмечено стрелками, нанесенными на витках этих ка- тушек. Сердечник состоит из четырех участков, соединенных по- следовательно друг с другом. Первый участок имеет длину 1\ и площадь поперечного сечения Sb второй участок — длину 12 и площадь поперечного сечения S2 и т. д. По правилу правой руки первая и третья катушки создают магнитные поля одного направления (в направлении движения часовой стрелки), а вторая катушка создает встречное магнит- 244
ное поле (против движения часовой стрелки). Все три катушки совместно создают некоторое результирующее магнитное поле, средняя магнитная линия которого показана пунктиром. Направ- ление ее здесь совпадает с направлением движения часовой стрелки. Магнитный поток в рассматриваемой цепи можно определить, исходя из закона Ома для магнитной цепи, который в данном случае может быть сформулирован так: магнитный поток в последовательной магнитной цепи прямо пропорционален алгебраической сумме на- магничивающих сил (ампер-витков), действующих в цепи, и обратно пропорционален сумме маг- нитных сопротивлений всех ее участков: ф = 4?-. (127) где Ф77-магнитный поток; EW— алгебраическая сумма намагничивающих сил .(ампер- витков) ; ^Rm—сумма магнитных сопротивлений отдельных участков последовательной магнитной цепи. Представим формулу (127) в развернутом виде: Ф = , (128) 245
где ри, рь2, |1з и |14 — магнитные проницаемости соответственно первого, второго, третьего и четвертого участков магнитной цепи. Из формулы (128) следует, что Ф/j I Ф^2 I Ф^З t Ф/4 Г Г I Г 1^2 P-3S3 P-4S4 1 1 2 21 8 * * 3’ или, иначе, B\l\ . B2l2 . Въ13 I ^4/4 T r I r H—— H—— + —— W, + w3/o, Pl P-2 P*3 Pd 11 2 2 r 3 3, так как n Ф1 о ________ Фг • ^1 = ^-; б2=-^ит,д. Но, как известно, В^ __ Т_г В% — = пл -- =Н2 и т. д., Р1 Х Р2 2 ’ где Нь Н2 ит. д. — напряженности магнитного поля соответ- ственно на первом, втором, третьем и четвертом участках маг- нитной цепи. Следовательно, равенству (128) можно придать такой вид: HJt + Н212 4- Ня13 + H4lt = w/i — w2/2+ wsf3. (129) Величины 7/iZi, H2h и т. д., входящие в левую часть послед- него равенства, называются магнитными напряжениями и обо- значаются соответственно символами С7М1, &м2 й т. д. Выясним размерность магнитного напряжения: И#] = • метр = ампер, или, иначе, rLr/1 ампер-виток [HI] =-----------метр = ампер-виток. В соответствии с этим за единицу измерения магнитного на- пряжения в системе единиц МКСА принят 1 ампер (1 ампер- виток) . Магнитное напряжение HI — величина алгебраическая. Если при обходе по магнитной цепи будем перемещаться на данном участке ее в направлении магнитного потока, то магнитное на- пряжение на этом участке необходимо будет брать с положи- тельным знаком. Наоборот, если при обходе по магнитной цепи будем перемещаться на данном участке навстречу магнитному потоку, то магнитное напряжение на этом участке будем брать с отрицательным знаком. 246
Теперь равенство (129) мы можем сформулировать так: алгебраическая сумма намагничивающих сил (ампер-витков) в данной магнитной цепи (магнитном контуре) равна алгебраи- ческой сумме магнитных напряжений (ампер-витков), расходуе- мых на всех участках этой цепи (маг- нитного контура). Нетрудно видеть, что формула (129) аналогична формуле второго закона Кирхгофа для . электрической цепи, где аналогом алгебраической суммы намагничивающих _сил (ампер-витков) ТсоА является алгебраическая сумма электродвижущих сил ££, а аналогом алгебраической суммы магнитных на- пряжений £Н1 (ампер-витков, расхо- дуемых на участках цепи) — алгебраи- ческая сумма падений напряжения I/г Рис m Разветвленная на отдельных участках электрической магнитная цепь цепи. В соответствии с этим форму- лу (129) называют по аналогии с формулой второго закона Кирх- гофа для электрической цепи формулой второго закона Кирхгофа для. магнитной цепи. Первый закон Кирхгофа для магнитной цепи. Разветвленной магнитной цепью называется цепь, в которой магнитный поток распределяется по нескольким направлениям. На рис. 129 пока- зана разветвленная магнитная цепь, где общий магнитный по- ток Ф, создаваемый намагничивающей силой катушки, распре- деляется по двум направлениям. Так как магнитные линии замкнуты на себя, то общий маг- нитный поток, создаваемый намагничивающей силой катушки, равняется сумме магнитных потоков, имеющихся в параллельных ветвях: Ф = ФХ+Ф2, (130) где Ф— общий магнитный поток в неразветвленном участке магнитной цепи; Ф|ИФ2—магнитные потоки соответственно в первой и второй параллельных ветвях. Нетрудно видеть, что формула (130) аналогична формуле первого закона Кирхгофа для разветвленной электрической цепи, где аналогом магнитного потока Ф является ток I. В соответ- ствии с этим формулу (130) называют по аналогии с формулой первого закона Кирхгофа для электрической цепи формулой пер- вого закона Кирхгофа для магнитной цепи. Распределение магнитного потока в параллельных магнитных цепях. Магнитные напряжения на параллельных магнитных вет- вях равны друг другу: > (131) 247
гдеЛ^и/^— напряженности магнитного поля соответственно в первой и второй параллельных ветвях; и /2— средние длины соответственно первого и второго участков параллельных ветвей. Но, как известно, Подставив эти* выражения для Н\ и Н2 в равенство (131), получим Ф ._А_ =ф ._А_ 1 hSi 2 p2V или, иначе, = ^2 где Фх и Ф2 — магнитные потоки соответственно первой параллельных ветвей; RmX и ^т2 — магнитные сопротивления соответственно второй параллельных ветвей. Из равенства (132) получаем: ф2 “ Rml ’ т. е. магнитный поток распределяется в лельных ветвях обратно пропорциональ нитным сопротивлениям этих ветвей. (132) и второй первой и (133) парад- но м а г- § 80. РАСЧЕТ ПРОСТОЙ МАГНИТНОЙ ЦЕПИ Рассчитать магнитную цепь — это значит по заданному маг- нитному потоку (магнитной индукции), кривым намагничивания и размерам магнитной це- пи определить намагничиваю- щую силу (общее число ам- пер-витков) , необходимую для создания заданного ре- жима в магнитной цепи. Допустим, что задана простая магнитная цепь, со- стоящая из пяти последова- тельно соединенных участков цепи, размеры которых (дли- ны /1, Z2, Z3, U и /о, а также площади поперечных сече- ний Si, S2, S3, S4 и So) показаны на рис. 130. Ма- териалом первых четырех 248
участков цепи является электротехническая сталь, кривая намагничивания которой известна (рис. 131). Пятым участком магнитной цепи служит воздушный зазор, имею- щий толщину /о и площадь поперечного сечения So. Требуется определить, ка- кую намагничивающую си- лу, или, иначе, какое чис- ло ампер-витков должна иметь катушка с током, положенная на один из участков цепи, чтобы по- лучить в. этой цепи маг- нитный поток Ф. Рис. 131. Кривая намагничивания электро- технической стали Порядок расчета заданной магнитной цепи сводится к сле- дующему: 1. По заданному магнитному потоку Ф и поперечным сече- ниям Si, S2, S3, S4 и So отдельных участков цепи определяют для этих участков величины магнитных индукций по формулам о __ Ф1 . р __ Фг . р _ Фз . р __ Ф4 р ____ Ф 2. Пользуясь кривой намагничивания электротехнической стали (рис. 131), находят напряженности магнитного поля /7Ь //2, Н3 и Н4 по соответствующим им величинам магнитной ин- дукции В], B2l В3 и В4. 3. Напряженность магнитного поля в воздухе Hq определяют по формуле н0=-^=4о-=4^^о=8'1о55о- ° 4тс 4тс и — и ”10’' 4. Определив напряженности магнитного поля во всех уча- стках магнитной цепи, находят магнитные напряжения (ампер- витки), расходуемые на отдельных участках цепи: > ^2^2> #3*3^4^4 И 5. Затем определяют намагничивающую силу F (полные ампер-витки) по формуле F = 4- Н212 + ^з^з 4“ Н414 4- HqIq. 249
6. Но намагничиваю- щая сила F, как из- вестно, равна произ- ведению числа витков w катушки на величину тока I в них, т. е. F — 'wl. Следовательно, зада- ваясь током /, можно определить необходи- мое число витков ка- тушки Пример 75. В воздушных зазорах магнитной цепи (рис. 132) необходимо со- здать магнитный поток Ф= 0,00125 в-сек. Верхняя часть сердечника магни- топровода П-образной формы выполнена из электротехнической стали марки Э11, нижняя часть — из чугуна. Кривые намагничивания этих мате- риалов приведены на рис. 131 и 133. Верхняя часть магнитопровода отде- лена от нижней двумя воздушными зазорами, имеющими одну и ту же тол- щину /0 и одинаковые поперечные сечения, равные поперечному сечению стального сердечника верхней части магнитопровода. Размеры магнитопро- вода на рис. 132 указаны в миллиметрах. На каждый из вертикальных стержней верхней части магнитопровода надета катушка. Катушки одина- ковы как по количеству витков, так и по сопротивлению их обмоток. Соеди- нены они друг с другом последовательно, так что через их обмотки проте- кает один и тот же ток. Требуется определить, какое число витков должна иметь каждая из катушек, если эти катушки создают в магнитопроводе маг- нитные поля одинакового направления, а ток, протекающий через их об- мотки, I = 0,5 а. Решение. 1) Определяем длины участков средней магнитной линии (пунктирная линия на рис. 132) магнитопровода. а) Длина верхней части магнитопровода равна Zi = 300 + 250 + 300 = 850 ^ = 0,85 м. б) Длина нижней части магнитопровода равна /2 = 250 мм = 0,25 м. в) Толщина двух воздушных зазоров равна 2Z0 = 2 • 0,5 = 1 мм — 0,001 м. 2) Вычисляем площади поперечных сечений отдельных участков магни- Топровода, которые в данном случае все равны друг другу: = S2 = 50 = 5 = 25 • 100 = 2500 мм2 = 0,0025 м2. 250
3) Находим величины маг- нитной индукции магнитного поля во всех участках магни- топровода; в данном сЛучае они всюду одинаковы из-за одинаковых значений магнит- ного потока Ф и площадей поперечных сечений: = В2 = в^ = — = о ____ 0,00125 _е в-сек ~ 0,0025 • 4) Пользуясь кривыми на- магничивания (рис. 131 и 13’3), выясняем, какая величина на- пряженности магнитного поля должна быть в сердечнике магнитопровода. а) В верхней части магнитопровода (в электротехнической стали марки Э11) магнитной индукции В\ = 0,5 —— соответствует напряженность маг- нитного поля Н\ = 175 —. м б) В нижней части магнитопровода (в чугуне) магнитной индукции В = 0,5 —соответствует напряженность магнитного поля Hi = 2200 . в) В воздушных зазорах напряженность магнитного поля =—==4-ю5 0 Ро 10’ а м * 5) Определяем магнитные напряжения (ампер-витки), расходуемые на отдельных участках магнитной цепи. а) В верхней части магнитопровода Я,/, = 175-0,85 s 150 ампер (ампер-витков). б) В нижней части магнитопровода Я2/2 = 0,25-2200 = 550 ампер (ампер-витков) в) В воздушных зазорах Яо-2/о = 400 000-0,001 =400 ампер (ампер-витков). 6) Находим намагничивающую силу (полное число ампер-витков) в маг- нитной цепи: F = + Н212 + 2Н010 = 150 + 550 + 400 = = 1100 ампер (ампер-витков). 7) Определяем число витков двух катушек «месте взятых: w = -С- = = 2200 витков. 4 v,D 251
Рис. 134. Магнитная цепь с магнитным рассеянием Число витков каждой катушки равно = 11С0 витков. Примечание. Если бы в данной магнитной цепи происхо- дило магнитное рассеяние, т. е. часть магнитного потока, созда- ваемого катушками с током, за- мыкалась бы через воздух, а не через магнитную цепь, то необ- ходимо было бы учесть эту утеч- ку магнитного потока с помощью коэффициента рассеяния, кото- рый обычно определяется экспе- риментальным путем. Для маг- нитных цепей, составленных только из ферромагнитных материалов и имеющих малые воздушные зазоры, коэф- фициент рассеяния принимается равным единице. Пример 76. В воздушном зазоре магнитной цепи, изображенной на рис. 134, необходимо создать магнитную индукцию Во = 0,4----— . Определить, какой ток должен протекать по виткам катушки, если число витков катушки ф = 1000. Материал магнитопровода — электротехническая сталь марки Э11, для которой кривая намагничивания приведена на рис. 131. Длина воздуш- ного зазора /о = 0,002 средняя длина магнитной линии в стали равна /ст= 1,2 м, площади поперечного сечения стали и воздушного зазора оди- наковы и равны S = 0,0025 м2. Коэффициент магнитного рассеяния К =1,25. Решение. Определяем полезный магнитный поток Фо, т. е. тот по- ток, который проходит через воздушный зазор: Фо = 505 = 0,4-0,0025 = 0,001 в-сек. Определяем полный магнитный поток, создаваемый заданной катушкой: Ф = /<Ф0 = 1,25-0,001 =0,00125 в-сек. Находим магнитную индукцию в стали: п ____ Ф ____ 0,00125 _р г в-сек ^ст — "у — о,ОО25 “ м2 * По кривой намагничивания (рис. 131) для магнитной индукции г в-сек Вст = 0,5—находим напряженность магнитного поля, которая в дан- ном случае равна Нст = 175 —. м Находим ампер-витки, расходуемые в стали: Яст^ст = 175-1,2 = 210 а (ампер-витков). Теперь определяем напряженность магнитного поля в воздушном зазоре: Н — — °’4 — 106 а 0 Р'О п м То7 252
Находим ампер-витки, расходуемые в воздухе: Яо/о = • 0,002 = 635 (ампер-витков). Намагничивающая сила катушки равна + HqIq = 210 + 635 =845 (ампер-витков). Ток в катушке должен быть установлен следующей величины: /= — = 0,345 а. w 1000 ’ Из рассмотренного примера видно, что число ампер-витков, израсходованных в воздушном зазоре, больше, чем в стали, при- мерно в три раза, в то время как длина воздушного зазора меньше длины средней магнитной линии встали в 600 раз. Это объясняется тем, что магнитная проницаемость воздуха значи- тельно меньше магнитной проницаемости стали. § 81. РАСЧЕТ РАЗВЕТВЛЕННОЙ МАГНИТНОЙ ЦЕПИ Рассчитать разветвленную магнитную цепь — это значит по заданному магнитному потоку (магнитной индукции) в каком- либо участке разветвлений магнитной цепи, по размерам уча- стков магнитной цепи и по кривым намагничивания рассчитать число ампер-витков (намагничивающую силу), необходимое для получения заданного магнитного режима. В основу расчета разветвленной магнитной цепи положены первый и второй законы Кирхгофа для магнитной цепи. Покажем порядок расчета разветвленной магнитной цепи на примере. Рис. 135. Разветвленная магнитная цепь 253
Пример 77. Допустим, что задана разветвленная магнитная цепь (рйс. 135). На среднем стержне ее посажена катушка, создающая намагничивающую силу (ампер-витки) в этой цепи. В правом стержне цепи имеется воздуш- ный зазор, в котором необходимо создать магнитный поток Фг Размеры магнитопровода на рис. 135 указаны в сантиметрах. Материалом магнитной цепи является электротехническая сталь, кривая намагничивания которой показана на рис. 131. Магнитным рассеянием можно пренебречь. Решение. 1) Определяем средние длины участков магнитной цепи: а) для правого стержня 4 = 2-15 +20 — 4 — 0,05 = 45,95 см «0,46 м\ б) для левого стержня /2 = 2-15 + 20 —4 = 46 гл/= 0,46 м\ в) для среднего ст^жня /3 = 20 — 2 = 18 см = 0,18 м\ г) для воздушного зазора /0 = 0,05 см = 0,0005 л/. 2) Определяем площади поперечных сечений отдельных участков маг- нитной цепи: а) для правого стержня S, = 2 • 2 = 4 см? = 0.С004 м?-, б) для левого стержня 52 = 2 • 2 = 4 см2 = 0,0004 м2; в) для среднего стержня S3 = 4 • 2 = 8 см? = 0,0008 м\ г) для воздушного зазора 50 = 2 • 2 = 4 см? = 0,0004 м\ 3) Находим магнитную индукцию в правом стержне: , __ Ф1 ___ 2-10* ____п _ в-сек 1 — 5? “ 4-10—4 “"U’° л/2 • 4) Пользуясь кривой намагничивания (рис. 131), находим для Bi = = 0,5 в С^- соответствующую ей напряженность магнитного поля Н\ = 175 —. л/2 м 5) Далее определяем магнитное напряжение в правом стержне: НХ1Х = 175-0,46 = 80 а (ампер-витков). 6) Магнитная индукция в воздушном зазоре такая же, как и в правом стержне, т. е. 254
а поэтому напряженность магнитного поля в воздушном зазоре будет равна ^о = —= ¥- = 4-105-^. и |л0 4тс м То7 7) Далее определяем магнитное напряжение в воздушном зазоре: Н01й = 4-105-5-10-4 — 200 а (ампер-витков). 8) Для определения напряженности магнитного поля в левом стержне составим уравнение применительно ко второму закону Кирхгофа для внеш- него контура, обходя его по часовой стрелке: Hth + /7о/о - Н212 = 0, откуда/находим, что и Hik + М/о + 200 rm а =------------------------Т,---- = 0,46 " = 610 — • 9) По кривой намагничивания (см. рис. 131) находим, что напряжен- ности магнитного поля /72 = 610 — соответствует магнитная индукция 10) Теперь находим магнитный поток в левом стержне: Ф2==5252 = 1,1 -4-10"4 = 4,4- 10't в-сек. 11) Магнитный поток в среднем стержне найдем, применив формулу первого закона Кирхгофа для магнитной цепи: Ф3 = Ф1 4-Ф2 = 2-10“4 4-4,4-10-4 = 6,4-10“4 в-сек. 12) Далее находим магнитную индукцию в среднем стержне: 13) По кривой намагничивания (рис. 131) находим, что магнитной ин- в-сек дукции —- 0,8 —соответствует напряженность магнитного поля Н3 = 320 — . я 14) Магнитное напряжение на среднем стержне равно /73/3 = 320-0,18^ 58 а (ампер-витков). 15) Находим намагничивающую силу (полное число ампер-витков) маг- нитной цепи, составив уравнение для правого контура в соответствии со вто- рым законом Кирхгофа: р = Нх1} 4- /70/0 4- /73/3 = 80 4- 2С0 4- 58 = 338 а (ампер-витков,). « 16) Теперь можно определить число витков катушки: F 338 w = — = Q-j: = 676 витков. 255
Примечание. Если бы мы определяли намагничивающую силу, ис- ходя из уравнения второго закона Кирхгофа, составленного для левого кон- тура, то получили бы такую же величину намагничивающей силы: F = H2l2 + ЛЦ3 = 610 • 0,46 + 320 • 0,18 = 338 а и соответственно то же число витков катушки. § 82. ЭЛЕКТРОМАГНИТЫ Если внутрь катушки с током ввести стальной сердечник, то последний намагнитится и создаст свое собственное магнитное поле, которое, накладываясь на поле катушки, создаст некоторое результирующее магнитное поле. Так как магнитная проницае- Рис. 136. Электро- магнит стержневой формы Рис. 137. Под- ковообразный электромагнит мость стали по сравнению с магнитной проницаемостью воздуха очень велика, то и магнитная индукция в стали будет значи- тельно больше, чем в воздухе. Катушки со стальными сердечниками, называемые электро- магнитами, обычно обладают более мощными магнитными по- лями, чем катушки без стали. На рис. 136 показан электромагнит стержневой формы с ци- линдрическим стальным сердечником, на который нанесена об- мотка, состоящая из w витков проволоки. Если через обмотку электромагнита пропустить ток, то в ка- тушке возникнет магнитное поле, которое намагнитит стальной сердечник, создав в нем некоторую магнитную индукцию. Поляр- ность электромагнита определяется правилом правой руки или правилом буравчика, применяемым для катушки. Приблизим к торцовой поверхности сердечника электромаг- нита плоский кусок стали (якорь) так, чтобы плоский торец сер- дечника и плоскость якоря расположились параллельно друг 256
другу. Линии магнитной индукции из северного полюса торца сердечника будут проникать в тело якоря; пройдя его, они воз- вратятся к южному полюсу сердечника и через последний зам- кнутся на себя. Под влиянием магнитного поля электромагнита якорь намагнитится так, что на его плоскости, обращенной к се- верному полюсу электромагнита, возникнет южный магнитный полюс. Разноименные магнитные полюсы сердечника и якоря бу- дут взаимодействовать, стремясь притянуться друг к другу. Сила, с которой электромагнит притягивает якорь, называется подъемной силой его. Подъемная сила электромагнита стержневой формы опреде- ляется формулой F = (134) ол где F—подъемная сила электромагнита в ньютонах; В—магнитная индукция между плоскостями сердечника электромагнита и якоря в вольт-секундах на квадрат- ный метр; 5 — площадь торца сердечника в квадратных метрах; л = 3,14. Пример 78. Магнитная индукция в воздушном зазоре между сердечником электро магнита и якорем В = 0,5 в-сек! м2, а площадь торца сердечника S = 25,12 см2. Определить подъемную силу электромагнита. Решение. По формуле (134) находим г. B2S 1А7 0,52-25,12-10“4 1А7 ОСА F = • 107 = —— 107 = 250, ньютонов. Для подковообразного электромагнита (рис. 137), который одновременно притягивает якорь двумя полюсами, подъемная сила увеличивается в два раза по сравнению со стержнеобраз- ным электромагнитом, т. е. F — S-10’. (135) Электромагниты нашли широкое применение в приборах, аппаратах и разного рода электромагнитных механизмах. Рас- смотрим в качестве примера применение электромагнита в элек- трическом звонке постоянного тока. Электрический звонок постоянного тока (рис. 138) состоит из подковообразного электромагнита /, перед полюсами которого расположен укрепленный на пружине якорь 2 электромагнита, снабженный на свободном конце ударником 3. Пружина 4 при- жимается к неподвижно укрепленному винту 5. При прохождении тока по обмотке электромагнита последний притягивает к себе якорь и этим разрывает цепь между пружи- ной 4 и контактным винтом 5. После разрыва цепи ток в элек- 17—1377 257
тромагните исчезает, й электромагнит теряет свои магнитные свойства. Вследствие этого якорь силой пружины отрывается от сердечнику электромагнита и возвращается в исходное положе- ние. Но как только пружина 4 прикоснется к контактному винту 5, цепь вновь замкнется, возникнет электрический ток в обмотке электромагнита и последний, намагнитившись, притянет якорь, Рис. 138. Электрический звонок по- стоянного тока: 1 — подковообразный электромагнит; 2 — якорь; 3 — ударник; 4 — пружина; 5 — винт разрывая цепь, и т. д. Таким образом, якорь электромагнита будет совершать колебательные движения некоторой определен- ной частоты между контактным винтом и сердечником электро- магнита. Ударник 5, перемещаясь вместе с якорем, бьет по чашке звонка, заставляя его издавать звук.
ГЛАВА XVII МЕХАНИЧЕСКОЕ ДЕЙСТВИЕ МАГНИТНОГО ПОЛЯ НА ПРОВОДНИКИ С ТОКАМИ § 83. МЕХАНИЧЕСКОЕ ДЕЙСТВИЕ МАГНИТНОГО ПОЛЯ НА ПРЯМОЛИНЕЙНЫЙ ПРОВОДНИК С ТОКОМ Магнитное поле действует с определенной силой на любой перемещающийся в нем электрический заряд. Если, например, электрон, перемещающийся в пространстве, попадает в магнит- ное поле, то он испытывает на себе действие силы магнитного поля, стремящейся изменить траекторию его движения. На рис. 139 . изображена картина искажения траекто- рии движущегося электро- на в однородном магнит- ном поле, магнитные линии которого направлены из-за плоскости чертежа к нам (точки на чертеже — следы пересечения магнитных ли- ний с плоскостью чертежа). Чем больше скорость Vo движения электрона и чем больше магнитная индук- ция В магнитного поля, тем движущийся электрон. В отличие от в магнитном поле си Рис. 139. Картина искажения прямоли- нейного движения электрона в магнит- ном поле больше сила F, действующая на поля на д в и- электростатического ла, действующая жущийся электрон, направлена не по каса- тельной к силовой линии, а перпендику- ляр н о е й. Если в металлическом проводнике отсутствует электрический ток, то все свободные электроны его движутся в междуатомном пространстве беспорядочно. При внесении этого проводника во внешнее магнитное поле на каждый движущийся электрон 17* 259
Рис. 140. На каждый из движущихся электронов действует сила, направленная перпендикулярно его движению и магнитным линиям будет действовать сила Д направленная перпендикулярно направ- лению его движения и магнитным линиям (рис. 140). Но так как свободных электронов в проводнике имеется громадное число и все они движутся беспорядочно, то результирующая сила, дей- ствующая на них в магнитном поле, в дюбой момент времени равна нулю. Совсем иная картина будет наблюдаться, если в магнитное поле поместить металлический проводник с током (рис. 141). В этом случае свободные электроны движутся общим потоком вдоль проводника направленно, упорядоченно, с некоторой рав- номерной скоростью v. Силы Д действующие на электроны, бу- дут направлены в одну сторону, и результирующая сила F будет равна их геометрической сумме. Под влиянием этой результи- рующей силы F проводник с током будет перемещаться, если она сможет преодолеть сопротивление движению проводника. Исследуем величину этой силы для случая, когда прямолиней- ный проводник с током помещен в однородное магнитное поле. Пусть прямолинейный проводник с током / (рис. 142) поме- щен в однородном магнитном поле. Проводник направлен от нас Рис. 141. Проводник с током в маг- нитном поле испытывает на себе действие механической силы F Рис. 142. Прямолинейный про- водник с током в магнитном поле 260
Рис. 143. Рисунок, поясняющий пра- вило левой руки за плоскость чертежа и перпендикулярно ей. Магнитные линии однородного магнитного поля направлены сверху вниз. Механи- ческая сила F, действующая на проводник с током, будет, на- правлена перпендикулярно направлению тока I и линиям маг- нитной индукции В, т. е. в данном случае влево. Для определения направления механической силы, действую- щей на помещенный в магнитном поле проводник с током, при- меняется правило левой руки: если ладонь левой руки поме- стить в магнитное поле так, чтобы она была направлена на- встречу линиям магнитной индукции и четыре вытянутых пальца ее указывали направление тока в проводнике, то отставлен- ный большой палец покажет направление действия механи- ческой силы на проводник с током (рис. 143). Направление механической силы, действующей на провод- ник с током в магнитном поле, можно объяснить и с точки зрения условных свойств линий магнитной индукции, а имен- но способности их сокращаться по длине, взаимно отталкивать- ся друг от друга при одинако- вых направлениях и взаимно притягиваться при разных на- правления?. Проводник с то- ком, внесенный в магнитное поле, сам создает свое соб- ственное магнитное поле, на- правление которого определяет- ся правилом буравчика, а по- этому линии магнитной индук- ции внешнего поля и поля тока имеют одинаковое направление ное — слева от него.., Следовательно, справа от проводника будем наблюдать сгущение линий магнитной индукции, а слева — разре- жение (рис. 144). Линии магнитной индукции, стремясь сокра- титься по длине и взаимно оттолкнуться одна от другой, будут выталкивать проводник влево, т. е. в ту сторону, где результи- рующее магнитное поле слабее. Если в однородное магнитное поле внесен прямолинейный проводник с током /, расположенный перпендикулярно магнит- ным линиям, то величина механической силы, действующей на этот проводник, определяется формулой справа от проводника и различ- F = BIl, (136) 261
где В—магнитная индукция в вольт-секундах на квадратный метр; F—механическая сила в ньютонах; / — ток в амперах; I — активная длина провода с током в метрах. Под активной длиной провода с током надо понимать ту часть проводника, которая непосредственно находится в магнит- ном поле. Пример 79. В однородное магнитное поле, в котором магнитная индукция В = 1 6помещен прямолинейный проводник с током, Z = 8 а, располо- женный под прямым углом к магнитным линиям. Определить механическую силу F, действующую на этот проводник в поле, если активная длина его Z = 1,2 м. Решение. По формуле (136) находим Р = ВЦ = 1-8 -1,2 = 9,6 ньютона. Если прямолинейный проводник рас- /,Т/1 ’ Г\\‘ f м положен в однородном магнитном поле А/ / * ’ \ 11 ин\\ под Углом а к магнитным линиям Рис. 145. Прямолинейный про- водник с током, расположен- ный под углом а к направле- нию магнитных линий Рис. 144. Проводник с током стремится пере- мещаться в ту сторону, где результирующее поле слабее (рис. 145), то механическая сила, действующая на этот провод- ник, определяется формулой F — BIl-sin а: (137) Пример 80. Прямолинейный проводник с током /= 12 а и активной дли- ной / = 0,8 м помещен в однородное магнитное поле так, что его направле- ние составляет с направлением магнитных линий поля угол а = 30°. Опре- делить магнитную индукцию магнитного поля, если этот проводник испы- тывает на себе действие силы В = 2,4 ньютона. Решение. По формуле (137) находим ___ F ___________ 2,4________________- в-сек //-sin а"“ 12-0,8 0,5 м* 262
§ 84. МЕХАНИЧЕСКОЕ ДЕЙСТВИЕ МАГНИТНОГО ПОЛЯ НА КОНТУР С ТОКОМ Пусть во внешнее однородное магнитное поле внесен плоский, прямоугольной формы, контур с током,у которого две параллель- ные стороны расположены вдоль магнитных линий, а две дру- гие— перпендикулярно им (рис. 146). Применяя формулу (137) F — BH’sina. к каждой стороне контура в отдельности, найдем: 1) 2) 3) на сторону ab действует сила F1 = B//1-sin90° = B//1; на сторону Ьс действует сила F& = ВН2 • sin 0° = 0; на сторону cd действует сила В2 = В11е81п90° = В11^ Рис. 146. Контур с током во внешнем магнитном поле 4) на сторону da действует сила = ВП2 • sin 0° = 0. Согласно правилу левой руки сила Л направлена влево от плоскости контура и перпендикулярно ей. Наоборот, сила F2 плоскости контура и перпендикулярно ей. Обе силы, равные по абсолютной величине, но противоположно направленные, стре- мятся повернуть контур вокруг его оси. Такая система двух сил является парой сил. Вращающий момент ее равен произведению одной из сил на расстояние (плечо) между точками приложения этих сил: направлена вправо от Л4 = Л/2, (138) где F] — сила в ньютонах; /2 — плечо пары сил в метрах. Но поэтому Так как — *>, 263
то M = BIS, (139) где В—магнитная индукция внешнего поля в вольт-секундах на квадратный метр; 5 — площадь контура в квадратных метрах; I — ток в амперах; М — вращающий момент в ньютонометрах. Пример 81. Определить вращающий момент для рамки, имеющей пло- щадь S = 40 сл<2, в обмотке которой протекает ток I = 12 а, если она поме- щена в однородное магнитное поле В = 0,2 и расположена так, что плоскость ее совпадает с направлением магнитных линий поля. Решение. По формуле (139) находим М == BIS = 0,2 • 12 • 0,004 = 9,6 • 10“3 ньютонометров. Если плоскость контура с током не совпадает с направлением магнитных линий (рис. 147), то величина вращающего момента, Рис. 147. Плоскость кон- тура с током не совпадает с направлением магнитных линий действующего на него, определится формулой М = BIS-sin а, (140) где а—угол, составленный нормалью/? (перпендикуляром к плоскости кон- тура) с направлением магнитных линий. Если вместо контура взять плоскую катушку, имеющую w витков, то вра- щающий момент для катушки будет в w раз больше, чем для одного витка: М = BAS*w-sina. (141) Необходимо отметить, что катуш- ка с током всегда стремится установиться во внешнем магнитном поле так, чтобы ее пронизывал максимальный магнитный поток и направление ее собственного магнитного поля и поля внешнего совпали внутри катушки. Свойство рамки с током взаимодействовать с внешним маг- нитным полем широко используется в электротехнике, в част- ности в электродвигателях, в магнитоэлектрических измеритель- ных приборах и т. д. 264
§ 85. МЕХАНИЧЕСКАЯ СИЛА ВЗАИМОДЕЙСТВИЯ ПРОВОДНИКОВ С ТОКАМИ Проводники с токами, расположенные близко один к друго- му, взаимно притягиваются или отталкиваются. Направление си- лы их взаимодействия зависит от пространственного расположе- ния этих проводников и от направления токов в них. Пусть два прямолинейных проводника, по которым проте- кают токи Д и Л, расположены параллельно один другому и так, что эти токи имеют различные направления (рис. 148). А А Рис. 149. Проводники с токами одинаковых направлений притя- гиваются друг к другу Рис. 148. Проводники с токами раз- личных направлений отталкиваются друг от друга Каждый из этих токов создает свое собственное магнитное поле. Направления магнитных линий этих полей определяются прави- лом буравчика. Из рис. 148 видно, что магнитные линии в про- странстве между проводниками имеют одинаковые направления, а поэтому стремятся оттолкнуться одна от другой, вследствие чего проводники с токами тоже будут отталкиваться один от другого. На рис. 149 показаны два параллельных проводника с тока- ми одинакового направления. В этом случае направления маг- нитных линий в пространстве между проводниками различны, что приводит к взаимному притяжению их, а следовательно, и к взаимному притяжению проводников с токами. Таким образом, проводники с токами одинаковых направле- ний притягиваются, а различных направлений — взаимно оттал- киваются. Теперь выясним, какова величина силы взаимодействия про- водников с токами. Допустим, что прямолинейный проводник с током А очень длинный. Тогда магнитная индукция в точках, расположенных в воздухе на расстоянии а метров от этого про- водника, будет равна = <142> 265
где В— магнитная индукция в вольт-секундах на квадратный метр; /,— ток в амперах; — магнитная проницаемость вакуума (практически воз- духа); а— расстояние от оси проводника в метрах; Н— напряженность магнитного поля в амперах на метр. Пусть второй прямолинейный проводник с током' 1г, располо- женный параллельно первому проводнику, находится от него на расстоянии а метров. Так как второй проводник находится в магнитном поле первого проводника с током Ц, то механическая сила, действующая на второй проводник, может быть определе- на согласно формуле (136) Р = В^1^г> но так как то г=2^ 10_7 (143) Пример 82. Определить силу F, действующую на прямолинейный про- водник с током /2 = 15 а, длина которого /2 = 0,8 м, если он расположен на расстоянии а = 0,05 м от другого прямолинейного весьма длинного провода, по которому течет ток Zj = 12 а. Оба проводника расположены параллельно один другому. Решение. с 2-12-15-0,8-10—7 F = ——--------=----------------= 57,6 • 10 5 ньютонов. а 0,0э ’ Если в формуле р == |Q_7 а положить /г=1 м, а = 2 м и Л = /2 — 1 а, то с 2-1 (а)1 (а) , , , 1П_7 Г = = 10 ’ ньютона. На основании последнего соотношения единице величины то- ка (1 амперу) можно дать такое определение: один artnep есть величина неизменяющегося электрического тока, который, проте- кая по каждому из двух параллельных прямолинейных прово- дов ничтожно малого кругового сечения, расположенных на рас- стоянии двух метров друг от друга в вакууме, создает между этими проводниками силу взаимодействия в одну десятимилли- онную долю ньютона на каждый метр длины. 266
§ 86. РАБОТА\СИЛ МАГНИТНОГО ПОЛЯ Механические силы, действующие на проводник с током во внешнем магнитном поле, перемещая этот проводник, совершают некоторую работу. Пусть внешнее однородное магнитное поле направлено от нас за плоскость чертежа крестиками на рис. 150. В этом магнитном поле имеются два ого- ленных металлических проводни- ка, расположенных параллельно друг к другу и присоединенных к зажимам источника электриче- ской энергии с постоянной элек- тродвижущей силой. Цепь зам- кнута легким пустотелым метал- ей, что обозначено X X I X X X JJ X fc— а — лическим проводником цилиндри- Рис 150> Под действием силы F ческой формы, который легко проводник с током перемещается может катиться по оголенным (катится) по оголенным проводам проводам. Если источник электрической энергии включен в цепь, то в цилиндрическом подвижном проводнике возникает ток / и на него действует сила F = BIl} где I — активная длина подвижного проводника в метрах. Направление силы F можно определить по правилу левой руки. Если под влиянием силы F подвижный проводник будет перемещаться, то работа источника электрической энергии при переносе проводника на расстояние а будет равна А = Fa = ВПа, (144) где А — работа в джоулях. Пример 83. В однородном магнитном поле, обладающем магнитной индук- цией б = 0,5 а , перемешается проводник с током /=12 а, имеющий активную длину I = 0,4 м. Определить работу, совершаемую источником электрической энергии, если проводник переместился на расстояние а = 0,25 м. Решение. По формуле (144) находим А = ВПа = 0,5-12 -0,4-0,25 = 0,6 дж. Формулу (144) можно представить в несколько ином виде, выразив ее через ток / и магнитный поток Ф. А = ВПа = BIS = /Ф, (145) где Ф—магнитный поток, пересеченный проводником с током / при его движении в магнитном поле. 267
Из формулы (145) следует, что работа механической силы при переносе проводника с током в магнитное поле равняется произведению тока /, протекающего в этом проводнике, на маг- нитный поток Ф, перерезанный проводником при его движений в магнитном поле. Пример 84. Прямолинейный проводник с током I = 15 а, перемещаясь в магнитном поле, перерезал магнитный поток Ф = 4 в-сек. Требуется опре- делить работу, затраченную источником электрической энергии на переме- щение этого проводника. Решение. По формуле (145) находим, что Л = /Ф = 15-4 = 60 дж.
ГЛАВА XVIII ЭЛЕКТРОМАГНИТНАЯ ИНДУКЦИЯ § 87. ИНДУКТИРОВАННАЯ ЭЛЕКТРОДВИЖУЩАЯ СИЛА В ПРЯМОЛИНЕЙНОМ ПРОВОДНИКЕ, ПЕРЕМЕЩАЮЩЕМСЯ В МАГНИТНОМ ПОЛЕ Предположим, что прямолинейный проводник, например ку- сок медной проволоки ab (рис. 151), приводимый в движение внешней механической силой, перемещается в однородном маг- нитном поле с постоянной скоростью v. Направление магнитных линий на рис. 151 показано крестиками, т. е. магнитные ли- нии уходят за плоскость черте- жа перпендикулярно ей, а про- водник ab движется перпенди- кулярно направлению магнит- ных линий, в направлении век- тора скорости V. При заданном поступатель- г/ хххххххх + - -----г х --------- х + Ест “ Ь ном движении проводника все его свободные электроны будут вместе с ним поступательно пе- ремещаться в магнитном поле ХХХХХХХХ Рис. 151. Перемещение проводника ab в магнитном поле при помощи внешней силы с той же скоростью v. Но, как известно, каждый электрон, движущийся во внеш- нем магнитном поле, испытывает на себе действие силы F, стре- мящейся отклонить его в сторону от прямолинейного движения (см. рис. 139). Величина этой силы пропорциональна скорости v поступательного движения электрона и магнитной индукции В внешнего магнитного поля и зависит также от угла, под кото- рым электрон движется по отношению к направлению магнит- ных линий. Если он движется в направлении, перпендикулярном магнитным линиям, то сила, действующая на него, будет макси- мальной, и она будет равна нулю, если электрон движется в на- правлении магнитной линии. Следовательно, все свободные электроны проводника, движу- щиеся поступательно вместе с ним, будут испытывать на себе 269
действие сторонних сил, стремящихся/отклонить их в сторону от первоначального направления их /поступательного движения. Под влиянием этих сил они будут/перемещаться с одного конца проводника на другой, и в результате один конец проводника зарядится отрицательным электричеством (избыток электронов), а другой — положительным (недостаток электронов). Разъеди- ненные электрические заряды создадут в проводнике электро- статическое поле, и на концах проводника возникнет разность потенциалов (напряжение U). Приток электронов на один из концов проводника будет про- должаться до тех пор, пока не уравновесятся противоречивые силы. С одной стороны, — это сторонние силы EQT, стремящиеся перемещать движущиеся электроны в магнитном поле, и, с дру- гой стороны,— это силы электростатического поля Еэ разъеди- ненных электрических зарядов, размещенных по концам провод- ника, стремящиеся вернуть электроны на прежние места (см. рис. 151). Сторонние силы, перенося электрические заряды с одного конца проводника на другой, навстречу действию сил электри- ческого поля, будут совершать некоторую работу за счет той энергии, которую расходует механическая сила, перемещающая этот проводник в магнитном поле. Эта работа, если не прини- мать во внимание некоторых потерь энергии, например на выде- ление тепла, пойдет на образование потенциальной энергии элек- тростатического поля разобщенных электрических зарядов в проводнике. Величина этой энергии, приходящаяся на единич- ный заряд, численно равна напряжению U на концах проводника. Если сторонние силы, перенося электрический заряд q с од- ного конца проводника на другой, затратили работу, равную Л, то работа, совершаемая ими при переносе единичного заряда, будет в q раз меньше: * = (146> где Е — величина работы, отнесенная к единичному заряду, — индуктированная электродвижущая сила. Если в формуле (146) работа А выражена в джоулях, а элек- трический заряд q — в кулонах, то индуктированная электродви- жущая сила будет выражена в вольтах. Пример 85. Сторонние силы, возникшие в проводнике, движущемся во внешнем магнитном поле, перемещая с одного конца этого проводника на другой количество электричества <7 = 0,001 к, совершают работу Д = 0,05дяс. Определить индукторов энную электродвижущую силу в проводнике, Решение. По формуле (146) находим Е — — — 0’05 — so а С q 0,001 “ W в' Когда наступит равновесие между сторонними силами и си- лами электрического поля в проводнике, индуктированная элек- 270
тродвижущая сила в нем его концах: численно равна напряжению на Следует отметить, что индуктированную электродвижущую силу нельзя сравнивать с какой-либо силой, например с дейст- вием насоса, качающего воду, так как электродвижущая сила характеризует собой только ту работу, которую совершают сто- ронние силы, перенося единичный заряд по проводнику. Итак, если проводник переме- щается в магнитном поле, то на его концах накапливаются разноименные электрические заряды, электрическое поле которых обладает некоторым запасом потенциальной электриче- ской энергии. Внутри проводника со- здаются поле сторонних сил и элек- трическое поле разобщенных элек- трических зарядов, взаимно уравно- вешивающие друг друга. Теперь допустим, что провод- ник ab, в котором индуктирована Рис. 152. Электрическая цепь, в которой индуктируются электродвижущая сила и ток электродвижущая сила Е, своими концами присоединяется к ка- кому-либо приемнику электрической энергии, например к элек- трической лампочке, образуя вместе с ним замкнутую электриче- скую цепь (рис. 152). Так как на концах проводника ab имеется напряжение Г/, то под этим напряжением окажется подключенная к проводнику лампочка. А это значит, что во внешней цепи (лампочка с сое- динительными проводниками) возникнет электрическое поле, а следовательно, и электрический ток. Электроны с конца провод- ника 6, заряженного отрицательно, через внешнюю цепь (через лампу) будут перемещаться к концу проводника а, заряженного положительно, где они будут нейтрализованы. Но как только часть отрицательных и положительных зарядов проводника ab исчезнет, тотчас же уменьшится напряжение на концах этого проводника, т. е. ослабнет электрическое поле в нем. Сторонние силы теперь получат перевес над силами электрического поля внутри проводника ab и электродвижущая сила Е станет боль- ше напряжения U на концах проводника. Это приведет к тому, что внутри проводника ab возобновится движение свободных электронов с положительно заряженного конца его к отрица- тельно заряженному концу. Таким образом, во всей замкнутой цепи будет существовать электрический ток проводимости, кото- рый в данном случае называется индуктированным током. Он будет проходить в цепи до тех пор, пока в проводнике ab будет 271
существовать поле сторонних сил, т. е/ пока проводник будет пе- ремещаться в магнитном поле при ромощи какой-либо внешней механической силы. / Следует обратить внимание на/ то, что работа индук- тированного тока в данной цепи совершается за счет механической работы внешних сил, пе- ремещающих проводник ab в магнитном поле. Чем больше индуктированный ток в цепи, тем больше внешняя сила затрачивает энергии на перемещение проводника ab в магнитном поле. При наличии тока в рассматриваемой цепи напряжение на концах проводника ab будет меньше, чем индуктированная электродвижущая сила в этом проводнике, так как часть элек- тродвижущей силы будет израсходована внутри самого провод- ника ab: Uab = E-U., где Е—индуктированная электродвижущая сила в провод- нике ab\ Uo—падение напряжения внутри проводника ab\ Uab—напряжение на концах проводника расходуемое во внешней цепи. В рассматриваемой цепи электроны движутся от отрицатель- но заряженного конца проводника ab (от минус-зажима) к по- ложительно заряженному концу его (к плюс-зажиму) по внеш- ней цепи. Но согласно принятому условию индуктированный ток в данной цепи имеет направление от плюс-зажйма проводника ab к минус-зажиму по внешней цепи и затем внутри него от ми- нус-зажима к плюс-зажиму. Возникновение в проводнике индуктирован- ной электродвижущей силы называется элек- тромагнитной индукцией. Это явление было открыто в 1831 г. английским физиком М. Фарадеем (1791—1867). По- мимо электромагнитной индукции, Фарадей открыл закон элек- тролиза, явление диамагнетизма и сделал ряд научных работ в области физики и химии. Фарадей считал, что причиной возникновения индуктирован- ной электродвижущей силы в проводнике является процесс «п е- ресечения» проводником магнитных линий при его переме- щении во внешнем магнитном поле. Если проводник перемещается во внешнем магнитном поле и на пути своего движения пересекает магнитные линии его, то в этом проводнике индуктируется электродвижущая сила. Индуктированная электродвижущая сила может возникнуть и в неподвижном проводнике, если он находится в перемещаю- щемся магнитном поле. Следовательно, для создания индуктиро- ванной электродвижущей силы необходимо движение проводки- 272
ка относительно внешнего магнитного поля или магнитного по- лй — относительно неподвижного проводника. Если проводник движется в магнитном поле, не пересекая его магнитных линий, т. е. скользит вдоль них, то индуктированная электродвижущая сила в нем будет равна нулю. Направление индуктированной электродвижущей силы в пря- молинейном проводнике, движу- щемся в магнитном поле перпен- дикулярно магнитным линиям (рис. 153), определяется правилом правой руки: если ладонь правой руки расположить в магнитном поле навстречу магнитным линиям так, чтобы отставленный большой палец указал направление движе- ния проводника, то остальные че- тыре вытянутых пальца покажут направление индуктированной электродвижущей силы. Рис. 153. Рисунок, поясняющий правило правой руки Рис. 154. Прямолинейный проводник движется под углом а к магнитным линиям Величина индуктированной электродвижущей силы Е в пря- молинейном проводнике пропорциональна величине магнитной индукции В, скорости движения проводника v и активной длине проводника /, т. е. той части проводника, которая пересекается магнитными линиями поля: E = Blv, (147) где Е — индуктированная электродвижущая сила в вольтах; В—магнитная индукция в вольт-секундах на квадратный метр; I—активная длина проводника в метрах; v—скорость движения проводника в метрах в секунду. Если прямолинейный проводник при движении в магнитном поле пересекает магнитные линии не под прямым углом, а под каким-то иным углом а (рис. 154), то величина индуктирован- ной электродвижущей силы в нем определяется по формуле E = Blv> sin а. (148) 18—1377 273
Следовательно, если а = О, т. е. Проводник, перемещаясь в магнитном поле, скользит вдоль магнитных линий,^о индукти- рованная электродвижущая сила в нем будет равна нулю, так KaKsinO° = O. перемещается м2 воднике, если Пример 86. Прямолинейный проводник, активная длина которого I = 0,8 в однородном магнитном поле, имеющем магнитную индукцию . Определить индуктированную электродвижущую силу в про- проводник, двигаясь со скоростью v = 2 , пересекает маг- пол я под углом а = 30°. нитные линии Решение. По формуле (148) находим £* = B/^-sina = 1,2*0,8*2*0,5 = 0,96 в. § 88. ИНДУКТИРОВАННАЯ ЭЛЕКТРОДВИЖУЩАЯ СИЛА В ЗАМКНУТОМ КОНТУРЕ И КАТУШКЕ Индуктированная электродвижущая сила возникает во вся- ком замкнутом4 контуре, если магнитный поток, пронизывающий его, изменяется с течением времени. Покажем это на следую- щем простом опыте. Пусть катушка, имеющая большое число витков проволоки, замкнута на гальванометр (прибор, измеряю- щий малые токи) с двусторонней шкалой (рис. 155). Если на- Рис. 156. При возрастании магнит- ного потока индуктированный ток направлен против часовой стрелки, а при убывании магнитного потока— по часовой стрелке Рис. 155. При приближе- нии постоянного магнита к катушке и удалении, от нее в катушке индук- тируется электродвижу- щая сила чать приближать постоянный стержневой магнит к катушке, на- пример, северным полюсом N, то стрелка гальванометра откло- нится от нулевого положения, например, вправо от нуля. Это значит, что в цепи катушка — гальванометр возник электриче- ский ток. При прекращении движения магнита стрелка гальва- нометра возвращается к нулевому положению, что свидетель- 274
ствует о прекр£щении\тока в катушке. Если, наоборот, начать удалять магнит от катушки, то стрелка гальванометра вновь от- клонится, но в другую сторону, т. е. влево от нуля. Чем быстрее движется магнит или чем быстрее изменяется магнитный поток, пронизывающий катушку, тем больший ток индуктируется в катушке. Направление индуктированного тока в катушке зависит от направления движения магнитов, или, иначе говоря, от направ- ления движения магнитных линий, пронизывающих катушку. Э. X. Ленц сформулировал закон, устанавливающий направ- ление индуктированного тока в электрической цепи. Этот закон, вошедший в электротехнику под названием правила Ленца, гла- сит: индуктированный ток в проводнике (контуре) возникает всякий раз такого направления, при котором он противодейст- вует причине, вызвавшей его. Если, например, катушку (контур) пронизывает некоторый уменьшающийся во времени магнитный поток, то в катушке (контуре) индуктируется ток такого на- правления, при котором он создает свое собственное магнитное поле, противодействующее уменьшению основного магнитного поля, т. е. направление магнитного поля тока и направление ос- новного магнитного поля в этом случае должны совпасть. Наоборот, если магнитный поток, пронизывающий катушку (контур), увеличивается, то индуктированный ток имеет такое направление, при котором он создает магнитный поток, направ- ленный навстречу внешнему магнитному потоку, т. е. противо- действующий его росту. Следовательно, если число магнитных линий, пронизывающих площадь, ограниченную контуром, уве- личивается, т. е. ДФ>0 (читать: дельта фи больше нуля), то для наблюдателя, смотрящего по направлению магнитных ли- ний, индуктированный ток направлен против часовой стрелки (рис. 156). Наоборот, если число магнитных линий уменьшается (ДФ <0), то для наблюдателя, смотрящего в направлении маг- нитных линий, индуктированный ток направлен по часовой стрелке. Применим правило Ленца к рассмотренному выше опыту с катушкой. Если постоянный магнит приближается к катушке своим северным полюсом, то в ней индуктируется ток такого на- правления, который своим магнитным полем противодействует движению магнитного поля постоянного магнита и в соответ- ствии с этим движению самого магнита. Следовательно, в ка- тушке возникает индуктированный ток такого направления, что на конце катушки, обращенном к приближающемуся северному полюсу магнита, возникает также северный полюс, противодей- ствующий приближению последнего. Если начать удалять маг- нит от катушки, то индуктированный ток в ней изменит свое на- правление на обратное и в соответствии с этим на конце катуш- ки, обращенном к удаляющемуся северному полюсу магнита, 18* 275
возникнет южный полюс, противодействующий удалению пос- леднего. Величина индуктированной электродвижущей силы в контуре прямо пропорциональна скорости изменения магнитного потока, пронизывающего этот контур. Если* за время Д/ (дельта t) секунд магнитный поток изме- нился на величину ДФ (дельта фи) вольт-секунд, то величина индуктированной электродвижущей силы в вольтах численно ДФ равна отношению , т. е. Е ДФ м * (149) Рис. 157. Напра- вления положи- тельного магнит- ного потока и по- ложительной ин- дуктированной электродвижущей силы связаны пра- вилом буравчика Если в формуле (149) положить, что ДФ=1 вольт-секунде, а = 1 секунде, то Е = 1 вольту. Следовательно, если в контуре магнитный поток изменяется на 1 вольт-секунду в 1 секунду, то в этом контуре индуктируется электродвижущая сила в 1 вольт. Теперь выясним значение знака минус в формуле (149) пе- ДФ ред отношением . Так как индуктированная электродвижущая сила в контуре может иметь различные направления, то ее надо считать вели- чиной алгебраической, т. е. при одном ее направлении в контуре она должна условно считаться положительной, а при обратном направлении — отрицательной. Точно так же и магнитный по- ток— величина алгебраическая, так как он по отношению к кон- туру может иметь разные направления, т. е. может входить в с другой стцроны. Какие же направления ин- дуктированной электродвижущей силы и маг- нитного потока принять за положительные? Считают, что условно принятый положительный магнитный поток, пронизывающий контур, и условно принятая положительная электродви- жущая сила, индуктированная в этом контуре, связаны друг с другом правилом буравчика правовинтовой системы, т. е. если вращатель- ное движение буравчика совпадает с положи- тельным направлением электродвижущей силы в контуре, то поступательное движение бурав- чика совпадаете положительным направлением магнитного потока, пронизывающего этот кон- тур (рис. 157). Стало быть, если ДФ > 0, т. е. прирост магнитного потока положительный, то индуктированная электродвижущая сила будет отрицательной, так как все выра- / ДФ \ жение имеет отрицательный знак. 276
А это значит, что она с магнитным потоком составит левовинтовую си- стему (рис. 158, а). На- оборот, если ДФ < 0, т. е. прирост магнитного по- тока отрицательный, то индуктированная электро- движущая сила будет положительной, так как личина положительная. А это значит, что в данном случае магнит- ный поток составляет с Рис. 158. При ДФ > 0 электродвижущая сила отрицательна, а при ДФ<^0 положи- тельна электродвижущей силой правовинтовую систему (рис. 158,6). Если вместо одного контура (витка) взять катушку, имею- щую w витков, то индуктированная в ней электродвижущая си- ла будет в w раз больше, чем в каждом из ее отдельных вит- ков, Т. ве г-. ач/ Е = — w —гг вольт. (150) Пример 87. В катушке, имеющей число витков w = 500, магнитный поток, пронизывающий ее, изменяется равномерно на величину ДФ = —0,2 в-сек в течение Д^ = 0,5 сек. Определить индуктированную электродвижущую силу в катушке. Решение. По формуле (150) находим = — 500 = 200 в. Формула (150) имеет важный физический смысл. Если в кон- туре индуктируется электродвижущая сила, то это значит, что в проводниках контура возникло электрическое поле. Под влия- нием сил индуктированного электрического поля свободные элек- троны получают направленное и упорядоченное движение вдоль участков контура, т. е. в контуре возникает индуктированный ток. Индуктированное электрическое поле возникает не только в проводниках замкнутого контура, но и во всем пространстве, где есть переменное магнитное поле. Это легко обнаружить, поме- щая контур в различные области переменного магнитного поля. На основании сказанного можно сделать следующий вывод: в пространстве, где есть изменяющееся магнитное поле, возни- кает (индуктируется) изменяющееся электрическое поле. Эти поля взаимозависимы, взаимообусловлены, органически срязаны в едином электромагнитном поле. Взаимодействие этих полей сводится к взаимному переходу их друг в друга, к взаимному обмену энергией, что и является причиной перемещения электро магнитной энергии в пространстве. 277
Индуктированное электрическое поле, о котором здесь идет речь, коренном образом отличается от электростатического поля. Это различие в основном сводится к следующему: 1. Электростатическое поле создается системой относительно неподвижных электрических зарядов в пространстве, а индукти- рованное электрическое поле —чв результате изменения магнит- ного поля. 2. Электрические линии электростатического поля имеют на- чало на положительных электрических зарядах и концы — на от- рицательных зарядах, а силовые линии индуктированного элек- трического поля замкнуты на себя. Они охватывают магнитные линии изменяющегося магнитного поля. 3. Электростатическое поле не сопровождается магнитным полем, а индуктированное электрическое поле неразрывно свя- зано с изменяющимся магнитным полем, и всякое изменение электрического поля сопровождается возникновением переменного магнитного поля. 4. Работа сил электростатического поля при переносе элек- трического заряда по любому замкнутому контуру равна нулю, а работа сил индуктированного поля по замкнутому контуру от- лична от нуля. Это значит, что электродвижущая сила в любом замкнутом контуре, помещенном в электростатическое поле, равна нулю, а в индуктированном электрическом поле она отлична от нуля и пропорциональна скорости изменения магнитного потока, пронизывающего контур. 5. В электростатическом поле действуют силы взаимодействия электрических зарядов — силы центральные, т. е. силы, направ- ленные по прямой линии; соединяющей центры взаимодействую- щих зарядов, а в индуктированном электромагнитном поле дей- ствуют силы не электростатического происхождения — силы сто- ронние, обусловленные действием сил магнитного поля на дви- жущийся электрический заряд и сил электрического индуктиро- ванного поля. Всякому изменению магнитного поля противодействует ин- дуктированное электрическое поле, которое стремится создать магнитное поле, попутное исчезающему и встречное возрастаю- щему магнитному полю. § 89. ЭЛЕКТРОДВИЖУЩАЯ СИЛА САМОИНДУКЦИИ Допустим, что в катушке, имеющей w витков, протекает по- стоянный ток I (рис. 159). Этот ток создает в данной катушке постоянный магнитный поток Ф, пронизывающий ее витки. Произведение величины магнитного потока Ф, пронизываю- щего витки катушки, на число витков w этой катушки назы- вается числом потокосцеплений, или, иначе, пото- ком, сцепленным с витками катушки, и обозначает- ся буквой ф (читать «пси»). 278
Следовательно, ф = о»Ф, (151) где w — число витков катушки; Ф — магнитный поток в катушке в вольт-секундах; Ф — число потокосцеплений (поток, сцепленный с витками катушки) в вольт-секундах. Если данная катушка не имеет сердечника из ферромагнит- ного материала, то число потокосцеплений ф в катушке пропорци- онально величине тока / в ее витках, т. е. ф = даФ = LIt (152) где L—коэффициент пропорциональности, называемый ин дуктивностью, или, иначе, коэффициентом само индукции катушки. - Выясним физический смысл индуктивности (коэф- фициента самоиндукции) ка- тушки, ее размерность и еди- ницы измерения. Из формулы (152) сле- -дует, что Л=4 = ^, (153) т. е. индуктивность катушки численно равна потоку, сцеп- ленному с витками катушки, при величине тока, равной единице. Исходя из формулы (153), сти катушки: Рис. 159. При замыкании цепи в ка- тушке индуктируется электродвижущая сила самоиндукции выясним размерность индуктивно- г 71 Г Ф 1 вольт-секунда = m = —: ом * секУнда=генРи; т. е. размерность индуктивности в системе единиц МКСА выра- жается в генри. В соответствии с этим за единицу измерения индуктивности в системе единиц МКСА принят 1 генри (1 гн, 1 Н). Если ф — 1 в-сек, I — 1 а, то , Ф 1 вольт-секунда , £ = Т = —Гампер ~ = 1 ГеН₽И< 279
т. е. катушка обладает индуктивностью в 1 генри, если в ней при токе, равном одному амперу, поток, сцепленный с ее витками, равен 1 вольт-секунде. Таблица 31 Единицы измерения индуктивности в системе единиц МКСА Наименование величины и ее обозначение Название единицы Обозначение Соотноше- ние с основной единицей русское между- народное г Индуктивность (коэффициент 1 самоиндукции) L | генри миллигенри микрогенри гн мгн мкгн Н mH Р-Н х ае IT Г о о Зная индуктивность L катушки, величину постоянного тока I в ней, а также число витков w катушки, можно определить ве- личину магнитного потока Ф в этой катушке по формуле ф=-^’ (154) где Ф—магнитный поток в вольт-секундах; I — величина тока в амперах; L—индуктивность в генри. Теперь допустим, что в рассматриваемой катушке протекает переменный ток /, т. е. такой ток, который с течением времени изменяется по величине. В соответствии с этим в катушке будет переменный магнитный поток, так как его величина Ф пропор- циональна величине тока /, создающего его. Но если витки катушки пронйзывает переменный магнитный поток, то в ней должна индуктироваться электродвижущая сила. Возникновение в катушке индуктированной электродвижу- щей силы под влиянием изменения ее собственного магнитного потока называется самоиндукцией, а возникающая при этом электродвижущая сила — электродвижущей силой самоиндукции. Величина индуктированной электродвижущей силы самоин- дукции катушки пропорциональна скорости изменения потока, сцепленного с витками этой катушки, т. е. = (155) где eL — электродвижущая сила самоиндукции; Дф — приращение потока, сцепленного с витками катушки, за элементарный отрезок времени Д/. Но, как было выяснено выше, поток ф, сцепленный с витками катушки, равен ф = 280
где L — индуктивность катушки, поэтому формуле (155) можно придать такой вид: 4 = ' (156) т. е. величина электродвижущей силы самоиндукции в катушке пропорциональна скорости изменения тока в ней. Следовательно, чем быстрее изменяется ток в катушке, тем при прочих равных условиях больше будет в ней электродвижу- щая сила самоиндукции. Пример 88. В катушке, индуктивность которой L — 0,2 гн, ток убывает равномерно за каждые 0,01 сек. на 2 а. Определить индуктированную элек- тродвижущую силу самоиндукции в катушке. Рт'Ш е н и е. В данном случае Д/ =—2 а и Д/ = 0,01 сек. По формуле (156) найдем д/ __о ^ = -^#=-0,2. ^ = 40 в. Исходя из формулы (156), можно дать единице измерения индуктивности, равной одному генри, такое определение: катуш- ка обладает индуктивностью, равной 1 генри, если при равномер- ном изменении тока в ней на 1 ампер в каждую секунду индук- тируется электродвижущая сила самоиндукции в 1 вольт. Знак минус, стоящий в правой части формулы (149), отра- жает собой известное правило Ленца, которое в данном случае можно сформулировать так: электродвижущая сила са- моиндукции всякий раз имеет такое направление в катушке, при котором она. противодействует изменению величины тока в ней. Если, например, ток в катушке возрастает, то электродвижу- щая сила самоиндукции направлена навстречу току, противодей- ствуя его росту, и, наоборот, когда ток в катушке убывает, она направлена попутно с ним, противодействуя его убыванию. Ис- точник электрической энергии в первом случае совершает неко- торую работу, преодолевая сопротивление электродвижущей си- лы самоиндукции. Эта работа идет на создание энергии магнит- ного поля катушки. Во втором же случае, когда электродвижу- щая сила действует попутно с током, энергия магнитного поля переходит в энергию электрического поля, расходуемую в дан- ной цепи. С явлением самоиндукции мы встречаемся во всех электри- ческих цепях, обладающих индуктивностью, в которых проходит изменяющийся по величине ток. Для примера рассмотрим режим в цепи с катушкой индук- тивности при подключении ее к источнику электрической энер- гии с постоянным напряжением и при отключении от него. 281
нения тока в катушке при подключении ее к источнику электриче- ской энергии с постоян- ным напряжением Рис. 161. График изме- нения тока в катушке при отключении ее от источника электриче- ской энергии с постоян- ным напряжением Если катушку, обладающую некоторой индуктивностью и омическим сопротивлением, подключить к источнику электриче- ской энергии с постоянным напряжением, то ток в катушке бу- дет нарастать не сразу, а плавно, непрерывно изменяя свою ве- личину по строго определенному закону (рис. 160). Причина по- добного изменения тока в катушке — электродвижущая сила са- моиндукции. В результате нарастания тока в катушке увеличи- вается в ней и магнитный поток, создаваемый этим током. Из- меняющийся же магнитный поток индуктирует в катушке элек- тродвижущую силу самоиндукции, направленную противополож- но току и потому противодействующую его нарастанию. Чем больше индуктивность катушки, тем больше электродвижущая сила самоиндукции в ней и тем медленнее нарастает ток в ка- тушке при подключении ее к источнику с постоянным напряже- нием. Практически ток в катушке достигает своего постоянного значения обычно в течение долей секунды. Если катушку, по которой течет постоянный ток /, отключить от источника электрической энергии и замкнуть на некоторое сопротивление л, то ток в катушке не исчезнет мгновенно (скач- ком), а будет более или менее плавно спадать до нуля (рис. 161). Причина этого — электродвижущая сила самоиндук- ции. Когда ток в катушке уменьшается, одновременно с этим уменьшается и магнитный поток, создаваемый им. В результате в катушке индуктируется электродвижущая сила самоиндукции, поддерживающая ток в цепи, так как она направлена попутно с ним. Чем больше индуктивность катушки, тем медленнее спадает ток в ней при отключении ее от источника электрической энергии с постоянным напряжением. Чтобы судить о том, насколько быстро исчезает ток в ка- тушке при отключении ее от источника и замыкании на сопро- тивление Гь ввели величину, называемую, постоянной времени т (читать «тау»). Постоянная времени т — это отрезок 282
времени, в течение которого ток в катушке уменьшится в е = 2,718 (основание натуральных логариф- мов) раз по сравнению с начальным его значе- нием. Постоянную времени т можно определить по следующей формуле: (157) где т— постоянная времени в секундах; г—омическое сопротивление катушки в омах; f\— омическое сопротивление, на которое замыкается ка- тушка после ее отключения от источника; L— индуктивность катушки в генри. Пример 89. Определить постоянную времени х цепи, если ее активное сопротивление г = 2 ом, индуктивность L = 0,01 гн, а сопротивление, на ко- торое она замыкается, равно Г\ = 8 ом. Решение. Применяя формулу (157), находим L 0,01 плл1 t+t;=2+8=0-001 сек- т = т. е. ток в катушке уменьшится в 2,718 раз по сравнению с начальным через 0,001 сек. С явлением самоиндукции приходится считаться во всех элек- трических цепях, где есть изменяющийся по величине ток. Если индуктивность катушки велика, то при больших скоростях изме- нения тока в ней индуктированная электродвижущая сила само- индукции может достигать очень больших величин. Чтобы индуктивность катушки практически свести к нулю/ применяют так называемую бифилярную намотку катушек (рис. 162). В этом случае в каждых двух соседних витках ток имеет противоположные направления, что создает два встречных магнитных потока, уравновешивающих друг друга. Вследствие этого индуктированная электродвижущая сила в катушке прак- тически сводится к нулю при всяких изменениях тока. В момент отключения катушки от источника электрической энергии индуктированная электродвижущая сила может оказать- ся чрезмерно большой, и при этом может произойти пробой изо- ляции витков катушки. Ток в катушке после отключения ее от источника электрической энер- гии называется экстратоком размыкания. Он обычно длит- ся всего несколько долей се- кунды. При размыкании цепи, обладающей индуктивностью, в месте разрыва ее образуется искра, через которую проходит экстраток размыкания. Эта искра, обладающая большей темпера- Рис. 162. Катушка с бифилярной намоткой 283
турой, обжигает разрывающиеся контакты, например в ключе телеграфного аппарата М-44. При частых замыканиях и размы- каниях они окисляются и проводимость их'резко уменьшается. Поэтому контакты обычно покрывают слоем тугоплавкого не- окисляющегося материала, например платины. § 90. ИНДУКТИВНОСТЬ ЦИЛИНДРИЧЕСКИХ катушек L Цилиндрическая катушка с однослойной обмоткой Допустим, что в цилиндрической однослойной катушке, имею- щей число витков w, длину I и диаметр цилиндрического карка- са D, проходит постоянный ток I (рис. 163). Рис. 163, Цилиндрическая однослойная катушка Предположим также, что длина катушки I значительно боль- ше (в десятки раз) ее диаметра D. В этом случае напряжен- ность магнитного поля внутри катушки равна где Н— напряженность магнитного поля катушки в амперах на метр; /— величина тока в катушке в амперах; I—длина катушки в метрах. Магнитная индукция В = ИЯ = ^-^-, где В— магнитная индукция в вольт-секундах; (а— магнитная проницаемость в генри на метр. ' Магнитный поток в катушке где Ф— магнитный поток в вольт-секундах; S—площадь поперечного сечения катушки в квадратных метрах. 284
Поток, сцепленный с витками катушки, равен где ф — поток, сцепленный с витками катушки, в вольт-секундах. Из последнего выражения находим индуктивность катушки г__ ф __ w2/p£ ~ и ’ или L=^- (158) где L — индуктивность в генри. Пример 90. Определить индуктивность однослойной цилиндрической ка- тушки, имеющей число витков w = 500, длину I = 0,25 м, площадь попереч- ного сечения цилиндрического каркаса S = 0,0004 м2 и магнитную проницае- 4п гн мость сердечника р = . 1U* м Решение. Применяя формулу (158), находим г w2pS 5002 • 4п • 0,0004 л ллле £ = —f— = —1О7.о,25— = 0,0005 генри. Если длина I катушки мало отличается от диаметра цилинд* рического каркаса ее, то в формулу (158) вводят поправочный коэффициент k, зависящий от отношения диаметра D катушки к ее длине I. Таблица 32 Зависимость между коэффициентом k и отношением диаметра катушки D к ее длине I D 1 k D 1 k D I k D I k 0,00 1 0,40 0,85 0,85 0,72 2,5 0,47 0,05 0,98 0,45 0,83 1,00 0,69 3,0 0,43 0,10 0,96 0,50 0,82 1,10 0,67 3,5 0,39 0,15 0,94 0,55 0,80 1,20 0,65 4,0 0,36 0,20 0,92 0,60 0,79 1,40 0,61 4,5 0,34 0,25 0,90 0,65 0,77 1,60 0,58 5 0,33 0,30 0,88 0,70 0,76 1,80 0,55 6 0,28 0,35 I 0,87 0,75 0,75 2,00 0,53 7 0,26 В табл. 32 приведены значения поправочного коэффициента k D , для разных значении отношения —. Формула для расчета од- 285
нослойных цилиндрических катушек в этом случае принимает следующий вид: L==k^-. (158') Формуле (158') можно придать несколько иной вид, сделав следующие подстановки: Тогда формула (158х) принимает следующий вид: L = k^D4^r-\Q'\ (159) где — относительная магнитная проницаемость сердечника катушки; — число витков, приходящееся на один метр длины ка- тушки; I — длина катушки в метрах; L — индуктивность катушки в генри. Пример 91. Определить индуктивность однослойной цилиндрической ка- « » витков тушки, у которой I = 0,4 м, D = 0,04 ле, Wo = 500-—- и относительная магнитная проницаемость сердечника катушки рг=1. Решение. По табл. 32 находим значение коэффициента Л, который при -у- = = 0,1 составляет 0,96.- По формуле (159) находим L = k^wlD4^rA^ = = 0,96 • 3,143 • 5002 • 0,Q42 • 0,4 • 1 • 1СГ7 1,5 • 10"4 гн. Рис. 164. Многослойная цилин- дрическая катушка в разрезе 2. Цилиндрическая катушка с многослойной обмоткой Для увеличения индуктивности катушки на ее каркас наносят обмотку, состоящую из несколь- ких слоев (рис. 164). Индуктивность многослойной цилиндрической катушки опреде- ляется по формуле L = m*LCB, (160) 286
где т— число всех слоев обмотки катушки; £ср — индуктивность ОД- НОСЛОЙНОЙ катуш- ки, у которой диа- метр равен полу- сумме наружного и . внутреннего диаметров много- слойной катушки: _______ ^внутр 4" ^нар Ь'ср = 2 • Пример 92. Требуется опреде- лить индуктивность много- слойной цилиндрической ка- тушки, у которой число слоев Рис. 165. Катушки индуктивности с обмот- ки — 6, диаметр каркаса D = кой в виде спиральной ленты из вожжен- = 0,05 м и длина намотки (ка- ного в керамику серебра тушки) / = 0,12 м. Толщина провода с изоляцией d = 0,5 мм и относительная магнитная проницаемость сердечника катушки = 1. Решение. Толщина шести слоев провода md = 6 • 0,5 = 3 мм = 0,003 м. Наружный диаметр обмотки 7)нар = ^внутр 4" 2-0,3 = 5 + 0,6 = 5,6 см = 0,056 м. Средний диаметр обмотки ^внутр + ^нар ^ср = 2 Отношение диа-метра £>ср к ^ср 5 4- 5,6 = —£— = 5,3 см = 0,053 м. длине / катушки 0,053 ~Т~ “ "ojT = 0,44- По табл. 32 определяем коэффициент k, который для ~ = 0,44 ра- вен 0,83. Число витков wq на один метр длины катушки W<> = ~d ~ 0^005 ~ 2С00, Индуктивность катушки согласно формулам (159) и (160) равна £ = /п2£ср = т,2къ2муЭЧрг • Ю'7 = = 62 - 0,83 • 3,142 • 20002 • 0,0532 • 0,12 • 1 • 10~7 ж 0,04 гн. В технике связи применяются самые разнообразные типы ка- тушек как с постоянной, так и с переменной индуктивностью. На рис. 165 показаны однослойные цилиндрические катушки с постоянной индуктивностью, применяемые в радиотехнике для УКВ (ультракоротких волн). Отличительная особенность этих катушек та, что остов их изготовляется из керамики, а обмотка 287
тушка с сер- дечником из магнитодиэлек- трика, показан- ная в разрезе тушки, равен представляет собой спиральную ленту из вожжен- ного в керамику серебра. Индуктивность таких катушек колеблется от сотых долей микрогенри до нескольких десятков микрогенри. На рис. 166 показана катушка индуктивности с сердечником из магнитодиэлектрика (карбо- нильное железо, порошкообразный пермаллой, альсифер и др.), широко применяемая в радио- технике. Вдвигая или выдвигая сердечник, можно соответственно увеличивать илИ уменьшать индук- тивность катушки*. § 91. ЭЛЕКТРОДВИЖУЩАЯ СИЛА ВЗАИМОИНДУКЦИИ Допустим, что две катушки расположены в непосредственной близости одна от другой (рис. 167). Одна из этих катушек, первичная, имеет число витков * Wi, а другая, вторичная,— w2. Первичная катушка присоединена к источнику электрической энергии с по- стоянным’ напряжением Ulf и в ней течет постоянный ток Д, со- здающий постоянный магнитный поток ФР Часть этого магнит- ного потока Ф1>2 пронизывают витки не только первичной, но и вторичной катушки. Следовательно, поток, сцепленный с витками вторичной ка- Ф1, 2 — W2^1> 2* А так как магнитный поток Ф1>2 пропорционален току Д, то и поток, сцепленный с витками вторичной катушки, также про- порционален току Д: Ф1,2=®’2Ф1.2=М71, (161) Рис. 167. Две катушки индуктивно связаны друг с другом 288
где М — коэффициент пропорциональности, называемый вза- имной индуктивностью катушек, или, иначе, коэф- фициентом взаимоиндукции их. Из формулы (161) следует, что = (162) ч ч т. е. взаимная индуктивность двух индуктивно связанных кату- шек численно равна потоку, сцепленному с витками одной из этих катушек, при величине тока, равной единице, в другой ка- тушке. Исходя из равенства (162), выясним размерность взаимной индуктивности: [Л4] = [1^-] = = ом-секунда = генри, т. е. размерность взаимной индуктивности в системе единиц МКСА выражается в генри. В соответствии с этим за единицу измерения взаимной индук- тивности в системе единиц МКСА принят 1 генри (1 гн, 1 Н). Если в формуле (162) положить, что ф1,2 = 1 в-сек и Л = 1 а, то будем иметь: * Ф1.2 1 в-сек . Л/ = -4^ = — = 1 генри, ii 1 а г ’ т. е. две катушки обладают индуктивностью в 1 генри, если при токе, равном одному амперу, в одной из них поток, сцепленный с витками второй катушки, равен 1 вольт-секунде. Теперь допустим, что в первичной катушке протекает пере- менный ток 1\. В соответствии с этим магнитный поток Фь соз- даваемый им, а также и поток фЪ2, сцепленный с витками вто- ричной катушки, являются переменными во времени. Но если по- ток ф1,2, сцепленный с витками вторичной катушки,— перемен- ный, то в витках этой катушки должна индуктироваться электро- движущая сила, называемая электродвижущей силой взаимоиндукции. Величина электродвижущей силы взаимоиндукции во второй катушке пропорциональна скорости изменения потока, сцеплен- ного с витками этой катушки, т. е. ^2=-Т> (163) где —электродвижущая сила взаимоиндукции во вторичной катушке; Дфь2—приращение потока, сцепленного с витками вторичной катушки, за элементарный отрезок времени ДЛ 19—1377 289
Но, как известно, Фь2 — » где Л4 — взаимная индуктивность катушек; поэтому формуле (163) можно придать такой вид: (164) т. е. индуктированная электродвижущая сила взаимоиндукции во вторичной катушке пропорциональна скорости изменения тока в первичной катушке. Следовательно, чем быстрее изменяется ток в первичной ка- тушке, тем больше величина электродвижущей силы взаимоин- дукции во вторичной катушке. На основании формулы (164) можно дать такое определение единице измерения взаимной индуктивности — 1 генри. Две ка- тушки обладают взаимной индуктивностью в 1 генри, если при равномерном изменении тока в одной из них на 1 ампер в 1 се- кунду в другой катушке индуктируется электродвижущая сила взаимоиндукции в*1 вольт. Знак минус, стоящий в формуле (164), отображает собой из- вестное правило Ленца, которое в данном случае можно сформулировать так: индуктированная электродвижущая сила взаимоиндукции во вторичной катушке всегда имеет такое на- правление, при котором электромагнитные процессы, создавае- мые ею, противодействуют всем изменениям тока первичной ка- тушки, а следовательно, и изменениям магнитного потока, созда- ваемого им. Если, например, вторичная цепь замкнута, то элек- тродвижущая сила взаимоиндукции создает в ней переменный ток /2, который в свою очередь создает свой собственный маг- нитный поток, действующий навстречу потоку, создаваемому то- ком первичной катушки. § 92. ВЗАИМНАЯ ИНДУКТИВНОСТЬ ДВУХ КАТУШЕК Определим взаимную индуктивность двух катушек, разме- щенных на тороидальном (кольцевом) сердечнике (рис. 168). Допустим, что по первичной катушке, имеющей число витков wh течет постоянный ток Л. Полагаем также, что весь магнит- ный поток Фь создаваемый током /ь целиком пронизывает вит- ки w2 вторичной катушки. Магнитный поток Ф1; создаваемый током 1\ первичной ка- тушки, равен 1 / ’ где Ф, — магнитный поток в первичной катушке в вольт-се- кундах; 290
Ц— ток в первичной катушке в амперах; р-—магнитная проницаемость тороидального сердечника в генри на метр; ^ = ^о = Ил4^-1О-7 I—длина средней магнитной силовой линии в сердечнике катушек в метрах; 5—площадь поперечного сечения сердечника в квадрат- ных метрах. Поток 2, сцепленный с витками вторичной катушки, в этом случае равен Отсюда находим взаимную индуктивность катушек по фор- муле или = (165) где М — взаимная индуктив- ность в генри. Пример 93. Две катушки с числом витков wi = 200 и = 300 размещены на тороидальном сер- дечнике (рис. 168). Определить взаимную индуктивность этих ка- тушек, если длина средней маг- нитной линии в системе катушек Z = 0,628 м, площадь поперечного сечения тора катушек S= 10* 10 ~4 мость сердечника катушки |xr = 1. Решение. По формуле (165) Рис. 168. Две катушки, расположенные на тороидальном сердечнике, индук- тивно связаны друг с другом м1 и относительная магнитная проницае- находим 107-0,628 / Теперь выясним соотношение между индуктивностями двух индуктивно связанных катушек (рис. 168) и их взаимной индук- тивностью. Как известно, первичная катушка имеет индуктивность, равную I ’ индуктивность второй катушки равна j 19* 291
Перемножив последние два равенства, получим Извлекая корень квадратный из полученного выражения, на- ходим,'что т. е. взаимная индуктивность двух катушек при отсутствии магнитного рассеяния в их магнит- ной цепи равна корню квадратному из произве- дения величин индуктивностей этих катушек. Однако при наличии магнитного рассеяния Vl^>m и в общем случае М = кУЦЬ2, (166) где k — так называемый коэффициент связи двух индуктивно связанных катушек, величина которого может изменять- ся от нуля до единицы: Пример 94. Определить коэффициент связд двух катушек, если их индук- тивности соответственно равны Li = 0,01 гн и L2 = 0,09 а их взаимная индуктивность М = 0,024 гн. Решение. По формуле (166) находим . М 0,024 0,024 А о Я = - ;____1 = Z ________- = = 0,8. Уцц J/0,01-0,09 0,03 Рис. 169. Воздушный трансформатор высокой частоты Индуктивно связанные катушки широко применяют- ся в электротехнике, и в частности в технике связи, в виде трансформаторов, ин- дукционных катушек, варио- метров и т. д. На рис. 169 показана си- стема двух индуктивно свя- занных катушек, посаженных на один цилиндрический кар- кас, применяемая в качестве воздушного трансформатора высокой частоты в радиотех- нике- 292
§ S3. ПОСЛЕДОВАТЕЛЬНОЕ И ПАРАЛЛЕЛЬНОЕ СОЕДИНЕНИЕ КАТУШЕК ИНДУКТИВНОСТИ Если несколько катушек индуктивности соединены последова- тельно и так, что они друг на друга индуктивного влияния не оказывают (рис. 170), то общая индуктивность всей цепи равна сумме индуктивностей отдельных катушек: А = £1 + А2 + А3 + ... + (167) При параллельном соединении катушек, не связанных индук- тивно друг с другом (рис. 171), общая индуктивность опреде- ляется формулой + + + (>«>) 0 0- Рис. 170. Последовательное соединение катушек индук- тивности Рис. 171. Параллель- ное соединение кату- шек индуктивности Пример 95. Три катушки с индуктивностями Ц = 0,3 гн, L2 = 0,6 гн и £з = 0,8 гн соединены параллельно. Определить общую индуктивность всей цепи, если катушки не связаны индуктивно друг с другом. Решение. По формуле (168) находим L ~~ Ц + А2 + L3 “ 0,3 + 0,6 + 0,8 “ 4 ’ откуда L = -Л- = 0,16 гн. Если катушки индуктивности соединены последовательно и индуктивно связаны друг с другом, то их общая индуктивность определяется по формуле £ = Z1 + £2±2M (169) где L— общая индуктивность катушек; ZjhZ2—индуктивности отдельных катушек; М— взаимная индуктивность катушек. 293
Знак плюс, стоящий перед величиной 2М, относится к со- гласному включению катушек, т. е. к тому случаю, когда собственный магнитный поток в каждой катушке совпадает с направлением магнитного потока другой катушки, пронизываю- щего ее. Если же эти потоки направлены навстречу друг другу, т. е. Рис. 172. Вариометр катушки включены встреч- н о, то перед величиной 2М ставится знак минус. На рис. 172 изображен прибор, называемый варио- метром, дающий возмож- ность плавно изменять об- щую индуктивность двух его катушек, соединенных после- довательно, путем изменения их взаимной индуктивно- сти. Поворачивая подвиж- ную катушку внутри непо- движной, можно плавно из- . менять коэффициент взаи- моиндукции катушек. Когда катушки расположены па- раллельно друг другу, их индуктивное взаимодействие бывает наибольшим; когда же они расположены своими плоскостями под прямым углом, их индуктивное взаимодействие равно нулю. Если направления то- ков в катушках при параллельном расположении последних оди- наковы, то общая индуктивность вариометра максимальна: L — Lx + Z»2 + 2Af, а если токи имеют различные направления, то общая индуктив- ность вариометра минимальна: L = A, -|- L2 — 27И. Пример 96. Определить, в каких пределах изменяется общая индуктив- •ю-сгь вариометра, если индуктивности его катушек соответственно равны Ci -= 8• 10—6 гн и £2 = 2*10~бги и максимальный коэффициент связи кату- шек k = 0,6. Решение. Определяем взаимную индуктивность катушек по фор- луле (166): М = k]/= 0,6 У8-10"5-2-10-5 = 2,4• 10-5 гн. Максимальное значение индуктивности L = LX +L2 -J- 2/W = 8 - IO"5 4-2-10-5 + 2-2,4-10's = = 14,8-10“5 гн. 294
Минимальное значение индуктивности. L = Lx + L2 — 2М = 8 • IO'5 + 2 • IO"5 — 2 • 2,4 • 1 О-5 = 5,2 • 10~5 гн. § 94. ЭНЕРГИЯ МАГНИТНОГО ПОЛЯ Магнитное поле, как некоторый вид материи, обладает энер- гией. Энергия магнитного поля распределена во всем объеме этого поля с некоторой конечной плотностью, пропорциональной ква- драту напряженности магнитного поля: Ц70=^^-2, (170) где —объемная плотность энергии магнитного поля в джоулях на кубический метр; Н—напряженность магнитного поля в амперах на метр; |л0 = 4тс. ю-7— —магнитная проницаемость вакуума; ptr—относительная магнитная проницаемость сре- ды, где распределено магнитное поле. Если известно распределение объемной плотности энергии магнитного поля, то можно определить энергию всего поля пу- тем суммирования ее по отдельным элементарным объемам, что. обычно выполняется приемами высшей математики. Ограничим- ся подсчетом энергии магнитного поля для простейшего случая, когда оно однородно. В этом случае энергия магнитного поля во всем ее объеме распределена с одинаковой плотностью и может быть определена по формуле IF=IT0V, (171) где W — энергия магнитного поля в джоулях; Wq—объемная плотность энергии в джоулях на кубический метр; V— объем, занимаемый магнитным полем, в кубических метрах. Пример 97. Требуется определить энергию, запасенную в магнитном поле весьма длинной цилиндрической катушки, если длина катушки /=1,5 м, площадь поперечного сечения цилиндрического каркаса катушки S = 10 кв. см, число витков катушки w = 150 и величина тока в обмотке катушки I = 2 а. Сердечник катушки сделан из материала, относительная магнитная проницае- мость которого рГ= 1. Решение. Так как каТушка относительно длинная (длиннее диаметра каркаса в несколько десятков раз), то можно считать, что магнитное поле внутри катушки однородно и вся энергия его практически сосредоточена внутри самой катушки. Напряженность магнитного поля катушки согласно формуле (НО) равна //=^- = 1^2 = 200 — . I 1,5 м 295
Объемная плотность энергии магнитного поля в катушке равна Ц7о = = »-4к-10~7- 200^ 25,12 • 10-8 • Объем, занимаемый магнитным полем внутри катушки, V = si= 10-10-*-1,5= 1,5*10-» м\ Магнитное поле катушки обладает энергией W = 1Г0 V = 25,12 • 1О-3 • 1,5 • 10~3 = 3,8 10~5 дж. Энергию магнитного поля катушки можно определить, исхо- дя из величины индуктивности L катушки и тока I в ней. Согласно формуле (171) энергия однородного магнитного поля катушки равна Н7=1Г0Ц где /7 = ^. и v = Sl. Принимая во внимание последние соотношения, можно напи- сать, что W = 1Г0 V = SI = SI, или ту/________________________ W ~~ I ’ 2 * Но, как известно, I ' Следовательно, выражению для определения энергии магнит- ного поля катушки можно придать такой вид: = (172) где W—энергия магнитного поля катушки в джоулях; L—индуктивность катушки в генри; /—величина тока в обмотке катушки в амперах. Пример 98. Определить энергию магнитного поля катушки, если ее индук- тивность L = 0,005 гн, а ток, протекающий в ее обмотке, / = 2 а. Решение. Применяя формулу (172), находим 117= = °’005'2- = 0,01 дж. 296
Формуле (172) можно придать несколько иной вид, выразив ее через ток I и магнитный поток Ф> пронизывающий катушку. Как известно, / __ гс'Ф _ ф L -~~1Т > а поэтому = (173) где ф —число потокосцеплений в катушке в вольт-секундах; / — ток в катушке в амперах; W— энергия магнитного поля в катушке в джоулях. Пример 99. Определить энергию магнитного поля катушки, если число потокосцеплений в катушке ф = 8-10~4 в-сек, а ток в ней / = 5а. Решение. Применяя формулу (173), находим = < = 8'10~4,5=2-10~3 дж.
ЧАСТЬ ВТОРАЯ ПЕРЕМЕННЫЕ ТОКИ ГЛАВА XIX ОСНОВНЫЕ ПОЛОЖЕНИЯ ТЕОРИИ ПЕРЕМЕННОГО ТОКА § 95. ОБЩЕЕ ПОНЯТИЕ О ПЕРЕМЕННОМ ТОКЕ Переменным током называется такой электрический ток, ко- торый с течением времени изменяется по величине и направле- нию или, в частном случае, изменяется по величине, сохраняя свое направление в цепи неизменным. Так как переменный ток в общем случае меняется в цепи не только по величине, но и по направлению, то обычно одно из направлений переменного тока в цепи считают условно положи- тельным, а другое, противоположное первому, — отрицательным. В соответствии с этим и величину переменного тока в первом случае считают положительной, а во втором — отрицательной. Следовательно, можно сказать, что переменный ток — ве- личина алгебраическая; знак его определяется тем, в каком направлении в рассматриваемый момент времени проте- кает переменный ток в цепи — в положительном или отрица- тельном. i (ампер) Рис. 173. Развернутая диаграмма переменного синусоидального тока 298
Величина переменного тока, соответствующая данному момен- ту времени, называется мгновенным значением переменного тока. Максимальное мгновенное значение переменного тока, кото- рого он достигает в процессе своего изменения, называется ам- плитудой тока. График зависимости переменного тока от времени называется развернутой диаграммой переменного тока. На рис. 173 приведе- на развернутая диаграмма переменного тока, изменяющегося с течением времени по величине и направлению. По горизонталь- ной оси Ot отложены в определенном масштабе отрезки време- ни, а по вертикальной оси — величины тока: вверх от начальной точки 0 — положительные, а вниз — отрицательные. Следова- тельно, часть развернутой диаграммы тока, расположенная выше оси времени Ot, характеризует изменение положительных вели- чий тока во времени, а часть, расположенная ниже оси времени Ot, — изменение отрицательных величин. Из диаграммы видно, что в момент времени / = 0 ток равен нулю (/ = 0). Затем он с течением времени растет в положительном направлении, в мо- ', Т мент времени / = — достигает максимального значения, после т чего убывает по величине и в момент времени t = -у становится равным нулю. Затем, пройдя через нулевое значение, ток меняет свой знак на противоположный, т. е. становится отрицательным, растет по абсолютной величине, достигает максимума при з /=уТ, после чего убывает и при t = T становится равным нулю. Развернутая диаграмма переменного тока дает наглядное представление о том, как изменяется величина данного перемен- ного тока во времени, и, кроме того, дает возможность опреде- лить значение тока для любого заданного момента времени, не прибегая к математическим вычислениям. Например, из развер- нутой диаграммы, приведенной на рис. 173, видно, что в момент т т времени t = -g-ток I = 2,8 а, в момент t = -j- ток I = 4 а, в мо- мент t— -у Т ток 1 = —2,8 а и т. д. § 96. НЕКОТОРЫЕ ОСНОВНЫЕ ОПРЕДЕЛЕНИЯ ПЕРЕМЕННОГО ТОКА Периодическим переменным током называется такой электри- ческий ток, который через равные промежутки времени повто- ряет полный цикл своих изменений, возвращаясь к своей исход- ной величине. На рис. 174 приведена развернутая диаграмма периодическо- го переменного тока. Здесь мы видим, что через равные проме- 299
жутки времени Т график тока воспроизводится полностью без каких-либо изменений. Время Г, в течение которого переменный периодический ток совершает полный цикл своих изменений, возвращаясь к своей исходной величине, называется периодом переменного тока. Величина, обратная периоду, называется частотой перемен- ного тока: (174) где f— частота переменного тока; Т — период переменного тока. Если в формуле (174) выразить время Т в секундах, то бу- дем иметь: I Рис. 174. Развернутая диаграмма периодического переменного тока т. е. размерность частоты пере- менного тока выражается в сек ’ Частота переменно- го тока численно рав- на числу периодов в секунду. За единицу измерения часто- ты переменного тока принят 1 герц (1 гц, 1 Hz). Частота переменного тока равна одному герцу, если период этого тока равен одной секунде. В табл. 33 -приведены единицы измерения частоты перемен- ного тока с указанием их условных обозначений и соотношения с основной единицей. Таблица 33 Единицы измерения частоты переменного тока Наименование величины и ее обозначение Название единицы Обозначение Соотноше- ние с основной величиной русское между- народное герц гц Hz — Частота / килогерц кгц kHz 103 гц мегагерц мггц MHz 10б гц гигагерц ггц GHz 109 гц За стандартную частоту переменного тока в сильноточной электротехнике принята частота f = 50 гц. В технике связи при- меняются частоты более -высокие, и в частности в радиотехни- ке — порядка миллионов и миллиардов герц. 300
§ 97. ПЕРЕМЕННЫЙ СИНУСОИДАЛЬНЫЙ ТОК Исключительно важную роль в электротехнике играют сину- соидальные переменные токи. Синусоидальным током называется периодический перемен- ный ток, который с течением времени изменяется по закону си- нуса. Синусоидальный ток — элементарный, т. е. его уже нельзя разложить на другие, более простые переменные токи. Наоборот, всякие сложные периодические токи можно разложить на ряд си- нусоидальных токов, что мы увидим далее, при изучении слож- ных переменных токов. Так как синусоидальный переменный ток — элементарный, т. е. наиболее простой, то естественно, что изложение теории пе- ременных токов обычно начинается с синусоидального перемен- ного тока. В простейшем случае переменный синусоидальный ток мате- матически выражается формулой Z = /„-sin4 (175) где 1т — амплитуда синусоидального тока. Переменная величина со/ в этом. выражении обозначает неко- торый угол, называемый фазой синусоидального тока. Фаза со/ изменяется пропорционально времени /. Множитель со, входящий в выражение фазы со/,— величина постоянная, называемая угловой частотой синусоидального тока. Угловая частота со синусоидального тока зависит от частоты / этого тока и определяется формулой <0 = 2^/ = ^-, (176) где <о— угловая частота синусоидального тока; /—частота синусоидального тока; Т—период синусоидального тока; 2к—центральный угол окружности, выраженный в радианах. Исходя из формулы (176), можно определить размерность угловой частоты: м = [т]=^- <177> так как 2 я — угол в радианах — является безразмерной вели- чиной. В соответствии с этим фаза со/ синусоидального тока измеряется радианами. 301
Напомним, что 1 радиан = 57° 17', угол 90° = -|- радиан, 180° = я радиан, 270° радиан и 360° = 2и радиан, 'где it = 3,14 радиан. Зная амплитуду тока 1т, а также угловую частоту ш, можно определить мгновенное значение переменного синусоидального тока для любого момента времени, пользуясь формулой (175). Пример 100. Требуется определить мгновенное значение переменного си- нусоидального тока i = Iт * sin tot, если известно, что 1т~ 12 а, частота f = 50 гц, а время t = 0,0025 сек. Решение. Угловая частота тока ш = 2к/=2-3,14-50 = 314—. J > сек Фаза тока = 314 • 0,0025 = 0,785 рад. = 0,785тс180° = 45°. Мгновенное значение тока /= 12-sin 45°= 12-0,707 = 7,484 а. Зная мгновенное значение синусоидального тока, можно по- строить его развернутую диаграмму, т. е. нарисовать график за- висимости тока от времени t или от угла со/, величина которого изменяется пропорционально времени. На рис. 175, а приведена векторная диаграмма синусоидального тока, выражающая зави- симость тока от времени /, а на рис. 175, б — векторная диа- грамма, выражающая зависимость этого тока от угла со/. В по- следнем случае мы видим, что периоду Т соответствует угол 2я, т половине периода -%----угол я и т. д. Рис. 175. Развернутые диаграммы синусоидального тока: а — зависимость тока от времени; б — зависимость тока от угла tot 302
В общем случае может оказаться, что в момент времени t = 0, с которого мы начинаем наблюдение за изменением переменного синусоидального тока, мгно- венное значение этого тока не равно нулю. Формула для определения мгновенно- го значения переменного си- нусоидального тока в этом случае принимает следую- щий вид: Z = /m-sin(a)/ + Ф), (178) Рис. 176. Развернутая диаграмма сину- соидального тока где (а>7 + ф)—фаза переменного синусоидального тока; Ф—угол, называемый начальной фазой пере- менного синусоидального тока. Если в формуле (178) положить / = 0, то будем иметь o)Z = O, о>7 + ф = ф и it_Q = Im sin ф. Следовательно, начальная фаза — это фаза сину- соидального тока в момент времени Z = 0. Пример 101. Известно, что амплитуда переменного синусоидального тока 1т=% а, а начальная фаза его ф=30°. Требуется определить мгновенное значение этого тока в момент времени 7 = 0. Решение. Применяя формулу (178), находим ii=Q = Im'sinф = 8• sin30° = 8-0,5 = 4 а. Начальная фаза переменного синусоидального тока может быть положительной (ф>0) или отрицательной (ф<0) величи- ной. В первом случае мгновенное значение синусоидального тока в момент времени t = 0 положительно, а во втором — отрица- тельно. На рис. 176 показана развернутая диаграмма переменного си- нусоидального тока, имеющего начальную фазу ф =-^-) т. е. тока, определяемого по формуле I =7^-sin • Мгновенное зна- чение его в момент времени t = 0 равно * Т * Т lf=0 т ’ SIH ~2~ t т. е. равно положительной амплитуде тока. 303
Рис. 177. Развернутая диаграмма сину- соидального тока На рис. 177 представлена развернутая диаграмма пере- менного синусоидального то- ка, имеющего начальную фа- зу ф = —~ f т. е. тока, опре- деляемого по формуле Мгновенное значение это- го тока в момент времени t = 0 равно h = 0~ * sin 2~ т. е. оно равно отрицательной амплитуде тока. Два переменных синусоидальных тока совпадают по фазе, если они имеют одинаковые фазы и, следовательно, одновременно достигают своих нулевых и максимальных значений одинакового знака. Например, токи Zi=/1/72-sin со/ и Z2 = I2m ’ s^n совпадают по фазе. На рис. 178 представлены развернутые диаграммы этих токов. Два переменных синусоидальных тока сдвинуты по фазе отно- сительно друг друга, если они имеют различные фазы. Например, токи Zi =/b72-sin((i)Z + и Z2 = /2/72-sin сдвинуты по фазе относительно друг друга на угол , так как 304
В этом случае ток h в своих изменениях опережает по фазе ток /2 на угол или> иначе, ток /2 отстает по фазе относительно тока Zi на угол -у. На рис. 179 представлены развернутые диа- граммы двух переменных токов, сдвинутых по фазе относительно друг друга, на угол -у . § 98. ПРОСТЕЙШИЙ СПОСОБ ПОЛУЧЕНИЯ СИНУСОИДАЛЬНОЙ ЭЛЕКТРОДВИЖУЩЕЙ СИЛЫ Переменный синусоидальный ток может проходить в такой электрической цепи, где действует синусоидальная электродвижу- щая сила. Рис. 180. Контур abed вращается вокруг своей оси тп с равномерной скоростью в однородном магнитном поле Рассмотрим простейший способ получения переменной сину- соидальной электродвижущей силы. Допустим, чточв однородном магнитном поле (рис. 180) вра- щается, вокруг своей оси тп прямоугольный контур abed. Движе- ние его .совершается с равномерной угловой скоростью w в на- правлении против движения часовой стрелки, как это показано стрелкой на левой стороне рис. 180. Стороны ab и cd, равные по- рознь /, являются активными сторонами контура, так как они при своем движении в магнитном поле пересекают магнитные линии и в них индуктируется электродвижущая сила. Другие две сто- роны, Ьс и da, контура, равные порознь а,— не активные, так как 20—1377 3 05
они при своем движении в магнитном поле скользят вдоль маг- нитных линий и в них электродвижущая сила не индуктируется. Назовем положение контура abed в магнитном поле нейтраль- ным, когда он своей плоскостью расположен перпендикулярно магнитным линиям, а плоскость 00, в которой он находится в этот момент времени, назовем нейтральной плоскостью. Активные стороны ab и cd контура, перемещаясь по окружно- а сти радиусом -у, пересекают магнитные линии под некоторым углом а, изменяющимся с поворотом контура. Величина а в лю- бой момент времени равна углу at поворота контура относи- тельно нейтральной плоскости, так как эти углы имеют взаимно перпендикулярные стороны. Сам же угол wt при вращении кон- тура изменяется пропорционально времени /, так как угловая ско- рость со вращения контура по условию постоянна (со = const). В каждой из активных сторон ab и Ьс контура индуктируется электродвижущая сила, величину которой можно определить по формуле ег = Blv • sin (tit и е2 = Blv • sin (со/ + к) == — Blv • sin ю/, где ех и е2—мгновенные значения электродвижущих сил, ин- дуктированных соответственно в активных сто- ронах ab и cd контура abed, в вольтах; В—магнитная индукция магнитного поля в вольт- секундах на квадратный метр; Z—длина каждой из активных сторон ab и cd кон- тура в метрах; v—линейная скорость движения активных сторон а контура по окружности радиусом у-в метрах в секунду; (tit и —углы, под которыми активные стороны контура пересекают магнитные линии. Так как электродвижущие силы, индуктированные в активных сторонах контура, действуют согласно друг с другом, что отме- чено стрелками на правой стороне рис. 180, то результирующая электродвижущая сила, индуктируемая в контуре, будет равна е = 2Blv • sin (tit, (179) т. е. индуктированная электродвижущая сила в контуре изме- няется по синусоидальному закону. Следовательно, если контур вращается в однородном магнит- ном поле с равномерной угловой скоростью, то в нем индукти- руется синусоидальная электродвижущая сила. Преобразуем формулу (179), выразив ее через максимальный магнитный поток , пронизывающий контур abed. Линейная скорость v активных сторон ab и cd контура abed 306
равна произведению радиуса вращения ~ контура на угловую Л т-г скорость ф, т. е. v = ф. Подставив данное выражение для v в формулу (179), получим е = 2В1 ~ ф • sin фА Но la = S, где S — площадь контура abed, и, следовательно, В1а = В8=Фт, где Фт— максимальный магнитный поток, про- низывающий контур abed. В соответствии с этим последняя формула приобретает сле- дующий вид: е = фФш • sin ф А (180) где фФ^—амплитуда синусоидальной электродвижущей силы; —фаза синусоидальной электродвижущей силы; ф—угловая частота синусоидальной электродвижущей силы, в данном случае равная угловой скорости вра- щения контура в магнитном поле. Если в однородном магнитном поле будет вращаться с равно- мерной скоростью рамка, имеющая w витков, то индуктирован- ная электродвижущая сила в ней будет в w раз больше, чем в контуре, имеющем только один виток, т. е. в этом случае элек- тродвижущая сила, индуктированная в рамке, будет равна е = w® Фт- sin фА (^81) Пример 102. В однородном магнитном поле, магнитная индукция которого В = 0,8---—, вращается квадратной формы рамка, каждая сторона которой I = 0,5 м. Определить электродвижущую силу, индуктированную в рамке, если рамка имеет число витков w = 50 и вращается со скоростью п = = 1500 об/мин. Решение. Площадь рамки 5 = Z2 = 0,52 = 0,25 м\ Максимальный магнитный поток, пронизывающий рамку, ФЛ/2 = BS = 0,8 • 0,25 = 0,2 в-сек. Угловая частота, в данном случае равная угловой скорости вращения рамки, равна 2кп 2-3,14-1500 1 1 Ф = -рТГ = ----Еп--- = 157 ---- . G0 60 сек Электродвижущая сила в рамке е — • sin ш/ = 50 • 157 • 0,2 • sin 1571, или е= 1570- sin 157/ вольт. 20* 307
§ 90. ВЕКТОРНАЯ ДИАГРАММА СИНУСОИДАЛЬНОГО ТОКА При расчете цепей переменного тока приходится производить сложение и вычитание синусоидальных токов или электродви- жущих сил, имеющих одинаковую частоту, но в общем случае различные амплитуды и начальные фазы. Решение подобных за- дач значительно облегчается, если применить метод вектор- ных диаграмм, основанный на изображении синусоидаль- ного тока или электродвижущей силы с- помощью вращающихся векторов. Возьмем прямоугольную систему осей MON (рис. 181). Ось ОМ, расположенную по горизонтальному направлению вправо от точки О, назовем начальной осью, так как от нее мы будем отсчитывать углы поворота векторов тока и элек- тродвижущей силы. При этом условимся откладывать положи- тельные углы от оси ОМ в направлении, противоположном на- правлению вращения часовой стрелки, а отрицательные углы — по часовой стрелке. Теперь предположим, что задан синусоидальный ток, мгно- венное значение которого определяется формулой /= sin w/. Зададимся некоторым произвольным масштабом тока и в этом масштабе представим амплитуду заданного тока /т в виде неко- торого отрезка прямой, который обозначим в виде вектора тока 1т. Совместим начало вектора тока 1т с точкой О пересе- чения осей ОМ и NN' на рис. 181, а сам вектор тока 1т распо- ложим вправо от точки О по оси ОМ. Так как вектор тока 1т со- вмещен по направлению с осью, то угол между ними равен нулю. Теперь допустим, что в момент времени t = 0 век- тор тока начал вращаться вокруг точки О в направ- лении против движения часовой стрелки с постоян- ной угловой скоростью (0. Спустя время h вектор тока 1т займет положе- ние на плоскости, опреде- ляемое углом (D^i поворо- та его относительно на- чальной оси ОМ (начала отсчета углов). Этот угол легко определить, если известны угловая ско- рость со вращения векто- ра тока 1т и фиксирован- ное время /1. Если из конца вектора тока 1т опустить перпен- 308
дикуляр на вертикальную ось ON, то он на этой оси отсечет отре- зок Оа, равный проекции вектора Iт на эту ось. Из прямоуголь- ного треугольника 01 та находим, что Оа = 7^-sin о)^. Но, как известно, Im-sin есть мгновенное значение задан- ного синусоидального тока, соответствующее моменту времени t±. А это значит, что отрезок Оа (проекция вектора тока 1т на вер- тикальную ось NN') пропорционален мгновенному значению тока I. Следовательно, если заранее нанести на вертикальную ось AW' масштаб тока, то можно определить мгновенное значе- ние тока для момента времени а также для любого другого момента времени (/2, /з и т. д.). Графическое изображение синусоидаль- ного тока с помощью вращающегося вектора тока 1т называется векторной, диаграммой этого тока. Пользуясь векторной диаграммой тока Imt легко построить его развернутую диаграмму. На рис. 182 показан метод построе- ния развернутой диаграммы переменного синусоидального тока по его векторной диаграмме. Здесь слева, изображена векторная диаграмма тока 1т, а справа — его развернутая диаграмма I = = 7m-sin о)С Если начальная фаза синусоидального тока отлична от нуля, например ф > 0, то в момент времени t = 0 вектор тока 1т за- нимает на векторной диаграмме положение под углом ф к на- чальной оси. На рис. 183 показана векторная диаграмма тока / = /wsin (ot + ф). 309
Если начальная фаза синусоидального тока отрицательна, т. е. ф < 0, то в момент времени t = 0 вектор тока 1т занимает на векторной диаграмме положение под углом — ф к начальной оси. На рис. 184 показана векторная диаграмма тока / = 7m-sin (ш/—ф). Теперь посмотрим, как осуществляется сложение и вычитание синусоидальных токов с помощью векторных диаграмм. Допустим, что требуется сложить два синусоидальных тока одинаковой частоты: h = Цт • sin + фт) и z2 = 12т * sin + Ф2) • Так как эти токи имеют одинаковую частоту, то их векторы 11т Рис. 184. Векторная диаграмма синусоидального тока i = 1т • sin (ш/ — ф) 310
и 12т можно изобразить на одной и той же векторной диаграмме (рис. 185). Отложим вектор /1от на векторной диаграмме под углом ф1 к начальной оси, а вектор 12т — под углом фг. Затем сложим геометрически векторы Iim и 12гп, как обычные векторы, по правилу параллелограмма и получим некоторый результирую- щий ток Iт “Ь^2/72- (182) Применяя известную из курса тригонометрии формулу для определения стороны косоугольного треугольника, можно выра- зить зависимость 1т от 1Хт и 12т в виде формулы 1т = У Р1т + Р2т + 2Z1OT/Sm • cos (ф2 — ) , (183) т. е. амплитуда результирующего синусоидаль- ного тока равняется корню квадратному из суммы квадратов амплитуд слагаемых сину- соидальных токов плюс удвоенное произведе- ние этих амплитуд на косинус угла разности начальных фаз этих токов. Начальную фазу ф резуль- тирующего тока можно опре- делить по тангенсу этого угла из прямоугольного тре- угольника ONIm (рис. 185): tg^ = Л/к-sin ф1 + 72^-5Ш ф2 (184^ Л/Я-СОЗф! + 72/й-СО5ф2- ' ' Определив tg ф, найдем угол ф по тригонометриче- ским таблицам для танген- сов. После того как будут определены 1т и ф, можно на- писать выражение для мгно- венного значения результирующего синусоидального тока: i = (а)/ + ф). Пример 103. Требуется определить результирующее напряжение, полу- чаемое от сложения двух синусоидальных электродвижущих сил одинаковой частоты, если известно, что = 60 • sin( wZ + 30°) и е2 = 40 • sin( wZ + 60°). Решение. В заданном примере £1^=60 в, Е2т = 40 в, ф; = 30° и ф2 = 60°. 311
Применяя формулу (183), найдем амплитуду результирующего синусо- идального напряжения: Ет = У ^m+^m + 2^2m-cos(<p2-<p1) = = У 602 -F 402 4- 2 • 60 • 40 cos (60° — 30°) s 96 в. Применяя формулу (184), найдем начальную фазу результирующего си- нусоидального напряжения: , , 4- - sin ф2 60-sin 30° + 40-sin 60° Z?iw-cos ipj + E2m-cos ф2 60-cos 30° + 40-cos 60° __ 60-0,5 + 40.0,866 60-0,866 + 40.0,5 “u>yuo- По тригонометрическим таблицам находим, что ф = 42°10'. Итак, результирующая синусоидальная электродвижущая сила равна г = 96 -sin (otf+ 42°10'). Когда требуется найти разность двух синусоидальных токов одинаковой частоты, например токов + Ф1) И z2 = /2m-sin (<0^ + ф2), на векторной диаграмме необходимо произвести геометрическое вычитание векторов токов /1Я1 и /2пг: 1т = 1хт-1гт- (185) Известно, что вычесть из одного вектора другой — значит к первому прибавить второй, но взятый с обратным знаком, т. е. с противоположным направлением. Следовательно, чтобы вычесть из* вектора Iim вектор /2ш, надо провести на векторной диаграмме вектор — /2^, т* е« вектор, 312
равный по абсолютной величине вектору 12т, но противополож- ный ему по знаку, а затем векторы /1П2 и (—У2/я) геометрически сложить: + (—Л/п) • На рис. 186 показана векторная диаграмма разности векто- ров токов Цт и 12т. Приведенные выше положения о геометрическом сложении векторов синусоидальных токов одинаковой частоты можно рас- пространить на все синусоидальные величины, применяемые в теории переменных токов, например, на синусоидальные элек- тродвижущие силы, синусоидальные напряжения и т. д. В даль- нейшем мы широко используем метод векторных диаграмм для исследования режимов в цепях переменного синусоидального тока. § 100. ДЕЙСТВУЮЩЕЕ ЗНАЧЕНИЕ ПЕРЕМЕННОГО СИНУСОИДАЛЬНОГО ТОКА Если все положительные и отрицательные мгновенные зна- чения синусоидального переменного тока сложить, то сумма их будет равна нулю, так как каждому положительному мгновен- ному значению синусоидального тока всегда найдется равный ему по абсолютной величине отрицательный мгновенный ток (рис. 187). Но если алгебраическая сумма всех мгновенных значений синусоидального тока за период равна нулю, то и среднее зна- чение этого тока за период также равно нулю: Следовательно,сред- нее значение синусои- дального тока за пе- риод не может служить для измерения этого то- ка. Если амперметр, ко- торый измеряет среднее значение тока, включить в цепь переменного си- нусоидального - тока, то он покажет величину тока, равную нулю, хотя фактически этот ток су- ществует в цепи и про- изводит некоторую ра- боту, например выде- ляет тепло. Возникает вопрос, что же в данном случае Рис. 187. Алгебраическая сумма мгновенных значений тока за период равна нулю 313
может служить мерой измерения синусоидального тока и какую, величину тока показывают приборы, включенные в цепь перемен- ного синусоидального тока, если они не измеряют среднее его значение за период. Чтобы судить о величине переменного синусоидального тока, его сравнивают с током постоянным по их тепловому действию. Два ток а, из которых один синусоидальный, а другой постоянный, эквивалентны по тепло- вому действию, если они, протекая по одинако- вым сопротивлениям, за одинаковые отрезки времени выделяют одинаковое количество тепла. Например, если постоянный ток /пост = 5 а, протекая через сопротивление г= 10 ом, за время t= 10 сек. выделит ко- личество тепла Q = 0,24 /I 2ост rt = 0,24 • 52 • 10 • 10 — 600 малых калорий, то переменный синусоидальный ток, эквивалентный данному постоянному току, выделит в сопротивлении г == 10 ом за время /=10 сек. то же количество тепла. Следовательно, величину синусоидального переменного тока /пер, эквивалент- ного данному постоянному току /пост = 5 а, необходимо считать равной также пяти амперам, т. е. Лтер === Л10СТ === 5 Л. Величину переменного тока, эквивалентную постоянному то- ку и, следовательно, равную ему численно, называют действую- щим значением переменного синусоидального тока. Следовательно, действующее значение переменного синусои- дального тока численно равно току постоянному, эквивалент- ному данному синусоидальному току, т. е. выделяющему порознь с ним в одинаковом сопротивлении за одинаковый отрезок вре- мени одинаковое количество тепла. Математический анализ, который мы здесь не можем при- вести, показывает, что величина действующего значения пере- менного синусоидального тока находится в строго определенной зависимости от амплитуды этого тока: 1 = ^, (186) т. е. действующее зна ч^н ие/ переменного сину- соидального ток-а в У 2 Раз меньше амплитуды этого тока. Пример 104. Определить действующее значение переменного синусоидаль- ного тока, если амплитуда этого тока 1т — 28,2 а. Решение. Применяя формулу (186), находим I = = — = 20 а 1 У 2 1.41 ' 314
Амперметр, например тепловой, электромагнитной или элек- тродинамической системы, включенный в цепь переменного сину- соидального тока, показывает действующее значение тока. В некоторых случаях практики приходится интересоваться средним значением переменного синусоидального тока за поло- вину периода, например за положительную полуволну сину- соиды. Математический анализ показывает, что среднее значение переменного синусоидального тока за половину периода равно 1 (т}=^^ = 0,637/т. (187) Пример 105. Определить среднее значение переменного синусоидального тока за половину периода (за положительную полуволну синусоиды), если амплитуда тока 1т = 25 а. Решение. Применяя формулу (187), находим I = 0,6371т = 0,637• 25 = 15,9 а. ср ы При исследовании ^кривых переменного тока важно бывает знать отношение действующего значения тока к среднему за по- ловину периода (положительную полуволну). Это отношение для синусоидального тока равно k =--------= -^-= —7== l-1L J ( г \ — / 2/2 Мт) к ” Аналогично действующему значению переменного синусои- дального тока можно говорить о действующем значении пере- менной синусоидальной электродвижущей силы или переменного синусоидального напряжения. __ Действующее значение напряжения в у 2 меньше его ампли- туды. Вольтметр тепловой, электромагнитной или электродина- мической системы, включенный в цепь переменного тока, пока- зывает действующее значение синусоидального напряжения. Пример 106. Определить амплитуду переменного синусоидального напря- жения, если действующее значение его U = 100 в. Решение. Амплитуда напряжения £/„=)/2 £/=*1,41-100 = 141 в.
ГЛАВА XX ПРОСТАЯ ЦЕПЬ ПЕРЕМЕННОГО ТОКА С ИНДУКТИВНОСТЬЮ И АКТИВНЫМ СОПРОТИВЛЕНИЕМ § 101. АКТИВНОЕ СОПРОТИВЛЕНИЕ Активным сопротивлением в цепях переменного тока назы- вают сопротивление, в котором происходит необратимый про- цесс превращения электрической энергии в какой-либо иной вид энергии, например в тепловую. Примером активного сопротивле- ния может служить сопротивление проволоки реостата, проводов цепи, нити электрической лампы и т. д., где электрическая энер- гия преобразуется в тепловую энергию, а последняя рассеи- вается. Вполне очевидно, что эта тепловая энергия не может быть преобразована в электрическую энергию в данной цепи. О величине активного сопротивления проводника в общем случае можно судить по той мощности, которая развивается источником электрической энергии в этом проводнике: г = -7г, (188) где г — активное сопротивление проводника в омах; / — действующее значение переменного тока в этом провод- нике в амперах; Р — мощность, расходуемая в активном сопротивлении про- водника, в ваттах. Пример 107. Мощность, потребляемая в активном сопротивлении потре- бителя, Р = 800 вг, а действующее значение тока в нем / = 2,5 а. Опре- делить активное сопротивление этого потребителя. Решение. По формуле (188) находим _ Р __ 800 __ 800 _ Г ~~ Р ~ 2,52 — 6,25 — 128 ом. 316
§ 102. ЦЕПЬ ПЕРЕМЕННОГО ТОКА С АКТИВНЫМ СОПРОТИВЛЕНИЕМ Пусть некоторое активное сопротивление г подключено к источнику электрической энергии (рис. 188), обладающему переменным синусоидальным напряжением u = (1’89) где и—мгновенное значение напряжения в вольтах; Um— амплитуда напряжения в вольтах; со—угловая частота; t— время в секундах; со/— фаза напряжения, выраженная в радианах. В данном случае для мгновенных значений переменного на- пряжение и переменного тока справедливы законы Ома, Кирхгофа и Ленца'—ДжОуля, так как всякий переменный ток в течение очень малого отрезка времени Д/ можно рассматривать как ток постоянный. Последнее тем справедливей, чем меньше промежу- ток Д/, и абсолютно справедливо, если Д/1 принять за бесконечно малый промежуток времени. Следовательно, на основе закона Ома i = (190) где I — мгновенное значение тока в амперах. Подставив в формулу (190) вместо и его выражение из фор- мулы (189), получим / = — -sin а)/, (191) или, иначе, i = Im-sin Ч (192) где 1т = ~ — амплитуда тока. (193) На основании формулы (192) можно сделать следующие выводы: 1. В цепи с активным сопротивлением при синусоидальном напряжении прохо- дит синусоидальный переменный ток. 2. Напряжение и ток в цепи с актив- ным сопротивлением совпадают по фазе, т. е. одновременно достигают своих нуле- вых и максимальных значений одинако- вого знака. Рис. 188. Простая цепь переменного тока с активным сопротивле- нием 317
Рис. 189. Векторная и плоская диа- граммы тока и напряжения для цепи с активным сопротивлением мулы (193) разделить на у 2 , На рис. 189 приведены векторная и плоская диа- граммы напряжения и тока для цепи с активным со- противлением. Из формулы (193) следует, что амплиту- да тока Iт пропорциональна амплитуде напряжения Um, а следовательно, и дей- ствующее значение тока I должно быть пропорцио- нальным действующему зна- чению напряжения U. Если левую и правую части фор- то получим lm Um Т и , или I = — J/2 /2 -Г ’ Г (194) Формула (194) является математическим выражением за- кона Ома для цепи с активным сопротивлением. Мгновенная мощность р, поглощаемая в активном сопротивлении г, равна произведению мгновенного значения на- пряжения на мгновенное значение тока: р = ui = UmIm • sin2 о>А (195) Из формулы (195) следует, что мгновенная мощность изме- няется пропорционально sin2 (о^, т. е. это не синусоидальная вели- чина. На, рис. 190 показан график мгновенной мощности р. Максимальное значение мгновенной мощности равно Р I Среднее значение мощности, потребляемой активным сопротивлением за период Г, равно половине мгновенной максимальной мощности: р ______ UrJm *ср 2 2 = .2^ /2, У 2 9 или Pcp = UI, (196) Рис. 190. График мгновенной мощности для цепй с активным сопротивлением 318
т. е. средняя мощность, потребляемая активным сопротивлением за период, равна произведению действующего значения напря- жения на действующее значение тока. Следует отметить, что в отличие от цепи постоянного тока в цепи переменного тока с активным сопротивлением энергия от источника подается к потребителю не непрерывным равно- мерным потоком, а в виде двух импульсов за период. О вели- чине энергии, потребляемой активным сопротивлением, можно су- дить по величине площадок, ограниченных осью времени и гра- фиком мгновенной мощности (рис. 191). Величина этой энергии за период Т равна W = UH\ (197) где W — энергия в джоулях. Так как согласно закону Ома U=Ir, то вместо выражения (197) можно написать W = I2rT. Энергия в таких активных со- противлениях, как реостаты, обыч- ные металлические провода и т. п., расходуется на образование теп- ловой энергии. Пример 108. К цепи с активным со- противлением г = 20 ом приложено напряжение и = 169,2 • sin 314/. Опреде- лить режим работы тока в данной цепи. Решение. Действующее напряже- ние (198) Рис. 191. Величина энергии, по- требляемая активным сопроти- влением, определяется площадью заштрихованных площадок _ Um ___ 169,2 _ ]/2 Ml 120 в. Действующее значение тока U 12° — Г ZY 20 — 6 а- Средняя мощность за период P=UI= 120-6 = 720 etn. Частота переменного тока , О) 314 г-п f ~ 2я — 2-3,14 — гЦ- 319
Период переменного тока т= 4= 4т = 0,02 сек. Т оО ’ Энергия, расходуемая током за период Tt UZ= РГ = 720-0,02 = 14,4 дж. § 103. ИНДУКТИВНОСТЬ В ЦЕПИ ПЕРЕМЕННОГО ТОКА Допустим, что в катушке без железного сердечника, обла- дающей индуктивностью L и активным сопротивлением г 0 (рис. 192), проходит переменный синусоидальный ток i = Im-sin Он сопровождается переменным синусоидальным магнитным потоком, совпадающим с ним по фазе (рис. 193). Рис. 193. Магнитный поток в катушке без стали совпа- дает по фазе с переменным током Рис. 192. Цепь переменного тока с катушкой индуктив- ности, активное сопроти- вление которой практически равно нулю Переменный магнитный поток индуктирует в катушке элек- тродвижущую силу самоиндукции eLi пропорциональную скоро- сти изменения тока: eL = -l%, (199) где eL — индуктированная электродвижущая сила самоиндук- ции в вольтах; А/ — скорость изменения тока в амперах в секунду; L — индуктивность катушки в генри. Знак минус, стоящий в формуле (199), отображает правило Ленца, согласно которому индуктированная электродвижущая сила самоиндукции всякий раз возникает такого направления, при котором она противодействует всем изменениям тока. Если 320
ток возрастает, то. опа направле- на навстречу ему, а если умень- шается,— попутно с током. На основании этих соображе- ний построим развернутую диа- грамму электродвижущей силы самоиндукции, пользуясь развер- нутой диаграммой тока. В первую четверть периода ток I нарастает в положительном направлении и электродвижущая сил а с а мои н д у кци и, п роти во дей - ствуя его росту, должна быть направлена навстречу ему, т. е. должна иметь отрицательный знак. Величина тока I имеет наи- большую скорость изменения во времени при переходе через нуле- вое значение и наименьшую — при переходе через максимальное значение. На рис. 194 показано, времени А/ приращение тока А/ Рис. 194. Скорость нарастания Д/ л тока по мере приближения тока к максимуму уменьшается что за один и тот же отрезок больше при значениях тока, близких к нулевому. Следовательно, электродвижущая сила самоиндукции макси- мальна при переходе через нуль, т. е. в момент, когда отношение Ы максимально, и она равна нулю, когда ток I переходит через максимум, т. е. когда -д^=0 (рис. 195). Во вторую четверть периода ток I, оставаясь положитель- ным, убывает по величине. Электродвижущая сила самоиндук- ции, противодействующая его убыванию, должна быть направ- синусоидального тока и индуктированной синусо< идальнбй электродвижущей силы самоиндукции 21—1377 321
лена попутно с Током, т. е. должна Также иметь положительный знак. Величина тока I по мере приближения к нулю увеличи- д/ вает скорость своего изменения, и достигает максимального значения при переходе через нуль. Следовательно, электродви- жущая сила самоиндукции в начале второй четверти периода нарастает в положительном направлении *и к концу второй чет- верти периода, когда ток проходит через нуль, достигает макси- мального положительного значения. В третью четверть периода ток I становится отрицательным и растет по абсолютному значению, уменьшая этот рост по мере приближения к максимуму. В связи с этим электродвижущая сила самоиндукции, противодействуя росту тока, имеет положи- тельное направление и с ростом его уменьшается, так как при- ращения тока Д/ в единицу времени становятся все меньшими и меньшими по мере приближения тока к амплитуде. В четвертую четверть периода док I убывает, стремясь к нулю. Электродвижущая сила самоиндукции, противодействуя его убы- ванию, имеет одинаковое направление с ним, т. е. она по знаку отрицательна, как и ток. Ток I по мере приближения к нулю увеличивает интенсивность своего изменения, достигая макси- мума при переходе через нуль. В соответствии с этим электро- движущая сила самоиндукции7 по мере приближения тока I к нулю растет и при переходе через нуль становится максималь- ной по абсолютной величине. Сопоставляя развернутые диаграммы тока I и электродвижу- щей силы самоиндукции eL (рис. 195), нетрудно видеть, что электродвижущая сила самоиндукции eL отстает по фазе от тока на четверть периода или, иначе, на угол у. Следовательно, если переменный синусоидальный ток, проте- кающий по обмотке катушки, определяется по формуле i = /w-sin atf, то электродвижущая сила самоиндукции в катушке будет опре- деляться по формуле eL = Еьт •sin ----2") • (200) Величина электродвижущей силы самоиндукции, как извест- но, пропорциональна скорости изменения тока; ^ = -1-^-, (201) т. е., иначе сказать, она пропорциональна частоте тока, так как чем больше частота тока, тем больше скорость его изменения во времени. 322
Одновременно с этим величина электродвижущей силы само- индукции зависит, как это видно из формулы (201), от индуктив- ности L катушки. Чем больше индуктивность катушки, тем больше при прочих равных условиях электродвижущая сила самоиндукции в катушке. Следовательно, электродвижущая сила самоиндукции ка- тушки пропорциональна частоте тока в катушке и величине индуктивности ее. В соответствии с этим формула для определе- ния действующего значения электродвижущей силы самоиндук- ции имеет следующий вид: El=^LI, (202) где El — действующее значение электродвижущей силы само- индукции в вольтах; /—действующее значение тока, протекающего в катуш- ке, в амперах; L—индуктивность катушкц в генри; со — угловая частота тока. Пример 109. Определить действующее значение электродвижущей силы самоиндукции в катушке, если ток, протекающий в обмотке катушки, /=2 й, индуктивность катушки L = 0,01 гн и угловая частота со = 1000— . Решение. Применяя формулу (202), находим: EL = (dZZ = 1000 • 0,01 • 2 = 20 в. Помножив левую и правые части равенства (202) на И2 , получим выражение для амплитуды электродвижущей силы са- моиндукции V2 EL = V2 &LI, или, иначе, Еьт = ^1т, (203) где ЕЬт—амплитуда электродвижущей силы самоиндукции; 1т—амплитуда тока. В соответствии с этим формуле (200) для определения мгно- венного значения электродвижущей силы самоиндукции можно придать такой вид: eL = ELm * sin ---y) = * sin ----r) • (204) Так как активное сопротивление катушки по условию равно нулю, то падение напряжения в нем также равно нулю. В этом случае все напряжение, приложенное к катушке, целиком идет на преодоление электродвижущей силы самоиндукции катушки. 21* 323
А это значит, что мгновенные значения напряжения и и электро- движущей силы eL в любой момент равны по абсолютной вели- чине и противоположны по знаку: u = — eL, (205) или согласно формуле (204) м = — a>LIm • sin (a>t--. Но, как известно из тригонометрии, — sin (at----= sin , а поэтому и — <n>LIm sin (®t 4- . (206) На основании формулы (206) можно сделать следующий вывод: если в катушке индуктивности проходит синусоидальный ток 1 = Im-sin то напряжение и, приложенное к этой катуш- ке, также синусоидально и опережает по фазе ток на угол . Угол сдвига фаз напряжения и и электродвижущей силы самоиндукции eL равен л, так как (®t+ т*)-—r)=1t’ т. е. напряжение и опережает по фазе электродвижущую силу самоиндукции eL на угол л. На рис. 196 приведены векторная и развернутая диаграммы напряжения, тока и электродвижущей силы самоиндукции для рассматриваемого нами режима в катушке индуктивности. Из формулы (206) следует, что максимума напряжение и достигает в тот момент, когда sin (vt + — 1. Следовательно, амплитуда напряжения (207) Разделив левую и правую части последнего равенства на К2, получим выражение для действующего значения напряжения: = u>L , /2 у 2 или, иначе, U = IwL. (208) 324
Отсюда можем получить формулу закона Ома для чисто ин- дуктивной цепи: ' = £. ' (209) где / — действующее U — действующее к катушке. значение тока в катушке; значение напряжения, приложенного Рис. 196. Векторная и развернутая диаграммы тока, напряжения и электродвижущей силы са- моиндукции Величина <oL, входящая в формулу (209), представляет собой индуктивное сопротивление катушки и обычно обозначается сим- волом xL = a>£, (210) где xL — индуктивное сопротивление в омах; L — индуктивность катушки в генри; <о — угловая частота в . Из формулы (210) следует, что напряжение U, приложенное к катушке, равно произведению тока / на индуктивное сопротив- ление катушки, т. е. в рассматриваемом случае оно целиком расходуется в индуктивном сопротивлении катушки. Но, с дру- гой стороны, известно, что напряжение Ц, приложенное к катуш- ке, целиком компенсируется электродвижущей силой самоиндук- ции El катушки, т. е. |47| = |££| = /<оД 325
Отсюда следует, что индуктивное сопротивление катушки обусловлено противодействием элек- тродвижущей силы самоиндукции. Из двух катушек, по которым протекает одинаковый сину- соидальный ток, большим индуктивным сопротивлением обла- дает та катушка, в которой индук- . S' тируется большая электродвижущая j / сила самоиндукции. S' Следовательно, индуктивное со- противление по своей природе отли- S^ чается от активного (например, ак- тивиого сопротивления металличе- ского проводника), обусловленного Рис. 197. График зависимости противодействием току элементар- индуктивного сопротивления ных частиц вещества. катушки от частоты тока Из формулы (210) следует, что индуктивное сопротивление катуш- ки прямо пропорционально частоте тока, протекающего через катушку, и индуктивности катушки. На рис. 197 показан график зависимости индуктивного сопро- тивления катушки от частоты тока. Пример 110. Определить индуктивное сопротивление катушки, обладаю- щей индуктивностью L — 0,02 гн, при частотах fi = 50 гц, ft = 1000 гц и = 1 000 000 гц, а также определить ток в катушке, если приложенное к ней напряжение U = 125,6 в и активное сопротивление катушки г = 0. Решение. По формуле (210) находим: xLi = = 2 • 3,14 • 50 • 0,02 = 6,28 ом; xl* = <»2L = = 2-3,14-1000-0,02 = 125,6 ом; xl9 = = = 2-3,14-1 000000-0,02 = 125600 ом. Ток в катушке; U 125,6 Qn „ -------6^8- = 20a’ Li ' — 125,6 _ 1 2 ““ 125,6 — 1 a> /s— 125 600“ 0,001 a' Из приведенного примера видно, какое влияние на величину тока в катушке оказывает частота приложенного к ней напряже- ния. При очень высоких частотах ток через катушку, обладаю- щую большой индуктивностью, практически не проходит. При частоте f= 0 индуктивное сопротивление равно нулю. Пример 111. Определить индуктивность катушки, активное сопротивле- ние которой г = 0, если при напряжении U = 220 в, приложенном к ней, и частоте / = 50 гц ток в ней / = $ а. 326
Решение. Индуктивное сопротивление катушки т и 220 .. xL = = _ = — = 44 ол/, * / э откуда t 44 44 44 п 1 /1 £ =— ="-й—г = о"о 1л-сА —0>И гн. со 2тс/ 2-3,14-50 > 'Мгновенная мощность р в катушке равна произве- дению мгновенного значения напряжения йь.на мгновенное значе* ние тока I. Следовательно, для данного случая р -ui — Um- sin (wt + Im • sin о>/. Из курса тригонометрии известно, что sin (mt + = sin ш/- cos-^- + cos sin = cos так как cos -у = 0 и sin -у- = 1, а поэтому р — UmIm • cos mt • sin <oA Но, как известно, 2 sin mt • cos mt = sin 2®/, а поэтому . . . sin 2cof Sin <0/ • COS — И p _ Ujnlm,. sjn _ ILnL . . sjn 2o)^, ^2 |/2 J/2 ИЛИ p = tZZsili 2юД (211) Из формулы (211) следует, что мгновенная мощность, разви- ваемая током в катушке, есть величина синусоидальная, но из- меняющаяся с двойной частотой по сравнению с частотой тока и напряжения, так как фаза ее равна 2ш/. В момент времени t = 0 величина р = 0, так как I = 0, Т а и = Um. В момент t = -j- величина р — 0, так как I = Im< 327
Т 3 а = 0. Точно так же р = 0 при t = , t = -^ Т и ^ = 7*. В первую четверть периода мгновенная мощность положительна, так как произведение положительного напряжения на положи- тельный ток — величина положительная. Во вторую четверть периода мгновенная мощность отрицательна, так как мгновенные значения напряжения и тока имеют различные знаки. В третью четверть периода мгновенная мощность положительна, так как мгновенные значения напряжения и тока одинакового отрица- тельного знака. И, наконец, в четвертую четверть периода мгно- венная мощность отрицательна, так как мгновенные значения тока и напряжения имеют противоположные знаки. На рис. 198 показан график мгновенной мощности р. Рис. 198. График изменения мгно- венной мощности для катушки, в которой активное сопротивление Г = О Рис. 199. Колебания энергии в цепи с катушкой индуктив- ности, активное сопротивление которой Г = О Среднее значение мощности за период, очевидно, будет равно нулю; ЛР=0, (212) так как при сложении всех положительных и отрицательных зна- чений мощности за период, изменяющийся по синусоиде, полу- чим сумму, равную нулю. Это значит, что среднее значение энергии, потребляемой ка- тушкой за один период, равно нулю. В те моменты времени, когда мгновенная мощность положительна (первая и третья четверти периода), катушка забирает энергию от генератора, запасая ее в своем магнитном поле: (213) где W — энергия магнитного поля в джоулях, запасаемая за четверть периода; L— индуктивность в генри; 1т — максимальная величина тока в амперах. 328
В момент времени, когда мгновенная мощность отрицательна (вторая и четвертая четверти периода), катушка сама как бы становится генератором электрической энергии и возвращает энергию генератору. Заштрихованные площадки на рис. 199, ограниченные осью времени и кривой мгновенной мощности р, дают представление о течении энергии в цепи с катушкой, активное сопротивление которой практически равно нулю. Площадки, расположенные выше линии абсцисс, определяют положительную энергию, т. е. энергию, поступающую от источника к катушке, а площадки, расположенные ниже линии абсцисс, определяют отрицательную энергию, т. е. энергию, возвращаемую катушкой источнику. Следует особо отметить, что хотя источник электрической энергии питает током катушку, средняя мощность, развиваемая в ней, равна нулю, так как катушка не потребляет электрической энергии. Значит, здесь происходит перекачивание энергии от источника к катушке и обратно, т. е. источник работает беспо- лезно в смысле отдачи энергии. В отличие от активного сопротивления индуктивное сопротив- ление называется реактивным, т. е. таким, в котором происходит обратимый процесс — колебание энергии от источника электри- ческой энергии к катушке и обратно. Мощность, развиваемая током в реактивном сопротивлении катушки, является реактивной мощностью. Ее среднее значение за период равно нулю. О величине реактивной мощности в цепи чаще всего судят по ее амплитуде, которую обычно и называют просто реактивной мощностью. Следовательно, если мгновенная мощность в цепи с катуш- кой индуктивности, активным сопротивлением которой можно пренебречь, равна р = UI-sin 2ш/, (214) то реактивная мощность в этой цепи равна Pr=UI, (215) где U—напряжение в вольтах; I—ток в амперах; Рг—реактивная мощность (амплитуда реактивной мощно- сти) в вольт-амперах. Реактивная мощность в отличие от активной измеряется в вольт-амперах, называемых реактивными вольт-амперами. 1 вольт-ампер реактивный (1 вар) = 1 вольт-1 ампер. 1 киловольт-ампер реактивный (квар) = 1000 вар. Пример 112. К катушке индуктивности, обладающей индуктивностью L = 0,05 гн и активным сопротивлением г 0, приложено напряжение U = ЮО в. Определить мгновенное значение мощности в цепи для момента 329
(Времени / = 78,5*10“б сек., реактивную мощность и максимальную энергию, запасаемую магнитным полем катушки, если угловая частота напряжения 0 = 1000—. сек Решение. Индуктивное сопротивление цепи равно хь = = 1000 • 0,05 = 50 ом. Действующее значение тока равно /=-L_ioo = 2 а 1 х 50 2 а‘ и Мгновенная мощность для момента времени t = 78,5 • 10~6 сек. равна p = UI-sin ф/ = 100 • 2 • sin 1000 • 78,5 • 10“5 = 200 • sin 0,785 == = 200-sin 45°= 200* КД = 141 вар. Реактивная мощность равна Pr = UI= 100-2 = 200 вар. § 104. ЦЕПЬ ПЕРЕМЕННОГО ТОКА С ИНДУКТИВНОСТЬЮ И АКТИВНЫМ СОПРОТИВЛЕНИЕМ Теперь допустим, что катушка индуктивности, обладающая индуктивностью L, имеет такое активное сопротивление г, кото- рым практически пренебречь нельзя. В схеме, изображенной на рис. 200, активное сопротивление катушки показано отдельно от нее, т. е. мы рассматриваем катушку с г и L как цепь с после- довательно соедийенными активным сопротивлением г и индук- тивным сопротивлением xL = coL. Пусть в цепи катушки существует переменный синусоидаль- ный ток i = /m-sin Напряжение V, приложенное к рассматриваемой цепи, рас- ходуется в активном г и индуктивном wL сопротивлениях. Мгновенное значение напряжения, расходуемого в активном сопротивлении, равно иа = /r-sin<o/. Мгновенное значение напряжения, расходуемого в индуктив- ном сопротивлении, равно uL = • sin (+ -у- 330
Мгновенное значение напряжения, приложенного Кб всей цепи, равно алгебраической сумме мгновенных значений напря- жений на активном и индуктивном сопротивлениях: « = иа + uL — Imr-sin и/ + 7т<в£ • sin (wt + -у-). (216) Так как слагаемые этой суммы — синусоидальные величины одинаковой частоты, то результирующее напряжение также дол- жно быть синусоидальным и иметь ту же частоту, что и слагае- мые напряжения. А это значит, что для определения действую- щего значения напряжения, приложенного к данной цепи, можно на векторной диаграмме геометрически сложить действующие значения напряжений: 1) Uа =1г — действующее значение падения напряжения в активном сопротивлении; 2) Ul=IwL — действующее значение падения напряжения в индуктивном сопротивлении. Рис. 200. Цепь переменного тока с индуктивностью и активным сопротивлением Рис. 201. Векторная диаграмма тока и напряжений для цепи с г и L, соединенными последова- тельно диаграммы строили, применяя До сих пор мы векторные амплитуды синусоидальных величин. В дальнейшем же для удобства будем строить векторные диаграммы, применяя дей- ствующие значения величин. Для этого в обычной векторной диаграмме все векторы надо уменьшить в /2 , так как U = = 7^;/ = #ит'д' Действующее значение тока I одинаково в активном сопро- тивлении г и в катушке с индуктивным сопротивлением xL, так как они соединены последовательно. Отложим вектор тока I на векторной диаграмме по горизон- тали, так как начальная фаза тока I — 1т • sin wf равна нулю (рис. 201). Напряжение Ua совпадает по фазе с током, поскольку оно приложено к участку цепи, обладающему только активным со- 331
противлением, а поэтому вектор Ua отложим на векторной диа- грамме в направлении вектора тока I. Напряжение UL опережает ток I по фазе на угол по- скольку оно приложено к участку цепи, обладающему только индуктивным сопротивлением, а поэтому вектор UL отложим на векторной диаграмме в направлении, перпендикулярном век- тору I, и вверх, т. е. на угол у против часовой стрелки. Сложив геометрически Ua и UL, получим действующее зна- чение результирующего напряжения U, приложенного ко всей цепи. Рис. 202. Тре- угольник на- . пряжений Рис. 203. Треугольник сопротивлений Выделим из векторной диаграммы прямоугольный треуголь- ник, составленный из векторов Ua, UL и U, который назы- вается треугольником напряжений (рис. 202). Из треугольника напряжений на основании теоремы Пифагора находим соотно- шение между напряжениями: (217) Пример 113. Вольтметры, подключенные к активному сопротивлению г и катушке L, активное сопротивление которой можно принять равным нулю, дают показания £7* == 30 в и UL = 40 в. Определить результирующее на- пряжение, приложенное ко всей цепи. Решение. По формуле (217) находим U = Vu^ + Ul = V 302 + 402 = 50 в. Разделив все стороны треугольника напряжений на величину тока I, получим так называемый треугольник сопротивлений (рис. 203). Из треугольника сопротивлений имеем где» Z — полное сопротивление всей цепи в омах. 332
Пример 114. Катушка индуктивности имеет активное сопротивление г =12 ом и индуктивное сопротивление = 9 ом. Определить полное со- противление катушки. Решение. По формуле (218) находим Z = Уг2 + (о)£)2 = lz 122 + 92 = 15 ом. Зная напряжение U, приложенное к цепи, и полное сопро- тивление Z цепи, можно найти величину тока по формуле I= U _ U Z 1/>2 + (О)£)2 ’ Формула (219) является математическим выражением закона Ома для цепи с индуктивным и активным сопротивлениями, соединенными последовательно. Пример 115. Определить ток в цепи с активным сопротивлением г = 30 ом, индуктивностью L = 0,04 гн, если угловая частота <о = 1000 и напряжение U, приложенное к цепи, равно 120 в. Решение. По формуле (219) находим V r* + (а)!)2 ]/302 + (1000 • 0,04)2 Из треугольника сопротивлений находим угол сдвига фаз ф между напряжением U и током I по формуле cos <р = -4- = Z .. - (220) ' Z /г2+(о)£)2 4 или по формуле tg?=v- <221> Определив cos <р или tg по тригонометрическим таблицам можно найти и угол <р. Так как напряжение U опережает по фазе ток / на угол <р, то последний положителен. Следовательно, если / = /m-sin at, то u = Um-sin + Мгновенная мощность, развиваемая током в цепи, равна p = ui=Um‘sin (pt + ср) 1т-sin atf и уже не является синусоидальной величиной (рис. 204). 333
Рис. 204. График изменения мгновенной мощ- ности в цепи с последовательно соединенными г и L На рис. 204 приведены развернутые диаграммы тока и на- пряжения, а также график мгновенной мощности для рассматри- ваемой цепи. Если все стороны треугольника напряжений (рис. 202) по- множить на действующее значение тока /, то получим так на- зываемый треугольник мощностей (рис. 205). Треугольник мощностей геометрически связывает между со- бой три мощности: активную Ра, реактивную Рг и полную Активная мощность Ра равна произведению активной состав- ляющей напряжения Ua на величину тока 1\ Pa=UaL Из треугольника напряжений имеем: Ua — U • cos ср, а по- этому активную Рис. 205. Треуголь- ник мощностей мощность можно выразить следующей фор- мулой: Ра = UP cos ср, (222) где Ра—активная мощность в ваттах; U — напряжение в вольтах; I—ток в амперах; cos ср — косинус угла сдвига фаз между током и напряжением, который в данном случае носит название коэффициента мощности. cos ср = -5—7zx. (223) т Уг2+(О)£)2 v J 334
Так как U = IZ и cos ср = -^-, то формуле (222) можно при- дать такой вид: Pa = IZI±-> или, иначе, Ра = Рг. (224) т. е. активная мощность в цепи с индуктивно- стью и активным сопротивлением расходуется в активном сопротивлении ее. Реактивная мощность Рг равна произведению реактивной составляющей напряжения UL на величину тока Z: Pr = ULI. Из треугольника напряжений имеем: UL = U> sin <р, а по- этому реактивную мощность можно выразить следующей фор- мулой: = L7-sin <р, (225) где U — напряжение в вольтах; I — ток в амперах; Рт — реактивная мощность в вольт-амперах реактивных (вар). Полная мощность Pt равна произведению напряжения U на величину тока I: Pi = UI. (226) Полная мощность в отличие от реактивной, измеряемой вольт-амперами реактивными (вар), измеряется просто вольт- амперами (ва). Из треугольника мощностей находим соотношение между полной мощностью Рь активной мощностью Ра и реактивной мощностью Рг: = (227) т. е. полная мощность, развиваемая током в цепи с активным сопротивлением и индуктивностью, равняется корню квадрат- ному из суммы квадратов активной и реактивной мощностей. На рис. 204 показаны заштрихованные площадки, ограничен- ные осью времени и графиком мгновенной мощности р. Пло- щадки, расположенные выше оси времени, пропорциональны по- ложительной энергии, т. е. той энергии, которая за соответствую- щий отрезок времени поступает от генератора электрической энергии в катушку (в цепь), а площадки, расположенные ниже 335
оси времени, пропорциональны отрицательной энергии, т. е. той энергии, которая за определенные отрезки времени возвращается из магнитного поля катушки к генератору. Разность площадок представляет собой площадку, пропорциональную той энергии, которая за один период расходуется в активном сопротивлении цепи на необратимые процессы*. Пример 116. Катушка с активным сопротивлением г = 80 ом и индук- тивным сопротивлением wL = 60 ом подключена к зажимам с напряжением U = 120 в. Определить ток в цепи, угол сдвига фаз тока и напряжения, коэффициент мощности, полную, активную и реактивную мощности и энер- гию, затрачиваемую в цепи за один период, если угловая частота тока ш = юоо—. сек Решение. Полное сопротивление катушки Z = Vr2+&L)z = /802 4- 602 = 100 ом. Ток в цепи Коэффициент мощности г 80 n о cosep^-j- = !об = о,8. Угол сдвига фаз находим по таблицам косинусов. Для cos ср = 0,8 ср = 36°50'. Полная мощность pi = UI =120-1,2 — 144 ва. Активная мощность Ра = UI• cos <? = 120-1,2-0,8 = 115,2 вт. Реактивная мощность Pt = UI- sin<р = 120-1,2-0,6 = 86,4 ва. Период переменного тока Т = — = = 0,00628 сек. со 1000 ’ Энергия, расходуемая в цепи за время одного периода Т, равна W = РаТ= 115,2 -0,00628 = 0,725 дж. Напряжение, теряемое в активном сопротивлении, Ua = Ir= 1,2-80 = 96 в. Напряжение, теряемое в индуктивном сопротивлении, £7Д = /ш£ = 1,2-60 = 72 в. 336
§ 105. ЦЕПЬ С НЕСКОЛЬКИМИ ПОСЛЕДОВАТЕЛЬНО СОЕДИНЕННЫМИ АКТИВНЫМИ СОПРОТИВЛЕНИЯМИ И ИНДУКТИВНОСТЯМИ Две катушки индуктивности, обладающие активными сопро- тивлениями Г1 и Г2 и индуктивностями Li и L2, соединены после- довательно (рис. 206). В этой цепи проходит переменный сину- соидальный ток i = Im* sin <о/. Напряжение U, приложенное к цепи, расходуется в двух активных сопротивлениях (77^ = 7^ и Ua2 = Ir2} и двух реак- тивных сопротивлениях (UL1 = I^LX и UL2 = I (о£2). Для того чтобы по этим четырем составляющим напряжения определить результирующее напряжение <7, надо Uab Ua2) UL1 и сложить геометри- чески на векторной диаграм- ме. ri % L2 н—Jt'TTV—Н— ^аг ---------0 U $ . Рис. 206. Цепь переменного тока с последовательно соеди- ненными активными сопроти- влениями и индуктивностями Рис. 207. Векторная диаграмма для цепи с последовательно соединен- ными активными сопротивлениями и индуктивностями Отложим вектор тока I по горизонтали, так как начальная фаза тока по условию равна нулю (рис. 207). Напряжение UaX = Ii\ совпадает по фазе с током 7, так как оно приложено к активному сопротивлению /ч, а поэтому вектор UaX отложим в направлении вектора тока 7. Напряжение ULX = опере- жает ток I по фазе на угол у, так как оно приложено к уча- стку цепи, обладающему только индуктивным сопротивлением (oLi, а поэтому вектор ULX отложим в направлении, перпендику- лярном вектору I, вверх из конца вектора Uai. Сложив геометри- чески векторы UaX и ULb получим действующее значение напря- жения 171, приложенного к первой катушке. Напряжение Ua2 совпадает по фазе с током I, так как оно приложено к активному сопротивлению г2, а поэтому вектор Ua2 отложим параллельно вектору тока I, приложив его к вектору <71. Напряжение UL2 опережает ток 7 по фазе на угол £, 22-1377 337
так как оно приложено к участку цепи, где имеется только ин- дуктивное сопротивление (dL2, а поэтому вектор UL2 отложим на векторной диаграмме в направлении, перпендикулярном век- тору I, вверх, приложив его к концу вектора Ua2. Сложив гео- метрически векторы Ua2 и UL2, получим действующее значение напряжения t/2, приложенного ко второй катушке. Сложив геометрически напряжения lh и С72, получим резуль- тирующее напряжение U, приложенное ко всей цепи. Выделим из векторной диаграммы треугольник напряжений (рис. 208), Рис. 208. Треугольник напряжений Рис. 209. Треугольник сопротивлений у которого один катет равен сумме напряжений Ual и Ua2, а второй — сумме напряжений UL1 и UL2. Согласно теореме Пифагора U=V(Ua + U02y + (ULi + (228) Разделив все стороны треугольника напряжений на величину тока I, получим треугольник сопротивлений (рис. 209), из кото- рого найдем Z = И(Г! + Г2)2 + (<»Л + <^2)2, (229) где Z — полное сопротивление цепи в омах. Из треугольника сопротивлений определяем tg <р: (230) и отсюда находим угол ср по таблицам тангенсов. Этот угол будет положительным, так как напряжение, приложенное к цепи, опережает по фазе ток. Его можно определить также и через cos <р из треугольника сопротивлений: Г1 + Лл COS <Р = 7 . 338
Зная напряжение U и полное сопротивление цепи Z, можно определить величину тока / по формуле z U (231) ’ ]/(Л + + (<°^1 + ^з)2 Формула (231) является математическим выражением закона Ома для цепи с последовательно соединенными катушками, об- ладающими индуктивностями и активными сопротивлениями. Полная мощность, развиваемая током в цепи, Pt = UI. Активная мощность Pa = f7/-cos<p. Реактивная мощность Pr = UI- sin ср. В общем случае, когда в цепи имеется несколько активных сопротивлений и индуктивностей, соединенных последовательно, величина тока в' цепи определяется формулой Т U V (2 r)2 + (2 o>L)2 ’ где Sr—сумма всех активных сопротивлений цепи; So)Z — сумма всех индуктивных сопротивлений цепи. Пример 117. Последовательно соединены две катушки индуктивности, у которых активные сопротивления равны и = 4 ом и г2 = 6 ом, а индук- тивные сопротивления соответственно равны wLi = 2 ом и <о£2 = 6 ом. Определить полное сопротивление цепи, величину тока, сдвиг фаз тока и на- пряжения и мощность, расходуемую в цепи, если напряжение U, приложен- ное к ней, равно 120 в. Решение. Полное сопротивление в цепи Z = /(г, + г2)2 + («)Л + ш£2)2 = |/(4+6)2 + (2+6)2 = 12,8 ом. Величина тока в цепи /_Е=_120_ 1 ~ z 12,8 У’4 а' Коэффициент мощности cos ? = -£-=^| = 0,781. Угол сдвига фаз определяем по таблице косинусов; для cos у = 0,781 у = 38°40'. Активная мощность Ра = UI-cos ? = 120 • 9,4 • 0,781 = 880 вт. 22*
ГЛАВА XXI ПРОСТАЯ ЦЕПЬ ПЕРЕМЕННОГО ТОКА С ЕМКОСТЬЮ И АКТИВНЫМ СОПРОТИВЛЕНИЕМ § 106. ПОДКЛЮЧЕНИЕ КОНДЕНСАТОРА К ИСТОЧНИКУ ЭЛЕКТРИЧЕСКОЙ ЭНЕРГИИ С ПОСТОЯННЫМ НАПРЯЖЕНИЕМ И РАЗРЯД КОНДЕНСАТОРА ЧЕРЕЗ АКТИВНОЕ СОПРОТИВЛЕНИЕ 1. Процесс заряда конденсатора Допустим, что задана электрическая цепь, состоящая из последовательно соединенных. активного сопротивления г и кон- денсатора, обладающего емкостью С. Индуктивность цепи будем считать настолько малой, что ею практически можно пренебречь (L = 0). Подключим заданную цепь к источнику электрической энер- гии с постоянным напряжением U (рис. 210). Как только цепь замкнется, тотчас же в ней появится электрический ток i кон- денсатор начнет заряжаться. По мере того как будет протекать ток в цепи, электрические заряды на обкладках конденсатора начнут расти. На обкладке конденсатора, соединенной с положи- тельным полюсом источника, будут накапливаться положитель- ные заряды (недостаток электронов), а на обкладке, соединенной с отрицательным полюсом, — отрицательные заряды (избыток электронов). Эти заряды конденсатора создадут свое собственное электростатическое поле, на- правленное навстречу электрическому полю, заряжающему конденсатор. Чем больше возрастают, заряды конденсатора, тем больше их электростатическое поле, тем с большей силой оно противодей- ствует полю источника электрической энергии и тем меньше становится зарядный ток в цепи. В момент, когда силы электро- статического поля 2?ст сравняются с силами электрического поля Ед источника тока: 340
электрический ток в цепи станет равным нулю {I — 0). Это зна- чит, что конденсатор зарядился полностью, т. е. напряжение на его обкладках Uc стало равным напряжению U на зажимах источника, зарядившего его, т. е. ис = и. Электрический заряд конденсатора в этом случае достиг своей максимальной величины при заданных выше условиях: Q = O/, где Q — электрический заряд конденсатора в кулонах; С — емкость конденсатора в фарадах; 9 U — напряжение на Обкладках конденсатора в вольтах. В начальный момент времени заряда конденсатора (при £ = 0), когда поля, противодействующего заряду конденсатора, еще нет из-за отсутствия зарядов на обкладках конденсатора, электрический зарядный ток в цепи максимальный и равен ^=/0=4> где —ток при t = 0 в амперах; U— напряжение источника электрической энергии в вольтах; г—активное сопротивление цепи в омах. Затем ток I в цепи начинает убывать, стремясь в конечном итоге к нулю. Напряжение на обкладках конденсатора Uc в начальный мо- мент времени (при t = 0) заряда конденсатора равно нулю (£7С = 0) из-за отсутствия зарядов на его обкладках. Затем оно по мере поступления зарядов на обкладки конденсатора растет, стремясь в конечном итоге к величине напряжения на зажимах источника электрической энергии (t/c=t7). заряжается от источника электрической энергии с постоянным’ напряже- нием Рис. 211. Графики зависимости напряжения ис и тока i от вре- мени при заряде конденсатора от источника электрической энер- гии с постоянным напряжением 341
На рис. 211 приведены графики зависимости тока i и напря- жения ис от времени при заряде конденсатора от источника электрической энергии постоянным напряжением для случая, когда индуктивность цепи L = 0. Чем больше емкость С и активное сопротивление г конден- сатора, тем больше необходимо времени на заряд конденсатора. Чтобы судить о том, насколько быстро заряжается конденсатор, применяют величину, называемую постоянной времени (т). Постоянная времени т — это отрезок времени, в течение кото- рого, начиная от / = 0, зарядный ток уменьшается в е = 2,718 раз. Она определяется по формуле т = гС, (232) где т — постоянная времени цепи в секундах; С — емкость конденсатора в фарадах; г—активное сопротивление цепи в омах. Пример 118. -Определить постоянную времени цепи, имеющей емкость С = 2 мкф и активное сопротивление г = 100 ом. Решение: Применяя формулу (232), находим т = гС = 100 -2 -10“6 = 0,0002 сек., т. е. через две десятитысячные доли секунды зарядный ток уменьшится по сравнению с его начальным значением (при / = 0) в 2,718 раз. Электрические заряды конденсатора создают в диэлектрике заряженного конденсатора электростатическое поле, которое об- ладает энергией, равной W.= сис 2 ’ где Wd — энергия электростатического поля конденсатора в джоулях; С — емкость конденсатора в фарадах; Uc — напряжение на зажимах (обкладках) конденсатора в вольтах. Эта энергия — потенциальная; она преобразуется в другие виды энергии при разряде конденсатора. При заряде конденсатора источник электрической энергии отдает свою энергию не только непосредственно конденсатору (тот накапливает ее в своем электрическом поле), но и актив- ному сопротивлению цепи, нагревая его. При этом, как показали расчеты, энергия источника всякий раз делится между ними по- ровну независимо от параметров г и С, т. е. коэффициент полез- ного действия при заряде конденсатора всегда равен 50%!. 342
2. Разряд конденсатора Если заряженный конденсатор отключить от источника элек- трической энергии и замкнуть на активное сопротивление г, то он может разряжаться (рис. 212). В момент времени t = 0 (начальный момент разряда конден- сатора) напряжение на его обкладках равно некоторому началь- ному напряжению 670, которое создает начальный разрядный ток, практически равный Затем по мере разряда конденсатора начинает убывать его заряд, а вместе с ним и напряжение Uc на обкладках конденса- тора; одновременно с этим уменьшается и величина.ч.разрядного тока i в цепи. На рис. 213 показан график изменения напряже- Рис. 212. Конденсатор раз- ряжается через активное сопротивление Рис. 213. График изменения напря- ' жения йс при разряде конденсатора через активное сопротивление ния на зажимах конденсатора при его разряде через активное сопротивление. Графики изменения разрядного тока I и заряда конденсатора q аналогичны графику напряжения. По мере разряда конденсатора постепенно исчезает его электрическое поле, отдавая свою энергию активному сопротивлению, которое преобразует ее в тепловую энергию. Амперметр, включенный в цепь разряжающегося конденса- тора, покажет наличие мгновенного разрядного тока. Стрелка прибора отклонится в сторону, противоположную той, в которую она отклонялась при заряде конденсатора, и затем вернется в нулевое положение. Разряд конденсатора продолжается малые доли секунды. О быстроте разряда можно судить по постоянной времени, опре- деляемой по формуле (232). В заключение следует подчеркнуть, что постоянный ток не может проходить через конденсатор по той простой причине, что 343
диэлектрик конденсатора имеет сопротивление, измеряемое обыч- но миллионами, или миллиардами омов. Например, пластинка слюды с площадью один квадратный сантиметр и толщиной один миллиметр имеет сопротивление для постоянного тока порядка десяти тысяч миллиардов омов. § 107. ЦЕПЬ ПЕРЕМЕННОГО ТОКА С ЕМКОСТЬЮ Рис. 214. Конденсатор в цепи пере- менного тока Если конденсатор подключить к источнику электрической энергии, обладающему синусоидальным напряжением (рис. 214), то в цепи пойдет переменный синусоидальный ток, который будет длиться до тех пор, пока в цепи действует переменное си- нусоидальное напряжение. При этом вне обкладок конденса- тора, т. е. в проводниках цепи, соединяющих конденсатор с источником электрической энер- гии, будет существовать . п е- ременный электриче- ский ток проводимо* с т и, создаваемый движением свободных электронов этого участка цепи. Свободные элек- троны под влиянием сил пе- ременного электрического поля совершают колебания в цепи, перезаряжая обкладки конденса- тора. В соответствии с этим заряд конденсатора и электрическое поле, создаваемое им в диэлектрике конденсатора, будут изме- няться по синусоидальному закону. Под влиянием сил этого электрического поля в атомах диэлектрика происходит смещение электронов то в одну, то в другую, противоположную, сторону в зависимости от направления действия переменных сил поля. Это направленное колебательное движение электронов в атомах диэлектрика под влиянием сил переменного электрического поля является электрическим током смещения в диэлектрике конден- сатора. Ток смещения- в диэлектрике обладает рядом свойств, прису- щих току проводимости, например магнитными, тепловыми и т. д. В рассматриваемом нами случае мы имеем в одном участке цепи (в металлических проводах) ток проводимости, а в другом (в диэлектрике конденсатора) :—ток смещения. Однако и в этом случае ток в цепи замкнут на себя, так как ток смещения допол- няет ток проводимости и совместно с ним образует замкнутые на себя линии тока. Величина тока в диэлектрике в точности равна величине тока проводимости в подводящих проводах, соединяющих ксн- 344
денсатор с источником электрической энергии, поскольку нити тока неразрывны. Необходимо помнить, что природа переменного тОка в диэ- лектрике конденсатора (ток смещения) отличается от природы тока в подводящих проводах (ток проводимости). Если бы через диэлектрик конденсатора прошел ток проводимости, то это за- вершилось бы пробоем диэлектрика, в результате чего конденса- тор пришел бы в негодность. Незначительные же (в доли микро- ампера) токи проводимости, проходящие через диэлектрик (ток утечки конденсатора), обычно в расчет не принимаются. Но так как в диэлектрике конденсатора есть ток смещения, то можно сказать, что переменный ток проходит через конденса- тор. Если бы в нем почему-нибудь возник ток проводимости, то это значило бы, что в диэлектрике конденсатора возникло явле- ние электрического пробоя. Амперметр, включенный в цепь переменного тока с конденса- тором, укажет на постоянное присутствие тока в цепи, величина которого зависит от приложенного напряжения, емкости конден- сатора и частоты колебаний приложенного напряжения. При наличии на зажимах конденсатора переменного напря- жения величина заряда q конденсатора также переменная. Если за очень малый промежуток времени Д/ заряд на обкладках конденсатора изменился на величину Д</, то мгновенное значение тока в цепи равно * = #• (233) Но, как известно, q = CU, где q—электрический заряд конденсатора в кулонах; U— напряжение на обкладках конденсатора в вольтах; С— емкость конденсатора в фарадах. С ростом заряда конденсатора растет и напряжение на его обкладках, поэтому Д^ = СДв, где Д^—приращение заряда в кулонах; Да—приращение напряжения в вольтах. Подставив выражение для Д<? в формулу (233), получим / = (234) Из формулы (234) следует, что переменный ток в цепи с емкостью пропорционален скорости изменения приложенного к конденсатору напряжения. 345
Пусть к конденсатору приложено синусоидальное напряже- ние (рис. 215) и = t/^-sin со/. В первую четверть периода напряжение и нарастает в поло- жительном направлении, и в соответствии с формулой (234) ток в цепи также должен быть положительным. Скорость изменения Рис. 215. Векторная и развернутая диа- граммы тока и напряжения для цепи с ем- костью напряжения и наиболь- шая при переходе че- рез нуль и равна нулю, когда напряжение ра£- но максимуму. Следо- звательно, величина то- ка Z, пропорциональ- ная скорости измене- ния напряжения, дол- жна иметь максимум при переходе напряже- ния через нулевое зна- чение и равна нулю при переходе напряже- ния череа максимум (см. рис. 215). Во вторую четверть периода напряжение на зажимах источника электрической энергии уменьшается от макси- мального значения до нуля; конденсатор разряжается, посылая ток навстречу при- ложенному к нему напряжению. Следовательно, убывающее напряжение источника продолжает оставаться положительным, а ток становится отрицательным и нарастает по мере увеличе- ния скорости изменения напряжения. Когда напряжение будет проходить через нуль, Ьи будет максимальным и ток в цепи до- стигнет максимального значения. В третью четверть периода напряжение меняет свой знак, становясь отрицательным, и в то же время растет по абсолютной величине, достигая максимума. С момента перехода напряжения через нуль начинается перезаряд конденсатора и ток имеет оди- наковый знак с напряжением, т. е. он отрицателен, сохраняя свой знак, который имел во вторую четверть периода. По мере уве- личения напряжения и скорость его изменения падает и стано- вится равной нулю, когда напряжение переходит свой максимум. А это значит, что в третьей четверти периода ток изменяется от максимума до нуля. В четвертую четверть периода напряжение уменьшается по 346
абсолютной величине от максимума до нуля, конденсатор разря- жается навстречу приложенному напряжению, т. е. при отрица- тельном напряжении ток становится положительным. По мере уменьшения напряжения скорость изменения его растет и в соот- ветствии с этим растет величина разрядного тока, изменяясь от нуля до максимума. Сопоставив развернутые диаграммы тока I в цепи с конденса- тором и напряжения w, приложенного*к конденсатору (рис. 215), нетрудно видеть, что ток / опережает по фазе напря- жение и на угол , т. е. на четверть периода. Следовательно, если мгновенное значение напряжения, при- ложенного к конденсатору, определяется по формуле u = Um-sin со/, то мгновенное значение тока будет определяться формулой Z = /,n-sin^+ -уУ (235) Как известно, величина тока в цепи с конденсатором пропор- циональна скорости изменения напряжения и на зажимах кон- денсатора, т. е. ки д/ i = C (236) а это значит, что величина тока в цепи с конденсатором при не- изменной емкости последнего пропорциональна частоте прило- женного к конденсатору напряжения, так как чем больше эта частота, тем больше скорость изменения напряжения во времени. Величина тока в цепи с конденсатором зависит и от емкости конденсатора: чем больше емкость, тем больше при прочих рав- ных условиях величина тока в цепи с конденсатором. Следовательно, величина тока в цепи с конденсатором про- порциональна частоте напряжения и емкости конденсатора. В со- ответствии с этим действующее значение тока I = ®CU, (237) где I— действующее значение тока в амперах; U—действующее значение напряжения в вольтах; С— емкость в фарадах; о»— угловая частота. Помножив левую и правую части равенства на И 2, полу- чим выражение для амплитуды тока: К27=/2шС67, 347
или, иначе, im = <x>cum, (238) где 1т— амплитуда тока; Um—амплитуда напряжения. В соответствии с этим формуле (235) можно придать такой вид: i = №>CUm • sin (»>t + ч где I — мгновенное значение тока. Преобразовав выражение (237), получим формулу закона Ома для чисто емкостной цепи: U 1 ’ (239) (оС где / — действующее значение тока; U — действующее значение напряжения. Величина представляет собой емкостное сопротивление конденсатора и обычно обозначается символом хс: = <24°) где хс — емкостное сопротивление в омах; С — емкость конденсатора в фарадах; со — угловая частота в ——. Емкостное сопротивление конденсатора — это не сопротивле- ние диэлектрика конденсатора прохождению через него тока проводимости, которое обычно измеряется многими миллионами или миллиардами ом. Емкостное сопротивление конденсатора обусловлено противодействием электрического поля диэлектрика конденсатора, которое направлено навстречу электрическому полю источника электрической энергии, осуществляющему пере- заряд конденсатора. В результате этого все напряжение U, при- ложенное к конденсатору, целиком компенсируется электродви- жущей силой Ес конденсатора, т. е. | U\ = । Ес|. Но, с другой стороны, согласно формуле (239) т. в. напряжение, приложенное к конденсатору, расходуется на преодоление емко- стного сопротивления конденсатора. Следовательно, 348
Рис. 216. Графики зависимости ем- костного сопротивления конденса- тора от угловой частоты т. е. напряжение, приложенное Отсюда мы можем сделать вывод, что емкостное сопротивле- ние конденсатора обусловлено противодействием электродвижу- щей силы конденсатора. Из двух конденсаторов, внутри которых протекает одинако- вый синусоидальный ток, большим емкостным сопротивлением обладает тот, в котором возни- кает большая электродвижу- щая сила. Из формулы (240) следует, что емкостное сопротивление конденсатора обратно пропор- ционально угловой частоте <о напряжения, приложенного к конденсатору, и емкости С конденсатора. На рис. 216 приведен гра- фик зависимости емкостного со- противления конденсатора от угловой частоты. Если угловая частота w = 0, к конденсатору, постоянно, то емкостное сопротивление конден- сатора равно бесконечности (хс = = -уу = -у = оо). Это понятно, так как при постоянном напряжении, приложенном к конденсатору, ток в конденсаторе равен нулю. Наоборот, если угловая частота стремится к бесконечности (о)->оо), то емкост- ное сопротивление конденсатора стремится к нулю (хс = ~^- = = ^с = °У Пример 119. К цепи, состоящей из конденсатора емкостью 6 = 0,1 мкф, приложено синусоидальное напряжение U = 42,3 • sin 105 t. Определить вели- чину мгновенного и действующего тока в цепи, если активным сопротивле- нием цепи можно пренебречь. Решение. Емкостное сопротивление Хс^~С = 106-0,1-10-в = 100 ом- Мгновенное значение тока z=^-sin flO51 + 4-) = 0,423-sin (1057 4- -£-) . 1vu \ A ] \ л 1 Действующее значение тока / = ^ = ^|3=0,3 а. |/2 1.41 ’ Мгновенная мощность р, развиваемая током в конденсаторе, равна произведению мгновенного значения напряжения (и) и мгновенного значения тока (/), т. е. р = ui = Um-sin wtlm-sin((241) 349
Рис. 217. График изменения мгновен- ной мощности для цепи с емкостью После некоторых преоб- разований, аналогичных при- веденным § 103, получим: /7 = 677-sin 24 (242) т. е. мгновенная мощ- ность — величина си- нусоидальная, изме- няющаяся с двойной частотой по сравне- нию с частотой тока (напряжения). На рис. 217 приведен график мгновенной мощно- сти вместе с развернутыми диаграммами тока и напряжения для рассматриваемого нами случая. Среднее значение мощности за период равно нулю: ^ср = так как все положительные значения мгновенной мощности при суммировании со всеми отрицательными значениями ее дают нуль. Средняя мощность, потребляемая конденсатором за период, равна нулю. Это значит, что конденсатор при отсутствии актив- ного сопротивления энергию не потребляет. В то время, когда мощность положительна (первая и третья четверти периода), конденсатор берет энергию от источника, за- пасая ее в электрическом поле диэлектрика: W =_____— где W9—энергия электрического поля конденсатора в джоулях; С—емкость конденсатора в фарадах; Um— максимальное значение напряжения, приложенного к конденсатору, в вольтах. Когда же мгновенная мощность отрицательна (вторая и чет- вертая четверти периода), конденсатор сам как бы становится источником электрической энергии, возвращая ее из электриче- ского поля источнику и облегчая работу последнего. " Заштрихованные площадки, ограниченные осью времени и гра- фиком мгновенной мощности (см. рис. 217), дают представление о движении энергии в цепи с конденсатором, где активное сопро- тивление г равно нулю. Площадки, расположенные выше линии времени, определяют положительную энергию, поступающую из источника в конденсатор, а площадки, расположенные ниже этой линии,— отрицательную энергию, возвращаемую конденсатором 350
источнику. Следовательно, конденсатор не потребляет энергии источника. Емкостное сопротивление в отличие от активного является реактивным и называется реактивным емкостным сопротив- лением. Мощность, развиваемая током в цепи с конденсатором, актив- ным сопротивлением которого можно пренебречь, является реак- тивной мощностью. Величина ее равна амплитуде в выражении для мгновенного значения мощности, т. е. Pr = UI, (243) где Рг — реактивная мощность в вольт-амперах реактивных (вар); U — напряжение в вольтах; /— ток в амперах. Активная мощность в данном случае равна нулю, так как Ра = Рг = /2.0 = 0. § 108. ЦЕПЬ ПЕРЕМЕННОГО ТОКА С ЕМКОСТЬЮ И АКТИВНЫМ СОПРОТИВЛЕНИЕМ Допустим, что в цепи, состоящей из последовательно соеди- ненных емкости С и активного сопротивления г (рис. 218), про- ходит переменный синусоидальный ток i = /m-sin Напряжение V, приложенное к данной цепи, расходуется в двух сопротивлениях: активном г и емкостном . В активном сопротивлении мгновенное значение напряжения равно иа = Imh sin <»/. Рис. 218. Цепь перемен- ного тока с последовательно соединенными активным со- противлением и емкостью В емкостном сопротивлении мгновенное значение напряжения равно Мгновенное значение напряжения, приложенного ко всей цепи,, равно ал- гебраической сумме мгновенных значе- ний напряжений на активном и емко- стном сопротивлениях: и = иа = = Imr-sin <0/ + 1т • sin . 351
Так как слагаемые здесь— синусоидальные величины одина- ковой частоты, то результирующее их напряжение также должно быть синусоидальным и иметь ту же частоту со, что и слагаемые напряжения. А это значит, что для определения действующего значения напряжения, приложенного к данной цепи, можно на векторной диаграмме геометрически сложить действующие значе- ния напряжений: 1) Ua = Ir—падение напряжения в активном сопротив- лении; 2) UC = I ------падение напряжения в емкостном сопротив- лении. Отложим вектор тока / на векторной диаграмме по горизон- тальному направление, так как по условию начальная фаза тока равна нулю (рис. 219). Напряжение Ua совпадает по фазе с то- тока и напряжения для цепи с по- следовательно соединенными ак- тивным сопротивлением и ем- костью Рис. 220. Тре- угольник на- пряжений ком, а поэтому вектор Ua отложим на векторной диаграмме в направлении вектора I. Напряжение Uc отстает по фазе от тока / на угол -у, так как оно приложено к участку цепи, обла- дающему только емкостным сопротивлением, а пЬэтому век.- тор Uc отложим в направлении, перпендикулярном вектору /, и вниз, т. е. на угол -у- по часовой стрелке. Сложив геометрически Ua и t/c, получим действующее зна- чение результирующего напряжения (7, приложенного ко всей цепи. Выделим из векторной диаграммы прямоугольный тре- угольник напряжений, составленный из векторов Uai Uc и U (рис. 220). Применяя теорему Пифагора, из треугольника напряжений находим соотношение между Ua> Uc и U\ и = уи2а + ис- (244) 352
Пример 120. Напряжение на зажимах конденсатора Uc = 80 в, а на за- жимах активного сопротивления, соединенного последовательно с конденса- тором, Ua = 60 в. Определить напряжение, приложенное ко всей цепи. Решение. По формуле (244) находим и = у/= /бО2 + 802 = 100 в. Разделив все стороны треугольника напря- жений на величину тока /, получим тре- угольник сопротивлений (рис. 221). Из этого треугольника найдем (245) Рис. 221. Треуголь- ник сопротивлений где Z— полное сопротивление в омах; г— активное сопротивление в омах; —~— емкостное сопротивление в омах. Пример 121. Конденсатор при некоторой заданной частоте имеет емкост- ное сопротивление = 45 ом, а последовательно соединенное с ним ак- тивное сопротивление г = 25 ом. Определить полное сопротивление цепи. Решение. По формуле (245) находим 2 = 1/ + У ==4/252 +452 = 51,5 Г \ <оС / Зная полное напряжение, приложенное к цепи, и полное со- противление ее, можно определить ток в цепи по формуле (246) Формула (246) является математическим выражением з а- к о н а Ома для цепи с емкостью и активным сопротивлением, соединенными последовательно. Пример 122. Определить ток в цепи с активным сопротивлением г = 60 ом и емкостью С = 100 мкф, соединенными последовательно, если частота тока / = 50 гц и напряжение, приложенное во всей цепи, (У =120 в. Решение. Емкостное сопротивление цепи равно Хс~ шс ~2izfC~ 2-3,14-50-100-ю-e-31-8^ Ток в цепи 120 1^602 -1-31,82 120 _ 1 7Д „ 68 — 1 а‘ 23-1377 353
Из треугольника сопротивлений находим угол <? по тангенсу ср 1 tg?=“7“ (247) Так как напряжение отстает от тока по фазе на угол ср, то последний отрицателен, и его надо отложить на векторной диа- грамме в сторону вращения часовой стрелки от вектора тока /. Таким образом, если i = I- sin то и = f/^-sin (a)Z — <р). Мгновенная мощность, развиваемая током в цепи, равна р = ui = Um • sin (ф/ — ср) 1т • sin и не является сину- соидальной величиной. Рис. 222. График изменения мгновен- ной мощности в цепи с последова- тельно соединенными г и С Ра-иа1 Рис. 223. Треуголь- ник мощностей На рис. 222 показаны развернутые диаграммы тока I и на- пряжения и, а также график мгновенной мощности р для рас- сматриваемого случая последовательного соединения активного сопротивления г и емкости С. Умножив все стороны треугольника напряжений (рис. 220) на величину тока /, получим треугольник мощностей (рис. 223). Из треугольника мощностей находим: 1) активная мощность в цепи равна Ра = UaI = UI • cos ср = Рг> (248) 2) реактивная мощность в цепи равна Pr = UcI = UI- sin ср; (249) 354
3) полная мощность в цепи равна Pi = VPa + Pr=^ (250) где Ра~~ активная мощность в ваттах; Рг— реактивная мощность в вольт-амперах реактив- ных (вар)\ Pt—полная мощность в вольт-амперах (ва); cos ? = — коэффициент мощности; U—напряжение в вольтах; I— ток в амперах. Положительная энергия, т. е. та, которая поступает от источ- ника к конденсатору, соединенному последовательно с активным сопротивлением, больше, чем отрицательная энергия, которую конденсатор возвращает источнику. Разность этих количеств энергии есть как раз та энергия, ко- торая поглощается в активном сопротивлении цепи. На рис. 222 показаны площадки, ограниченные графиком мгновенной мощно- сти и осью времени. Площадки, расположенные выше оси вре- мени, определяют величину положительной энергии, а пло- щадки, лежащие ниже этой линии,— величину отрицательной энергии. Пример 123. Конденсатор, обладающий емкостью С = 0,2 мкф, соединен последовательно с активным сопротивлением г = 400 ом. Определить ток в цепи и мощность, расходуемую в цепи, если напряжение, приложенное к цепи, U = 125 в и угловая частота <о = 10 000 —. Решение. Емкостное сопротивление цепи равно . Хс = ~^С ~ 10 000-0,2-10-6 = ом- Полное сопротивление цепи z = j/r2 + (хс)2 '= И4002 + 5002 = 640,3 ом. Ток в цепи М’й = °’1951г- Коэффициент мощности Г ' 400 пглл COS ? = = W = 0,624. Угол сдвига фаз тока и напряжения определим по tg?: tg?=^- = ^ = l,25 и <р = 51°20'. Полная мощность = 67=125-0,195 = 24,4 ва. 23* 355
Активная мощность ра = t/z-cos ? = 125-0,195-0,624 = 15,4 вт. Если цепь составлена из нескольких последовательно соеди- ненных активных сопротивлений и емкостей (рис. 224), то расчет режима в этой цепи аналогичен расчету цепи с несколькими активными сопротивлениями и индуктивностями, приведенному в § 105. В этом случае эквивалентное активное сопротивление цепи равно R&= Л 4~ г2 + • • • + гп. (251) Рис. 224. Цепь переменного тока с последовательно соединенными активными сопротивлениями и емкостями Рис. 225. Векторная диа- грамма тока и напряжений для цепи с последовательно соединенными активными сопротивлениями и емко- стями Эквивалентное реактивное сопротивление цепи Хс э:=: —4—г—h • • • 4—• (252) с 9 юС1 соС2 ^Сп v 7 Полное сопротивление цепи = (253) Действующее значение тока /=4- <254) Угол сдвига фаз между током и напряжением можно опреде- лить по тангенсу этого угла: tg ? = -%=• (255) 356
На рис. 225 приведена векторная диаграмма тока и напря- жений для цепи с последовательно соединенными активными со- противлениями и емкостями. Пример 124. Цепь переменного тока состоит из последовательно соеди- ненных сопротивлений г\ = 35 ом, Г2 = 25 ом, х^ = 75 ом и хС2 = 45 ом. Определить ток в цепи, сдвиг фаз напряжения и тока и активную мощ- ность, если напряжение U, приложенное к цепи, равно 125 в. Решение. По формуле (253) находим полное сопротивление цепи Z = /(Л + r2)2 + (xCl + хсу = /(35 + 25)2 + (75 + 45)2 = —134,2 ом. Ток в цепи / = ^- = -^ = 0,935 а. Коэффициент мощности cos <f> = = j^2 = 0,446, угол <p = 63°31'. Активная мощность = /2 (Г1 + = 0)9352 (35 + 25) = 52,5 вт.
ГЛАВА XXII ПРОСТАЯ ЦЕПЬ ПЕРЕМЕННОГО ТОКА С ПОСЛЕДОВАТЕЛЬНО СОЕДИНЕННЫМИ ИНДУКТИВНОСТЬЮ, ЕМКОСТЬЮ И АКТИВНЫМ СОПРОТИВЛЕНИЕМ § 109. КОЛЕБАТЕЛЬНЫЙ И АПЕРИОДИЧЕСКИЙ РАЗРЯД КОНДЕНСАТОРА I. Собственные незатухающие колебания в контуре Допустим, что имеется электрическая цепь, в которой можно осуществить заряд конденсатора от источника электрической Рис. 226. Схема цепи для заряда конденсатора и последующего его раз* ряда через катушку индуктивности энергии с постоянным напряжением U, а затем и разряд его через катушку ин- дуктивности (рис. 226). Конденсатор об- ладает емкостью С, а катушка — индук- тивностью L. Активное сопротивление ка- тушки будем вначале считать практически равным нулю, т. е. возьмем идеальный случай разряда конденсатора. Если переключатель К поставить в по- ложение /, то конденсатор начнет заря- жаться. Заряд будет продолжаться до тех пор, пока напряжение на обкладках конденсатора не станет равным напряже- нию на зажимах источника электриче- ской энергии. В результате заряда конденсатора в его электрическом поле накопится потенциальная электрическая энергия, равная си2 W э т 2 ’ (256) где lF9/n— энергия электрического поля конденсатора в джоулях; Um—конечное (максимальное) напряжение на зажимах (обкладках) конденсатора в вольтах; С—емкость конденсатора в фарадах. 358
После того как конденсатор зарядится полностью до макси- мального напряжения Umy отключим его от источника электри- ческой энергии и в момент / = 0 присоединим к катушке индук- тивности L. Как только цепь, состоящая из последовательно соединенных конденсатора й катушки индуктивности, будет зам- кнута, конденсатор начнет разряжаться. Возникший в обмотке катушки разрядный ток конденсатора создаст в катушке магнитное поле, которое будет нарастать по мере увеличения тока. Изменяющееся во времени магнитное поле индуктирует в об- мотке катушки электродвижущую силу самоиндукции, которая согласно правилу Ленца противодействует нарастанию тока (рис. 227). Силы электрического поля конденсатора, перемещая электри- ческие заряды по цепи, преодолевают сопротивление сил индук- ционного электрического поля, так как напряжение на обклад- ках конденсатора и электродвижущая сила самоиндукции направ- лены навстречу друг другу. При этом силы электрического поля конденсатора совершают работу, в результате чего энергия электрического поля конденсатора переходит в энергию магнитного поля ка- тушки. Благодаря противодействию нарастанию тока со стороны электродвижущей силы самоиндукции разрядный ток конденсатора изменяется плавно, без скачков. По мере разряда конденсатора его раз- рядный ток все более возрастает и с ним вместе растет энергия магнитного поля ка- тушки за счет убыли энергии исчезающего электрического поля конденсатора. В момент времени, когда конденсатор разрядится полностью, т. е. когда его заряд и напряжение на его зажимах станут рав- ными нулю, электрическое поле конденсато- ра целиком преобразуется в магнитное поле катушки. Ток в цепи в этот момент до- стигнет максимального значения 1т, а магнитное поле тока за- пасет в себе максимальное количество энергии, равное lP W ** м т 2 Рис. 227. Разрядный ток i и э. д. с. eL на- правлены на- встречу друг другу (257) где W„m—энергия магнитного поля катушки в джоулях; 1т—максимальное значение тока в амперах; L— индуктивность катушки в генри. Так как в процессе разряда конденсатора энергия ёго элек- трического поля не расходовалась ни на какие необратимые про- 359
Цессы (например, на образование тепла в обмотке катушки), то вся она преобразовалась в энергию магнитного поля тока ка- тушки индуктивности, т. е. W = W ™ Э т ™ М или, иначе, Clfi ы2 2 “ 2 ’ Итак, в результате полного разряда конденсатора потенциаль- ная энергия его электрического поля (энергия поля относительно неподвижных зарядов) преобразовалась в кинетическую энергию магнитного поля тока (энергию поля, обусловленного движущи- мися электрическими зарядами — током). Но как только исчезло электрическое поле конденсатора, тот- час же начнут уменьшаться ток в цепи и его магнитное поле, так как исчезли силы, создающие и поддерживающие этот ток. Изменяющееся (исчезающее) магнитное поле опять начнет индуктировать электродвижущую силу самоиндукции. Однако на этот раз ее направление будет противоположным тому, которое она имела при разряде конденсатора, так как теперь она сог- ласно правилу Ленца должна быть направлена одинаково с исче- зающим током, противодействуя его убыванию. Теперь силы ин- дукционного электрического поля будут действовать уже в на- правлении движения электрических зарядов, способствуя их перемещению в цепи. В результате начнется заряд конденсатора (рис. 228). ^Электрические заряды, накапливающиеся на обкладках кон- денсатора, создают электрическое поле конденсатора, которое на- правлено навстречу индукционному электрическому полю, осу- ществляющему заряд конденсатора. Перемещая электрические заряды по цепи, силы индукционного электрического поля прео- долевают противодействие сил электрического поля конденсатора и при этом совершают некоторую работу за счет энергии исче- зающего магнитного поля. По мере заряда конденсатора зарядный ток становится все меньше, а с ним вместе убывает и его магнитное поле, преобра- зуя свою энергию в энергию электрического поля конденсатора. В момент времени, когда электрический ток в цепи станет равным нулю, магнитное поле его исчезнет, а вся энергия этого поля преобразуется в энергию электрического поля конденсатора. Конденсатор в этом случае полностью зарядится, и напряжение на его обкладках вновь станет равным Um. Однако теперь по- лярность конденсатора изменилась по сравнению с предыдущей, так как обкладка конденсатора, которая при разряде была заря- жена положительно, теперь заряжена отрицательно, а отрица- тельно заряженная обкладка — положительно, 360
' Итак, в момент времени, когда ток в цепи стал равным нулю, энергия магнитного поля целиком перешла в энергию электриче- ского поля конденсатора. После того как ток в цепи стал равным нулю и магнитное поле его целиком преобразовалось в электрическое поле конден- сатора, последний начинает вновь разряжаться. Как и при пре- дыдущем разряде конденсатора, разрядный ток нарастает в цепи плавно, с ним вместе непрерывно растет его магнитное поле. Рис. 229. Разряд- ный ток i и э. д. с. eL направлены на- встречу друг Другу Рис. 228. Зарядный ток / и э. д. с. ет L направлены по- путно друг с дру- гом Рис. 230. Заряд- ный ток i и э. д. с. eL направлены по- путно друг с дру- гом Электродвижущая сила самоиндукции направлена навстречу на- растающему току, противодействуя его росту (рис. 229). Силы электрического поля конденсатора преодолевают сопротивление сил индукционного электрического поля и совершают работу. В результате энергия электрического поля конденсатора перехо- дит в энергию магнитного поля тока. Когда конденсатор пол- ностью разрядится, напряжение на его обкладках станет равным нулю, ток в цепи достигнет максимального значения, а электри- ческое поле конденсатора преобразуется в магнитное поле ка- тушки, передав ему всю свою энергию. Затем вновь будет происходить заряд конденсатора (рис. 230*). Ток в цепи начнет убывать по мере заряда конденса- тора. Силы индукционного электрического поля, перемещая элек- трические заряды по цепи, будут преодолевать сопротивление электрического поля заряжающегося конденсатора. Следова- тельно, силы индукционного электрического поля будут совер- шать работу, и при этом энергия магнитного поля тока катушки будет преобразовываться в энергию электрического поля конден- сатора. Когда ток в цепи станет равным нулю, конденсатор пол- ностью зарядится, напряжение на его обкладках достигнет мак- симальной величины Um, а энергия магнитного поля тока ка- тушки целиком преобразуется в энергию электрического поля конденсатора. 361
Таким образом, электрическое состояние конденсатора верну- лось к исходному, соответствующему моменту времени t = О, когда начался разряд. Следовательно, полный цикл перезаряда конденсатора закончился. Однако, как показывает опыт, переза- ряд конденсатора будет повторяться периодически, т. е. через равные промежутки времени. Преобразование потенциальной энергии электрического поля конденсатора в кинетическую энер- гию магнитного поля тока катушки и обратно в данном идеаль- ном случае будет продолжаться сколь угодно долго, так как по условию активное сопротивление цепи равно нулю, а поэтому по- терь энергии на необратимые процессы здесь нет. Рис. 231. Векторная и развернутая диаграммы напряжения п, э. д. с. eL и тока i колебатель- ного разряда конденсатора Рассмотренный нами процесс разряда конденсатора назы- вается незатухающим колебательным разрядом, так как ампли- туда напряжения Um на зажимах конденсатора, амплитуда тока 1т в цепи, а также максимальное значение энергии IF972 электрического поля конденсатора и максимальное значение энергии WKtn магнитного поля катушки с течением времени остаются неизменными. Как показывает математический анализ, незатухающий колебательный разряд конденсатора является электромагнитным процессом, изменяющимся во времени по синусоидальному закону; значит, напряжение на зажимах конденсатора, ток в цепи, электродви- жущая сила — синусоидальные величины. На рис. 231 приведены векторная и развернутая диаграммы напряжения на зажимах конденсатора, тока и электродвижущей силы самоиндукции для случая незатухающего колебательного разряда конденсатора. Здесь мы видим, что ток в цепи изменяется по закону 362
i = Im - sin wt, напряжение на зажимах конденсатора опере- жает по фазе ток на угол 'у и изменяется по закону u = Um-sin (о>/ + , а электродвижущая сила самоиндукции отстает от тока по фазе на угол и изменяется по закону еь =ELm-sin • Частота незатухающего колебательного разряда конденсатора называется частотой собственных незатухающих колебаний контура. Она зависит от емкости С конденса- тора и индуктивности L катушки. Определим частоту собственных незатухающих колебаний кон- тура, исходя из условия равенства максимальных значений энер- гии электрического поля конденсатора и магнитного поля ка- тушки: CU2 Ы2 ^ит___ п1т , — 2 * Применяя известную нам формулу закона Ома для опреде- ления величины тока на участке цепи с конденсатором, актив- ным сопротивлением которого можно пренебречь, найдем вели- чину тока в конденсаторе, а следовательно, и в любом участке данной цепи, так как рассматриваемая нами цепь (контур) — п осл едовательная: / = -y- = t7<oC, CD С где U—действующее значение напряжения на зажимах кон- денсатора; /—действующее значение тока в конденсаторе (в цепи); 1 ---емкостное сопротивление цепи; со — угловая частота; С—емкость конденсатора. Помножив.левую и правую части последнего равенства на 1^2, получим 4 = ^о)С. (259) Подставив вместо 1т его выражение в формулу (258), по- лучим CU2m LU2mu>2C2 2 — 2 ’ 363
или 1 = <o2ZC. ? Отсюда получим формулу для определения угловой ча- стоты собственных незатухающих колебаний контур а: 1 со = ——. VLC (260) В соответствии с этим частота собственных неза- тухающих колебаний контура равна (О __ 1 2* ~ 2it]/Zc (261) и период собственных незатухающих колеба- ний контура равен j-=2k/ZC, , (262) где L — индуктивность в генри; С — емкость в фарадах; Т — период в секундах; f — частота в герцах. Пример 125. Определить частоту собственных незатухающих колебаний в контуре, если индуктивность катушки L = 4« 10“4 гн, емкость С = = 1,6* 10“9 ф и активное сопротивление г = 0. Решение. Применяя формулу (260), найдем угловую частоту соб- ственных незатухающих колебаний в контуре: <0 = -4= = —— 1 ---= 1,25 • 10е — . VLC 1/4-10—1,6- 10-е сек Частота собственных незатухающих колебаний в контуре равна . «о 1,25- 10б пп /_ 2я _ 2 314 - 1,99-10. гц. Период собственных незатухающих колебаний в контуре равен т = -у =. 1,99’. ю» = 5,10-6 сек- = 5 мксек- Посмотрим, каковы величины индуктивного и емкостного со- противлений в колебательном контуре при наличии в нем соб- ственных незатухающих колебаний. Индуктивное сопротивление т 1 г xl-^L- L -у с . 364
Ёмкостное сопротивление 1 1 л /17 Хс~ о>С ~ 1 — 1 С ’ утсс Следовательно, индуктивное и емкостное сопротивления ко- лебательного контура при наличии в нем собственных незатухаю- щих колебаний равны: %L = % С === • Сопротивление, равное порознь индуктивному и емкостному сопротивлениям колебательного контура при наличии в нем соб- ственных незатухающих колебаний, называется волновым сопро- тивлением р контура: (2бз> где р— волновое сопротивление контура в омах; L— индуктивность в генри; С— емкость в фарадах. Пример 126. Определить волновое сопротивление колебательного контура, если индуктивность контура Л = 9*10~2 гн, а емкость С = 4*10~8 ф. Решение. Применяя формулу (263), находим ,/ТГ >/9’10-2 Р - с - у 4.10-8 - 1500 ом. Теперь посмотрим, по какому закону изменяется энергия элек- трического поля конденсатора и энергия магнитного поля ка- тушки в колебательном контуре при наличии в нем собственных незатухающих колебаний. Мгновенное значение энергии электрического поля конденса- тора равно 1FZ CU'2 + C^-cos2^ М/э— 2 —' 2 — 2 Мгновенное значение энергии магнитного поля катушки равно W7 — Lp — LUm-sin “О2 _ • sin 2 — 2 2 — 2 Из полученных формул видно, что мгновенные значения энер- гий электрического поля конденсатора и магнитного поля катуш- ки несинусоидальны, так как они зависят соответственно от ква- драта конуса и синуса угла 365
Рис. 232. Графики колебания энергии электриче- ского \УЭ и магнитного WM полей Выясним, чему равна сумма мгновенных значений энергий электрического и магнитного полей для любого данного момен- та времени: Clfim cos2 a>t Lpm • sin2 u>t U7=U7a+U7M=—- m 2 Но так как CU^ _ Llm _ 2 — 2 ’T0 1Г сп2 nf1 (cosW + sin2^) = -^ Следовательно, cifl lP (264) т. е. сумма трического полякатушки наличии в нем собственных незатухающих ко- лебаний — величина постоянная, не зависящая от времени ксимальной магнитного поля. Это понятно, так как энергия в контуре не теряется на необратимые процессы. На рис. 232 приведены графики изменения во времени мгно- венных значений энергии электрического и магнитного полей и суммарной величины этих мгновенных значений энергии в коле- бательном контуре при наличии в нем собственных незатухающих колебаний. мгновенных значении поля в конденсатора колебательном энергии элек- и магнитного контуре при и равная энергии порознь величинам ма- электрического поля и 366
II. Собственные затухающие колебания в контуре Теперь рассмотрим электромагнитные колебания в таком контуре, в котором активным сопротивлением пренебречь нельзя. Допустим, что к конденсатору емкостью С, заряженному до напряжения Um, подключается в момент времени t = 0 катуш- ка, имеющая индуктивность L и активное сопротивление г (рис. 233). Как только цепь, состоящая из конденсатора и катушки, бу- дет замкнута, тотчас же начнется разряд конденсатора. Однако электромагнитный процесс разряда конденсатора уже будет но^ сить иной характер по сравнению с тем, что наблюдалось при’ наличии в контуре собственных незатухающих колебаний. Те- перь в контуре будут возникать потери энергии на необратимые процессы — на преобразование электрической энергии конден- сатора не только в магнитную, но и тепловую, которая будет рассеиваться в окружающее пространство. Если активное сопротивление контура меньше двойного вол- нового сопротивления его, т. е. г<2р = 2/4> (265) то разряд конденсатора носит колебательный характер. Одйако эти колебания в отличие от собственных незатухающих колеба- ний представляют собой затухающие колебания контура. Это значит, что амплитуда Um напряжения на зажимах конденса- тора и амплитуда 1т тока в контуре закономерно уменьшаются с каждым новым периодом. Энергия, запасенная в электриче- ском поле конденсатора, расходуется в процессе колебательного разряда на тепловые потери в активном сопротивлении контура, а колебания в контуре спустя незначительное время после нача- ла разряда конденсатора практически пре- кращаются (затухают). Колебания подобного рода называются собственными затухающими ко- лебаниями контура. Графически они изображаются кривой, называемой з а- тухающей синусоидой (рис. 234). Частота собственных зату- хающих колебаний контура за- висит не только от индуктивности А, емко- сти С, но и' от активного сопротивления г контура. Чем больше активное сопротивле- ние контура, тем меньше при всех прочих равных условиях частота собственных зату- хающих колебаний контура. Рис. 233. Разряд кон- денсатора через цепь с г и L 367
о t Рис. 234. Затухающая синусоида тока Угловая частота собственных затухающих колебаний контура определяется по формуле 1 . / 1 г2 Ф VLC у 4Р2 ’ где р = |/ — волновое сопротивление контура. (266) Из формулы (266) следует, что чем больше активное сопро- тивление контура, тем меньше угловая частота собственных за- тухающих колебаний контура. Если активное сопротивление контура равно нулю, то (0 = (00 1 VLC ’ т. е. угловая частота собственных колебаний контура становится максимальной возможной при заданных значениях L и С; в то же время это будет уже частота собственных незатухающих ко- лебаний контура. Пример 127. Определить угловую частоту, частоту и период собственных затухающих колебаний в контуре, если известно, что индуктивность L = = 0,01 гн, емкость С — 1 мкф и активное сопротивление контура г = 100 ом. Решение. Применяя формулу (266), находим угловую частоту соб- ственных затухающих колебаний контура: 1 |/. г2 = 1 ,/ г2 = 1 Л 1002 уьс ' W~VLcV 4_£_ 1/0,01 • 10-6 \ / . 0,01 с V 10-6 = 1041/1—0,25 = 8,65-10’ —. сек Частота собственных затухающих колебаний равна со 2тГ = 8,65-103 2-3,14 = 1,38-103 гц. Период собственных затухающих колебаний равен 7 = T = W® = 7'25'10-‘ 368
Степень затухания колебаний в контуре зависит от величины потерь в нем энергии на образование тепла в активном сопро- тивлении. Следовательно, чем больше активное сопротивление контура, тем при прочих равных условиях интенсивнее будет происходить затухание колебаний в контуре. Интенсивность затухания колебаний в контуре характери- зуется величиной, называемой затуханием контура. Затухание контура численно равно отношению активного со- противления к волновому сопротивлению контура: 4/ = (267) где d — затухание •* кон- тура; г— активное сопро- тивление контура в омах; р — волновое сопро- тивление контура в омах. Величина, обратная за- туханию контура, назы- вается качеством (доброт- ностью) контура: Q = 4-, (268) где Q — качество контура; d — затухание кон- тура. Чем выше качество кон- а тура, тем менее интен- сивно затухают собствен- ные колебания в контуре. На рис. 235 для сравне- ния приведены графики собственных затухающих колебаний при различных значениях качества (доб- ротности) контура и гра- фик собственных незату- хающих колебаний его. В радиотехнике обыч- но стремятся к тому, что- бы качество контура было высоким, т. е. чтобы в Рис. 235. Графики собственных коле- баний при. различных значениях каче- ства (добротности) контура: а — незатухающие колебания; б и в — затухаю- щие колебания (Q> > Q2) 24—1377 369
нем потери энергии за каждый период колебаний были сведены к минимуму и чтобы частота его собственных затухающих коле- баний приближалась к частоте его собственных незатухающих колебаний. Колебательные контуры считаются относительно хо- рошими, если их качество Q > 100, и плохими, если Q < 20. Пример 128. Активное сопротивление колебательного контура г = 2 ом, индуктивность L — 2 • 10“4 гн, емкость С = 5* 10—9 ф. Определить качество (добротность) контура и затухание его. Решение. Волновое сопротивление контура -/"Г i/ 2* Ю-4 опп р - J/ с - у 5,10-0 - 200 ом. Затухание контура Добротность (качество) контура ®-4-i = 100- В каждом колебательном контуре могут существовать только затухающие собственные колебания, так как на практике нет таких контуров, которые не имели бы активного сопротивления. Если же в контуре происходят незатухающие колебания, то они, как правило, бывают вынужденными, т. е. навязан- ными какой-либо сторонней электродвижущей силой. В этом случае источник электрической энергии периодически доставляет в контур необходимое количество энергии, чтобы скомпенси- ровать потери ее на необратимые процессы, например на выде- ление тепла в активном сопротивлении. Затухающие колебания в контуре можно сравнить с зату- хающими колебаниями маятника. Если бы маятник при колеба- ниях не имел никаких потерь своей заранее запасенной потен- циальной энергии (например, на трение в подшипниках, на со- противление воздуха), то теоретически его колебания должны были бы быть незатухающими и продолжаться сколь угодно долго. Если же. в системе маятника есть потери энергии при его ко- лебаниях, то последние будут затухающими. С каждым новым периодом колебаний амплитуда их будет уменьшаться, и в кон- це концов колебания маятника прекратятся. Чем больше потерь энергии в маятнике на необратимые про- цессы, тем интенсивнее происходит затухание колебаний. Чтобы сделать их незатухающими, необходимо за каждый период ко- лебаний сообщать маятнику такое количество энергии, которое он потерял в течение этого периода. Например, в часах эту энергию маятник получает от раскручивающейся сжатой пру- жины или от силы тяжести гирь, спускающихся вниз. 370
III. Апериодический разряд конденсатора Допустим, что активное сопротивление контура больше двой- ного волнового сопротивления, т. е. г > 2р. Выясним, чему будет равна угловая частота собственных за- тухающих колебаний в кон- туре для данного случая. и и Согласно формуле (266) t Ш “ VLC j/1-"Зр7 • Но по условию г > 2р, . или г2>4р2, или > 1. - В соответствии с этим под- 0----------•------ 7ТТ t коренное выражение Рис. 236. Изменение напряжения конден- сатора при его апериодическом разряде в последней формуле — ве- личина отрицательная. Следовательно, в данном случае угловая частота — мнимое число: . 1 1/ ® VLC * 4Р2 ’ где / = И—1 —мнимая единица. Но если угловая частота собственных затухающих колеба- ний контура — число мнимое, значит, в данной цепи колеба- тельный разряд невозможен. В ней будет происходить так на- зываемый апериодический разряд конденсатора. При апериодическом разряде конденсатора напряжение на его обкладках монотонно убывает от своей начальной величины до нуля (рис. 236). Разрядный ток в цепи вначале увеличивает- Рис. 237. Изменение тока во времени при апериодическом разряде конденсатора ся от нуля до некоторого максимума, а затем умень- шается, стремясь к нулю (рис. 237). Предельное минималь- ное активное сопротивле- ние контура, в котором при данных индуктивно- сти L и емкости С не бу- дет происходить колеба- тельный разряд конденса- тора, равно двойному ВОЛ- 24* 371
новому сопротивлению контура, т. е. г = 2р. В этом случае угловая частота равна нулю: (0 VLC * 1 4Р2 1 уьс о, V\=\ т. е. колебательный процесс в контуре невозможен. Итак, если активное сопротивление контура равно двойному волновому сопротивлению кон- тура или больше него, то в данном контуре раз- ряд конденсатора носит апериодический.ха- р а кт е р. Подобно тому как маятник не может совершать собственные колебания в среде с большим сопротивлением его движению, например в вязкой среде, так и в контуре, обладающем боль- шим активным сопротивлением, не может быть осуществлен ко- лебательный разряд конденсатора из-за больших потерь энергии в этом сопротивлении. § 110. ВЫНУЖДЕННЫЕ КОЛЕБАНИЯ В ЦЕПИ С ИНДУКТИВНОСТЬЮ, ЕМКОСТЬЮ И АКТИВНЫМ СОПРОТИВЛЕНИЕМ, СОЕДИНЕННЫМИ ПОСЛЕДОВАТЕЛЬНО Чтобы в цепи, состоящей из индуктивности L, емкости С и активного сопротивления г, происходили незатухающие синусои- дальные колебания, необходимо в этой цепи иметь источник электрической энергии с переменным синусоидальным напряже- нием. Он создаст в цепи переменный синусоидальный ток, для- щийся практически сколь угодно долгое время. Частота незату- хающих синусоидальных колебаний (тока) будет равна частоте приложенного к цепи синусоидального напряжения. Эта частота для рассматриваемой цепи будет вынужденной (навязанной) и в общем случае не совпадающей с частотой собственных незату- хающих колебаний этой цепи (контура). Рассмотрим основные соотношения электрических величин в цепи с индуктивностью, емкостью и активным сопротивлением, со- единенными последовательно, при наличии в ней вынужден- ных синусоидальных колебаний. Допустим, что в' цепи (рис. 238), состоящей из по- следовательно соединенных ин- дуктивности L, емкости С и активного сопротивления г, проходит синусоидальный пе- ременный ток Рис. 238. Схема цепи с последова- , тельно соединенными г, L и С I = 1т • sill 372
создаваемый источником электрической энергии переменного тока. Выясним, каково должно быть в этом случае напряжение, приложенное к цепи. Напряжение U, приложенное к данной цепи, расходуется в трех сопротивлениях: активном г, индуктивном xL = и ем- костном хс = . Ь CD С Напряжение, расходуемое в активном сопротивлении, совпа- дает по фазе с током, и его мгновенное значение равно иа =Imr-sw (tit. Напряжение, расходуемое в индуктивном сопротивлении, опе- режает по фазе ток на угол , и его мновенное значение равно «д = • sin (a>t + . Напряжение, расходуемое в емкостном сопротивлении, от- стает по фазе от тока на угол -у, и его мгновенное значение равно Мгновенное значение полного напряжения, приложенного ко всей цепи, равняется алгебраической сумме мгновенных значе- ний напряжений, расходуемых на активном, индуктивном и ем- костном сопротивлениях: и'=иа + uL + ис = 1т г-sin <ot + Im <t>L • sin (ш/ + J + + Zm~sin Так как все слагаемые в этом выражении — синусоидальные величины одинаковой частоты, то и сама сумма является сину- соидальной величиной той же частоты. А это значит, что к цепи приложено синусоидальное напряжение той же частоты, что и частота синусоидального тока в этой цепи. Теперь выясним, каково численное соотношение напряжений, имеющихся в цепи. Для этой цели построим векторную диаграм- му тока и напряжений. Вектор тока / отложим на векторной диаграмме по началь- ной оси, так как начальная фаза тока, по условию равна нулю. Вектор падения напряжения в активном сопротивлении Ua = Ir отложим по направлению вектора тока /, так как эти величины совпадают по фазе. Вектор падения напряжения в индуктивном 373
сопротивлении UL = I^L отложим вверх под углом у к векто- ру тока /, так как это напряжение опережает ток по фазе на угол . Вектор падения напряжения в емкостном сопротивле- нии отложим вниз под углом у к вектору тока /, так как это напряжение отстает от вектора тока на угол у. Сложив геомет- рически векторы Ua, UL и UCy получим вектор полного напря- жения U, приложенного ко всей цепи: U=Ua + UL + Uc. Рис. 239. Векторная диа- грамма для сличая, когда UL > UC Рис. 240. Векторная диа- грамма для случая, когда UL<UC На рис. 239 приведена векторная диаграмма для случая, ког- да UL > £7С, а на рис. 240 — для случая, когда UL < Uc. Выделим из векторных диаграмм, изображенных на рис. 239 и 240, треугольники напряжений (рис. 241). Из треугольника напряжений, показанного на рис. 241, а, + (269) а из треугольника напряжений, показанного на рис. 241,6, U=]/Ul + (UC-UL)\ (270) Но так как (UL— UC~)2 = (UC— UL)2, то формулы (269) и (270) равнозначны. В силу этого независимо от того, будет ли UL > Uc или, наоборот, UL < Uc, для определения полного на- пряжения обычно применяют формулу (269). 374
Разделив стороны треугольников напряжений (рис. 241, а и 241,6) на величину тока /, получим треугольники сопротивлений (рис. 242). Из треугольника сопротивлений, показанного на рис. 242, а, находим, что полное сопротивление Z равно * (271) а из треугольника сопротивлений, показанного на рис. 242, б, на- ходим, что z = / + <2?2) (1 \ 2 /1 \ 2 соА—— a)Lj , то формулы (271) и (272) равнозначны, а поэтому в дальнейшем для определения полного сопротивления мы будем применять формулу (271). Рис. 241. Тре- угольники напря- жений: а — для случая, когда UL > Uq ; б — для случая, когда <U с Рис. 242. Треугольники со- противлений: а — для случая, когда > Uq\ б — для случая, когда < Uq Разность индуктивного и емкостного сопротивлений цепи*на- зывается полным реактивным сопротивлением цепи и обозначается буквой х: x-xl—xc = wL — -^, (273) где х— полное реактивное сопротивление цепи в омах. Следовательно, формулу для определения полного сопротив- ления цепи в рассматриваемом случае можно записать так: Z = -|/r2 + x3 . (274) Формула закона Ома для рассматриваемой нами цепи при- нимает следующий вид: J = и =_____________£7________ Z <275) 375
Угол сдвига фаз тока / и напряжения U найдем из треуголь- ника сопротивлений (рис. 242) по тангенсу этого угла ___1_ = <276) Если > 0, то угол ср > О, что соответствует индук- тивному режиму в цепи, т. е. случаю, когда ток отстает по фазе от напряжения на угол ср. Если же оз£—то ? чт0 соответствует емкостному режиму в цепи, т. е. случаю, когда ток опережает по фазе напряжение на угол ср. Определив угол сдвига фаз тока / и напряжения Г7, прило- женного к цепи, можем найти мгновенное значение этого напря- жения по формуле u = Um-sin(®t + <р). (277) Теперь посмотрим, какова мощность, развиваемая источни- ком электрической энергии в данной цепи. Мгновенная мощность, развиваемая источником электриче- ской энергии в цепи, равна p=±ui = 6^-sin (о)/ + ср) /m-sin (278) Если ср > 0, т. е. цепь имеет индуктивный характер нагрузки, то выражение (278) принимает следующий вид: р = UI • cos ср — UI • cos + ср). (279) Если же ср < 0, т. е. в цепи имеет место емкостный характер нагрузки, то выражение (278) принимает следующий вид: p = UI* cos ср — UI-cos (2<о/ — ср). (280) Следовательно, независимо от режима в данной цепи (индук- тивного или емкостного) мгновенная мощность в ней — несинусоидальная величина, изменяющаяся с двойной частотой по сравнению с частотой тока. Активная мощность в данной цепи равна Pa = UI-zos^ (281) где cos <Р = = —----- г - . (282) 1/г2+(ш£- г v соС / 376
Реактивная мощность P, = 67-sin<p, (283) где <о£ — sin ? = -г “== • (284) Эта мощность в рассматриваемой цепи характеризует коле- бания энергии только между источником и всей цепью и не учи- тывает местных колебаний энергии между магнитным полем ка- тушки и электрическим полем конденсатора. Это видно из того, о)А---~ . . соС что sin <р = sm --------- и, следовательно, реактивная мощ- ность Рг зависит не от абсолютных величин индуктивного (coL) и емкостного сопротивлений, а от их разности Если эта разность велика, то относительно велики величины sin с? и Рп и, наоборот, если она мала, то относительно мала и реак- тивная мощность. Если, например, максимальная энергия, запасаемая в маг- нитном поле тока катушки, больше максимальной энергии, запа- саемой в электрическом поле конденсатора, т. е. то магнитное поле тока катушки, исчезая, передает не всю свою энергию нарастающему электрическому полю конденсатора, а только часть ее, равную максимальной энергии этого поля, т. е. CU2Cm —2—• Остающаяся часть энергии магнитного поля, равная L/2 CI fl —2------2— > возвращается генератору электрической энергии. Наоборот, когда магнитное поле нарастает, оно получает необ- си2Ст ходимую ему энергию —%— от исчезающего электрического LIm CUCm поля конденсатора, а остальную часть энергии —%--------%— по- лучает от генератора. Таким образом, чем меньше разность меж- ду максимальными значениями энергии магнитного поля тока катушки и электрического поля конденсатора, тем меньшее ко- личество энергии колеблется между всей цепью и генератором электрической энергии. В частном случае, когда максимальное количество энергии, запасаемой в магнитном поле тока катушки, 377
численно равно максимальному количеству энергий, запасаемой в электрическом поле конденсатора, эти поля полностью обеспе- чивают друг друга энергией и не нуждаются в получении ее от генератора. В заключение следует отметить, что местные колебания энер- гии в цепи между магнитным полем тока катушки и электриче- ским полем конденсатора иногда могут значительно превысить по своей мощности колебания энергии между всей цепью в це- лом и генератором электрической энергии. Это возможно в том случае, когда индуктивное сопротивление катушки coL и емкост- ное сопротивление конденсатора будут порознь значительно больше активного сопротивления г цепи и в то же время будут относительно мало различаться между собой по абсолютной вели- чине. В этом случае падения напряжения на индуктивном сопро- тивлении UL и на емкостном сопротивлении Uc. могут превысить не только падение напряжения в активном сопротивлении, но и напряжение, приложенное ко всей цепи. В самом деле, если о)£^>ги —уг г, то Ir и /—или, иначе, UL Ua и Uc Ua. Если же одновременно с этим близко по абсолютной величине к —U , то wL------U и в соответствии с этим UL — Uс — малые величины, поэтому и= 1^ + (<4-^с)2 uL и аналогично u=Vu^ + (uL-ucy С ис, т. е. напряжения на индуктивности UL и на емкости Uc в рас- сматриваемом случае значительно превышают напряжение, при- ложенное к цепи. Пример 129. Цепь переменного тока состоит из последовательно соеди- ненных активного сопротивления г = 20 ом, катушки, имеющей индуктивность L = 0,3 гн, и конденсатора, емкость которого С — 4 мкф. Определить режим в этой цепи, если известно, что действующее значение напряжения, прило- женного к цели, U = 120 в, а угловая частота его со = 1000 —— . сек Решение. Индуктивное сопротивление катушки xL = toL = 1000 • 0,3 = 300 ом. _ Емкостное сопротивление конденсатора хГ = \ 1П_Г = 250 ом. с тС 1000-4-10 6 Полное реактивное сопротивление цепи X = xL — хс = 300 — 250 = 50 ом* 378
Полное сопротивление цепи Z = ]/г2 + х2 = }/202 + 502 = 53,8 ом. Действующее значение тока /=_z =й§ = 2,23 а- Эо,о Напряжение на активном сопротивлении Ua = 1г = 2,23-20 = 44,6 в. Напряжение на катушке UL = IxL = 2,23-300 = 669 в. Напряжение на конденсаторе Uc = 1хс = 2,23-250 = 557,5 в. Угол сдвига фаз тока I и напряжения U находим, определив tgc?: tg<p = *L - *с _ 300-250 _ г “ 20 откуда по таблицам тангенсов находим, что <р =68° 10'. Коэффициент мощности равен cos? = ^- = 5^=0.372. Z Оо,о Активная мощность Ра = Ш-cos= 120-2,23-0,372 = 100 вт. Реактивная мощность Р. = t/7-sin ? = 120-2,23^ = 250 вар. Оо,о Полная мощность Pt = Ul= 120-2,23 = 270 ва. Максимальная энергия, запасаемая в магнитном поле катушки, равна Максимальная энергия, запасаемая в электрическом поле конденсатора, №. _ = = 4 Ю-(1/2.557.5Г _ Количество энергии, которой взаимно обмениваются магнитное поле тока катушки и электрическое поле конденсатора, W = w9 т = 1,25 дж. 379
Количество энергии, которая колеблется между генератором электриче- ской энергии и всей цепью, IT = т - W9 т = 1,5-1,25 = 0,25 дж. Количество энергии, расходуемой <в активном сопротивлении за четверть периода, ^тепл = Ра = Ра = °-157 дж‘ § 111. РЕЗОНАНС НАПРЯЖЕНИЙ Если к цепи, состоящей из последовательно соединенных ак- тивного сопротивления г, индуктивности L и емкости С (см. рис. 238), приложить синусоидальное напряжение и = Um • siiW, то в ней возникнут вынужденные электромагнитные колебания, частота которых будет совпадать с частотой приложенного к цепи напряжения. Переменный ток, возникший в этой цепи, будет также изменяться по закону синуса с частотой, рав- ной частоте приложенного напряжения, т. е. он будет равен z = Zm-sin(a>/ —?). Если в данной цепи индуктивное сопротивление больше ем- костного, то угол сдвига фаз тока и напряжения — положи- тельная величина и в соответствии с этим режим в цепи носит индуктивный характер. Наоборот, если индуктивное сопротивле- ние меньше емкостного, то угол сдвига фаз тока и напряже- ния — отрицательная величина и режим в цепи носит емкостный характер. Величину угла ср сдвига фаз тока и напряжения можно опре- делить по тангенсу этого угла г Изменяя величину L, или С, или угловой частоты со, можно добиться того, что тангенс угла <р станет равным нулю: tgcp=O. А если это так, то необходимо, чтобы угол <р тоже был ра- вен нулю, так как tg 0 = 0. Если же угол <р равен нулю, то это значит, что в данной цепи, состоящей из последовательно соединенных активного со- противления, индуктивности и емкости, ток и напряжение совпа- дают по фазе. Явление совпадения по фазе тока и напря- жения в цепи с индуктивностью и емкостью называется электрическим резонансом. 380
Различают две разновидности электрического резонанса в це- пях — резонанс напряжений и резонанс токов. Резонанс напряжений возникает в цепи с последовательно соединенными индуктивностью и емкостью, а резонанс токов — в цепи с параллельным соединением их. В настоящем параграфе мы рассмотрим явление резонанса напряжений, а в последующем, когда ознакомимся с режимами в параллельных цепях переменного тока, рассмотрим также и явление резонанса токов. Выясним, при каких условиях возникает резонанс напряже- ний, чем характерно это явление, каковы соотношения электри- ческих величин при нем и некоторые другие положения, имею- щие практический интерес. Итак, резонансом напряжений в цепи переменного тока, со- стоящей из последовательно соединенных индуктивности L, ем- кости С и активного сопротивления г, называется режим, при ко- тором ток и напряжение совпадают по фазе. При резонансе напряжений ток и напряжение изменяются синфазно, т. е. они имеют одинаковую частоту колебаний, изме- няются по одному и тому же синусоидальному закону и одно- временно достигают своих нулевых и максимальных значений одинакового знака. Благодаря этому в цепи создается макси- мальный размах электрических колебаний, т. е. максимальное значение тока при заданном напряжении, практически макси- мальные величины энергии электрического поля конденсатора и магнитного поля катушки, а также максимальные напряжения на индуктивности и емкости. Вначале установим, при каких условиях может возникнуть резонанс напряжений в цепи. Если в цепи с последовательно соединенными индуктив- ностью L, емкостью С и активным сопротивлением г возникает резонанс напряжений, то угол <р сдвига фаз тока и напряжения, а следовательно, и тангенс этого угла должны быть равны нулю, т. е. Последнее равенство может быть справедливым только в том случае, когда WqL~~^C = °’ или, иначе, . (285) 381
т. е. явление резонанса напряжений в цепи возникает тогда, ког- да в ней равны индуктивное (ооь и емкостное сопротивления. Иначе можно сказать, что резонанс напряжений в цепи воз- никает тогда, когда полное реактивное сопротивление цепи ста- новится равным нулю: ^=ф0£—L = o. 0 <*>ос Теперь, исходя из формулы (285), определим угловую ча- стоту тока (напряжения) в цепи при наличии в ней резонанса напряжений: (^LC = 1, ИЛИ (Оо==£^> или 0)0 VLC ’ (286) где ш0— резонансная угловая частота; L — индуктивность в генри; С—емкость в фарадах. Исходя из формулы (286), определим резонансную частоту колебаний в цепи (в контуре): с = % _ 1 0 2ти 27TJ/ZC’ (287) где fo — резонансная частота в герцах. Нетрудно видеть, что формула (286) для определения резо- нансной угловой частоты в точности совпадает с формулой (260) для определения угловой частоты собственных незатухаю- щих колебаний в цепи. Следовательно, можно сказать, что резо- нанс напряжений — это такое явление в цепях переменного тока, при котором наблюдается совпадение частоты вынужден- ных колебаний с частотой собственных незатухающих колебаний цепи. Пример 130. Определить резонансную угловую частоту wo и частоту fo цепи, индуктивность которой L = 4*10—4 гн, а емкость С =1,6*10-9 ф. Решение. По формуле (286) находим резонансную угловую частоту ШО---= 1 -----= 1,25 • 106 _L . VLC V4-10-6-1,6-10-» сек Резонансная частота равна , <»0 1250000 ОЛПЛЛЛ /о - 2я = 2.3,14 s 200000 гц. 382
Полное реактивное сопротивление цепи при резонансе напря- жений равно нулю, т. е. X = <o0L----^ = 0. и <*>оС Это объясняется тем, что электродвижущая сила самоиндук- ции катушки El и электродвижущая сила конденсатора Ес чис- ленно равны, но сдвинуты по фазе относительно друг друга на угол 180° (находятся в противофазе), а потому взаимно ком- пенсируются. Следовательно, индуктивное и емкостное сопротив- ления, будучи численно равными, но разными по знакам, также взаимно компенсируются. Равенство нулю полного реактивного сопротивления цепи при резонансе напряжений приводит к тому, что полное сопротивле- ние всей цепи равно ее активному сопротивлению. Следовательно, цепь в целом по отношению к генератору ведет себя как цепь с чисто активным сопротивлением z =/г2 + Л2 = = л (288) В соответствии с этим действующее значение тока в резо- нансной цепи при заданном напряжении U становится макси- мальным: / = 4 = (289) Это полностью соответствует тому, что при резонансе напря- жений получаются наиболее мощные электрические колебания в цепи вследствие совпадения по фазе тока и напряжения, а так- же совпадения вынужденной частоты колебаний и частоты соб- ственных незатухающих колебаний. Мгновенное значение тока при заданном напряжении u==Um-sm<&t будет определяться выражением i = -^--sin со/ = Im' sin «)/, поскольку ток и напряжение по условию совпадают по фазе. Падение напряжения на активном сопротивлении г согласно формуле (289) равно Ua0 = и, (290) т. е. падение напряжения на активном сопро- тивлении резонансной цепи Ua равно напря- жению, приложенному к цепи. Падение напряжения на индуктивном сопротивлении резо- нансной цепи равно Q — I 383
й падение напряжения на емкостном сопротивле- нии равно U ___т 1 UC0~ Л) ШоС- □а=и А так как по условию при резонансе напря- —— to женин индуктивное сопротивление равно емкост- ному, то и падение напряжения на ин- дуктивном сопротивлении в этом случае равно падению напряжения на емкостном сопротивлении, т. е. Рис. 243. Век- торная диа- грамма тока и напряжений для случая ре- зонанса напря- жений = (291) На рис. 243 приведена векторная диаграмма тока и напряжений для случая резонанса напря- жений. Здесь показано, что активная составляю- щая напряжения Ua совпадает по фазе с то- ком /о, напряжение UL опережает по фазе ток на угол у напряжение Uc отстает по фазе от тока на угол у и, наконец, полное напряжение U, равное по вели- чине £7Л, совпадает по фазе с током. Напряжения и С7со численно равны друг другу и нахо- дятся в противофазе, а поэтому их геометрическая сумма равна нулю. Это и приводит к тому, что все напряжение U, приложен- * ное к цепи, расходуется при резонансе напряжений на преодоле- ние активного сопротивления, т. е. U = Ua. Если при резонансе напряжений окажется, что индуктивное сопротивление <о0А цепи и равное ему емкостное сопротивление уу Окажутся порознь значительно больше активного сопротив- ления цепи г, то и падения напряжения в них UL 0 и £7С0 по- рознь будут значительно больше активной составляющей напря- жения Ua и, следовательно, значительно больше приложенного к цепи напряжения (Л Это видно из следующих математических соотношений: если 1 ш0С ’ то 1йг W — Л> Шос; но так как — Uа — U\ I0®0L — UL 0; Iо — Uс 0, 384
го £ZCt7ro = £7co. Перенапряжения, возникающие на катушке и на конденса- торе при резонансе напряжений, обусловлены тем, что внутри цепи создаются относительно мощные местные колебания энергии между магнитным полем тока катушки и электрическим полем конденсатора. Теперь посмотрим, каков энергетический баланс в цепи (контуре) при резонансе напряжений. Если напряжение, приложенное к резонансному контуру, из- меняется по закону M = t7m-sin то.мгновенное значение тока в контуре при резонансе напряже- ний определяется по формуле i = Zm-sin со/, так как ток и напряжение совпадают по фазе. Мгновенные значения напряжений на индуктивности и емко- сти в данном случае выражаются формулами «L==i/Z,m,Sin(^ +-J)» «С = UCm' Sin —-у). В соответствии с этим максимальное количество энергии, за- пасаемой в магнитном поле катушки за четверть периода, равно LI2 = (292) где —максимальная энергия, запасаемая в магнитном по- ле катушки, в джоулях; 1т — амплитуда тока в амперах; L — индуктивность катушки в генри. Максимальное количество энергии, запасаемой в электриче- ском поле конденсатора, равно cifl = (293) где — максимальная энергия, запасаемая в электрическом поле конденсатора, в джоулях; — амплитуда напряжения на конденсаторе, в вольтах; •С — емкость конденсатора в фарадах. 25—1377 3 85
Нетрудно показать, что максимальное количество энергии, запасаемой а катушке при резонансе напряжений, равно макси- мальному количеству энергии, запасаемой в электрическом поле конденсатора. Так как TO 2 = 2 г ’ 2* С* или 2 ” 2 ’ T. е. W9m=WMm, (294) Нетрудно также показать, что в любой момент времени при резонансе напряжений общее количество энергии, сосредоточен- ной в электрическом и магнитном полях резонансного контура, постоянно и равно максимальному количеству энергии, запасае- мой магнитным полем катушки или электрическим полем кон- денсатора. Мгновенное значение энергии магнитного поля катушки равно z/а LfL -sin2(D/ Мгновенное значение энергии электрического поля конденса- тора равно С Г ft Си^Ст-sin2 ((at-о-) W. = }------!-L = cos’ 2 2 Сумма мгновенных значений энергии магнитного и электриче- ского полей в резонансном контуре при резонансе напряжений равна lP си1 sin2 wt 4-• cos2 ш/. A A Но так как 2 = 2 ’ 386
то /А lP„ 1ГМ + lF9 = -^(sin2^ + cos2a>/) = -^- = —(295) На рЛ. 244 приведены ного, электрического полей от времени при резонансе напряжений. При резонансе напря- жений цепь по отношению к генератору является чи- сто активной и генератор посылает в цепь энергию, необходимую только для покрытия активных по- терь. Что же касается энергии, потребной для со- хранения магнитного поля графики изменения энергии магнит- и суммы этих энергий в зависимости Рис. 244. Графики изменения электри- ческой W9 и магнитной WM энергии во времени в катушке, и энергии, по- требной для создания электрического поля кон- денсатора, то генератор разгружен от них, так как катушка и конденсатор взаимно обме- ниваются энергией. Энергия исчезающего магнитного поля ка- тушки переходит в энергию нарастающего электрического поля конденсатора, и, наоборот, энергия исчезающего электрического поля конденсатора переходит в энергию нарастающего магнит- ного поля катушки. Чем меньше активное сопротивление цепи, тем при прочих равных условиях мощнее электромагнитные колебания в цепи при. резонансе напряжений, тем большим количеством энергии обмениваются электрическое и магнитное поля цепи и тем соот- ветственно больше напряжения на катушке индуктивности и на конденсаторе и больше величина тока в цепи. Индуктивное и емкостное сопротивления при резонансе на- пряжений порознь равны волновому сопротивлению цепи: XLO = V = = (296) ^»=Xc=ZC7=/?=p' (297) VLC ° где р — волновое сопротивление цепи. Затухание цепи (контура), как известно, равно отношению активного сопротивления г цепи к ее волновому сопротивле- нию р: d = f, (298) где d — затухание цепи. я 25* 387
Помножив числитель и знаменатель последней формулы на величину резонансного тока /0, получим V _ иа, U _ и * (299) т. е. затухание цепи численно равно отношению величины напряжения, приложенного к цепи при резонансе напряжений, к величине падения напряжения на индуктивном или на емкостном сопротивлении ее. Добротность (качество) — это величина, обратная затуханию цепи, т. е. Ч=Т = ЧГ=ЧГ. (300) где Q — добротность (качество) цепи (контура). Следовательно, добротность Q цепи — это величина, показы- вающая, во сколько раз напряжение на индуктивности и емкости в отдельности больше напряжения, приложенного ко всей цепи. В радиотехнических контурах стремятся добиться того, что- бы добротность контура была относительно большой, так как в этом случае при резонансе напряжений напряжение на конден- саторе (на катушке) во много раз превысит напряжение, прило- женное к контуру, что с успехом используется для усиления электромагнитных колебаний в радиотехнических цепях. Пример 131. К колебательному контуру, состоящему из индуктивности £=1,6*10“"3 гн, емкости С = 25«10“12$ и активного сопротивления г — 20 ом, приложено синусоидальное напряжение, действующее значение которого U = 1,2 в. Определить режим в этом контуре, если в нем возник резонанс напря- жений. Решение. Резонансная угловая частота а>о = = —------ = 5-10« —. VLC J/1,6- 10-э-25 10—« сек Волновое сопротивление контура Р 25-10“12 = 8000 ом. Затухание контура г 20 d -7-8000 “°’0025- Добротность контура 388
Напряжение на индуктивности и емкости контура U. n = Ur. = QU = 400-1,2 =- 480 в. ьи си Индуктивное сопротивление контура <d0L = 5-106.1,6-ю-з = 8000 ом. Емкостное сопротивление контура —“Т5" в "<"Тпв~Л ш-12 = 8000 ом* (о0с 5-106.25.10 12 Полное реактивное сопротивление контура х = <ogL----i-=r - 8000 — 8000 = 0. “0С ‘ Сдвиг фаз тока и напряжения Л 1 х 0 Л ф = 0, так как tg ф = — = —— = 0. т & т г 20 Величина тока в контуре '•--г-я-0'06»- Активная мощность, развиваемая током в контуре, Ра = /£г = 0,062-20 = 0,072 вт. Максимальная энергия, запасаемая в магнитном поле катушки, ш и‘т 1,6-Ю-з (1/2-0,06)2 w„m = — =------------2------- “ 5,76 ’10 дж- Максимальная энергия, запасаемая электрическим полем конденсатора, = К Ю-у -480)- Н 5,76.10- 6» Реактивная мощность в контуре р, = /2Лд - 0,062-8000 = 28,8 ва, т. е. она численно превышает активную мощность, развиваемую источником электрической энергии в контуре, примерно в 390 раз. При исследовании резонансных явлений и определении доб- ротности колебательных контуров важную роль играют резо- нансные кривые, выражающие зависимость действующего значе- ния тока или напряжения на индуктивности (емкости) цепи от частоты приложенного к цепи напряжения, индуктивности или емкости. Рассмотрим резонансную кривую тока, выражающую зависи- мость действующего значения тока от частоты. 389
Допустим, что задана цепь, состоящая из последовательно соединенных активного сопротивления г, индуктивности L и ем- кости С. К этой цепи подключен источник электрической энер- гии с переменным синусоидальным напряжением, частоту кото- рого мы можем плавно изменять от нуля до максимальной ве- личины — теоретически до бесконечно большой величины. • Как известно, действующее значение тока в заданной цепи можно определить по формуле I = ...... . (301) / / 1 \2 V 7 иг2+ Не изменяя действующего значения напряжения (7, активно- го сопротивления г, индуктивности L и емкости С, будем плавно изменять частоту cd (угловую частоту) генератора.от нуля до бесконечности и проследим, как будет в соответствии с этим из- меняться величина тока 7, определяемая формулой (301). Допустим, что частота f, а следовательно, и угловая часто- та ш равны нулю. Тогда при (о = 0 Х£ = ш4 = 0-£ = 0; хс = -^ = -Щ- = ео; XL — Хс = 0 — со = — оо; 2 = Р4Г2 + (— оо)2 = оо и в соответствии с этим ток равен нулю: Далее полагаем, что частота f, а следовательно, и угловая частота со равны бесконечно большим величинам. Тогда при со = оо со£ — оо • L = оо; —= 0; . wG оо-С 0)7, — = оо — 0 = оо; 2 =•• V Г2 + (оо)2 = оо и в соответствии с этим ток равен нулю: Итак, действующий ток в заданной цепи как при со = 0, так и при со = оо равен нулю. А это значит, что ток / при изменении частоты со от 0 до оо должен возрастать, достигать своего максимума при каком-то значении ш, а затем убывать до 0. Из формулы (301) видно, что действующее значение тока / в заданной цепи достигает максимума тогда, когда знаменатель 390
имеет минимальное значение. Нетрудно видеть, что максимума ток достигнет тогда, когда = 0, или, иначе, когда wZ, = -L. СО С А это означает, что максимума ток I достигнет в данной цепи при резонансе напряжений. Это и понятно, так как именно при резонансе напряжений электромагнитные колебания в цепи достигают наибольшего размаха. Рис. 245. Резонансная кривая тока Следовательно, ток I становится максимальным при угловой частоте w, равной резонансной угловой частоте 1 (I) = О)0 = . ° VLC Величина этого максимума тока I равна На рис. 245 приведена резонансная кривая тока, выражаю- щая зависимость тока / от частоты ю. На форму кривой резонанса влияет Затухание цепи d. Чем оно меньше, т. е. чем меньше активное сопротивление цепи по сравнению с ее волновым сопротивлением р, тем кривая резо- нанса будет острее и выше при всех прочих равных условиях, и, наоборот, она будет более пологой и ниже при относительно большом затухании цепи. На рис. 246 приведены две резонансные кривые тока для од- ной и той же цепи, но имеющей различные активные сопротив- ления, или, иначе, различные затухания, причем во всех случаях к ней было приложено одно и то же синусоидальное напряжение. 391
Рис. 245. Резонансные кривые тока: 1 — при малом затухании; 2 — при большом зату- хании только резонансной частоты, При остром резонансе, когда резонансная кривая имеет резко выраженную пи- кообразную форму, контур слабо отзывается на часто- ты, отличающиеся от его соб- ственной частоты, но зато при резонансе напряжений в нем возникают колебания с относительно большой ампли- oj тудой. Наоборот, при боль- шом затухании контура ам- плитуда колебаний при резо- нансе напряжений относи- тельно мала и контур легко воспринимает колебания не но и широкую полосу соседних с ней частот. Резонансная кривая в этом случае получается Тупой. В радиотехнических цепях колебательные контуры подби- раются обычно так: чтобы они хорошо пропускали определен- ную полосу частот. Например, при радиовещании для высокока- чественного приема звуковой передачи полоса частот радиопри- емника должна составлять 9000 гц. Полосу частот, хорошо пропускаемую колебательным конту- ром, называют полосой пропускания контура или шириной кри- вой резонанса. Полосу пропускания контура определяют по резонансной кри- вой, принимая за границы боковые частоты, при которых ток со- ставляет 70% резонансного (рис. 247). Зависимость полосы пропускания колебательного контура от затухания определяется по формуле /пР = ^/о, (302) W /,,р — полоса пропускания контура в герцах; а — затухание контура; /о — резонансная частота в герцах. Рис. 247. Полоса пропускания контура 392
Пример 132. Колебательный контур настроен в резонанс на частоту /0=Ю6 гц. Определить полосу пропускания контура, если затухание его d = 0,01. Решение. Применяя формулу (302), нахо- дим, что полоса пропускания контура fnp = df0 = 0,01 • 106 = 10000 гц. Из формулы (302) следует, что для узкой полосы пропускания контура необ- ходимо иметь малое затухание контура, или, иначе, высокое его качество (доброт- ность). Иногда приходится вести радиотехни- ческие передачи при очень широкой по- лосе частот (например, при телевизион- ной передаче полоса пропускания изме- ряется несколькими миллионами герц). В этих случаях контур должен работать на весьма высокой резонансной частоте. Пример 133. Колебательный контур настроен в резонанс на частоту fo = 5-IO7 гц. Опреде- лить полосу пропускания контура, если затухание Korfrypa tZ = 0,1. Решение. Применяя формулу (3Q2), на- Рис. 248. Схема антен- ного контура ходим /пр = dfQ = 0,1 -5-107 = 5-10б гц. Явление резонанса напряжений широко применяется в ра- диотехнике. Как известно, переносные радиостанции малой мощ- ности обычно работают в широком диапазоне радиочастот. По- этому радиостанцию надо настраивать на ту или иную рабочую волну. Чтобы добиться наиболее мощного излучения радиостан- ции, ее антенный контур необходимо настроить в резонанс коле- баниям, создаваемым генератором. На рис. 248 показан антенный контур радиостанции, индук- тивно связанный с катушкой Lr генератора через катушку свя- зи Lz. Помимо этой катушки, антенный контур имеет катушку индуктивности LK для грубой настройки и конденсатор С пере- менной емкости для точной настройки на заданную волну. Путем подбора ««гисла секций катушки LK и емкости конден- сатора С добиваются возникновения в антенном контуре резо- нанса напряжений, который возникнет в нем только в том слу- чае, если частота вынужденных колебаний генератора будет рав- на резонансной частоте антенного контура. Тогда ток в антен- ном контуре достигнет максимального значения, о чем можно судить по яркому накалу индикаторной лампочки Л. Напряже- ния на катушке и конденсаторе станут максимальными. Все это способствует наиболее эффективному радиоизлучению антенны. Убедившись, что антенный контур настроен в резонанс, вы-' включают индикаторную лампочку выключателем В.
ГЛАВА XXIII ЦЕПИ ПЕРЕМЕННОГО ТОКА С ПАРАЛЛЕЛЬНО СОЕДИНЕННЫМИ ПОТРЕБИТЕЛЯМИ § 112. ЦЕПЬ ПЕРЕМЕННОГО ТОКА С ДВУМЯ ПАРАЛЛЕЛЬНО СОЕДИНЕННЫМИ КАТУШКАМИ ИНДУКТИВНОСТИ Рассмотрим цепь переменного тока, состоящую из двух парал- лельно соединенных катушек индуктивности (рис. 249). Одна из этих катушек имеет активное сопротивление п и индуктивность Li, а другая — активное сопротивление г2 и индуктивность L2. Напряжение, приложенное к цепи, синусоидально, и его дейст- вующее значение равно U. Определим ток в неразветвленном участке цепи /о, если известны токи в параллельных ветвях /1 и /2. Так как в общем случае токи А и А в параллельно соединен- ных катушках не совпадают по фазе, то результирующий ток /о равен геометрической сумме этих токов: /о = А + 4 (зоз) В участке цепи с первой катушкой: 1) полное сопротивление катушки Zx = V Г] 4- (ш£,)2; 2) ток в ней 1 V + (шЛ,)» 3) сдвиг фаз (<pi) тока и напряжения определяем по тан- генсу: В участке цепи со второй катушкой: 1) полное сопротивление катушки z2 = /4 + (w£2)2; 394
2) ток в ней //^ + (<^2)2 3) сдвиг фаз (ср2) тока и напряжения определяем по тан- генсу: Зная величины токов h и /2, углы <pi и <р2 сдвига их фаз по отношению к напряжению, построим векторную диаграмму то- ков и напряжений. Рис. 249. Схема цепи перемен- ного тока, состоящей- из двух параллельно соединенных кату- шек индуктивности Рис. 250. Векторная диаграмма напряжения и токов при парал- лельном соединении катушек индуктивности Отложим по горизонтальной оси вектор напряжения Г7, а за- тем под углами и к нему — векторы токов /1 и /2 (рис. 250), Так как токи в катушках отстают по фазе от приложенного напряжения U, то векторы /1 и /2 должны быть повернуты на углы <р! и <р2 по часовой стрелке от начальной оси. Чтобы получить результирующий ток /о, сложим векторы Л и /2 геометрически по правилу параллелограмма. Из получен- ного параллелограмма определим результирующий ток /о, яв- ляющийся одной из его диагоналей, применяя известную фор- мулу тригонометрии Ч = Ч + Ч + 2/Л • COS (®2 — <Р1) или /о = / Ч + Ч + 2/,/2 • cos (®2 - <р,). (304) В частном случае, если <pi = <р2, т. е. в катушках имеется одинаковый сдвиг фаз, то . —?i = 0 и cos(?2 — <&) = !, 395
и тогда /0 = /7? 4-/2 4-2/Л = ]/(/, 4- /2)2 = Л + Ц, т. е. результирующий ток /о равен арифметической сумме токов составляющих. Из векторной диаграммы нетрудно понять, что /0. COS ср = А • COS ср! + /2 • COS ср2, Рис. 251. Векторная диаграмма напряжения и токов для случая параллельного соединения не- скольких катушек Так как напряжение, пр: откуда cos? = A^Pi + k^?) (305) 4) где ср — угол сдвига между током /о и напряжением U. Если цепь состоит из совокупности нескольких параллельно соединенных катушек индуктивно- сти, то результирующий ток бу- дет равен геометрической сумме векторов составляющих токов (рис. 251). В этом случае можно ограничиться геометрическим спо- собом решения: задавшись мас- штабом для тока, сложить все век- торы токов по правилу много- угольника (результирующий ток равен замыкающей стороне много- угольника, построенного на векто- рах составляющих токов) и полу- ченный результирующий вектор тока определить по масштабной линейке. Угол сдвига фаз опреде- ляется по транспортиру. оженное ко всем катушкам, оди- наково, то. откуда <306> т. е. токи в параллельных ветвях распределяются обратно про- порционально полным сопротивлениям ветвей. Пример 134. Даны две катушки, соединенные параллельно и подключен- ные к источнику с напряжением U — 120 в. Угловая частота ш = 1000 —— . сек Одна из катушек имеет активное сопротивление г\ = 30 ом и индуктивность Li = 0,04 гн, другая — активное сопротивление г2 = 80 ом и индуктивность 396
L2 = 0,06 гн. Требуется определить токи в параллельных ветвях, ток в нераз- ветвленном участке цепи, сдвиг фаз между токами и напряжением и мощ- ность, расходуемую в цепи. Решение. Величина тока в первой катушке у ^4-(coZ,^ 1/30- + (1000-0,04)2 Сдвиг фаз определим по формуле (oLi 1000-0,04 t tgT1 = —---------------------------3Q—= 1.333, откуда по таблицам тангенсов найдем ?1 = 53°10'. Величина тока во второй катушке I = U = 120 = 12 а 2 J/80*+ (1000-0,Об)2 Сдвиг фаз <рз определим по формуле . 1000-0,06 п„ ------80------°’75’ откуда по таблице тангенсов найдем ?2 « 36°50'. • Ток в неразветвленном участке цепи определяем по формуле (304) /0 = ]/ Zi + + 2/^2-соь (с?2—<Pi) = = ]/2,42 + 1,22 + 2.2,4- 1,2-си> (53°10' - 36°50') ~ = ]/ 5,76 + 1,44 + 2-2,4-1,2-0,96 « 3,56 а. Коэффициент мощности для всей цепи cos т = A-cos ?, + /;• cosy, = 2,4-0,64- 1,2-0,8 = ад5 /0 3,6 Сдвиг фаз между током /о и напряжением U можем найти по таблице косинусов. Для cos ф = 0,65 у « 49°20'. Мощность, расходуемая в первой катушке, Р, = /2Г, = 2,42-30 = 172,8 вт. Мощность, расходуемая во второй катушке, Р, = /|г2 = 1,22-80 = 115,2 вт. Мощность, расходуемая во всей цепи, Р = Р, + Р, = 172,8 + 115,2 = 288 вт. 397
§ 113. АКТИВНЫЙ И РЕАКТИВНЫЙ ТОК Задана цепь, состоящая из активного сопротивления г и ин- дуктивности L, соединенных пследовательно друг с другом. Если к данной цепи приложено синусоидальное напряжение « = Um’sin (of, то мгновенное значение тока будет равно Z = /m-sin (W —<р). Действующее значение тока г= и а и % J/V2 + (шД)2 Коэффициент мощности г г cos <₽ — -7 — . . т угг + (ш£)2 Активная мощность Pa = UI -cos <р, реактивная мощность Pr = UI' sin <? и полная мощность Pt = UI. На рис. 252 приведена векторная диаграмма тока и напря- жения для данного случая, где ток / показан отстающим по фазе от напряжения на угол <р. Запишем формулы активной и реактивной мощностей сле- дующим образом: Pa = U (/-cos <р); Pr = U(I-sin <р), т. е. отнесем множители cos <р и sin <р непосредственно к вели- чине тока I. Величина 1 - cos <р называется активной составляющей то- ка (4): /e = 7.cos<p, (307) и формулу для определения активной мощности можно записать так: Pa = UIa- (308) 398
Величина I • sin <p называется реактивной составляющей то- ка (/,): Ir = h sin ср, (309) и в соответствии с этим формулу для определения реактивной мощности можно записать так: Pr = UIr (310) Если в векторной диаграмме (рис. 252) разложить вектор тока / на две взаимно перпендикулярные составляющие, из кото- рых одну совместить с направлением вектора 67, а другую про- вести перпендикулярно ему, то первая из них будет равна I • cos <р, т. е. активной составляющей тока, а вторая 1 • sin <р, Рис. 252. Векторная диаграмма тока и напряжения для цепи с последовательно соединен- ными г и L с разложением тока на активную и реактив- ную составляющие Рис. 253. Треуголь- ник токов т. е. реактивной составляющей тока. Следовательно, вектор ак- тивной составляющей тока 1а совпадает по фазе с приложенным напряжениехМ 67, а вектор реактивной составляющей тока 1Г сдвинут по фазе по отношению к напряжению U на угол — у . Выделим из векторной диаграммы треугольник ОПаУ называе- мый треугольником токов (рис. 253). Из треугольников токов, применяя теорему Пифагора, получим соотношение между <о- ком I и его активной и реактивной составляющими I = V (/.си> + (/.Sin ?)* = / ра + /2. (311) Пример 135. В катушке индуктивности ток /=12 а и угол сдвига фаз напряжения и тока — 60°. Определить активную и реактивную составляю- щие гоков, а также активную и реактивную мощность, если напряжение 0= 120 в. Решение. Активная составляющая тока 1а — /• cos ф = 12 • cos 60° «= 6 а. Реактивная составляющая тока lr -=• /-sin 12-sin60° « 12*0,866 » 10,4 а. 399
Активная мощность Ра -= (J!a ~ 120-6 - 720 вт. Реактивная мощность Pr - UIr - 120-10,4 == 1248 ва. Активная и реактивная составляющие токов физического со- держания не имеют, и их надо рассматривать как вспомогатель- ные математические величины, облегчающие расчеты в электри- ческих цепях переменного тока. § 114. АКТИВНАЯ И РЕАКТИВНАЯ ПРОВОДИМОСТИ Допустим, что задана цепь переменного тока, состоящая из последовательно соединенных активного сопротивления г и ин- дуктивности L. Определим активную составляющую тока для этой цепи, вы- разив ее через напряжение V и через сопротивления этой цепи. Как известно, /a = /.Coscp, (312) где cos ? = 4'> Z = Vr2 + (<о£)2 — полное сопротивление цепи. Ток / по закону Ома равен / = -^ . Следовательно, формулу (312) можно записать так: т т V г Ia = I.ZQsy = — — f ИЛИ Л = (313) Выясним размерность величины : Г r 1 __ ом _ _L L Z2 J ~ ом2 ~~ ом ’ т. е. величина имеет размерность проводимости. А так как . эта величина входит в формулу для определения активной со- ставляющей тока, то она называется активной проводимостью и обозначается буквой g: k g = -^, (314) 1 где g— активная проводимость в ; г— активное сопротивление цепи в омах; Z— полное сопротивление цепи в омах. 400
Из формулы (314) следует, что активная проводимость g не является величиной, обратной активному сопротивлению, так как она зависит не только от активного сопротивления г, но и от индуктивного o)L: & Z2 г2 + (юЛ)2 Таким образом, формулу (313) для определения активной составляющей тока можно записать так: Л = (315) т. е. активная составляющая тока 1а равна произведению напря- жения U на активную проводимость g. Теперь определим реактивную составляющую тока, выразив ее через напряжение U и сопротивления этой цепи. Как известно, /, = /-sin?, (316) где sin ср = , т и и в то же время I = Следовательно, формулу (316) можно записать так: или 4 = ^. (317) Выясним размерность величины : г шЛ П ом _L_ L Z2 J ом2 им * coL « т. е. величина имеет размерность проводимости. А так как эта величина входит в формулу для определения реактивной со- ставляющей тока, то она называется реактивной проводимостью цепи и обозначается буквой Ь: ь = (318) , 1 где b — реактивная проводимость в —; — индуктивное сопротивление цепи в омах; Z — полное сопротивление цепи в омах. 1377 401
Из формулы (318) следует, что реактивная проводимость b не является величиной, обратной реактивному сопротивлению цепи, так как она зависит не только от реактивного сопротив- Рис. 254. Треугольник про- водимостей ления (dL, но и от активного г: А__ <о£ _ <oL 0 — ~ Г2 + (шЛ)2 ‘ Таким образом, формулу для опре- деления реактивной составляющей тока можно записать так: Ir = Ub, (319) т. е. реактивная составляющая тока 1Г равна произведению на- пряжения U на реактивную проводимость цепи Ь. Если все стороны треугольника токов разделить на напряже- ние (7, то получим так называемый треугольник проводимостей (рис. 254), в котором g— активная проводимость, b — реактив- ная проводимость и У — полная проводимость. Из треугольника проводимостей находим Y = Vg* + &, (320) т. е. полная проводимость У равняется корню квадратному из суммы квадратов активной и реактивной проводимостей. Так как У = -^-,что видно из треугольника проводимостей, то к=4"> (з2о т. е. полная проводимость цепи есть величина, обратная ее пол- ному сопротивлению. В частном случае, если активным сопротивлением цепи мож- но практически пренебречь, что часто бывает в практике, фор- мулы (314), (318) и (320) приобретают следующий вид: & г2 + (<о£)2 = 0 + (о»£)2 °’ wL a>L 1 ’= г2 + (ш£)2 ~ 0 + (ф£)2 ’ Y = Vg* + А2 = V0 + & = Ь. Пример 136. Катушка индуктивности обладает активным сопротивлением г = 20 ом и индуктивностью L = 0,04 гн. Определить активную, реактивную и полную проводимости катушки, если частота тока в катушке /' = 50 гц. Решение. Активная проводимость катушки г ______________ г * 20 0 036 1 g “ Z2 — г2 + (ш£)2 = 202 + (2-3,14-50-0,04)» “ ’ ом ' 402
Реактивная проводимость шЛ <*L 2.3,14 50.0,04 0 0295-L* Z2 ” /-2 4- (<dL)2 “ 202 + (2.3,14.50.0,04)2 = ’ ом * Полная проводимость У ш, Vgt 4- />2 = /0,0362 + 0.02252 = 0,042 —. § 116. ОПРЕДЕЛЕНИЕ СОПРОТИВЛЕНИЯ, УГЛА СДВИГА ФАЗ, МОЩНОСТИ И ВЕЛИЧИНЫ ТОКА В ЦЕПИ ПОСРЕДСТВОМ ЕЕ ПРОВОДИМОСТЕЙ Если заданы проводимости цепи g, b и У, то по формулам (314), (318) и (321) можно определить сопротивления цепи: откуда r=gZ2, или г = -Д; (322) Ь = откуда ®L=gZ2, или = (323) Z« 4-. или Z = . (324) У ’ у? + ь -2 4 > По формулам (308) и (310) можно определить мощности через проводимости. Активная мощность Р —UI Но согласно формуле (315) Ia = Ug, а поэтому Pa=*b2g. (325) Реактивная мощность Рг = ^г Но согласно формуле (319) Ir=Ub, а поэтому Pr = U2b. (326) Полная мощность Pt = у/р2 4- Р2 = t/2 = U2Y. (327) Коэффициент мощности cos <р = , R 1 а так как г = -у^ и Z = -y-, то cos? = —Lr=-y . (328) У*-у 26* 403
Аналогично найдем выражение и для sin <р: sin <р = -^ = —у- = 4" • (329) /2_ 1 Y Из формулы (329) и (328) определим tg tg<p = ^=A = —• (330) 6 т cos tp У У g v ’ Величину тока в цепи найдем по формуле / = = = (331) •Пример 137. Активная проводимость цепи g = 0,04—, реактивная про- ом водимость b = 0,03, напряжение, приложенное к цепи, [7=125 в. Опре- делить сопротивление цепи, мощность, ток в цепи и сдвиг фаз между током и напряжением. Решение. Полная проводимость цепи У = Vg* + b2 = J/0,042 + 0,ОЗ2 = 0,05 . Ток в цепи Z = UY = 125’0,05 = 6,25 а. А кти в н ое сопротивлен ие g 0,04 1Г Л_ =------------------------------ = 16 пл/ Реактивное сопротивление л у. Активная мощность Ра = U*g~ 1252-0,04 = 625 вт. Реактивная мощность Рг = U2b = 1252’0,03 = 468,75 ва. Коэффициент мощности а 0 04 cos у =-р-= ^? = 0,8 и = 36°50 . 0,052 0.03 § 116. ПРИМЕНЕНИЕ МЕТОДА ПРОВОДИМОСТЕЙ К РАСЧЕТУ ЦЕПИ С ПАРАЛЛЕЛЬНО СОЕДИНЕННЫМИ КАТУШКАМИ ИНДУКТИВНОСТИ Для расчета цепей переменного тока с параллельно соединен- ными потребителями можно воспользоваться векторными диа- граммами. Однако этот метод обладает тем недостатком, что он не всегда дает требуемую точность решений. Поэтому при рас- 404
чете цепей переменного тока обычно прибегают к методам ана- литического расчета, например к методу проводимости, символи- ческому методу и др. В данном разделе мы ознакомимся с ме- тодом проводимостей, который, как увидим далее, можно успеш- но применить для расчета па- раллельных и смешанных цепей переменного тока. Вначале покажем примене- ние метода проводимостей для расчета параллельной цепи, со- стоящей из двух параллельно соединенных катушек индуктив- ности (рис. 249). Рис. 255. Векторная диаграмма на- пряжения и токов для случая парал- лельного соединения двух катушек индуктивности Построив векторную диа- грамму токов и напряжения (рис. 255), разложим все токи в ней на активные и реактив- ные составляющие. Из векторной диаграммы нетрудно видеть, что активная со- ставляющая общего тока равна сумме активных составляющих токов в параллельных ветвях: / • COS <f = Ц • COS <Pj + /2 • COS ?2 — I0l + Iai, или согласно формуле (315) Ug = Ugx + Ug* откуда, сократив на V, получим £ = £+£2- (332) т. е. активная проводимость g всей параллельной цепи равна сумме активных проводимостей параллельных ветвей ее. Далее, из векторной диаграммы находим, что I-sin ? = Л • sin ?! + /2 • sin ?2 = 1п + /,2, С или согласно формуле (319) Ub = Ubx + Ub2, откуда, сократив на U, получим £ = £1 + Л2, « (333) г. е. реактивная проводимость цепи равна сумме реактивных проводимостей параллельных ветвей ее. Полную проводимость всей цепи найдем по формуле (320) У = V ё^+b2 = Г(& +g2)2 + to +А)2. (334) 405
(335) (336) (337) (338) прово- Величина тока 1 в неразветвленном участке цепи будет равна I = UY = U |/ (g, -^2)г 4-(^ + ^)2 Активную мощность определим по формуле (325) ^ = ^ = ^2(^+^)> реактивную мощность — по формуле (326) Р, = (М = (/<(Ш), а коэффициент мощности в цепи — по формуле (328) СОЗ Т = 4- а + е’ = . V (£1 + £2? + + Ь2)2 Таким образом, зная активную, реактивную и полную димости цепи, нетрудно рассчитать величину тока в неразвет- вленном участке цепи, мощность, расходуемую в цепи, и сдвиг фаз тока и напряжения, применяя аналитический метод реше- ния задачи при помощи проводимостей. Пример 138. Две катушки индуктивности, обладающие активными сопро- тивлениями Г1 = 30 ом, г2 = 25 ом и индуктивностями L\ — 0,04 гн и L2=* <=0,025 гя, соединены параллельно и подключены к источнику электрической энергии с напряжением (7= 120 в и угловой частотой о> = 1000 — . Опре- делить ток в неразветвленном участке цени, активную и реактивную мощно- сти и сдвиг фаз между общим током и напряжением. Решение. Активная проводимость первой катушки , г' 30 - 0 012 1 + (C0LJ2 = 302 + (1000-0,04)2 U,U1Z ом • Активная проводимость второй катушки г2 25 — 0 09 1 gi= r3 + “ 252 + (1000-0,025)2 ’ ом ' Общая активная проводимость цепи g » gl + g, = 0,012 + 0,02 - 0,032 —. ОМ Реактивная проводимость первой катушки __ 1000-0,04 __ 1 1 = 4- («.LJ2 “ 302 + (Ю00 0,04)2 - u-uw ом- Реактивная проводимость второй катушки г “ % + (a>£s)2 252 + (1000-0.025)2 “ ’ ом ‘ зе
Общая реактивная проводимость цепи ь = bl + Ьг = 0,016 + 0,02 - 0,036 —. ом Общая проводимость всей цепи У = yg2 + *2 = у (0,032)2 + (0.036)2 = 0,0482 . Ток неразветвленного участка цепи / = UY - 120-0,0482 = 5,78 а. Активная мощность Ра « [pg = ISO2-0,032 « 460 вт. Реактивная мощность pr » (pb = 1202-0,036 = 518 ва. Полная мощность PL e IPY = 1202-0,0482 = 694 ва. Коэффициент мощности Угол сдвига фаз = 48°20'. § 117. ЦЕПЬ ПЕРЕМЕННОГО ТОКА С ПАРАЛЛЕЛЬНО СОЕДИНЕННЫМИ ИНДУКТИВНОСТЬЮ И ЕМКОСТЬЮ и активным сопротивлением rlt а U Рис. 256. Схема цепи, состоящей из парал- лельно соединенных катушки индуктив- ности и конденсатора Допустим, что задана цепь переменного тока, состоящая из двух параллельных ветвей. В одной из них имеется катушка, об- ладающая индуктивностью L\ в другой — конденсатор, имеющий емкость Сг и последовательно с ним со- единенное активное сопро- тивление г2 (рис. 256). К этой цепи приложено синусоидальное напряже- ние и = Uт • sin дей- ствующее значение кото- рого равно U. Найдем ток в неразветвленном участке цепи, применив для этого метод проводи- мостей. 407
Для построения векторной диаграммы токов и напряжения определим токи в параллельных ветвях и углы сдвига фаз токов и напряжения. В первой ветви ток Л найдем по формуле 1 V 4 + (<°М)г Угол <р] сдвига фаз тока Л и напряжения U будет положи- тельным, поскольку ток Л отстает по фазе от напряжения. Этот угол определим по его тангенсу: tg?i=7T- Во второй ветви ток 72 найдем по формуле Угол сдвига фаз тока /2 и напряжения U будет отрицатель- ным, потому что юк /2 опережает по фазе напряжение U. Этот угол определим также по его тангенсу: 1 . * tg?2 = -7- / Определив токи 71 и /2, а также углы <Pi и <р2, построим век- торную диаграмму токов и напряжения U (рис. 257). Для этой цели отложим вектор напряжения U по начальной оси, так как по условию начальная фаза напряжения равна нулю. Затем под углом 91 к вектору напряжения U отложим вектор тока 7Ь при- няв во внимание, что ток /1 отстает от напряжения U по фазе. Рис. 257. Векторная диаграмма напряжения и токов для цепи с параллельно соединен- ными катушкой индуктивности и конденса- тором 408
После этого отложим под углом <р2 к вектору напряжения U вектор тока /2, приняв во внимание, что ток /2 опережает напря- жение U по фазе. Сложив геометрически векторы токов Л и /2, получим вектор тока I неразветвленного участка цепи: Л+ 4 = А (339) Разложим на векторной диаграмме (рис. 257) все токи на активные и реактивные составляющие. Активные составляющие токов, имеющихся в параллельных ветвях, совпадают по фазе с напряжением U. Их величины оп- ределим по формулам Ila = Iv cos ?! = Ug{ = и, п + (шм)2 Ла = /2 • cos ?2 = Ug2 = и ——. Активная составляющая тока / в неразветвленном участке цепи равна арифметической сумме активных составляющих то- ков параллельных ветвей: 1а = Ла + Ла = Л1 COS ?, + Л ’ COS ?2 = U (gt + g2). Реактивная составляющая тока Л в катушке индуктивности отстает по фазе от напряжения U на угол -у- .Ее величину опре- делим по формуле 11г = Ц • sin ?! = Ub[=и , <fl£1,—. lr 1 n 1 4 + (ш£1)2 Реактивная составляющая тока Ц в конденсаторе опережает напряжение U по фазе на угол 4. Ее величину определим по формуле 1 Л, = л• sin ?2 = Ub2 = и— . Л + (4г Нетрудно видеть, что реактивные составляющие токов парал- лельных ветвей /1г и 12г находятся в противофазе (сдвинуты по фазе на угол 180°). А это значит, что реактивная составляющая тока в неразветвленном участке цепи равняется разности реак- тивных составляющих токов /1л и /2г. 409
В рассматриваемом случае реактивная составляющая тока в первой ветви /1Л больше реактивной составляющей тока во второй ветви /Зп что видно из векторной диаграммы (рис. 257). Следовательно, реактивная составляющая тока 1Г в нераз- ветвленном участке цепи равна Отсюда находим ^ = ^1-^2, (340) где Ь — результирующая реактивная проводимость всей цепи. Рис. 258. Треугольник токов На основании формулы (340) сделаем такой вывод: резуль- тирующая реактивная проводимость двух параллельно соединен- ных ветвей, из которых одна содержит индуктивность, а дру- гая — емкость, равна разности реактивных проводимостей этих ветвей. Выделим из векторной диаграммы треугольник токов (рис. 258), из которого найдем, что / = = |/(/1в + /2а)2 4- (/1г - /2г)2 . (341) Разделив все стороны треугольника токов на величину на- пряжения U, получцм треугольник проводимостей (рис. 259). т Рис. 259. Треугольник проводимостей Из треугольника проводимостей находим, что у = |/ £2 + £2 = V (Й 4-&J2 4-(Л-^)2 . (342) Т. е. полная проводимость равняется корню квадратному из сум- мы квадратов результирующих активной и реактивной проводи- мостей. 410
Обший ток в неразветвленном участке цепи определим по формуле I = UY = UV(gy +g2)2 + (^-^)2. (343) Угол сдвига фаз между током / в неразветвленном участке цепи и напряжением U найдем по его тангенсу из треугольника проводимостей: <з44> отсюда по тригонометрическим таблицам определим угол <р. В рассматриваемом случае угол > 0, так как /1г>/2г и’ следовательно, Ь\ > Ь2. В соответствии с этим цепь в целом для Рис. 260. Векторная диаграмма токов для цепи с параллельно соединенными катушкой индуктивности и конденсатором- Рис. 261. Треугольник токов генератора является потребителем с индуктивно-активным ха- рактером нагрузки. Если бы реактивная составляющая тока во второй ветви была больше, чем в первой то Депь в целом для гене- ратора являлась бы потребителем с емкостно-активным харак- тером нагрузки. Векторная диаграмма для этого случая изобра- жена на рис. 260, треугольник токов — на рис. 261 и треуголь- ник проводимостей — на рис. 262. В этом случае реактивная проводимость в первой ветви была бы меньше, чем во второй В соответствии с этим результирующая реактивная проводимость в цепи (&i — b2 < 0) стала бы отрицательной ве- 411
личиной. Но так как (Ь± — Ь2у = (Ь2— &i)2 и эти выражения имеют положительные значения, то для определения полной про- водимости цепи сохраняет силу формула (342), примененная выше для случая, когда Ьг > 62. Рис. 262. Треугольник проводимостей Следовательно, полная проводимость и в данном случае, ко- гда bi < Ь^, определяется по формуле г=Г(^+^)2 + (^-^)2 и в соответствии с этим ток в неразветвленном участке цепи равен I=uy = . Угол сдвига фаз тока / в неразветвленном участке цепи ина- пряжения U, приложенного к цепи, в данном случае будет отри- цательным, так как Ьг <^Ь2: и, следовательно, ?i < 0, т. е. цепь в целом для источника элек- трической энергии является потребителем с емкостно-активным характером нагрузки. Полная мощность, развиваемая источником электрической энергии в цепи, равна pi==UI = CPY. (345) Активная мощность, развиваемая источником электрической энергии в цепи, равна Ра = UI- cos y = = (gx +g2). (346) Реактивная мощность, развиваемая источником электриче- ской энергии в цепи, равна P, = L7-sin<p = C/2£ = £72(^—М- (347) Здесь необходимо обратить внимание на то, что реактивная мощность Рг характеризует колебание энергии между источни- ком электрической энергии и всей цепью и не учитывает мест- ных колебаний энергии между магнитным полем тока катушки 412
и электрическим полем конденсатора. Это видно из того, что ве- личина реактивной мощности согласно формуле (347) зависит от разности реактивных проводимостей (61 — Z>2), а не от абсо- лютных значений Ь\ и 62. Если разность (&i— fe2) велика, то ве- лика и реактивная мощность, т. е. источник электрической энер- гии обменивается со всей цепью большим количеством энергии; наоборот, если эта разность мала, то источник обменивается со всей цепью малым количеством энергии. Максимальное^ количество энергии, запасаемой магнитным полем катушки, равно Максимальное количество энергии, запасаемой электрическим полем конденсатора, равно W = w эт 2 ‘ Если энергия электрического поля конденсатора и энергия магнитного поля катушки относительно велики и при этом чис- ленно мало отличаются друг от друга, то местные колебания энергии между этими полями будут более мощными, чем коле- бания реактивной энергии между всей цепью и генератором электрической энергии. В этом случае токи в параллельных ветвях могут значитель- но превысить ток в неразветвленном участке цепи из-за относи- тельно больших величин реактивных составляющих токов в па- раллельных ветвях по сравнению с реактивной составляющей тока в неразветвленном участке цепи. Пример 139. К катушке индуктивности, обладающей активным сопро- тивлением г\ = 80 ом и индуктивностью L = 0,03 гн, подключен параллельно конденсатор, имеющий емкость С = 25 мкф, и последовательно с ним вклю- чено активное сопротивление г2 — 20 ом. Определить токи в параллельных ветвях и в неразветвленном участке цепи, если приложенное к цепи напря- жение (7=120 в и угловая частота тока со = 2000 -— (рис. 256). Решение. Активная проводимость первой ветви 6 ________§2________ А АЛО 1 + * 802 + (2000-0,03)2 я=и’ииоол/- Активная проводимость второй ветви 63 / 1 \ 2 / 1 \ 2 2 + 202 + GoOO-25-Ю-в) Общая активная проводимость всей цепи g “ gi + g, - 0,008 + 0,025 = 0,033 —. 413
Реактивная проводимость первой ветви "L f 2000 0,03 nmfi_L 1 f2 + (<»Z,)2 80’ + (2000-0,ОЗ)2 ' ом" Реактивная проводимость второй ветви 2000-25. Ю-e 0|025Д. / 1 X2 о М 2000 -25-10-6J Общая реактивная проводимость всей цепи ь = bt — b2 = 0,006 — 0,025 = -0,019 —. Так как общая реактивная проводимость всей цепи имеет отрицатель- ный знак, то режим в цепи носит емкостный характер, т. е. ток 7 в нераз- ветвленном участке цепи опережает по фазе приложенное к цепи напря- жение (7. Полная проводимость всей цепи Y = у + Л2 = {/(0,033)* + (-0,019)2 = 0,038 —. Ток в неразветвленном участке цепи 7 = UY ~ 120-0,038 «4,56 а. Сдвиг фаз между током и напряжением в неразветвленном участке цепи , b -0,019 = —°-576- По тригонометрической таблице тангенсов находим ф . -30°. Активная мощность, расходуемая в цепи, ра = U2g « 120* - 0,033 = 475,2 вт. Полная проводимость первой ветви - V + b'i - V (0.008)2 + (0,006)2 „ 0,01 Ток в первой ветви Л « UYX - 120-0,01 « 1,2 а. Активная мощность, расходуемая в первой ветви, Pai " U2gt =- 1202-0,008 - 115,2 67П. Полная проводимость второй ветви У3 - у > , 62 =. у 0,0252 4- 0,02^2 - 0,0354 —. 2 ОМ 414
Ток во второй ветви /2 » Д/У2 =- 120-0,0354 =- 4,25 а. Активная мощность, расходуемая во второй ветви, Ра2 = U*g2 = 1202 • 0,025 = 360 вт. Сдвиг фаз тока и напряжения в первой ветви . bi 0,006 л 7- 07)08= °-75- откуда угол ?i = 36°50'. Сдвиг фаз тока и напряжения во второй ветви . Ь2 -0,025 1 tgf»= gj = 0,025 = 11 откуда угол <р8 ® —45®. § 118. РЕЗОНАНС ТОКОВ Резонансом токов называется такой режим в цепи перемен- ного тока с параллельно соединенными индуктивностью и емко- стью, при котором ток в неразветвленном участке цепи и напря- жение, приложенное к этой цепи, совпадают по фазе. Рассмотрим, при каких условиях может возникнуть резонанс токов, каковы характерные особенности этого явления и какое практическое значение оно имеет в электротехнике. Допустим, что задана цепь переменного тока, состоящая из катушки индуктивности и конденсатора, так что они вместе с активными сопротивлениями образуют замкнутый контур (рис. 256). Катушка индуктивности обладает активным сопро- тивлением Г1 и индуктивностью L, а конденсатор — активным сопротивлением г2 и емкостью С. Генератор, питающий током данный контур, имеет на своих зажимах синусоидальное напря- жение, действующее значение которого равно U и угловая ча- стота 0)0. Если в данной цепи возникает резонанс токов, значит, угол сдвига фаз между током /0 неразветвленного участка цепи и на- пряжением [7, приложенным к цепи, равен нулю, т, е. <р = 0. Следовательно, и тангенс этого угла должен быть также равен нулю, т. е. tg ? = 0. Подставив в последнее равенство вместо tg его выражение, согласно формуле (344) получим tg<p = ^T^=0’ (348) где gi и Ьх — соответствено активная и реактивная проводимости параллельной ветви с ийдуктивностью L и актив- ным сопротивлением гх; и *-соответственно активная и реактивная проводимо- сти параллельной ветви с емкостью С и активным сопротивлением г2. 415
Равенство (348) справедливо только в том случае, когда — bi = 0, т. е. когда = (349) так как в этом случае tg —-— = tg 0 = 0. gl + gt * Принимая во внимание равенство (349), можно сделать такой вывод: резонанс токов возникает в контуре в том случае, если реактивные составляющие проводимости параллельных ветвей с индуктивностью и емкостью численно равны друг другу. Помножив левую и правую части равенства (349) на дей- ствующее значение напряжения U, приложенного к цепи, полу- чим равенство иьх = иьъ или, иначе, Л1 = Л2, (350) где 1л = иьх—реактивная составляющая тока в параллельной' ветви с индуктивностью; 1гъ = иЬг—реактивная составляющая тока в параллельной ветви с емкостью. Из равенства (350) следует, что при резонансе токов реак- тивные составляющие токов параллельных ветвей с индуктивно- стью и емкостью равны друг другу. Как известно, ток в неразветвленном участке рассматривае- мой цепи в общем случае определяется по формуле (343) I = U • Принимая во внимание, что при резонансе токов — &2 = 0, получим /о = г/И(£14-£2)2 + О, или, иначе, /о = £/(&+ &) = Ugi + Ug2 = Ini 4- гаг, (351) где Ial = Ugx—активная составляющая тока в параллельной ветви с индуктивностью; Ia2 = Ug2—активная составляющая ветви с емкостью. Следовательно, ток в неразветвленном участке цепи при ре- зонансе токов равен сумме активных составляющих токов в па- раллельных ветвях. 416
Рис. 263. Векторная диаграмма токов и напряжения для случая резонанса токов На рис. 263 приведена векторная диаграмма токов и напря- жения для рассматриваемого случая резонанса токов. Здесь век- тор U приложенного к резонансному контуру напряжения на- правлен по начальной оси, так как начальная фаза его принята равной нулю. Вектор тока Л параллельной ветви с индуктив- ностью отстает по фазе на угол <pi от напряжения U, а вектор тока /2 параллель- ной ветви с емкостью опере- жает -по фазе на угол <р2 на- пряжение U. Реактивные со- ставляющие токов парал- лельных ветвей 1п и 1г2 рав- ны" по абсолютной величине, но находятся в противофазе. Вектор тока /о неразветвлен- ного участка цепи совпа- дает по фазе с напряже- нием U и равен по абсо- лютной величине сумме ак- тивных составляющих то: ков 1Ха и Ii0 параллельных ветвей. Теперь определим резонансную частоту, т. е. ту частоту, при которбй при заданных параметрах цепи L, г2 и С наступает резонанс токов. Для этой цели воспользуемся равенством (349) — ^2> или, иначе, 1 COqL <Л0С Отсюда после несложных преобразований получим или - ^LCrl + -^ = ^ + ^fJ, ИЛИ 27—1377 417
откуда находим, что или (352) где too — резонансная угловая частота. В соответствии с этим резонансная частота (353) Исследуем формулу (352) для определения резонансной угло- вой частоты о)0, рассмотрев несколько частных случаев. Первый случай. Допустим, что активные сопротивления параллельных ветвей с индуктивностью и емкостью равны друг другу: Тогда резонансная угловая частота (о0 равна т. е. в данном случае формула для определения резонансной угловой частоты о)0 совершенно сходна с формулой для опреде- ления резонансной частоты при резонансе напряжений. Второй случай. Допустим, соотношение параметров па- раллельной цепи таково, что выполняется следующее равен- ство: fl fl — fl — Г\ Г(2 Г С • "Тогда формула (352) принимает вид Г _ __1 1 / ______С _ 1 о _ о ш° ~ утё |/ Г2 _ — у~ьс ' 0 ~~ 0 ’ (355) Мы получили неопределенную величину. Это значит, что при заданных значениях параметров п, r2, L и С резонанс? токов в контуре наступит при любой частоте со. 418
Третий случай. Допустим, соотношение параметров па- раллельной цепи таково, что и г2^>^г- г2 —— ч 77 Тогда подкоренное выражение ---------в формуле (352) /1--с отрицательно, а следовательно, <оо будет величиной мнимой: “° VLC (356) где / = V — 1 — мнимая единица. Значит, при заданном соотношении параметров параллельной цепи- резонанс токов невозможен ни при какой частоте. Пример 140. В цепи, схема которой изображена на рис. 256, L — 0,01 гн и С = 100 мкф. Определить резонансную, частоту контура для следующих четырех вариантов: 1) и = 8 ом, г2 = 6 ом; 2) п = 8 ом, г2 = 8 ом; 3) Г1 = 10 ом, г2 = 10 ом; 4) Г1 = 12 ом, г2 = 3 ом. Решение. По формуле (352) находим: Первый вариант ТЛ~4 , -.Л-то^ уьс |/ г2 £ /0,01-100.10-е I/ 0,01 F 2 С г 100-10-6 1 0,001 = 1000-4- = 750 —. 8 сек Второй вариант 1 “°" VTZ Третий 1 (Ол = --- V LC /0,01-100-10-6 1. 0,001 = 1000 — . сек 82______ 100-10-6 82______ 100-10-6 вариант 1 /0,01; 100 «10-6 Ю-______ 100-10-6 102_____212L- 100-10-6 1 1 °, = 0 0,001 о “ о т. е. резонанс токов возможен при любой частоте, 271 419
Четвертый вариант <о0 = _______1 /0,01-100-10-6 122__ °»01 100-10-6 82_____2^- 100-10-6 1 /44 0,001 V 36 1105 /—1, т. е. резонанс токов невозможен ни при какой частоте. На практике чаще всего приходится иметь дело с резонанс- ными контурами, в которых активное сопротивление в парал- Рис. 264. Резонансный контур лельной ветви с емкостью рав- но нулю (Г2 = О), а активное сопротивление г± в параллель- ной ветви с индуктивностью значительно меньше реактив- ного сопротивления ее при резонансе токов, т. е. ri<Co)0L. Рассмотрим подобный кон- тур (рис. 264) и проанализи- руем явление резонанса токов в нем. Если в таком контуре ак- тивное сопротивление в парал- лельной ветви с индуктивностью значительно меньше индуктив- ного сопротивления ее при резонансе токов, т. е. ri<^(o0L, то с достаточной точностью можно считать, что реактивная проводи- мость этой ветви.при резонансе токов равна (O0Z/ ______&QL ______ 1 ] ~ r*+(u0Ly ~ (<o0L)2"”a)0L’ (357) Реактивная проводимость параллельной ветви в данном случае равна 1 1 с емкостью (358) так как по условию г2 = 0. Условием возникновения резонанса токов является равенство реактивных проводимостей параллельных ветвей контура, т. е. Ь1 = Ьъ ' или согласно формулам (357) и (358) 4z=“«c' ' <iK|j 420
Из равенства (359) определим резонансную угловую частоту: “°= уте ’ Следовательно, если активное сопротивление параллельной ветви с индуктивностью значительно меньше реактивного сопро- тивления ее, а активное сопротивление ветви с конденсатором практически равно нулю, то угловая резонансная частота кон- тура определяется формулой, совершенно сходной с формулой для определения резонансной угловой частоты при резонансе на- пряжений. Это значит, что в данном случае резонансная угловая частота контура равняется угловой частоте собственных незатухающих колебаний его.- Чем меньше активное сопротивление ri параллельной ветви с индуктивностью по сравнению с ее индуктивным сопротивле- нием q)0L при резонансе токов, тем более точной становится формула (360). Если, например, ri меньше ®qL в десятки раз, то погрешность расчета угловой резонансной частоты шо по фор- муле (360) составит менее одного процента. Так как по условию ri<Oo£, то и активная составляющая проводимости gt ветви с индуктивностью будет значительно меньше реактивной составляющей Ь± этой ветви. Ведь если Л С то , 1 << , 0 ..8 , ч + (<*>о02 ч + (ш(Л)2 или, иначе, gi^bi. А если это так, то и активная составляющая тока /а1 в параллельной ветви с индуктивностью должна быть значительно меньше реактивной составляющей тока /н в этой ветви, поскольку при или, иначе, Лг Следовательно, при величину тока 71 в параллель- ной ветви с индуктивностью с достаточной точностью можно считать равной его реактивной составляющей /н, т. е. А так как реактивные составляющие токов в параллельных ветвях с индуктивностью- и емкостью при резонансе токов равны, то в нашем случае, когда и Гг — 0, токи А и /2 в этих ветвях практически равны друг другу: А ® А- 421
Как уже упоминалось, ток 70 в неразветвленном участке цепи при резонансе токов равен сумме активных составляющих токов параллельных ветвей, т. е. А?1 + Лг2 4 Но в данном случае активная составляющая тока в парал- лельной ветви с емкостью равна нулю, поскольку Ia. = Ug2=U = и----А-тт=0. 4+ (Л. °+(-7Ч \шоС/ \<о0С/ Следовательно, ток в неразветвленном участке цепи будет равен активной составляющей тока в параллельной ветви с индуктив- ностью, т. е. J — Г — JJа — « [J h “ Ugl ~ + (V)2 U (шоЛ)2 > так как и, следовательно, & = Но активная составляющая тока 1а* в параллельной ветви значительно меньше всего тока Л этой ветви; поэтому ток в не- разветвленном участке цепи при резонансе токов значительно меньше токов параллельных ветвей с индуктивностью и емко- стью, если и г2 = 0, т. е. 2- Рис. 265. Векторная диа- грамма для случая ре- зонанса токов По той причине, что при резонансе токов реактивные состав- ляющие токов параллельных ветвей с индуктивностью и емко- стью взаимно компенсируются, а токи в этих ветвях обычно значительно больше тока в неразветвленном участке цепи, этот электрический резонанс и назван резонан- сом токов. На рис. 265 приведена векторная диа- грамма для рассматриваемого случая, когда в резонансном контуре fi<CoJoL и /*2 = 0. Пример 141. Цепь составлена из параллельно соединенных катушки индуктивности, обладаю- щей индуктивностью L = 5 • 10~4 гн и актив- ным сопротивлением г\ = 5 ом, а также конден- сатора, обладающего емкостью С = 2 • 10“9 ф и активным сопротивлением, практически равным нулю (рис. 264). Требуется определить режим при резонансе токов в дан- ной цепи, если к ней приложено синусоидальное напряжение, действующее значение которого U = 120 в. Решение. Определяем резонансную угловую частоту по формуле (360) Фо -----— = < 1 -10» — . YLG / 5-10-4-2-10~9 се>< 422
Индуктивное сопротивление катушки индуктивности при резонансе токов равно соо£ = 106«5* 10~4 = 500 ом, т. е. оно в сто раз больше актив- ного сопротивления г\ = 5 ом катушки, а поэтому мы были вправе приме- нить формулу (360) для определения резонансной угловой частоты соо. Реактивная ^проводимость параллельной ветви с индуктивностью со- гласно формуле (357) равна bi = ^1 = 106.5-10—4 = 0,002 "ом‘ Активная составляющая проводимости в параллельной ветви с индук- тивностью согласно неравенству г\ ыоЬ равна г 5 5 1 gi = (о>0£)2 ” (106-5-10—4)2 = 25-104 = °’00002 • Реактивная составляющая тока в параллельной ветви с индуктивностью равна /Г1 = = 120-0,002 = 0,24 а. Активная составляющая тока в параллельной ветви с индуктивностью равна /Я1 = Ugi = 120-0,00002 = 0,0024 а, т. е. она меньше реактивной составляющей тока /Г1 в сто раз. Реактивная составляющая тока в параллельной ветви с емкостью равна /г2 = Ub2 = U&C2 = 120-106-2-10—э = о,24 а, т. е. она равна реактивной составляющей тока в параллельной ветви с ин- дуктивностью, что и следовало ожидать при резонансе токов. Активная составляющая тока в параллельной ветви с емкостью равна нулю, так как ^2=^2 = 77-0 = 0. Ток 1\ в параллельной ветви с индуктивностью равен А = V = V 0.00242 + 0,242 я 0,24 а, т. е. он практически равен своей реактивней составляющей. Ток /г в параллельной ветви с емкостью равен А = Vх ^2 + zr2 = ]/о + о,242 = 0,24 а. Следовательно, в рассматриваемом случае токи в параллельных ветвях при резонансе токов практически равны: /1 = 72 = 0,2,4 а. Ток в неразветвленном участке цепи /о равен активной составляющей тока в параллельной ветви с индуктивностью, так как активная составляю- щая тока в параллельной ветви с емкостью равна нулю: /0 = /Д1 « 0,0024 а, т. е. ток /о в неразветвленном участке цепи в данном случае в сто раз меньше тока в каждой из параллельных ветвей резонансного контура. ъ- 423
Теперь определим полное сопротивление резонансного кон- тура при резонансе токов по отношению к генератору, питаю- щему этот контур энергией. Для этой цели используем формулу == Лп== отсюда находим, что ^рез /о gi т. е. резонансное сопротивление контура по отношению к генера- тору при 1\ <С и г2 = 0 является величиной, обратной активной проводимости параллельной ветви с индуктивностью. Подставив вместо gi его выражение в последнее равенство, получим , = J 1 ~ 1 __ (“(Л)2 ре3 Si п п п г? + (ш0£)2 (<°о^)2 так как гт<Со)0Ь. В рассматриваемом случае а поэтому последнему равенству можем придать такой вид: или, иначе, (361) где Zpe3— резонансное сопротивление контура в омах; L— индуктивность контура в генри; С— емкость контура в фарадах; т\— активное сопротивление контура в омах. Исходя из.формулы (361), можно сделать следующий вывод: резонансное сопротивление контура по отношению к генератору при гт <О0£ и г2 = 0 ^обратно пропорционально активному со- противлению Г1 параллельной ветви с индуктивностью, т. е. чем меньше это активное сопротивление, тем больше резонансное сопротивление контура. В пределе, когда активное сопротивление Г1 контура равно нулю, резонансное сопротивление Zpe3 контура становится рав- ным бесконечности и в соответствии с этим ток /0 в неразвет- 424
вленном участке цепи становится равным нулю. В этом идеаль- ном случае электромагнитные колебания в контуре будут неза- висимы от источника электрической • энергии, т. е. в контуре будут существовать собственные незатухающие колебания. ч Чем больше активное сопротивление резонансного контура,, тем больше в нем потерь энергии на. необратимые процессы (выделение тепла в активном сопротивлении) за каждый пе- риод, тем, следовательно, больше энергии должен доставлять источник в контур, чтобы поддержать в нем вынужденные неза- тухающие колебания. Поэтому генератор с ростом активного со- противления контура должен посылать больший ток, что и осу- ществляется в результате уменьшения резонансного сопротивле- ния при возрастании активного сопротивления. Пример 142. Резонансный контур имеет индуктивность £ = 8*10~4 гн, емкость С =12,5*10-8 ф и активное сопротивление и = 2 ом. Требуется определить резонансное сопротивление Zpe3 контура и величину тока /0 в не- разветвленном участке цепи, если действующее значение напряжения, прило- женного к цепи, U = 80 в. » Решение. Резонансное сопротивление контура согласно формуле (361) равно 7 Я 10—4 ZPe3 = ^-2.182,5°i0-e=3200 Q" Ток в неразветвленном участке цепи равен U zpe3 80 3200 = 0,025 а. Так как по условию и в соответствии с этим резо- нансная угловая частота (о0 = 1 1/Тс’ то индуктивное сопротивле- ние параллельной ветви контура с индуктивностью для случая резонанса токов можно выразить следующей формулой: xL^~^L-y=L, или, иначе, xlo — L С • (362) Аналогично этому емкостное сопротивление параллельной ветви контура с емкостью для случая резонанса токов можно выразить формулой х _J_______1 *с0~а>оС~ 1 > удес z 425
или, иначе, (363) I Следовательно, если в резонансном контуре 1\ <С aj0L, то индуктивное и емкостное сопротивления параллельных ветвей контура при резонансе токов численно равны друг другу и равны порознь волновому сопротивлению контура Р = (364) где р — волновое сопротивление резонансного контура в омах; L — индуктивность в генри; С — емкость в фарадах. Выразим вышеприведенные основные формулы, характери- зующие режим в резонансном контуре при резонансе токов, через волновое сопротивление. Резонансное сопротивление контура согласно формуле (361) равйо = = (365) Ток равен в неразветвленном участке цепи при резонансе токов = = £ = (366) г Ток равен в параллельной ветви с индуктивностью, где (o0L, = (367) Ток в параллельной ветви с емкостью равен (368) (D0C Разделив величину тока /х на величину тока /о, получим h_=U_ • _g_ 4 р ' р2 Л ' Аналогично этому, разделив тока Го, получим величину тока /2 на величину 4_ = _t£ . h Р ’ Р2 П ’ 426
Следовательно, т,о к и в параллельных ветвях резо- нансного контура Д'и I2 при резонансе токов порознь во столько раз больше тока /о нераз- ветвленного участка цепи, во сколько раз вол- новое сопротивление контура р больше его активного сопротивленият. Но, как известно, отношение волнового сопротивления р кон- тура к его активному сопротивлению ri— это добротность (ка- чество) контура «=^-> где Q— добротность (качество) контура. Следовательно, добротность Q контура — это величина, пока- зывающая, во сколько раз ток в резонансном контуре больше тока в неразветвленном участке цепи при резонансе токов: = (369) z0 *0 Величина, обратная добротности (качеств^ резонансного контура, называется затуханием контура (d); или, иначе, а h 4 ’ (370) т. е. затухание резонансного контура — это величина, показываю- щая, какую долю тока контура составляет ток в неразветвлен- ном участке цепи при резонансе токов. Пример 143. Резонансный контур составлен из катушки индуктивности, обладающей индуктивностью L — 3 • 10—6 гн и активным сопротивлением r\ = 1 ом, и параллельно соединенного с катушкой конденсатора, имеющего емкость С = 3 • Ю~9 ф и активное сопротивление г2 = 0. Определить вол- новое сопротивление, добротность, затухание контура, а также ток в нераз- ветвленно-м участке цепи и ток в контуре, если действующее значение при- ложенного к данной цепи синусоидального напряжения U = 20 в. Решение. Волновое сопротивление контура Р 3-10-5 3-10-9 = 100 ом. Добротность контура j-= 100. Затухание контура ж = о.«|- 427
Ток в неразветвленном участке цепи согласно формуле (366) равен /0 = L/-^ = 20--1^ = 0,002 а. - Ток в колебательном контуре Л « Z2 = Q1q = 100-0,002 « 0,2 а. Так как резонансный контур для генератора электрической энергии представляет нагрузочное сопротивление чисто активного характера, то активная мощность, развиваемая им в резонансном контуре, может быть определена по формуле Ра = UIQ = 20-0,002 = 0,04 вт. Реактивная мощность, развиваемая в каждой из параллельных ветвей контура, равна Pr = Uh к 1Л2 = 20-0,2 - 4 ва. Таким образом, в рассматриваемом случае, когда и < реактивная мощность в контуре значительно больше активной. Пример 144. Резонансный контур обладает волновым сопротивлением р = 160 ом и активным, сопротивлением и = 2 ом. Требуется определить активную и реактивную мощность в контуре при резонансе токов, если дей- ствующее значение напряжения, приложенного к цепи, U = 127 в. Решение. Резонансное сопротивление контура о2 1602 Zpe3 = 2r = 2^ = 12800 ом. i Ток в неразветвленном участке цепи г V 127 Z° “ Zpe3 " 12800 ~ 0,01 й‘ Добротность контура ^- = 8°. и 2 Ток в резонансном контуре Л ж Z2 = QZ0 = 80-0,01 = 0,8 а. Активная мощность, развиваемая генератором в резонансном контуре, равна ра =± UIQ = 127-0,01 = 1,27 вт. Реактивная мощность в контуре Рг=иЦ = 127-0,8 = 102 ва, т. е. она больше активной мощности в 80 раз. В отличие от резонанса напряжений при резонансе токов сумма энергий электрического и магнитного полей контура не яв- ляется постоянной величиной. Это значит, что имеются такие Промежутки времени, когда энергия электрического и магнитного полей контура расходуется частично или полностью в активном сопротивлении контура, и, следовательно, имеются также такие 428
промежутки времени, когда генератор электрической энергйи по- полняет запас энергии, израсходованной электрическим и маг- нитным полями контура. Однако обмена реактивной энергией между генератором электрической энергии и магнитным полем контура здесь не происходит. Генератор в общей" сложности за период доставляет в контур такое количество энергии, которое было израсходовано в активном сопротивлении контура. В частном случае, когда активными сопротивлениями контура можно пренебречь, в контуре совершается обмен энергией между электрическим и магнитным полями. При этом сумма энергий электрического и магнитного полей сохраняется для любого мо- мента времени постоянной по величине, а максимальные энергии этих полей численно равны друг другу: Lit cut WKm = ^ = ^=W3m, (371) где Wwm—максимальная магнитная энергия; UZ9 т— максимальная электрическая энергия. Весьма большое значение для практики имеют резонансные кривые токов, отображающие графически зависимость токов в цепи от частоты вынуж- денных колебаний. На рис. 266 приведены графики зависимости то- ка / неразветвленного участка цепи, тока А па- раллельной ветви с ин- дуктивностью и тока А па- раллельной ветви с емко- стью от частоты выну- жденных колебаний в кон- туре для случая, когда Рис. 266. Графики изменения токов Л, А и I при изменении угловой частоты о> активное сопротивление катушки относительно ма- ло, а активное сопротив- ление ветви конденсатора равно нулю. Здесь график тока А изображен гиперболой, поскольку ток А изменяется практически обратно пропорционально частоте. При со = О А = -уг, а при со = оо А = 0. График зависимости тока А от частоты изображен прямой линией, так как ток А изменяется пропорционально ча- стоте. При со = О А = 0, а прщсо->оо /2-> оо. График зависи- мости тока / от частоты (резонансная кривая) изображен в виде U-образной кривой. При (о=0 /==—> а при со-> о^ 7->оо; при СО = (О о = 1 VTC , где coo — резонансная частота, ток I имеет минимальную величину. Чем меньше активное сопротивление 429
Рис. 267. Графики изменения токов Zn /2 и 1 при изменении емкости от нуля до бесконеч- ности Q ............... » Q) Рис. 268. Резонансные кривые для контуров с различными потерями энергии: чем больше активное сопротивление кон- тура, тем более пологая кри- вая контура, тем меньше ток I при резонансе токов, и в идеальном случае, когда гх; = 0, кривая тока I будет в точке w = (о0 сопри- касаться с осью О). > На рис. 267 представлены графики зависимости токов Л Л и Z2 резонансной цепи от емкости контура. Здесь график тока Л (ток ветви с индуктивностью) изображен прямой линией, парал- лельной оси емкости С, так как Л = const, т. е. не зависит от емкости С при постоянстве действующего значения синусоидаль- ного напряжения, приложенного к цепи. График тока Л (ток в ветви конденсатора) изображен также прямой линией, так как этот ток изменяется пропорционально емкости (l2=U(^C), График тока / (резонансная кривая) изображен в виде U-об- разной кривой. При со = 0 /== у-, при w->oo I -> оо и при резонансной емкости Со==~^-£- ток Z имеет минимальную ве- личину. Если при всех прочих равных условиях изменять активное сопротивление контура, то резонансная кривая этого контура также будет изменяться: при увеличении сопротивления она станет более пологой, а при уменьшении его — более остроконеч- ной. На рис. 268 приведено несколько резонансных кривых тока для контуров с различными активными сопротивлениями. Явление резонанса токов широко используется в радиотехни- ческих цепях. Здесь резонансный контур обычно питается от генератора электрической энергии, обладающего весьма большим внутренним сопротивлением, например, от генераторной или уси- лительной электронной лампы. Если внутреннее сопротивление генератора, питающего резонансный контур, значительно больше резонансного сопротивления контура, то последний приобретает резко выраженные резонансные свойства. Допустим, что внут- реннее сопротивление генератора Rt постоянно независимо от частоты электрических колебаний в цепи, т. е. /?z = const, и при этом оно значительно больше сопротивления контура Z, т. е. 430
Rt^>Z. В соответствии с этим ток I в неразветвленном участке цепи сохраняется практически неизменным по величине при из- менении частоты электрических колебаний в контуре, так как электродвижущая сила генератора не зависит от частоты элек- трических колебаний. Следовательно, Е Е / = 7?7+z~K7 = const- где I—ток в неразветвленном участке цепи; Е— электродвижущая сила генератора; Z— сопротивление колебательного контура; /?z— внутреннее сопротивление генератора. Если изменять частоту электрических колебаний в контуре, то сопротивление контура начнет резко изменяться, достигая макси- мума при резонансе токов. Но поскольку ток неразветвленного участка цепи I = const, то с изменением сопротивления колеба- тельного контура будет соответственно изменяться и напряжение на зажимах контура U = IZ, + гд£~и—напряжение на зажиме контура; I — ток в неразветвленном участке цепи; Z — сопротивление контура. А так как сопротивление контура при резонансе токов (резо- нансное сопротивление) относительно великог то велико и напря- жение на контуре. В соответствии с этим становятся большими и токи в параллельных ветвях резонансного контура, потому что Л ==: ^рез^1 ^2 == ^рез^2> где t7pe3напряжение на контуре при резонансе токов. На рис. 269 изображены графики изменения тока I в нераз- ветвленном участке цепи и напряжения U на резо- нансном контуре при из- менении частоты f колеба- ний в контуре. Следовательно, радио- технические контуры резко увеличивают свое сопро- тивление при резонансе токов. В соответствии с этим резко усиливается напряжение на контуре и значительно увеличива- ются токи в параллельных ветвях, в то время как ток в неразветвленном участке цепи практически тока / в неразветвленн2М участке цепи от частоты f 43J
сохраняет свое действующее значение неизменным.-Резкое же уве- личение мощности электромагнитных колебаний в резонансном контуре используется для усиления электромагнитных колебаний. Резонанс токов важную роль играет и в сильноточной тех- нике. Известно, что активная мощность в цепи переменного тока определяется по формуле Pc = 67-cos% где cos <р — коэффициент мощности. Если данная мощность передается при cos?=l, т. е. при ср = 0, то ток в цепи равен Если же мощность Ра передается, например, при cos ср == 0,5, то ток в цепи равен /==_^__=2— т. е. он по сравнению с первым случаем увеличился в два раза. Итак, чем меньше коэффициент мощности cos ср, тем при всех прочих равных условиях больше ток в цепи, питающий при- емники электрической энергии. С ростом же тока в цепи растут непроизводительные тепловые потери в ней. Помимо этого, чем меньше cos ср в цепи, тем большая доля тока приходится на его реактивную составляющую, обусловленную реактивной энергией, которой периодически обменивается генератор с потребителями. При резонансе же токов генератор совершенно разгружается от реактивной составляющей тока, необходимой, например, для катушки индуктивности, так как он получается за счет реактив- ной энергии, которой обмениваются магнитное поле катушки и электрическое поле конденсатора. На практике стремятс^ как можно больше увеличить cos ср. Для этой цели к потребителям, требующим реактивного тока индуктивного характера, подключают параллельно необходимую емкость такой величины, чтобы она смогла своим реактивным током скомпенсировать реактивную составляющую тока в ветви с индуктивностью. Если по тем или иным причинам не удается добиться полного резонанса токов, то все же этим способом можно значительно улучшить коэффициент мощности цепи.
ГЛАВА XXIV ПРИМЕНЕНИЕ МЕТОДА КОМПЛЕКСНЫХ ЧИСЕЛ В ТЕОРИИ ПЕРЕМЕННЫХ ТОКОВ § 119. ОБЩИЕ ЗАМЕЧАНИЯ Несложные задачи по переменному току можно решать с большой точ- ностью методом проводимости. Но при решении сложных задач этот метод нерационален, так как требует слишком много времени на вы- числения. Графический метод решения задач прост, не требует много времени, но не всегда дает необходимую точность решений. В теории переменных токов при решении сложных задач успешно при- меняется метод комплексных чисел. Удобство этого метода заключается в том, что при нем геометрические действия над векторами заменяются алгебраическими действиями над ком- плексными числами, соответствующими этим векторам. Заменяя векторы на- । । । । I । । । । пряжения, тока и т. д. комплексными п «н +о +? + числами, удается решать задачи на 4 u I j о ч сложные цепи по формулам, аналогич- ным тем, которые применяются при Рис. 270. Числовая ось относи- решении задач на сложные цепи по- тельных чисел стоя иного тока. В алгебре мы имеем дело с так называемыми относительными числами, т. е. с числами положительными и отрицательными. Всякое алгебраическое число характеризуется абсолютной величиной и знаком, например, температура воздуха ^=+10°Ц, /2=“"5ОЦ, ток h = +5 а, «2 = —3 а, напряжение и\ = 4-100 в, и2 =—60 в и т. д. Всякое алгебраическое число можно изобразить графически в виде отрезка прямой линии, отложенного в некотором масштабе на числовой оси от начальной точки в ту или иную сторону в зависимости от знака этого числа. На рис. 270 показана прямая линия (числовая ось), на кото- рой отмечена начальная точка 0 (начало отсчета чисел). Вправо от этой точки 0 в некотором масштабе отложены положительные алгебраические числа, а влево — отрицательные. Изображая алгебраическое число направ- ленным вдоль числовой оси отрезком прямой, мы ему в известной степени приписываем свойства вектора. Поэтому действия над алгебраическими чи- слами можно заменить действиями над векторами, изображающими их. На рис. 271 показан пример сложения двух алгебраических чисел гра- фическим методом. При сложении двух векторов, изображающих алгебраиче- ские числа, надо к концу первого слагаемого вектора приложить начало второго вектора, сохранив направление последнего неизменным. Сумма 28-1377 433
двух векторов определится величиной отрезка, имеющего начало в точке О и конец, совпадающий с концом второго слагаемого вектора. Вычитание векторов есть действие, обратное их сложению. Вы- честь из одного вектора другой — это значит к первому из них прибавить второй, но с обратным знаком (направлением). На рис. 272 показан пример вычитания двух алгебраических чисел графическим методом. Числа арифметические и обычные алгебраические называются числами действительными, или вещественными, если они дают пред- ставление о реальном количестве единиц тех или иных величин, например 5 л/, 3 сек., 2 а, просто число 8 и т. д. А = 6 о 5=4 Д + 0 = 70 .. ° *" : 0-----------------;------•- С'-10 О-------- Рис. 271. Графический метод сложения двух алгебраиче- ских чисел Рис. 272. Графический метод вычитания двух алгебраи- ческих чисел § 120. МНИМОЕ ЧИСЛО Корень квадратный из любого числа есть такое число, которое, будучи помножено само на себя, должно дать произведение, равное подкоренному числу. Но, как известно, нет такого числа, которое было бы помножено само на себя издало бы произведение с отрицательным знаком, а поэтому извлечь квадратный корень из отрицательного числа невозможно. Преобразуем алгебраическое выражение —9 следующим образом: ]/=9 = /9НП = 3 Алгебраическое выражение J/ —1 принято называть мнимой еди- ницей и обозначать символом i или /. В дальнейшем будем пользоваться символом /, оставив символ I для обозначения мгновенного значения пере- менного тока. Итак, (372) Тде / — мнимая единица. Следовательно, /=9 = 3 У~\ = ЗД т. е. выражение У—9 равно трем мнимым единицам. Всякое число, состоящее из совокупности мнимых единиц, называется мнимым числом, например 2/, щ и т. д., где п — вещественное число. Мнимое число в отличие от вещественного не имеет физического смысла, и его надо рассматривать как подсобную математическую величину. 434
Мнимое число есть число алгебраическое, т. е, оно может быть поло- жительным или/отрицательным, например: /=25 = 5/ и —J/^Лб = —4 У~\ = —4/ Над мнимым числом можно производить алгебраические действия, например: 5/4-3/= 8/; 5/-3; = 2у; 4;-(-1У') = 5/. JA j2A=~A А "I 11 О Рис. 273. Вектор, соответ- ствующий данному числу, при умножении на /2 пово- рачивается против часовой стрелки на 180°, не меняя своей абсолютной величины А О Рис. 274. Вектор, умноженный на /’, поворачивается против часовой стрелки на 90° Возведение в степень мнимой единицы приводит к следующим резуль- татам: У2 = J j = • Г=1 = -1; / = /7 = -М “ / = J*J —JJ = —J2 = +1; J° = /У = + М = i и т. д., При возведении в квадрат мнимой единицы получается вещественная отрицательная единица. Следовательно, при умножении вещественного (действительного) числа на /2 мы меняем знак этого числа на обратный, например: З/2 = —3; —5/2 =; +5 и т. д. Значит, вектор, соответствующий этому числу, при умножении его на р поворачивается на 180° (рис. 273), не меняя своей абсолютной величины. V 4 3 2 ?• О 432 / ;. / 2 3 4 ’ “7 Рис. 276. Графиче- ский метод сло- жения мнимых чи- сел 2 3 4 Рис. 275. Оси веществен- ных и мнимых величин в комплексной числовой плоскости 28* 435
Умножить вектор на j2 — это значит повернуть его на 180°. В соответ- ствии с этим условно принято считать, что при умножении вектора на j он поворачивается против движения часовой стрелки на 90° (рис. 274). При умножении вектора на р он поворачивается против часовой стрелки на угол 180°, при умножении на р — на угол 270° и т. д. При умножении вектора на —/ вектор поворачивается на угол —90°, что равносильно умножению вектора на р = —/, т. е. повороту его против движения часовой стрелки на угол 270°. \ Для того чтобы графически ..представить мнимые числа, их изображают в виде отрезков прямой (векторов), наносимых на ось мнимых чисел, которая, как это вытекает из предыдущих соображений, должна быть распо- ложена перпендикулярно оси действительных величин (рис. 275). Плоскость с нанесенными на ней осями действительных и мнимых чисел называется комплексной числовой плоскостью. Над векторами, изображающими мнимые числа, можно совершать дей- ствия геометрического сложения и вычитания, как и над векторами, изобра- жающими вещественные величины (рис. 276). > § 121. КОМПЛЕКСНОЕ ЧИСЛО Число, состоящее из действительных и мнимых единиц, называется ком- плексным числом, например: 3 4- j • Д; cl — jb, где 3 и а— действительные числа; 4 и b— действительные числа, являющиеся коэффициентами при мнимой единице; J— мнимая единица. Комплексное число иногда сокращенно называется комплексом. Этим названием мы будем пользоваться в дальнейшем. Всякому комплексу на комплексной числовой плоскости соответствует некоторый вектор. Рассмотрим это положение на конкретном примере. Пусть требуется представить в виде вектора комплекс Z = 3 4-/4, где Z—комплексное число; 3— действительное число; /4— мнимое число. Нанесем на ось действительных чисел заданное в комплексе действи- тельное число 3 (рис. 277) и на ось мнимых чисел — мнимое число / • 4. Оба эти числа в сумме дают заданный комплекс Z. Следовательно, если мы сложим геометрически векторы, соответствующие действительному числу 3 и мнимому числу j-4, то должны получить результирующий вектор Z, соот- ветствующий заданному комплексу. Численное значение комплекса называется его модулем. Модуль комплекса легко определить геометрически. Вектор, соответствующий данному комплексу, является геометрической суммой двух векторов слагаемых, расположенных под прямым углом друг к другу, а поэтому его можно определить по теореме Пифагора. На- пример, длй комплекса А — а + jb (рис. 278) его модуль |Л| определим пе формуле \А] = У а2 4- ‘(373) т. е. модуль комплекса равен корню квадратному из суммы квадратов дей- ствительного числа комплекса и коэффициента при его мнимом числе. 436
Пример 145. Определить модули комплексов 4 = 0,8 + у' 0,6 и В = —12 + (—у-9). Решение. По формуле (373) находим | Л| = /0,82 + 0,62 = 1,0; |В| = /122 + 92 = 15. Угол а, составленный вектором А с положительной полуосью действи- тельных величин, называется аргументом комплекса (рис. 278). Положительные аргументы комплекса надо откладывать против движения часовой стрелки от положительной полуоси действительных чисел, а отрица- тельные аргументы — в направлении вращения часовой стрелки. Рис. 277. Вектор, соответ- ствующий комплексу Z = 34- + /•4 Аргумент а комплекса А — а + jb гевсу: tg Рис. 278. Модуль комплекса А = а 4- ,jb равен | А | = = ]А2 + £2 278) можно определить по тан- где а — действительная часть комплекса; b — коэффициент при мнимой части комплекса; а — аргумент комплекса. Пример 146. Определить модуль и аргумент комплекса А = 8 + /*6. Решение. Модуль комплекса |Л| = ]/82 4- 62 = 10. Аргумент комплекса находим по тангенсу: tg а = 1 = 0,75, о откуда по таблице для тангенсов находим, что а = 36°50'. 437
Пример 147. Определить модуль и аргумент комплекса Л=—3 —у-З. Решение. Модуль комплекса 1 Л] = ]/32"+32 = 4,23. Для определения аргумента надо вначале найти, где в комплексной пло- скости будет расположен вектор, соответствующий комплексу А. Судя по действительной части комплекса (—3) и по коэффициенту при мнимой части его (—3), вектор А должен быть расположен в третьем квадранте, т. е. угол а лежит в пределах от 180 до 270°. Аргумент определяем по тангенсу: Этому тангенсу (tg а = 1) соответствуют два угла* ф = 45° и ф = 180° + + 45° = 225°. Первый из них для нас не подходит по соображениям, высказанным выше, а поэтому принимаем ф = 225°. Выражение комплекса вида А = а + jb называется алгебраической формой изображения его. Если задан модуль и аргумент комплекса, то легко найти действительную и мнимую части комплекса и его алгебраическую форму. Из прямоугольного треугольника (см. рис. 278) а = |Л| • cos а; b = |Л| • sin а. (374) Следовательно, комплекс А = а + jb можно представить в следующем виде: А — а + jb = |AJ-cosa + у.]А| «sin а. (375) Форма комплекса, записанная формулой (375), называется тригоно- метрической формой изображения его. Пример 148. Модуль комплекса |Л| = 12 и аргумент его а = 20°30'. Определить действительную и мнимую части комплекса и представить его в тригонометрической и алгебраической формах. Решение. Тригонометрическая форма комплекса А = 12-cos20°30' + у-12-sin 20°30'. Действительная часть комплекса а = 12-cos20°30' = 12.0,94 = 11,28. Коэффициент при мнимой части комплекса b = 12-sin20°30' = 12-0,35 = 4,2. Алгебраическая форма комплекса Л = 11,28+ у-4,2. Помимо рассмотренных выше алгебраической и тригонометрической форм комплексных чисел, применяется еще показательная форма их. Комплексное число в показательной форме выражается произведением модуля комплекса на множитель е;а: Л = )Л| е* , (376) 438
где А — комплексное число; |А| — модуль комплексного числа; а — аргумент комплекса; j—мнимая единица; е=2,718—основание натуральных логарифмов. Комплекс, выраженный в показательной форме, можно преобразовать в тригонометрическую форму: А = |Л| е;а = Hl-cosa + j Hl-sin a. (377) Если аргумент комплекса отрицателен, то А = |Л| е—= |Л| • cos a — у |Л| • sin a. Пример 149. Задан комплекс Л = 20 е7*е45°- Требуется представить его в тригонометрической и алгебраической формах. Решение. По формуле (377) находим А - 20е;'45° = 20.cos 45° + ;-20-sin45° = 14,14 + j-14,14. Пример 150. Комплекс Л = 30 — j • 40 представить в показательной форме. Р е ш е н и е. Модуль комплекса |Л| /302+ 402 = so, а аргумент комплекса —40 a - аге tg = аге tg (—1,333) == -53° 10'. OU Следовательно, А = 30 —;-40 = 5Ое-'-53°10'. . ТС , - / V Пример 151. Выразить комплекс Л = 20е в тригонометрической и алгебраической формах. .ТС Л = 20е 2=20-cos-y—у • 20» sin-у в ““./•20. § 122. ДЕЙСТВИЯ НАД КОМПЛЕКСНЫМИ ЧИСЛАМИ Комплексные числа — это числа алгебраические, а поэтому, над ними можно производить все алгебраические действия. Два комплексных числа равны друг другу, если их действительные и мнимые части порознь равны друг другу. Если, например, а = с и b = d, то а + jb = с + jd, 1. Сложение комплексов При сложений комплексов надо сложить отдельно их действительные и мнимые части по правилу алгебраического сложения: (а + jb) + (с + Jd) = (а + с)+ j(b + d). (378) Если комплексы изображены графически в комплексной плоскости при помощи векторов, то сумме комплексов буд1?г соответствовать вектор, являю- щийся геометрической суммой векторов составляющих комплексов. 439
Пример 152. Найти сумму, модуль и аргумент комплексов А = 4 + j-4 и В = 5 4- j-& Решение. Сумма комплексов С = А + В == 4 + у’-4 + 5 + у’-8 = 9 + J-12; |С| = J/92 + 122 = 15. Аргумент суммы комплексов находим по тангенсу: 19 1’333> у откуда а = 53° 10'. 2. Вычитание комплексов Вычесть из одного комплекса другой — это значит к первому комплексу прибавить второй с обратным знаком: (а + jb) - (с + jd) = (а + jb) + (-с - Jd) = (а - с) + J (b - d), (379) т. е. при вычитании из одного комплекса другого надо из действительной части первого комплекса вычесть действительную часть второго комплекса, и полученная разность будет действительной'частью разности комплексов, а затем из мнимой части первого комплекса вычесть мнимую часть второго комплекса, и полученная разность будет мнимой частью разности комплексов. При вычитании векторов, соответствующих заданным комплексам, надо к вектору, изображающему комплекс-уменьшаемое, прибавить вектор ком- плекса-вычитаемого, но повернутый на 180°, и затем эти векторы сложить. Сумма этих векторов определит собой искомую разность комплексов. Пример 153. Найти разность, модуль и аргумент комплексов А = — 6 + 7-4 и В = 2 + у*-6. Решение. Найдем разность комплексов в алгебраической форме: С = А - В = (-6 + у‘-4) - (2 + у-6) = (-6 + 7-4) + (-2 -у’ -6) = -8 - J - 2. Модуль разности комплексов * |С| = /8М3 * * * 722 = 8,24. Аргумент разности комплексов находим по тангенсу. —9 tg«------н- = 0,25. --о Судя по знакам действительной и мнимых частей комплекса, вектор на- ходится в третьем квадранте, а поэтому угол ср = 180° + 14° 10'= 194° 10'. 3. Умножение комплексов При умножении комплексов они перемножаются согласно правилам умно- жения алгебраических многочленов, т. е. при умножении одного комплекса на другой надо все члены первого комплекса поочередно • помножить на все члены второго комплекса, а затем сделать приведение подобных членов: С = (а + Jb) (с + Jd) = ас + jad + jbc + j2bd = ас + j (ad + be) — bd = = (ac — bd) + 7 (ad + be). (380) 440
Модуль произведения комплексов | С | == V(ac — bd)2 + (ad + be)2. (381) Аргумент произведения определяется по тангенсу: Пример 154. Определить произведение комплексов А = 2 + j • 4 и В = 3 — j • 2. Решение. Находим произведение, перемножая комплексы по правилу умножения многочленов: С = ЛВ= (2+7-4) (3 —у-2) = 6 —/4 + ;• 12 — у2-8 = 6 + у.8 + 8= 14 + J-8. Модуль произведения комплексов 1С| = J/142 + 82 = 16,1. Аргумент произведения комплексов находим по тангенсу: о tg« = ^- = 0,571, откуда а = 29°50'. Произведение двух комплексов, выраженных в показательной форме, есть комплекс, модуль которого равен произведению модулей сомножителей, а ар- гумент— алгебраической сумме аргументов перемножаемых комплексов: С = АВ= |Л|е7’ |В|е^ = ]Д|.|В| = |С(е'?, (383) где |С| = |Л|.|В| и т = а + {3. Пример 155. Определить произведение комплексов Л = 25е;'60 и В = 4е~;'30. Решение. По формуле (383) находим z С - | Л] -|В| е/(а+?) = 25-4е' (60’30) = ЮОе-' 30, ИЛИ С- 100-cos 30° + у-100-sin 30° =-86,6 + ;«50. Два комплекса называются сопряженными, если они различаются только знаками перед мнимыми частями; например, комплексы А = а + jb и В = а — jb сопряженные. Перемножим эти комплексы, применяя обычные правила умножения: С = АВ = (а + jb)(d — jb) — jab + jab — J2b2 = a2 + b2, (384) т. e. произведение двух сопряженных комплексов есть число действитель- ное, равное сумме квадратов действительной части и коэффициента при мни- мой части одного из комплексов. 441
Пример 156. Найти произведение сопряженных комплексов 4 = 6 + у-8 и В = 6 —у-8. Решение. По формуле (384) находим С = 4В=(6 + У-8) (6 —у-8) = 62 4- 82 = 100. 4. Деление комплексов Если требуется разделить один комплекс (делимое) на другой (делитель), то поступают*следующим образом. Вначале освобождаются от мнимости в знаменателе (в делителе), умно- жая делимое и делитель на комплекс, являющийся сопряженным комплексом делителя; затем, производя соответствующие действия, приводят комплекс (частное) к наиболее простому виду. Предположим, требуется разделить комплекс А = a -f- jb на комплекс В = с + j d. Запишем действие деления комплексов в виде алгебраической дроби: Г = А = а + В с jd' Освободимся от мнимости в знаменателе, умножив числитель и знамена- тель дроби на комплекс, сопряженный с комплексом знаменателя, т. е. __ (я + jb)(c — jd) _ ас — jad + jbc + bd __ ас 4- bd be — ad C ~ (c + jd) (c — jd) + d2 c2 + d2 + 7 c2 + d2 ’ Пример 157. Требуется разделить комплекс A = 4 + /*8 на комплекс В = 5 + j • 2. Решение. Представим частное от деления заданных комплексов в виде алгебраической дроби: А = 4+У-8 В 5 + у-2 ’ Освободимся от мнимости в знаменателе, умножив числитель и знамена- тель дроби на комплекс 5 — / • 2: (4 + Л8)(5-Л2) 20~/.8 + У-40 + 16 36+Л32 , (5-Ь у -2) (5 —/-2) 52 + 22 29 ” + 7 ’ 1’1* Частное от деления двух комплексов, выраженных в показательной форме, есть комплекс, модуль которого равен частному от деления модуля комплекса делимого на модуль комплекса делителя, а аргумент равен алге- браической разности аргументов делимого и делителя: г*__ А ___ Ml _____ Ml (а—?) /оосч 1Qe ’ (385) где |С|-Ж " Пример 158. Требуется определить частное от деления двух комплексов: А == 25е? 2 и В = 10е? 4 . 442
Решение. По формуле (385) находим . / к тс \ . тс С = А= 25 е\2 4) =2,5е7 4 = 2,5-cos45° + /-2,5-sin45° = = 1,76 +/-1,76. 5. Возведение в степень комплекса При возведений в степень комплекса необходимо возвести в заданную степень его модуль, а аргумент помножить на показатель степени: С = (| Л] еу*)" = ([Л| )n ey/zot = |С|е'\ (386) где |С] = (]Л|)Л и т = т. Пример 159. Требуется возвести в третью степень комплекс Л=4 + /*3. Решение. Изобразим комплекс А в показательной форме. Для этого определим модуль и аргумент комплекса. Модуль комплекса |Л| = /42 + З2 = 5. 3 Аргумент комплекса найдем по тангенсу: tg а = = 0,75, откуда а = 36°50'. Определяем искомый комплекс: с= А3 = (5е^'36°50 )3 = 125е',110°30'. Л 6. Извлечение корня из комплекса При извлечении корня из комплекса необходимо извлечь корень дан- ной степени из модуля комплекса, а аргумент комплекса разделить на пока- затель корня: С = У |Л|е-'я -И 1Л]е? " = |С|еА, (387) где ]С| = У|Л| и 7 = ^-. Пример 160. Требуется извлечь корень квадратный из комплекса Л = 9+ 12. Решение. Изобразим комплекс Л в показательной форме. Для этого определим модуль и аргумент комплекса. Модуль комплекса |Л| = J/92 + 122 =Д5. Аргумент комплекса находим по тангенсу: 12 tga = -д- = 1,333, откуда а = 53° 10'. 443
Определяем искомый комплекс: /•53°10' с = У А = У15 е 2 = 3,873 е^26035'. § 123. КОМПЛЕКСЫ СИНУСОИДАЛЬНОГО ТОКА И НАПРЯЖЕНИЯ Допустим, что задан синусоидальный переменный ток, мгновенное значе- ние которого определяется выражением i = sin (®< + ф). Построим для этого тока векторную диаграмму в комплексной плоско- сти так, чтобы начальная ось диаграммы совпала вещественных величин. Вектор Рис. 279. Вектор тока 1т в ком- плексной плоскости с осью положительных амплитуды тока 1т отложим под углом + ф к начальной оси, или, иначе, оси вещественных положительных величин, в стерону против вращения часовой стрелки (рис. 279). Так как каждому вектору в ком- плексной плоскости соответствует неко- торое комплексное число, то и вектору тока соответствует некоторый ком- плекс. Модулем этого комплекса яв- ляется численное значение вектора 1т> а аргументом — угол <о^+ф. В соот- ветствии с этим комплекс, соответствую- щий вектору 1т, можно записать в по- казательной форме так: I е/(^+Ф) Представим теперь этот комплекс тока в тригонометрической форме: = 7m-cos (о/ + ф) + jlm-sin (<of + ф). (388) Нетрудно видеть, что выражение, стоящее при мнимой единице (/), есть выражение мгновенного значения заданного синусоидального переменного тока. Это и понятно, так как мгновенное значение синусоидального тока на векторной диаграмме определяется как проекция вектора 1т на вертикальную ось, перпендикулярную начальной оси, или в данном случае как проекция на ось положительных мнимых величин. В соответствии с этим условились комплекс мгновенного значения синусоидального пе- ременного тока i = sin (at + ф) представлять выраже- нием I (f) = 1т (389) где i(t)—условное обозначение комплекса мгновенного значения сину- соидального переменного тока; 1т — модуль комплекса; (<о/ + ф) — аргумент комплекса. Но если от комплекса м г н о в е н.н о г о значения сину- соидального тока необходимо перейти к обычному вы- ражению синусоидального тока, т. е. к выражению реального тока i = Im' sin (со/4- ф), необходимо представить комплекс мгновенного значения тока в тригонометрической 444
форме и из него взять выражение при мнимой единице, которое и будет выражением мгновенногю значения реального тока. Пример 161. Комплекс мгновенного значения синусоидального переменного тока равен i(t) = 15 • е \ /. Определить выражение для мгновенного значения тока. Решение. Выразим заданный комплекс тока в тригонометрической форме /(/) — 15-cos (<о/ + + у. 15• sin (со/ 4- . Коэффициент, стоящий при мнимой части комплекса, и есть выражение мгновенного значения реального тока, т. е. i = 15-sin (со/ + . Пример 162. Мгновенное значение тока определяется выражением (2 \ со t—х- тс 1. Требуется найти комплекс мгновенного значения этого о / тока. Решение. Преобразуем выражение для заданного мгновенного значе- ния тока, заменив в нем косинус синусом соответствующего угла: i = 5 • cos (со/-— тс = 5 • sin <ог--х- ) • \ о / \ о / В соответствии с этим комплекс мгновенного значения заданного,, тока будет иметь следующий вид: /(/) = 5е V 6 , или в тригонометрической форме / (t) = 5• cos (со/-+ j-5• sin (со/----5-) . Преобразуем выражение для комплекса мгновенного значения синусои- дального переменного тока i = Im sin (со/ + ф), представив его в следующем виде: Z (/) = lm (390) Множитель е;ш* показывает, что данный ток синусоидальный, так как е7и)* = cos со/ + j. sin со/. Выражение, стоящее при мнимой единице этого комплекса, есть sin со/. Выражение /we7'^, стоящее в качестве сомножителя при в ком- плексе мгновенного значения синусоидального тока, называется комплекс- ной амплитудой: 4 = (391) 445
где Im — условное обозначение комплексной амплитуды тока; 1т — модуль комплексной амплитуды тока (максимальное значение си- нусоидальной величины); Ф — аргумент комплексной амплитуды тока (начальная фаза синусои- дального тока). Чтобы показать, что рассматриваемый ток синусоидальный, над симво- лом 1т поставлена точка. Это правило — ставить точки над буквами, изобра- жающими комплексы,— распространяется на все синусоидальные величины, например синусоидальные напряжения, электродвижущие силы и т. д. _ Разделив левую и правую части комплексной амплитуды на)/2, по- лучим комплекс действующего значения синусоидаль- ного переменного тока: ^т_ __ е/ф V2 /2" ’ или I = (392) где /—комплекс действующего значения" синусоидального переменного тока; /—модуль комплекса действующего значения синусоидального тока; ф — аргумент комплекса действующего значения тока. Пример 163. Задан комплекс действующего значения синусоидального переменного тока /=10e7^0°. Требуется определить действующее значение тока, начальную фазу его, комплексную амплитуду тока, комплекс его мгно- венного значения и мгновенное значение тока. Решение. Действующее значение тока равно модулю комплекса задан- ного тока I = Юе7'300, т. е. I = 10 а. Начальная фаза равна аргументу тока, т. е. ф = 30°. Комплексная амплитуда тока jm = ]/f/e''30° = V2- Юе7'30’ = 14,le^30°. Комплекс мгновенного значения тока ime®eiwt = 14,1е',30° e;W = 14,le;‘(u,z+3°O). Мгновенное значение тока /= 14,1-sin (ш/ + 30°). Аналогично для переменного синусоидального напряжения можно напи- сать комплексы мгновенного значения, действующего значения и комплексную амплитуду. Например, если u=/7/„‘sin (со/—ф1), то: комплекс мгновенного значения напряжения комплекс действующего значения напряжения U = f/e-^i; комплексная амплитуда напряжения и я и 446
§ 124. СОПРОТИВЛЕНИЕ ЦЕПЕЙ ПЕРЕМЕННОГО ТОКА В КОМПЛЕКСНОЙ ФОРМЕ А) Вначале определим комплекс полного сопротивления цепи с последо- вательно соединенными активным г и индуктивным <о£ сопротивлениями. Допустим, что к рассматриваемой цепи приложено синусоидальное на- пряжение и = • sin И + ф). В соответствии с этим мгновенное значение тока в этой цепи будет равно I = (ш/ + ф — ср), где Z = ]/г2 + (шА)2— полное сопротивление цепи; <р<—угол сдвига фаз тока и напряжения, определяемый по его тангенсу tg <р = -у- . Выразим действующие значения заданных напряжения и тока в комплекс- ной форме: U= —комплекс действующего значения напряжения; — комплекс действующего значения тока. Если разделим комплекс действующего значения напряжения U на ком- плекс действующего значения тока /, то получим комплекс полного сопро- тивления цепи Z = Д = —е7<р = zej'f, (393) где Z — комплекс полного сопротивления цепи; Z—модуль комплекса полного сопротивления цепи, равный z = = JZг2 + (со/,)2; ср — аргумент комплекса полного сопротивления цепи, равный углу сдвига фаз тока и напряжения. Выразим комплекс полного сопротивления цепи в тригонометрической, а затем в алгебраической форме: Z — ze^ = z- cos<р + /г-sin<р. (394) Но, как известно, z*cos<p = r— активное сопротивление цепи, а z • sin ср = (оА — реактивное (индуктивное) сопротивление цепи; поэтому Z = ze^ = z-cos ср + yz-sin ср = г + j^L. (395) Следовательно, вещественная часть комплекса полного сопротивления цепи изображает здесь активное со- противление цепи, а мнимая часть — реактивное. Перед мнимой частью комплекса полного сопротивления цъпи с индук- тивным характером нагрузки стоит знак положительный. Пример 164. Требуется выразить в комплексной форме полное сопротивле- ние цепи, состоящей из последовательно соединенных активного сопротивле- ния г = 8 ом и индуктивности Л = 0,0012 гн, если угловая частота сину- соидального напряжения, приложенного к цепи, равна w = 5000 —— . 447
Решение. Индуктивное сопротивление xL = <оЛ = 5000-0,0012 = 6 ом. Комплекс полного сопротивления цепи в алгебраической форме равен Z — f = 8 -}- j -6. Модуль комплекса полного сопротивления цепи Z = ]/Г2 + (Ю£)2 = |/§2Тб2 = 10 ОМ. Аргумент комплекса полного сопротивления цепи найдем по его тангенсу: tg ср = = 0,75, SY г 8 ’ ’ откуда ср = 36°50'. Комплекс полного сопротивления цепи в тригонометрической форме будет иметь следующий вид: Z = 2:-cos ср 4- угг• sin ср = 10-cos 36°50' + j-10-sin 36ь50'. Комплекс полного сопротивления в показательной форме будет иметь следующий вид: Z = ге™ = Юеу-36°50'. Б) Теперь определим комплекс полного сопротивления для цепи с после- довательно соединенными активным г и емкостным сопротивлениями. Допустим, что к рассматриваемой цепи приложено синусоидальное на* пряжение и = Um • sin ( wZ + ф). В соответствии с этим мгновенное значение тока в цепи i = • sin (ш/ + ф + ср), где Z = у г2 + — полное сопротивление цепи; ср — угол сдвига фаз тока и напряжения (ток опе- режает напряжение по фазе на угол ср). Выразим действующие значения заданных напряжения и тока в ком- плексной форме: (j=U^—комплекс действующего значения напряжения; / = /е7(Ф“Нр) — комплекс действующего значения тока. Если разделим комплекс действующего значения напряжения (Л на ком- плекс действующего значения тока /, то получим комплекс полного сопро- тивления цепи Z = —Г- =------/, , . == —Г е л = ze I /е/(Ф+т) I (396) где £= -j — модуль комплекса полного сопротивления, равный z « 448
Ф—аргумент комплекса полного сопротивления цепи, равный углу сдвига фаз тока и напряжения; ф можно определить по его тан- . . а>С генсу tg <р — —у— . Выразим комплекс полного сопротивления цепи в тригонометрической, а затем в алгебраической форме: Z = ze Jcp=-г-cosф— y2?-sin<p. (397) Но, как известно, z * cos ф = г — активное сопротивление цепи и г • sin ф = — реактивное (емкостное) сопротивление цепи;- поэтому Z = « г-cos ф — уг-sin ф = г — (398) Следовательно, вещественная часть комплекса полного сопротивления изо- бражает здесь активное сопротивление цепи, а мнимая часть — реактивное (емкостное). Перед мнимой частью комплекса полного сопротивления цепи с емкост- ным характером нагрузки стоит знак отрицательный. Пример 165. Требуется выразить в комплексной форме полное сопротив- ление цепиг состоящей из последовательно соединенных ‘активного сопротив- ления г = 60 ом и емкостного сопротивления %с = = 40 ом. Решение. Комплекс полного сопротивления в алгебраической форме равен Z - г — J -U <= 60 — у-40. О) и .... Модуль комплекса полного сопротивления цепи равен г = j/rs + (-L-)2 = /602 + 4Q2 « 72,2 ом. Аргумент комплекса полного сопротивления цепи найдем по его тан- генсу: 1 шС —40 Л tg?- — --ад-»-0,666, откуда = —33°40'. Комплекс полного сопротивления в тригонометрической форме будет иметь следующий вид: Z = г-созф — Jz-sin ф == 72,2-cos 33°40'~у-72,2-sin ф. И, наконец, комплекс полного сопротивления в показательной форме бу- дет иметь вид Z «- ге~;<р « 72,2е~; *33°40\ 29—1377 449
В) Для цепи, состоящей из последовательно соединенных активного г, г 1 индуктивного o)L и емкостного сопротивлений, комплекс полного сопро- тивления цепи в алгебраической форме изобразится следующим образом: Z~r + jt <*>L------------------ где а-— вещественная часть комплекса (активное сопротивление цепи); J —мнимая часть комплекса, соответствующая полному реактив- ному сопротивлению цепи. Модуль комплекса численно равен полному сопротивлению заданной цепи: Z —Г. шС ) Аргумент комплекса полного сопротивления цепи найдем по его тан- генсу: cdL--- tg? = 1 Ш С откуда по тригонометрическим таблицам находим аргумент ср. Если (j)L> -^£-,то <₽>0 и перед мнимой частью комплекса надо поста- вить знак положительный; если же, наоборот, <oL < , то ср < 0 и перед мнимой частью комплекса надо поставить знак отрицательный. Следовательно, если wL > -Дт , то со С Z ® г + j(^L---= лг-cos <р -ь/г-sin <р, т 1 9 если соь < —тг , то соС Z = г — j(vL—= ге~;ф = г-cos ср — /г-sin <р. § 125. ПРОВОДИМОСТИ ЦЕПЕЙ ПЕРЕМЕННОГО ТОКА В КОМПЛЕКСНОЙ ФОРМЕ Комплекс полной проводимости цепи — это величина, обратная комплексу полного сопротивления цепи. А) Для цепи с активно-индуктивным характером нагрузки где У — комплекс полной проводимости цепи; Z — комплекс полного сопротивления цепи. 450
Преобразуем равенство (399), умножив числитель и знаменатель его на комплекс, сопряженный с комплексом полного сопротивления. Это даст нам возможность освободиться от мнимости в знаменателе: у _ г — jab __________________ f — J(oL f . юA _ y ~ (r + ja>L) — = Г2 + (ф£)? = Г2 + (<oZ,)2 ~J Г2 + (o>£)4 “ = g—jb, (400) где g Г2 + ((0£)2 A __ CO A r2 4- (col)2 Следовательно, проводимости — активная проводимость цепи; ♦ — реактивная проводимость цепи. вещественная часть комплекса полной цепи является активной проводимостью, а мнимая — реактивной. Перед мнимой частью комплекса полной проводимости цепи с активно- индуктивной нагрузкой стоит знак отрицательный. Модуль комплекса (400) равен y = \Y\ = V^ + b\ а аргумент комплекса находится по тангенсу этого угла . b В показательной и тригонометрической формах комплекс полной прово- димости для цепи с активно-индуктивным характером нагрузки имеет следую- щий вид: Y = g— jb = ye~jr* = j/-cos<p — jty-sintp, где у = 1 У | — модуль комплекса. Б) Для цепи с активно-емкостным характером нагрузки где У— комплекс полной проводимости цепи; Z — комплекс полного сопротивления цепи. Преобразуем равенство (401), умножив числитель и знаменатель его на комплекс, сопряженный с комплексом полного сопротивления цепи: + j---f + j---------------ТТ У <o(J g)(? f f 1 J-----‘FFu “ S + Jb, r2 + (—r \ J r где g «= « —7—=—г-5 — активная проводимость цепи; 1 , (DC 0 — ----7~i—П—реактивная проводимость цепи. + (~7Г) 29* 451
Перед мнимой частью комплекса полной проводимости цепи с активно- емкостной нагрузкой стоит знак положительный., В показательной и тригонометрической формах комплекс полной прово- димости для рассматриваемого случая будет иметь следующий вид: Y -= g + jb =» уе+я =* у • cos ср 4- jy • sin <р, где у = [ У1 = g2 + b2 — модуль комплекса полной проводимости; <р— аргумент комплекса, определяемый по формуле 1 tg ? = — • В) Если задана цепь с последовательно соединенными активным г, индук- тивным cdL и емкостным сопротивлениями, то выражение комплекса пол- ной проводимости этой ц^ри будет находиться в зависимости от соотношений индуктивного и емкостного сопротивлений этой цепи. Для случая, когда <dL > или X = &L--------------4г > 0 и ф > О, CD С CD С Y — g — jb=ye = у • cos ср — jy • sin ср. Для случая, когда coL < —4т или X = &L--------------4г < 0 и ф <0, соС соС У = g + Jb = = y^costf + Jy-sin <f, где у = | У | = V у2 + b2, г2 + (coL И Ь { IX \ / а угол ф находится по тангенсу его tgv г S 126. МОЩНОСТЬ СИНУСОИДАЛЬНОГО ТОКА В КОМПЛЕКСНОЙ ФОРМЕ Для того чтобы определить мощность электрического тока в комплекс- ной форме, необходимо комплекс напряжения помножить на комплекс, сопря- женный с комплексом тока: Pt = UI, (402) где Р[—комплекс полной (кажущейся) мощности; U— комплекс полного напряжения, приложенного к цепи; * I — комплекс, сопряженный с комплексом тока Л Предположим, что u = u' + ju" и i~r + ji", 452
где (J—комплекс напряжения; U'—действительная часть комплекса U\ U"—коэффициент при мнимой части комплекса [); I—комплекс тока; Г — действительная часть комплекса тока /; Г—коэффициент при мнимой части комплекса тока I. Сопряженный комплекс тока в данном случае принимает вид следовательно, мощность согласно формуле (402) будет определяться сле- дующим выражением: = (U' + JU") (/' - /7") = U'F - jUT + JU"Г 4- U"I"t или Pi = (UT + U'T) + j (U"r — U’l"), (403) или, иначе, Pi-Pa + JPr, (404) где Pa = UT + U"I", (405) Pr=U"r—U'I", (406) Pa— активная мощность, развиваемая током в цепи; Рг—реактивная мощность, развйваемая током в цепи. В формуле (403) перед мнимой частью комплекса будет знак плюс при индуктивном характере нагрузки и минус при емкостном характере. Мощность в комплексной форме можно также получить, умножив ком- плекс тока на комплекс, сопряженный с комплексом напряжения: Pi = Ui. (407) В этом случае мы получим тот же результат, что и по формуле (404), только с той разницей, что знак перед мнимой частью комплекса изменится на обратный. Пример 166. Определить мощность тока в цепи, если 77=80 + 7-100 и 7 = 4 + у-3. Решение. По формуле (402) находим Pt = UI = (80 + 7- 100) (4 — 7«3) = 320 — /-240 + 7-400 + 300 ® 620 + 7-160, или, иначе, ^ = ^ + 7^ = 620 +7-160, откуда Ра = 620 вт и Рг — 160 ва. Знак плюс у реактивной мощности показывает, что режим в цепи носит индуктивный характер. Полная (кажущаяся) мощность равна модулю комплекса: pl = + = ^6202 + 1602 = 640 ва. 453
§ 127. ОСНОВНЫЕ ЗАКОНЫ ПЕРЕМЕННОГО ТОКА В КОМПЛЕКСНОЙ ФОРМЕ I. Закон Ома в комплексной форме выражается для участка цепи следую- щей формулой: /=^, (408) где /—комплекс действующего значения тока; U— комплекс действующего значения напряжения, Z — комплекс полного сопротивления цепи. Пример 167. К цепи, состоящей из последовательно соединенных актив- ного сопротивления г = 40 ом и индуктивного сопротивления = = 30 ом, приложено синусоидальное йапряжение; комплекс действующего значения его равен U = 40 + j • 80. Требуется определить комплекс тока, мо- дуль комплекса и его аргумент. Решение. Выразим полное сопротивление цепи в комплексной форме: Z = г + >£ = 40 + у-зо. Согласно формуле закона Ома _ 40 + 80 (40 + У-80) (40 — У-30) 7 ” Z “ 40 4- у-30 402 + 302 1600 +/.3200-/.1200 + 2400 Z1 с „ =--------------2500---------------= (1,6+/0,8) а. Модуль комплекса тока / равен /1,62 + 0,82 = 1,79 а. Аргумент комплекса тока I найдем по его тангенсу: tg ф = у|- = 0,5, откуда ф = 26°32', и тогда ) 7-1,79еу,26°32'. Пример 168. Определить комплекс напряжения, приложенного к цепи, комплекс сопротивления которой равен Z = (45 — / • 20) ом, а комплекс тока / = 3 е',64°- Решение. Представим комплекс сопротивления цепи в показательной форме. Модуль сопротивления равен z = J/452 + 202 « 49,2 ом. Аргумент комплекса сопротивления найдем по его тангенсу 20 tg <р = —— = — 0,445 и — 24°. Следовательно, г = 49,2е~-'-24°. 454
Теперь определим комплекс напряжения U=IZ~ Зе^64°-49,2е~>,24° = 147,6еу’40° = (104,5 + 93,5) в. II. Первый закон Кирхгофа в комплексной форме формулируется сле- дующим образом: алгебраическая сумма комплексов токов для любой узловой точки цепи переменного тока равна нулю: S4-0. (409) В формуле (409) токи необходимо брать с положительными знаками, если положительное направление их обращено к данной узловой точке, и, наобо- если положительное направление их обра- рот, с отрицательными знаками, 0 If I '4 ч Рис. 280. Стрелки указывают положительные направления Рис. 281. Контур abcda сложной цепи переменного тока токов щено от узловой точки. Например, для узловой точки О некоторой сложной схемы (рис. 280) уравнение согласно первому закону Кирхгофа напишем так: S 4 = 4+ 4-4 = о. Пример 169. Определить ток Z, в схеме рис. 280, если Л = (6 + /-12) а и /2 = (2 + /• 3) а. . Решение. Согласно первому закону Кирхгофа А + 4 — 7з = 0» или 4 - 4 + /, = (6 + /.12) + (2 + J-3) =• 8 + ] 15 = 17е'-б2°. III. Второй закон Кирхгофа в комплексной форме формулируется сле- дующим образом: алгебраическая сумма комплексов электродвижущих сил, имеющихся в замкнутом контуре, равняется алгебраической сумме комплек- сов падений напряжений на всех участках этого контура: S ё = S iz. - (4Ю) На рис. 281 приведен замкнутый контур abcda, выделенный из некоторой сложной цепи переменного тока. Положительные направления электродви- жущих сил и токов в контуре обозначены стрелками. Уравнение второго за- кона Кирхгофа для такого контура имеет вид Ё2 = hz^ 455
Применяя законы Кирхгофа в комплексной форме, можно рассчитать слож- ные цепи переменного тока, что и будет показано в одном из следующих примеров. IV. Комплексный метод дает возможность при расчете параллельных це- пей переменного тока применять расчетные формулы, аналогичные формулам, применяемым при расчете параллельных цепей постоянного тока. Эквивалентное сопротивление нескольких параллельных ветвей в ком- плексной форме можно определить по формуле _L = _L + J_ + _L Z Z1 + Z. + Z3 ’ где Z — эквивалентное сопротивление в комплексной форме; Z2 и т. д. — комплексы полных сопротивлений параллельных ветвей. В частном случае, когда параллельная цепь состоит из двух ветвей, 7 — ^1^8 Zt + Z2 ’ т. е. комплекс полного сопротивления двух параллельных ветвей равняется частному от деления произведения комплексов полного сопротивления этих ветвей на сумму комплексов этих сопротивлений. § 128. ПРИМЕРЫ РАСЧЕТА ЦЕПЕЙ ПЕРЕМЕННОГО ТОКА ПРИ ПОМОЩИ КОМПЛЕКСНОГО МЕТОДА Пример 170. Две параллельно соединенные катушки обладают сопротив- лениями Zj = 5 + j • 8 и Z2 = 4 + j • 4.- Определить общий ток в цепи, если напряжение, приложенное к ней, U = 120 + j • 0. Решение, Полное комплексное сопротивление цепи = ZtZ2 = (5+7-8) (4 + у-4) = 20+у-20 + у.32- 32 _ + Z2 5 + 7*- 8 + 4 + 7’-4 9 + 7-12 _ (—12+/.52) (9 — ;. 12) _ Ю8 + у-144 + у-468 + 624 “ (9+ 7-12) (9 —7-12) “ 92 + 122 ~ = 516 225 61" = 2,3 + У'2,72- ) zzo Комплекс тока • 120 _ 120(2,3 —/-2,72) 276 —/-326,4 7 1 Z 5=1 2,3 + 7-2,72 2,32 + 2,722 12,7 21,4 J °' ' Модуль тока 7 = J/21,42 + 25,72 = 33,5 а. Сдкрг фаз между током и напряжением найдем по тангенсу: , 2’72 1 откуда по таблице тангенсов ср ~ 49°50'. Пример 171. Определить токи во всех участках сложной цепи (рис. 282), если Zi = 10 — / • 15; Z2 = 2 + j • 6 и Z3 = 3,33 + J • 2 и приложенное к цепи напряжение U = 120 + j • 0. 456
Решение. Определим эквивалентное сопротивление двух параллельных ветвей: v , _ Z1Z2 _ (Ю —/15)(2+/6) _20 + /60 — /30 + 90 - Z1’2 Zi + Za 10—/15+ 24-/6 “ 12—/9 110 +/30 = 12 — /9 * Освободимся от мнимости в знаменателе: _ (ПО +/30) (12 +/9) = 1320 +/990 +/360 — 270 Z‘,S- 122 + 92 225 = теории = 467 + Л6 Полное сопротивление цепи ^1,2,з = ^1,2 + Z3 = 4,67 + /6 + 3,33 + /2 = 8 + J-8. Комплекс тока f U _ 120 _ 120(8-7-8) 7._,. 7- '° ~ ZMj3 ~ 8 + J-8 “ 8^ + 82 =/’5 •/'7Д Модуль тока /0 = /7,52 + 7)52 = ю,б а. Комплекс падения напряжения в сопротивлении Z3 й3 = ioZ3 = (7,5 —7-7,5) (3,33 + 7-2) = 25 +7-15—7-25 + 15 = 40-7-10. Комплекс падения напряжения на параллельных ветвях Uab = U— 03 = 120 — (40 — 7-10) = 80 + 7-10. Комплекс тока в первой ветви j _ Uab _ 80 + 7-10 _ (80+ 7-10) (10+ 7-15) 1 Z, 10-7-15 102+152 _ 800 + 7-1200 + 7-100- 150 „ , . . “ 325" -2 + 7-4. Модуль тока первой ветви /, = /22 + 42 = 4,47 а. Комплекс тока во второй ветви 12 = /о - А = (7,5 — 7,5) - (2 + /4) = = 5,5 —/11,5. Модуль тока второй ветви 7, = /5,52 + 11,52 К 12,75 а. Рис. 282. Схема сложной цепи с одним источником электриче- ской энергии 457
Пример 172. Два генератора работают на общий приемник энергии, имею- щий сопротивление Z3 = 5,5 + / • 4 (рис/283). Электродвижущие силы генера- торов Ei — 120 е й ^2=110 в совпадают по фазе. Сопротивления обмоток генераторов равны друг другу: Z\ = Z2 = 0,5 + j • 2. Найти величину тока каждого генератора, ток в приемнике и напряжение на его зажимах. Решение. Дано: =120+ 7-0; £2=11О + /-О; ^ = 0,54-7-2; Z2 = 0,5 + ).2; Z8 = 5,5 +7*4. Применяя законы Кирхгофа, составим три уравнения с тремя неизвест- ными токами. Для этого в первую очередь зададимся положительными на- Рис. 283. Схема сложной цепи с двумя источниками электри- ческой энергии или правлениями токов, как это показано на рис. 283 стрелками. Уравнения составим, применяя первый закон Кирхгофа для узловой точки b и второй закон Кирхгофа для контуров abefa и bcdeb: ii + ia = ii (i) Ёх - IXZX + /»Z8; (II) £2-4Zs+ hZt, (III) A + h = A; 120 = A (0,5 + 7-2) + 4(5,5 + 7-4); 110 = (0,5 + 7-2) + /, (5,5 + 7-4). Подставим вместо А его выражение из уравнения (I) в уравнения (II) и (III): 120 = А (0,5 + 7-2) + (А + А) (5,5 + ;-4); НО = А (0,5 + 7-2) + (А + А) (5,5 + 7-4), или 120-А (6 + У-6) + А(5,5 + Л4); (II') 110 - А (6 + /-6) + А (5,5 + J-4). (ИГ) Помножим уравнение (II') на (6 +/-6), а (III')—на (5,5 + /*4) и вычтем из первого уравнения второе: _ 120 (6 + /-6) = /; (6 + 7-6) (6 + J-Q) + /, (5,5 + j-4) (6 + 7-6) ПО (5,5 + j-4) = 1\ (5,5 + J-4) (5,5 + /-4) + 4 (5,5 + J-4) (6 + J-6)____ 120 (6 + 7-6)-110(5,5 + 7-4) = А (6 + 7-6) (6 + 7-6)-А(5,5 + 7-4)(5,5+7-4) или 115 + 7-280 = А (— 14,25 + У-28), откуда 115 + 7-280 (115 + 7-280) (— 14,25-7-28) = — 14,25 +7-28 = 14,25» + 282 — 1639 — 7-3220 — 7-3990 + 7840 _ 6201-7-7210 203 + 784 “ 987 ~М 458
Подставив в уравнение (II') вместо /1 полученное выражение, найдем: 120 - (6,3 — У-7Э3) (6 + 7-6) + /2 (5,5 +7-4), или 120 = 37,8 + 7-37,8 — 7*43,8 + 43,8 + Z2 (5,5 + 7-4), откуда находим t _ 38,4 + 7*6 (38,4 + 7-6) (5,5 — 7*4) _ 235,2-7-120,6 2 5,5 + 7-4 = 5,52 + 42 ” 46,25 ’ 7 ’ Подставляя в уравнение (I) вместо Л и 72 полученные выражения, на- ходим ’ А = А + 4 = 6,3 - /-7.3 + 5,1 —y-2,6 = 11,4 — у-9,9. Напряжение на зажимах приемника найдем, вычтя из электродвижущей силы первого генератора величину падения напряжения в нем: = — ixzx = 120 — (6,3 — 7-7,3) (0,5 + 7*2) = = 120 —(3,15 + 7-12,6 — 7-3,65 + 14,6) = 102,25-7-8,95. Модуль тока первого генератора h = J/6,32 + 7,32 = 9,6 а. Модуль тока второго генератора /2 = ]/5,12 + 2,62 =х= 5,7 а. Модуль тока потребителя 73 « J/ll,42 + S,92 = 15,1 а. Мощность, развиваемая первым генератором, Pi » Ё*Ц = 120 (6,3 + 7-7,3) = 756 + 7-876. Мощность, развиваемая вторым генератором, Р2 - Е2*2 = ПО (5,1 + 7-2,6) =» 561 + 7-286. Мощность, поглощаемая потребителем, р9 ~ ад - (102,25-7-8,95) (11,4 + 7-9,9) = 1255 + 7-910. Активная мощность, развиваемая генераторами, РОа = Pia + Р2Л — 756 + 561 1317 вт. Активная мощность, получаемая потребителем, Р3а — 1255 вт. Активная мощность, потребляемая самими генераторами, Ргв » Роа — Р3а « 1317 — 1255 = 62 вт. 459
Проверка: Pra = fa 4- /®г2 = 9,62 -0,5 + 5,72-0,5 = 62 вт. Пример 173. Для схемы, изображенной на рис. 284, определить токй и напряжения во всех участках цепи, мощность, расходуемую во всей цепи, а также сдвиг фаз тока /о и напряжения U, если и = 1,5 ом; х„=2 ом; хТ1. = Ьощ хГс. = 6 ом; г = 3 ом; х._ = 3 ом; г = 2 ом; г4 = 3 ом; хС4 = 4 ом и ток /2 = 2 а. Решение. Выразим величины сопротивлений в комплексной форме: Zj = fl — jxci = 1,5 — J-2 = 2,5e--'’53°10'; Z3 = r2 + j (x.2 - x \ = 3 + Д4 - 6) = 3 - J-2 = 3,6e->-33°40'; Z3 = r3 + jxL2 = 2 + j-3 = 3,6e^56°20'; Z4 = 4 - jxCi = 3 - j-4 = 5e->- 53°10'. Рис. 284. Схема сложной цепи Вектор тока /2 направим по оси мнимых величин в сторону положитель- ной полуоси, т. е. /2 = // 2 = 2е7-9°° = у.2. Напряжение между точками с и d (Jcd = /2Z2 = 2е^9°о-3,6е_-'-33°40' = 7,2е'-56°20' = 4 + >-6. Комплекс тока /8 .. Uca _ 7,2е>-56°20' ’ Z3 - з,6е'-56°20' = 2е-'-°° = 2 + J0 = 2. Комплекс тока Z, 4 = /; + 4 = у.2 + 2 = 2,82е'-45°. 460
Комплекс напряжения l)ac Uac - AZt = 2,82еу',45О-2,5е7у,53°10' - 7,О5е“-у,8°10' = 7 - / Комплекс напряжения Uab иаъ = Ucd + йас = 4 + J Q + 7 - J = 11 + J-5 = 12,1е>-24030'. Комплекс тока /4 z*r 19 1 * 24°30' 4 = = 12’1е А . = 2,42е-''77°40' = 0,52 + J-2,36. ^4 5e“;‘w 1U Комплекс тока /0 неразветвленного участка цепи /0 = 4 + /4 = 2 + /2 + 0,52 + ;-2,36 = 2,52 + /-4,36 - 5е;>60’. Полная мощность, развиваемая источником электрической энергии в цепи, Pt = UabI0 = 12,1е7''24°30'-5е~-''60° - 6О,5е”^35°30' и 49,2—J-35,2. Активная мощность, развиваемая источником электрической энергии в цепи, Ра — 49,2 вт. Реактивная мощность, разбиваемая источником электрической энергии в депи, Рг = 35,2 ва. Модули токов равны: /0 = 5 а\ 11 = 2,82 а\ 1а = 2 а\ /8 = 2 д; Ц = 2,42 а. Модули напряжений равны: Uab ~ 12,1 a; Uac = 7,05 a; Ucd 7,2 в. Эквивалентное сопротивление всей цепи 7 — ^аЪ - 12,1е/'24°30 _ 2 _,.35°30' /о " 5е>-60° " ’ е Модуль комплекса эквивалентного сопротивления всей цепи Z3 = 2,42 ом. Сдвиг фаз тока Zo неразветвленного участка цепи и напряжения U = Ua^ приложенного к цепи, равен ?0=в— 35°30'. Следовательно, ток /о опережает по фазе напряжение U на угол 35°30'л т. е. для генератора вся цепь в целом представляет нагрузку емкостного характера.
ГЛАВА XXV МНОГОФАЗНЫЕ СИСТЕМЫ § 129. ТРЕХФАЗНАЯ СИСТЕМА Среди многофазных систем переменного синусоидального' тока наиболее широкое применение получила трехфазная си- стема. Трехфазной системой называется совокупность трех однофаз- ных цепей, в которых действуют три электродвижущие силы одинаковой частоты, сдвинутые по фазе относительно друг друга на угол 120° На рис. 285 приведена принципиальная схема простейшего генератора трехфазного тока. Здесь на неподвижном стальном Рис. 285. Принципиальная схема генератора трехфаз- ного тока кольцевом (тороидальном) сердечнике размещены три совершен- но одинаковые катушки, сдвинутые пространственно относи- тельно друг друга на 120°, т. е. начала (Я1, Н2 и Н3) обмоток катушек сдвинуты по окружности сердечника по отношению друг к другу на 120°. Точно так же размещены и концы (Ki, Kz и Кз) обмоток. 462
Кольцеобразный сердечник с размещенными на нем катуш- ками является, неподвижной частью генератора и называется статором. Внутри статора вращается вокруг своей оси двухполюсный электромагнит, называемый ротором генератора. Обмотка ротора генератора питается постоянным током от постороннего источника электрической энергии, обладающего постоянным напряжением. Следовательно, в системе данного ро- тора существует постоянное магнитное поле. Магнитный поток выходит из северного полюса ротора, проходит через воздушный за^ор, далее попадает в стальное кольцо (статор), разветвляется на два направления и затем, вновь соединясь в общий магнит- ный поток, проходит через другой воздушный зазор, через южный полюс и замыкается на себя внутри сердечника ротора. С помощью какого-либо первичного двигателя ротор генера- тора приводится во вращение с постоянной угловой скоростью. При своем вращении ротор увлекает за собой магнитное поле, создаваемое им, в результате чего в катушке статора изменяется магнитный поток, пронизывающий витки, и в них индуктируются электродвижущие силы. Конструктивно статор с катушками и ротор с полюсными наконечниками выполняются такими, что в катушках индуктируются синусоидальные электродвижущие силы, сдвинутые по фазе относительно друг друга на 120°. Сле- довательно, если в первой катушке индуктируется синусоидаль- ная электродвижущая сила ^i = ^iOT-sin Ч то во второй катушке будет индуктироваться электродвижущая сила ^2 = ^2m-sin(<o/— -у*) , в третьей катушке — электродвижущая сила ^3 = £3m-sin — у , ♦де еь е2 и е3— мгновенные. значения электродвижущих сил в от- . дельных катушках; EimtE2mviE3m— амплитуды электродвижущих сил в отдельных катушках. Если к каждой из катушек при помощи специальных приспо- соблений подключить потребители электрической энергии, то в этих цепях будут проходить следующие электрические токи (рис. 285); А = Am-sin И — ?1) z A = Am'Sin(®/—-уте—<р2) . (411) А = Ат • sin — у К — Тз) 463
где ib i2 и z3— мгновенные значения токов в первой, второй и третьей катушках; Ilmi hm и hm— амплитуды токов в катушках; и ?з—углы сдвига фаз между электродвижущими си- лами и токами в катушках. Трехфазная система называется симметричной, если амплитуды электродвижущих сил в отдельных фазах генератора одинаковы по величине, т. е. Е<=Е2=Е2т = Ет. itn 2т т В соответствии с этим = Ет • sin sin (<•>/ — -|*) (412) es = £m-sin(<o/ — | Трехфазная система, схематически изображенная на рис. 285, является электрически не связанной, т. е. в ней от- дельные фазы представляют собой совершенно независимые друг от друга электрические цепи. Такая трехфазная система имеет мало преимуществ по сравнению с однофазной системой, а поэтому ее на практике не применяют. Трехфазная система называется электрически связан- ной, если ее отдельные фазы соединены между собой электри- чески. Подобная система обладает значительными преимуществами по сравнению с однофазной системой, так как она требует мень- шей затраты металла на провода при передаче одной и той же мощности. На рис. 286 показана часть схемы обычного трехфазного генератора. Здесь мы видим неподвижную часть генератора — статор, в пазы которого укладываются три отдельные обмотки, смещенные относительно друг друга на 120 электрических граду- сов, т. е. на одну треть двойного полюсного деления. Для нг^ глядности обмотка первой фазы показана густо заштрихованной, обмотка второй фазы — слабо заштрихованной и обмотка тре- тьей фазы — незаштрихованной. Внутри статора помещена подвижная часть генератора — ротор, на цилиндрической поверх- ности которого находятся электромагниты, питающиеся постоян- ным током от постороннего источника электрической энергии. Магнитное поле электромагнитов вращается вместе с ротором, ца пути своего движения пересекает магнитными линиями непо- движные проводники статорной обмотки и индуктирует в них электродвижущую силу. Приоритет изобретения трехфазной системы принадлежит русскому изобретателю М. О. Доливо-Добровольскому, В 1891 г. 464
Доливо-Доброаольский демонстрировал свою трехфазную уста- новку, организовав с помощью ее передачу электрической энер- гии на расстояние примерно в 175 км. Успех этой опытной пере- дачи электроэнергии на большое по тому времени расстояние обеспечил широкое внедрение трехфазной системы в электротех- ническую практику. В наши дни, когда электрификация разви- вается бурными темпами, когда электрическая энергия от мощ- ных электростанций передается на многие сотни километров, трехфазная система электропередачи, как и прежде, играет перво- степенную роль. Рис. 286. Схема генератора трехфазного тока В настоящее время трехфазная система является стандарт- ной во многих государствах мира и в том числе у нас в Совет- ском Союзе. Работая в области совершенствования и развития трехфазной системы передачи электрической энергии, Доливо-Добровольский создал трехфазный генератор и разработал трехфазные системы, соединенные в «звезду» и «треугольник». Асинхронный электро- двигатель трехфазной системы, изобретенный Доливо-Доброволь- ским, нашел весьма широкое применение в электротехнической практике благодаря его высоким эксплуатационным качествам и простоте конструкции. Одно из важнейших преимуществ трехфазной системы перед однофазной — меньшая затрата металла на провода при пере- даче одной- и той же мощности (вес меди, идущей на провода и обмотки генераторов трехфазной системы, примерно на 25% меньше веса меди, необходимой для двух проводов однофазной системы). Кроме того, передача электрической энергии по прово- дам при трехфазной системе осуществляется с меньшими поте- рями. 30-1377 465
Другое весьма важное преимущество трехфазйой системы — возможность получения вращающегося магнитного поля, с по- мощью которого осуществляется работа ряда простых по кон- струкции и удобных в эксплуатации машин и приборов. § 130. ТРЕХФАЗНАЯ СИСТЕМА, СОЕДИНЕННАЯ <ЗВЕЗДОЙ» Если в схеме, изображенной на рис. 285, начала всех трех фаз генератора (Hi, Hz и Яз) соединить в одну общую точку, называемую нулевой точкой генератора (0г), а к концам фаз генератора присоединить линейные провода, идущие к по- требителям, то получим трехфазную систему, соединенную «звездой». В этом -случае три обратных провода, имеющихся в трехфазной электрически не связанной системе, можно заме- нить однйм общим обрат- Рис. 287. Схема соединенной „звездой" трехфазной четырехпроводной системы с нулевым проводом ным проводом, обслужи- вающим одновременно все три ф а з ы и называемым нулевым или ней- тральным прово- дом. На рис. 287 при- ведена подобная схема, называемая т р е х ф а з- ной четырехпровод- ной схемой, соеди- ненной «звездой». Здесь потребитель элек- трической энергии, питаю- щийся трехфазным током, соединен также по схеме «звезда»; его три фазы своими началами соединены в одну об- щую точку, называемую нулем приемника 0п, и концами при- соединены к линейным проводам. Нулевая точка генератора 0г соединена с нулевой точкой приемника 0л нулевым проводом. На рис. 2Й8 показана обычная схема четырехпроводной трехфазной системы, соединенной «звездой»; здесь стрелками обозначены направления положи- тельных токов. Условно будем считать, что положительные токи в линейных проводах направлены от генератора к приемнику, а в нулевом проводе — от приемника к генератору. В соответ- ствии с этим для нулевой точки генератора или приемника пер- вый закон Кирхгофа выразится следующей формулой: Ч* ^2 Ч* “7" Ah т. е. мгновенное значение тока в нулевом про- воде равно алгебраической сумме мгновенных т о к о в в ф а з а х тр ех ф а з н о й це п и. 466
Рис. 288. Схема четырехпроводной трех- фазной системы, соединенной „звездой4* Для действующих зна- чений токов первый за- кон Кирхгофа можно вы- разить формулой 4+ 4 +Л = 4 (413) т. е. действующее значение.тока в ну- левом проводе' рав- но геометрической сумме действую- щих значений то- ков в фазах трех- фазной системы. Так как все токи в трехфазной системе одинаковой частоты, то их можно изобразить на одной и той же векторной диаграм- ме. На рис. 289 показана векторная диаграмма токов и электро- движущих сил соединенной «звездой» трехфазной системы с ну- левым проводом при неравномерной нагрузке фаз. Ток /0 на век- торной диаграмме получен путем геометрического сложения векторов токов Л, /2 и /3. Сопротивление фаз генератора обычно мало, а поэтому и падение напряжения в них обычно незначительно. В силу этого трехфазную систему можно считать симметричной не только по отношению к электродвижущим силам в фазах генератора, но и по отношению к напряжениям на зажимах генератора, т. е. можно положить, что напряжения на зажимах трехфазного гене- ратора по величине равны: ult = uu = ust = ut. Если пренебречь падением напряжения в линейных проводах и нулевом проводе, то напря- Рис. 289. Векторная диаграмма токов и электродвижущих сил в трехфаз- ной системе, соединенной „звездой4* жения на зажимах для потре- бителя будут равны напряже- ниям на зажимах фаз генера- тора. Следовательно, при сим- метричной системе Ц = Ц = г73 = £/г, где С/i, U2 и U» — напряжения на фазах потребителя. Наличие нулевого провода в трехфазной системе, соединенной «звездой», обеспечивает независимость режи- ма работы одной фазы 30* 467
Рис. 290. Схема включения освети- тельной нагрузки в трехфазную четырехпроводную систему, соеди- ненную „звездой* потребителя от дру- гой фазы, так как при малом падении напряжения в проводах напряжения на фазах потребителя относи- тельно мало изменяются с изменением нагрузки фаз. Поэтому там, где нагрузка фаз неравномерна, как Пра- вило, применяется соединен- ная «звездой» трехфазная система с нулевым прово- дом. На рис. 290 приведена схема включения осветитель- ной нагрузки в трехфазную четырехпроводную систему, соединенную «звездой»; лампы по- казаны включенными между линейными проводами и нулевым проводом. Если известны сопротивления отдельных фаз приемника и напряжения на их зажимах, то по формуле закона Ома можно определить токи в отдельных фазах приемника и затем ток в ну- левом проводе: 7-А. Z—A- Т -UL Zx> Z2 ’ — 23 И _ _ _ __ Л + 4 + 4 = 4, где /ь/2и/3 — векторы токов отдельных фаз приемника; Zo—вектор тока нулевого провода; Ub U2 и 17% — напряжения на фазах приемника; Zb Z2 и Z3— сопротивления фаз приемника. Пример 174. Напряжения на фа-1 зах приемника, соединенных «звез- дой» с нулевым проводом, одинаковы и равны (7=120 в. Сопротивления фаз приемника активные и равны: И = 10 ом, г2=15 ом и гз = 20 ом. Определить токи в фазах приемника и нулевом проводе. Решение. Находим токи в фа- зах приемника: 120 10 4 = U 12 а; 120 15 = 8 а\ Рис. 291. Векторная диаграмма то- ков U _ 120 /а - г3 20 = 6 а. 468
Токи совладают йо фазе с й выражениями в фазах, а поэтому они сдвй- нуты по фазе относительно друг друга на угол 120°. Построим векторную диаграмму токов 71, /2 и /3 (рис. 291) и, спроектировав векторы токов на взаимно перпендикулярные оси х и у, найдем: Ix в 1\х + 1%х 4“ /зх = 04-/2* cos 30° — /3j cos 30° =» = 8 .^-6-4 = >,73.. /у = /1у + fly + /Зу = Л — /2-cos60° — /3-cos60° = 12 —8-0,5 —6-05 = 5 а. Отсюда по теореме Пифагора найдем, что ,0 = ]/ 4 + 4 = /t732 4- 52 = 5,3 а. Во всякой трехфазной системе различают линейные и фазо- вое напряжения и токи. Линейным напряжением (£/л) называется напряжение между линейными проводами (линей- ными зажимами), а линейным током (/л) называется ток, протекающий по линейному проводу (рис. 288). Фазовым напряжением (£7ф) назы- вается напряжение на фазе, т. е. напряжение между линей- ным и нулевым проводами в схеме, соединенной «звездой», а фазовым током Оф) — ток, протекающий по фазе ге- нератора или приемника. Из рис. 288 видно, что линейный ток в трехфазной системе, со- единенной «звездой», равен току фазовому, т. е. . Ал1==Аф1> Ал 2 ^ф2> АлЗ АфЗ*(414) Рис. 292. Схема трехфазной цепи соединенной „звездой", без нулевого провода Выясним соотношение между линейными и фазовыми напря- жениями. Линейное напряжение между первым и вторым линейными проводами обозначим t/li2, между вторым и третьим — £72,з и между третьим и первым —6/зл (рис. 292), где порядок индексов согласован с положительным направлением линейных напряже- ний. Например, положительное направление линейного напряже- ния £71>2—от зажима 1 к зажиму 2, линейного напряжения £72,з — от зажима 2 к зажиму 3 и линейного напряжения £/зл— от’зажима 3 к зажиму /. Обозначим мгновенные значения по- тенциалов точек 0, 1, 2 и 3 соответственно через <Pi, ?2 и Мгновенное значение линейного напряжения между каждой парой линейных проводов равняется алгебраической разности мгновенных значений потенциалов соответствующих линейных зажимов (Л 2 и 3): 2 — w2, з — ?2 ?з‘, йз, 1 — <Рз 469
Мгновенное значение фазового напряжения равняется алгеб- раической разности мгновенных значений потенциалов точки (зажима) конца фазы и нулевой точки (нулевого зажима): щ —?0; щ = ?2—«з = ?3—то, где и2 и из — мгновенные значения фазовых напряжений соот- ветственно первой, второй и третьей фаз. Отсюда находим, что ?1=«1—?о; т2=и2—То; Тз=—То- Принимая во внимание последние соотношения, мгновенные значения линейных напряжений можно выразить следующими формулами: й1,2 == Ti Тг “ (^1 То) (й2 То) = и2, 3 = ?2 Тз “ (#2 То) (й3 То) ~ и2 й8> из, 1= Тз Ti= (йз То) (% То) =аз т. е. мгновенные значения линейных напряжений равны алге- браической разности мгновенных значений соответствующих фа- зовых напряжений. Но если мгновенные значения линейных напряжений равны алгебраической разности фазовых напряжений, то действующее значение линейных напряжений равняется геометрической раз- ности соответствующих фазовых напряжений. Следовательно, для определения действующих значений ли- нейных напряжений надо на векторной диаграмме произвести действия геометрического вычитания над соответствующими век- торами фазовых напряжений (рис. 293): Ц,2 = Ц-^2; ^3 = ^2-^ ^1==^з-Ц. (415) Для определения^инейного напряжения 4/л1>2 надо из__вектора вычесть вектор £7ф2, т. е., иначе говоря, к вектору £/ф1 при- бавить вектор—£7ф2 (рис. 293). Проведя вектор_— С/ф2, равный и противоположный по направлению вектору С7ф2, сложим его геометрически с вектором С7ф1. Полученная диагональ паралле- лограмма даст нам величину_и положение на векторной диаграм- ме линейного напряжения С7лЪ2, т. е. напряжения между первой и второй фазами. Проектируя обе стороны параллелограмма £/ф1 и —TJqi на его диагональ, получим результирующий вектор, т. е. вектор линейного напряжешгя. Аналогично этому построены векторы линейных напряжений и ил2Л. Если трехфазная система симметрична относительно напря- жений, т. е. С7ф1 = 67ф2 = С7фз = С/ф, и они сдвинуты по фазе относительно друг друга на угол -у- -тс, то линейное напряжение 470
будет в У 3 раз больше напряжения фазового. Это соотноше- ние легко получить из векторной диаграммы (рис. 293), из! кото- рой следует, что ^л1,2 = Ц>1 • cos 30° + иф2 • cos 30° = 2Ц, • = Уз иф. Итак, для симметричной относительно напряжений трехфаз- ной системы, соединенной «звездой», ия = Узиф. (416) Пример >175. Определить линейное напряжение в трехфазной, симметрич- ной относительно напряжений системе, если t/ф = 127 в. Решение. По формуле (416) находим ив = Из иф = Уз • 127 = 220 в. Нагрузка на фазах трехфазной системы называется равно- мерной, если токи во всех фазах одинаковы по величине и сдви- нуты по фазе на одинаковые углы относительно своих напряже- ний, т. е. сопротивления фаз одинаковы по величине и по харак- теру: / Z, = Z2 === Z3 = Z. 471
Следовательно, если при равномерной нагрузке фаз 4==Z„-sina)/; /2=/OT-sin(®Z---g-тс); Z, = /OT-sin(arf — TO ' A + i2 + h =Im’ [sin+ sin (atf— 120°) + sin (®t— 240°)] = — Im-(sin a)/+ sin ю/.cos 120° — cos atf-sin 120° + + sin a)/, cos 240° — cos <d/• sin 240°) = = /m*^sin(of—0,5- sin a)/—^~cos«)/—0,5-sina^+ - cos =0, т. e. алгебраическая^сумма мгновенных значе- ний токов в трехфазной системе, соединенной «звездой», при равномерной нагрузке фаз равна нулю. Но если сумма мгновенных значений токов равна нулю, то геометрическая сумма Действующих значений токов при равномерной нагрузке фаз также будет равна нулю: А + А 4- А = А== 0. Эту формулу легко проверить путем построения векторной диаграммы токов (рис. 294). Если результирующий ток в нуле- вом проводе при равномерной нагрузке фаз равен нулю, то отпа- дает необходимость в самом нулевом проводе. В этом случае можно ограничиться только тремя проводами, т. е. перейти к трехфазной трехпроводной системе. Это даст большую экономию меди при передаче одной и той же мощности потребителю. При равномерной нагрузке фаз трехфазная система симме- трична относительно напряжений и линейное напряжение стано- вится в ИЗ раз больше фазовых. Трехфазную трехпроводную систему, соединенную «звездой», применяют там, где нагрузка фаз строго равномерна, например, когда к трехфазной сети подключается трехфазный электродви- гатель (рис. 295). Если же. при трехфазной трехпроводной си- стеме, соединенной «звездой», нагрузка оказывается неравномер- ной, то это ведет к перераспределению напряжений на фазах приемника в соответствии с их нагрузками и система перестает быть симметричной. В этом случае могут .возникнуть перенапря- жения на отдельных фазах приемника. Например, если в соеди- ненной «звездой» трехфазной системе. без нулевого провода, 472
I Рис. 295. Схема трехфа^ной трехпроводной системы, соединенной „звездой* Рис. 294. Геометрическая сумма действующих зна- чении токов при равно- мерной 'нагрузке фаз равна нулю равномерно нагруженной, одна из фаз приемника случайно зако- ротится, то на остальных фазах приемника напряжение возра- стет в У 3 раз против нормального, т. е. фазовые напряжения станут равными по величине линейным напряжениям. Пример 176. В трехфазной трехпроводной системе фазовое напряжение' на каждой фазе потребителя равно [7=127 в, если фазы нагружены равно- мерно. Определить, как оно распределится на фазах приемника, если пер- вая из фаз приемника окажется закороченной. Решение: Напряжение на первой фазе U\ = 0, так как сопротивление фазы равно нулю. Напряжения на второй и третьей фазах будут равны; и2 = и3 = иуъ = 127-уз = 220 в. § 131. ТРЕХФАЗНАЯ СИСТЕМА, СОЕДИНЕННАЯ «ТРЕУГОЛЬНИКОМ» При соединении «треугольни- ком» обмотки фаз генератора со- единяются так, что начало каж- дой из фаз соединяется с концом предыдущей фазы и к точкам со- единения фаз присоединяются ли- нейные провода (рис. 296). На рис. 297 показана схема трехфазной системы, в которой обмотки фаз генератора и фазы потребителя соединены «треуголь- ником». Из рис. 297 видно, что фазы генератора образуют зам- кнутый на себя контур. Однако если трехфазная система сим- метрична, то для любого момента Рис. 296. Схема трехфазпой си- стемы, соединенной „треуголь- ником* 473
времени в треугольнике фаз генератора алгебраическая сумма мгновенных значений электродвижущих сил равна нулю, так как (2 \ ш/---з~^ + + --Т 1')==0. а поэтому опасности короткого замыкания фаз генератора здесь нет. В несимметричной трехфазной системе при разомкнутой внешней цепи в треугольнике фаз генератора проходит ток, вели- чина которого зависит от того, насколько несимметрична си- стема. Условимся в схемах трехфазной системы, соединенной «тре- угольником», положительными направлениями токов в фазах ге- нератора (рис. 297) считать направления против движения часо- Рис. 297. Схема генератора и потребителя трехфазной системы, соединенной „треуголь- ником* вой стрелки. линейных проводах положительными направле- ниями токов будем считать направления от генератора к прием- нику, а в фазах приемника — направление движения часовой стрелки. В соответствии с этим уравнения первого закона Кирхгофа для вершин (/) треугольников фаз генератора и приемника мо- жем записать следующим образом: Аг == Ал “Ь Аг И Ап = Ал 4“ Ап И Т. Д., откуда Ал= Аг Аг и Ал= Ап Ап, (417) т. е. ток в первом линейном проводе равен геометрической раз- ности токов первой и третьей фаз генератора или первой и тре- тьей фаз приемника. 474
Аналогично этому Ал Аг — Дг И Ал = Ап Ап> Ал ~ Аг Аг и Ал== Ап Ап- Следует обратить внимание на то, что для определения линейных токов надо брать 1зл геометрическую разность по- следующего и предыдущего то- ков, например, чтобы опреде- лить второй линейный ток /2л, надо из второго фазового то- ка /2г вычесть геометрически первой фазовый ток /1г и т. д. При соединении «треуголь- ником» каждая фаза прием- ника находится под линейным Рис. 298. Векторная диаграмма токов трехфазной системы, соединенной „треугольником* Напряжением, а поэтому, если пренебречь падением напряжения в проводах, всякое изменение нагрузки в той или иной фазе потребителя не влияет на величину напряжений в других его фазах. Это — одно из основных достоинств таких систем. При соединении «треугольником» линейное и фазовое на- пряжения равны друг другу, что очевидно из построения самой схемы, где фазовое и линейное напряжения — по сути дела одно и то же. На рис. 298 приведена векторная диаграмма токов трехфаз- ной системы, соединенной «треугольником», и доказаны линей- ные токи как геометрические разности фазовых токов. Исходя из векторной диаграммы, можем написать, что h Рис. 299. Векторная диаграмма токов и напряжений трех- фазной системы, со- единенной „треуголь- ником* 4л=Ar-cos 30° + + /Зг • cos 30° = 2Zr *у - Уз Zr, где /1л — линейный ток в линейном проводе первой фазы; /1Г, 4г — фазовые токи соответственно в первой и третьей фазах генератора. Аналогично /2л=Гз’/г и /3л=Уз’/г> т. е. при равномерной нагрузке фаз тре- угольника линейные токи в ИЗ раз больше фазовых токов: /л = Из"/ф, (418) 475
где 7Л—линейный ток; /ф— фазовый ток. На рис. 299 показана векторная диаграмма токов и напря- жений. Линейные напряжения представлены здесь в виде замк- нутого равностороннего треугольника, а фазовые токи показаны отстающими от напряжений по фазе (случай индуктивных на- грузок фаз). Геометрическая сумма линейных токов для любой нагрузки фаз равна нулю: 71л + 72л + 73л = (7, —73) 4- (72 — 4) 4- (-73 — 72) = 0. Этого, конечно, нельзя сказать про токи фазовые, таи как геометрическая сумма фазовых токов «треугольника» равна Рис. 300. Схема включения освети- тельной нагрузки в трехфазную си- стему, соединенную „треугольником* нулю только при равномерной нагрузке фаз. . Если заданы напряжения на фазах приемника и сопро- тивления фаз, то величина тока в фазах определится по закону Ома: г _ _ и, . ,17, А — > А — Zt • '3~ Z3‘ На рис. 300 показана схема включения осветительной на- грузки—электрических ламп— «треугольником». Пример 177. Нагрузка соединена «треугольником», и сопротивления фаз равны между собой: г( = г2 = г3 = Ц ом. Определить фазовые и линей- ные токи в этой трехфазной системе, если линейное напряжение у потре- бителя £7л=220 в.. Решение. Фазовое напряжение п.ри соединении «треугольником» равно линейному напряжению: Фазовые токи (7-77л-220 в. , , , U 220 „ h °” А •= h ж -у “ ff “ 20 а. Линейные токи Лл-/2л = /8л“ /3.20 = 34,6 а. Соединение «треугольником» выгодно применять для потре- бителей в том случае, когда необходимо на его фазы подать более высокое напряжение. Например, если трехфазный асин- хронный электродвигатель включить «звездой», каждая из его фаз будет под напряжением 127 в, а в случае соединения «тре- угольником» — 220 в, что значительно повысит мощность элек- тродвигателя. 476
§ 132. МОЩНОСТЬ ТРЕХФАЗНОГО' ТОКА Мгновенная мощность трехфазной системы равна сумме мгновенных мощностей отдельных фаз: Р = А + А + А = «Л + «2^2 + (419) Если все фазы нагружены равномерно, то р = Um-sin (<о/ —<р) +• + Um• sin(otf--g-id /m-sin (ю/-g-к— <p)4- (4 \ /4 \ 0)Z-------------------g-z — cpl. После ряда тригонометрических преобразований, которые мы здесь опускаем, получим Р = "2” ^nJm ’ COS ?> или р = 3UI • cos (420) Из формулы (420) следует, что мгновенная мощ- ность равномерно нагруженной трехфазной системы не зависит от времени, т. е. она по- стоянна. Многофазная система, в которой мгновенная мощность не зависит от времени, называется с истемой уравновешен- ной. Следовательно, равномерно нагруженная трехфазная си- стема — это система уравновешенная. Если мгновенное значение мощности не зависит от времени, то среднее значение мощности равно ее мгновенному значению: P = 3L//-cos<^ (421) При соединении «звездой» = и = а поэтому Р — У'З (7л7л-соз<р. Аналогично при соединении «треугольником» /ф=-^ и (7Л = Ц>, 477
а поэтому Р=Уз £7л7л-со8<р, (422) где <р — угол сдвига фаз тока и напряжения в фазах приемника. Пример 178. В трехфазной системе, соединенной «звездой» и нагружен- ной равномерно, линейное напряжение ил = 120 в, а линейный, ток/л = 15 а. Определить мощность, развиваемую током в этой системе, если коэффициент мощности' cos = 0,8. Решение. По формуле (421) находим Р = уз t/A-cos? = 1,73-120-15-0,8 = 2491 вт. Пример 179. Электродвигатель трехфазного тока с полезной мощностью Р\ = 25 л. с., коэффициентом полезного действия 7] = 83,5% и коэффициентом мощности cos ср = 0,8 подключен к трехфазной сети, в которой линейное на- пряжение 1/л = 220 в. Определить,, какой ток потребляет данный электродви- гатель при полной нагрузке и под каким напряжением находятся его фазо- вые обмотки, соединенные «звездой». Решение. Полезная мощность электродвигателя Л = 35-736 = 18 400 вт. Полная мощность, потребляемая электродвигателем, Л _ 18400 Р — — Л QQt- Оио вт. т] 0,835 Линейный ток Р 22 036 уз ил- cos ф уз -220-0,8 = 72,4 а. Фазовое напряжение /з 220 1,73 е 127 в. § 133. ВРАЩАЮЩЕЕСЯ МАГНИТНОЕ ПОЛЕ ТРЕХФАЗНОГО ТОКА Допустим, что три одинаковые плоские катушки расположены в про- 2 странстве так, что их оси составляют углы, равные -g-к (рис. 301). Катушки неподвижно закреплены и находятся под напряжением трехфазной сети, так что токи в них равны /1 " <»>/; Z2 = /OT.sin(<o/ —-у-я); (4 \ со/ — -у- я ) . О / - 478
Если в катушках нет стальных сер- дечников, то магнитаый поток в каждой из них изменяется - пропорционально току, т. е. 0J == Ф^-sin coft Ф5 = Фт-5т фэ = фт'£‘п Определим результирующий магнит- ный поток, создаваемый системой из трех заданных катушек, питающихся трехфазным током. Как известно, положительный ток в катушке и положительный магнит- ный поток; создаваемый током катушки, связаны правилом правого винта. Если вращательное движение винта совпадает Рис. 301. Три катушки располо- жены в пространстве под углом 120° одна относительно другой с направлением положительного тока в витках катушки, то поступательное движение винта совпадает с на- правлением положительного магнитного поля в катушке. На рис. 301 по- ложительные направления токов обозначены крестиками и точками в сечении проводов катушек. Крестик показывает, что ток направлен от нас за пло- скость рисунка, а точка — что он направлен из-за плоскости рисунка к нам. В соответствии с этим положительные магнитные потоки катушек будут сдвинуты пространственно относительно друг друга на угол 120°. На рис. 302 эти магнитные потоки представлены в виде векторов Ф1/и, Ф2/я и Ф3/и (век- торов амплитуд магнитных потоков). Эти векторы, расположенные по отноше- Рис. 302. Векторы положительных магнитных потоков сдвинуты на 120° один относительно другого 479
нию друг к другу под углом 120°, т. е. так же, как осп катушек, не явля- ются вращающимися векторами в системе катушек, так как каждый из них направлен по оси своей катушки и может менять свое направление только на противоположное. Однако заданные магнитные потоки изменяются по за- кону синуса, а поэтому каждый из них периодически меняет свое направле- ние в катушке на противоположное. Чтобы определить результирующее магнитное поле в системе, надо сло- жить векторы геометрически. Для этой цели разложим каждый вектор магнит- ного потока на две составляющие, из которых одну направим по оси х, другую — по оси у (рис. 302). При этом полагаем, что положительная полу- ось Ох направлена в сторону вектора Ф^, т. е. положительного потока пер- вой катушки, а положительная полуось Оу направлена перпендикулярно оси х и против движения часовой стрелки от положительной полуоси Ох. Разложив векторы амплитуд магнитных потоков Ф^, Ф2/д и Ф3/л по осям х и у, получим * COS 0 Ф1/7Р mx = -<^<os60° = -%; (фзт)х = — фзт • cos 60° = — (ф1Л=ф1т-соз 90° = 0; (Ф2т)у = — Ф2т • COS 30° = —Ф2т; (фз Д = + ф3т • COS 30° = Ф3и. Но каждый из магнитных потоков синусоидален, а поэтому мгновенные зна- чения составляющих магнитных потоков будут определяться следующими выражениями; (ф1)л = ф)т-81п со/; (фг)х *= (Ф2,Л • sin (со/ — -у- «) = — • sin (a>t — ~ к); (фз)х = (ф3т)л • Sin (со/ — у К ) = — • sin (со/ — Д- к) . Для определения мгновенного значения результирующего магнитного по- Хока необходимо последние три равенства сложить алгебраически. Принимая во внимание, что по условию | Ф^ | = ГФ2Л? | = | Фа/И | = Ф^, т. е. численные значения амплитуд магнитных потоков равны друг другу, Получим (Фо)х ==Фт-8Ш • sin (----5- к)---^--sinful/---i-v) = = Фт^зш со/—^-(sin со/, cos 120° — cos со/, sin 120°) — — -у (sin со/ - cos 240° — cos сб/. sin 240°) J = Л . 1 . . , уз . . 1 . , уз Д = ФД81ПСо/ 4- — • sin со/ cos со/ 4- -у sin со/— cos со/), 480
или, иначе, (Фо1г = 1Л Ф^-sin со/, где (Ф0)д. — составляющая результирующего магнитного поля по оси х. Ана- логично определяем составляющую результирующего магнитного поля но оси у: (Ф1\ = ^ijy-sin <о/ = 0, так как (Ф1/72)у = 0; (2 X 1/3 / 2 ~з"К)------2"^m-sin^0,C—-у * (Фз)у = (фзт)у • sin (<*t — «) = Ф3яг • sin (at — -у ") Следовательно, (Ф0)у = (Ф1)у + (Ф2)у + (фз)у = 0 - iy Ф^-sin «) + + -Гу Фт-sm (W— -у к), или, иначе, (ФО)У = ¥£-Фт [sin(<0t - 240°) - sin(o>f - 120°)] = = ®m(sin<i>f-cos240° — cosшбsin240° —sin<o6cos 120° + cos<o/-sin 120°) = 1/3 Л ( 1 . . , /3 , , 1 . , , уз Д •= Фот f — -y sin at + -^y- • COS at + у Sin at + -?-y- • COS at j, откуда (Ф0)у = 1,5Ф,„- cos cot Следовательно, составляющие результирующего магнитного поля имеют одинаковые амплитуды, равные Фо = 1,5Ф^. Однако составляющая по оси х изменяется по закону синуса, а составляющая по оси у — по закону ко- синуса. Чтобы получить результирующий магнитный поток для любого момента времени /, необходимо геометрически сложить мгновенные значения состав- ляющих результирующего магнитного поля по осям х и у. Так как эти со- ставляющие пространственно расположены под углом 90° относительно друг друга, то будем складывать их по правилу прямоугольного треугольника (рис. 303): Фо - V(Фо)ж + (фо)^ = 1/(1.5Фет-Sin at)* + (1,5Фт.СО5 aty, или Фо - 1,5ФЛ. (423) Из полученной формулы следует, что величина результирующего маг- нитного потока не зависит от времени и в 1,5 раза больше амплитуды лю* бого из магнитных потоков, создаваемых заданными катушками. 31—1377 481
Теперь определим, какое же Направление в пространстве имеет резуль- тирующий магнитный роток. Из рис. 303 видно, что вектор Фо составляет с положительной полуосью Ох угол а, который определим из формулы или, иначе, tga = (Фр)у (Фр)х 1,5Ф^-С08 со/ 1,5Ф^-8т со/ tg а = Ctg со/ откуда « « у — <о/, т. е. угол, составленный вектором Фо результирующего магнитного потока с положительной пблуосью Ох, изменяется пропорционально времени. Рис. 303. Величина и на- правление результирую- щего магнитного потока Ф Рис. 304. Вращающийся магнит при своем движении увлекает за собой металлический диск В момент времени t = 0 а = у, т. е. вектор Фо по своему направле- нию совпадает с положительной полуосью Оу, В момент времени, когда . тс тс тс _ , ы = у » а = 2— у = 0, т. е. вектор Фо совпадает по направлению с по- те ложительной полуосью Ох. В момент времени, когда w/ = it, а= —я = тс _ и = —2, т. е. вектор Фо совпадает по направлению с отрицательной полу- осью Оу, и т. д. Это означает, что вектор Фо результирующего магнитного потока, оставаясь неизменным по величине, вращается по часовой стрелке с постоянной угловой скоростью io. Следовательно, за один период тока вектор Фо делает один полный оборот в системе из трех заданных кату- шек, питающихся трехфазным током. Магнитное поле подобного рода назы- вается вращающимся магнитным полем трехфазного тока. Рассмотренное нами магнитное поле двухполюсное, оно напоминает маг- нитное поле стержневого магнита, вращающегося вокруг своей поперечной оси, проходящей через его середину. 482
Для того чтобы переменить направление вращения магнитного поля в системе катушек, питающихся трехфазным током, надо поменять местами концы проводов каких-либо двух катушек, например, первую катушку, ра- нее подключенную к первой фазе генератора, подключить к его второй фазе, а вторую катушку, ранее подключенную ко второй фазе генератора, подключить к первой его фазе. Вращающиеся магнитные поля широко используются на практике, на- пример, в асинхронных электродвигателях переменного тока, индукционных электроизмерительных приборах, с которыми подробно ознакомимся впо- следствии. Если во вращающееся магнитное поле внести металлическое тело, на- пример металлический диск, могущий вращаться вокруг своей оси, то маг- нитное поле, перемещаясь относительно диска, будет индуктировать в нем вихревые токи. Благодаря взаимодействию этих токов с вращающимся магнитным полем диск начнет вращаться в ту же сторону, что и магнит- ное поле. Вращающееся магнитное поле при своем движении как бы увле- кает за собой диск. На рис. 304 показан простейший опыт подобного рода. Если вращать постоянный магнит перед диском, то диск также будет вра- щаться в ту же сторону, что и магнит. Так как металлический диск обла- дает некоторой инерцией, т. е. создает сопротивление своему движению, то, естественно, число его оборотов будет меньше, чем число оборотов вращаю- щегося магнитного поля. Чем тяжелее диск, тем больше сопротивление его движению и тем меньше число его оборотов по сравнению с оборотами вращающегося магнитного поля. Такое движение диска по отношению к движению вращающегося магнитного поля называется асинхронным, т. е. не совпадающим по числу оборотов. § 134. ДВУХФАЗНОЕ ВРАЩАЮЩЕЕСЯ МАГНИТНОЕ ПОЛЕ Двухфазным током называется совокупность двух однофазных токов, сдвинутых по фазе относительно л друг друга на у г о л у, т. е. z’i = Im'sin <dZ; Предположим, что две одинаковые плоские катушки расположены в про- страйстве так, что их оси взаимно перпендикулярны (рис. 305). Если такая система катушек питается двухфазным током, то в ней создаются два магнитных потока: Ф1 = Ф^-sin coZ; Ф3 = Фот 8т — y i Эти магнитные потоки пространственно расположены относительно друг друга под углом 90°, а поэтому результирующий магнитный поток будет равен их геометрической сумме Фо=у (®OT-sina>0* + ®OT-sin(®< —-у)2 • sin Г <oZ-------у j = — • cos a)Z? Но 31* 483
а поэтому Фо = у (Ф,„- SIH шО2 + ( —Ф,л-соь «>/)’, ИЛИ Фо - Фл>. (424) т. е. результирующий магнитный поток двух заданных катушек, питающихся двухфазным током, не зависит от времени и численно равен максимальному магнитному потоку любой из заданных двух катушек. Рис. 305. Две ка’тушки расположены Рис. 306. Величина и направление в пространстве под углом 90° одна результирующего магнитного по- к другой тока Фо Теперь определим, какое направление имеет результирующий магнит- ный поток Фо в пространстве. Из рис. 306 видно, что . — Ф^-COSCD^ л 6 Ф^-Sina)/ ’ или, иначе, tg“ = tg т)' Отсюда находим, что а = at — А т. е. угол а, составленный вектором Фо с положительной полуосью Ох, из- меняется пропорционально времени. При t = 0 а = —90°, т. е. вектор Фо совпадает с направлением отри- цательной полуоси Оу. В момент, когда йЛ = 90°, а = 90®— 90° = 0, т. е. в этот момент времени вектор Фо совпадает с направлением положительной полуоси Ох. В момент, когда о)/=180°, а =180° — 90° = 90°, т. е. в этот момент времени вектор Фо совпадает с направлением положительной полу- оси Оу и т. д. Это означает, что вектор Фо результирующего магнитного 484
поля, сохраняя свое численное значение неиз- менным, вращается против часовой стрелки с постоянной угловой скоростью со. Следова- тельно, вектор Фо за один период тока делает один оборот в системе катушек, питающихся двухфазным током. Магнитное поле подобного рода называется вращающимся магнитным полем двухфазного тока. Для получения двухфазного вращающего- ся магнитного поля чаще всего прибегают к системе двух параллельно соединенных кату- шек, из которых одна обладает большим ак- тивным сопротивлением, а другая — большой индуктивностью (рис. 307). В таких катушках • я токи сдвинуты по фазе примерно на угол Рис. 307. Схема соединения катушек для получения двухфазного вращающегося магнитного поля по отношению друг к другу, и если катушки к пространственно расположены под углом у, то в системе их возникает двухфазное вращающееся магнитное поле. Вращающееся магнитное поле двухфазного тока применяется в некото- рых измерительных приборах переменного тока, с которыми мы ознако- мимся впоследствии.
ГЛАВА XXVI КАТУШКИ И ТРАНСФОРМАТОРЫ СО СТАЛЬНЫМИ СЕРДЕЧНИКАМИ § 135. ОБЩИЕ ПОЛОЖЕНИЯ В электротехнических установках весьма часто приходится встречаться с такими цепями переменного тока, в которых вклю- чены катушки со стальными сердечниками, например дроссель-, ные катушки, трансформаторы, автотрансформаторы, разного рода электромагнитные механизмы и т. д. Электромагнитные процессы в таких катушках сложнее, чем в катушках без стальных сердечников. Если в катушке без стального сердечника Мощность потерь имеется только в активном сопротивлении обмотки, то в катушке со стальным сердечником возникает еще мощность потерь на перемагничивание стали и на вихревые токи, индуктированные в ней. Если в катушке без стального сердечника существует про- порциональная зависимость между токами и магнитным потоком, то в катушке со стальным сердечником эта зависимость, изобра- жаемая графически циклической кривой перемагничивания стали (петля гистерезиса), не пропорциональная. А это значит, что при синусоидальном напряжении, приложенном к катушке со сталь- ным сердечником, ток в ней в общем случае несинусоидален. Следовательно, катушка со стальным сердечником искажает си- нусоиду тока. Если изменять положение стального сердечника в катушке или величину воздушного зазора в нем, то параметры катушки будут изменяться и, в частности, будет резко изменяться индук- тивность катушки. А это значит, что при неизменном действую- щем значении синусоидального напряжения, приложенного к ка- тушке, можно изменять в ней в широких пределах режим путем изменения величины воздушного зазора* в сердечнике, 486
§ 136. МОЩНОСТЬ ПОТЕРЬ НА ПЕРЕМАГНИЧИВАНИЕ СТАЛИ Перемагничивание стали связано с потерей энергии, так как магнитные силы, стремящиеся ориентировать элементарные маг- нитики в стали, встречают с их стороны некоторое противодей- ствие повороту. I Мощность потерь на перемагничивание стали зависит от ве- личины максимальной магнитной индукции Вт от числа циклов f перемагничивания в секунду, от веса G и сорта стали. о .в-сек Если максимальная магнитная индукция не превышает , то мощность потерь на перемагничивание стали можно опреде- лить по формуле pnep = [a i£j Вт + р (Ви)’] G, (425) где Рпер — мощность потерь на перемагничивание стали в ват- тах; Q — вес стали в килограммах; Вт — максимальная магнитная индукция в вольт-секун- дах на квадратный метр; /—число циклов перемагничивания в секунду; а и р — коэффициенты, учитывающие сорт стали, значения которых приведены в табл. 34. Если максимальная магнитная индукция в стали превышает , в-сек 1 ~м2 , то мощность потерь на перемагничивание можно опре- делить по формуле Pnep = a^^G, (426) где а — коэффициент, учитывающий сорт стали, значение кото- рого приведено в табл. 34. Таблица 34 Значение коэффициентов, учитывающих сорт стали Сорт стали Толщина листа стального сердеч- ника, мм а Р а Сталь динамная 1,0 0,9 3,5 4,4 0,5 0,9 3,5 4,4 0,35 0,9 3,8 4,7 Сталь трансформаторная 0,35 0,3 2,1 2,4 0,5 0,4 2,6 3,0 487
Пример 180. Определить мощность потерь на перемагничивание сердеч- ника из трансформаторной стали, если максимальная магнитная индукция п Л в-сек в сердечнике Вт = 1,2 —, число циклов перемагничивания стали в секунду f = 50, вес железа G = 10 кг и толщина стальных листов сердечника d = 0,5 мм. Решение. По табл. 34 для трансформаторной стали при толщине ее листов d = 0,5 мм коэффициент с = 3,0. По формуле (426) находим Рпер = «B^G = З--^ 1.2М0 = 21,6 вт. § 137. МОЩНОСТЬ ПОТЕРЬ НА ВИХРЕВЫЕ ТОКИ Если в переменное магнитное поле внести массивный кусок металла, то в нем будет индуктироваться электродвижущая сила и появятся индуктированные токи. Эти токи будут рас- пространяться по массивному проводнику в виде замкнутых вих- ревых токов. Согласно правилу Ленца вихревые токи, как токи индуктированные, противо- действуют причине, вызвавшей их. Сле- довательно, направление их таково, что они стремятся противодействовать всем изменениям внешнего магнитного поля, индуктирующего их. На рис. 308 пока- зано направление вихревых токов в мас- сивном проводнике при АФ>0, т. е. при возрастании внешнего магнитного потока, пронизывающего проводник. Так как сопротивление массивного проводника очень мало, то вихревые токи в проводнике могут достигнуть очень большой величины, если не принять мер борьбы с ними. Вихревые токи чаще всего бывают паразитными, бесполезно поглощающими электрическую энергию, которую они превра- щают в тепловую энергию. Одна из самых рациональных мер борьбы с вихревыми то- ками состоит в расслоении массивных проводников; при этом вредное влияние вихревых токов сводится к минимуму. Сердеч- ники трансформаторов, якорей генераторов обычно собирают из тонких листов железа (стали) толщиной 0,35—0,5 мм, изолиро- ванных друг от друга тонким слоем лака или какой-либо другой изоляции. Железные сердечники в катушках очень часто изго- товляют из пучка железных прутьев, изолированных друг от друга тем или иным способом. Чтобы вихревые токи были мини- мальными, поверхности железных листов сердечников распола* гают параллельно магнитным линиям. 488 Рис. 308. Вихревые токи, индуктированные в мас- сивном проводнике при АФ >0
Незначительная примесь кремния в железе (несколько про- центов) увеличивает удельное сопротивление железа и этим уменьшает вихревые токи. ..В некоторых случаях вихревые токи применяются как токи полезные, например, в металлургии при плавке металлов, при электрическом торможении подвижных частей некоторых измери- тельных приборов. Мощность потерь на вихревые токи в расслоенных стальных сердечниках зависит от максимальной магнитной индукции, от числа циклов изменения магнитной индукции в секунду, от тол- щины стальных листов сердечника и от веса его: f = °в. Т ^tnj Q (427) где Рв, т— мощность потерь на вихревые токи в ваттах; Вт—максимальная магнитная индукция в вольт-секун- дах на квадратный метр; /— число циклов изменения магнитной индукции в се- кунду; G— вес железного сердечника в килограммах; ав,т—коэффициент, учитывающий материал стального сердечника, величина которого показана в табл. 35. Таблица 35 Значения коэффициента» учитывающего материал сердечника Сорт стали Толщина листа, мм °в. т 0,35 3,2 Сталь динамная | 0,5 5,6 1,0 22,4 Сталь трансфор- | 0,35 0,6 маторная [ 0,5 Пример 181. Определить мощность потерь на вихревые токи в сердеч- нике из трансформаторной стали весом G =» 10 кг, если переменная магнит- ная индукция при своих изменениях в стали достигает величины Вот= = 1,2 ) число циклов изменения магнитной индукции в секунду / = 50 и толщина листов трансформаторной стали равна 0,5 мм. Решение. Для трансформаторной стали при толщине листов 0,5 мм по табл. 35 коэффициент ав, т==1,2. По формуле (427) находим величину мощности потерь на вихревые токи / f \2 /50 V G=1’2(w-1-2) -W-4,32 em. 489
§ 138. КАТУШКА СО СТАЛЬНЫМ СЕРДЕЧНИКОМ В ЦЕПИ ПЕРЕМЕННОГО ТОКА Допустим, что к катушке со стальным замкнутым сердечни- ком приложено синусоидальное напряжение u = Um-s\\\ (urt+ 4--^) (рис. 309). Исследуем режим в этой катушке, считая вначале активное сопротивление обмотки катушки и магнитный поток рассеяния в системе катушки практически настолько ма- лыми, что ими можно пренебречь. Переменное синусоидальное напряжение, приложенное к ка- тушке, создает в ней переменный ток, в общем случае не сину- соидальный. Так как активное сопротивление обмотки катушки равно нулю(гм = 0), то все напряжение, приложенное к ка- тушке, расходуется только в ее индуктивном сопротивлении. Значит, это напряжение полностью компенсируется электродви- жущей силой самоиндукции, возникшей в катушке в результате изменений магнитного потока, создаваемого переменным током. Следовательно, в любой момент времени мгновенное значение напряжения, приложенного к катушке, должно быть численно равным и противоположным по знаку электродвижущей силе са- моиндукции: eL = — и = — Um-sin(o>t + = -у). На основании последнего равенства можно сделать вывод, что при синусоидальн ом напряжении, прило- женном к катушке со стальным сердечником, электродвижущая сила самоиндукции в ней является величиной синусоидальной, отстаю- щей от напряжения по фазе на угол л. Но электродвижущая сила самоиндукции в катушке пропорциональна скорости изме- нения магнитного поля в этой катушке, а поэтому, если электродвижущая сила са- моиндукции синусоидальна, то и магнитный поток, создающий ее, также должен изме- няться по синусоидальному закону во вре- мени и в то же время должен опережать ее по фазе на угол у- : Ф = Фт • sin <оД На рис. 310 приведены векторные и раз- вернутые диаграммы синусоидальных на- Рис. ЗОЭ. Катушка со пряжений, электродвижущей силы самоин- стальным сердечни- дукции и магнитного потока для катушки ком со стальным сердечником. 490
Ф 0 Е Рис. 310. Векторные и развернутые диа- граммы напряжения, магнитного потока и электродвижущей силы самоиндукции для катушки со сталь- ным сердечником, активное сопротивле- ние обмотки которой равно нулю Установлено, что максимальная вели- чина магнитного потока Фт в катушке за- висит от величины действующего значения синусоидального напряжения U, приложен- ного к катушке, от частоты ее f и от числа витков w катушки: Ф = (428) т 4,44/w 4 ' Пример 182. Определить максимальную величину магнитного потока (амплитуду магнитного по- тока) в катушке со стальным сердечником, если действующее значение синусоидального напряжения, приложенного к ней, (/ = 220 в, частота ее / = 50гг{ и число витков w = 1000. Решение. Применяя формулу (428), находим фл» = 4,44/те, = 4,44-50-1000 = 0,001 в'СеК‘ Теперь выясним, каким же должен быть ток в катушке, чтобы он мог создавать в ее сердечнике синусои- дальный магнитный поток Ф = Фот-8тшА Как известно, между током и магнитным потоком в катушке со стальным сердечником существует нелинейная зависимость, т. е. магнитный поток в ней изменяется не пропорционально току, а поэтому при наличии в ней синусоидального магнитного потока ток в ней должен быть несинусоидальным. Для построения графика зависимости тока от времени для катушки со стальным сердечником необходимо воспользоваться графиком, выражающим зависимость между током и магнит- ным потоком, т. е. кривой, аналогичной кривой перемагничива- ния стали. На рис. 311 с левой его стороны приведен график зависи- мости магнитного потока Ф от тока I, а с правой стороны при- ведена синусоида магнитного потока Ф, выражающая зависи- мость его от времени. Покажем, как с помощью этих двух гра- фиков можно построить график зависимости тока I от времени. На левой стороне рис. 311 находим, что магнитному потоку Ф = Ф1 = 0 соответствует ток Zi, магнитному потоку Ф2—ток t2, магнитному потоку Ф3=ФОТ— ток i2=Im, магнитному по- току Ф4 — ток Ц = 0, магнитному потоку Ф5 = 0 — ток — is и т. д. Затем на правой стороне рис. 311, где нанесена синусоида магнитного потока Ф, для каждого значения магнитного по- тока Ф откладываем на перпендикулярах к оси времени вели- чины тока I, соответствующие величинам данных магнитных по- токов. Например, для магнитного потока Ф = 0 отметим на вер- тикальной оси ток 1\, для магнитного потока Ф2 — ток t2, для Фот — ток 1т и т. д. Затем по полученным для кривой токаточ- 491
ф Рис. 311. Построение графика несинусоидального тока для катушки со стальным сердечником с помощью циклической кривой перемагничива- ния стали кам сможем построить график зависимости тока от времени, так как для каждого мгновенного значения магнитного потока нам известно соответствующее ему мгновенное значение тока, взятое из графика зависимости магнитного потока от тока (левая часть рис. 311). Из графика тока мы видим, что ток I и магнитный поток Ф достигают одновременно своих максимальных значе- ний, но нулевых значений они достигают не одновременно — ток I достигает нулевого значения несколько раньше, чем маг- нитный поток Ф. Это объясняется’ отставанием намагничивания стали от намагничивающей силы, или, иначе сказать, явлением гистерезиса. Итак, ток в катушке со стальным сердечником, к которой приложено синусоидальное напряжение, в общем случае неси- нусоидален. Следовательно, катушка со стальным сердечником искажает синусоиду тока, и это искажение при всех прочих равных условиях будет тем сильнее; чем до большего магнитного насыщения будет доводиться сталь- ной сердечник при его перемагничивании. Исследование режима в катушке со стальным сердечником при наличии в ней несинусоидального тока связано с извест- ными затруднениями. Для облегчения этой задачи обычно пере- ходят от несинусоидального тока к эквивалентному синусоидаль- ному току катушки. Синусоидальным током, эквивалентным данному несинусоидальному периодическому т о к у, называют такой ток, который имеет оди- наковое с ним действующее значение, один а- ковуючастоту и развивает одинаковую с ним активную мощность в данной цепи. 492
Эквивалентный синусоидальный ток для катушки со сталью можно найти по двум его взаимно перпендикулярным состав- ляющим — намагничивающей / и активной 1а. Намагничивающая составляющая тока создает синусоидаль- ный магнитный поток Ф. Она является синусоидальной величи- ной, совпадающей по фазе с создаваемым ею магнитным пото- ком Ф. Ее определяют следующим образом: 1) по заданному действующему значению U синусоидального напряжения, приложенного к катушке, определяют максималь- ное значение Фт магнитного потока в сердечнике катушки по формуле (428) Ф = т 4,44/w ’ 2) затем по основной кривой намагничивания заданного сорта стали (рис. 312) определяют максимальные ампер-витки AWm, соответствующие полученному максимальному магнит- ному потоку Ф^; 3) по полученным максимальным ампер-виткам AWm нахо- дят действующее значение намагничивающей составляющей тока /и по формуле / - J/2 w ’ где w — число витков катушки. Активная составляющая тока 1а развивает мощность, необхо- димую для покрытия мощности потерь в стали на перемагничи- вание и вихревые токи. Она также является синусоидальной величиной, опережающей по фазе магнитный поток Ф, а следо- вательно, и намагничивающую составляющую тока на угол у. Следовательно, в данном случае, когда активное сопротивление обмотки катушки практически равно нулю (гм = 0), активная составляющая тока 1а со- впадает по фазе с напря- жением U, приложенным к катушке. Величину ак- тивной составляющей то- ка 1а можно определить по формуле / ==^- где Ра— мощность потерь в ваттах на перемагничивание (430) Рис. 312. График зависимости магнитного потока Ф от ампер-витков катушки стали и на вих- ревые токи, ко- 493
торая может быть рассчитана по формулам (425) и (426) или измерена ваттметром; U — действующее значение напряжения, приложенного к ка- тушке, в вольтах; 1а — активная составляющая тока в амперах. Определив намагничивающую составляющую тока / и ак- тивную составляющую его 1а, можно найти и действующее зна- чение эквивалентного синусоидального тока по формуле / = (431) т.е. действующее значение эквивалентного сину- соидального тока в катушке со стальным сердечником равняется корню квадратному из суммы квадратов активной 1а и намагничи- вающей Z составляющих его. Если активное сопротивление га обмотки катушки таково, что им пренебречь нельзя, то часть напряжения, приложенного к катушке, будет затрачена в этом сопротивлении, а другая часть его будет затрачена на преодоление электродвижущей силы самоиндукции. В этом случае в формуле (428) для определения максималь- ного магнитного потока Фт надо вместо напряжения U, прило- женного к катушке, взять часть его UL, равную электродвижу- щей силе самоиндукции в катушке: ф т 4,44/w * Формула (430) для определения активной составляющей тока 1а в этом случае также несколько видоизменяется: f _ ^ст иг ’ Li где UL — та часть напряжения, которая компенсирует электро- движущую силу самоиндукции. Надо заметить, что в рассматриваемом случае, когда гм #= О, активная составляющая тока 1а будет совпадать по фазе не с полным напряжением I/, приложенным к катушке, а с напря- жением ULi которое, как и активная составляющая тока 1аУ опережает по фазе магнитный поток Ф на угол у. Чем больше мощность потерь на перемагничивание и вихревые токи в стали катушки, тем больше активная составляющая тока. На рис. 313 показана векторная диаграмма токов и напря- жений катушки со стальным сердечником, порядок построения которой сводится к следующему. Откладываем вектор магнит- ного потока Ф в некотором произвольном направлении, напри- 494 '
мер в горизонтальном и вправо. Намагничивающая составляю- щая тока / , создающая этот магнитный поток, совпадает с ним по фазе, а поэтому откладываем вектор по направле- нию вектора магнитного потока Ф. Индуктированная электро- движущая сила самоиндукции в катушке отстает по фазе от магнитного потока на угол , а поэтому вектор электродвижу- щей силы самоиндукции EL отложим вниз под углом к век- Рис. 313. Векторная диаграмма токов и напряжений для ка- тушки со стальным сердеч- ником тору магнитного потока Ф. Та часть приложенного к катушке напряжения UL, которая компенсирует электродвижущую силу самоиндукции, находится с ней в противофазе, а поэтому вектор UL отложим вверх перпендикулярно вектору магнитного потока Ф. Актив- ная составляющая тока /Л, обуслов- ленная мощностью потерь на вихре- вые токи и перемагничивание стали катушки, совпадает по фазе с на- пряжением UL, а поэтому вектор 1а проводим в направлении вектора напряжения UL. Результирующий ток / — эквивалентный синусоидаль- ный ток катушки — равен геометри- ческой сумме тока намагничения и активной составляющей тока /а, а поэтому вектор тока I опреде- ляем путем геометрического сложе- ния векторов тока и 1а. Та часть напряжения Ua> которая составляет падение напряжения в активном сопротивлении обмотки ка- тушки, совпадает по фазе с током I катушки, а поэтому вектор напряжения Ua прикладываем началом к концу вектора UL и проводим параллельно вектору тока I. Напряжение, приложенное к катушке, равно геометрической сумме падений напряжений UL и Ua. Сложив геометрически векторы UL и Ua, найдем вектор напряжения С7, приложенного к катушке. Из векторной диаграммы видно, что угол сдвига фаз напря- жения U и эквивалентного синусоидального тока I в катушке в общем случае меньше -у-. Это и понятно, так как в катушке происходят потери энергии на необратимые процессы — в актив- ном сопротивлении обмотки катушки и в стали на вихревые токи и на перемагничивание. Чем больше эти потери, тем меньше угол и тем, следовательно, больше мощность потерь в ка- тушке. Мощность потерь на перемагничивание и вихревъге токи в катушке со сталью зависит отмаксимальной магнитной 495
индукции Вт в стали, от частоты f тока, от сорта стали ивеса. Величину этой мощности можно рассчитать по формулам (425) и (427). Величина максимальной магнитной индукции Вт в стали катушки зависит от величины приложенного напряже- ния: чем больше это напряжение, тем больше магнитный по- ток Ф в стали, максимальная магнитная индукция Вт в ней, мощность потерь на пере- магничивание и вихревые токи. Полное сопротивле- ние Z катушки можно определить, исходя из формулы закона Ома Рис. 314. Катушка с замкнутым стальным сердечником Z = -y. (432) Эквивалентное актив- ное сопротивление ка- тушки можно определить, исходя из активной мощ- ности Ра, расходуемой в катушке: ₽.=4e-%L+r-» <433> гдеРст—мощность потерь в стали на перемагничивание и вих- ревые токи; г„—активное сопротивление обмотки катушки. Зная полное сопротивление Z и эквивалентное активное со- противление А*э, можно определить реактивное сопротивле- ние xL: xL = y/Z^-Rl, а отсюда — и индуктивность L катушки: («4) где L — индуктивность катушки в генри. Пример 183. Требуется определить режим в катушке со стальным сер- дечником (рис. 314), если к ней приложено синусоидальное напряжение, действующее значение которого U = 50 в и частота f = 50 гц. Катушка имеет число витков w = 100. Сердечник катушки набран из листовой электротехни- ческой стали и имеет площадь поперечного сечения S = 23 см2. Активное со- противление обмотки катушки rM = 1 ом. Все размеры, показанные на рис. 314, даны в сантиметрах. Воздушного зазора сердечник не имеет. Маг- нитным рассеянием в катушке пренебрегаем. 496
Решение. Поскольку активное сопротивление обмотки катушки отно- сительно мало, полагаем, что все напряжение, приложенное к катушке, ком- пенсируется электродвижущей силой самоиндукции, т. е. U , « U =- 50 в. Lt Максимальный магнитный поток в сердечнике катушки определим по формуле Ф„ = , /fz = < лл 5Л ,ЛЛ « 0.0023 в-сек. т 4,44/10 4,44-50-100 Величина максимальной магнитной индукции в сердечнике катушки равна в _ __ 0,0023 _ в-сек S ~ 0,0023 ~ м* * По кривой намагничивания находим удельные ампер-витки (напряжен- ность магнитного поля), которые для максимальной магнитной индукции п , в-сек ОЛЛ а-в Вт в 1----т— равны 300 ------. т м2 г м Полные ампер-витки (магнитодвижущая сила), необходимые для созда- ния данного магнитного режима в стальном сердечнике катушки, равны АП7СТ = Л ¥Гср « 300-0,4 « 120 а-в. где Zcp—длина средней магнитной линии в стали. Отсюда находим максимальное значение намагничивающей составляю- щей эквивалентного синусоидального тока: _Л\17ст_120 V/Л w ~ 100= ’ Намагничивающая составляющая эквивалентного синусоидального тока равна Г = = -TJT = 0,84 а. и /2 Ml Определяем вес стального сердечника, принимая удельный вес стали равным 7,8 ^3- : G = 37-7,8 = 0,23-4-7,8 = 7,16 кг. Находим мощность потерь на перемагничивание стали по формуле Роер - «пер £ &mG = 3-^. Р-7,16 - 10,74 вт, где <?пер = 3. Находим мощность потерь на вихревые токи в стали по формуле (f X 2 / 50 \ 2 G = 1,2^1) -7,16 = 2,15 вт. где <тв.т = 1,2. Находим общую мощность потерь в стали на перемагничивание и вихре- вые токи: Рст == Рпер + т = 10,74 + 2,15 » 12,9 ш. 32—1377 497
Теперь определим активную составляющую эквивалентного синусоидаль- ного тока по формуле Эквивалентный синусоидальный ток катушки I = / = /0,262 + 0,842 _ о,88 а. Падение напряжения в активном сопротивлении обмотки катушки = /гм = 0,88-1 = 0,88 в, т. е. оно имеет такую величину, которой можно пренебречь. Активная мощность, развиваемая источником электрической энергии в катушке, равна Ра = Лт + /Ч = 12,9 + 0,882-1 = 13,7 вт. Коэффициент мощности Р 13 7 C0S* = 7f =50Ж8 = 0’314- Полное сопротивление катушки 7 и 50 Z = T= ОЖ=57 ом- Эквивалентное активное сопротивление катушки Р 13 7 ^ = -# = 0^ = 17’7ол- Индуктивное сопротивление катушки хь - |/ Z2 — = J/572 — 17,72 = 54 ом. Индуктивность катушки х. 54 L= —^ = — = 0,172 гн. а) 014 Очень часто на практике применяются катушки, стальные сердечники которых имеют воздушные зазоры. Такие катушки обычно называют дроссельными. Если при неизменном действующем значении напряжения, приложенного к дроссель- ной катушке, изменять величину воздушного зазора в стальном сердечнике, то изменится магнитное сопротивление магнитопро- вода катушки и ее индуктивность. На рис. 315 показан график зависимости индуктивности дроссельной катушки от воздушного зазора ее сердечника. Из этого графика видно, что с увеличе- нием воздушного зазора индуктивность катушки падает вначале более резко, а потом замедленно. В соответствии с этим изме- няется и реактивное сопротивление катушки, что в свою очередь приводит к изменению тока в ее обмотке. Следовательно, регу- лируя воздушный зазор в сердечнике катушки, можно регули- 49§
ровать ток в ней, сохраняя неизменным действующее значение напряжения на ее зажимах. На рис. 316 показан один из типичных дросселей, применяе- мых в радиотехнических схемах. Сердечник дросселя изготовлен из специальной трансформаторной стали или из мягкого листо- вого железа толщиной менее одного миллиметра. Отдельные листы сердечника изолированы друг от друга тонкой папирос- ной бумагой или слоем шеллач- ного спиртового лака, чтобы свести к минимуму потери на вих- ревые токи. Каркас катушки склеивается из электрокартона или Сердечник Рис. 316. Внешний вид дрос- сельной катушки, применяе- мой в радиотехнике Рис. 315. График зависимости индуктивности дроссельной ка- тушки от воздушного зазора ее сердечника картона. Щеки каркаса делаются из толстого картона. Обмотка катушки кладется ровными слоями, и при этом между несколь- кими рядами проволоки размещается прокладка из электрокартона, пропитанного шеллачным лаком. Это делается для улучшения изоляции обмотки и предохранения витков от пробоя высоким напряжением. Воздушный зазор в сердечнике катушки обычно заполняется картонной прокладкой, толщина которой колеблется от долей миллиметра до нескольких миллиметров. Крепление нескольких листов сердечника производится при помощи болтов, пропускае- мых через отверстия в пластинах, или же при помощи деревян- ных пластин, стягиваемых шурупами. Потери энергии в катушках со сталью невелики, а поэтому их весьма выгодно применять в цепях переменного тока вместо реостатов для поглощения излишнего напряжения. § 139. ОБЩЕЕ ПОНЯТИЕ О ТРАНСФОРМАТОРАХ Трансформатором называется электротехническое устройство (аппарат), преобразующее переменный ток одного напряжения в переменный ток другого напряжения. 32* 499
Трансформаторы играют Важную роль в электротехнике и, в частности, в технике связи. Например, мощные повышающие трансформаторы дают возможность передавать электрическую Рис. 317. Схедаа трансформатора энергию с малыми потерями на очень большие расстояния. С по- мощью понижающих трансфор- маторов эту энергию можно пре- образовать для использования ее многочисленными потребите- лями переменного тока низкого напряжения. Трансформаторы — важные элементы радиоцепей, без которых немыслима работа радиотехнических устройств и во- обще всех электрических средств связи. Советская электропромышленность освоила полностью вы- пуск всех типов трансформаторов, необходимых для удовлетво- рения нужд электротехники. Советские конструкторы разрабо- тали оригинальные конструкции трансформаторов высокого на- пряжения, предназначенных для линий передачи электрической энергии на сверхдальние расстояния. Освоено производство вы- сококачественных трансформаторов для ртутных преобразова- тельных подстанций, мощных трансформаторов, применяемых в электротермии, и ряда иных типов трансформаторов, предна- значенных для обслуживания разного рода электротехнических устройств. Приоритет изобретения трансформатора принадлежит заме- чательному русскому изобретателю П. Н. Яблочкову. В 1876 г. П. Н. Яблочков выдвинул идею трансформации переменного тока, а спустя шесть лет, в 1882 г., первый транс- форматор Яблочкова — прототип современных трансформаторов переменного тока — демонстрировался на электротехнической выставке в Петербурге и получил всеобщее признание. В том же 1882 г. на московской промышленной выставке другой русский изобретатель, лаборант физической лаборатории Московского университета И. Ф. Усагин демонстрировал транс- форматор переменного тока своей конструкции, за что получил от Комитета выставки диплом с надписью «За успешные опыты электрического освещения через посредство отдельной индукции и в поощрение дальнейшей разработке этой методы». Простейший трансформатор состоит из замкнутого сталь- ного сердечника, собранного из листовой стали, на котором раз- мещены две обмотки, индуктивно связанные друг с другом (рис. 317). Одна из этих обмоток, имеющая число витков подключается к источнику электрической энергии с переменным напряжением. Она называется первичной обмоткой транс- 500
форматора., Другая обмотка, имеющая число витков назы- вается вторичной. Назначение первичной обмотки трансформатора — получать электрическую энергию от генератора и передавать ее во вто- ричную обмотку. Назначение же последней — преобразовывать (трансформировать) напряжение (повышать или понижать) первичной обмотки и при заданном номинальном напряжении питать потребители электрической энергией, получаемой от пер- вичной обмотки. Различают два основных режима работы трансформатора: при холостом ходе и при нагруске. Рассмотрим эти два режима. § 140. РАБОТА ТРАНСФОРМАТОРА ПРИ ХОЛОСТОМ ХОДЕ Если концы вторичной обмотки трансформатора оставить разомкнутыми, то трансформатор будет работать без нагрузки, или, иначе говоря, будет находиться в режиме холостого хода. Этот режим мало отличается от режима работы обычной ка- тушки со стальным сердечником. Под влиянием переменного синусоидального напряжения щ = t7lz72-sin(aj/ + , приложенного к первичной обмотке трансформатора, в ней в общем случае возникнет какой-то пе- риодический несинусоидальный ток, действующее значение ко- торого равно I. Если активное сопротивление первичной обмотки трансфор- матора относительно мало, что бывает довольно часто на прак- тике, то переменный ток создает в стальном сердечнике такой синусоидальный магнитный поток Ф = Ф^-sin wt, при кото- ром в первичной обмотке индуктируется электродвижущая сила самоиндукции, уравновешивающая полностью приложенное к этой обмотке напряжение: et= — Ulm • si и + -J-) = • sin (ш/ — -у), ИЛИ где Ц—действующее значение напряжения, приложенного к первичной обмотке; Ег—действующее значение электродвижущей силы само- индукции в первичной обмотке. Одновременно магнитный поток Ф = Фт • sin пронизывает и витки вторичной обмотки трансформатора, где в соответствии с этим индуктируется электродвижущая сила взаимоиндукции Е2. Так как электродвижущие силы Ei и Е2 индуктируются одним и тем же магнитным потоком, то £1 _ Et wt ’ 501
т. е. индуктированные электродвижущие силы в первичной и вто- ричной обмотках трансформатора пропорциональны числу вит- ков этих обмоток. Но так как при холостом ходе трансформатора напряже- ние U2 на разомкнутых зажимах вторичной обмотки равно элек- тродвижущей силе Е2, а напряжение Uh приложенное к первич- ной обмотке, практически равно электродвижущей силе Еь то можем положить, что U2 w2 Приняв во внимание, что 'в мощных трансформаторах паде- ние напряжения в обмотках относительно мало ’при нагрузке, можно считать, что равенство (435) справедливо не только при холостом ходе трансформатора, но и при его нагрузке. Отношение первичного напряжения U\ к вторичному U2 при холостом ходе трансформатора, равное отношению числа витков первичной обмотки к числу витков вторичной, называется коэф- фициентом трансформации k. Если коэффициент трансформации £<1, то трансформатор является повышающим, так как вторичное напряжение U2 больше первичного U\ (£/2>£Л при &<1). Наоборот, если коэффициент трансформации Л>1, то трансформатор является понижающим, так как в этом случае вторичное напряже- ние U2 меньше первичного U\ (U2<Ui при 1). Пример 184. К первичной обмотке трансформатора подано напряжение, действующее значение которого равно Ui = 25 в. Число витков первичной обмотки Wi = 250, а вторичной ш2 = 2000. Определить коэффициент транс- формации трансформатора и напряжение на его зажимах при холостом ходе. Решение. Коэффициент трансформации равен Л = _ 250 w2 ~~ 2000 = 0,125, т. е. трансформатор является повышающим. Напряжение на зажимах вторичной обмотки при холостом ходе равно U _ = ILl 2 Wi k 2» 0,125 = 200 в. Величина магнитного потока в сердечнике трансформатора зависит от величины действующего значения напряжения U\, приложенного к первичной обмотке трансформатора. Макси- мальное значение магнитного потока определяется, как и для обычной катушки со стальным сердечником, по формуле (428) Ф =—— т 4,44 fwi Пример 185. Определить максимальное значение магнитного потока в сер- дечнике трансформатора, если напряжение, приложеннъе к его первичной 502
обмотке, Ui = 125 в, частота f = 50 гц и число витков первичной обмотки wi = 500. Решение. По формуле (428) находим Ф„ - = 1,125-10-’ в-сек. т 4,44-50-500 Рис. 318. Векторная диа- грамма токов и напря- жений при холостом ходе трансформатора Магнитный поток при холостом ходе трансформатора соз- дается током его первичной обмотки, называемым током хо- лостого хода /о трансформатора. Величину его можно определить по намагничивающей и активной составляющим, как и для обычной катушки со стальным сердечником, применяя формулы (429) и (430). Ток холостого хода трансформатора обычно составляет несколько процентов номинального рабочего тока первичной его цепи. Чтобы уменьшить ток холостого хода, обычно в качестве материала для сердеч- ников трансформатора применяют транс- форматорную сталь, обладающую боль- шой магнитной проницаемостью. Сталь- ной сердечник рассчитывается обычно так, чтобы он не работал в режиме маг- нитного насыщения, поскольку в этом случае возникает большая мощность по- терь стали из-за большой величины маг- нитной индукции и, кроме того, увеличи- вается ток холостого хода из-за относи- тельно малой магнитной проницаемости сердечника. На рис. 318 приведена векторная диа- грамма токов и напряжений при холо- стом ходе трансформатора для случая, когда магнитным рассеянием в трансфор- маторе можно пренебречь. На диаграмме показаны: Ф — вектор магнитного потока трансформатора; Ех— электродвижущая сила самоиндукции в первичной об- мотке трансформатора; Е2 — электродвижущая сила взаимоиндукции во вторичной обмотке трансформатора; /0 — ток холостого хода трансформатора; Ual — та часть первичного напряжения, которая компенси- рует электродвижущую силу самоиндукции Е^ в первичной об- мотке; Uar — падение напряжения в активном сопротивлении пер- вичной обмотки трансформатора; Ux — напряжение, приложенное к первичной обмотке; 503
<Pi — сдвиг фаз тока холостого хода /0 и первичного напря- жения Ui. Так как ток холостого хода трансформатора относительно мал, то мощность потерь в активном сопротивлении первичной обмотки обычно мала. Следовательно, мощность потерь холо- стого хода трансформатора складывается главным образом из потерь на перемагничивание и на вихревые токи в стальном сердечнике: ^0=tzi4,cos?0, (436) где Ро— мощность потерь в трансформаторе при холостом ходе в ваттах; Ц— номинальное напряжение трансформатора в вольтах при холостом ходе трансформатора; /0—ток холостого хода трансформатора в амперах; cos <р0— коэффициент мощности трансформатора при холостом ходе. § 141. РАБОТА ТРАНСФОРМАТОРА ПРИ НАГРУЗКЕ Допустим, что к зажимам вторичной обмотки трансформа- тора подключен потребитель, обладающий полным сопротивле- нием Z. В результате этого во вторичной цепи возникнет ток /2, величину которого можно определить по формуле закона Ома Т _ Uj z » где Ц— напряжение на зажимах вторичной обмотки трансфор- матора. Вполне очевидно, что напряжение U2 на зажимах вторичной обмотки трансформатора будет меньше, чем электродвижущая сила взаимоиндукции Ё2, из-за падения напряжения во вторич- ной обмотке трансформатора: Z72==£2-72Z2, (437) т. е. напряжение на зажимах вторичной обмотки нагруженного трансформатора равняется геометрической разности электродви- жущей силы взаимоиндукции Е2 и падения напряжения I2Z2 во вторичной обмотке. Ток 12, возникший во вторичной цепи под действием электро- движущей силы взаимоиндукции, согласно правилу Ленца соз- дает собственный магнитный поток в сердечнике трансформа- тора, направленный навстречу основному магнитному потоку, создаваемому током 1\ первичной обмотки. Следовательно, при переводе трансформатора с холостого хода на нагрузку результирующий магнитный поток в сердеч- нике трансформатора в первый момент несколько снижается 504
из-за размагничивающего действия ампер-витков wJz вторич- ной обмотки. Однако это приводит к тому, что электродвижущая сила самоиндукции Е\ первичной обмотки уменьшается, соответ- ственно уменьшаются индуктивное и полное сопротивления пер- вичной обмотки и возрастает ток 71. Этот ток возрастает до та- кой величины, при которой результирующий магнитный поток вновь становится таким же, как и при холостом ходе трансфор- матора. Итак, ток /1 в первичной обмотке нагруженного трансформа- тора увеличивается по сравнению с током холостого хода /о ровно настолько, чтобы скомпенсировать размагничивающие ампер-витки вторичной обмотки и сохранить прежнее значение магнитного потока, равное потоку холостого хода. Наоборот, если трансформатор будет разгружаться (будет уменьшаться ток /2), то в соответствии с этим будут уменьшаться размагни- чивающие ампер-витки W2I2, что приведет к увеличению резуль- тирующего магнитного потока в сердечнике трансформатора. В соответствии же с этим возрастет электродвижущая сила са- моиндукции в первичной катушке, увеличатся индуктивное и полное сопротивления в ней, что приведет к снижению тока Ц в первичной обмотке. Этот ток будет уменьшаться до тех пор, пока результирующий магнитный поток в трансформаторе не станет вновь равным магнитному потоку холостого хода. Следовательно, всякое изменение нагрузки (тока А) транс- форматора приводит к такому изменению тока Ц в первичной цепи, при котором результирующий магнитный поток в сердеч- нике трансформатора остается равным магнитному потоку хо- лостого хода. Для сохранения постоянства результирующего магнитного потока в сердечнике трансформатора в режимах от холостого хода до полной нагрузки его необходимо такое соотношение между ампер-витками первичной и вторичной обмоток, чтобы их геометрическая сумма равнялась ампер-виткам первичной об- мотки при холостом ходе трансформатора, т. е. + (72w2) = /0®! = const, (438) где ZjWt — ампер-витки первичной обмотки трансформатора при нагрузке; /2да2—размагничивающие ампер-витки вторичной обмотки трансформатора; Iowl — ампер-витки первичной обмотки трансформатора при холостом ходе. Разделив все члены равенства (438) на число витков u>t, по- лучим следующее соотношение токов в трансформаторе: (439) 505
т. е. ток 71 в первичной обмотке трансформатора равняется геометрической сумме двух токов — тока холостого хода 70 трансформатора и тока =------^"4, который компенсирует раз- магничивающее действие тока вторичной обмотки трансформатора. Ток 7'1 находится в противофазе с током 72, т. е. сдвинут по фазе относительно него на угол 180°, и численно отличается w2 от него в ---раз. Так как ток холостого хода трансформатора мал по сравне- нию с номинальным рабочим током 1\ первичной цепи трансфор- матора, то в уравнении (439) с достаточной для практики точ- ностью можно считать, что или, иначе, /1 w2 1 /2 Wi k ’ (440) т. е. токи первичной и вторичной обмоток трансформатора обратно пропорциональны числам витков этих обмоток. Надо все же помнить, что пренебрежение величиной тока хо- лостого хода 70 приводит нас к равенству ZjW, =Z2W2, которое выражает только приближенное равновесие ампер-вит- ков первичной и вторичной обмоток трансформатора. Пример 186. Определить ток в первичной обмотке трансформатора, если во вторичной цепи его имеется ток /2 = 5 а, число витков первичной обмотки ic'i = 300, а вторичной и>2 = 30. Решение. Применяя формулу (440), находим 7 = = о 5 д 1 Wi 300 ’ Мощность, развиваемая источником электрической энергии в первичной цепи трансформатора, равна Pl = Ц/1-СО8?1. Мощность, развиваемая во вторичной цепи трансформатора, равна P2 = t/2/2-cos <р2. Мощность Pi, развиваемая источником электрической энер- гии, слагается из мощности Рг, передаваемой из первичной цепи во вторичную, мощности Р„ потерь в активном сопротивлении 506
первичной обмотки и мощности Рст потерь в стальном сердеч- нике на перемагничивание и вихревые токи: Pi — Рг + Рм + PCv Мощности Р„ и Рст потерь в первичной цепи трансформа- тора относительно малы. Например, в мощных трансформаторах они составляют не более одного процента. Поэтому, если ими практически пренебречь, тр будем иметь следующее соотношение мощностей в трансформаторе: Р\ Рг, или, иначе, U1I1-cos<f1^L72l2’COs<f2. (441) Но так как в этом случае = или UXI2 = U2I2, то на основании равенства (441) получим COS?i = COSCp2 И Т1 = ?2, т. е. угол сдвига фаз между током Ц и напряжением первич- ной цепи трансформатора практически равен углу сдвига фаз между током Л и напряжением U2 вторичной цепи трансформа- тора. Обычно не весь магнитный поток, создаваемый током первич- ной обмотки, пронизывает целиком все витки вторичной об- мотки. Часть этого потока, называемая потоком рассеяния Ф51 первичной обмотки, замыкается через воздух. Этот поток рас- сеяния не индуктирует электродвижущую силу во вторичной об- мотке и является по сути дела бесполезным магнитным потоком. Одновременно с этим существует и поток рассеяния Ф52 во вторичной обмотке трансформатора, который также замыкается через воздух и не пронизывает витки первичной обмотки. Магнитное рассеяние в трансформаторах нежелательно, так как оно вызывает дополнительное падение напряжения в об- мотке трансформатора, что несколько снижает напряжение на зажимах потребителя. § 142. ВЕКТОРНАЯ ДИАГРАММА НАГРУЖЕННОГО ТРАНСФОРМАТОРА В основу построения векторной диаграммы возьмем вектор результирующего рабочего магнитного потока Ф, который, как известно, сохраняет неизменное действующее значение при всех режимах работы трансформатора, если действующее значение напряжения U\, приложенного к первичной обмотке трансфор- матора, неизменно. Действующее значение результирующего маг- нитного потока Ф находим по формуле Ф^ ~ — — 434/^1 4,44/^ • 507
Рис. 319. Векторная диаграмма для на- груженного трансформатора при индук- тивной нагрузке Полагая, что начальная фаза результирующего магнитного потока Ф равна нулю, т, е. Ф = Фт • sin отложим вектор Ф на векторной диаграмме (рис. 319) по начальной оси. Вектор тока холостого хода /0 трансформатора отложим под углом а к вектору результирующего магнитного потока Ф в сто- рону опережения, т. е. против движения часовой стрелки. Вели- чину угла « найдем по его тангенсу tga = ^,’ (442) где 1а = — активная составляющая тока холостого хода (см. формулу 430); [ == — намагничивающая составляющая тока (см. фор- и у 2w мулу 429). Результирующий магнитный поток Ф индуктирует в первич- ной катушке электродвижущую силу самоиндукции Е\, а во вторичной обмотке — электродвижущую силу взаимоиндук- ции Е2; каждая из них отстает от магнитного потока Ф на угол В соответствии с этим отложим векторы Е\ и Е2 на векторной диаграмме под углом -у- к вектору Ф в сторону отставания (по часовой стрелке). Электродвижущая си- ла Е2 показана на вектор- ной диаграмме несколько большей, чем электродви- жущая сила Ei, т. е. в данном случае мы строим векторную диаграмму для повышающего трансфор- матора' Полагая, что нагрузка во вторичной цепи транс- форматора носит индук- тивный характер, отложим вектор тока 12 вторичной обмотки трансформатора, отстающий от вектора электродвижущей силы Е2 на некоторый угол ф2. Под действием тока /2 во вторичной обмотке воз- никает магнитодвижущая 508
сила, или, иначе сказать, ампер-витки /2^2, размагничивающее действие которых уравновешивается соответствующим числом ампер-витков первичной обмотки. Поэтому ток Л первичной об- мотки на векторной диаграмме представим как геометрическую сумму двух токов: тока холостого хода /о, который не зависит от общей нагрузки, и тока /'р зависящего от нагрузки трансформа- тора, находящегося в противофазе с током /2 вторичной цепи трансформатора и создающего в первичной обмотке трансфор- матора ампер-витки I'iWi, равные и противоположные по знаку ампер-виткам вторичной обмотки Itw2, т. е. 7^ =—/2w2. (443) В соответствии с этим ток Ц на векторной диаграмме пока- жем как геометрическую сумму двух токов, 10 и Г с А = /о + Л. где |/;] = |-g-Z2| иЛ = -^72. Если нагрузка трансформатора (ток /2) будет изменяться но величине, а также будет изменяться и угол ф2 сдвига фаз между током 12 и электродвижущей силой Е2, то соответственно будут изменяться составляющая тока Г\ в первичной цепи трансформа- тора, а следовательно, величина самого тока 1\ и углы сдвига фаз между ним и напряжением C7i, приложенным к первичной обмотке трансформатора. Магнитный поток рассеяния во вторичной обмотке Ф52 сов- падает по фазе с током /2, а поэтому на векторной диаграмме отложим вектор Ф52 в направлении вектора тока /2. Магнитный поток рассеяния Ф$2 индуктирует электродвижу- щую силу самоиндукции рассеяния ES2> отстающую от него на угол . В соответствии с этим вектор ES2 отложим на век- торной диаграмме под углом к вектору Ф52 в сторону от- ставания от него. Электродвижущая сила Е2 во вторичной катушке равна гео- метрической сумме трех составляющих: £2 =ЛГ2 + (444) где Е2—электродвижущая сила взаимоиндукции во вторичной обмотке трансформатора; /2г2—падение напряжения в активном сопротивлении вто- ричной обмотки, совпадающее по фазе с током /2; £7$2— та часть электродвижущей силы, которая компенси- рует электродвижущую силу рассеяния ES2 и нахо- дится с ней в противофазе; U2— напряжение на зажимах вторичной обмотки трансфор- матора. 509
В соответствии с этим наносим на векторную диаграмму век- торы С/а2 =Z2r2; C/S2 =—£52 и С/2. Напряжение С/2 на векторной диаграмме показано отстаю- щим по фазе от электродвижущей силы £2 на угол (ф2— <р2), а ток Z2 показан отстающим от напряжения С/2 на угол <р2 из-за индуктивного характера нагрузки. Угол <рг зависит от параметров внешней цепи и определяется через его тангенс по формуле tg?2 = -7-» (445) где г — активное сопротивление потребителя; х— реактивное (индуктивное) сопротивление потребителя. Ток /1 первичной обмотки трансформатора создает магнитный поток рассеяния который совпадает с ним по фазе, а по- этому отложим на векторе тока It в векторной диаграмме век- тор Ф51. Магнитный поток рассеяния ФЛ индуктирует электродвижу- щую силу самоиндукции рассеяния ES1, которая отстает от по- тока ФЛ на угол у-, а поэтому вектор ESi на векторной диа- грамме отложим под углом -у к вектору Ф51 в сторону отста- вания от него. Напряжение СД, прилОЖёййое к первичной обмотке транс- форматора, равно геометрической сумме трех составляющих: Ц = + + (446) где C7t— напряжение, приложенное к первичной обмотке транс- форматора; С/Д—та часть напряжения С/ь которая компенсирует элек- тродвижущую силу самоиндукции Et и, следовательно, находится в противофазе с ней; USi—та часть напряжения, которая компенсирует электро- движущую силу самоиндукции рассеяния ESI и, сле- довательно, находится с ней в противофазе; Ua—та часть напряжения, которая расходуется в активном сопротивлении первичной обмотки и совпадает по фазе с током /1. Наносим на векторную диаграмму векторы ULl, USb UM и строим по ним вектор напряжения СД. Угол <pi на диаграмме — это угол сдвига фаз тока 1\ и на- пряжения С/i в первичной обмотке трансформатора. Он положи- телен, так как трансформатор имеет нагрузку индуктивного ха- рактера. $10
§ 143. ВНЕШНЯЯ ХАРАКТЕРИСТИКА ТРАНСФОРМАТОРА С изменением нагрузки трансформатора изменяется и напря- жение U2 на его вторичных зажимах. Зависимость величины напряжения U2 на вторичных зажимах трансформатора от нагрузки (тока Л) называется внешней ха- рактеристикой трансформатора. При увеличении тока Л увели- чивается и ток Л в первичной цепи его. Одновременно с этим растут соответственно магнитные потоки рассеяния Ф$( и Ф$2, а также электродвижущие силы самоиндукции рассеяния ESl и Es2, индуктированные этими потоками. Увеличиваются также напряжения USl и US2,____компенсирующие электродвижущие силы ES1 и Е$2, так как USi— — ES1 и U$2 = — ES2. Одновре- менно с ростом токов Ii и 12 увеличиваются падения напряжений и в активных сопротивлениях первичной (Uai = 7/м1) и вторич- ной ( £/а2 =/2гм2) обмоток трансформатора. Так как действующее значение напряжения Ut, приложен- ного к первичной обмотке трансформатора, сохраняется постоян- ным, а падения напряжений Ual и USi с ростом тока Ц растут, то часть этого напряжения ULb которая компенсирует электро- движущую силу самоиндукции Ег, должна* уменьшиться. А это произойдет' только в том случае, если уменьшится электродви- жущая сила самоиндукции Ei, поскольку UL1 — —Ei. Уменьше- ние же электродвижущей силы самоиндукции Ei непосредственно связано с уменьшением результирующего магнитного потока Ф, потому что 4,44/»! 4,44/Wj Но если результирующий магнитный поток (рабочий поток) Ф в сердечнике трансформатора уменьшится, то это повлечет за собой уменьшение электродвижущей силы Е2, индуктированной во вторичной обмотке трансформатора. Следовательно, если с ростом нагрузки трансформатора умень- шается электродвижущая сила Е2 во вторичной обмотке транс- форматора и в то же время увеличиваются падения напряжения в ней Ua2 = I2r№2 и US2 = —ES2, то напряжение на зажимах вторичной обмотки трансформатора U2 должно уменьшиться. Из векторной диаграммы (рис. 319) видим, что это уменьшение бу- дет тем значительней, чем больше будет отставать на фазе ток 12 от электродвижущей силы Е2, т. е. чем более резко будет выра- жен индуктивный характер нагрузки трансформатора. Наоборот, если трансформатор имеет нагрузку емкостного характера, то с увеличением нагрузки (тока 12) напряжение на вторичных за- жимах трансформатора будет изменяться в сторону возрастания его. На рис. 320 приведена векторная диаграмма для работаю- щего трансформатора при емкостной нагрузке. Здесь показано, 511
Рис. 320. Векторная диаграмма для работающего трансформатора при емкостной нагрузке что токи Д и Л опережают по фазе соответствующие им напряжения Ui и Ui. Напряжение U2 несколько больше элек- тродвижущей силы Е2, а сдвиг фаз тока Г2 и напряжения U2 больше, чем сдвиг фаз этого тока и электродвижущей силы Е2. На рис. 321 приведены три внешние характеристики транс- форматора соответственно для трех различных режимов работы Рис. 321. Внешние характеристики трансформатора его: при ® < 0 (емкостный характер нагрузки), при <р = 0 (чисто активный ха- рактер нагрузки) и <р > О (индуктивный характер на- грузки). Надо сказать, что паде- ние напряжения во вторич- ной обмотке силовых транс- форматоров в пределах от холостого хода до нормаль- ной нагрузки составляет не более двух — трех процентов номинального напряжения трансформатора. 512
§ 144. КОЭФФИЦИЕНТ ПОЛЕЗНОГО ДЕЙСТВИЯ ТРАНСФОРМАТОРА При работе трансформатора полная мощность, потребляемая его первичной обмоткой, слагается из полезной мощности, по- требляемой во вторичной цепи, и из потерь в самом трансформа- торе на вихревые токи и перемагничивание, а также из потерь в обмотках его катушек: РХ = Р2 + РС + /fr + (447) Р, = 672/2-cos <р21 Рис. 322. Кривая зави- симости к. п. д. транс- форматора от его на- грузки где Р2— мощность, расходуемая во вторичной цепи потребите- лем; Рс—мощность, расходуемая в стали трансформатора; —мощность, расходуемая в активном сопротивлении пер- вичной обмотки трансформатора; /|г2— мощность, расходуемая в активном сопротивлении вто- ричной обмотки трансформатора. Магнитный поток в трансформаторе, как мы выяснили выше, не изменяется с изменением нагрузки, а поэтому и величина магнитной индукции сохраняется неизменной, если действующее значение напряжения (/, приложенного к первичной цепи, неизменно. Но при по- стоянстве магнитной индукции В потери на вихревые токи и перемагничивание постоянны, т. е. потери в стали транс- форматора не зависят от его нагрузки, в отличие от потерь в меди обмоток транс- форматора и /|г2, растущих с уве- личением нагрузки. Коэффициент полезного действия трансформатора очень высок. Для мощ- ных трансформаторов, применяемых в силовых установках, он достигает при- мерно 99%. Коэффициент полезного действия трансформаторов малой мощности, применяемых в технике связи, колеблется в пределах 50—70%. Наибольший коэффициент полезного действия трансформа- тора получается при равенстве потерь в стали и меди: Рс = РА+Р2г2, (448) что соответствует нормальной. нагрузке трансформатора (рис. 322). 83-1377 513
§ 145. ТИПЫ ТРАНСФОРМАТОРОВ В современной электротехнике широкб применяются самые разнообразные трансформаторы, начиная от маломощных, при- меняемых в технике связи, и кончая мощными силовыми транс- форматорами, применяемыми в осветительных, силовых установ- ках и в линиях передачи электроэнергии на дальние расстояния. Трансформаторы однофазного тока по своей конструкции подразделяются на два основных тиТТа — стержневой и броневой. Рис. 323. Стержневой трансформатор В стержневом трансформаторе первичная и вто- ричная обмотки размещены на общем неразветвленном^сталь- ном сердечнике (рис. 323). В броневом трансформаторе (рис. 324) катушки насажены на средний стержень разветвленного стального сер- дечника так, что магнитопровод замыкается вокруг этих катушек. Чтобы свести потери на перемагничивание и вихревые токи в трансформаторе к минимуму, сердечники трансформаторов из- готовляют из специальных сортов стали и собирают их из от- дельных листов стали толщиной 0,35—0,5 мм, изолированных друг от друга лаком. Для уменьшения магнитного рассеяния в трансформаторе и намагничивающего тока при сборке сердечника трансформатора воздушные зазоры между отдельными частями сердечника сво- дят к минимуму. С этой целью стальные листы сердечников трансформатора собирают внакладку или впритык, чтобы они тесно соприкасались друг с другом. На рис. 325 показаны де- тали трансформатора Ш-образной формы и порядок сборки сер- дечника трансформатора. Для достижения хорошей изоляции в трансформаторе его ка- тушки тщательно изолируют как одну от другой, так и от сер- дечника. Для этого их собирают на специальном каркасе из ка- 514
кого-либо изоляционного материала (см. рис. 325), например из специального изоляционного картона. Провода для обмоток трансформатора обычно берут медные с очень хорошей изоляцией. Если обмотки трансформатора рас- полагаются коаксиально на сталь- ном сердечнике, то, как правило, обмотка высокого напряжения рас- полагается поверх обмотки низкого напряжения. Трансформаторы малой мощно- сти обычно рассчитаны на естествен- ное воздушное охлаждение. Наобо- Рис. 325. Трансформатор Ш-образной формы: I — пластины; 2 — катушки Рис. 324. Броневой трансформатор рот, трансформаторы высоких мощностей, выделяющие значи- тельное количество тепла при работе, требуют специального охлаждения, поэтому их обмотки обычно погружают в трансфор- маторное масло. На рис. 326 показаны маломощные трансформаторы со сталь- ными сердечниками, предназначенные для обслуживания цепей Рис. 326. Трансформаторы низкой частоты 33* 515
Рис. 327. Трансформатор высокой частоты с сердеч- ником из магнитодиэлек- трика Рис. 329. Мощный трансформатор 516 Рис. 328. Трансформатор трехфазного тока с низкими частотами электриче- ских колебаний. Трансформаторы данного- типа широко применяют- ся в технике связи.. На рис. 327 показан маломощ- ный трансформатор с сердечником из магнитодиэлектрика, предназ- наченный для обслуживания цепей с высокими частотами электриче- ских колебаний. Наличие магни- тодиэлектрического сердечника в этом трансформаторе сводит к ми- нимуму мощность потерь на пере- магничивание и вихревые токи. Трансформаторы данного типа ши- роко применяются в радиотехнике. На рис. 328 показан трансфор- матор трехфазного тока, применя- емый в сильноточной электротех- нике. На рис. 329 показан мощный трансформатор, применяемый в линиях передачи высокого напря- жения на большие расстояния,
Рис. 330. Схема автотрансформатора § 146. АВТОТРАНСФОРМАТОР Автотрансформатором называется такой трансфор- матор, в котором первичная и вторичная обмотки объединены в одну общую электрическую цепь. На рис. 330 приведена схема автотрансформатора, где источ- ник электрической энергии подключен ко всем виткам «ь обмотки автотрансформатора, а потреби- тель — к некоторой части этой обмотки, обладающей числом ВИТКОВ W2. При прохождении перемен- ного тока по обмотке автотранс- форматора возникает перемен- ный магнитный поток, индук- тирующий в этой обмотке элек- тродвижущую силу, величина которой пропорциональна числу витков обмотки. Следовательно, если во всей обмотке автотрансформатора, имеющей число вит- ков ИУ1, индуктируется электродвижущая сила Е2, то в части этой обмотки, имеющей число витков Шг, индуктируется электродви- жущая сила Е2. Соотношение величин этих электродвижущих сил такое, что (449) £1 _ Wt Ег w, ’ где fe — коэффициент трансформации. Так как падение напряжения в активном сопротивлении об- мотки автотрансформатора относительно мало, то им практиче- ски можно пренебречь и считать справедливыми равенства £7^ = Ех и £72 = Е2, где £7]—напряжение источника электрической энергии, подан- ное на всю обмотку автотрансформатора, имеющую число ВИТКОВ Wi, £72— напряжение, подаваемое на потребитель, снимаемое с той части обмотки автотрансформатора, которая обла- дает числом витков о»2. Следовательно, (450) Ui___ иг — wt— к> т. е. напряжение Ult приложенное со стороны источника элек- трической энергии кд всем виткам обмотки автотрансформа- тора, во столько раз больше напряжения £72, снимаемого с части обмотки, обладающей числом витков W2, во сколько раз число витков Wi больше числа витков 517
Если к автотрансформатору подключен потребитель электри- ческой энергии, то под влиянием напряжения U2 в нем возникает электрический ток, действующее значение которого обозначим /2. В соответствии с этим в первичной цепи автотрансформатора бу- дет ток, действующее значение которого обозначим 1\. Однако ток в верхней части обмотки автотрансформатора (см. рис. 330), имеющей число витков — w2), будет отличаться от тока в нижней ее части, имеющей число витков w2. Это объяс- няется тем, что в верхней части обмотки протекает только ток Л, а в нижней части — некоторый результирующий ток, представ- ляющий собой разность токов Л и /2. Дело в том, что согласно правилу Ленца индуктированное электрическое поле в обмотке автотрансформатора направлено навстречу электрическому полю, созданному в ней источником электрической энергии. Поэтому токи 1\ и /2 в нижней части обмотки автотрансформатора на- правлены навстречу друг другу, т. е. находятся в противофазе. Сами токи Л и /2, как и в обычном трансформаторе, связаны соотношением Л J_ /2 k или (451) Так как в понижающем автотрансформаторе Wi > то Z2 > Zi и результирующий ток в нижней части обмотки авто- трансформатора равен Следовательно, в той части обмотки автотрансформатора, 'с которой подается напряжение на потребитель, ток значительно меньше тока в потребителе, т. е. /2—/,«/2 Это позволяет значительно снизить расход энергии в обмотке автотрансформатора на нагрев ее проволоки и применить провод для обмотки меньшего сечения, т. е. снизить расход цветного ме- талла, уменьшить вес и габариты автотрансформатора. Если автотрансформатор повышающий, то напряжение со сто- роны источника электрической энергии подводится к части вит- ков обмотки трансформатора, а на потребитель подается напря- жение со всех его витков. Автотрансформаторы нашли широкое применение в телефон- ных аппаратах, радиотехнических устройствах, для питания вы- прямителей и т. д.
ГЛАВА XXVII ЦЕПЬ ПЕРЕМЕННОГО ТОКА С ПРИЛОЖЕННЫМ НЕСИНУСОИДАЛЬНЫМ НАПРЯЖЕНИЕМ § 147. ПРИМЕНЕНИЕ РЯДА ФУРЬЕ К ИССЛЕДОВАНИЮ СЛОЖНЫХ ПЕРИОДИЧЕСКИХ КОЛЕБАНИЙ В электротехнике часто встречаются цепи, в которых имеются негармонические колебания, или, иначе сказать, н е- с и ну сои д а л ь,н ы е токи и напря жени я. Например, в цепи, имеющей катушку со стальным сердечником, при синусои- дальном напряжении, приложенном к катушке, протекает неси- нусоидальный ток. Что же касается радиотехники, телефонии, телеграфии, то там, как правило, приходится иметь дело со сложными электри- ческими колебаниями. В основу исследования сложных периодических колебаний по- ложена теорема Фурье, согласно которой периодическую слож- ную кривую можно разложить в ряд на бесконечно большое чис- ло отдельных синусоидальных (гармонических) составляющих. Применение ряда Фурье к исследованию сложных периодиче- ских колебаний сводится к тому, что путем соответствующих математических приемов представляют данную несинусоидаль- пую величину, например, напряжение, приложенное к цепи, в виде суммы отдельных синусоидальных величин (гармоник). Число членов суммы (ряда) обычно берут небольшим, так как все остальные, отброшенные члены бесконечного ряда при сум- мировании дают такую малую величину, которой практически можно пренебречь. После этого определяют мгновенные значения тока, создаваемого каждой отдельной гармоникой напряжения, а затем эти значения складывают алгебраически и получают формулу для определения мгновенного значения результирую- щего несинусоидального тока. Разложить сложное периодическое колебание на ряд гармони- ческих колебаний — это значит определить отдельные гармони- ческие колебания, составляющие его, т. е. определить амплитуды 519
и начальные фазы отдельных гармонических колебаний, или, иначе, гармоник. Частоты отдельных гармоник сложного колебания кратны ча- стоте основного сложного колебания. При этом первая, основная гармоника имеет частоту колебаний, одинаковую с частотой сложного колебания. Вторая гармоника имеет двойную частоту, третья — тройную и т. д. Порядковое число гармоники показы- вает, во сколько раз частота ее колебаний больше основной ча- стоты колебаний, равной частоте первой гармоники. Если есть некоторая постоянная составляющая сложного ко- лебания, то ее частота принимается равной нулю и ее считают нулевой гармоникой. Амплитуды отдельных гармоник сложного колебания различ- ны и зависят от формы сложной кривой. Напряжение, изменяющееся периодически по некоторой слож- ной кривой, согласно теореме Фурье можно представить в виде следующего ряда: и = и0 + и1т • sin + Ф1) + и2т sin (2ш/ + ф2) + • • • + -l-^m-sin (W + Ф*) + ..., где и— мгновенное значение напряжения, изменяющегося периодически по некоторой сложной кривой; £70—постоянная составляющая напряжения; Uim—амплитуда первой гармоники; — фаза первой гармоники; начальная фаза первой гармоники; Ukm — амплитуда 6-й гармоники; (6ю/ + ф*) — фаза 6-й гармоники; —начальная фаза 6-й гармоники. Рис. 331. Кривая несинусоидального напряжения, со- стоящая из совокупности первой и третьей гармоник с начальными фазами, равными нулю 520
u,ut,u2 Рис. 332. Кривая несинусоидального напряжения, со- стоящая из совокупности первой и третьей гармоник, имеющих различные начальные фазы Ниже приведены гармоники нескольких наиболее типичных несинусоидальных величин: 1. На рис. 331 представлена кривая несинусоидального на- пряжения, состоящего из двух нечетных гармоник, содержащих синусы, с начальными фазами, равными нулю: и = «1 + «3 = Ulm • sin wt + U3n • sin 3<о/. (452) 2. На рис. 332 представлена кривая несинусоидального на- пряжения, состоящего из двух нечетных гармоник, содержащих синусы, с различными начальными фазами: и = «] 4- и3 — Ulm • sin + U3m• sin (3<о/ + к). (453) 3. На рис. 333 представлена кривая напряжения при однопо- лупериодном выпрямлении синусоиды. Эта кривая расклады- вается в ряд вида « = -^-(1 4- 4 -sinш/—'-Д- •cos2<o/ — A -cos4<oZ— It \ Z 1*0 0*0 2 2 \ — ^7-sin6<u/ — y-Q-cos8<o^ —...). (454) О • / / • У f 4. На рис. 334 представлена кривая напряжения при двух- полупериодном выпрямлении синусоиды, которая раскладывается в ряд вида __ Wm К 2-cos2<d/ 2*cos 4<о^ 2-cos6a>£ \ к V ЬЗ 3-5 Г7 •••/• 521
Рис. 333. Кривая напряжения при однополупериодном выпрямлении синусоиды и Рис. 334. Кривая напряжения при двухполупериодном выпрямлении синусоиды 5. На риё. 335 представлен график несинусоидального напря- жения, подводимого к телеграфному аппарату М-44 при посылке точек, разлагаемый в ряд вида « = 44 4- fsin 4- 4- • sin Зю/ 4- 4- • sin 4- 4- • sin 7&t 4- Z \ О Э / 4-... 4- у- • sin k®t 4-...), (456) где U—напряжение, приложенное к началу линии; и—мгновенное значение напряжения; k— произвольное нечетное число. Из формулы (456) следует, что постоянная составляющая равна половине приложенного напряжения и что в ряде Фурье имеются гармоники только нечетного порядка: k= 1, 3, 5 и т. д. Рис. 335. График несинусоидального напряжения в цепи при посылке точек 522
= 12 в. Пример 187. Разложить периодическую кривую напряжения (рис 335), если приложенное напряжение U = 24 в, а период колебаний Т = -у сек. Решение. Постоянная составляющая U° ~ 2 2 Амплитуда первой гармоники 2 £7 _ 2J24 тс 3,14 Угловая частота первой гармоники 2тт <•>! - у- - 15,3 в. ^-=18,84-*-. 1 сек *3 Амплитуда третьей гармоники „ 2U 2-24 Utm it-3 3,14-3 511 в' Угловая частота третьей гармоники <„3 = 3^ = 3« 18,84 = 56,52 — . сек Амплитуда пятой гармоники Г7 W 2-24 QnR = к-5 = 3,14-5 = 3,06 в’ Угловая частота пятой гармоники О, = 5Ш1 = 5 • 18,84 = 94,2 —. сек Амплитуда седьмой гармоники 2U 2-24 U’,m~ л-7” 3,14-7 ~2’ 9 Угловая частота седьмой гармоники <0, = 7®! = 7-18,84 = 131,88 — . сек Амплитуда девятой гармоники П __ 2‘24 1 7 а U*m~ п-9 3,14-9“ *' ’ Угловая частота девятой гармоники О, = 9<»t = 9 -18,84 = 169,56 — v 1 'сек и т. д. Следовательно, мгновенное значение напряжения определится формулой и = 12 4- 15,3-sin 18,84/ 4- 5,1 • sin 56,52/ 4- 3,06-sin 94,2/ 4- 4- 2,19.sin 131,88/ 4- 1,7-sin 169,56/ 4-... 523
На рис. 336 показаны графики постоянной составляющей пер- вой, третьей, пятой и седьмой гармоник и результирующее коле- бание для рассмотренного нами случая периодической посылки точек телеграфным аппаратом М-44. Для того чтобы судить о степени искажения кривой тока (на- пряжения), т. е. отличия ее от синусоиды, пользуются некоторы- ми общепринятыми коэффициентами. Коэффициент формы/Гф характеризует собой величи- ну отношения действующего зна- Постоян. составляющая Седьмая гармоника Рис. 336. Графики напряжения и его отдельных гармоник при посылке точек чения тока (напряжения) к сред- нему значению тока (напряжения) за положительную полуволну: ТТЛ' <457> Чт) где / — действующее значение тока; t I Т\ — среднее значение тока за первую половину периода; Кф — коэффициент формы. Для синусоидального тока коэффициент формы равен Коэффициент амплитуды К.ь характеризует вели- чину отношения амплитуды к действующему значению тока (на- пряжения): Ка=^> (458) где 1т — амплитуда тока; I — действующее значение тока; Ка — коэффициент амплитуды. Для синусоидального тока коэффициент амплитуды равен к. ' 1т /2 Коэффициент искажения КИ характеризует вели- чину отношения действующего значения первой гармоники тока к действующему значению несинусоидального тока: (459> 524
где Д — действующее значение первой гармоники тока; I — действующее значение несинусоидального тока; /Си — коэффициент искажения. Для синусоиды коэффициент искажения /Си= 1. Ниже приведены для сравнения коэффициенты формы, ам- плитуды и искажения для некоторых типичных сложных кривых напряжения. Из приведенной таблицы видно, что кривая напряжения тре- угольной формы имеет все коэффициенты более близкие к коэф- фициентам синусоиды, чем кривая напряжения прямоугольной формы. § 148. РЕЖИМ В ЦЕПИ ПРИ ПРИЛОЖЕННОМ К НЕЙ НЕСИНУСОИДАЛЬНОМ НАПРЯЖЕНИИ Допустим, что к цепи, состоящей из последовательно соеди- ненных активного сопротивления г, индуктивности L и емкости С (рис. 337), приложено несинусоидальное напряжение вида. и = UQ + C7lm-sinсо/ 4- Ц^-sin 3otf + + (460) Чтобы определить закон изменения мгновенного значения тока в данной цепи, необходимо определить мгновенное значение 525
тока, создаваемого каждой из отдельных гармоник напряжения, а затем, сложив их алгебраически, получим искомое решение. В рассматриваемой цепи постоянная составляющая напряже- ния Uq, как нулевая гармоника, не создает тока, так как емкост- ное сопротивление в цепи при угловой частоте со0 = 0 будет бес- конечно большим: _ 1 __ 2 _ 1 _ хсо— Woc ~ 0-С~ О ~ °° Первая (основная) гармоника напряжения создает ток в цепи, мгновенное значение которого 1\ определяется по формуле z’i= 'sin — 'Pi) = Ат •sin — <Р1)> где — угол сдвига фаз напряжения и тока первой гармоники, определяемый по его тангенсу Рис. 337. Цепь с последова- тельно соединенными г, L и С г 1 ojL------ , <оС tg?l = —— Zi—полное сопротивление цепи для тока первой гармоники, определяемое по формуле Третья гармоника напряжения создает ток, мгновенное значе ние которого /3 можно определить по формуле = J/3CT-sin^-y3) = sin (3i)/ __ где <рз — угол сдвига фаз напряжения и тока третьей гармоники, определяемый по его тангенсу tg<p3 = Зш/. — О CD С Z3 — полное сопротивление цепи для тока третьей гармоники, определяемое по формуле Zs=|/r"+(3»Z--3Jr)’. Гармоника напряжения Uk создает ток в цепи, мгновенное значение которого ik определяется по формуле 526
где Zk — полное сопротивление цепи &-й гармоники тока, опре- деляемое по формуле и — угол сдвига фаз тока и напряжения, определяемый по тангенсу его 1 г Складывая алгебраически мгновенные значения токов отдель- ных гармоник, получаем выражение для мгновенного значения несинусоидального тока в заданной цепи: i = + 4 + ... + ik + • •. =/lm-sin («>/—ft) + + /3m-sin (3oj/ — ©3) + ... +4m-sin(W — ft) + ... (461) Действующее значение тока для любой гармоники тока равно амплитудному значению тока этой гармоники, деленному на К2: Д = -4^ J Л — ~^= и т- Д » 1 J/2 2 ]/2 где /1, /г и т. д. — действующие значения токов отдельных гар- моник. Действующее значение несинусоидального тока равно корню квадратному из суммы квадратов действующих значений токов отдельных гармоник и постоянной составляющей: / = ]/72 + /f + /R--- + 7T7. . (462) Аналогично для действующего значения несинусоидального напряжения U = УЦ + Щ + ЩЛ- , (463) т. е. действующее значение несинусоидального напряжения равно корню квадратному из суммы квадратов действующих значений отдельных гармоник и постоянной составляющей. Пример 188. К цепи приложено • несинусоидальное напряжение и = 150-sin ад/ 4- 50 sin 3<о/ 4- 30-sin 5<о/. Определить показание вольтметра, подключенного к зажимам этой цепи. Решение. Действующие значения напряжений отдельных гармоник и, = = 106,3 в; и3 = ~ = 35,4 в и t/6 = - 21,26 в. /2 J/2 /2 Действующее значение несинусоидального напряжения и = уtf+7^+7^ = ]/'1063Г+ 35,42 + 21,26» = 114 в. 527
Мощность, развиваемая несинусоидальным током, равна сумме мощностей, развиваемых отдельными гармониками тока. Если активное сопротивление цепи равно /?, то мощность, расхо- дуемая в цепи, P = PQR + I*R + P2R + ... + /2/? = (/g + ZJ + /| + --+^)^ = = PR, (464) т. е. мощность равна произведению квадрата действующего зна- чения несинусоидального тока на активное сопротивление. Но эту же мощность можно выразить формулой Р= UI-cos <р, где U—действующее значение несинусоидального напряжения; /—действующее значение тока; cos <р—эквивалентный коэффициент мощности. Пример 189. К цепи, состоящей из последовательно соединенных актив- ного сопротивления г = 40 ом и индуктивности L = 0,03 гн, приложено не- синусоедальное напряжение и — 40 + 84,6 • sin со/ + 28,2 ♦ sin 3 со/. Определить мгновенное и действующее значения тока в данной цепи, а также действующее значение напряжения и активную мощность, развивае- мую током в ней, если угловая частота первой (основной) гармоники со = Решение. Нулевая гармоника напряжения создает постоянную со- ставляющую тока, равную Первая гармоника напряжения создает ток в цепи, амплитуда которого равна I = -И™. = = 84,6 = 1,692 а. А У fL + (ш£)2 У 402 + (1000 • 0,03)2 Действующее значение тока первой гармоники равно Сдвиг фаз напряжения и тока первой гармоники находим по его тан- генсу , Хх 1000-0,03 tg ср< = —- =----—----= 0,7э, & т1 г 40 откуда по тригонометрическим таблицам находим ъ = 36°50'. Мгновенное значение тока первой гармоники равно /j «= /lz72-sin (ш/— cpj в 1,692-sin (1000/—36°50')- 528
Третья гармоника напряжения создает ток в цепи, амплитуда которого равна l3m = = Uam----------= 28;2rt ....— = 0,283 а. т Z3 Уг2 + (3в>£)2 J/40* + (3-1600-0,03)2 Действующее значение тока третьей гармоники равно /з = -^ = ~ °-2 « 3 УЧ 1-41 Сдвиг фаз напряжения и тока третьей гармоники находим по его тан- генсу tg?, = ^ = ±2200^03.2,25, 6 Td г 40 откуда по тригонометрическим таблицам находим <Рз « 66°. Мгновенное значение тока третьей гармоники равно Z8 = /3w-sin (Зш/ — ?3) = 0,2-sin (3000/ — 66°). Мгновенное значение несинусоидального тока определяется по формуле I = /0 + q + /3 = 1 + 1,692-sin (1000/ — 36°50') 4- 0,2-sin (3000/ — 66°). Действующее значение несинусоидального тока равно / . у /2 + /2 + /2 = )/ Р + 1,22 + 0,22 = 1,58 д. Активная мощность, развиваемая несинусоидальным током в цепи, равна Ра = Рг = 1,582-40 = 99,2 вт. § 149. РЕЗОНАНС НАПРЯЖЕНИЙ В ЦЕПИ ПРИ НЕСИНУСОИДАЛЬНОМ ПРИЛОЖЕННОМ НАПРЯЖЕНИИ Пусть задана цепь, состоящая из последовательно соединен- ных г, L и С. К ней приложено некоторое несинусоидальное на- пряжение, например: « = Ulm • sin 4- U-Sm • sin 4- U5m • sin 5ш/. Полное сопротивление цепи зависит от величин г, L, С и ча- стоты, а поэтому оно различно для различных гармоник. Для первой гармоники оно равно Z, = |/г2 4- (&L —У- «>С/ ’ для третьей гармоники Zs = ]/ г2 + (3<oL — ; 34-1377 529
для пятой гармоники ^=)/г> + (5»£-32гг)’ И т. д. При некоторых значениях емкости С и индуктивности L для какой-либо из гармоник, например k-й, индуктивное сопротивле- ние цепи может оказаться равным емкостному сопротивлению ее: k<»L = ~. (465) В этом случае полное сопротивление цепи для й-й гармоники будет равно активному сопротивлению цепи, т. е. Zk = г, ток fe-й гармоники достигнет максимального значения и может пре- высить значения токов всех остальных гармоник. Тогда возни- кает резонанс напряжений k-й гармоники со всеми присущими ему явлениями. Пример 190. К цепи, обладающей емкостью С = 2 мкф, индуктивностью L = 0,02 гн и активным сопротивлением г = 0,1 ом, приложено несинусо- идальное напряжение и = 162,2• sin mt 4- 18,1 • sin 3a>t + 6,5-sin + 3,3-sin 7<dZ, где co = 1000 . сек Определить амплитуду гармоник тока в данной цепи. Решение. Реактивные сопротивления для гармоник: 1. Для первой гармоники X, - .1 - - llW.| lF,- - - « 2. Для третьей гармоники “ 3a>L - 3^7 “ 3-1000-°’02 - З-ЮоЛ-Ю-в ~ ~ 106 °М- 3. Для пятой гармоники Х‘ - М ~ -5Й7 - 5'100Мда - 3-1000*2. 10~. - °- 4. Для седьмой гармоники Х’ - Ы W ’ '00Мда - 7.1000 *2.10-. ” Ю Амплитуды гармоник тока: 1. Для первой гармоники J62’2 -пял а. J/0,12 + 4802 2. Для третьей гармоники 18,1 = 0,17 а. 1/0,12 + 1062 530
3. Для пятой гармоники _^=6!5 = 65. '5т ZB 0,1 °5 4. Для седьмой гармоники — Uim Z, 3,3 1/0,12 + 692 = 0,048 а. Для пятой гармоники будет резонанс напряжений. Ее ток значительно превышает токи всех остальных гармоник, взятых в отдельности. Напряжение, приложенное к емкости и индуктивности, в силу возник- шего резонанса напряжений пятой гармоники значительно превосходит дей- ствующее значение напряжения, приложенного , ко всей цепи: / U5l = U5C = IkuL = 65’5 -1000-0,02 = 6500 в. При несинусоидальной форме кривой на- пряжения, приложенного к цепи, может быть столько случаев резонанса напряжений, сколько гармоник содержит в себе несинусоидальное напряжение, приложенное к цепи. Изменяя плавно в этой цепи какую-либо из величин, например емкость, можно снять кривую так называемого сложного резонанса (рис. 338). Рис. 338. Кривая сложного резонанса § 150. ПОНЯТИЕ ОБ ЭЛЕКТРИЧЕСКИХ ФИЛЬТРАХ В технике проводной связи, при телефонировании и телегра- фировании, широко применяется одновременная работа .по одной и той же линии при помощи переменных токов различных ча- стот. Например, по одной паре бронзовых проводов, помимо обычных телефонных разговоров на частотах от 200 до 3000 гц, можно осуществить несколько телеграфных передач на частотах от 3000 до 10 000 гц и далее вновь телефонные разговоры на вы- соких частотах от 10 000 до 40 000 гц. При этом отдельные одно- временно ведущиеся передачи протекают без каких-либо помех друг другу. Уплотнение телефонно-телеграфных передач по одной и той же паре проводов дает значительную экономию проводов и рас- ширяет эксплуатационные возможности телефонно-телеграфных установок. Для многократного телефонирования и телеграфирования применяются дополнительные приборы, называемые электри- ческими фильтрами. Электрические фильтры разделяют на конце линии одни частоты от других и направляют каждую в свой аппарат. Эти фильтры широко используются и в других отраслях электротехники, например в радиотехнике, в технике сильных токов и т. п. Принцип действия фильтров основан на свойстве индуктив- ности задерживать токи высших частот и пропускать токи низ- 34* 531
ших частот и постоянную составляющую тока, а также на свой- стве емкости пропускать токи высших частот, задерживать токи низших частот и совершенно не пропускать постоянной состав- ляющей тока. Всякий фильтр представляет собой комбинацию емкости С и индуктивности L, дающую возможность пропустить к аппарату какую-либо одну из частот или полосу частот и задержать все остальные или, наоборот, пропустить все частоты, кроме за- данной. Рассмотрим наиболее типичные схемы электрических филь- тров. Фильтры нижних частот Фильтры нижних частот пропускают токи частот ниже опре- деленной заданной частоты. На рис. 339 приведена схема Т-образного фильтра нижних частот, содержащего в себе катушки индуктивности Li и L2 и кон- денсатор С. Токи низких частот легко проходят через катушки индуктивности и с трудом — через конденсатор. Наоборот, токи Рис. 340. Характеристика фильтра нижних частот Рис. 339. Схема Т-образного фильтра нижних частот высоких частот легко проходят через конденсатор и задержи- ваются катушками. Поэтому фильтр нижних частот оказывает возрастающее сопротивление токам всех частот выше резонанс- ной. На рис. 340 приведена характеристика фильтра нижних ча- стот. По горизонтальной оси отложены частоты f, а по вертикаль- ной — величины тока /, соответствующие данным частотам. Фильтры верхних частот Фильтры верхних частот пропускают токи частот выше неко- торой определенной заданной частоты. На рис. 341 приведена Т-образная схема фильтра верхних частот, содержащего конденсаторы С\ и С2 и катушку индуктив- ности L. Токи высоких частот легко проходят через конденсаторы и с трудом—через катушку индуктивности. Наоборот, токи низ- 532
——г Сг» ' L 0. — 4 ... Рис. 341. Схема Т-образного фильтра верхних частот Рис. 342. Характеристика фильтра.верхних частот ких частот легко проходят через катушку индуктивности и за- держиваются конденсаторами. Вследствие этого фильтр высокой частоты оказывает все возрастающее сопротивление токам всех частот меньше резонансной. На рис. 342 приведена характеристика фильтра верхних ча- стот. Здесь мы видим, что с ростом частоты ток, пропускаемый фильтром, возрастает. Полосовые фильтры Полосовые фильтры пропускают определенную частоту или полосу частот, задерживая все остальные частоты. На рис. 343 изображена схема простейшего полосового филь- тра, состоящего из последовательно соединенных катушки индук- тивности и конденсатора. Подобный фильтр свободно пропускает ток частоты, совпадающей с его резонансной частотой 2it]/ LC ’ задерживая в той или иной степени токи других частот. На рис. 344 показана характеристика полосового фильтра. Из этой характеристики видно, что чем более пологая резонанс- ная кривая, тем большую полосу частот пропускает фильтр. Это не всегда желательно, особенно в тех случаях, когда нужно эту полосу частот свести к минимуму или, например, пропустить только определенную частоту. В этом случае схему полосового фильтра усложняют. 0 — 0 Рис. 343. Схема простей- шего полосового фильтра Рис. 344. Характеристика полосового фильтра 533
На рис. 345 приведена схема полосового фильтра, состоящая из двух катушек индуктивности и двух конденсаторов. Участок цепи с индуктивностью L\ и последовательно соединенной с ней емкостью Ci настраивают на резонанс напряжений для ча- стоты, которую желают пропу- стить к аппарату. Контур, име- ющий индуктивность L2 и ем- кость Сг, настраивают на ре- зонанс токов заданной частоты, так что этот контур для коле- баний заданной частоты оказы- вает очень большое сопротив- ление. В результате фильтр бу- Рис. 345. Схема полосового фильтра дет пропускать строго опреде- ленную частоту, все же остальные частоты, более высокие и низ- кие, чем резонансная, не будут допущены к тому или иному при- бору. Заграждающие фильтры Заграждающие фильтры задерживают токи определенной ча- стоты или полосы частот и пропускают токи всех частот, лежа- щих за пределами этой полосы. На рис. 346 показана простейшая схема заграждающего фильтра. Здесь контур с емкостью С\ и индуктивностью L\ на- страивается на резонанс токов для некоторой заданной частоты, которую желают задержать, не допустить в аппарат. Участок цепи с индуктивностью L2 и емкостью С2 настраи- вают на резонанс напряжений заданной частоты. Следовательно, заданная частота встречает очень большое сопротивление в кон- туре L\ и С\ и незначительное — в ответвлении L2y С2у благодаря чему колебания этой частоты отсеиваются, не заходя в прием- ник. Все же остальные частоты более или менее свободно про- ходят в приемник. Рис. 346. Простейшая схема заграждающего фильтра Рис. 347. Характеристика за- граждающего фильтра На рис. 347 показана характеристика заграждающего филь- тра. Чем более пологая кривая характеристики фильтра, тем большую полосу частот задерживает фильтр.
ГЛАВА XXVIII ВЫПРЯМИТЕЛИ ПЕРЕМЕННОГО ТОКА § 151. ОБЩИЕ ПОЛОЖЕНИЯ Выпрямителями переменного тока называются приборы, пре- образующие переменные токи в токи постоянные или пульсирую- щие. В современной электротехнике, и в частности в технике связи, применяются разнообразные типы выпрямителей переменного тока, например кенотроны, ртутные и полу- проводниковые выпрямители, вибропреобразо- ватели и некоторые иные. Большинство выпрямителей переменного тока, применяемых на практике, являются элек- трическими вентилями, т. е. приборами, кото- рые обладают практически односторонней про- водимостью. Ток одного направления (прямой ток) проходит через них относительно свободно, Направление тока Рис. 348. Схемати- ческое изображе- ние выпрямителя а ток противоположного направления (обратный ток) — очень слабо. Например, кенотрон пропускает через себя ток только в на- правлении от анода к катоду, селеновый выпрямитель — от селена к катодному сплаву и т. д. На рис. 348 показано условное изо- бражение полупроводникового (твердого) выпрямителя перемен- ного тока. Здесь острие стрелки показывает направление тока через выпрямитель (направление прямого тока). В нормальных условиях эксплуатации обратный ток в выпря- мителях переменного тока настолько мал по сравнению с пря- мым током, что им практически можно пренебречь. Однако в том случае, когда на выпрямитель подается переменное напряжение, превышающее номинальное, обратный ток становится ощутимым, и при некотором предельном обратном напряжении выпрямитель пробивается и теряет свое вентильное свойство. Основными характеристиками выпрямителей переменного тока являются: допустимая температура, плотность тока и допусти- мое обратное напряжение на вентильный элемент. В последующих параграфах мы познакомимся с некоторыми типами выпрямителей и их основными характеристиками. 535
§ 152. КЕНОТРОН На рис. 349, а показана схема устройства двухэлектродной электронной выпрямительной лампы, называемой кенотроном. Кенотрон состоит из стеклянного баллона, внутри которого соз- дано высокое разряжение воздуха. В баллоне помещены два электрода, из которых один называется анодом, а другой като- дом. Анодом служит металлический цилиндр, изготовляемый обычно из никеля, тантала или молибдена. Катод, помещенный внутри цилиндрического анода, представляет собой тонкую ме- таллическую нить, изготовляемую обычно из вольфрама. Из бал- лона лампы выведены три проводника, из которых два — это концы нити катода и один — проводник, присоединенный к аноду. Все три выводных проводника припаяны к соответствующим штырькам, впрессованным в цоколе лампы. На рис. 349,6 показана принципиальная схема двухэлектрод- ной электронной лампы с подключенными к ней анодной БА и накальной Бн батареями. Рассмотрим применительно к этой схеме, в чем заключается принцип вентильного действия двух- электродной электронной лампы. Катод лампы К питается током от батареи накала 2>н. Ток, проходя через нить катода, нагревает ее, и она получает способ- ность излучать в окружающее пространство электроны. Выле- тевшие с поверхности катода электроны образуют вокруг него нечто вроде электронного облачка, создающего пространственный объемный заряд. Анод лампы соединен с положительным полюсом анодной батареи БА, другой полюс которой присоединен к катоду лампы, поэтому между анодом и катодом лампы имеется некоторая раз- ность потенциалов, равная напряжению на зажимах анодной батареи БА. Следовательно, в пространстве между анодом А и Рис. 349. Кенотрон 536
Рис. 350. Характеристика кенотрона • катодом К лампы имеется электрическое поле Е, направленное от анода к катоду. Электроны, вырвавшиеся с поверхности катода, под влия- нием сил электрического поля движутся к аноду А и, осаждаясь на его поверхности, компенсируют положительные заряды анода. Если убыль электронов на катоде и убыль положительных заря- дов ,на аноде будут пополняться от анодной батареи Б^ то электроны с катода станут непрерывным потоком перемещаться внутри лампы, т. е. через лампу ‘будет проходить термоионный электрический ток, называемый анодным током. Зависимость анодного тока в электронной лампе от анодного напряжения при постоянном токе накала нити лампы является одной из основных характеристик двухэлектродной электронной лаМпы. На рис. 350 изображена кривая этой зависимости. Здесь по горизонтальной оси отложено анодное напряжение Ua в воль- тах, а по вертикальной оси — анодный ток 1а в миллиамперах. Из этой кривой видно, что между напряжением Ua и током 1а не существует прямой пропорциональной зависимости. Анодный ток, соответствующий анодному напряжению в диапазоне от нуля до нескольких вольт, растет очень медленно и неравно- мерно. Затем, с ростом напряжения примерно до нескольких де- сятков вольт, анодный ток увеличивается более быстро и при- том равномерно и пропорционально анодному напряжению. И, наконец, с дальнейшим ростом анодного напряжения интен- сивность роста анодного тока спадает. Кривая роста тока круто поворачивает вправо и, загибаясь, становится почти горизон- тальной. То предельное значение, которого достигает анодный 537
ток при своем увеличении с ростом анодного напряжения, назы- вается током насыщения. Сложную зависимость анодного тока 1а от анодного напря- жения Ua можно объяснить следующим образом. Когда анодное напряжение равно нулю, то анод не имеет положительного потенциала, электроны, вылетающие с катода лампы, не притягиваются анодом и образовавшееся вокруг ка- тода электронное облачко отталкивает электроны, вылетающие с поверхности катода. С увеличением напряжения на аноде он заряжается положительно и притягивает к себе часть электро- нов из электронного облачка, т. е. в цепи создается анодный ток. Но при анодном напряжении от 0 до 10 в анодный ток мал, так как еще очень слабо электрическое поле внутри лампы и электронное облачко оказывает большое противодействие вылету электронов с катода лампы. При увеличении напряжения от 10 до 50 в электрическое поле сильно растет, электронное облачко частично рассеивается, электроны получают возможность более свободно вылетать с катода лампы и анодный ток увеличивается. При дальнейшем увеличении анодного напряжения от 50 до 80 в электрическое поле становится настолько сильным, что элек- тронное об!лачко совершенно исчезает и электроны, покидающие катод, беспрепятственно достигают анода. Дальнейшее увеличе- ние напряжения не увеличивает анодного тока, так как все вы- летающие с катода электроны попадают на анод, и в анодной цепи устанавливается ток насыщения. Если необходимо дальше увеличивать анодный ток, то надо увеличить эмиссию, или излу- чение электронов с катода, повысив температуру его путем уве- личения тока накала. Теперь изменим полярность электродов электронной лампы, подключив к электроду А лампы отрицательный полюс анодной батареи Бм а к электроду К — положительный полюс ее. В этом случае электроны, вылетающие из накаленной нити лампы, не только не будут притягиваться электродом Д, но даже станут отталкиваться от него. Следовательно, направленного движения потока электронов внутри лампы от электрода К к электроду А не будет, а это значит, что электрический ток в цепи бата- реи БА—лампа протекать не будет. Таким образом, двухэлек- тродная электронная лампа (кенотрон) обладает вентильными свойствами, т. е. способностью пропускать через себя ток только в том случае, если на ее электрод А (анод) подан положитель- ный потенциал, а на электрод К (катод) — отрицательный потен- циал от анодной батареи БА. Если к электродам кенотрона подключить переменное сину- соидальное напряжение, то при подаче на анод положительной полуволны напряжения (потенциала) в кенотроне будет прохо- дить анодный ток, а при подаче отрицательной полуволны тока не будет. 538
На рис. 351 показана схема двухэлек- тродного кенотрона, подключенного к ис- точнику электрической энергии с синусои- дальной электродвижущей силой. На рис. 352 показаны синусоида напряжения, подаваемого на электроды кенотрона, и график коммутированного напряжения U на активном сопротивлении R, включен- ном в анодную цепь лампы. Нетрудно видеть, что ток во внешней цепи кенотрона (рис. 351) проходит от нити лампы (катода) через нагрузочное сопротивление R к аноду лампы. Следовательно, кенотрон, включенный в цепь переменного тока, можно рассма- Рис. 351. Схема включе- ния двухэлектродного кенотрона в сеть пере- менного тока тривать по отношению к нагрузочному сопротивлению R (потребителю) как ис- точник пульсирующего тока, т. е. перемен- ного по величине, но неизменного по на- правлению тока. При этом катод кенотро- на служит гюложительным, а анод — отрицательным полюсом этого источника. Существенный недостаток схемы однополупериодного выпря- мителя переменного синусоидального тока заключается в том, что здесь использована только одна положительная полуволна подаваемого на электроды синусоидального напряжения, в ре- зультате чего постоянная составляющая выпрямленного напря- жения относительно мала: ' £7= 0,318 £4, (466) Рис. 352. Однополупериодное вы- прямление переменного синусоидаль- ного напряжения двухэлектродным кенотроном где U — постоянная составля- ющая выпрямленного напряжения; Uт — амплитуда пульсирую- щего напряжения. Для улучшения работы ке- нотрона применяются схемы с двухполупериодным выпрямле- нием — с двумя спаренными кенотронами или с одним ке- нотроном, имеющим два анода и один катод (двуханодный ке- нотрон). На рис. 353 приведена одна из практических схем двухполу- периодного выпрямления с по- мощью двуханодного кенотрона. 539
Здесь кенотронный выпрямитель получает электрическую энергию от сети переменного тока через первичную обмотку си- лового трансформатора Тр. Понижающая вторичная обмотка трансформатора, имеющая на своих выходных зажимах напря- жение в несколько вольт, питает переменным током нить накала кенотрона. Повышающая вторичная обмотка, имеющая на за- жимах напряжение обычно в несколько сот вольт, подключена к кенотрону так, что один из ее концов присоединен к первому 1, а другой — ко второму 2 аноду лампы. Средняя же точка этой обмотки соединена через нагрузочное сопротивление R с катодом кенотрона. Таким образом, повышающая обмотка трансформа- тора разделена на две секции, из которых каждая связана с анодной цепью кенотрона только через свой анод. Рис. 353. Схема двухполупериодного выпрямления с помощью двуханодного кенотрона Если одна из секций повышающей обмотки трансформатора, например секция 1, подает на свой анод 1 положительный по- тенциал, то в это же время секция 2 подает на свой анод 2 отрицательный потенциал. Спустя некоторое время, наоборот, секция 1 будет подавать на свой анод 1 отрицательный потен- циал, а секция 2 — подавать на свой анод 2 положительный по- тенциал. В результате этого попеременно будут выпрямляться как положительная, так и отрицательная полуволны синусои- дального напряжения (рис. 354). Постоянная составляющая коммутированной синусоиды при двухполупериодном выпрямлении равна и = 0,636 Um, (467) где U — постоянная составляющая коммутированной сину- соиды напряжения; Um — амплитуда пульсирующего напряжения. 540
Так как на каждый из анодов двуханодного кено- трона подается только по- ловина электродвижущей силы, индуктированной в повышающей обмотке си- лового трансформатора, то соответственно этому и выпрямленное напряже- ние, подаваемое кенотро- ном на потребитель, будет заниженным. Чтобы повысить его, применяют схемы с че- тырьмя однополупериод- ными кенотронами. Здесь за один полупериод рабо- тают два соединенных по- следовательно кенотрона, а за другой полупериод — два других кенотрона. Таким образом, потре- битель получает напряже- ние вдвое большее по. срав- нению с тем, которое он получал бы от-двухполу- ние переменного синусоидального напря- жения двуханодным кенотроном: а — напряжение трансформатора; б — напряжение» выпрямленное первым анодом; в — напряжение, вы- прямленное вторым анодом; г — напряжение на на- грузочном сопротивлении периодного кенотрона. Кенотрон подает на нагрузочное сопротивление пульсирующее напряжение, содержащее постоянную и переменную составляю- щие. Однако для питания, например, анодных цепей радиоприем- ника требуется постоянный ток, т. е. ток, не изменяющийся во вре- мени по величине. Следовательно, возникает необходимость как-то отсеять от пульсирующего тока его переменную составляющую, не допустить ее в нагрузочное сопротивление кенотрона, пропу- Рис. 355. Принципиальная схема кенотронного выпрямителя со сгла- живающим фильтром 541
стив туда только постоянную составляющую. Для этой цели при- меняют сглаживающие фильтры, включаемые между кенотроном (выпрямителем) и нагрузочным сопротивлением. ' На рис. 355 показана принципиальная схема кенотронного выпрямителя со сглаживающим фильтром, где схема фильтра выделена пунктирной четырехугольной рамкой. Сглаживающий фильтр состоит из дроссельной катушки Др и конденсаторов Ci и С2. Дроссельная катушка Др имеет стальной сердечник, обычно с воздушным зазором; на сердечник насажена обмотка, имею- щая несколько тысяч витков проволоки. Такой дроссель низкой частоты имеет индуктивность порядка 20—30 гн. Конденсаторы сглаживающего фильтра, из которых один подключается параллельно выходу кенотрона, а другой — парал- лельно нагрузочному сопротивлению R кенотрона, обычно имеют большую емкость—10 мкф и больше. Для этой цели чаще всего применяют электролитические конденсаторы, обладающие большими емкостями и в то же время относительно небольшими размерами и весом. Необходимо напомнить, что электролитические конденсаторы имеют строго определенную полярность, т. е. они играют роль конденсатора только в том случае, когда плюс источника элек- трической энергии, в данном случае кенотрона, соединен с поло- жительным полюсом конденсатора (центральный вывод конден- сатора), а минус — с отрицательным (с корпусом конденсатора). В этом случае постоянная составляющая пульсирующего тока, создаваемого кенотроном, не проходит через конденсатор. Конденсатор подключенный параллельно выходу кено- трона, отделяет значительную долю переменной составляющей пульсирующего тока, которая, минуя остальную часть фильтра, замыкается через кенотрон. Дроссель Др, как обладающий большим индуктивным сопро- тивлением, значительно уменьшает проходящую через него пере- менную составляющую пульсирующего тока и почти беспрепят- ственно пропускает постоянную составляющую его. И, наконец, конденсатор С2 подобно конденсатору Ci отде- ляет значительную долю переменной составляющей пульсирую- щего тока, так что через нагрузочное сопротивление проходит уже практически постоянный ток. Надо сказать, что конденсатор Ci сглаживающего фильтра заряжается весьма быстро в тот отрезок времени, когда нара- стает подаваемое на него кенотроном пульсирующее напряжение, и, наоборот, разряжается относительно медленно в тот отрезок времени, когда пульсирующее напряжение, подаваемое кенотро- ном, убывает. Это объясняется тем, что конденсатор не может разрядиться через кенотрон, так как электроны с его об- кладки не могут перемещаться от анода к катоду кенотрона. Поэтому единственный путь разряда конденсатора Ci — это дрос- 542
сель Др и нагрузочное сопротивление R. Но эта цепь обладает, как правило, большим сопротивлением из-за нагрузочного сопро- тивления /?, а поэтому разряд конденсатора Ci протекает отно- сительно медленно. Он еще не успевает разрядиться полностью, как уже снова начинает заряжаться кенотроном до амплитуд- ного значения напряжения. В результате постоянная составляю- щая выпрямленного напряжения на выходе фильтра повышается. Следовательно, конденсатор Ci сглаживающего фильтра не только сглаживает пульсацию напряжения, но и’ повышает его постоянную составляющую. Рис. 356. Графики напряжения на выходе сглаживающего фильтра На рис. 356 показаны для сравнения графики напряжения на нагрузочном «сопротивлении однопол у пер иодного кенотрона, ра- ботающего со сглаживающим фильтром и без него. Сглаживающий фильтр не дает идеального постоянного на- пряжения на нагрузочном сопротивлении кенотрона. Это объяс- няется тем, что сопротивление дросселя не является бесконечно большим для переменной составляющей пульсирующего тока и что емкостное сопротивление конденсаторов для этой составляю- щей не бесконечно мало. Поэтому на нагрузочном сопротивле- нии создается напряжение с некоторой, правда относительно малой, переменной составляющей. Для практики считается вполне приемлемым, если фильтр ослабит переменную составляющую пульсирующего напряжения кенотрона примерно в 50 раз. В этом случае напряжение, пода- ваемое с нагрузочного сопротивления на аноды ламп радиопри- емника, не будет создавать звуковой фон переменного тока низ- кой частоты колебаний в репродукторе приемника. Чем выше частота пульсирующего тока, тем больше индук- тивное сопротивление дросселя и меньше емкостное сопротивле- ние конденсатора для переменной составляющей этого тока и тем лучше работа сглаживающего фильтра. Отсюда следует, что двухполупериодная схема кенотрона (двуханодного кено- 543
Рис. 357. Кенотроны трона) дает лучшее выпря- мление пульсирующего тока, чем однополупериодная, так как в первом случае частота переменной составляющей этого тока в два раза боль- ше, чем во втором случае. Кенотрон обладает срав- нительно большим внутрен- ним сопротивлением — в не- сколько сотен или тысяч ом. Следовательно, его надо рассматривать как источник пульсирующего тока со срав- нительно большим внутрен- ним сопротивлением. Вели- чина напряжения на зажимах кенотрона сильно зависит от его нагрузки из-за относительно большого падения напряжения внутри кенотрона. Если нагрузка (ток) кенотрона мала, что бы- вает при большом внешнем сопротивлении, то напряжение на выходных зажимах становится относительно большим и возни- кает опасность пробоя конденсаторов фильтра. Эта опасность особенно возрастает при холостом ходе выпрямителя, когда он не дает тока во внешнюю цепь. Отсюда следует весьма важный практический вывод: выпрямитель никогда не следует включать под напряжение электросети без нагрузки. 'Таблица 36 Основные данные некоторых типов кенотронов Тип кенотрона Число анодов^ Напряже- ние накала, в Ток накала, а Максимально допустимый выпрямленный ток, ма Максимально допустимая амплитуда обратного напряжения, в Среднее внут- реннее сопро- тивление на анод, ом 5Ц4С 2 5,0 2,0 125 1350 150 6Х6С 2 6,3 0,3 16 420 500 1Ц1 1 0,7 0,185 1,0 10 000 6000 На рис. 357 показан внешний вид кенотронов типа Щ1С и 6Х6С. § 153. РТУТНЫЕ ВЫПРЯМИТЕЛИ Ртутный выпрямитель, изображенный на рис. 358, состоит из герметически закрытой стеклянной или металлической колбы, наполненной парами ртути и имеющей четыре электрода. Основ- ные аноды выпрямителя — два верхних электрода Д1 и Л2, изго- товленные из железа или графита. Нижние два электрода 544
К и Л3 — ртутные, из них элек- трод К — катод ртутного выпря- мителя, а электрод А3 — вспомо- гательный анод для зажигания ртутного выпрямителя. Наличие двух анодов Д1 и А2 обеспечи- вает двухполупериодное выпрям- ление переменного^ тока выпрями- телем. Питание от сети перемен- ного тока выпрямитель получает через трансформатор Гр. Один из проводов, по кото- рым подается постоянный ток от выпрямителя к потребителю R, подключен к катоду выпрямителя, а другой — к средней точке трансформатора. Вспомогатель- ный электрод Л3 подключен к виткам трансформатора через до- полнительное сопротивление г и выключатель В. Принцип действия ртутного выпрямителя сводится к следую- щему. Для того чтобы запустить в ход выпрямитель, необходимо подать переменное напряжение н нуть выключатель В в цепи вспо Рис. 358. Схема ртутного выпря- мителя трансформатор, затем замк- >гательного электрода Л3 и слегка наклонить колбу выпрямителя, чтобы часть ртути из электрода Л3 перешла к электроду /(. Когда оба эти электрода будут электрически соединены тонкой струйкой стекающей рту- ти, то в цепи со вспомогательным электродом Л3 возникнет элек- трический ток. При возвращении колбы в вертикальное поло- жение и разрыве ртутной струйки в цепи вспомогательного элек- трода образуется электрическая дуга, которая перекидывается и на главные электроды А и Л2. Действие выпрямителя осно- вано на свойстве., электрической дуги, возникающей между ртутью и твердым электродом, пропускать ток лишь в направле- нии от твердого электрода к ртути. Источником излучения электронов в катоде является светя- щееся пятно на поверхности ртути. Электроны, вылетевшие с ка- тода, направляются к тому аноду, который в данный момент имеет положительный потенциал. По пути часть электронов сталкивается с молекулами паров ртути и ионизирует их. Поло- жительные ионы движутся под влиянием сил поля к катоду, сталкиваются с ним и отдают ему свою кинетическую энергию. Последняя преобразуется в нем в тепловую энергию, и этим поддерживается относительно высокая температура светящегося 35—1377 545
пятна на ртути. Благодаря относительно высокой температуре светящегося пятна (несколько сот градусов) катода и достаточно сильному электрическому полю между катодом и слоем положи- тельных ионов, окружающих катод, создается эмиссия электро- нов катодом. Светящееся пятно на катоде и, следовательно, электрическая дуга могут существовать в ртутном выпрямителе только при на- личии в нем тока. Выключение тока, даже на весьма малое время, связано с прекращением работы выпрямителя. Чтобы его вновь заставить работать, необходимо заново повторить пусковой процесс зажигания электрической дуги. Электрическая дуга ртутного выпрямителя может погаснуть и при наличии в нем тока, если он станет меньше минимального, в несколько ампер. Для бесперебойной работы выпрямителя при резко изменяю- щейся нагрузке в мощных ртутных выпрямителях введены вспо- могательные электроды, работающие на некоторое постоянное сопротивление. В этих же целях в схеме ртутного выпрямителя предусмотрен дроссель L (рис. 358), который сглаживает пуль- сацию выпрямленного тока и препятствует быстрому спаду тока, когда переменный ток проходит нулевое значение. На рис. 359 показана вольтамперная характеристика ртутного выпрямителя. Здесь можно Ъидеть, что при малых напряжениях на выпрямителе между током / и напряжением U существует линейная зависимость (участок кривой оа). При этом режиме скорость движения электронов внутри колбы выпрямителя от- носительно мала из-за слабого электрического поля и поэтому ионизация молекул ртути практически отсутствует. Ток в выпря- мителе обусловлен только потоком электронов, движущихся от катода к аноду. Дальнейший рост напряжения от точки а до точки b не влияет на величину тока в выпрямителе, так как при данном напряжении ток, обусловленный эмиссией электронов катодом, достигает насыщения, т. е. все электроны, излучаемые Рис. 359. Вольтамперная характеры стика ртутного выпрямителя катодом, попадают на анод. Однако при данном режиме электроны еще не достигают такой скорости движения, при которой они смогли бы осуществить ионизацию мо- лекул ртути. Когда же на- пряжение на выпрямителе достигает величины, соответ- ствующей точке b на кри- вой oabc, наступает процесс ионизации молекул ртути, сопровождаемый резким уве- личением тока в выпрями- теле. Теперь ток / снова изменяется по линейному 546
закону в зависимости от напряжения U (участок be), но кру- тизна подъема его значительно больше, чем на участке оа. Ртутные выпрямители широко применяются в электротехнике. Они обладают рядом положительных свойств. Их коэффициент полезного действия доходит до 80—85%. Они могут выпрямлять очень большие токи, в сотни или даже тысячи ампер. Падение напряжения в них практически не зависит от величины на- грузки. Однако наряду с этим ртутные выпрямители имеют ряд суще- ственных недостатков. Они чувствительны к перегрузкам, пре- кращают работу при малой нагрузке (2—3 а) из-за прекращения эмиссии катодом электронов, а при превышении номинального напряжения в них возникает явление обратного зажигания. § 154. МЕДНО-ЗАКИСНЫЙ ВЫПРЯМИТЕЛЬ Медно-закисные выпрямители относятся к категории полупро- водниковых (твердых) выпрямителей переменного тока. На рис. 360 схематически изображено устройство медно-за- кисного выпрямительного элемента. Он состоит из медного ди- ска 1 толщиной 1 —1,5 мм, играющего роль катода выпрямитель- ного элемента. На одну из сторон диска нанесен путем термиче- ской обработки тонкий слой закиси меди 2 толщиной 0,08— 0,1 мм, играющий роль анода. К этому слою крепко прижат свинцовый диск 3 толщиной 0,3—0,5 мм, служащий для предо- хранения закиси меди от повреждения и для электрического кон- такта с ней. Шайба 4 и радиаторная пластина 5 служат для отвода тепла от выпрямительного элемента. Шайба 6 и трубка 8 изолируют электроды от болта 10, на котором собраны все де- тали выпрямительного элемента. Вентильное действие медно-закисного выпрямительного эле- мента в основном сводится к следующему. В результате терми- ческой обработки медного диска ме- жду ним и закисью меди обра- зуется запирающий слой толщиной 10-4—10“5 мм, обладающий одно- сторонней проводимостью. Свобод- ные электроны относительно легко проникают через этот слой из мед- ного диска (катода) 1 к закиси меди (аноду) 2, а в обратном направле- нии в нормальных условиях эксплу- атации электроны весьма слабо про- ходят через этот слой. Это значит, что прямой ток проходит через мед- но-закисный выпрямительный эле- мент от закиси меди 2 к меди 1. На рис. 361 приведена вольтам- Рис. 360. Медно-закисный вы- прямительный элемент 35* 547
перная характеристика медно-закисного выпрямителя. Так как обратный ток выпрямителя относительно мал, то масштабы пря- мого и обратного токов взяты различными: масштаб прямого тока — в амперах, а обратного Рис. 361. Вольтамперная характеристика медно-закисного выпрямителя — в миллиамперах на миллиметр. Напряжение, получаемое от медно-закисного выпрями- тельного элемента, не пре- вышает 6 в, а поэтому для выпрямления более высоких напряжений эти элементы соединяются последователь- но в столбики. На рис. 362 показан внешний вид такого столбика. Рис. 362. Столбик медно-закисного выпрямителя Во избежание пробоя полупроводника обратное напряжение в выпрямителе не должно превышать 5—10 в на один элемент. Плотность тока в медно-закисном выпрямительном элементе при окружающей температуре 20° Ц не должно превышать 0,044-0,06 а!см2, а при искусственном охлаждении — 0,14- 4-0,2 а/см2. Допустимая температура элемента — не выше 50° Ц. С повы- шением температуры вентильные свойства выпрямителя сни- жаются, а поэтому рекомендуется не переходить за пределы до- пустимой для элемента температуры. Коэффициент полезного действия медно-закисных выпрями- телей колеблется в пределах 60—70%. К достоинствам медно-закисного выпрямителя следует отне- сти простоту конструкции, удобство эксплуатации и запуска. Недостатки его — малая величина обратного напряжения, боязнь перегрева, низких температур и относительно большой вес. § 155. СЕЛЕНОВЫЙ ВЫПРЯМИТЕЛЬ Селеновый выпрямитель относится к категории полупровод- никовых (твердых) выпрямителей переменного тока. 543
На рис. 363 схематически изображено устройство селенового выпрямительного элемента. Он состоит из никелированного сталь- ного или алюминиевого диска 1 толщиной 0,7—1,5 мм, покры- того с одной стороны слоем селена 2 толщиной 0,05—0,1 мм. В свою очередь на селен нанесен тонкий покровный слой сплава из кадмия, олова и висмута 3 толщиной 0,05 мм, играющий роль Рис. 363. Селеновый выпря- мительный элемент Рис. 364. Вольтамперная характери- стика селенового выпрямительного элемента катода Шайба 4 служит для отвода тепла, а пружины 5 яв- ляются токоотводами. Действие селенового выпрямительного элемента в основном сводится к следующему. Между селеном и покровным слоем 3 имеется запирающий слой толщиной 10”4—ДО”5 мм, который об- ладает практически односторонней проводимостью. Свободные электроны относительно легко проникают через него в направле- нии от покровного слоя (катода) к никелированному диску (ано- ду), а в обратном направлении в нормальных условиях эксплуата- ции они проходят через этот слой весьма слабо. Следовательно, прямой ток в селеновом выпрямительном элементе проходит в направлении от никелированного диска к покровному слою. На рис. 364 приведена вольтамперная характеристика селе- нового выпрямительного элемента. Так как обратный ток его значительно меньше прямого, то масштабы плотностей этих токов взяты различными, а соответственно этому и масштабы напря- жений взяты также различными. Напряжение, получаемое от одного элемента, не должно пре- вышать 15 в, иначе может появиться сильный обратный ток 549
(т. е. ток в направлении от катодного сплава к аноду), который может вывести элемент из строя. Действующее значение допустимого обратного напряжения на элемент не должно превышать 20 в. Плотность тока при окружающей температуре 20° Ц не дол- жна превышать 0,04—0,05 а/см2. ‘ Максимальная допустимая температура — не выше 70° Ц. Коэффициент полезного действия селенового выпрямителя ко- леблется в пределах 50—80 %. Селеновые элементы, соединенные последовательно, образуют селеновые столбики. Селеновые выпрямители по своим качествам выше, чем медно-закисные. Они рассчитаны на более высокое обратное напряжение, допускают больший нагрев и имеют меньший вес и больший к. п. д. Достоинства селеновых выпрямителей — большой срок служ- бы (до 25000 рабочих часов), механическая прочность, простота конструкции, удобство эксплуатации и пуска; недостатки — спо- собность терять вентильные свойства при бездействии в течение нескольких месяцев и увеличение внутреннего сопротивления по мере эксплуатации. § 156. ВИБРАЦИОННЫЙ ПРЕОБРАЗОВАТЕЛЬ Вибрационные преобразователи предназначаются для пита- ния анодных цепей радиостанций малой мощности. На рис. 365 приведена принципиальная схема вибропреобразователя. В нее входят три основных элемента: вибратор, состоящий из якоря Я, электромагнита Э и четырех контактов а, Ь, с и d, с которыми Рис. 365. Принципиальная схема преобразователя 550
контактирует якорь при своих колебаниях; повышающий транс- форматор Тр, обмотки которого имеют выводы от своих средних точек О] и О2, и фильтр, сглаживающий колебания высокого напряжения, получаемого со вторичной обмотки трансформатора и выпрямленного вибратором. Вибропреобразователь получает электрическую энергию от аккумуляторной батареи, т. е. питается постоянным током низ- кого напряжения. Если замкнуть рубильник Р, то ток 1\ поте- чет от плюс-зажима батареи низкого напряжения через обмотку электромагнита Э в нижнюю часть первичной обмотки транс- форматора Тр до точки 01 и отсюда по соединительному провод- нику влево через дроссель Др — к минус-зажиму источника элек- трической энергии. Под влиянием тока электромагнит Э срабо- тает, притянув к себе якорь вибратора, который, заняв нижнее положение, замкнет контакты а, Ъ и при этом обмотка электро- магнита Э будет шунтирована якорем. Тогда ток 1\ от точки 2 пройдет через якорь вибратора, минуя обмотку электромагнита, и через контакт Ь, точку 3 схемы, соединительный проводник, нижнюю половину первичной обмотки трансформатора, среднюю точку 01 и дроссель придет к минус-зажиму источника электри- ческой энергии. Пока в нижней половине первичной обмотки трансформатора ток возрастает, протекая по схеме вверх к точке 01, во вторич- ной обмотке индуктируется ток /2, который от средней точки 02 трансформатора проходит по соединительному проводнику схемы вниз к контакту а вибратора, а отсюда через якорь, точки 2 и 1 схемы, фильтр и плюс-зажим выхода вибропреобразователя — к потребителю и затем возвращается через фильтр к средней точке трансформатора 02. Шунтирование обмотки электромагнита якорем ведет к тому, что катушка теряет свои электромагнитные свойства и якорь в силу своей упругой деформации отрывается от контактов а, Ь, стремясь занять исходное положение. Но вследствие инерции якорь переходит свое положение устойчивого равновесия, кон- тактирует с контактами с, d вибратора и занимает верхнее поло- жение. Тогда ток /1 от источника электрической энергии низкого напряжения проходит через рубильник Р, точки 1 и 2, якорь Я, контакт d и верхнюю половину первичной обмотки трансформа- тора к средней точке 0\ и отсюда, через дроссель Др — к минус- зажиму источника электрической энергии. При нарастании тока в верхней половине первичной обмотки трансформатора во вто- ричной обмотке индуктируется ток /2, который идет от средней точки 02 трансформатора к контакту с вибратора и отсюда через якорь Я, точки 2 и 1 — к фильтру и потребителю, а затем воз- вращается через , фильтр к средней точке 02 трансформатора. Следовательно, в цепи высокого напряжения ток вторичной об- мотки трансформатора проходит к потребителю в том же-направ- лении, что и в предыдущем случае, т. е. когда якорь соприка- 551
сался с контактами а, 6, и вибратор выпрямляет ток высокого напряжения, индуктированный во вторичной обмотке трансфор- матора. При отрыве якоря от контактов а, b цепь, шунтирующая катушку электромагнита Э, разрывается и электромагнит, воз- действуя на якорь, заставляет его вновь возвратиться в нижнее положение и т. д. В результате якорь совершает механические колебания определенной частоты, а во вторичной обмотке транс- форматора происходит непрерывный процесс генерирования тока высокого напряжения, который выпрямляется при помощи ви- братора.
ГЛАВА XXIX ЭЛЕКТРОМАГНИТНОЕ ПОЛЕ § 157. КРАТКИЕ СВЕДЕНИЯ ИЗ ИСТОРИИ РАЗВИТИЯ УЧЕНИЯ ОБ ЭЛЕКТРОМАГНИТНОМ ПОЛЕ Современное учение об электромагнитном поле возникло не сразу, а развивалось постепенно, в соответствии с потребностями общества в более интенсивном и разностороннем использовании электрической энергии. Прошло много веков с тех пор, когда пытливый ум человека впервые среди разнообразнейших явлений природы подметил электрические и магнитные явления. До XIX века электрические и магнитные явления изучались как совершенно обособленные и не зависимые друг от друга, о внутренней причинной связи их еще ничего не было известно. Это был век, когда в науке безраздельно господствовала мета- физика и классическая механика, когда пространство и время считались обособленными категориями, не зависимыми от мате- риального мира, когда признавался принцип дальнодействия как единственно правильная теория, объясняющая механизм взаимо- действия материальных тел. Все это наложило свой отпечаток и на учение об электричестве и магнетизме. Например, чтобы объяснить природу магнитных явлений с позиции метафизики и классической механики, понадобилось ввести гипотезу о нали- чии в природе невесомого вещества — магнетизма, от количества которого зависит сила всякого магнита. Начало XIX века ознаменовалось рядом открытий в области электромагнитных явлений. Английский физик М. Фарадей от- крыл в 1831 г. явление электромагнитной индукции. В результате своих исследований в области электромагнитных явлений Фара- дей предложил гипотезу, которая и была положена в основу раз- вития учения об электромагнитном поле. Согласно этой гипотезе среда, в которую помещены взаимодействующие электрические заряды, магнитные полюсы или проводники с токами, принимает непосредственное участие в их взаимодействии и является как > 553
бы передаточным звеном в этом физическом процессе. Простран- ство, в котором размещены взаимодействующие электрические заряды или магнитные полюсы и т. д., находится в состоянии особого напряжения, которое мы можем обнаружить, наблюдая действие электрических и магнитных сил. Чем больше вели- чина этих сил, тем в состоянии большего напряжения, большей деформации находится та среда, в которой наблюдаются эти силы. Русский академик Э. X. Ленц открыл один из важнейших за- конов электромагнитной индукции, вошедший в науку под назва- нием правила Ленца. В своих работах Ленц обосновал глубокую органическую связь между электрическими и магнитными явле- ниями и этим содействовал укреплению и развитию идеи об электромагнитном поле. Ленц впервые в науке установил прин- цип обратимости электрических машин и этим способствовал раз- витию практической электротехники. Английский физик Максвелл облек в математическую форму идею об электромагнитном поле. Уравнения Максвелла устанав- ливают математическую зависимость между электрическими и магнитными силами в электромагнитном поле. Решения этих уравнений показали, что электромагнитные возмущения в про- странстве должны распространяться волнообразно с некоторой конечной скоростью, зависящей от среды, в которой происходит распространение. Электромагнитные волны, по Максвеллу, дол- жны преломляться и отражаться от поверхностей физических тел так же, как и световые лучи. Русский физик Н. А. Умов разрешил ряд физических про- блем, сыгравших важную роль в развитии учения о распростра- нении электромагнитной энергии в пространстве. Он предложил теорему о том, что «количество энергии, проходящей через эле- мент поверхности тела в единицу времени, равно силе давления или напряжения, действующей на этот элемент, умноженной на скорость движения элемента». Несколько позднее эта теорема Умова была применена к электромагнитным явлениям. В области исследования электромагнитных колебаний и их практического применения исключительно велика заслуга рус- ского физика А. С. Попова — изобретателя радио. А. С. Попов впервые в мире осуществил в 1895 г. радиотелеграфную пере- дачу и этим положил начало развитию новой отрасли электро- техники — радиотехники. Наш народ гордится тем, что родиной радио является наша страна. Изобретение радио А. С. Поповым весьма ценно в том отно- шении, что оно органически связало теоретические исследования электромагнитного поля с практикой и этим способствовало грандиозному прогрессу развития радиотехники. Русский физик П. Н. Лебедев дал ряд весьма важных поло- жений в области теоретического и практического исследования электромагнитных полей. Им впервые был практически установ- 554
лен факт давления световых лучей, и этим окончательно была доказана справедливость электромагнитной теории света. «Всю жизнь я воевал с Максвеллом, — говорил Кельвин Ти- мирязеву, — не признавая светового давления, и Ваш Лебедев заставил меня сдаться перед его опытами». Исключительно велика заслуга советских ученых в развитии учения об электромагнитных явлениях, в деле исследования во- просов практического применения электромагнитных полей. Мы гордимся именами таких советских ученых, как академики и профессора М. А. Бонч-Бруевич, Н. Д. Папалекси, М. В. Шулей- кин, А. А. Чернышев, В. А. Веденский, В. Ф. Миткевич, В. А. Фок, А. М. Берг, члены-корреспонденты Академии наук СССР В. П. Вологдин, А. Н. Щукин и др., которые своими на- учными трудами и изобретениями в области радио во многих случаях далеко опередили зарубежных ученых и инженеров. § 158. ЭЛЕКТРОМАГНИТНОЕ ПОЛЕ Среди разнообразнейших явлений природы важную роль играют электромагнитные явления. Особый вид материи, с помощью которого осуществляются электромагнитные явления (электромагнитная форма движения материи), называется электромагнитным полем. Электромагнитное поле обладает рядом свойств, наглядно подтверждающих его материальную сущность. Например, оно, как и всякое вещественное тело, обладает массой, энергией, ко- личеством движения, производит давление на физические тела, возникает за счет вещества и, наоборот, преобразуется в веще- ство. Электромагнитное поле обладает также рядом таких свойств, которые отличают его от вещества. Например, масса свободного электромагнитного поля — не механического происхождения, т. е. свободное электромагнитное поле не имеет массы покоя и вся его масса обусловлена движением. Электромагнитное поле в отличие от вещества проницаемо, т. е. в одном и том же объеме пространства может быть несколько электромагнитных полей, образующих совместно единое электромагнитное поле, напряженность которого определяется как геометрическая сумма напряженностей составляющих полей. Современная наука установила, что электромагнитное поле как некоторый вид материи обладает одновременно такими про- тиворечивыми свойствами, как прерывность и непрерывность. Свойство прерывности (корпускулярности) электромагнит- ного поля проявляется в том, что оно в электромагнитных явле- ниях выступает как совокупность элементарных невещественных частиц — фотонов (поток фотонов), мчащихся в пространстве со скоростью света. Фотоны в отличие от элементарных веществен- ных частиц, например электронов, протонов и т. д., не обладают 555
массой покоя. Они находятся в постоянном движении, и их масса — электромагнитного происхождения. Всякая остановка движения фотона связана с его исчезновением и переходом его массы и энергии в массу и энергию какой-либо иной материаль- ной частицы. Свойство непрерывности (волновое свойство) электромагнит- ного поля проявляется в том, что оно выступает в электромаг- нитных явлениях в форме электромагнитных волн. Наличие у электромагнитного поля одновременно двух проти- воречивых свойств — прерывности и непрерывности —свиде- тельствует о том, что в действительности электромагнитное поле не является ни частицами, ни волнами в буквальном смысле этого слова. Электромагнитное поле — это вид мате- рии, в котором диалектически сочетаются про- тиворечивые свойства — прерывности и непре- рывности; само поле во всех своих проявлениях выступает как единство этих противополож- ностей. Установлено, что энергия фотона электромагнит- ного поля пропорциональна -частоте электро- магнитных колебаний, свойственных данному потоку фотонов: = Av, (468) где —энергия фотонов в эргах; v—(читать «ни»)—частота электромагнитных колеба- ний в герцах; h= 6,624 • 10-27 эрг * сек — коэффициент пропорциональ- ности, называемый коэффициентом Планка. Пример 191. Частота электромагнитных колебаний рентгеновых лучей равна у = 25*1018 гц. Требуется определить энергию фотонов этих лучей. Решение. Применяя формулу (468), находим У7ф = Ь = 6,624-10~27-25-1018 = 1,6-10~7 эрг. Чем больше энергии несет данный фотон, тем больше величина его электромагнитной массы: /Пф = ^, (469) где П^ф—энергия фотона в эргах; с—скорость света; Шф—масса фотона в граммах. Пример 192. Требуется определить массу фотона рентгеновых лучей, если энергия его равна №ф = 1,6 • 10—7 эрг. Решение. Применяя формулу (469), находим iR.io^e г. * с2 (3-1010)2 ’ 556
Таким образом, масса фотона исчезающе мала. В соответ- ствии с этим и масса электромагнитных полей также относи- тельно мала по своей величине. Зная мощность излучения электромагнитного поля осцилля- тором (излучателем) и частоту электромагнитных колебаний поля, можно определить общее количество фотонов, излучаемых осциллятором за единицу времени, и количество фотонов, прохо- дящих за единицу времени через какую-либо поверхность. Пример 193. Точечный осциллятор излучает электромагнитное поле, раз- вивая мощность Р = 100 вт при частоте колебаний v = 106 гц. Требуется определить, какое количество фотонов ежесекундно излучает осциллятор, ка- кова их масса и какое количество фотонов проходит ежесекундно через 1 кв. см площадки, расположенной перпендикулярно к направлению распро- странения электромагнитного поля и удаленной от осциллятора на расстоя- нии г = 5 км. Решение. Энергия каждого фотона данного электромагнитного поля равна 1Гф = Ь = 6,624-10”27- 106 « 6,6-10~21 эрг. Электромагнитное поле ежесекундно уносит с собой энергию W = Pt = 100-1 = 100 дж = 109 эрг. Масса электромагнитного поля, излучаемого в течение одной секунды, равна W 109 tn = _ = ——------~ 1 1. ю-12 г с2 (3-1019)2 ~ 1,1 Следовательно, если осциллятор будет работать непрерывно с неизмен- ной мощностью в течение одного года, то электромагнитное поле, отпочко- вавшееся от него, унесет с собой массу, равную примерно нескольким сто- тысячным долям грамма. Число W фотонов, излучаемых осциллятором за одну секунду, равно W 109 " = 4^ = б^ГПР51 ~ 1,5 ’1029 Ф°тонов- Поверхность шара радиусом г = 5 ог=5«105 см, в центре которого расположен осциллятор, равна S = 4лг2 = 4-3,14 (5-Ю6)2 = 3,14-1012 о. см. Число фотонов, проходящих ежесекундно через 1 кв. см шаровой по- верхности, равно „ = 4 4.8.10Ю 5 3,14-1012 кв. см т. е. за одну секунду через площадку в 1 кв. см, удаленную на 5 км от осциллятора, проходит 4,8 миллиарда миллиардов фотонов. Из рассмотренного примера видно, что в каждом кубическом сантиметре электромагнитного поля содержится громаднейшее количество фотонов, примерно равное числу молекул газа в 1 куб. см при атмосферном давлении. Естественно, что при 557
таких условиях весьма трудно заметить корпускулярность элек- тромагнитного поля и оно воспринимается как нечто непрерыв- ное, сплошное, распространяющееся в виде электромагнитных волн. Волновые свойства электромагнитного поля обусловлены вол- новыми свойствами его фотонов. Поток фотонов, перемещаю- щихся в пространстве со скоростью света, проявляется в форме волнообразного движения материи (электромагнитных волн). Исследование электромагнитных полей показывает, что, будем ли мы рассматривать электромагнитное поле как поток движущихся дискретных частиц (фотонов) или как некоторое сплошное, непрерывное волнообразное движение его, всякий раз движение его подчиняется одним и тем же законам. Например, явления интерференции и дифракции наблюдаются как в том, так и в другом случае. Расчет энергии электромагнитного поля при- водит к совершенно одинаковым результатам, будем ли мы исхо- дить из энергии каждого фотона этого поля или объемной плот- ности энергии его как материального образования, распростра- няющегося в пространстве в форме электромагнитных волн. Если энергия фотонов электромагнитного поля относительно велика, например энергия фотонов гамма-лучей или рентгеновых лучей, то длины электромагнитных волн этих лучей весьма малы (миллиардные доли сантиметра). В этом случае электромагнит- ное поле проявляет главным образом свои корпускулярные свой- ства. Фотоны его при встрече с элементарными вещественными частицами, например электронами, ядрами атомов, бомбарди- руют их и отдают им частично или полностью свою массу и энергию. Если, например, фотон гамма-лучей до встречи с элек- троном обладал энергией ГГф = Ау, то после столкновения с ним его энергия и масса уменьшатся. Согласно закону сохранения энергии и массы для данного случая и7ф = Ь = Лу' + -^- , (470) где Av—энергия фотона до столкновения с электроном; Av'—энергия фотона после столкновения с электроном; mv2 —2----кинетическая энергия электрона,- полученная им в ре- зультате столкновения с фотоном. Если масса фотона до столкновения с электроном была равна ftv = —, то после столкновения с электроном она стала рав- ной . Убыль массы фотона связана с переходом ее в электромагнитную массу электрона. Частота колебаний рентгеновых лучей в результате столкно- вений фотонов с электронами снижается в соответствии с умень- шением их энергии, т. е. качество лучей становится несколько 558
иным. Скорость же движения фотонов после столкновения их с электронами остается по-прежнему равной скорости света. Если фотоны электромагнитного поля обладают относительно малой энергией, например фотоны электромагнитных полей, из- лучаемых антеннами радиостанций, то длины волн этих полей относительно велики (единицы, сотни и даже тысячи метров). Электромагнитные поля в этом случае проявляют главным обра- зом свои волновые свойства. Зависимость длины волны электромагнитного поля от коли- чества энергии каждого из его фотонов определяется формулой he где X — длина электромагнитных волн в сантиметрах; 1^ф — энергия фотонов в эргах; h — 6,624 • 10-27 эрг • ерк — постоянная Планка; с—З’Ю10-^—скорость света. Пример 194. Определить длину волн электромагнитного поля, если каж- дый фотон его несет энергию, равную М7ф = 6,6 • 10~20 эрг. Решение. Применяя формулу (471), находим he 6,624-10-27-3-1010 6,6-10-20 « 3000 см = 30 м. § 159. СВОБОДНОЕ ЭЛЕКТРОМАГНИТНОЕ ПОЛЕ Подобно веществу, которое может быть качественно различ- ным, существуют качественно различные электромагнитные поля. Электромагнитное поле называется стационарным, если оно сопровождается постоянным током в электрической цепи. Характерная особенность этого поля та, что оно неразрывно свя- зано со своими вещественными источниками — движущимися за- рядами. Электромагнитное поле называется к в а з и ста ци о н а р- ным (т. е. как бы стационарным), если оно сопровождается то- ками низкой частоты. Оно, как и стационарное поле, неразрывно связано со своими вещественными источниками — движущимися электрическими зарядами. Масса и энергия стационарного и квазистационарного элек- тромагнитных полей возникают за счет массы и энергии вещест- венных источников этих полей. Если поле исчезает, то обычно оно отдает целиком всю массу и энергию вещественным части- цам, являющимся носителями той или иной энергии,* например тепловой и т. д. В отличие от квазистационарного электромагнитного поля, создаваемого токами низкой частоты, свободное элек- тромагнитное поле создается быстро колеблющимися 559
источниками поля, например токами высокой частоты в антенне радиопередающей станции. В этом случае электромагнитное поле благодаря высокой частоте своих колебаний и своей инертности, обусловленной его массой, не успевает целиком возвращать массу и соответственно энергию вещественным источникам и ча- стично отпочковывается от них. Эта часть электромагнитного поля, отделившаяся от своих вещественных источников (быстро движущихся зарядов в антенне), становится независимой от них и распространяется самостоятельно в пространстве (рис. 366) в виде совокупности электромагнитных волн. Электромагнитные волны, как и все иные физические волны, обусловлены некоторым колебательным процессом, в данном случае электромагнитными колебаниями. Рис. 366. Электромагнитное поле „отпочковы- вается* от антенны и становится свободным Электромагнитные колебания — это периодические измене- ния во времени и пространстве электрического и магнитного полей. Установлено, что всякое свободное электромаг- нитное поле состоит из совокупности взаимо- обусловленных, неразрывно связанных друг с другом электрического и магнитного полей как некоторых материальных образований. Если электрическое поле как одна из составляющих электромагнитного поля исчезает, то оно создает магнитное поле, являющееся дру- гой составляющей электромагнитного поля, и при этом передает ему свою массу и энергию. Наоборот, когда исчезает магнитное поле, оно создает электрическое поле, передавая ему свою массу и энергию. Этот процесс взаимного превращения электрического и магнитного полей друг в друга в свободной электромагнитной 560
волне протекает непрерывно с частотой, равной частоте электро- магнитных колебаний. Следовательно, если в данной области пространства имеется переменное электрическое поле, то оно создает в этой области и соседних с нею переменное магнитное поле, которое в свою оче- редь создает переменное электрическое поле, а это вызывает маг- нитное поле и т. д. Непрерывный процесс обмена массой и энер- гией электрического и магнитного полей с каждым новым коле- банием захватывает все новые области пространства, благодаря чему электромагнитное поле распространяется в форме электро- магнитных волн с некоторой конечной скоростью. Движущей силой развития свободного элек- тромагнитного поля является взаимодействие его двух противоречивых, взаимообусловлен- ных и органически связанных сторон его — элек- трического и магнитного полей. Электромагнитные волны характеризуются периодичностью во времени и пространстве. Рис. 368. Напряженность электриче- ского поля изменяется в направлении оси OZ по синусоидальному закону Рис. 367. Напряженность элек- трического поля изменяется во времени по синусоидальному закону Периодичность во времени означает, что напря- женности электрического и магнитного полей как составляющих электромагнитного поля изменяются во времени по периодиче- скому закону, например по синусоидальному. На рис. 367 пока- зана развернутая диаграмма напряженности электрического поля, изменяющейся во времени по закону синуса. Периодичность в пространстве означает, что в направлении своего распространения электромагнитная волна изменяется в зависимости от расстояния по периодическому за- кону, например по синусоидальному. На рис. 368 показан гра- фик изменения напряженности электрического поля в направле- нии оси OZ по синусоидальному закону. Число электромагнитных колебаний в одну секунду назы- вается частотой электромагнитных колебаний. 36—1377 561
Путь, проходимый электромагнитной волной за один период колебаний, называется длиной волны. На рис. 369 показана длина волны X; она равна расстоянию между двумя одинако- выми фазами колебания, например, между двумя соседними по- ложительными амплитудами напряженности электрического поля. Скорость распространения электромагнитных волн в вакууме равна с = 3’108-^-, а во всех прочих средах она несколько ниже и находится в зависимости от диэлектрической и магнит- f ной проницаемости среды. лтч Если известна скорость / 1 \ / ! \ распространения электромаг- / | \ / ‘ \ нитных волн и период элек- 0 е V ~г | ^—**2 тромагнитных колебаний, то । \ / j длину электромагнитной вол- । 1 ны можно определить по L------л --------J формуле Рис. 369. Длина электромагнитной волны = (472) где X —длина волны в метрах; v — скорость распространения электромагнитных волн в метрах в секунду; Т — период в секундах. Пример 195. Определить длину электромагнитных волн, распространяю- щихся в вакууме (практически в воздухе), если частота электромагнитных колебаний v = 106 гц. Решение. Период электромагнитных колебаний т- v = тк “10-6 сек- Длина электромагнитной волны X = сТ = 3 • 108•10—6 в 300 м. В природе существует весьма большое разнообразие свобод- ных электромагнитных полей, различающихся по частоте своих электромагнитных колебаний, или, иначе, по длине электромаг- нитных волн. Качество электромагнитных полей зависит от их количествен- ной характеристики — частоты электромагнитных колебаний. Надо заметить, что закон перехода количества в качество здесь находит свое яркое отображение. В табл. 37 приведен спектр электромагнитных волн с указа- нием их основных качественных характеристик. 562
Спектр электромагнитных волн Таблица 37 Наименование волн Частота колебаний, ги, Длина волны, м Основные свойства Длинные ВОЛНЫ 1,0—10* 3-109—3-104 Распространяются вдоль проводов элек- трических цепей Радиоволны 104—ЗЮ1» ЗЮ4—10“2 Излучаются антен- нами радиопередаю- щих устройств Инфракрасные лучи 3.1010—4 • 1014 10—2—7,5 • 10~7 Обладают тепло- вым действием Световые лучи 4Ю14—7,5-1014 7,5-10—7—4 -IO-7 Оказывают дей- ствие на зрительные органы Ультрафиоле- товые лучи 7,5-1014—7,5-1016 4-Ю-7—4-10—9 Обладают химиче- ским действием Рентгеновские лучи 7,5-Ю1»—2Ю1а 4-10-9—1,5.10-и Обладают способ- ностью проникать че- рез физические тела, не пропускающие световых лучей Гамма-лучи ‘З-Ю19—1021 1,5-10-“—3-10-» Элементы этого по- ля могут „рождать* пары — электрон и позитрон и возникать в результате взаим- ного погашения этих пар. Обладают боль- шой способностью проникать в физиче- ские тела § 160. ЭНЕРГИЯ ЭЛЕКТРОМАГНИТНОГО ПОЛЯ Всякое свободное электромагнитное поле как некоторый вид материи обладает массой и энергией. Электромагнитная энергия есть мера электромагнитной формы движения материи — распространения электромагнитного поля! Следовательно, зная закономерность изменения энергии и’ соответственно массы электромагнитного поля, можно судить о том, как протекает то или иное электромагнитное явление, как оно видоизменяется и переходит в иные формы движения ма- терии. 36* 563
£ Рис. 370. Взаимное расположение векторов Е, Н и 5* в некоторой точ- ке О электромагнитного поля Если известен закон изменения в пространстве и времени напряженностей электрического и магнитного полей как состав- ляющих электромагнитного поля, то это значит, что известно само электромагнитное поле, направление передачи электромаг- нитной энергии в пространстве и т. д. Важной величиной, харак- теризующей интенсивность дви- жения электромагнитной энер- гии в пространстве, является мощность потока этой энергии, отнесенная к единице поверх- ности, расположенной перпен- дикулярно направлению элек- тромагнитной энергии. Мощность потока электромагнитной энер- гии, отнесенная к еди- нице поверхности, чис- ленно равна тому количеству энергии, которое переносится в единицу времени черезединицу поверхности, расположенную перпендикуляр- но вектору скорости распространения энергии. Мощность потока электромагнитной энергии, отнесенная к единице поверхности, — величина векторная, так как она характеризуется не только численным значением, но и направ- лением в пространстве. Мысль о потоке переносимой в пространстве энергии и о мощности потока этой энергии, отнесенной к единице поверх- ности, впервые высказал в 1874 г. известный русский физик Н. А. Умов (1846—1915). Английский ученый Пойнтинг спустя одиннадцать лет применил теорему Умова о мощности потока движущейся в пространстве энергии к электромагнитным явле- ниям. Поэтому вектор, характеризующий мощность потока пере- носимой электромагнитной энергии, отнесенной к единице по- верхности, называется вектором Умов а—П о й н ти н г a (3). Если в данной точке электромагнитного поля известны направ- ления векторов напряженности электрического и магнитного полей Е и Я, то направление вектора Умова—Прйнтинга в этой точке поля можно определить по правилу правого винта. Если поворачивать вектор напряженности электрического поля Е до совпадения с вектором напряженности магнитного поля Н по кратчайшему пути согласно повороту винта правой системы, то поступательное движение винта покажет направле- ние вектора Умова—Пойнтинга 3* в данной точке, т. е. направ- ление (луч), по которому через данную точку в данный момент времени передается электромагнитная энергия (рис. 370). 564
Численное значение вектора Умова—Пойнтинга можно опре- делить по формуле S = ЕН-sin (ЕН), (473) где 5—численное значение вектора Умова—Пойнтинга, выра- женное в ваттах на квадратный метр; Е — напряженность электрического поля в вольтах на метр; Н — напряженность магнитного поля в амперах на метр; (Е Н) — угол, составленный векторами Е и Н в данной точке поля. Пример 196. Электрическая лампа излучает в окружающее пространство электромагнитное поле, несущее с собой поток электромагнитной энергии (лу- чистой энергии). Требуется определить, какова мощность электромагнитного излучения этой лампы, если известно, что во всех точках, расположенных на расстоянии г = 1 м от лампочки, действующее значение напряженности элек- трического поля Е = 34,64 , а напряженность магнитного поля Н = = 0,092—. Направления векторов £ и Я в этих точках взаимно перпен- дикулярны. Решение. Численное значение вектора Умова — Пойнтинга в любой точке шаровой поверхности радиусом г = 1 м, равно 5 = EH sin (Ей) = 34,64-0,092 = 3,19-^- . Мощность потока электромагнитной энергии, приходящаяся на шаровую поверхность радиусом г = 1 м, равна Р = Ss = 3,19-4к.I2 = 3,19-12,56 « 40 вт, т. е. мощность электромагнитного излучения лампы равна 40 вт. § 161. ПЛОСКИЕ ЭЛЕКТРОМАГНИТНЫЕ ВОЛНЫ В ОДНОРОДНОМ ДИЭЛЕКТРИКЕ Простейший вид электромагнитных волн — плоские волны в однородном диэлектрике. Электромагнитная волна называется пло- ской, если в любой момент времени во всех точ- ках любой плоскости, расположенной перпен- дикулярно к направлению распространения волны, векторы напряженности электриче- ского Е и магнитного Н полей имеют одинако- вые значения. Следовательно, эти векторы в плоской волне изменяются по величине только в направлении распространения волн и во времени. 565
Пример плоской волны — электромагнитная волна, излучае- мая антенной передающей радиостанции; если эту волну рас- с w сматривать на относитель- • но большом расстоянии 1 от антенны — в несколько километров и более. Допустим, что плоская Z' >\ электромагнитная волна 0 Г_______\t распространяется в одно- п 77 родном диэлектрике, на- /1 пример в воздухе, в напра- \ / влении некоторой оси OZ \ / и при этом электромаг- нитные колебания в ней _ _ совершаются по закону Рис. 371. Векторы Е и Н изменяются синуса. Это значит, что по закону синуса напряженности электриче- ского Е и магнитного Н полей изменяются по закону синуса и совпадают по фазе. Следо- вательно, если напряженность электрического поля Е изменяется по закону Е = Ет* sin ш/, то в соответствии с этим напряженность магнитного поля будет изменяться по закону Н = f/m-sin atf, т. е. напряженности электрического Е и магнитного Н полей в плоской волне в диэлектрике изменяются синфазно во времени. На рис. 371 приведены развернутые диаграммы напряженно- стей электрического Е и магнитного Н полей для рассматривае- мого случая. Так как в рассматриваемом нами диэлектрике нет потерь электромагнитной энергии на необратимые процессы, то ампли- туды напряженностей электрического Ет и магнитного Нт полей неизменны во времени и пространстве. Ет = const и Нт = const. Отношение амплитуд электрического Ет и магнитного Нт полей в плоской волне — вели- чина постоянная и определяется равенством -fc = l/V = const' ,47,) 1 ф где е0 = '47{.9~4()9 — диэлектрическая проницаемость вакуума (практически воздуха); 566
4п гн ( р0 =-уог— —магнитная проницаемость вакуума (прак- тически воздуха). Следовательно, для воздуха Нт 4тс ю7—!— 4л-9-109 = 377 ом. (475) Векторы напряженности электрического Е и магнитного Н полей в любой точке поля плоской волны взаимно перпендику- лярны и расположены в плоскостях, перпендикулярных направ- лению распространения электромагнитных волн. При этом век- торы Е, Н и направление распространения волны (ось OZ) образуют правовинтовую систему, т. е. если вектор Е вращать до совпадения с вектором Н по кратчайшему пути согласно пово- роту винта правовинтовой системы, то поступательное движение винта будет совпадать с направлением распространения электро- магнитной волны. Скорость распространения электромагнитной волны в вакууме (практически в воздухе) можно определить по формуле (476) или V — —, . У 4л.9.109 * То7" = З.Ю8—. Длина плоской волны может быть определена по формуле Х = ^Г, (477) где X—длина волны в метрах; 7' = -^-—период в электромагнитных колебаниях в секундах. Поскольку электромагнитное поле не только изменяется во времени по синусоидальному закону, но и перемещается в про- странстве вдоль оси OZ с постоянной скоростью у = 3 • 108 напряженности электрического Е и магнитного Н полей изме- няются в направлениях, параллельных оси OZ, по синусоидаль- ному закону. На рис. 372 показаны графики (синусоиды) зависимости ве- личин Е и Н от расстояния z, расположенные во взаимно пер- пендикулярных плоскостях. Так как напряженности электрического Е и магнитного Н полей изменяются по закону синуса в зависимости не только от 567
времени, но и расстояния по оси OZ, то мгновенное значение напряженности электрического поля выражается следующей формулой: Е = Ет-sin (о>/ — kz), (478) где z — расстояние от начала OZ до точки наблюдения; k — ш |/ so р-о — так назы^ймое волновое число, или, иначе, фазовый коэффициент, выраженный в радианах на метр. Рис. 372. Графики зависимости величин Е и Н от рас- стояния Мгновенное значение напряженности магнитного поля Н = ]/-^Е = /-^Em-sin(<o/-fe). (479) Мгновенное значение вектора Умова—Пойнтинга S = ЕEI — Е„-sin (<ot — kz) l/ — Ет- sin (wt — kz), ИЛИ S = E^ • sin2 (<o/ — kz). (480) Среднее значение вектора Умова—Пойнтинга за период (481) где Ед — действующее значение синусоидальной напряженности Е электрического поля. §68
Плотности энергии электрического и магнитного полей соот- ветственно равны ^э.о = ^и 1ГМ.О = -^, ’ (482) где 1^Э1О—объемная плотность энергии электрического поля в джоулях на кубический метр; Е—напряженность электрического поля в вольтах на метр; £ =______ 0 4к 109 м 1 i U^M,o—Объемная плотность энергии магнитного поля в джоулях йа кубический метр; /7—напряженность магнитного поля в амперах на метр; __ гн Го -- То?- 77 • Легко показать, что объемные плотности энергии электриче- ского и магнитного полей в рассматриваемом случае равны: Р-сД2 _ / i/‘V р \2 _ Р-о-о рг _ еоЕ2 2 — 2 [io J 2|*о 2 ’ т. е. 1F,.O = 1ГМ.О. Пример 197. Плоская электромагнитная волна, падая на плоскость физи- ческого тела, расположенную перпендикулярно к направлению распростра- нения луча, отдает ежесекундно 0,05 малых калорий тепла на каждый ква- дратный сантиметр этой плоскости. Определить напряженности электриче- ского и магнитного полей этой плоской электромагнитной волны и плот- ность энергии этих полей. Решение. Определим численное значение вектора Умова — Пойнтинга, т. е. количество джоулей, приходящихся на каждый квадратный метр поверх- ности за каждую секунду: $ = 10000 „ 9nRn дж = 9nRn вт_ Следовательно, 0,24 сек-м2 м2 * ЕН =2080 Но так как & 1/ - 2080. то Н) 569
или 2080 = 78-104 и Е = У78-104 - 886 — . м Е2 = 4к-9- 109-4к-10-’ Напряженность магнитного поля Е = 886 1 __ 2 35 — 4л-30 ’ л/ ’ Объемная плотность энергии электрического поля т еоЕ2 1 8862 1П_Л дж = — = 4я,9.109 • -г = 3-48-10 6 Объемная плотность энергии магнитного поля п/ == . ЛИ. — 3 48« 10~6 § 162. ПЛОСКИЕ ВОЛНЫ В ПРОВОДЯЩЕЙ СРЕДЕ Процесс распространения плоских электромагнитных волн в проводящей среде значительно отливается от аналогичного про- цесса в диэлектрике. Допустим, что плоская электромагнитная волна, распростра- няющаяся в однородном диэлектрике, падает на плоскую по- верхность металлической среды, расположенную перпендикуляр- но распространению данной волны. Рассмотрим, какие изменения претерпевает эта волна вследствие проникновения в данную ме- таллическую среду. Установлено, что скорость распространения элек- тромагнитныхволн в металлических средах (при всех прочих равных условиях) значительно меньше, чем в диэлектрике. При этом чем больше удельная проводи- мость 7 и магнитная проницаемость р. металлической среды и чем больше период Т электромагнитных колебаний, тем меньше скорость распространения электромагнитных волн в этой среде. Эта зависимость скорости распространения электромагнитных волн в проводящей среде от параметров ее 7 и р., а также от пе- риода Т электромагнитных колебаний выражается следующей формулой: («) где н—магнитная проницаемость среды в генри на метр; 7— удельная проводимость в — ; Т — период в секундах; v— скорость распространения электромагнитных волн в мет- рах в секунду. 570
Так как скорость распространения электромагнитных волн в металлической среде меньше, чем в диэлектрике, то и длина волны в ней будет меньше, чем в диэлектрике. Длину волны в проводящей среде определим по формуле или (484) т. е. чем меньше период Т электромагнитных колебаний и чем больше магнитная проницаемость у. и удельная проводимость 7 металлической среды, тем (при прочих равных условиях) меньше длина волны (рис. 373). Рис. 373. Плоская волна при переходе из воздуха в металли- ческую среду изменяет свою длину и затухает Пример 198. Плоская электромагнитная волна падает из воздуха на пло- скую поверхность медной пластины, расположенной перпендикулярно к на- правлению распространения волны. Определить длину волны к в меди и ско- рость v распространения ее там, если удельная проводимость меди 7 = = 5 - 107 0J^~— » магнитная проницаемость ее у = yry0 = 1 • 4я- 10“7 , а частота электромагнитных колебаний v = 5 • 10б гц. Решение. Период электромагнитных колебаний Скорость распространения электромагнитных волн 4ж.10-’-5-10’-2-10-’_ 1000 сек ‘ 571
Длина волны в меди X = vT = 1000-2-10”7 = 2-10—4 м = 0,2 мм. в то время как в воздухе скорость распространения электромагнитных волн а длина волны (при заданной частоте) ^•воза = ^возд У = 3 - 109-2-10 7 — 60 М. В металлической среде в отличие от диэлек- трика электромагнитные волны весьма быстро затухают. Это объясняется тем, что электромагнитное поле, войдя во взаимодействие с металлом, быстро расходует свою массу и энергию, преобразуя ее в массу и энергию вещественных частиц металла. В соответствии с этим глубина проникновения электромагнитных полей в металлическую среду обычно изме- ряется долями миллиметра. Установлено, что убывание амплитуд напряженностей элек- трического Ет и магнитного Нт полей в зависимости от рас- стояния г, пройденного электромагнитным полем в металличе- ской среде, происходит по экспоненциальному закону £m = 5mOe-p\ (485) где Етй— амплитуда напряженности электрического поля на плоскости раздела диэлектрика (воздуха и металла), т. е. при z = 0; 2 — расстояние, пройденное электромагнитным полем в металлической среде от некоторой начальной точки z-О, расположенной на плоскости раздела диэлек- трика и металлической среды; е=2,718—основание натуральных логарифмов; Р— коэффициент затухания электромагнитной волны, оп- ределяемой по формуле р = )/4“. (486) Из формулы (486) следует, что чем больше магнитная прони- цаемость [1, удельная проводимость 7 и угловая частота со элек- тромагнитных колебаний, тем больше коэффициент затухания р и тем, следовательно, быстрее затухает электромагнитное поле в проводящей среде. Пример 199а. Определить минимальную толщину стенки электромагнит- ного экрана, сделанного из алюминия ^7 = 3,6 ♦ 107 , если при прохо- ждении через него электромагнитных волн электромагнитное поле должно быть ослаблено в 200 раз. Угловая частота электромагнитных колебаний равна со = 1»1 • 106—. г сек 572
Решение. Если электромагнитное поле по выходе из экрана должно ослабнуть в 200 раз, то это значит, что амплитуда напряженности электри- ческого и магнитного полей должны порознь также уменьшиться в 200 раз, т. е. = 0-005. Коэффициент затухания в рассматриваемом случае равен ,/ __ ./4тг. 10—7-3,6-107-1,1 - 10в _ m3 1 У 2 “г 2 “ л/ Следовательно, е-5ооог = 0,005, или, иначе, е^ооог = 200. Логарифмируя последнее равенство, получаем 5000г 1g 2,718 = 1g 200, или 5000г-0,436 = 2,302, откуда г = ~ 0,00106 м « 1,1 мм, ZZoxj т. е. для того чтобы ослабить заданное электрическое поле в 200 раз, необ- ходимо на его пути поставить алюминиевый экран толщиной в 1,1 мм. Для того чтобы можно было судить о том, как быстро зату- хает электромагнитная волна в проводящей среде, введена вели- чина, называемая глубиной погружения d электромагнитной вол- ны. Под последней подразумевают то расстояние, на протяжении которого амплитуды напряженностей электрического Ет и маг- нитного Нт полей уменьшаются в е = 2,718 раз по сравнению с амплитудами Е^ и HmQ на границе перехода из воздуха (ди- электрика) в данную металлическую среду. В табл. 38 приведены для сравнения величины глубин погру- жения и скорость распространения электромагнитных волн в ме- ди при различных частотах электромагнитных колебаний. Таблица 38 Частота колебаний, гц Длина волны в воздухе, м Скорость распростра- нения электромагнит- ной волны в меди, Mfсек Глубина погружения электромагнитной волны в медь, м 1010 0,03 42 000 0,67-10-6 10* 3,0 4 200 6,7-Ю-б 106 300 420 67-Ю-б 50 6-10-6 3 9,5-10-3 573
В табл. 39 приведены для сравнения глубины погружения электромагнитных волн в различные металлы. Таблица 39 Длина волны в воздухе, мм Глубина погружения электромагнитной волны * в металлах, м медь | алюминий | никель 0,03 0,67-10—6 0,86-10-6 2,9-10-6 3,0 6,7-10-6 8,6-10-6 29-10-6 300 67-10-6 86-10-6 290-10-6 6-106 9,5-10-3 Г2,.1-10-з 42-Ю-з Из вышеприведенных таблиц видно, что глубина погружения электромагнитных волн в металлы очень мала. Для коротких ра- диоволн она измеряется десятитысячными долями миллиметра, для длинных волн — несколькими долями сантиметра в зависи- мости от длины волны и удельной проводимости металла. В заключение отметим, что и в полупроводящих средах так- же происходит относительно быстрое затухание электромагнит- ных волн. Например, электромагнитные волны длиной X = 300 м при проникновении из воздуха в морскую воду уменьшают ам- плитуду своих колебаний примерно в миллион раз на расстоя- нии нескольких метров от поверхности воды. § 163. СТАЦИОНАРНОЕ ЭЛЕКТРОМАГНИТНОЕ ПОЛЕ Стационарным электромагнитны м полем на- зывается поле в цепях постоянного тока. Отличительная особенность такого поля та, что оно неразрыв- но связано с электрической цепью и распространяется от генера- тора к потребителю вдоль соединительных проводов по окру- жающему эти провода диэлектрику, например по воздуху. Электромагнитная энергия, которую несет стационарное элек- тромагнитное поле от генератора к потребителю, расходуется ча- стично в соединительных проводах, а остальная, более значитель- ная часть ее отдается электромагнитным полем потребителю. Электромагнитное поле, проникнув извне в провода электри- ческой цепи, создает в них электрический ток. А так как это по- ле стационарное, то и ток в цепи стационарный, т. е. постоянный. Следовательно, постоянный ток и стационарное электромагнитное поле неразрывно связаны дру.г с другом в электрической цепи. При этом ста- ционарное электромагнитное поле преобразует свою энергию в проводниках цепи с помощью электрического тока в тепловую энергию. 574
Рассмотрим стационарное электромагнитное поле в системе линии электропередачи, по которой протекает некоторый посто- янный ток I, На рис. 374 показана такая линия в разрезе чертежа. В ле- вом проводе (прямом) ток уходит от нас за плоскость чертежа, т. е. от генератора к потребителю, что и обозначено на сечении провода крестиком. В правом проводе (обратном) ток идет к нам из-за плоскости чертежа, т. е. от потребителя к генератору, что обозначено точкой. Магнитные силовые линии показаны Рис. 374. Магнитные и электрические поля системы проводов линии пе- редачи энергии сплошными линиями. Их направление легко определить по пра- вилу буравчика. Электрические силовые линии — пунктирные. Они начинаются на положительно заряженном проводе (прямом) и оканчиваются на отрицательно заряженном (обратном). Если определить направление вектора Умова — Пойнтинга для точки М, расположенной вблизи прямого провода, и для точ- ки N, расположенной вблизи обратного провода, то нетрудно убедиться в том, что в каждой из них он направлен за плоскость чертежа, т. е. от генератора к потребителю. А это значит, что электромагнитное поле распространяется вдоль прямого и обратного проводов линии электропередачи как некоторых направляющих от ге- нератора к потребителю. Если бы провода линии были идеальными, т. е. сопротивление их было бы равно нулю, то в них не было бы потерь энергии. 575
Электромагнитное поле не проникало бы в провода, и внутри них не было бы составляющей вектора напряженности электрического поля, направленной вдоль осей этих проводов. Вектор напряжен- ности электрического поля Еп Е в этом идеальном случае 6 “71 был бы направлен нор- / мально (перпендикулярно) /р о к поверхности проводов. ---------------1 Однако каждый про- / I ) В°Д обладает тем или । иным сопротивлением, в I котором электромагнитное \ j Н______________________________________\ поле неизбежно теряет -----------------------------------------------------------------/ свою энергию вследствие _-------------------------------преобразования ее в теп- Рис. 375. Вектор Е тмеет две составляю- ловую энергию, nine, Et и Еп В этом случае вектор напряженности электриче- ского поля уже не будет перпендикулярным поверхности провода и его можно в любой точке поверхности проводов разложить на две составляющие (рис. 375): E=E„ + Et, (487) где Еп—нормальная, т. е. перпендикулярная поверхности про- _ вода, составляющая; Et—тангенциальная составляющая, направленная по поверх- ности провода параллельно его оси. В соответствии с двумя составляющими вектора напряженно- сти электрического поля мы имеем две составляющие вектора Умова — Пойнтинга. Одна из них, тангенциальная составляющая 5Z, обусловлена векторами Еп и Н и равна St—EnH-sin (ЁЛ„Н). (488) Тангенциальная составляющая 5Z вектора Умова — Пойнтин- га дает возможность определить количество электромагнитной энергии, переносимой вдоль проводов от генератора к потреби- телю. Другая составляющая вектора Умова—Пойнтинга —формаль- ная составляющая Sn—обусловлена векторами Et и Н и равна Sп = EtH-sin &Н). (489) Нормальная составляющая S„ вектора Умова — Пойнтинга позволяет определить количество электромагнитной энергии, вно- симой в провод через его поверхность из внешней среды. 576
Подсчитаем, какое количество электромагнитной энергии про- никает извне в провод на его участке длиной I метров. Для это- го определим величины Et и Н на поверхности провода. Напряженность магнитного поля на поверхности цилиндриче- ского провода, как известно, равна где Н— напряженность магнитного поля в амперах на метр; /—величина тока в проводе в амперах; R— радиус провода в метрах. Тангенциальная составляющая Et вектора напряженности электрического поля в точках на поверхности провода численно равна падению напряжения на единицу длины провода, т. е. (490) где Et— тангенциальная составляющая вектора напряженности электрического поля в вольтах на метр; U— напряжение на участке провода длиной в I метров в вольтах; I—длина провода в метрах. Векторы Et и Н на поверхности провода взаимно перпенди- кулярны, а поэтому sin (£,Я) = 1. Следовательно, нормальная составляющая вектора Умова— Пойнтинга S„ равна U 1 (вт\ п _ E.tt1 — t \л^) ' Чтобы определить, какую мощность поглощает провод на уча- стке длиной I метров, надо величину Sn помножить на боковую поверхность цилиндра (провода) s = 2тс/?/, и тогда получим Р Sns t или P = UI, <491) где Р — мощность, поглощаемая проводом на участке длиной I метров, в ваттах; U— напряжение в вольтах; /— ток в амперах. 37—1377 577
Полученная нами формула в точности совпадает с известной формулой, определяющей мощность потерь в проводе на тепло- вую энергию: p=UI = Fry где г—сопротивление провода длиной I метров в омах. Если электромагнитная энергия переносится от источника ее к потребителю вдоль проводов по окружающему пространству (по диэлектрику), то это значит, что поток электрической энер- гии вытекает из источника в окружающее пространство. Рис. 376. Электромагнитная энергия переносится в направлении линий вектора Умова — Пойнтинга На рис. 376 схематически показана картина линий S вектора Умова—Пойнтинга, т. е. условных линий, в направлении которых переносится электромагнитная энергия от химического источника электрической энергии к внешней цепи, состоящей из совокупно- сти проводников. Направление электрических линий электромаг- нитного поля здесь показано стрелками на пунктирных линиях, а направление магнитных линий электромагнитного поля внутри контура — точками, а вне контура — крестиками. В заключение следует отметить, что примитивное представле- ние о том, что электромагнитная энергия переносится электрона- 578
ми внутри объемов проводов от генератора к потребителю напо- добие того, как вода несет гидравлическую энергию по трубопро- воду, ошибочно. Носителем электромагнитной энергии, передат- чиком ее от генератора к любому потребителю является электро- магнитное поле с присущей ему как некоторому материальному объекту массой и энергией. При этом распространение электро- магнитного поля, как правило, происходит по диэлектрику, и вся- кое проникновение его в проводящие тела, например провода ли- нии передачи или проводники потребителя, связано с потерей электромагнитным полем его массы и энергии путем преобразо- вания в массу и энергию каких-либо иных материальных частиц. % § 164. ПОВЕРХНОСТНЫЙ ЭФФЕКТ Если в проводе протекает постоянный электрический ток /, то он распределяется по любому поперечному сечению S этого провода с одинаковой плотностью / х ] = у = const. Это значит, что при постоянном электрическом токе стацио- нарное электромагнитное поле проникает извне во всю толщу провода и его напряженность во всех точках поперечного сечения одинакова по величине и направлению. Это справедливо и для квазистационарных электромагнитных полей, возникающих при токах низкой частоты. Например, для переменного тока, имеющего частоту /= 50 гц, с большой степенью точности можно считать, что ток распределяется с одинаковой плотностью по всему попе- речному сечению провода. Сопротивление провода в этом случае можно рассчитывать по обычной формуле, применяемой для рас- чета сопротивлений проводов при постоянном токе: Однако при переходе от токов низкой частоты к токам высо- кой частоты, порядка многих тысяч или даже миллионов перио- дов в секунду, равномерность распределения ^плотности тока по сечению провода нарушается, и чем больше частота, тем больше неравномерность распределения плотности тока. Ток в этом слу- чае как бы оттесняется от оси провода к его периферии (поверх- ности), так что плотность тока в точках, близлежащих к оси про- вода, становится практически равной нулю, в то время как у по- верхности провода она становится максимальной. Явление оттеснения токов высокой частоты от оси провода к его периферии называется поверхностным эффектом. 37* 579
Поверхностный эффект объясняется тем, что при токах высо- кой частоты электромагнитное поле не успевает глубоко проник- нуть в толщу провода вследствие своей инерции, относительно малой скорости распространения в металлах и затухания. Его действие в данном случае проявляется только в тончайшем-слое у поверхности провода. Поэтому активное сопротивление прово- да увеличивается, так как не через все сечение провода проходит ток. Опытом установлено, что чем больше частота переменного то- ка, магнитная проницаемость, удельная проводимость и радиус провода, тем больше проявляет себя поверхностный эффект и тем большим становится активное сопротивление провода. * Отношение активного сопротивления провода при токах высо- кой частоты колебаний (при поверхностном эффекте) к сопро- тивлению его при постоянном токе определяется по формуле где г— активное сопротивление провода при поверхностном эф- фекте в омах; г0—сопротивление провода при постоянном токе (практиче- ски и при токах низкой частоты) в омах; а— радиус провода в метрах; «)—угловая частота синусоидальных колебаний в 1 удельная проводимость материала провода в ~ ; гн (л— магнитная проницаемость провода в — . Пример 1996. Требуется определить, во сколько раз увеличится активное сопротивление медного цилиндрического провода вследствие поверхностного эффекта, если радиус провода а = 0,5 см, удельная проводимость его -у = 1 гн ю 6« 107 и магнитная проницаемость р. = 1 • 4п-10~7 —. Ча- стота тока в проводе равна f = 2«106 гц. Решение. Угловая частота тока о = 2к/= 4к* 10е —. 7 сек Отношение активного сопротивления провода при частоте / = 2 • 106 гц к сопротивлению при постоянном токе определим по формуле (492) г _ аУ^Т _ 0,005 У4п• 10« ♦ 4* -10~7 •6 ЛО7" _ Го “ 2/2 ~ 2/2 — ’ т. е. активное сопротивление провода при частоте f = 2 • 106 гц увеличилось в 54 раза по сравнению с его сопротивлением при постоянном токе. 580
В радиотехнике, где приходится иметь дело с токами высокой частоты, часто вместо сплошных проводников применяются труб- чатые, покрытые по поверхности серебром. Так как при относительно высоких частотах, применяемых в технике проводной связи, центральная часть провода не исполь- зуется из-за явления поверхностного эффекта, то ее часто заме- няют более дешёвым и в то же время механически более креп- ким материалом. Так называемые биметаллические провода имеют внутренний объем из стали, а внешнюю оболочку из меди. Явление поверхностного эффекта широко применяется в элек- тротехнике для поверхностной закалки металлов. Крупнейшим специалистом в этой области был член-корреспондент Академии наук СССР В. П. Вологдин (1881 —1953).
ЧАСТЬ ТРЕТЬЯ ЭЛЕКТРИЧЕСКИЕ ИЗМЕРЕНИЯ ГЛАВА XXX ЭЛЕКТРИЧЕСКИЕ ИЗМЕРИТЕЛЬНЫЕ ПРИБОРЫ § 165. ЗНАЧЕНИЕ ЭЛЕКТРИЧЕСКИХ ИЗМЕРЕНИЙ Для контроля за правильностью работы электротехнических установок, испытания их, определения параметров электрических цепей, учета расходуемой электрической энергии и т. д. произво- дят различные электрические измерения. В технике связи, как и в технике сильных токов, электриче- ские измерения имеют важное значение. Всякую телефонно- телеграфнуюлинию нужно периодически проверять с точ- ки зрения соответствия предъявляемым к ней требованиям. На- пример, для надежной работы линии связи требуется периодиче- ски измерять сопротивления проводов и изоляции. В случае пор- чи изоляции и вследствие этого большой утечки тока в землю необходимо путем соответствующих измерений быстро отыскать место повреждения линии. Для нормальной работы аппаратуры связи, например телеграфных аппаратов, радиостанций и т. д., надо тщательно наблюдать за их режимом работы, пользуясь для этого элек- троизмерительными приборами — амперметрами, вольтметрами и т. д. При эксплуатации источников электрической энергии — генераторов, аккумуляторных батарей, гальваниче- ских элементов — необходимо также тщательно следить за пра- вильностью их работы, за соответствием их данным установкам и т. д., что достигается неослабным контролем над ними при по- мощи электроизмерительных приборов. Приборы, с помощью которых измеряются различные элек- трические величины: ток, напряжение, сопротивление, мощность и т. д., — называются электрическими измеритель- ными приборами. 682
§ 166. КЛАССИФИКАЦИЯ ЭЛЕКТРОИЗМЕРИТЕЛЬНЫХ ПРИБОРОВ ПО ПРИНЦИПУ ИХ ДЕЙСТВИЯ По принципу действия электроизмерительные приборы под- разделяются на следующие основные типы: 1. Приборы магнитоэлектрической системы, основанные на принципе взаимодействия катушки с током и внешнего магнитного поля, создаваемого постоянным маг- нитом. 2. Приборы электродинамической системы, основанные на принципе электродинамического взаимодействия двух катушек с токами, из которых одна неподвижна, а другая подвижна. 3. Приборы электромагнитной системы, в кото- рых используется принцип взаимодействия магнитного поля не- подвижной катушки с током и подвижной железной пластинки, намагниченной этим полем. 4. Тепловые измерительные приборы, исполь- зующие тепловое действие электрического тока. Нагретая током проволока удлиняется, провисает, и вследствие этого подвижная часть прибора получает возможность повернуться под действием пружины, выбирающей образовавшуюся слабину проволоки. 5. Приборы индукционной системы, основанные на принципе взаимодействия вращающегося магнитного поля с токами, индуктированными этим полем в подвижном металличе- ском цилиндре. 6. Приборы электростатической системы, ос- нованные на принципе взаимодействия подвижных и неподвиж- ных металлических пластин, заряженных разноименными элек- трическими зарядами. 7. Приборы термоэлектрической системы, представляющие собой совокупность термопары с каким-либо чувствительным прибором, например магнитоэлектрической си- стемы. Измеряемый ток, проходя через термопару, способствует возникновению термотока, воздействующего на магнитоэлектри- ческий прибор. 8. Приборы вибрационной системы, основанные на принципе механического резонанса вибрирующих тел. При за- данной частоте тока наиболее интенсивно вибрирует тот из якорь- ков электромагнита, период, собственных колебаний которого сов- падает с периодом навязанных колебаний. В табл. 40 приведены условные обозначения электроизмери- тельных приборов по принципу их действия, а в табл. 41 — некоторые условные обозначения, наносимые на шкалы при- боров. 583
Таблица 40 Условные обозначения электроизмерительных приборов по принципу их действия Система прибора Условный знак Система прибора Условный знак Магнитоэлектрическая Индукционная СЕ и CO U Электродинамическая г—Д——! Jj пдт/тплртптг1пдг* v о а и- с/Лсктрос тати ческа я Электромагнитная Термоэлектрическая п UczsU Тепловая \ 1 У Вибрационная \!/ Таблица 41 Условные обозначения, наносимые на шкалы электроизмерительных приборов Пояснение к условному обозначению Условный знак Пояснение к условному обозначению Условный знак Постоянный ТОК Частота 50 гц r\jS0 Переменный ток Изоляция прибора испытана на 2000 в ^2н8 Постоянный и пере- менный ток Вертикальная уста- новка прибора 1 Двухфазный ток Горизонтальная уста- новка прибора W Трехфазный ток , Наклонная установка прибора под углом 60° 584
§ 167. КЛАССИФИКАЦИЯ ЭЛЕКТРОИЗМЕРИТЕЛЬНЫХ ПРИБОРОВ по роду измеряемой величины Электроизмерительные приборы классифицируются и по роду измеряемой ими величины, так как приборы одного и того же принципа действия, но предназначенные для измерения разных величин могут значительно отличаться друг от друга по своей конструкции, не говоря уже о шкале прибора. В табл. 42 приведен перечень условных обозначений наибо- лее употребительных электроизмерительных приборов. Таблица 42 Условные обозначения электроизмерительных приборов по роду измеряемых ими величин Наименование электроизмеритель* ногр прибора Условный знак прибора Измеряемая величина Наименование единиц измерения Амперметр А Ток Ампер Миллиамперметр mA Ток 7 Миллиампер Гальванометр G Ток Микроампер Вольтметр V Напряжение Вольт и э. д. с. Омметр У Сопротивление Ом Меггер MQ Сопротивление Мегом Ваттметр W Электрическая Ватт, киловатт мощность Счетчик электрической энер- Kwh Электрическая Киловатт-час гии энергия Частотомер f Частота тока Герц § 168. КЛАССИФИКАЦИЯ ЭЛЕКТРОИЗМЕРИТЕЛЬНЫХ ПРИБОРОВ ПО СТЕПЕНИ ИХ ТОЧНОСТИ Абсолютной погрешностью прибора называют разность между показанием прибора и истинным значением измеряемой величи- ны. Например, абсолютная погрешность амперметра равна « = /-/.., (493) где 8— (читать «дельта») — абсолютная погрешность в ампе- рах; /—показание прибора в амперах; /э—истинное значение измеряемого тока в амперах. Если / > /9, то абсолютная погрешность прибора положи- тельна, а при /, она отрицательна. Поправкой прибора называют величину,-которую надо приба- вить к показаниям прибора, чтобы получить истинное значение измеряемой величины. Из формулы (493) следует, что /, = /-8=/ + (-8). 4 585
Следовательно, поправка прибора — это величина, численно равная абсолютной погрешности прибора, но противоположная ей по знаку. Например, если амперметр показал / = 5 а, а абсо- лютная погрешность прибора равна 3 = 0,1 а, то истинное зна- чение измеряемой величины равно 7 = 5+ (—0Л) =4,9 а. Приведенной погрешностью прибора называется отношение абсолютной погрешности к наибольшему возможному отклоне- нию показателя прибора (номинальному показанию прибора). Например, для амперметра Р= * . 1С0%=-+-1С0%, (494) *N lN где Р— приведенная погрешность в процентах; IN—номинальное показание прибора. Точность прибора характеризуется величиной его максималь- ной приведенной погрешности. Согласно ГОСТ 1845—52 приборы "по степени их точности разделяются на 7 классов: 0,1; 0,2; 0,5; 1,0; 1,5; 2,5 и 4,0. Если, например, данный прибор имеет класс точности 1,5, то это значит, что его максимальная приведенная погрешность рав- на 1,5%. Электроизмерительные приборы, имеющие классы точности 0,1 и 0,2, как наиболее точные, применяются там, где требуется весьма большая точность измерения. Если прибор имеет приведенную погрешность выше 4%, то он считается внеклассным. § 169. ЧУВСТВИТЕЛЬНОСТЬ И ПОСТОЯННАЯ ПРИБОРА Чувствительностью прибора называют отношение углового или линейного перемещения указателя прибора, приходящееся на едйницу измеряемой величины. Если чикала прибора равномерна, то чувствительность его по всей .шкале одинакова. Например, чувствительность амперметра, имеющего равномерную шкалу, определяется формулой 5 = -^, (495) где >$“—чувствительность амперметра в делениях на ампер; Д/—приращение тока в амперах или миллиамперах; Да— приращение углового перемещения показателя прибора в градусах или миллиметрах. Если шкала прибора неравномерна, то чувствительность при- бора в различных областях шкалы различна, так как одному и тому же приращению (например, тока) будут соответствовать разные приращения углового или линейного перемещения пока- зателя прибора. 586
Величина, обратная чувствительности прибора, называется постоянной прибора. Следовательно, постоянная прибора — это цена деления прибора,^-или, иначе, величина, на которую должен быть помножен отсчет по шкале в делениях, чтобы получить из- меряемую величину. Например, если постоянная прибора равна 10 (десять миллиампер на деление), то при отклонении его указателя на а =10 делений измеряемая величина тока равна 1 = ю- 10= 100 ма. § 170. МОЩНОСТЬ ПОТЕРЬ ЭНЕРГИИ В ПРИБОРАХ Электроизмерительные приборы потребляют при работе энер- гию, которая в них преобразуется обычно в тепловую энергию. Мощность потерь зависит от режима в цепи, а также от системы и конструкции прибора. Если измеряемая мощность относитель- но мала, а следовательно, относительно малы ток или напряже- ние в цепи, то мощность потерь энергии в самих приборах может заметно влиять на режим исследуемой цепи и показания прибо- ров могут иметь довольно большую погрешность. При точных измерениях в цепях, где развиваемые мощности сравнительно малы, необходимо знать мощность потерь энергии в приборах. В табл. 43 приведены средние величины мощности потерь энер- гии в различных системах электроизмерительных приборов. Таблица 43 Таблица мощности потерь энергии в электроизмерительных приборах Система прибора Вольтметры на 100 в, etn Амперметры на 5 а, вт Магнитоэлектрическая 0,1— 1,0 0,2— 0,4 Электромагнитная 2,0- 5,0 2,0— 8,0 Индукционная ' 2,0— 5,0 1,0- 4,0 Электродинамическая 3,0- 6.0 3,5—10,0 Тепловая 8,0—20,0 2,0— 3,0 § 171. ПРИБОРЫ МАГНИТОЭЛЕКТРИЧЕСКОЙ СИСТЕМЫ Электроизмерительные приборы магнито- электрической системы основаны на принципе взаимодействия подвижной катушки 1 с током и магнитного поля, создаваемого постоянным магнитом 2 (рис. 377). Известно, что плоская катушка 1 с током, поме- щенная в магнитное поле постоянного магни- та 2, стремится установиться так, чтобы ее про- 587
низы вал максимальный магнитный поток и что- бы направление ее собственного магнитного поля совпало с направлением магнитного поля постоянного магнита внутри нее. Если площадь плоской катушки равна S, число витков ее — w, магнитная индукция, создаваемая постоянным магнитом,— В, то при токе в катушке, равном /, она испытывает вращающий момент М = 'wBIS • sin а, (496) где М— вращающий момент в ньютонометрах; В—магнитная индукция в ; число витков катушки; —площадь рамки в квадратных метрах; а—угол, составленный перпендикуляром к плоскости рам- ки с направлением магнитных линий; /—ток в амперах. Обычно конструкция магнитоэлектрического прибора такова, что величины, входящие в формулу (496), постоянны, за исклю- чением тока /, который зависит от нагрузки в цепи. Полагая, что wBS • sin а = k = const, получим M = kf, (497) т. е. вращающий момент, hS 6 3 6 о------- Рис. 377. Схематическое изображе- ние магни!оэлешрическо! о прибора приложенный к катуш- ке, пропорционален ве- личине тока /. Под влия- нием этого вращающего мо- мента катушка поворачивается в магнитном поле. Этому по- вороту противодействуют спи- ральные пружины 3, сжимаю- щиеся или разжимающиеся при повороте рамки. При установ- лении равновесия сил, стремя- щихся повернуть рамку, и сил сопротивления упругих спираль- ных пружин рамка займет определенное положение в маг- нитном поле, повернувшись на некоторый угол. Угол поворота рамки пропорционален току /, протекающему по ее виткам, а поэтому прибор можно отгра- 588
дуировать так, что его показания будут соответствовать величине измеряемого тока. Вместе с катушкой поворачивается стрелка (указатель) при- бора 4, конец которой перемещается по шкале 5 прибора с на- несенными на нее делениями измеряемой величины. Так как вра- щающий момент пропорционален току / в первой степени, то можно иметь на приборе шкалу равномерную, т. е. такую, для которой каждое деление соответствует одинаковому углу поворо- та подвижной части прибора. Если изменить направление тока в катушке прибора, то из- менится направление действия сил на катушку и стрелка прибо- ра будет отклоняться в обратном направлении. Следовательно, прибор требует соблюдения полярности или же наличия двусто- ронней шкалы. Постоянный магнит 2 прибора изготовляют из высокосортной кобальтовой, хромистой стали или никелево-алюминиевого спла- ва, обладающих большой задерживающей силой. Магнитная ин- дукция, создаваемая ими в воздушном зазоре магнитной цепи прибора, достигает десятых долей вольт-секунды на квадратный метр. Полюсные надставки 6 делают из мягкой литой ста- ли, и им придают такую форму, чтобы линии магнитного поля были радиальны к поверхности цилиндра 7, помещенного между ними. Цилиндр, выполняющий роль железного сердечника ка- тушки, изготовляют из мягкой ли- той стали. Он неподвижно укреп- лен в приборе внутри катуш- ки (рамки). Воздушный зазор между полюсными надставками и цилиндром делается очень ма- лым, чтобы обеспечить постоян- ство магнитной индукции и ради- альное направление сил магнит- ного поля, пронизывающего ци- линдр. Каркас рамки изготов- ляют из алюминия. На него на-- несена обмотка, состоящая из витков тонкой медной или алю- миниевой проволоки с шелко- вой изоляцией. Вес всей подвиж- ной части магнитоэлектрического прибора не превышает нескольких граммов. Спиральные пружины 3 рамки изготовляются из бронзы. Их назначение — создавать про- Рис. 378. Конструкция основных деталей магнитоэлектрического прибора 589
тиводействующий момент и в • то же время быть токопроводя- щими частями прибора. При повороте рамки в ее каркасе индуктируются вихревые токи, которые поглощают энергию движения подвижной части прибора, благодаря чему достигается быстрое успокоение стрел- ки прибора. На рис. 378 приведена конструкция одного из типов магнито- электрических приборов. Здесь ясно видны подковообразный Рис. 379. Переносный вольтметр магнитоэлектрической системы магнит 1, полюсные надставки 2, рамка прибора 4, стальной ци- линдр 3 и другие детали. На рис. 379 показан переносный вольт- метр магнитоэлектрической системы. Магнитоэлектрические приборы пригодны для измерения только постоянных токов. Поэтому у зажимов приборов обычно имеются отметки (+) и (—); зажим со знаком (+) надо вклю- чать в сторону положительного полюса источника, а зажим со знаком (—) — в сторону отрицательного полюса. Класс точности лучших образцов приборов магнитоэлектриче- ской системы 0,1—0,2, т. е. предельно высокий. Чувствительность 590
их также велика. Например, стрелочные гальванометры имеют чувствительность порядка нескольких делений на один микро- ампер, а нестрелочные.—значительно большую. Магнитоэлектрические приборы получили широкое распро- странение в электротехнике. Их применяют в качестве ампер- метров, вольтметров и омметров. Благодаря большой точности и чувствительности магнито- электрические приборы применяются в качестве гальванометров, т. е. приборов, измеряющих токи очень малой величины и обла- дающих большой точностью. Достоинства магнитоэлектрических приборов следующие: большая точность, малое влияние внешних магнитных полей, ма- лое потребление энергии, равномерность шкалы. Недостатки магнитоэлектрических приборов: пригодность только для постоянного тока, сложность конструкции, высокая стоимость. § 172. ПРИБОРЫ ЭЛЕКТРОДИНАМИЧЕСКОЙ СИСТЕМЫ Электродинамические при б ор ы‘осно ваны на принципе взаимодействия двух катушек с то- ками (рис. 380). Одна из этих катушек Л, неподвижно закреп- ленная в приборе,< состоит из небольшого числа витков толстой проволоки, а другая, подвижная, — S, обладающая весьма ма- лым весом, состоит из большого числа витков тонкой проволоки. Если по катушкам проходят токи, то между магнитными по- токами катушек возникает сила взаимодействия, стремящаяся установить подвижную катушку так, чтобы через плоскость ее проходил максимальный магнитный поток и чтобы направления магнитных полей обеих кату- шек Внутри них совпали. Сила взаимодейст- вия катушек в резуль- тате действия магнит- ных’ потоков пропор- циональна произведе- нию токов Ли Л, проте- кающих по ним: F = kl^ (498) где F—сила взаимодействия ' катушек; k — коэффициент, завися- щий от числа витков катушек и взаимного расположения послед- них. Рис. 380. Принцип устройства при- бора электродинамической системы 591
На рис. 381 показано внутреннее устройство электродинами- ческого прибора. Здесь мы видим неподвижную катушку /, со- стоящую из двух секций, последовательно соединенных друг с другом. Секция, находящаяся ближе к нам, показана для на- глядности срезанной. Наличие двух секций у неподвижной ка- тушки дает возможность получения внутри нее равномерного магнитного поля. Внутри этой катушки помещена подвижная ка- тушка 2, жестко связанная с осью прибора 3. На верхнем конце оси 3 закреплена спиральная пружина 4 из фосфористой брон- зы, служащая для создания противодействующего момента по- вороту подвижной части прибора и для подвода тока к подвиж- ной катушке. К верхнему концу оси 3 прикреплен также указа- тель 5 прибора. К нижнему концу оси 3 прикреплено крыло 6 Рис. 382. Ваттметр электро- динамической системы 381. Внутреннее устройство Рис. электродинамического прибора воздушного успокоителя, которое при повороте оси перемещает- ся в воздушной камере 7 (показана открытой). Движению кры- ла 6 оказывает сопротивление воздух в камере 7, и этим осуще- ствляется торможение движения подвижной части прибора. На рис. 382 показан внешний вид одного из типов электро- динамических ваттметров. Этот ваттметр относится к приборам без стали. На практике существуют и электродинамические при- боры со сталью, называемые ферродинамическими при- борами. В них для усиления магнитного потока внутри кату- шек применены магнитопроводы из мягкой стали или из специ- альных сплавов. Приборы со сталью имеют то преимущество, что они имеют значительно больший вращающий момент, мало зави- симы от посторонних магнитных полей. Однако они менее точны по сравнению с приборами без стали. §92
Рис. 383. Схема включения в цепь электродинами- ческого амперметра с последовательно соединен- ными катушками Катушки в электродинамических приборах в зависимости от их назначения и требований, предъявляемых к ним, соединяются последовательно или параллельно. В вольтметрах электродинамической системы катушки, как правило, соединены последовательно. В электродинамических амперметрах применяется как последовательное, так и парал- лельное соединение катушек: если амперметр предназначен для измерения величин тока, не превышающих 0,5 а, то его катушки соединяются последовательно (рис. 383); если же амперметр предназначен для измерения величин тока выше 0,5 а, то его катушки соединяются параллельно. Если в электродинамическом приборе обе катушки соединены последовательно, то по ним проходит один и тот же измеряемый прибором ток (рис. 384). Следовательно, в этом случае сила вза- имодействия катушек пропорциональна квадрату величины тока, проходящего в катушках: F = kP. (499) Рис. 384. Схема включения в цепь электродинамиче- ского ваттметра 38-1377 593
Это значит, что угол отклонения подвижной катуш- ки пропорционален квадрату величины тока, т. е. шкала электродинамического амперметра должна быть не- равномерной (квадратичной). Если электродинамический прибор используется как ватт- метр, то неподвижная катушка прибора подключается последо- вательно с потребителем электрической энергии.и через нее про- ходит весь ток I потребителя. Подвижная катушка ваттметра присоединяется параллельно цепи так, чтобы на ее концах было приложено напряжение, рав- ное напряжению на зажимах потребителя. В подвижной катушке протекает ток Z2, пропорциональный приложенному напряжению (рис. 383). Сила взаимодействия катушек прямо пропорциональна токам /1 и/2в них, т. е. F = klj2. А так как где R2 — сопротивление подвижной катушки, то F = кЦ ~ = к1С7/1 = kj>, (500) где Р—мощность, потребляемая в цепи; — коэффициент пропорциональности. Из формулы (500) следует, что угол поворота под- вижной катушки пропорционален мощности Р, расходуемой в цепи; следовательно, шкала электродинамического ваттметра должна быть равномерной. Если одновременно изменить направление токов в неподвиж- ной и подвижной катушках, то направление силы взаимодейст- вия катушек сохранится неизменным. А это значит, что при пи- тании катушек переменным током подвижная катушка все время стремится повернуться в одном и том же направлении. Следова- тельно, электродинамический прибор пригоден как для постоян- ного, так и для переменного тока. К достоинствам электродинамических приборов следует отне- сти их пригодность к измерению постоянного и переменного тока (напряжения), возможность измерять ими мощность, высокий класс точности — 0,2 (для приборов без стали). Недостатки электродинамических приборов (без стали) — за- висимость показаний от внешних магнитных полей, даже такого слабого, как поле земного шара, чувствительность к перегруз- кам, сложность конструкции и увеличение погрешности прибора с ростом частоты, 594
§ 173. ПРИБОРЫ ЭЛЕКТРОМАГНИТНОЙ СИСТЕМЫ В электромагнитных приборах используется взаимодействие катушки с током и намагниченным им стальным сердечником, имеющим форму легкой пластинки (лепестка). На рис. 385 приведена схема устройства одного из наиболее употребительных электромагнитных приборов. Здесь показана не- подвижная катушка 1, концы обмотки которой присоединены к зажимам 2—2 прибора. В катушке имеется узкая щель 3, в кото- рую входит стальная пластинка 4 при втягивании ее внутрь маг- нитного поля катушки. Эта пластинка посажена эксцентрично на подвижную ось 5 прибора и жестко скреплена с ней. При пово- роте пластинки ось также поворачивается, а вместе с ней пово- рачивается и стрелка 6 прибора. Спиральная пружина 7, закреп- ленная одним концом на оси 5, а другим — на корпусе прибора, имеет назначением создавать противодействующий момент в при- боре. Воздушный успокоитель состоит из пустотелой коробки 8. Внутри коробки перемещается воздушный поршенек 9, приводи- мый в движение осью 5, с ко- торой он жёстко скреплен. На рис. 386 показан внеш- ний вид одного из типов элек- тромагнитных амперметров. IIIIIIIIIIIHIIIIn Рис. 386. Амперметр электромагнит- ной системы Рис. 385. Схема устройства электро- магнитного прибора 38* 595
Сила, с которой железный сердечник (пла- стинка) втягивается внутрь катушки, пропор- циональна току/ и величине магнитной индук- ци и В п о л я к а ту ш к и: F = kBI, (501) где k — коэффициент пропорциональности, Степень намагничения железной пластинки А прибора прак- тически пропорциональна величине тока. Следовательно, сила, действующая на пластинку А в магнитном поле катушки и стре- мящаяся втянуть ее внутрь катушки, примерно пропорциональна квадрату тока: F = kP. Это значит, что угол поворота стрелки (подвижной части) при- бора при относительно малых токах практически пропорционален квадрату тока: а = С72, (502) и в соответствии с этим шкала прибора при малых величинах измеряемого тока обычно приближается к квадратичной. Электромагнитные приборы устаревших типов имеют обычно низкий класс точности (1,5—2,5) из-за несовершенства конструк- ции и невысокого качества их ферромагнитных материалов. В настоящее время советская электропромышленность выпускает электромагнитные приборы класса точности 0,5, применяя в них высококачественные железо-никелевые сплавы, типа пермаллоя. Для устранения влияния внешних магнитных полей на показа- ния электромагнитных приборов последние делаются астатиче- скими. Астатический прибор электромагнитной системы состоит из двух катушек й двух сердечников, сидящих на общей оси. Под влиянием магнитного поля катушек каждый сердечник втяги- вается в свою катушку, при этом они поворачивают ось со стрел- кой согласованно, т. е. в одну и ту же сторону. Внешние магнит- ные поля воздействуют на сердечники таким образом, что один из них стремится повернуться по часовой стрелке, а другой — против часовой стрелки. Благодаря этому результирующий вра- щающий момент подвижной системы прибора равен нулю и при- бор не реагирует на внешнее магнитное поле. Достоинства электромагнитных приборов следующие: пригод- ность для постоянного и переменного тока, возможность измере- ния относительно больших токов без шунтирования или транс- формации тока, надежность в эксплуатации, простота конструк- ции. Недостатки электромагнитных приборов: неравномерность шкалы, зависимость показаний от влияния внешних магнитных полей. 596
§ 174. ПРИБОРЫ ЭЛЕКТРОСТАТИЧЕСКОЙ СИСТЕМЫ В электростатических приборах используется взаимодействие проводников, заряженных разноименным электричеством. На рис. 387 показано устройство одного из применяющихся в современной практике электростатических вольтметров. Две пло- ские, изолированные друг от друга неподвижные металлические пластины 1 и 2 расположены параллельно друг другу. Через зажимы 7 и 8 они присоединяются к точкам цепи, между кото- рыми требуется измерить напряжение. Между этими пластинами Рис. 387. Устройство электростатиче- ского вольтметра Рис. 388. Схематическое изо- бражение электростатического измерительного прибора свободно подвешена пластина 3, электрически соединенная с пла- стиной 1. К нижней части пластины 3 присоединена тяга 4, ко- торая другим своим концом механически соединена с тонкой ме- таллической нитью 5. Если данный прибор подключить под на- пряжение электрической цепи, то пластины 1 и 3 зарядятся одноименным электрическим зарядом, а пластина 2 — зарядом противоположного знака. Это приводит к тому, что пластина 3 отталкивается от пластины 1 и притягивается к пластине 2. Чем больше напряжение на зажимах 7 и 8 прибора, тем больше величины электрических зарядов на пластинах /, 2 и 3, тем с большей силой отталкивается пластина 3 от пластины 1 и при- тягивается к пластине 2. Противодействующий момент в при- боре создается силой тяжести противовеса 6. На рис. 388 схематически изображено устройство электроста- тического прибора, в котором подвижные пластины поворачи- ваются внутри камер, образованных системой параллельно распо- ложенных неподвижных пластин. Подвижные и неподвижные пла- стины здесь также взаимодействуют благодаря наличию на них 597
разноименных электрических зарядов. Сила взаимодействия про- порциональна квадрату напряжения, приложенного к зажимам прибора: F = kU\ (503) где k — коэффициент пропорциональности, зависящий от кон- струкции прибора. Чувствительность прибора увеличивается с числом камер, с увеличением рабочих поверхностей подвижных секторов и не- подвижных пластин и с уменьшением расстояния между ними. Если подвижным секторам придать соответствующую форму, то в электростатическом приборе можно получить почти равно- мерную шкалу. Электростатические приборы можно использовать только как вольтметры. Достоинства электростатических приборов следующие: отсут- ствие потребления приборами электрической энергии, независи- мость показаний приборов от магнитных полей, от частоты изме- ряемого напряжения и от температуры окружающей среды. Недостатки электростатических приборов: зависимость пока- заний приборов от внешних электрических полей, неравномер- ность шкалы, влияние влажности воздуха на показания прибора. § 175. ПРИБОРЫ ТЕРМОЭЛЕКТРИЧЕСКОЙ СИСТЕМЫ В приборах термоэлектрической системы используется термо- электрическое явление. Если спаять концы, например, медной и висмутовой проволок и нагреть место спая, оставив другие концы их холодными, то свободные электроны начнут переходить из медной проволоки в висмутовую. В результате этого медная проволока зарядится по- ложительным электричеством, а висмутовая — отрицательным. Переход свободных электронов из медной проволоки в висму- товую будет продолжаться до тех пор, пока не уравновесятся две противоречивые силы. С одной стороны, это силы стороннего электрического поля, перемещающие свободные электроны из медной проволоки в висмутовую. С другой стороны, это силы электростатического поля системы разобщенных разноименных электрических зарядов,' размещенных на холодных концах мед- ной (положительный заряд) и висмутовой (отрицательный за- ряд) проволок. В момент наступления равновесия этих сил тер- моэлектродвижущая сила Е в системе медь — висмут будет чис- ленно равна разности потенциалов на холодных концах медной и висмутовой проволок, т. е. \E\ = \U\, или E = -U, где Е — термоэлектродвижущая сила; U — разность потенциалов на холодных концах медной и висмутовой проволок. 598
Рис. 389. Схема термоэлектриче- ского прибора Система, состоящая из спаянных между собой разнородных металлов и создающая термоэлектродвижущую силу, называется термопарой. В современной электротехнике наиболее употребительны тер- мопары: медь — константан, дающая термоэлектродвижущую си- лу, равную примерно 40 мкв на 1°Ц нагретого спая; железо — константан — 50 мкв на Г Ц; манганин — константан — 50 мкв на 1°Ц. Термоэлектродвижущая сила, развиваемая в термопаре, про- порциональна разности темпера- тур нагретого и холодного спаев металлов. Если холодные концы термо- пары, например медь — висмут, соединить, то в этой замкнутой цепи возникнет _ электрический ток, который будет протекать через нагретый спай от висмута к меди. Электрический ток, возникший в цепи под действием сто- ронних термоэлектрических сил, называется термоэлектри- ческимтоком. Термоэлементы нашли широкое применение в современной электротехнике, и в частности в термоэлектрических измеритель- ных приборах. Термоэлектрический прибор состоит из магнитоэлектрического прибора и термопреобразователя. В свою очередь термопреобра- зователь состоит из термопары или термобатареи и электриче- ского нагревателя. На рис. 389 приведена принципиальная схема термоэлектри- ческого измерительного прибора с контактным термопреобразо- вателем. Нагреватель термопреобразователя представляет собой токопроводящую проволочку 1—2, через которую проходит весь измеряемый прибором ток /. Термопара 3 своим спаем припаяна к проволочке 1—2. .Свободные (холодные) концы термопары присоединены к зажимам 4 и 5, а к последним в, свою очередь подключен миллиамперметр (амперметр или гальванометр) маг- нитоэлектрической системы. Таким образом, термопара термо- преобразователя совместно с миллиамперметром образует замк- нутую цепь. Измеряемый ток I, проходя по проволочке 1—2, нагревает ее, а следовательно, нагревает и спай термопары. В результате этого в термопаре создается термоэлектродвижущая сила, возни- кает постоянный ток, который отклоняет стрелку магнитоэлек- трического прибора (миллиамперметра) на некоторый угол. Термопреобразователь, показанный на рис. 389, называется контактным, так как его термопара имеет непосредственный электрический контакт с проволочкой 1—2 (подогревателем). 599
Рис. Л90. Амперметр с тер- мопреобразователем Достоинство его заключается в том, что его термопара более чувствительна к изменению температуры подогрева- теля. Но в то же время он обладает существенным недостатком: при изме- рении токов высокой частоты эти токи могут ответвляться в цепь постоянного тока термопары. Этим недостатком не обладают бесконтактные термопреобра- зователи, потому что их термопары не имеют электрического контакта с подо- гревателем. Но в то же время они ме- нее чувствительны, поскольку влияние температуры подогревателя на их тер- мопары несколько слабее. Для увеличения чувствительности термоэлектрических приборов в них применяют термоэлектрические бата- реи, состоящие из ряда последователь- но соединенных термоэлементов. 4 Постоянный термоэлектрический ток пропорционален термоэлектродвижу- щей силе, а последняя в свою очередь пропорциональна температуре нити нагревателя, т. е. квадрату измеряемого тока. Следовательно, постоянный термоэлектриче- ский ток пропорционален квадрату измеряемого тока. А это зна- чит, что показания магнитоэлектрического прибора в цепи с тер- мопреобразователем будут пропорциональны квадрату тока: а = С72, т. е. шкала термоэлектрического прибора неравномерная. Термоэлектрические приборы весьма успешно применяются при измерении токов высокой частоты, нередко до нескольких мегагерц. Эти приборы гораздо чувствительнее тепловых и дают меньшую погрешность измерений благодаря относительно корот- кой проволоке подогревателя. Поэтому тепловые измерительные приборы постепенно вытесняются из практики термоэлектриче- скими. Термоэлектрические приборы весьма чувствительны к пере- грузкам, а поэтому необходимо весьма внимательно относиться к эксплуатации этих приборов, так как при незначительном от- клонении от номинального тока в сторону его повышения термо- преобразователь легко перегорает. На рис. 390 показан общий вид амперметра с термопреобразователем, выделенным отдельно от магнитоэлектрического прибора. Достоинства термоэлектрических приборов следующие: при- годность как для постоянного, так и для переменного тока, неза- 6G0
виеимость показаний от частоты тока, малое потребление энер- гии, большая точность и чувствительность. Недостатки термоэлектрических приборов: чувствительность к перегрузкам током, неравномерность шкалы, относительно ма- лый срок службы. § 176. ПРИБОРЫ ВИБРАЦИОННОЙ СИСТЕМЫ К приборам вибрационной системы относятся частотомеры, при помощи которых измеряется частота переменных токов. Вибрационный частотомер основан на ис- пользовании механического резонанса вибри- рующего тела На рис. 391 схематически показан принцип действия вибрационного частотомера. Электромагнит А питается током некоторой частоты и создает переменное магнитное поле той же частоты. Перед полюсной надставкой М электромагнита расположена система якорьков b в виде упругих тонких сталь- ных пластинок, заделанных одними концами на общей основе а (на рисунке показана только одна такая пластинка). Каждая из этих пластинок имеет свой собственный период (частоту) коле- баний. За. один период колебаний тока каждая из пластинок дважды испытывает притягивающую силу электромагнита. Но эта вынуждающая к колебаниям сила больше всего воздействует на ту пластинку, которая имеет собственную частоту колебаний, совпадающую с частотой колебаний вынуждающей внешней си- лы. По наибольшей амплитуде колебаний пластинки можно су- дить о числе колебаний электрического тока в цепи. Для того чтобы легко можно было наблюдать за колебания- ми вибрирующих пластинок частотомера, концы их снабжают окрашенными в белый цвет площадками. На рис. 392 показана шкала вибрационного частотомера, включенного для измерения частоты 50 гц. Вибрационные частотомеры применимы для измерения толь- ко низких частот. На рис. 393 показан внеш- ний вид вибрационного часто- томера щитового типа. Достоинства вибрационных приборов следующие: относи- 45 50 55 Рис. 391. Прибор вибра- ционной системы Рис. 392* Шкала вибрацион- ного частотомера, включенного для измерения частоты 50 гц 601
Рис. 393. Вибрационный частотомер расход электрической энергии в тельно малая зависимость показаний от изменения напряжения в сети, на- дежность в работе, про- стота конструкции. Недостатки вибрацион- ных приборов: зависи- мость их показаний от внешних механических ви- браций, прерывность шка- лы, невозможность изме- рения высоких частот. § 177. ЭЛЕКТРОДИНАМИ- ЧЕСКИЕ СЧЕТЧИКИ Счетчиками электриче- ской энергии называют приборы, учитывающие цепях. На рис. 394 показана принципиальная схема электродинамического счетчика постоян- ного тока. Через неподвижные катушки В\ и В2 проходит весь ток А, по- требляемый электрической цепью. В магнитном поле, которое со- здается током, проходящим через катушки, помещен якорь А. Он покоится на подпятнике а и может вращаться* свободно вокруг своей оси. Обмотка якоря секционирована, т. е. разбита на отдельные са- мостоятельные катушки — секции, концы которых присоединены к коллекторным пласти- нам К якоря. Ток /2 к об- мотке якоря подводится через неподвижные щет- ки 6, которые прижимают- ся к коллекторным пласти- нам. При вращении якоря щетки скользят по поверх- ностям 'коллекторных пла- стин. Для предохранения тонкой обмотки якоря от перегрева током последо- вательно с обмоткой вве- дено сопротивление /?1. Катушка С, включенная последовательно с якорем, создает допо л н ите л ь н ое магнитное поле, компен- сирующее механические Рис. 394. Принципиальная схема электро- динамического счетчика 602
потери энергии в счетчике, например на трение в подпятнике, тре- ние щеток о коллектор и т. п. Эта катушка называется компенса- ционной. Якорь включен в цепь параллельно, поэтому через его обмотку проходит ток, пропорциональный напряжению U. Принцип действия счетчика сводится к следующему. Щетки, соприкасаясь с парой коллекторных пластин, присо- единяют к сети одну из секций обмотки якоря, так что только в этой секции протекает ток /2, который взаимодействует с маг- нитным полем, создаваемым неподвижными катушками Bi и В2. Под влиянием силы их взаимодействия секция якоря, «оживлен- ная» током /2, стремится установиться в магнитном поле кату- шек Bi и В2 так, чтобы ее пронизывал максимальный магнитный поток и чтобы направления магнитных полей катушек Bi и В2 и самой секции внутри последней были одинаковыми. Но когда эта секция вместе с якорем повернется на некоторый угол а, щетки пе- рейдут на следующую пару коллекторных пластин, и теперь ока- жется «оживленной» током следующая очередная секция обмотки якоря, которая также повернется на некоторый угол а, и т. д. В результате якорь будет непрерывно вращаться вокруг своей оси с постоянной скоростью. Вращающий момент, развиваемый подвижной системой счет- чика, пропорционален величине магнитной индукции В, создавае- мой неподвижными катушками, и току /2 в обмотке якоря. Так как в системе неподвижных катушек нет железа, то магнитную индукцию можно считать пропорциональной величине тока Ц неподвижной катушки, а ток в якоре /2 — пропорциональным при- ложенному к счетчику напряжению U. Следовательно, вра- щающий момент подвижной системы счетчика пропорционален напряжению U в цепи и току нагрузки Л: M = kUIb (504) где k — коэффициент пропорциональности, зависящий от кон- струкции прибора. Вместе с якорем вращается сидящий на его оси алюминиевый диск D, который находится в магнитном поле постоянного маг- нита М. Индуктированные в диске вихревые токи тормозят вра- щение якоря и этим регулируют число оборотов якоря так, что оно пропорционально мощности, расходуемой в цепи: Р = с1пь (505) где пх — число оборотов счетчика в секунду; q — мощность, соответствующая одному обороту счетчика в секунду. 603
Вся же электрическая энергия, израсходованная в цепи, про- порциональна числу оборотов якоря счетчика: А = Сп, (506) где С — постоянная счетчика, соответствующая количе- ству электрической энергии, отмечаемому счетчиком за один обо- рот своей подвижной части. Вращение якоря передается счетному механизму, регистри- рующему электрическую энергию на циферблате. 'На точность показания электродинамического счетчика влияет изменение внешней температуры, изменяющее электрическое со- противление диска, посторонние магнитные поля и колебания на- пряжения в сети. § 178. ИНДУКЦИОННЫЕ СЧЕТЧИКИ I Рис. 395. Принципиальная схе- ма индукционного счетчика В цепях переменного тока широкое применение получили индукционные счетчики. й На рис. 395 схематически изображено устройство одного из типов индукционных счетчиков. На нижний железный сердечник П-образной формы надеты две по- следовательно соединенные катушки, состоящие из небольшого числа вит- ков толстой проволоки, по которым проходит весь ток нагрузки 1\. Так как магнитный поток Фь создавае- мый током /ь большую часть своего пути проходит по воздуху, то вели- чина его пропорциональна току Л и почти совпадает с ним по фазе. На другой сердечник, располо- женный выше первого, нанесена об- мотка из большого числа витков тон- кой проволоки, присоединенная па- раллельно к цепи. Ток /2, протекаю- щий по этой катушке, создает маг- нитный поток Ф2, который распреде- ляется по двум параллельным маг- нитным цепям. Часть этого потока Ф2 замыкается через диск, находящийся в междужелезном -пространстве. Дру- гая часть потока, Ф2, замыкается че- рез магнитный шунт, благодаря ко- торому увеличивается индуктивное сопротивление параллельной обмотки электромагнита, что необходимо для установления пропорцио- нальности между вращающим моментом подвижной системы счетчика и мощностью, расходуемой в цепи. 604
Магнитный поток Ф2, пронизывающий диск, отстает по фазе от напряжения U на угол, близкий к -у. Если ток Л отстает по фазе от приложенного напряжения на угол то потоки Ф1 и Ф2 сдвинуты по фазе один относительно другого на угол ----<?) . Магнитный поток Ф] индуктирует в подвиж- ном алюминиевом диске вихревые токи /в.т, ко- торые, взаимодействуя с магнитным потоком Ф2, создают вращающий момент, пропорцио- нальный произведению амплитуд магнитных потоков и синусу угла между ними: М = АФХФ2 • sin ---?) = АФхФ2 • cos (507) или Л/ = с UI • cos <р = сР (508) (так как Ф] и Ф2 пропорциональны соответственно величине тока / и напряжению (/), где с — коэффициент пропорционально- сти; Р — мощность, расходуемая в цепи. Постоянный магнит М, в магнитном поле которого вращается диск, создает в диске вихревые токи, поглощающие механическую энергию подвижной системы счетчика, и этим регулирует число оборотов счетчика К достоинствам счетчиков следует отнести допускаемую ими перегрузку, малое собственное потребление мощности (около 1,5 вт), прочность конструкции и невысокую стоимость. Недостатки счетчика: влияние частоты и температуры на по- казания его, относительное увеличение оборотов счетчика при ма- лой нагрузке и повышении напряжения.
ГЛАВА XXXI ЭЛЕКТРИЧЕСКИЕ ИЗМЕРЕНИЯ § 179. ИЗМЕРЕНИЕ ВЕЛИЧИНЫ ТОКА Для непосредственного измерения тока в цепи применяются приборы, называемые амперметрами. При относительно малых токах применяют миллиамперметры, дающие показания непосредственно в миллиамперах, или гальванометры, измеряющие токи порядка микроампер. Для измерения постоян- ных токов применяют ампер- метры магнитоэлектрической, электродинамической и теп- ловой систем. Переменные же токи можно измерять ампер- метрами всех систем, кроме Рис. 396. Схема включения амперметра магнитоэлектрической. с шунтом Амперметр вклю- чается в цепь так, чтобы через него проходил весь измеряемый ток, т. е. последовательно с тем участком цепи, где необходимо измерить величину тока. В тех случаях, когда желают расширить пределы измерения амперметра, его присоединяют параллельно к шунту (рис. 396). Чем меньше сопротивление шунта по сравнению с сопротив- лением амперметра, тем большая часть измеряемого тока отво- дится в шунт и меньшая — в амперметр. Коэффициентом шунта называется число, показы- вающее, во сколько раз предельный ток, измеряемый ампермет- ром - ром с шунтом, больше предельного тока, измеряемого ампермет- без шунта: ZZ — г , *а (509) где л*— коэффициент шунта; 1О — предельный ток, на который рассчитан амперметр без шунта; 606
/—предельный ток, на который рассчитывается амперметр с шунтом. Рассчитать шунт к амперметру — значит определить сопротив- ление его по заданному сопротивлению амперметра и коэффи- циенту шунта. Исходя из схемы, изображенной на рис. 396, можно написать, что /=4,4-4, (1) где /— предельный ток амперметра, на который рассчитывается амперметр с шунтом; /ш — ток в шунте; 1а—предельный ток, на который рассчитан амперметр без шунта; гш—сопротивление шунта; га—сопротивление амперметра. Разделив левую и правую части равенства (1) на /Л, получим: Но, принимая во внимание, что у~=/г и = вместо 'а *а последнего равенства можем написать, что откуда найдем, что ^ = ^1, (510) т. е. сопротивление шунта меньше сопротивле- ния амперметра в число раз, равное шунто- вому коэффициенту без единицы. Пример 200. Амперметр рассчитан на максимальный ток /д = 0,5 а. Тре- буется расширить пределы измерения данного амперметра до 1 — 5 а, т. е. подобрать к нему шунт, если сопротивление амперметра га = 0,81 ом. Решение. Коэффициент шунта Сопротивление шунта гш «= = 0,09 ом. 607
Рис. 397. Шунты для переносных приборов На рис. 397 показан внешний вид шунтов к переносным при- борам. Шунты монтируются внутри амперметра или же подклю- чаются к нему вне его корпуса; На шунте, прилагаемом к дан- Рис. 398. Схема включения амперметра для его про- верки ному амперметру, отмечается предельное значение того тока, ко- торый может измерить прибор с данным шунтом, или же цена деления прибора, например 1° = 0,1 а и т. д. Материалом для изготовления шун- тов обычно служит манганин, обладаю- щий большим удельным сопротивле- нием и очень малым температурным коэффициентом. Следует помнить, что амперметр вместе с шунтом имеет сопротивление Г —~ гагы Га + Г ш ’ что должно быть учтено при всякого рода измерениях, где прихо- дится считаться с сопротивлением амперметра. С течением времени в силу тех или иных причин величина погрешности амперметра возрастает и возникает необходимость Рис. 399. Кривая поправок градуировки амперметра 608
его проверки. Проверяют амперметр Ах, сличая его показания с показаниями эталонного амперметра А9, включенного после- довательно с ним в цепь (рис. 398)". Обычно при проверке амперметра строят кривую поправок (рис. 399): по одной оси откладывают показания проверяемого амперметра, а по другой оси — поправки, т. е. те величины, ко- торые надо прибавить к показаниям амперметра, чтобы получить истинное значение измеряемой величины. Как видно из графика, поправки могут быть положительными и отрицательными. § 180. ИЗМЕРЕНИЕ НАПРЯЖЕНИЯ Напряжение в цепи в зависимости от его величины измеряется вольтметром или милливольтметром. В цепях постоянного тока, когда требуется относительно боль- шая точность измерений, применяются вольтметры магнитоэлек- трической системы, при помощи которых можно измерить напря- жения от долей вольта до нескольких сотен вольт. Там, где не требуется большой точности измерений, применяются’вольтметры электромагнитной системы, при помощи которых можно измерять напряжения от единиц до нескольких сотен вольт, г*-" ----------и------------------«ч L------- ид ------п I I Рис. 400. Схема включения вольтметра с добавочным сопротивлением Высокие напряжения в несколько тысяч вольт обычно изме- ряются электростатическими вольтметрами. В цепях переменного тока применимы все типы вольтметров, за исключением вольтметров магнитоэлектрической системы. Вольтметр включается в цепь так, что его зажимы подключаются к- тем точкам, между которыми измеряется напряжение. Чтобы расширить пределы измерения вольтметра, его соеди- няют последовательно с добавочным сопротивлением (рис. 400). Чем больше добавочное сопротивление по сравнению с сопро- тивлением вольтметра, тем большая часть измеряемого напряже- ния будет израсходована в нем и меньшая — в самом вольтметре. Переводным множителем называется число, пока- зывающее, во сколько раз предельное напряжение, измеряемое вольтметром с добавочным сопротивлением, больше предельного 39—1377 609
напряжения, на которое рассчитан вольтметр без добавочного сопротивления: /И=4В’ где т — переводный множитель; UB—предельное напряжение, на которое рассчитан вольт* метр без добавочного сопротивления; U—предельное напряжение, на ,которое рассчитывается вольтметр с добавочным сопротивлением. Исходя из схемы, изображенной на рис. 400, можно напи- сать, что и=ил + ив, (1) где U—предельное напряжение, на которое рассчитывается вольтметр с добавочным сопротивлением; UB — предельное напряжение, на которое рассчитан вольт- метр без добавочного сопротивления; ил — напряжение, теряемое в дополнительном сопротив- лении; гв — сопротивление вольтметра; гд — добавочное сопротивление. Разделив левую и правую части равенства (.1) на UB, по- лучим: т т Л G Но, принимая во внимание, что = /и и , вместо последнего равенства можем написать, что /п = 14--^-, гв откуда найдем, что Гд = (/П—1)гв, (511) т. е. добавочное сопротивление больше сопро- тивления вольтметра в число раз, равное пере- водному множителю без единицы. Пример 201. Вольтметр рассчитан на максимальное напряжение UB = 3 в. Требуется рассчитать добавочное сопротивление К' вольтметру, если пределы намерения его необходимо расширить до величины £/ = 30 в. Сопротивление вольтметра гв = 500 ом. 610
Решение. Переводный множитель U 30 т”7л“Т 10. Добавочное сопротивление 4 = (т— 1)гв = (10— 1)-500 = 4500 ом. Добавочное сопротивление монтируется внутри вольтметра или же присоединяется к нему вне его корпуса. Оно обычно изго- товляется из манганиновой проволоки с изоляцией, намотанной на цилиндрической формы каркас или же на тонкую плоскую пла- стинку из изолирующего материа- ла. Чтобы сделать катушки доба- вочного сопротивления безындук- ционными, их выполняют в виде бифилярной обмотки. В некоторых случаях требует- ся, чтобы один и тот же вольт- метр был рассчитан на различные Рис. 401. Схема включения вольт- метра для его проверки пределы измерения напряжения. Тогда для него добавочные сопротивления вводят в виде от- дельных секций, которые включают по мере необходимости. Проверяют и градуируют вольтметр, сличая его показания с показаниями эталонного вольтметра Va, включенного парал- лельно с ним в цепь (рис. 401). В результате проверки вольт- метра к нему прилагают кривую поправок, аналогичную кривой поправок для амперметра (рис. 399). § 181. ИЗМЕРЕНИЕ СОПРОТИВЛЕНИИ МЕТОДОМ АМПЕРМЕТРА И ВОЛЬТМЕТРА Если, известно напряжение U, теряемое на данном участке цепи, и ток I, протекающий по, нему, то, пользуясь формулой закона Ома для участка цепи, можно определить сопротивление последнего: Следовательно, если даны показания вольтметра и ампер- метра, включенных в заданный участок цепи, то по ним можно определить сопротивление участка. Этот метод определения со- противления участка цепи называется ’ методом амперметра и вольтметра. Для измерения сопротивления гх по методу амперметра и вольтметра соберем цепь по схеме, приведенной на рис. 402. Эта цепь состоит из источника электрической энергии с постоянной электродвижущей силой и последовательно соединенных с ним 39* 611
искомым сопротивлением гх и амперметром Л. Вольтметр V под- ключен к зажимам источника электрической энергии. При этом способе включения приборов амперметр измерит ток Z, проходящий через измеряемое сопротивление а вольт- метр покажет напряжение, которое теряется не только в неиз- вестном сопротивлении гх, но и в амперметре А, обладающем со- противлением го: U = Irx + Ira. Отсюда следует, что г. = ^-га. (512) т. е. искомое сопротивление равно частному от деления напря- жения U на величину тока / без сопротивления амперметра га. Если сопротивление га <С гх, то сопротивлением га можно пренебречь и положить, что Если, же сопротивление га такого же порядка, что и сопро- тивление то сопротивлением га пренебрегать нельзя, так как можно допустить очень грубую ошибку при определении гх. Следовательно, измерить сопротивление методом вольтметра и амперметра согласно схеме р и с. 402 можно только в тех случаях, когда не- известное сопротивление гх значительно больше сопротивления амперметр а* га> Покажем это на простейших примерах. Пример 202. Вольтметр, включенный согласно схеме рис. 402, показал напряжение U = 100 в. а амперметр показал ток I = ОД а. Определить не- известное сопротивление гх, если сопротивление амперметра ra = 1 ож. Решение. По формуле закона Ома находим 1 гх = -у = = 1000 ом. Фактически же согласно формуле (512) гх = — га = 1000 — 1 = 999 ом, т. е. ошибка, допущенная при расчете гх по приближенной формуле, не превышает 0,1%. Пример 203. Вольтметр, включенный согласно схеме рис. 402, показал напряжение (7=10 в, а амперметр показал ток I = 5 а. Определить неиз- вестное сопротивление гх, если сопротивление амперметра Га = 1 ом. Решение. По формуле закона Ома находим 612
Фактически же согласно формуле (512) Л. “ = 2 - 1 = 1 ОМ. л j “ т. е. при измерении сопротивления гх по. схеме рис. 402_ была допущена ошибка на 100% против действительной величины. А это значит, что в дан- ном случае применять вышеуказанную схему для измерения сопротивле- ния гх рекомендовать нельзя. Если для определения сопротивления включить амперметр и вольтметр так, как это показано на рис. 403, то вольтметр покажет истинное значение напряжения, приложенное к неиз- Рис. 402. Схема измерения сопроти- Рис. 403. Схема измерения сопроти- вления по методу амперметра и вления по методу амперметра и вольт- вольтметра метра вестному сопротивлению гх. Амперметр же измерит ток /, р-авный сумме двух токов: /=4 + 4, где 4 — ток в сопротивлении гх, /в — ток в вольтметре. Так как 4 = /-4, г = х ix i-ib’ т. е. неизвестное сопротивление равно частному от деления на- пряжения на разность тока, показываемого амперметром, и тока, протекающего по вольтметру. Принимая во внимание, что вместо последнего равенства можно написать (512') 613
Если, гв то величиной ~ можно пренебречь по сравнению Гв С ТОКОМ I и положить, что Если же сопротивление гв того же порядка, что и гх, то вели- чиной пренебрегать нельзя, так как при определении гх будет 'в допущена грубая ошибка. Следовательно, схему включения а м п е р м ет р а, определения вольтм етр а и изображенную на рис. 403, для Неизвестного сопротивления гх можно рекомендовать только для случаев, когда это сопротивление значительно меньше сопротивления вольтметра гв. Пример 204. Вольтметр, имеющий сопротивление гв = 10 000 ом, вклю- чен по схеме рис. 403 и показал U = 100 в, а амперметр показал ток /=Ю а. Определить неизвестное сопротивление гх. Решение. По формуле закона Ома находим £7 100 in =10ол/- Фактически согласно формуле (512.) „ и 100 100 1ЛЛ1 Гх~ , и_ “ ]0 юо = 9,99 = 10,01 0М' 1 гв 10 000 т. е. ошибка, допущенная при расчете гх по приближенной формуле, не пре- вышает 0,1%. Пример 205. Вольтметр, имеющий сопротивление гв = 10 000 ом, включен по схеме рис. 403 и показал напряжение U = 100 в, а амперметр показал ток I = 0,02 а. Определить неизвестное сопротивление гх. Решение. По формуле закона Ома находим rx = -j- = ^ = 5000 ом. Фактически согласно формуле (512) r* = JUU_ = 0 021°°100 = (S = 10 000 ом’ гв ’ 10 000 т. е. ошибка допущена на значит, что эту схему для рекомендовать нельзя. 100% против действительной величины. А это определения сопротивления гх в данном случае § 182. ИЗМЕРЕНИЕ БОЛЬШИХ СОПРОТИВЛЕНИЙ МЕТОДОМ ВОЛЬТМЕТРА В тех случаях, когда требуется измерить очень большое сопротивление, например сопротивление изоляции провода, можно применить так назы- ваемый метод вольтметра. 614
На рис. 404 изображена схема включения вольтметра в цепь для изме- рения сопротивления изоляции провода (кабеля) по отношению к земле. Вольтметр для этой цели берется обычно с большим сопротивлением, от 20 000 до 40 000 ом, и со шка- лой от 200 до 300 в. Источник электрической энергии должен иметь электродвижущую силу, примерно равную 200—300 в, и малое внутреннее сопроти- вление (аккумуляторная бата- рея). Вначале вольтметр при- соединяют к зажимам источ- ника электрической энергии и Рис. 404. Схема измерения сопротивления изоляции провода методом вольтметра измеряют его напряжение при холостом ходе, которое при очень малом внутреннем сопротивлении источ- ника электрической Энергии го по сравнению с сопротивлением вольтметра равно электродвижущей силе источника: U = Е — IrQ « Е; где U — напряжение, измеренное вольтметром на зажимах источника элек- трической энергии при разомкнутой внешней цепи; /—ток, протекающий в цепи вольтметра при разомкнутой внешней цепи. Пример 206. Вольтметр, обладающий сопротивлением гв = 20 000 ом, подключенный к зажимам источника электрической энергии, при разомкну- той внешней цепи показал U = 200 в. Определить, какую ошибку мы допу- стили, если напряжение, измеренное вольтметром, приняли за электродви- жущую силу источника электрической энергии. Внутреннее сопротивление источника электрической энергии r0 = 1 ом. Решение. Величина тока в цепи вольтметра, присоединенного к зажи- мам источника электрической энергии при разомкнутой внешней цепи, равна Электродвижущая сила источника электрической энергии Е = U 4- lrQ = 200 4- 0,01 • 1 =* 200,01 в, т. е. мы допустили ошибку на 0,005%, если приняли напряжение V за ве- личину электродвижущей силы. Чем меньше внутреннее сопротивление го источника электрической энер- гии по сравнению с сопротивлением вольтметра гв, тем меньше разница между напряжением на зажимах источника электрической энергии, измерен- ной вольтметром при разомкнутой внешней цепи, и электродвижущей си- лой его. Измерив напряжение на зажимах источника электрической энергии при холостом ходе, подключают вольтметр последовательно с проводом, свобод- ный конец которого отключен от потребителя и изолирован от земли. Тай как один из полюсов источника электрической энергии заземлен, то при утечке тока через изоляцию на землю ток потечет от плюс-зажима источника электрической энергии через вольтметр, через изоляцию провода в землю и затем возвратится к минус-зажиму. Вольтметр, включенный по- следовательно с сопротивлением изоляции гх, отметит то напряжение UB, ко- торое теряется в нем самом. Остальная доля электродвижущей силы Е источника электрической энергии теряется в сопротивлении изоляции />, если пренебречь падением напряжения во всех остальных участках цепи. .615
Следовательно, на сопротивление изоляции гх будет затрачено напря- жение UX~E-UB. Как известно, напряжение в последовательной цепи распределяется про- порционально сопротивлениям участков цепи, т. е. в данном случае rx E — UB rB UB ’ откуда (513) Формула (513) дает возможность определить сопротивление изоляций кабеля, если известны rB, Е и UB.' Пример 207. Вольтметр, включенный в провод для измерения сопротив- ления изоляции последнего, показал UB = 5 в. Определить сопротивление изоляции провода, если сопротивление вольтметра гв = 25 000 ом, электро- движущая сила источника электрической энергии Е = 225 в. Решение. По формуле (513) находим р___и 90S_______5 Гх = Е -?-гв = -25 000 = 1 100 000 ом. § 183. ИЗМЕРЕНИЕ СОПРОТИВЛЕНИЙ ОММЕТРОМ Омметром называется электроизмеритель- ный прибор, с помощью которого можно непо- средственно измерять со- противление проводни- ков. Омметр представляет собой при- бор магнитоэлектрической системы, отградуированный непосредственно в омах. На рис. 405 приведена схема гра- дуировки омметра, где показаны: Рис. 405. Схема градуировки источник электрической энергии с омметра постоянной электродвижущей си- лой Е (один элемент или батарея элементов), магнитоэлектрический прибор 2, который предстоит проградуировать в омах, добавочное сопротивление /?доб к нему и переменное достаточно точное сопротивление /?э, которое можно изменять практически от нуля до бесконечности. Вначале устанавливают R3 = оо, т. е. практически размы- кают цепь, и тогда ток в цепи становится равным нулю. Стрелка прибора 2 занимает крайнее положение на шкале, соответ- ствующее сопротивлению /?э = оо. В соответствии с этим на шкале прибора против острия его стрелки Наносят оо. Затем устанавливают сопротивление /?9, например, равным R9 = 616
= 1 Мом, В этом случае в цепи будет проходить некоторый по- стоянный ток, который отклонит стрелку омметра (Q) на неко- торый угол. В соответствии с этщл на шкале прибора против острия стрелки его наносят число 1 Мом. Каждому значению переменного сопротивления /?э соответствует строго определен- ное положение стрелки прибора 2 на его шкале, которое и фиксируется величиной этого сопротивления. В конце градуи- ровки омметра переменное сопротивление /?э закорачивается, т. е. устанавливается равным 7?э = 0. В этом случае ток в цепи с ом- метром достигнет максимального значения, а стрелка прибора отклонится на максимальный угол от своего начального (оо) положения. В соответствии с этим на шкале прибора против острия стрелки его наносят 0. Шкала магнитоэлектрического прибора, проградуированного в омах, получается неравномерной, сильно сжатой в диапазоне больших сопротивлений. Это и понятно, так как чем больше пе- ременное сопротивление /?эпо сравнению с сопротивлением ом- метра 2 и дополнительным сопротивлением, тем грубее стано- вится прибор. Можно считать, что омметр дает показания с до- статочной для практики точностью, если измеряемое сопротивле- ние /?х находится в пределах 0,1 —10 сопротивления прибора 2 с добавочным сопротивлением /?доб. После того как омметр проградуирован, им можно измерять неизвестные сопротивления Rx. Для этой цели в цепи, изобра- женной на рис. 405, вместо сопротивления /?э включают неиз- вестное сопротивление Rx и по отклонению стрелки прибора узнают величину измеряемого сопротивления. Так как омметр 2 проградуирован при строго определенной постоянной электродвижущей силе, то, казалось бы, необходимо иметь такую же электродвижущую силу в цепи омметра и при измерении им сопротивлений. Однако это трудно осуществить, так как электродвижущая сила элементов с течением времени изменяется. Выходом из этого положения является регулировка омметра с помощью магнитного или электрического шунта. Магнитный шунт, помещаемый непосредственно внутри магнитоэлектрического прибора, представляет собой железную пластинку, которую можно особым механизмом приближать к по- люсным надставкам прибора или удалять от него и этим уве- личивать или уменьшать величину магнитной индукции магнит- ного поля подковообразного магнита прибора. Если электродви- жущая сила источника электрической энергии в приборе больше нормальной, то магнитный шунт надо приблизить к полюсным надставкам и этим уменьшить магнитную индукцию магнитного поля в пространстве, где расположена рамка прибора. Если же электродвижущая сила Е будет меньше нормальной, необходимо магнитный шунт несколько удалить от полюсных надставок под- ковообразного магнита прибора и этим увеличить магнитную ин- дукцию в междужелезном пространстве. 617
Электрический шунт омметра представляет собЬй пе- ременное сопротивление /?ш, включенное параллельно рамке при- бора. Величину этого сопротивления берут примерно в 10—20 раз большей по сравнению с сопротивлением самого магнитоэлектри- ческого прибора. Изменяя величину шунтирующего сопротивле- ния 7?ш, можно легко отрегулировать прибор в соответствии с его градуировкой. Прежде чем приступить к измерению сопротивлений омме- тром, необходимо его отрегулировать. При разомкнутой внешней цепи, когда измеряемое сопротивление = со, стрелка при- бора должна показывать на шкале со. Если она не показы- вает со, необходимо отверткой повернуть винт корректора, кото- рый имеется в каждом омметре, так, чтобы стрелка установилась на со. Затем устанавливают внешнее (измеряемое) сопротивле- ние равным нулю, т. е. закорачивают внешние зажимы омметра. Если стрелка омметра в этом случае не устанавливается на нуль, то это значит, что электродвижущая сила источника электриче- ской энергии, питающего током цепь омметра, отличается от нор- мальной. Тогда необходимо с помощью магнитного или электри- ческого шунта установить стрелку прибора на нулевое деление шкалы. После этого можно считать, что омметр отрегулирован и им можно измерять сопротивления. § 184. ИЗМЕРЕНИЕ СОПРОТИВЛЕНИЙ МЕТОДОМ МОСТИКА На рис. 406 показана принципиальная схема мостика, состоящая из четырех сопротивлений: г2, Гз и гх, называемых плечами мостика. Эти сопротивления образуют четырех- угольник, в одну из «диагоналей» которого включен гальвано- метр G, дающий возможность обнаружить токи, очень малые по величине. В другую «диагональ» четырехугольника включен источник электрической энергии с постоянной электродвижу- Рис. 406. Принципиальная схема мо стика щей силой. Под диагональю мы подразумеваем здесь не геометрическую диагональ как прямую линию, соеди- няющую противоположные вершины четырехугольника, а участок цепи, подсоеди- ненный к этим вершинам и нанесенный на чертеж произвольно. Сопротивления ri и г2 мо- стика называются его б а- лансными плечами, сопротивление Гз—с р а в н и- тельным и сопротивле- ние — измеряемым. 618
Гальванометр подключен к точкам с и d, а источник электри- ческой энергии — к точкам а и b схемы мостика. Если замкнуть ключи А"6 и /<г, то в схеме мостика возникнет электрический ток /, который от плюс-зажима источника электрической энергии идет до узловой точки а схемы, где разделяется на токи Д и 12 и далее идет уже по двум направлениям: / = Д + 72. В плечах мостика с сопротивлениями гх и г2 происходит не- которое падение напряжения = Дг£ И U2 = /2Г2* В общем случае потенциалы точек с и d различны из-за раз- личных величин падения напряжения U\ и U2, Если окажется, что U\>U2, то это значит, что потенциал точки с меньше по- тенциала точки d: В соответствии с этим ток в диагонали, имеющей гальвано- метр, течет от точки d к точке с, и стрелка гальванометра, от- клонившись от нулевого положения ^а некоторый угол, напри- мер, вправо, отметит присутствие тока в диагонали мостика. Если же U2^> U\t то потенциал точки d меньше потенциала точки с: В этом случае ток через гальванометр проходит от точки с к точке d, и стрелка гальванометра теперь отклонится влево от нуля, показывая этим присутствие тока в цепи гальванометра. Токи /3 и Д, проходящие через сопротивления Гз и гх мостика, сходятся в узловой точке b схемы:- 4 + 4 = Л и далее ток I течет к минус-зажиму источника электрической энергии. Направление и величина тока /г в цепи гальванометра зави- сят от разности потенциалов между точками с и d, а следова- тельно, и от величины сопротивлений плеч мостика. Можно подобрать сопротивления плеч мостика так, что по- тенциалы точек end будут равны друг другу; тогда <fc — = и тока в цепи гальванометра не будет: /г=о. 619
В этом случае должно быть выполнено требование Л = Л: И ^2 = 4, так как ток в цепи гальванометра отсутствует. Кроме того, должны быть соблюдены условия равенства падений напряжений М = /2г2 и /Х^ = /Зг8. Разделив последние два равенства одно на другое, получим 1 Л^2 ЛЛг 4Гв Но так как J\=IX и /2 = /з, то последнее равенство после упрощения принимает *вид П — 6 Гх 6 ’ откуда V3 = Vx, (514) т. е. условием равновесия*в схеме мостика, или, иначе, отсут- ствия тока в гальванометре, является равенство произведений со- противлений противоположных плеч мостика. Следовательно, пользуясь формулой (514), можно найти неизвестное сопротив- ление г,, если все остальные сопротивления плеч мостика из- вестны: r, = irs. (515) Отношение сопротивлений у- называется отношением балансных плеч мостика. Следовательно, чтобы измерить сопротивление при помощи мостика, надо в схеме его подобрать известные сопротивле- ния /'1, г2 и Гз так, чтобы установилось равновесие в схеме мо- стика, и затем по этим сопротивлениям определить по фор- муле (515). По методу измерения сопротивлений мостики разделяются на две основные группы. В одной из них устанавливается постоян- ное отношение балансных плеч , и равновесие в мостике до- стигается при помощи подбора сравнительного сопротивления г3. В другой группе мостиков устанавливается постоянство срав- нительного сопротивления, и равновесие в мостике достигается при помощи изменения отношения балансных плеч. 620
§ 185. ТИПЫ МОСТИКОВ На рис. 407 показана схема магазинного мостика завода «Электроприбор». Мостик имеет: три магазина сопротивлений, из которых со- противления Г1 и г2 — балансные плечи, а остальные сопротив- ления (единицы, десятки, сотни и тысячи омов) составляют срав- нительное сопротивление; гальванометр с отметкой на шкале Рис. 407. Схема магазинного мостика завода „Электроприбор" «Много» и «Мало» — магнитоэлектрической системы с двусторон- ней шкалой; коммутатор с гнездами ВГ, ДС, между которыми включено сопротивление 5000 ом, и гнездом НГ-, два ключа, из которых ключ Б — для включения батареи и ключ Г — для за- мыкания цепи гальванометра; шесть зажимов: к зажимам П и 3 подключается искомое сопротивление, к зажимам (+) и (—) — источник электрической энергии постоянного тока, к зажимам Hi] и НГ — наружный гальванометр в случае порчи гальванометра прибора. На рис. 408 показан внешний вид мостика завода «Электро* прибор». 621
Измерения мостиком завода «Электроприбор» в основном сво- дятся к следующему: 1. Батарею гальванических элементов присоединяют к зажи- мам (+) и (—). Напряжение питающей батареи и наиболее Рис. 408. Внешний вид мостика завода „Электро- прибор” выгодное соотношение балансных плеч мостика в зависимости от измеряемого сопротивления выбирают согласно данным, поме- щенным в табл. 44. Таблица 44 Измеряемое сопротивление, ом Наибольшее допускаемое напряжение батареи, в Вынутые штепсели на плечах мостика умножить разделить 0—10 2 1 1000 10—100 2 10 1000 100—1000 4 100 1000 1000—10 000 ч. 4 1000 1000 10 000—100 000 10 1000 100 100 000—1 000 000 10 юоэ 10 1 Мом — 10 Мом 20—40 1000 1 2. Измеряемое сопротивление подключают к зажимам П и 3 йостика. 3. Штепсель коммутатора, имеющего гнезда с отметкой ВГ. (внутренний гальванометр), ДС (дополнительное сопротивле- ние) и НГ (наружный гальванометр), вставляют в гнездо ДС, т. е. последовательно с внутренним сопротивлением гальвано- 622
метра вводят дополнительное сопротивление 5000 ом, предохра- няющее гальванометр от чрезмерно большого тока. 4. В балансных плечах мостика «умножить» и «разделить» штепсели вставляют в гнезда 100 и 100, т. е. отношение баланс- ных плеч берут равным единице, а в сравнительном сопротив- лении устанавливают сопротивление такого же порядка, как и измеряемое. 5. Нажимают ключ Б и после этого на короткий промежуток времени — ключ Г (размыкать эти ключи следует в обратном порядке, т. е. сначала надо разомкнуть ключ Г, а затем ключ Б). Стрелка гальванометра, отклонившись вправо или влево, пока- жет на шкале отметку «Мало» или «Много»: первая означает, что сравнительное сопротивление мало и его надо увеличить, а вторая — что оно велико и его надо убавить. Подбирая сравни- тельное'сопротивление, добиваются равновесия в мостике. Если в начале измерения будет замечено, что стрелка галь- ванометра не установлена на нуль, то это следует сделать при помощи регулировочного винта, расположенного на крышке галь- ванометра. 6. Добившись равновесия в мостике, увеличивают его чув- ствительность, для чего выключают дополнительное сопротивле- ние ДС, переставив штепсель из гнезда ДС в гнездо ВГ. Затем вновь устанавливают в мостике равновесие и окончательно определяют искомое сопротивление. 7. Если гальванометр мостика неисправен, то следует к за- жимам НГ— НГ подключить какой-либо другой гальванометр. В этом случае переставляют штепсель из гнезда ДС в гнездо ЯГ, и после этого делают окончательное измерение методом, изло- женным выше. 8. При работе с мостиком надо следить за тем, чтобы про- водники мостика не перегревались от чрезмерно больших токов. Для этого нужно подбирать соответствующей величины Напря- жение питающей батареи и не допускать замыкания ключей Б и Г на длительное время. На рис. 409 представлена схема универсального мостика УМВ с постоянным отношением балансных плеч, выпускаемого отече- ственной электропромышленностью. Все три плеча мостика составлены из магазинов сопротивле- ний рычажного типа. Магазин сопротивлений I путем установле- ния рычага _на тот или иной контакт дает возможность устано- вить некоторое постоянное отношение баласных плеч мостика Ri ' 1 Rt 100 п в пределах от ^L="iooo д0 ~аГ=-Т~‘ Сравнительное сопро- тивление /?з состоит из четырех последовательно соединенных ма- газинов сопротивлений и может быть взято в пределах 1—9999 ом, меняясь ступенями через один ом. Неизвестное (из- меряемое) сопротивление подключается к зажимам Дл. 623
Мостиком УМВ можно измерять сопротивления от 0,01 до 10е ом. Точность измерения1 не превышает +0,5%. На рис. 410 приведена Принципиальная схема линейного мо- стика. В нем балансные сопротивления и и г2 представляют со- бой сопротивления участков de и Ьс калиброванной проволоки. Перемещая движок С вдоль калиброванной проволоки bd, изме- няют длины ее отрезков de и Ьс и этим изменяют соответственно Рис. 410, Схема линейного мостика постоян- ного тока 624
балансные сопротивления г\ и г2. Следовательно, линейный мо- стик относится к категории мостиков с переменным отношением балансных плеч. В качестве сравнительного сопротивления Гз в нем применяется магазин сопротивлений. Так как калиброванная проволока bd линейного мостика со- вершенно однородна по составу и имеет всюду одинаковое попе- речное сечение, то отношение сопротивлений ее участков de и Ьс численно равно отношению длин этих участков, т. е. G Л /*2 ^2 где /1 — длина участка de калиброванной проволоки; /2 — длина участка Ьс калиброванной проволоки. В соответствии с этим в формуле для равновесия мостика можно отношение сопротивлений балансных плеч мостика '2 заменить отношением соответствующих длин калиброванной *2 проволоки. Следовательно, формулу для определения неизвестного со- противления гх можно представить в следующем виде: г, = |-г8. (516) *•2 Для измерения индуктивностей и емкостей применяются мо- стики переменного тока. Мы здесь не можем останавливаться на их теории, которая подробно рассматривается в специальной ли- тературе по электрическим измерениям. Ограничимся ^несколь- кими общими замечаниями. Как известно, основным условием равновесия мостика по- стоянного тока является равенство произведений сопротивлений его противоположных плеч. Равновесия такого мостика можно достигнуть изменением сопротивления* одного плеча, например сравнительного сопротивления, при постоянстве отношения ба- лансных плеч. В противоположность этому равновесие в мостике переменного тока при измерении индуктивности или емкости до- стигается изменением не менее двух его параметров, например активного сопротивления и одновременно с этим индуктивности или емкости. Условие равновесия в мостике переменного тока выражается системой двух независимых уравнений Z1Z3 = Z2Z4, (517) ?1 + ?3 = ?2 + ?4- (518) Уравнение (517) показывает, что для равновесия мостика пе- ременного тока необходимо равенство произведений численных значений полных сопротивлений противоположных плеч мостика. 40—1377 625
Рис. 411. Схема мостика УМ-2 для измерения емкости щего подбора индуктивностей биться равенства Уравнение (518) показывает, что для равновесия мостика перемен- ного тока необходимо равенство сумм углов сдвига фаз между со- ответствующими токами и напря- жениями противоположных плеч мостика. Из уравнения (518) можно сделать вывод, что если первое и третье плечи мостика являются участками цепи емкостного ха- рактера, т. е. ?i<0 и ?з<0, а вто- рое и четвертое плечи — участка- ми индуктивного характера, т. е. <?2>0 и ?4>0, то равновесия в мостике достигнуть нельзя. Если же первое и второе пле- чи — индуктивного характера, т. е. <Pi>0 и <?2>0, а третье и четвер- тое — емкостного, т. е. <рз<^0 и <р4<0, то путем соответствую- или емкостей можно до- ?1 + Фз = Ъ + ?4- На рис. 411 представлена схема для измерения емкости с по- мощью универсального мостика переменного тока типа УМ-2, выпускаемого заводом «Эталон». Здесь первое плечо состоит из активного сопротивления которое может быть взято равным 1; 10; 100; 1000; 10 000; 100 000 ом. Второе плечо представл1яет собой плавно изменяющееся переменное сопротивление Rcrl (ин‘ дексы CRL показывают, что это сопротивление используется для измерения не только емкости, но и активного сопротивления и индуктивности). Третье плечо мостика состоит из последова- тельно соединенных образцовой (точно выверенной) емкости Сн и образцового сопротивления /?н. Четвертое плечо надо пони- мать как эквивалентную схему конденсатора с идеальной емко- стью Сх и последовательно соединенным активным сопротивле- нием /?>, обусловливающим мощность потерь в диэлектрике кон- денсатора. Порядок измерения емкости мостика типа УМ-2 следующий. Вначале устанавливают образцовое активное сопротивление /?н в третьем плече мостика равным нулю (А?и = 0) и изменяют от- ношение балансных плеч мостика, добиваясь наименьшего KCRL показания нулевого прибора НП. После этого приступают к ре- гулировке сопротивления Ru, добиваясь дальнейшего уменьшения 626
показания нулевого прибора. Затем вновь переходят к регули- ровке отношения сопротивлении балансных плеч мостика (-5-), \ KCRL J потом опять к регулировке сопротивления /?н и т. д. Регули- ровка заканчивается тогда, когда ток в нулевом приборе ока- жется равным нулю, что будет свидетельствовать о наступлении момента равновесия в мостике. В данном частном случае, когда сопротивления первого и вто- рого плеч мостика — чисто активные, уравнения мостика прини- мают более простой вид: RxRcRL — RcRi£h = В£х' Отсюда находим, что (519) О С,—^С„ (520) Мостиком УМ-2 можно измерять емкости от 10 пфцо 100 мкф. Наименьшая погрешность измерения при определении емкости составляет примерно 1 %. Мостик УМ-2 дает возможность измерять и индуктивности от 10 мкгн до 100 гн. При этом наименьшая погрешность измерения составляет 1-т-2%. § 186. ИЗМЕРЕНИЕ МОЩНОСТИ ЭЛЕКТРИЧЕСКОГО ТОКА Электродинамические ваттметры пригодны для измерения мощности в цепях постоянного и переменного тока. Как было выяснено ранее, электродинамический ваттметр основан на принципе взаимодействия неподвижной и подвиж- ной катушек. Первая из них соединена последовательно с потребителем, и по ней про- ходит весь ток нагрузки. Вторая катушка присоеди- нена к потребителю парал- лельно, и по ней проходит ток, пропорциональный на- пряжению, приложенному к потребителю. Здесь можно различать два варианта под- ключения концов подвижной 40* Рис. 412. Схема измерения мощности электродинамическим ваттметром 627
катушки по отношению к концам неподвижной катушки. Если подвижная катушка включена по схеме, приведенной на рис. 412, то U = U+IRa, где ' U — напряжение на концах подвижной катушки; U — напряжение на потребителе; IRa — падение напряжения в неподвижной катушке; Ra — сопротивление неподвижной катушки. Мощность, которую зафиксирует ваттметр при заданной схеме включения подвижной катушки, определится формулой Р = (U + IRa) I = UI + PRa = P + PRa, где Р — мощность, зафиксированная ваттметром; Р— мощность, расходуемая в потребителе; PRa — мощность, расходуемая в неподвижной катушке. Рис. 413. Схема измерения мощности элек- тродинамическим ваттметром Следовательно, истинная мощность Р, развиваемая током в потребителе, меньше мощности Р', зафиксированной ваттме- тром, на величину мощности, расходуемой в неподвижной ка- тушке: P=P — PRa. (521) В схеме, изображенной на рис. 413, один из концов подвиж- ной катушки присоединен за неподвижной катушкой. В этом слу- чае напряжение на концах подвижной катушки равно напряже- нию на зажимах потребителя. Ток I в потребителе меньше тока в неподвижной катушке на величину тока Z, протекающего через подвижную катушку, т. е. где Г — ток в неподвижной катушке; / — ток в потребителе. 628
Мощность, которую зафиксирует ваттметр в этом случае, определяется формулой Р” = U (Z 4- i) = UI + Ui = Р + , Кв где Р"—мощность, зафиксированная ваттметром; Р—истинная мощность, расходуемая в потребителе; U2 ‘ * ~б---мощность, расходуемая в подвижной катушке; Кв RB — сопротивление подвижной катушки. Следовательно, истинная мощность Р, развиваемая током в потребителе, меньше мощности Р", зафиксированной ваттме- тром, на величину мощности, расходуемой в подвижной катушке: Р = Р’ — (522) Кв Лри точных измерениях мощности необходимо иметь в виду потери мощности в катушках ваттметра и, зная заранее сопро- тивление катушек, подсчитать эти потери и' внести поправки в показания ваттметра. Когда не требуется большая точность измерений мощности, показания ваттметра принимают за вели- чину измеряемой мощности. В ваттметрах обычно последовательно с подвижной катуш- кой подключается добавочное сопротивление гд. В этом случае при определении мощности, расходуемой в подвижной катушке, надо учесть и это добавочное сопротивление. У некоторых электродинамических ваттметров один из зажи- мов неподвижной и один из зажимов подвижной обмоток поме- чены звездочками. В этом случае для правильного включения ваттметра в цепь указанные зажимы соединяют друг с другом коротким, малого сопротивления проводом. Эти зажимы обычно называют генераторными, так как к ним непосредственно присо- единяется провод, идущий от одного из зажимов сети (генера- тора электрической энергии). При неправильном включении неподвижной и подвижной об- моток ваттметра в цепь стрелка прибора может отклоняться влево от нулевого деления. В этом случае концы подвижной об- мотки ваттметра надо переключить. § 187. ИЗМЕРЕНИЕ ЭЛЕКТРИЧЕСКОЙ ЭНЕРГИИ Для непосредственного измерения электрической энергии, расходуемой в потребителе, применяются счетчики электрической энергии. Ранее мы выяснили, что показания счетчиков как электроди- намической, так и индукционной систем пропорциональны чис- лам оборотов их подвижных частей (якорь, диск): W=Cn> 629
где — энергия, зарегистрированная счетчиком; п — число оборотов, совершенное подвижной частью счет- чика; С—постоянная счетчика — энергия, соответствующая од- ному обороту подвижной части счетчика. Для определения постоянной счетчика собирают схему, изо- браженную на рис. 414. Сняв показание U вольтметра, I амперметра, подсчитывают число оборотов счетчика п за некоторое время t секунд и затем определяют постоянную счетчика по формуле г квт-ч * * 3 600 000л 1 оборот * (523) где С — постоянная счетчика в киловатт-часах на один оборот; U — напряжение в вольтах; I — ток в амперах; t—время в секундах; п — число оборотов за время t секунд. Рис. 414. Схема определения постоянной счет- чика электрической энергии Обычно постоянную счетчика определяют как среднее арифме- тическое из постоянных для 10, 25, 50 и 100% номинальной на- грузки счетчика. В цепях постоянного тока измерение электрической энергии осуществляется при помощи счетчиков электродинамической си- стемы. В однофазных цепях переменного тока измерение электриче- ской энергии, помимо счетчиков электродинамической системы, осуществляется при помощи счетчиков индукционной системы, описание которых было приведено выше.
ЧАСТЬ ЧЕТВЕРТАЯ ХИМИЧЕСКИЕ ИСТОЧНИКИ ЭЛЕКТРИЧЕСКОЙ ЭНЕРГИИ ГЛАВА XXXII АККУМУЛЯТОРЫ § 188. ПРИНЦИП ДЕЙСТВИЯ АККУМУЛЯТОРОВ Работа всякого химического источника электрической энер- гии связана с прохождением электрического тока через электро- лит; являющийся одной из важнейших составных частей каждого из этих источников. Поэтому прежде чем рассматривать работу аккумуляторов и гальванических элементов, ознакомимся с хи- мическим действием тока. Рис. 415. Электролитическая диссоциация мо- лекулы Во всяком растворе солей, кислот и оснований (в электролите) происходит непрерывный процесс электролитической диссоциа- ции. Сущность его заключается в том, что молекулы-диполи воды (растворителя) электрически воздействуют на молекулы-диполи растворенного вещества и заставляют их распадаться на поло- жительные и отрицательные ионы (рис. 415). При электролитической диссоциации молекул сложных ве- ществ металлы и водород, как правило, выпадают в виде' поло- 63)
жительных ионов (катионов). Остальная же часть молекул ста- новится при этом отрицательным ионом (анионом). В частности, кислоты распадаются на положи- тельные ионы водорода и отрицательные ионы кислотных .остатков. Например, молекула серной кис- лоты распадается согласно следующему уравнению: H2SO4 2Н++ SO? ”, (524) где H2SO4—молекула серной кислоты; 2Н+—два положительных однозарядных иона водорода; SO"—отрицательный двухзарядный ион кислотного остатка. Соли распадаются на положительные ионы металла и отрицательные ионы кислотного остатка. Например, молекула цинкового купороса распадается согласно уравнению ZnSO4 "t Zn++ + SO", (525) где ZnSO4—молекула цинкового купороса; Zn++— положительный двухзарядный ион цинка; SO"—отрицательный двухзарядный ион кислотного остатка. Основания распадаются на положительные ионы металлов и отрицательные ионы гидро- ксила. Например, молекула едкого натрия распадается со- гласно уравнению NaOH Na+ + ОН~, (526) • где NaOH— молекула едкого натрия; Na+— положительный однозарядный ион натрия; ОН“—отрицательный однозарядный ион гидроксила. Аналогично распадается и молекула едкого калия: КОН-*К+ + ОН-, (527) где КОН—молекула едкого калия; К+— положительный однозарядный ион калия; ОН- — отрицательный однозарядный ион гидроксила. Знаки во всех вышеприведенных формулах указывают, что электролитическая диссоциация — процесс о б р а т и м ы й, т. е. молекула сложного вещества не только рас- падается, но и образуется вновь из совокупности разноименных ионов. Допустим, что в слабый раствор серной кислоты (в электро- лит) погружены два одинаковых электрода, например две плз- 632
типовые пластинки. Один из них, называемый анодом, соединим с положительным полюсом источника электрической энергии, а другой, называемый катодом, соединим с отрицательным полю- сом (рис. 416). В результате этого в цепи возникает электрический ток, который внутри электролита будет ионным. Рассмотрим, в чем заключается физическая сущность этого тока и каково его химическое дей- ствие. Независимо от того, существует ли в данном электролите ток или нет, в нем про- исходит непрерывный процесс электролити- ческой диссоциации: H2SO4 2Н+ + SO7 - (528) Под действием сил электрического поля, имеющегося в пространстве между электро- дами, положительные ионы водорода 2Нн“ перемещаются в направлении действия сил поля, т. е. от положительно заряженного электрода (анода) к отрицательно заря- женному (катоду). Наоборот, отрицатель- ные ионы кислотного остатка SO4 пере- I' Анод Катод Й й — 7 7 “ Рис. 416. Ионный ток Б мещаются навстречу электрическому полю, в электролите т. е. от катода к аноду. Таким образом, яв- ление электрического тока в рассматриваемом электролите выра- жается в форме перемещения потоков разноименных ионов, дви- жущихся в диаметрально противоположных направлениях. Положительные ионы водорода 2Н+, войдя в соприкоснове- ние с катодом, отнимают от него электроны и, нейтрализовав- шись, осаждаются на нем в виде нейтральных молекул водорода. Отрицательные ионы кислотного остатка SO~~, войдя в со- прикосновение с анодом, отдают ему свой излишек электронов, превращаясь в нейтральную молекулу SO4. Однако последняя не может существовать в свободном состоянии и немедленно всту- пает в химическое взаимодействие с молекулой воды Н2О, в ре- зультате чего образуется молекула серной кислоты H2SO4 и осво- бождается эл'ектронейтральный 'атом кислорода: SO4 + Н2О = H2SO4 + О. (529) Выделяющиеся атомы кислорода соединяются, образуя моле- кулы кислорода О2, которые осаждаются на аноде. Таким образом, в р^ультате прохождения электрического тока через раствор серной кислоты на аноде выделился кисло- род, а на катоде — водород. Так как они не вступают в химиче- ское взаимодействие с платиновыми электродами, то они осажда- 633
ются на электродах в виде пузырьков газа водорода и кисло- рода. Явление выделения на электродах химических веществ при прохождении электрического тока через электролит называется электролизом. Количество химических веществ, отлагающихся на электро- дах при электролизе, определяется законом Фарадея: количество Веществ, отлагающихся на электродах, прямо пропорционально количеству электричества, прошедшего через электролит. Закон Фарадея математически определяется следующей фор- мулой: Q = KIt, (530) где Q— количество вещества, отложившегося на электроде, в миллиграммах; /—ток в амперах; t— время в секундах; К— коэффициент пропорциональности, называемый электро- химическим эквивалентом. Электрохимическим эквивалентом вещества называют количе- ство вещества (в миллиграммах), которое выделяется на элек- троде при-прохождении через электролит электрического заряда, равного одному кулону. В табл. 45 приведены электрохимические эквиваленты неко- торых наиболее употребительных веществ. Таблица 45 Наименование Электрохимический Наименование Электрохимический вещества эквивалент вещества эквивалент Водород 0,0104 Медь 0,329 Кислород .... 0,083 Цинк 0,338 Железо ... . . 0,291 Свинец ..... 1,072 Никель 0,309 Серебро 0,118 Пример 208. Определить, какое количество меди выделится из раствора медного купороса при прохождении через него тока I = 8 а в течение t = 20 мин., если электрохимический эквивалент меди К = 0,329. Решение. По формуле (530) находим Q = ки = 0,329 • 8 • 20 • 60 = 3158,4 мг = 3,1584 г. Явление электролиза широко применяется на практике для получения чистых металлов из растворов их солей, например, получения химически чистой меди (электролитической меди) из раствора медного купороса, серебра — из раствора азотно- серебряной соли (AgNCh) и т. д., а также для покрытия метал- лических предметов слоем металла (гальваностегия). 634
Если, например, в водный раствор никелевой соли погрузить в качестве анода никелевую пластинку, а в качестве катода какой-нибудь металлический предмет, например, ложку, чайник и т. п., то в процессе электролиза на этом металлическом пред- мете (на катоде) будет осаждаться слой никеля. Спустя не- сколько минут после начала опыта он покроется тонким слоем никеля, т. е. будет никелирован. Анод (никелевая пластинка) в процессе никелирования будет постепенно растворяться. По- добным же образом происходит процесс серебрения металличе- ских предметов. Электролитом здесь служит раствор какой- нибудь серебряной соли, например AgNO3 (азотнокислое се- ребро), а анодом — серебряная пластинка. При помощи электро- лиза можно также легко осуществить золочение, омеднение, хро- мирование металлических предметов. Явление электролиза широко используется при рафинирова- нии (очистке) металлов и гальванопластике, т. е. снятии точных копий с различных предметов, например с медалей, монет, ста- туэток и т. д. Приоритет открытия гальванопластики принадлежит извест- ному русскому ученому Б. С. Якоби. В предисловии к своей книге о применении гальванопластики он писал: «Гальванопластика принадлежит исключительно России. Здесь она открыта, здесь и развивалась». Б. С. Якоби был одним из тех русских ученых, которые на заре развития русской электротехники способствовали тому, что она во многих своих областях занимала ведущее место среди научных достижений в России и за границей. Помимо открытия гальванопластики, Б. С. Якоби изобрел электродвигатель и при- менил его на практике, построил первый пишущий и буквопеча- тающий телеграфные аппараты и разработал ряд других научных и технических проблем. Итак, в' процессе прохождения тока через рассматриваемый электролит (раствор серной кислоты) на электродах выделились разнородные вещества: на аноде — кислород, а на катоде — во- дород. Но, как показывает опыт, электрохимическая система, со- стоящая из кислорода, раствора серной кислоты и водорода 2H|H2SOJO2 представляет собой гальваническую пару, способную преобразо- вать свою химическую энергию в электрическую. Если отсоеди- нить от электродов этой гальванической пары источник электри- ческой энергии, то обнаружим разность потенциалов между ними. При этом анод (электрод, покрытый водородом) будет иметь по- ложительный потенциал, а катод (электрод, покрытый кислоро- дом) — отрицательный. В электролите же между .анодом и като- дом образуется электрическое поле. 635
Явление изменения потенциалов электродов в результате от- ложения на них веществ при электролизе носит название поля- ризации электродов. Электродвижущая сила, возникшая в результате поляризации электродов, называется поляризационной электродвижущей си- лой. Она численно равна разности потенциалов между анодом и катодом при разомкнутой внешней цепи. > Если поляризованные электроды замкнуть внешним проводни- ком, то под влиянием поляризационной электродвижущей силы в цени появится ток. Однако направление его в электролите бу- "дет противоположным тому, которое наблюдалось, когда в цепи действовал посторонний источник постоянного тока. Это приве- дет к тому, что на аноде, покрытом кислородом, теперь будет выделяться водород, а на катоде, покрытом водородом,— кисло- род. Кислород, соединяясь химически с водородом, будет обра- зовывать как на аноде/ так и на катоде молекулы воды. Элек- трический ток в цепи будет проходить до тех пор, пока не израс- ходуются активные вещества на электродах — кислород и во- дород. Если бы мы пожелали возобновить работу данной электро- химической системы, то необходимо было бы вновь подключить электроды к источнику постоянного тока, путем электролиза от- ложить на них активные вещества (кислород и водород), затем разрядить систему на внешнее сопротивление и т. д. Таким обра- зом, в рассматриваемой электрохимической системе можно осу- ществить ряд обратимых электрохимических процессов. При под- ключении ее к источнику постоянного тока в ней происходит пре- образование электрической энергии в химическую энергию актив- ных веществ, отлагающихся на электродах; наоборот, при замы- кании этой системы на внешнее сопротивление (на потребитель) происходит преобразование химической энергии активных ве- ществ в электрическую. Рассмотренная нами электрохимическая система является про- стейшим электрическим аккумулятором, т. е. электрохимической системой, способной накапливать (аккумулировать) химическую энергию за счет электрической и вновь преобразовывать ее в электрическую, отдавая ее потребителю. Количество электричества, измеряемое обычно ампер-часами, которое может отдать аккумулятор при своем нормальном раз- ряде, называется емкостью аккумулятора. Емкость аккумулятора зависит от его конструкции и от продолжительности времени его разряда. Чем меньше разрядный ток, тем больше ампер-часов отдает аккумулятор. Отдачей аккумулятора по количеству элек- тричества называется отношение числа ампер-часов, полу- ченных от аккумулятора при разряде, к количеству электриче- ства, затраченному при его заряде. 636
Отдачей аккумулятора по энергии, т. е. коэффи- циентом полезного действия его, называется отношение энергии, полученной от аккумулятора при его разряде, к энергии, затра- ченной при заряде. Рассмотренный нами выше аккумулятор обладает весьма ма- лыми емкостью, отдачей по количеству электричества и коэффи- циентом полезного действия, а поэтому практического значения не имеет. На практике обычно применяются аккумуляторы, у ко- торых в процессе заряда образуется значительное количество активных веществ на электродах, т. е. имеется большая ем- кость. Такие аккумуляторы могут работать при относительно вы- соком коэффициенте полезного действия. Применяемые в современной практике аккумуляторы подраз- деляются на две основные категории — щелочные и кислотные. В свою очередь щелочные аккумуляторы подразделяются на кад- миево-никелевые и железо-никелевые. § 189. ЩЕЛОЧНЫЕ АККУМУЛЯТОРЫ I. Кадмиево-никелевые аккумуляторы 1. Устройство аккумулятора. На рис. 417 приведен внешний вид кадмиево-никелевого аккумулятора, а на рис. 418 показано его внутреннее устройство. Сосуд аккумулятора 1 изготовлен из листовой стали. Внутри сосуда помещены положительные 2 и от- рицательные 3 пластины, разделенные эбонитовыми изоляцион- ными палочками 6, Боковая изоляция 5 предохраняет пластины от соприкосновения со стенками сосуда. Пробка 4 прикрывает отверстие в аккумуляторе, служащее для заливки его электро- литом и для отвода газов. В качестве электролита в кадмиево-никелевом аккумуляторе применяется водный раствор едкого калия КОН или едкого на- трия NaOH. Для улучшения работы аккумулятора в электролит иногда добавляют едкого лития LiOH. Однако такой составной электролит несколько увеличивает внутреннее сопротивление аккумулятора и делает его менее холодостойким. В табл. 46 приведены электролиты, применяемые в кадмиево- никелевых аккумуляторах, с указанием температурных условий их эксплуатации. Положительные и отрицательные пластины кадмиево-никеле- вого аккумулятора одинаковы по своей конструкции (рис. 419). Они состоят из отдельных стальных никелированных рам, в кото- рые заделаны в виде ячеек пакетики из перфорированной стали с активной массой. В качестве активной массы положительных пластин приме- няется смесь из гидрата закиси никеля Ni(OH)2, гидрата окиси никеля Ni(OH)3 и некоторого количества графита (до 20%),уве- личивающего электропроводимость массы. 637
Рис. 417. Внешний вид Рис. 418. Внутреннее устройство щелочного аккумуля- щелочного аккумулятора тора Рис. 419. Пластины щелочного аккумулятора 638
Таблица 46 Электролиты, применяемые в кадмиево-никелевых аккумуляторах Предельные температуры при эксплуатации Электролит Удельный вес электролита Примечание От —40 до —15° Ц КОН 1,26—1,3 Зимний электролит От —15 до +259 Ц кон 1,19—1,21 От —15 до +40° Ц КОН 4- LiOH 1,19—1,21 От +15 до +35° Ц NaOH 1,17—1,19 Летний электролит От +40 до +60° Ц NaOH 4- LiOH 1,17—1,19 То же В качестве активной массы отрицательных пластин приме- няется смесь губчатого кадмия Cd с мия и 25Н-20% железа). Желе- зо увеличивает электропроводи- мость массы и предохраняет ее от спекания. 2. Электрохимические процес- сы при разряде аккумулятора. Допустим, что кадмиево-никеле- вый аккумулятор заряжен. В этом случае он представляет собой следующую электрическую си- стему: анод электролит катод (+)Ni(OH)3|KOH|Cd (-)> т. е. активным веществом ано- да (+) является гидрат окиси ни- келя Ni(OH)3, активной массой катода (—)—губчатый (пори- стый) кадмий '• Cd и электроли- том — раствор едкого калия КОН. железом Fe (75-4-80% кад- Рис. 420. Принципиальная схема разряда кадмиево-никелевого ак- кумулятора В электролите аккумулятора происходит непрерывный процесс электролитической диссоциации молекул KOH-tK+ + OH-. (531) Присоединим к зажимам аккумулятора внешнее сопротивле- ние (потребитель). В цепи возникнет электрический ток, и акку- мулятор начнет разряжаться. На рис. 420 показана принципиальная схема разряда кад- миево-никелевого аккумулятора. Положительные ионы калия К+ перемещаются в направлении электрического поля, т. е. от отрицательного электрода (катода) к положительному .(аноду). Отрицательные ионы гидроксила ОН- перемещаются навстречу электрическому полю, • т. е. от -анода к катоду. 639
Процесс на отрицательном электроде. С от- рицательного электрода (катода) электроны уходят во внешнюю цепь. Отрицательные ионы гидроксила ОН" отдают свои отрица- тельные заряды катоду, и в результате этого там возникает хи- мическая реакция, которую в молекулярном виде запишем так: Cd + 2OH = Cd (ОН)2, т. е. кадмий Cd, соединяясь химически с гидроксилом 2ОН, обра- зует совместно с ним гидрат окиси кадмия Cd (ОН) 2. Процесс на положительном электроде. Из внешней цепи на анод поступают свободные электроны, а из электролита — положительные ионы калия К+, которые отдают аноду свои положительные заряды. В результате этого на аноде возникает химическая реакция, которую в молекулярном виде запишем так: 2Ni (ОН)3 + 2К = 2Ni (ОН)2 + 2КОН, т. е. при разряде аккумулятора на аноде расходуется гидрат окиси никеля 2Ni(OH)3, а также калий 2К и одновременно с этим образуются гидрат закиси никеля 2Ni(OH)2 и едкий калий КОН. Следовательно, уравнение токообразующего процесса при раз- ряде кадмиево-никелевого аккумулятора можно записать в мо- лекулярном виде так: анод катод анод катод 2Ni (ОН)3 + 2КОН + Cd=2Ni (ОН)2 + 2КОН + Cd (ОН)2, (532) т. е. в результате разряда кадмиево-никелевого аккумулятора активная масса его отрицательного электрода — кадмий — пре- образуется в гидрат окиси кадмия Cd (ОН) 2, а активная масса положительного электрода — гидрат окиси никеля Ni (ОН) з — в гидрат закиси никеля. Концентрация электролита — едкого калия КОН — при раз- ряде аккумулятора не изменяется, так как сколько едкого калия расходуется вблизи катода, столько же его возникает вблизи анода. 3. Электрохимические процессы при заряде аккумулятора. При заряде аккумулятора его анод присоединяется к положитель- ному полюсу источника электрической энергии, а катод — к от- рицательному полюсу. В начале заряда аккумулятор представляет собой электрохи- мическую систему следующего состава: анод электролит катод Ni (ОН), | КОН | Cd (ОН)2. При подключении аккумулятора к источнику постоянного тока в цепи возникает электрический ток. Теперь положительные ионы 640
•калия К+ направляются внутри аккумулятора от анода к ка- тоду, а отрицательные ионы гидроксила ОН“—от катода к аноду. На рис. 421 показана принципиальная схема заряда кадмие- во-никелевого аккумулятора. Процесс на положительном электроде. Сво- бодные электроны уходят с ано- да, и одновременно с этим отри- цательные ионы гидроксила ОН“ попадают на анод и отдают ему свои отрицательные заряды. В ре- зультате на аноде возникает хи- мическая реакция, которая в мо- лекулярном виде может быть за- писана так: 2Ni (ОН)2 + 2 (ОН) = 2Ni (ОН)3, т. е. гидрат закиси никеля 2Ni(OH)2, соединяясь с гидрокси- лом 2(ОН), совместно образует гидрат окиси никеля 2Ni(OH)3. Процесс на отрица- тельном электроде. Сво- бодные электроны поступают от источника электрической энер- гии на отрицательный электрод, в результате чего там возникает Рис. 421. Принципиальная схема заряда кадмиево-никелевого ак- кумулятора химическая реакция, которую в молекулярном виде можно запи- сать так: Cd (ОН), -> Cd + 2 (ОН), т. е. гидрат окиси кадмия Cd (ОН) 2 в результате химической реакции распадается на губчатый кадмий Cd и гидроксил 2(ОН). Последний же, вступая в химическое взаимодействие с калием, образует молекулы едкого калия: 2 (ОН) + 2К = 2КОН. Следовательно, уравнение токообразующего процесса при за- ряде кадмиево-никелевого аккумулятора можно записать в моле- кулярном виде следующим образом: катод анод катод анод Cd (ОН)2 4- 2КОН 4- 2Ni (ОН)2 = Cd 4- 2КОН 4- 2Ni (ОН)3, т. е. в результате заряда кадмиево-никелевого аккумулятора на его катоде восстанавливается губчатый кадмий Cd, а на аноде — гидрат окиси никеля 2Ni(OH)a. 41—1377 641
Концентрация раствора едкого калия КОН сохраняется неиз- менной по тем же причинам, что и при разряде аккумулятора. Таким образом, заряд и разряд кадмиево-никелевого аккуму- лятора — обратимая электрохимическая реакция, которую можно записать в молекулярном виде следующим образом: 2Ni (ОН)3 + Cd -t 2Ni (ОН)2 + Cd (ОН)2. (533) заряд 4. Напряжение при заряде и разряде аккумулятора. На рис. 422 приведены кривые изменения напряжения при заряде и Рис. 422. Кривые изменения напряжения кад- миево-никелевого аккумулятора при его за- ряде и разряде разряде кадмиево-нике- левого аккумулятора. При заряде напряже- ние на зажимах та- кого аккумулятора вна- чале быстро растет, до- стигая 1,4 в. Затем в течение нескольких ча- сов оно относительно медленно поднимается до 1,5 в. После этого оно вновь относитель- но быстро достигает 1,65 в и, наконец, отно- сительно медленно ра- стет до 1,75—1,8 в. После того как акку- мулятор будет заряжен и отключен от источника электрической энергии, напряжение на его зажимах упадет с 1,8 до 1,5 в. При установке аккумулятора на разряд напряжение его вна- чале относительно быстро падает, достигая 1,3 в,ха затем уже медленно и равномерно понижается до 1,1 в. Если продолжить разряд аккумулятора дальше, то его напряжение будет умень- шаться, круто падая вниз. Среднее разрядное напряжение кадмиево-никелевого аккуму- лятора принимается равным 1,25 в. 5. Внутреннее сопротивление аккумулятора. Внутреннее со- противление кадмиево-никелевого аккумулятора относительно мало и практически измеряется сотыми или тысячными долями ома. По мере разряда аккумулятора его внутреннее сопротивление растет. Величину внутреннего сопротивления кадмиево-никелевого аккумулятора можно практически определить по следующей эмпирической формуле: 0,3 Г° — Q ’ (534) 642
где r0—внутреннее сопротивление аккумулятора в омах; Q— емкость аккумулятора в ампер-часах. 6. Отдача аккумулятора по емкости и по энергии. Отдача кадмиево-никелевого аккумулятора по его емкости (количеству электричества) колеблется в пределах 65—70%. Несколько ниже отдача аккумулятора по энергии. Она колеб- лется в пределах 50—55%, так как потеря энергии сильно зави- сит от разного рода необратимых процессов, например в элек- тролите и на электродах. Если разряд аккумулятора вести предельным током, то в со- ответствии с этим отдача аккумулятора как по емкости, так и по энергии будет уменьшена из-за более интенсивных химических реакций. 7. Типы кадмиево-никелевых аккумуляторов, применяемых на практике. В настоящее время в СССР выпускаются самые разно- образные типы кадмиево-никелевых аккумуляторов. В табл. 47 приведены типы кадмиево-никелевых аккумулято- ров, изготовляемых в СССР, с указанием их электрических дан- ных. Таблица 47 Основные данные кадмиево-никелевых аккумуляторов Тип аккумуля- тора Номинальная, емкость, а-ч Номинальный зарядный ток, а Номинальный разрядный ток, а Вес, кг без электро- лита с электро- литом АКН-2,25 2,25 0,56 0,28 0,28 0,33 НКН-10 10 2,5 1,25 0,60 0,74 НКН-22 22 5,5 2,75 1,35 1,67 НКН-45 45 11,25 5,65 2,18 2,72 НКН-60 60 15,0 25,0 7,5 3,70 4,60 НКН-100 100 12,5 5,10 6,50 Примечание. Первая буква условного обозначения аккумулятора (элемента) указывает практическое назначение его, а именно: А — анодный (для питания анодных цепей в радиотехнике), Н—накальный (для питания цепей накала радиоламп). Вторая и третья буквы указывают тип аккумуля- тора: КН — кадмиево-никелевый. Число, стоящее правее букв, указывает емкость аккумулятора в ампер-часах. Следовательно, если аккумулятор имеет условное обозначение НКН-100, то это означает, что он накальный (Н), кадмиево-никелевый (КН) и имеет емкость 100 ампер-часов. II. Железо-никелевые аккумуляторы Наряду с кадмиево-никелевыми аккумуляторами в практике применяются и железо-никелевые аккумуляторы (ЖН). По конструкции они мало отличаются от кадмиево-никелевых. В отличие от последних они имеют отрицательных пластин на одну больше, чем положительных. Это объясняется тем, что для 41* 643
получения соответствующей емкости аккумулятора необходимо взять несколько большее количество активной массы отрица- тельных пластин. Обычно отрицательные пластины железо-нике- левого аккумулятора соединены с корпусом, а вывод положи- тельного электрода тщательно изолирован от корпуса. В качестве электролита для железо-никелевых аккумуляторов применяется раствор едкого калия КОН или едкого натрия NaOH, в некоторых случаях с примесью едкого лития LiOH. Положительные пластины железо-никелевых аккумуляторов такие же, как у кадмиево-никелевых аккумуляторов. Следова- тельно, активной массой их в основном является гидрат окиси никеля Ni(OH)a. Активная масса отрицательных пластин железо-никелевых аккумуляторов отличается от активной массы кадмиеро-никеле- вых аккумуляторов. Она в основном состоит из смеси губчатого железа Fe с его окислами и небольшого количества процентов окиси ртути. Внутреннее сопротивление железо-никелевых аккумуляторов несколько больше, чем кадмиево-никелевых, и практически может быть определено по эмпирической формуле (535) где Q — емкость аккумулятора в ампер-часах; /?0 — внутреннее сопротивление аккумулятора в омах. Напряжение при разряде железо-никелевых аккумуляторов несколько ниже, чем у кадмиево-никелевых, из-за относительно большого внутреннего сопротивления, а напряжение при заряде выше. В конце заряда оно достигает примерно 1,8—1,9 в. Емкость железо-никелевых аккумуляторов при увеличении разрядного тока изменяется относительно мало. Отдача по количеству электричества равна примерно 65%', а по энергии — 45—50%. Железо-никелевые аккумуляторы обла- дают такими характеристиками, что во многих случаях практики могут вполне заменить кадмиево-никелевые аккумуляторы, на- пример в пределах температур от —10° Ц до -|-30о Ц. В табл. 48 приведены основные данные железо-никелевых аккумуляторов, изготовляемых нашей электропромышленностью. Таблица 48 Тип аккумуля- тора Номинальная емкость, а~ч Номинальный зарядный ток, а Номинальный разрядный ток, а Вес, кг без электро- лита с электро- литом ЖН-22 22 5,5 2,75 1,4 1,73 ЖН-45 45 11,25 5,65 2,4 2,85 ЖН-60 60 15,00 7,5 4,0 4,78 ЖН-100 100 25,00 12,5 5,6 6,80 644
Примечание. Буквы ЖН обозначают железо-никелевые. Число, стоящее после букв ЖН, указывает емкость аккумулятора в ампер-часах. Неисправности железо-никелевых аккумуляторов в основном те. же, что и у кадмиево-никелевых. Правила ухода и эксплуата- ции также мало отличаются от правил для кадмиево-никелевых аккумуляторов. На практике применяется много разнообразных типов железо- никелевых аккумуляторов, изготовляемых электропромышленно- стью СССР. В частности, в технике связи используются аккуму- ляторы ЖН-22, ЖН-45, ЖН-60 и ЖН-100. § 190. ЭКСПЛУАТАЦИЯ ЩЕЛОЧНЫХ АККУМУЛЯТОРОВ При составлении раствора электролита необходимо брать ди- стиллированную воду в количестве, потребном для получения заданной плотности электролита. В табл. 49 указано, какое количество едкого калия содер- жится в одном литре электролита при заданной его плотности. Таблица 49 Содержание едкого калия в одном литре электролита при заданной его плотности Плотность электролита Содержание едкого калия на 1 л электролита, г Плотность электролита Содержание едкого калия на 1 л электролита, г 1,18 242 1,25 335 1,19 255 1,26 349 1.20 269 1,27 362 li21 282 1,28 375 1,22 295 1,29 390 1,23 308 1,30 403 1,24 323 В табл. 50 указано, какое количество едкого натрия необхо- димо взять на один литр дистиллированной воды, чтобы полу- чить необходимую плотность электролита. Таблица 50 Содержание едкого натрия в одном литре электролита Плотность электролита Содержание едкого натрия на 1 л электролита, г 1,17 176,2 1,18 188,8 1,19 201,2 645
В табл. 51 указано, какое количество едкого натрия содер- жится в одном литре концентрированного раствора едкого на- трия различных плотностей. Таблица 51 Содержание едкого натрия в его^концентрированных растворах Плотность концентриро- ванного раствора Содержание едкого натрия в 1 л концентрированного раствора, г Плотность концентриро- ванного раствора Содержание едкого натрия в 1 л концентрированного раствора, г 1,241 267,4 1,383 484,1 1,252 281,7 1,397 507,9 1,263 296,8 1,410 530,9 1,274 311,9 1,424 556,2 1,285 327,7 1,438 582,0 1,297 344,7 1,453 610,6 1,308 361,7 1,468 639,8 1,320 380,6 1,483 669,7 1,332 399,6 1,498 700,0 1,345 419,6 1,514 732,9 1,357 441,0 1,530 766,5 1,370 462,1 — — Приготовлять электролиты для щелочных аккумуляторов ре- комендуется в стеклянной или керамической посуде. Запрещается приготовлять раствор в медной, свинцовой, луженой посуде или посуде, содержащей остатки серной кислоты. Аккумулятор можно заливать только остывшим электроли- том, температура которого не превышает 25° Ц. Уровень электролита в аккумуляторе должен быть выше верх- них краев пластин аккумулятора-примерно на 10 мм. По мере расхода электролита в аккумуляторе надо своевременно доливать аккумулятор дистиллированной водой или слабым раствором едкого калия. Запрещается держать аккумуляторы в помещениях, где име- ются кислотные исггарения, так как кислоты разрушают актив- ную массу пластин. Необходимо внимательно следить за чистотой' электролита, так как при выпадении осадков на дно сосуда аккумулятора пластины его могут закоротиться, вследствие чего напряжение на его зажимах и емкость будут значительно снижены. Слишком высокий уровень электролита в аккумуляторе и про- сачивание его через пробку или выводы приводит к обильному выделению ползучих солей. Поэтому надо периодически очищать аккумулятор от пыли и грязи, а металлические неокрашенные наружные части смазывать техническим вазелином. Рекомендуется аккуратно обращаться с электролитом, не до- пуская попадания едкой щелочи на руки или одежду, а в случае 646
обнаружения этого следует смыть едкую щелочь 2%’ раствором борной кислоты, а затем промыть чистой водой. Заряд аккумулятора производят от источника электрической энергии с постоянным напряжением или от сети переменного тока, но в последнем случае с обязательным использованием вы- прямителя или преобразователя тока. Перед началом заряда аккумулятора необходимо вынуть пробки из отверстий в его крышке, чтобы газы вышли из бака аккумулятора, а затем проверить уровень электролита в баке. Заряжать аккумулятор необходимо нормальным током, ука- занным в паспорте аккумулятора. Величину нормального тока заряда аккумулятора можно • определить по эмпирической фор- муле /зар = -Т’ <536) т. е. нормальный зарядный ток щелочного аккумулятора равен емкости аккумулятора, разделенной на 4. Время нормального заряда аккумулятора — не менее 6 часов. При необходимости форсирования заряда это время можно со- кратить до 4 часов: первые 2,5 часа заряжать аккумулятор двой- ным нормальным током, а остальные 1,5 часа — нормальным током. Заряд аккумулятора необходимо всякий раз доводить до конца, помня, что часто повторяющаяся недозарядка аккумулятора приводит к резкому снижению его емкости или даже порче его. Когда напряжение на зажимах аккумулятора достигнет 1,8 в, следует его продержать под током еще минут 30, а затем снять с заряда. О конце заряда аккумулятора следует судить по напряжению на его зажимах, но никоим образом не по так называемому «ки- пению» электролита. Так как при заряде щелочного аккумулятора наб’людается обильное выделение газов — кислорода и водорода (результат разложения воды4в электролите), то необходимо следить за тем, чтобы отверстия в пробках аккумулятора, предназначенные для выхода газов, не были засорены, иначе вследствие скопления га- зов внутри сосуда аккумулятора стенки его могут раздуться. За- винчивать пробки аккумулятора надо не раньше чем через 6 ча- сов после окончания заряда, чтобы газы полностью улетучились из аккумулятора. Разряжать аккумулятор следует током не выше нормального, обычно указываемого в паспорте аккумулятора. Практически можно считать, что нормальный разрядный ток щелочного акку- мулятора не должен превышать половины нормального 'заряд- ного тока его: т ____ ^зар Q *разр 2 8 ’ (537) 647
Рекомендуется разряд щелочного аккумулятора закончить, когда напряжение на его зажимах упадет до 1,1 в. В крайних случаях, когда нельзя своевременно прекратить разряд аккуму- лятора, допускается разряд его до 0,8 в. Как при заряде, так и при разряде щелочного аккумулятора необходимо следить за тем, чтобы электролит и активная масса пластин не перегрелись, так как это резко снижает емкость акку- мулятора. Например, если температура электролита аккумуля- тора будет доведена до 45° Ц, то номинальная емкость аккуму- лятора уменьшится примерно на 50%. Эксплуатация .щелочного аккумулятора при низких темпера- турах также ведет к резкому снижению емкости аккумулятора. В табл. 52 указано влияние низких температур на отдачу акку- мулятора по емкости. Таблица 52 Отдача кадмиево-никелевых аккумуляторов по емкости при низких температурах Температура, °Ц Отдача аккумулятора по емкости (в процентах) для случая заряда при нор- мальной температуре и раз- ряда при данной низкой температуре для случая заряда и разряда при данной низкой температуре —10 85 80 —20 75 70 —30 50-4-60 45 —40 20-7-30 5 Примечание. Нормальной температурой заряда аккумулятора счи- тается 25° Ц. § 191. КИСЛОТНЫЕ (СВИНЦОВЫЕ) АККУМУЛЯТОРЫ 1. Устройство аккумуляторов. В качестве сосудов кислотных аккумуляторов применяют эбонитовые, стеклянные или деревян- ные, выложенные свинцом, баки. Аккумуляторы с эбонитовыми баками (рис, 423) используются главным образом в переносных установках, со стеклянными баками (рис. 424) — в переносных и стационарных установках и с деревянными баками — в мощных стационарных установках. На рис. 423 показано внутреннее устройство кислотного акку- мулятора в эбонитовом баке. Здесь знаком плюс (-J-) отмечены положительные электроды (пластины), знаком минус (—)—от- рицательные. Деревянные и эбонитовые прокладки служат для изоляции пластин друг от друга. Пробка прикрывает отверстие, через которое заливают электролит в сосуд аккумулятора и выходят газы при работе аккумулятора. 648
Электролитом аккумулятора служит 24—33% раствор хими- чески чистой серной кислоты. Дополнительные пластины свинцовых аккумуляторов приме- няются трех видов: поверхностные, панцирные и намазные (ре- шетчатые) . Поверхностные положительные пластины представляют собой отлитые из чистого свинца пластины ребристого строения, так Рис. 423. Свинцовый аккумулятор переносный что их активная поверхность относительно велика. Путем ряда повторных зарядов и разрядов добиваются того, что на всей ак- тивной поверхности пластин отлагается активная масса — пере- кись свинца РЬО2 толщиной в несколько долей миллиметра. До- стоинство поверхностных пластин заключается в том, что они механически прочны, их активная масса не вываливается из бо- розд пластин. Недостатки их — малая удельная емкость (емкость аккумулятора на единицу веса пластин) и длительность формов- ки, связанная с относительно большим расходом электрической 649
Внутрь трубок вложены стержни Электролит свободно проникает энергии. На рис. 425 приведен образец одного из типов поверх- ностных пластин. Панцирные пластины состоят из эбонитовых трубок с полу- кольцевыми прорезями, соединенных в одну общую пластину. ~ из свинцово-сурьмяного сплава, к активной массе в трубках через прорези, а сама активная масса прочно удерживается в трубках и не вываливается при механических толчках. Панцирные пластины имеют значительно большую удельную емкость по сравнению с по- верхностными. Намазные (решетчатые) по- ложительные пластины состоят из решетки, сделанной из свин- Illllllllllllllllllllllllllllllllllllllllllllllllllllllllllllll lllllllllllllllllllllllllllllllllllllllllllllllllllllllllllllll lllllllllllllllllllllllllllllllllllllllllllllllllllllllllllllll lllllllllllllllllllllllllllllllllllllllllllllllllllllllllllllll llllllllllllllllllllllllllllllillllllllllllllllllllllllllllllll lllllllllllllllllllllllllllllllllllllllllllllllllllllllllllllll lllllllllllllllllllllllllllllllllllllllllllllllllllllllllllllll llllllllllllllllllillllllllllllllllllllllllllllllllllllllllllll lllllllllllllllllllllllllllllllllllllllllllllllll!!!lllllllllll lllllilllllllllllllllllllllllllllllllllllllllllllllllllllllllll lllllllllllllllllllllllllllllllllllllllllllllllllllllllllllllllll Рис. 425. Положительная пластина свинцового аккумулятора Рис. 424. Свинцовый аккумуля- тор стационарный цово-сурьмяного сплава. Ячейки этой решетки заполнены актив- ной массой в виде пасты, сделанной из окислов свинца на рас- творе серной кислоты. Затвердевшая паста после просушки прочно удерживается в ячейках. Достоинство намазных (решет- чатых) пластин заключается в том, что они дают относительно большую удельную емкость аккумулятора. Недостаток их — относительно малая механическая прочность, так как активная масса может вываливаться из ячеек при механических толчках или продолжительной эксплуатации аккумулятора. Отрицательные пластины свинцовых аккумуляторов изготов- ляют двух типов: коробчатые и намазные. * Коробчатые пластины представляют собой тонкую свинцовую коробку, состоящую из отдельных ячеек, внутрь которых вма- 650
Зана активная масса (паста). На рис. 426 представлен внешний вид коробчатой пластины. Намазные отрицательные пластины не отличаются по своей конструкции от намазных положительных пластин. 2. Электрохимические процессы при разряде аккумулятора. Свежезаряженный кислотный акку- , мулятор представляет собой сле- дующую электрохимическую си- стему: анод электролит катод (+)PbO2|H2SO4|Pb (-), т. е. положительный электрод (анод) имеет в качестве активной массы перекись свинца РЬО2, отрицатель- ный электрод — губчатый свинец РЬ; электролитом служит раствор серной кислоты H2SO4. В электро- лите происходит обратимый процесс электролитической диссоциации мо- Рис. 426. Отрицательная пла- стина свинцового, аккумуля- тора лекул H2SO4 2Н+ + sor При подключении к аккумулятору приемника электрической энергии в цепи возникает электрический ток. Во внешней цепи это будет ток проводимости, осуществляемый свободными элек- тронами, перемещающимися от отрицательного электрода (ка- тода) по внешней цепи к аноду. Внутри аккумулятора это будет ионный ток, осуществляемый движением ионов. В частности, положительные ионы водорода Н+ будут перемещаться в на- правлении электрического поля, т. е. от отрицательного элек- трода (катода) к положительному (аноду). Отрицательные ионы кислотного остатка SO4“- будут перемещаться навстречу элек- трическому полю, т. е. от анода к катоду. На рис. 427 показана принципиальная схема разряда кислот- ного аккумулятора. Процесс на положительном электроде. На по- ложительный электрод при разряде аккумулятора поступают свободные электроны из внешней цепи и одновременно с этим положительные ионы водорода Н+ из электролита. В результате этого там в присутствии серной кислоты H2SO4 возникает хими- ческая реакция, уравнение которой в молекулярном виде можно записать так: РЬО2 + 2Н + H2SO4 = PbSO4 4- 2Н2О, т. е. на положительном электроде во время разряда конденса- тора расходуется перекись свинца РЬО2, водород 2Н и серная 651
кислота H2SO4; одновременно на ней образуется сернокислый свинец (сульфат PbSO4), а в растворе у электрода выделяется вода Н2О. Процесс на отрицательном электроде. Сво- бодные электроны уходят с отрицательного электрода во внеш- нюю цепь, одновременно с этим на электрод поступают отрица- тельные ионы кислотного остатка SO~ В результате на отрица- тельном электроде возникает химическая реакция, уравнение ко- торой в молекулярной форме можно записать так: Pb + SO4 = PbSO4, т. е. губчатый свинец РЬ, соединяясь с кислотным остатком SO4, образует сернокислый свинец PbSO4. Рис. 427. Принципиальная схема разряда свинцового аккумулятора Рис. 428. Принципиальная схе- ма заряда свинцового аккуму* лятора Следовательно, уравнение токообразующего процесса при разряде кислотного аккумулятора можно записать в молекуляр- ной форме следующим образом: анод катод анод катод (+) РЬО2 + 2H2SO4 + Pb PbSO4 + 2Н2О + PbSO4. (538) Отсюда мы видим, что в процессе разряда аккумулятора его электроды, ранее имевшие разнородные вещества на своих по- верхностях — перекись свинца РЬОг н губчатый свинец РЬ, в конце разряда покрываются однородным веществом — серно- кислым свинцом PbSO4. Но как только исчезает разнородность веществ электродов аккумулятора, исчезает и его способность преобразовывать химическую энергию в электрическую. 652
По мере разряда свинцового аккумулятора в нем расхо- дуется серная кислота и образуется вода. Следовательно, плот- ность раствора серной кислоты к концу разряда аккумулятора уменьшается. По плотности раствора серной кислоты в аккуму- ляторе можно судить о степени его разряда. 3. Электрохимические процессы при заряде аккумулятора. Чтобы восстановить аккумулятор как источник электрической энергии, его необходимо зарядить. При заряде аккумулятора положительные ионы водорода 2Н+ перемещаются от анода1 к катоду, а отрицательные ионы кислотного, остатка — от катода к аноду. На рис. 428 показана принципиальная схема заряда кислот* ного аккумулятора. z Процесс на положительном электроде. С по- ложительного электрода убывают в сторону источника электри- ческой энергии свободные электроны и одновременно с этим из электролита на этот электрод прибывают отрицательные ионы кислотного остатка В результате на положительном элек- троде с участием воды Н2О происходит химическая реакция, уравнение которой в молекулярном виде можно прёдставить так: PbSO4 + 2Н2О + SO4 = РЬО2 + 2H2SO4, т. е. при заряде кислотного аккумулятора на положительном электроде расходуются сернокислый свинец PbSO4, вода Н2О и кислотный остаток SO4 и образуется перекись свинца РЬОг, а в электролите образуется серная кислота H2SO4. Процесс на отрицательном электроде. Свобод- ные электроны поступают на отрицательный электрод при за- ряде, одновременно с этим туда же поступают положительные ионы водорода 2Н+. В результате там возникает химическая реакция, уравнение которой в молекулярной форме можно за- писать так: . PbSO4-^H2 = Pb + H2SO4, т. е. на отрицательном электроде при заряде кислотного акку- мулятора расходуются сернокислый свинец PbSO4 и водород Нг и возникает губчатый свинец РЬ, а в электролите образуется серная кислота H2SO4. Следовательно, уравнение токообразующего процесса при за- ряде кислотного аккумулятора можно записать в молекулярной форме следующим образом: анод катод анод катод PbSO4 + 2Н2О + PbSO4 = РЬО2 + 2H2SO4 Ч- РЬ. (539) Отсюда мы видим, что в процессе заряда кислотного акку- мулятора его электроды, ранее имевшие однородное вещество 653
(сернокислый свинец PbSO4), в конце заряда покрываются раз- нородными веществами: перекисью свинца РЬО2 и губчатым свинцом РЬ. Следовательно, аккумулятор вновь приобретает свойства химического источника электрической энергии. По мере заряда аккумулятора в нем 'расходуется вода и об- разуется серная кислота, следовательно, плотность раствора сер- ной кислоты непрерывно растет. К концу заряда, когда сернокислый свинец на аноде почти полностью превращается в перекись свинца, а на катоде — в губ- чатый свинец, начинается быстро нарастающее выделение кис- лорода и водорода из аккумулятора. Бурное выделение пузырь- ков газа — водорода и кислорода с поверхности электролита внешне напоминает «кипение» жидкости. Сильное «кипение» аккумулятора—один из основных признаков того, что аккуму- лятор зарядился. Обычно для полного заряда аккумулятора не- обходимо дать ему «кипеть» не менее одного — двух часов. Чтобы снизить степень кипения аккумулятора, рекомендуется в конце заряда проводить его менее интенсивно, снизив несколь- ко величину зарядного тока. Электрические данные свинцового аккумулятора 1. Напряжение U на зажимах аккумулятора зависит от сте- пени его заряженности и от нагрузки. При заряде аккумулятора напряжение на его зажимах U всегда больше электродвижущей силы Е на величину падения напряжения Uq = IrQ в нем, т. е. U = E + IrQ, (540) и Тис. 429. Кривые изменения напряжения свин- цового аккумулятора при заряде (а) и раз- ряде (0 где г0— внутреннее со- противление ак- кумулятора; /—величина за- рядного тока. На рис. 429 показа- на характерная кри- вая а заряда кислот- ного аккумулятора. Здесь по горизонталь- ной оси отложено вре- мя заряда аккумулято- ра в часах, а по верти- кальной оси — величи- на напряжения на его зажимах в вольтах. Из графика видно, что «54
в начале заряда напряжение от 1,8 до 2,1 в растет очень бы- стро. Затем кривая зарядного напряжения становится более по- логой и на протяжении примерно 4—6 часов медленно увеличи- вается до 2,3 в. И потом, когда аккумулятор почти зарядился полностью, напряжение на его зажимах вновь резко растет до величины 2,7—2,8 в. В эти моменты крутого подъема напряже- ния на зажимах аккумулятора последний начинает бурно «ки- петь», т. е. интенсивно выделять с поверхности электролита газы — кислород и водород. Если прекратить заряд аккумулятора, отключив его от источ- ника электрической энергии, то напряжение на его зажимах тот- час же упадет примерно до 2,2—2,3 в, .При разряде аккумулятора напряжение на его зажимах меньше электродвижущей силы аккумулятора Е на величину па- дения напряжения Uq внутри него: U=E — IrQ) где I — величина разрядного тока. В начале разряда напряжение на зажимах аккумулятора от- носительно быстро падает до 1,95 в, затем в течение несколь- ких часов плавно уменьшается до 1,85 в (на рис. 429 кривая б) и после этого резко падает, стремясь к нулю. В зависимости от условий аккумуляторы иногда разряжаются в течение относи- тельно продолжительного времени. Например, аккумуляторные батареи, питающие телеграфные цепи, обычно разряжаются в течение нескольких суток относительно 1^алым разрядным то- ком. Разряжать аккумулятор ниже напряжения 1,85 вне рекомен- дуется, так как он может испортиться. Среднее разрядное напряжение свинцового аккумулятора для практических расчетов принимается равным U = 2 в. Указанный режим изменения зарядного и разрядного напря- жений на зажимах свинцового аккумулятора относится к слу- чаю, когда аккумулятор заряжается и разряжается нормальным током, свойственным данному типу аккумулятора. При больших токах химические процессы в аккумуляторах протекают более бурно, и в соответствии с этим графики зарядного и разрядного напряжений могут резко изменяться. 2. Величина электродвижущей силы аккумулятора зависит от состояния активных веществ его пластин и от концентрации раствора серной кислоты. Если в аккумуляторе имеется нормаль- ная плотность серной кислоты (1,18), то его электродвижущая сила Е = 2 в. Но так как при эксплуатации аккумулятора плот- ность раствора серной кислоты в нем изменяется в сравнительно малых пределах (от 1,15 до 1,2), то и электродвижущая сила аккумулятора изменяется в относительно малых пределах, при- мерно от 1,95 до 2,05 в. 655
Влияние температуры на величину электродвижущей силы аккумулятора очень мало, а поэтому им практически можно пре- небречь. 3. Внутреннее сопротивление аккумулятора зависит от разме- ров пластин аккумулятора, от расстояния между пластинами, от удельного сопротивления электролита и от температуры. Так как состояние электролита при разряде и заряде аккумулятора ме- няется, то в соответствии с этим меняется и внутреннее сопро- тивление аккумулятора. В конце заряда плотность раствора сер- ной кислоты больше, чем в конце разряда, а поэтому аккумуля- тор в заряженном виде имеет несколько меньшее внутреннее сопротивление, чем в разряженном.' Внутреннее сопротивление свинцовых аккумуляторов по срав- нению с внутренними сопротивлениями гальванических элемен- тов очень мало вследствие больших размеров пластин и очень малого расстояния между ними. Величина внутреннего сопротивления аккумуляторов изме- ряется сотыми и тысячными долями ома. 4. Отдача кислотного аккумулятора по количеству электриче- ства в среднем равна К = 0,9. 5. Коэффициент полезного действия кислотного аккумулятора в среднем равен tq = 0,7—0,75. Отечественная электропромышленность освоила выпуск раз- нообразных типов кислотных аккумуляторов: СТ (стартерные), С (стационарные), СК (стационарные, допускающие одночасо- вый разряд), PH (радионакальные), РАДАН (радиоанодные) и др. В табл. 58, помещенной в главе XXXIV, приведены спра- вочные данные некоторых типов батарей кислотных аккумулято- ров, выпускаемых нашей электропромышленностью. § 192. ЭКСПЛУАТАЦИЯ КИСЛОТНЫХ АККУМУЛЯТОРОВ При составлении раствора электролита необходимо брать дистиллированную воду в количестве, потребном для получения необходимой плотности раствора серной кислоты. В табл. 53 указано, какое количество серной кислоты содер- жится в одном литре электролита при заданной его плотности. В табл. 54 указано количество серной кислоты, содержащееся в одном литре концентрированного раствора серной кислоты. Чтобы получить из концентрированного раствора серной кис- лоты электролит заданной плотности, необходимо добавить в него некоторое количество воды, сообразуясь с данными, приведен- ными в табл. 53 и 54. При составлении раствора серной кислоты следует вначале налить в стеклянную посуду необходимое количество дистилли- рованной воды, а затем осторожно тонкой струйкой вливать в нее потребное количество химически чистой серной кислоты. 656
Таблица 53 Содержание серной кислоты в одном литре электролита при заданной его плотности Плотность электролита Содержание серной кислоты на 1 л электролита, г Плотность электролита Содержание серной кислоты на 1 л электролита, г 1,11 * 175 1,23 382 1,12 191 1,24 400 1,13 207 1,25 418 . 1Д4 223 1,26 435 1,15 239 1,27 454 1,16 257 1,28 472 1,17 275 1,29 490 1,18 292 1,30 510 1,19 310 1,31 529 1,20 328 1,32 548 1,21 355 1,385 672 1,22 364 — — Запрещается при составлении раствора серной кислоты лить в серную кислоту воду, так как капли воды, соприкасаясь с сер- ной кислотой, сильно нагреваются и испаряются. Вырываясь с поверхности воды, они уносят с собой (разбрызгивают) ка- пельки серной кислоты. Последние, попадая на кожу лица и рук, могут произвести ожоги или порчу одежды. В случае ожога сер- ной кислотой необходимо место ожога немедленно промыть рас- твором соды, а затем холодной водой. Качество и плотность электролита влияют на емкость акку- мулятора, а поэтому рекомендуется не повышать и не понижать процентное содержание серной кислоты в растворе более чем на 5%. Нормальной температурой электролита кислотного аккумуля- тора надо считать 25° Ц. Повышение температуры свыше 40° Ц недопустимо. Таблица 54 Содержание серной кислоты в ее концентрированных растворах Плотность кон- центрирован- ного раствора Содержание серной кислоты в 1 л концентрированного раствора, г 1,80 1560 1,81 1595 1,82 1638 1,83 1690 1,84 1750 42-1377 657
Уровень электролита в кислотном аккумуляторе должен быть выше верхнего края пластин примерно на 10 мм. Доливать элек- тролит в аккумулятор, как правило, надо пере# его зарядом. Заряжать аккумулятор необходимо током, ’не превышающим нормальный зарядный ток, отмеченный в паспорте аккумулятора. После того как напряжение на зажимах аккумулятора при ^за- ряде достигнет 2,4 в, рекомендуется снизить величину зарядного тока примерно в 2 раза. Заряд аккумулятора необходимо всякий раз доводить до конца, т. е. до того момента, когда напряжение на зажимах аккумулятора становится равным примерно 2,75 в и затем в те- чение последующих 20—30 минут сохраняется неизменным по величине. В этом случае плотность электролита достигает макси- мума и перестает также увеличиваться. В конце заряда в акку- муляторе наблюдается бурное «кипение» электролита. Разряжать аккумулятор в нормальных условиях следует то- ком, не превышающим нормальную величину разрядного тока, обычно указываемую в паспорте аккумулятора. Нельзя аккумулятор оставлять незаряженным более суток, так как пластины его будут покрываться не растворимыми в воде крупными кристаллами сульфата, резко снижающего емкость аккумулятора. Надо помнить, что кислотный аккумулятор в силу его само- разряда теряет ежедневно около 2% номинальной емкости, а по- этому, если его. своевременно не заряжать, он спустя некоторое время полностью разрядится и его пластины покроются слоем сульфата. При сульфатации отрицательные пластины покры- ваются белым налетом, а положительные приобретают желтова- тый оттенок. Сульфат нельзя снять с пластин путем повторных зарядов и разрядов. При сильной сульфатации пластин аккуму- лятор обычно выбывает из строя. При слабой сульфатации суль- фат с пластин можно удалить механическим способом, например соскоблить. После снятия сульфата рекомендуется залить акку- мулятор слабым раствором серной кислоты и зарядить относи- тельно слабым током. Если кислотный аккумулятор необходимо на несколько меся- цев оставить без эксплуатации, то рекомендуется его предвари- тельно разрядить, снизив напряжение на его зажимах до 1,7 в. Затем следует вылить электролит из аккумулятора, а последний промыть чистой водой так, чтобы в нем не осталось никаких сле- дов серной кислоты. Присутствие серной кислоты в аккумуляторе можно обнаружить с помощью лакмусовой бумаги, которая, как правило, краснеет при смачивании ее водой, содержащей в себе даже весьма малое количество серной кислоты. После тщатель- ной промывки аккумулятора чистой водой отверстие в аккумуля- торе следует плотно закрыть пробкой. Если кислотный аккумулятор необходимо хранить заряжен- ным, то рекомендуется не реже одного раза в месяц заряжать 658
его Полностью, так как в результате саморазряда аккумулятор может разрядиться полностью и его пластины в дальнейшем по- кроются сульфатом. Запрещается в аккумуляторном помещении зажигать огонь, курить, пользоваться рубильниками и иными выключателями тока, при размыкании которых может возникнуть электрическая искра. Это вызывается тем, что кислород и водород, выделяю- щиеся из электролита аккумуляторов, образуют гремучий газ» который при воспламенении может дать сильный взрыв.
ГЛАВА ХХХШ ГАЛЬВАНИЧЕСКИЕ ЭЛЕМЕНТЫ § 193. ОБЩИЕ ПОЛОЖЕНИЯ Гальваническим элементом называется источник электриче- ской энергии, в котором, так же как и в аккумуляторе, электри- ческая энергия получается за счет расхода химической энергии активных веществ. Однако в отличие от аккумулятора в гальва- ническом элементе происходит необратимый электрохимический процесс — процесс преобразования химической энергии в элек- трическую, а последняя уже не может быть преобразована опять в химическую. При изготовлении гальванического элемента в него заклады- ваются в готовом виде активные вещества, которые он расходует при работе. По мере расхода активных веществ емкость эле- мента убывает, и когда она достигает некоторого предельного минимума, гальванический элемент становится непригодным для дальнейшей эксплуатации. Всякий гальванический элемент состоит из двух разнородных по своим химическим свойствам элементов, погруженных в элек- тролит. В результате химического взаимодействия электродов с электролитом в элементе образуется гальваническая пара. Один из электродов заряжается положительным электричеством, а другой — отрицательным. Электродвижущая сила гальваниче- ского элемента численно равна разности потенциалов на его за- жимах при разомкнутой внешней цепи, т. е. напряжению его холостого хода. В процессе работы гальванического элемента внутри его элек- тролита проходит ионный электрический ток. В результате этого на электродах элемента отлагаются вещества, которые могут создавать поляризацию электродов. Электродвижущая сила поляризации, как правило, направ- лена навстречу основной электродвижущей силе элемента. Это приводит к уменьшению результирующей электродвижущей силы элемента и нарушает его нормальную работу. Поляризацию электродов элемента подразделяют на устра- нимую и неустранимую. 660
Неустранимая поляризация обусловлена каче- ственным изменением электролита и активных веществ электро- дов элемента. Она снижает электродвижущую силу элемента. В этом случае путем выключения из цепи элемента нельзя вос- становить его первоначальный химический состав и начальную электродвижущую силу. ‘ Устранимая поляризация вызывается многими причинами и происходит при всяком электродном процессе и, в частности, в любом химическом источнике электрической энер- гии. Но в отличие от неустранимой поляризации устранимая по- ляризация исчезает в элементе при отключении его от внешней цепи. 'Например, когда элемент работает, состав его электролита изменяется наиболее сильно вблизи его электродов, что вызывает уменьшение разности потенциалов между ними. Если элемент выключить из цепи, т. е. прекратить ток в нем, то состав элек- тролита выравняется и потенциалы электродов, а также и элек- тродвижущая сила элемента достигнут вновь примерно своего первоначального значения. В цепях борьбы с вредным явлением поляризации в гальва- нические элементы вводят химические вещества, называемые деполяризаторами, способные значительно снижать по- ляризацию. Работа деполяризатора в основном сводится к по- глощению веществ, создающих поляризацию. Примером деполя- ризатора может служить медный купорос CuSO4, применяемый в медно-цинковом элементе. Путем поглощения водорода и вы- деления взамен него меди на медном электроде деполяризатор предотвращает возникновение поляризационной электродвижу- щей силы. В современных гальванических элементах в качестве деполяризатора наиболее широко применяется двуокись мар- ганца МпО2. Существуют элементы и с марганцово-воздушной деполяризацией, где в качестве деполяризатора, помимо дву- окиси марганца, используется и кислород воздуха. Гальваниче- ские элементы благодаря их малому весу, портативности, по- стоянной готовности к действию, простоте конструкции и удоб- ству эксплуатации получили широкое применение на практике. § 194. ГАЛЬВАНИЧЕСКИЕ ЭЛЕМЕНТЫ С МАРГАНЦОВОЙ ДЕПОЛЯРИЗАЦИЕЙ Гальванические элементы с марганцовой деполяризацией под- разделяются на два основных типа — сухие (С) и водоналив- ные (В). I. Сухие элементы На рис. 430, а показан в разрезе сухой элемент. Сосудом для него является цинковая коробка, которая одновременно с этим служит отрицательным электродом элемента. 661
Цинковый электрод изготовляется из электролитического, т. е. химически чистого, цинка. Высокие требования, „предъявляемые к качеству цинка, объясняются тем, что примеси, например же- леза, меди, приводят к образованию в элементе гальванических пар, создающих местные разрядные токи. В результате самораз- ряда элемента он весьма быстро может выйти из строя. На дне цинковой коробки установлен положительный электрод из двуокиси марганца МпОг. Для токоотвода в нем применяется угольный стержень, вокруг которого расположена запрессованная масса двуокиси марганца. Для улучшения проводимости двуокись марганца механически смешивается с графитом, которого берут примерно 20% смеси. Механическая смесь двуокиси марганца с графитом обычно на^ зывается агломератом- деполяризатором. Пространство между агломератом и цинковой коробкой за- полнено пастой или студенистой массой, составленной из 20%' раствора хлористого аммония (нашатыря) NH4CI и муки с при- месью хлористого цинка. В верхней части цинковой коробки помещена картонная шайба, верх которой залит смолкой, закупоривающей элемент й скрепляющей его детали. Между верхней частью агломерата и 7 См Нашатырь еполяризатор Изоляция б Смоляная за ли она Фильтров бумага Угольный полюс 55 а сч Рис. 430. Сухой (а) и водоналивной (б) гальванические элементы 662
картонной шайбой оставлено свобод- ное пространство для сосредоточе- ния газов, которые отсюда выходят через тонкую стеклянную трубочку. К цинковой коробке припаян гиб- кий проводник, а на угольный элек- трод надет металлический колпачок, к которому также припаян гибкий проводник для присоединения к вне- шней цепи. Снаружи цинковая коробка за- щищена картонным футляром, на ко- тором обычно указываются основные данные элемента: его марка, элек- тродвижущая сила, емкость и др. (рис. 431). Электродвижущая сила сухого элемента в начале разряда равна примерно 1,5 в. Внутреннее сопро- тивление сухого элемента зависит от его размеров и степени изношенно- сти. Оно колеблется от 0,25 ом до нескольких ом в конце разряда. Рис. 431. Внешний вид сухого элемента типа ЗС-Л-ЗО II. Водоналивные элементы В отличие от сухого элемента, который готов к действию с мо- мента изготовления, водоналивной элемент необходимо перед на- чалом эксплуатации залить водой, так как активное вещество его электролита находится в сухом состоянии. На рис. 430, б показан в разрезе водоналивной элемент. Здесь в отличие от сухого элемента 'пространство между агломе- ратом и цинковой коробкой плотно заполнено пропускной бума- гой, а сверху деполяризатора помещен сухой хлористый аммоний NH4C1, прикрытый сверху картонной прокладкой. В верхней части элемента имеется стеклянная трубочка, про^ ходящая через смоляную заливку. Через эту трубочку при за^ ряде элемента вводится вода, которая, растворив сухой хлорид стый аммоний, образует электролит. Газы, образующиеся в элементе при его работе, отводятся че- рез тонкую стеклянную трубку. Водоналивной элемент считается окончательно готовым к экс- плуатации через 6 часов после его заливки "водой. Электродвижущая сила свежезаряженного водоналивного эле- мента равна 1,5 в. Внутреннее сопротивление водоналивного элемента колеблется от 0,5 ом до нескольких ом в конце разряда. 6^3
В табл. 55 приведены основные данные некоторых сухих и во- доналивных элементов, изготовляемых нашей электропромышлен- ностью. Таблица 55 Основные данные элементов с марганцовой поляризацией Условные обозначения элементов Началь- ная Э.Д.С., в Началь- ное напря- жение, в Началь- ная ем- кость, а-ч Срок хра- нения в заряжен- ном со- стоянии, месяцы Емкость в конце срока хранения, а-ч Вес элементов Вес воды для заливки водо- наливных элементов, кг сухих, кг водона- ливные в неза- литом виде, кг 1С-Л-3 1,5 1,4 3,1 12 2,2 0,145 2С-Л-9 1,5 1,42 9,0 12 6,5 0,3 — — ЗС-Л-ЗО 1,5 1,44 30,0 18 24,0 0,7 — ЗС-Х-ЗО 1,65 1,6 30,0 18 24,0 0,7 — — ЗС-У-ЗО 1,65 1,6 30,0 18 24,0 0,7 — 4С-Л-37 1,5 1,42 37,0 18 28,0 1,1 — — 1В-Л-3 1,5 1,4 3,1 12 2,2 — 0,14 0,030 2В-Л-8,5 1,5 1,42 8,5 12 6,0 — 0,28 0,045 ЗВ-Л-27 1,5 1,44 27,0 12 22,0 — 0,65 0,100 4В-Л-31 1,5 1,42 31,0 12 25,0 — 1,0 0,175 Примечание. Первая цифра, стоящая в условном обозначении эле- мента, указывает размер его (самый малый размер — 1 и самый боль- шой — 4). Буква С обозначает, что элемент сухой, а буква В — водоналивной. Буква Л обозначает, что элемент летний, буква X — хладостойкий и буква У — универсальный. Последние цифры условных обозначений указывают начальную емкость элемента. Например, условное обозначение элемента 4С-Л-37 говорит о том, что данный элемент — четвертого (4), т. е. самого большого, размера из существующих типов элементов, что он сухой (С), летний (Л), емкость его 37 ампер-часов. Ниже приведена табл. 56, показывающая, в каких пределах температур допускается эксплуатация элементов. Таблица 56 Температурные интервалы работы элементов Условные обозначения на элементе Наименование элемента Предельнее температуры при' эксплуатации элемента °Ц У Универсальный От —40 до +60 л Летний От —20 до +60 X Хладостойкий От —40 до +40 Номинальной емкостью гальванического элемента называется то количество электричества, которое он отдает при непрерывном разряде его на некоторое стандартное внешнее со- противление при средней температуре 20° Ц до того момента, когда напряжение на его зажимах окажется равным 0,7 в. 664
Элементы с марганцовой деполяризацией, предназначенные для питания радиотехнических цепей накала, рассчитываются на максимум разрядного тока 0,1—0,2 а. III. Процессы при работе элементов с марганцовой деполяризацией Свежезаряженный элемент с марганцовой деполяризацией представляет собой электрохимическую систему следующего со- става: анод электролит катод (+) MnO21NH4C11 Zn (—), т. е. электролитом в элементе служит водный раствор хлористого аммония NH4CI, активным веществом положительного электрода (анода) служит двуокись марганца МпОг. Двуокись марганца здесь играет роль также деполяризатора-. Активным веществом отрицательного электрода является цинк Zn. В электролите независимо от того, работает элемент или нет, происходит непрерывный процесс диссоциации молекул хлори- стого аммония NH4CI согласно следующему уравнению: NH4C1Z±NH^4-Cr, (541) т. е. молекула хлористого аммония NH4C1 распадается на поло- жительный однозарядный ион аммония NH^ и отрицательный ион хлора С1“. Если к гальваническому элементу подключить потребитель, то в цепи возникнет электрический ток. Свободные электроны на- чнут перемещаться по внешней цепи от катода к аноду. Положи- тельные ионы аммония NH^ будут перемещаться внутри элек- тролита от катода к аноду, а отрицательные ионы хлора С1“— от анода к катоду. Процесс на отрицательном электроде (ка- тоде). Положительные двухзарядные ионы цинка Zn++ поки- дают положительную пластину под влиянием сторонних сил. В электролите они вступают во взаимодействие с отрицатель- ными ионами хлора и совместно с ними образуют хлористый цинк: Zn++ + 2СГ = ZnCI2. (542) Цинковый электрод, лишившись части положительных ионов цинка Zn++, заряжается отрицательным электричеством (избы- ток электронов). Процесс на положительном электроде (ано- де). На положительный электрод поступают из внешней цепи 665
свободные электроны и, кроме этого, со стороны электролита — положительные ионы аммония NH+. В результате этого на аноде возникает химическая реакция, уравнение которой в мо- лекулярном виде запишем так: MnO2 + 2NH4 + Н2О = MnO + 2NH4 (ОН), (543) т. е. двуокись марганца МпО2, соединяясь с аммонием 2NH4 и водой Н2О, образует окись марганца МпО и гидроокись аммо- ния 2NH4(OH). Таким образом, токообразующую реакцию в элементе с мар- ганцовой деполяризацией можно в молекулярной форме предста- вить уравнением в следующем виде: MnO2 + 2NH4C1 + Н2О + Zn = MnO + ZnCl2 + 2NH4 (ОН). (544) По мере расхода двуокиси марганца МпО2 и цинка Zn галь- ванический элемент теряет свои активные веществу и свою ем- кость. Кроме полезных токообразующих реакций, в гальванических элементах происходят еще и побочные, вредные процессы. Не- которые из них являются неизбежными спутниками основных то- кообразующих процессов, другие вызываются неудовлетворитель- ным качеством материалов, дефектами конструкции и производ- ства, неправильной или небрежной эксплуатацией. Общий вред всех паразитных процессов в элементах заклю- чается в том, что они происходят постоянно и независимо от того, работает элемент или нет. Все они вызывают бесполезный расход активных веществ, из-за чего сокращается количество энергии, полезно отдаваемой элементом, т. е. сокращается срок службы элемента. Емкость не эксплуатируемого элемента в конце срока его хранения сокращается в среднем на 20—25%. Пара- зитные процессы в элементе, связанные с потерей, им емкости, называются саморазрядом элемента. § 195. ГАЛЬВАНИЧЕСКИЕ ЭЛЕМЕНТЫ С МАРГАНЦОВО-ВОЗДУШНОЙ ДЕПОЛЯРИЗАЦИЕЙ В гражданской технике связи получили применение гальвани- ческие элементы с марганцово-воздушной деполяризацией (МВД). Конструктивная особенность этих элементов по сравнению с обычными элементами марганцовой деполяризации состоит в том, что в них обеспечен доступ наружного воздуха к актив- ным веществам положительного электрода (агломерату). В силу этого деполяризация в данном элементе осуществляется за счет не только двуокиси марганца, но и кислорода воздуха. Это зна- чительно улучшает эксплуатационные качества элемента и уве- личивает его емкость по сравнению с обычными элементами мар- 666
ганцовой поляризации (при прочих равных условиях) примерно в 1,5—2 раза. Электрохимическая система элемента с марганцово-воздуш- ной деполяризацией следующая: О2 (+) MnO2 inh4ci I Zn (-), т. е. на аноде, помимо двуокиси марганца МпО2, имеются моле- кулы кислорода О2 воздуха. Агломерат элемента с марганцово-воздушной деполяризацией состоит на одну треть из двуокиси марганца, а остальная часть приходится на долю активированного угля и графита. Элементы с марганцово-воздушной деполяризацией боятся пе- регрузки больше, чем обычные элементы. При незначительной перегрузке током они уже работают, как и обычные элементы с марганцовой деполяризацией, т. е. воздушная деполяризация в них в этом случае мало ощутима.
ГЛАВА XXXIV СОЕДИНЕНИЕ ХИМИЧЕСКИХ ИСТОЧНИКОВ ЭЛЕКТРИЧЕСКОЙ ЭНЕРГИИ В БАТАРЕИ § 196. ОБЩИЕ ПОЛОЖЕНИЯ Электродвижущие силы химических источников электрической энергии относительно малы. Например, электродвижущая сила щелочных аккумуляторов и гальванических элементов марганцо- вой деполяризации менее 2 в, а кислотного аккумулятора — не- сколько выше. Незначительная величина электродвижущей силы не дает возможности применять одиночные элементы (аккуму- ляторы и гальванические элементы) для обслуживания потреби- телей, номинальное напряжение которых измеряется, например, десятками вольт. Чтобы можно было питать электрической энер- гией потребители подобного рода, необходимо соединить эле- менты в батарею так, чтобы электродвижущая сила ее была больше номинального напряжения потребителя. Предельная нагрузка элементов также весьма ограничена. На- пример, гальванический элемент с марганцовой деполяризацией не рекомендуется разряжать током более 0,1 а. Аккумуляторы малых емкостей также могут давать относительно малые разряд- ные токи. Следовательно, в тех случаях, когда потребитель рас- считан на относительно большой номинальный ток, одиночный элемент уже не в состоянии его обслужить. Поэтому приходится элементы соединять в батарею так, чтобы она давала величину тока, необходимую для' нормальной работы данного потребителя. И, наконец, в тех случаях, когда потребитель рассчитан на отно- сительно большие номинальные напряжения и ток (на относи- тельно большую мощность), можно опять путем соответствую- щего соединения элементов добиться нормальной его работы. В современной технике батареи аккумуляторов и гальваниче- ских элементов нашли самое широкое применение. Советская электропромышленность выпускает самые разнообразные типы батарей элементов, например, накальные (батареи, питающие то- ком нить накала электронных ламп), анодные (батареи, питаю- щие анодные цепи радиотехнических устройств), стартерные (ба- 668
тареи, применяемые для пуска двигателей) и т. д. С электриче- скими данными некоторых из них мы познакомимся в конце этой главы. § 197. ПОСЛЕДОВАТЕЛЬНОЕ СОЕДИНЕНИЕ ХИМИЧЕСКИХ ИСТОЧНИКОВ ЭЛЕКТРИЧЕСКОЙ ЭНЕРГИИ (ЭЛЕМЕНТОВ) При последовательном соединении элемен- тов (аккумуляторов или гальванических элементов) в бата- рею они соединяются друге другом в ряд раз- ноименными полюсами. Рис. 432. Батарея последовательно соединенных элементов и ее условное обозначение На рис. 432 показано последовательное соединение четырех гальванических элементов типа ЗС-Л-ЗО в батарею. Последовательное соединение элементов применяется в том случае, когда ток I потребителя не превышает номинального раз- рядного тока элемента, а номинальное напряжение потребителя U больше электродвижущей силы Ео (напряжения Uo) отдельного элемента. При последовательном соединении элементов элементы дол- жны обладать одинаковыми электрическими емкостями. Если батарея составлена из п однотипных элементов, т. е. эле- ментов, обладающих одинаковыми электродвижущей силой Ео и внутренним сопротивлением Го, то общая электродвижущая сила такой батареи равна £ = л£0, (545) 669
т. е. электродвижущая сила Е батареи последовательно соединен- ных одинаковых элементов равна электродвижущей силе одного элемента Ео, умноженной на число элементов п батареи. Полное внутреннее сопротивление батареи в этом случае равно /?о = лг0, (546) т. е. полное внутреннее сопротивление /?0 батареи последова- тельно соединенных одинаковых элементов равно внутреннему Рис. 433. Схема электрической цепи с последовательно со- единенными элементами и внешним сопротивлением сопротивлению одного элемента, умноженному на число элементов п батареи. Емкость батареи Q последова- тельно соединенных элементов рав- на емкости Qo одного элемента, так как ток, отдаваемый всей бата- реей в целом, равен току, отдавае- мому каждым элементом в отдель- ности. Если батарея последовательно со- единенных потребитель с сопротивлением г (рис. в цепи можно определить по формуле элементов питает током 433), то величину тока закона Ома пЕ0 пг0 + г ’ (547) Напряжение U на зажимах батареи последовательно соеди- ненных элементов равно U = пЕ§ — InrQ — п(Е — IrQ) = nUQ, т. е. напряжение U на зажимах батареи последовательно соеди- ненных элементов равно напряжению на зажимах отдельного эле- мента Uo, помноженному на число элементов п батареи. Электрический коэффициент полезного действия последова- тельно соединенных элементов равен — Рпол —____________— г — 1 (54g\ ~ Лэбщ ~ /2 (^0 + Г) ~ nrQ + Г Г. • /г —- |- 1 Воспользовавшись формулой (547), можно вывести формулу для расчета числа п элементов, необходимого для питания за- данным током I потребителя, обладающего сопротивлением г. Из формулы (547) пЕй = 1пгй + 1г, или пЕй — 1пг0 = 1г, 670
откуда йакодим, чтб Ео — 7/q (549) Пример 209. Определить количество гальванических элементов типа ЗС-Л-ЗО,, необходимое для составления последовательной батареи, если _элек- тродвижущая сила одного элемента Ео = 1,5 в, внутреннее сопротивление го = 0,8 ом, сопротивление потребителя г= 192 ом, а величина тока в потре- бителе должна быть равна / = 0,075 а. Определить для данного режима ра- боты батареи элементов напряжение на зажимах батареи и электрический коэффициент полезного действия батареи. Решение. Применяя формулу (549), находим число элементов ба- тареи: 1г п~ Е0-1г0 0,076-192 1,5-0,075-0,8 = 10 элементов. Напряжение на зажимах батареи U = пЕ$ — InfQ = 10-1,5 — 0,075-10-0,8 = 14,4 в. Электрический коэффициент полезного действия батареи Ъл = -------/--------- =---------т/-----— = 0,96, или 7)эл = 96%. Пример 210. Аккумуляторная батарея типа 10НКН-22М состоит из п, = 10 последовательно соединенных щелочных кадмиево-никелевых аккумуляторов, каждый из которых имеет электродвижущую силу Ео=1,25 в и номиналь- ную емкость Qo — 22 ампер-часа. Определить внутреннее сопротивление одного аккумулятора, внутреннее сопротивление всей батареи аккумулято- ров, величину тока в цепи и через сколько часов разрядится батарея, если потребитель, который питается от нее электрической энергией, имеет сопро- тивление г = 20 ом. Решение. Внутреннее сопротивление одного аккумулятора согласно формуле (534) равно = # = -> = 0,0135 ом. ЧТО Внутреннее сопротивление батареи аккумуляторов /?о = пго = Ю-0,0135 — 0,135 ом, . 20 т. е. оно очень мало по сравнению с внешним сопротивлением (в^-г: = 140 раз меньше), а поэтому при расчете тока внутренним сопротивле- нием батареи можно пренебречь/ Величина тока в цепи в этом случае равна пЕ0 _______ ггЕ0 _* 10-1,25 ж пг0 + г ~ 20 Так как емкость батареи последовательно соединенных аккумуляторов Q равна емкости Qo отдельного аккумулятора, входящего в батарею, то время полного разряда аккумуляторной батареи Q _ <?° 22 часа 1 I I ~ 0,625 ~d5’2 671
§ 198. ПАРАЛЛЕЛЬНОЕ СОЕДИНЕНИЕ ХИМИЧЕСКИХ ИСТОЧНИКОВ ЭЛЕКТРИЧЕСКОЙ ЭНЕРГИИ (ЭЛЕМЕНТОВ) При параллельном соединении элементов (аккумуляторов или гальванических элементов) в батареи? они соединяются друг с другом одноимен иыми полюсами. На рис. 434 показано параллельное соединение четырех галь- ванических элементов типа ЗС-Л-ЗО в батарею. Параллельное соединение элементов применяется в том слу- чае, когда напряжение потребителя равно напряжению на зажи- мах отдельного элемента, входящего в батарею, а ток потреби- теля / больше предельного разрядного тока /разр отдельного эле- мента, т. е. />/раЭр. Батарея параллельно соединенных элементов, как правило, собирается из одинакового типа элементов, так как иначе воз- можен разряд одних элементов батареи через другие. Электродвижущая сила Е батареи параллельно соединенных элементов равна электродвижущей силе Ей одного элемента: Е = Е0. - (550) Полное внутреннее сопротивление 7?0 батареи параллельно со- единенных элементов равно внутреннему сопротивлению Го одного элемента, деленному на число т элементов в батарее: = (551) Рис. 434. Батарея параллельно соединенных элемен- тов и ее условное обозначение 672
Если батарея параллельно соединен- ных элементов питает током потребитель с сопротивлением г (рис. 435), то вели- чину тока /, отдаваемого всей батареей элементов, можно определить по формуле закона Ома а ток /о, отдаваемый каждым элементом батареи, в этом случае равен 7o = -L, (553) Рис. 435. Схема электри- ческой цепи с парал- лельно соединенными элементами и внешним сопротивлением т. е. ток /0, протекающий через каждый отдельный элемент, равен току /, отда- ваемому всей батареей, разделенному на число элементов т батареи. В соответствии с этим емкость Q всей батареи параллельно соединенных элементов равна емкости Q'o одного элемента, умно- женной на число элементов т батареи: Q = mQ0. (554) Напряжение на зажимах батареи параллельно соединенных элементов равно £7 = Е0-/А=£0-/0г0, (555) т. е. напряжение U на зажимах батареи параллельно соединен- ных элементов равно электродвижущей силе Ео одного элемента минус падение напряжения в одном элементе. Электрический коэффициент полезного действия батареи па- раллельно соединенных элементов равен „ __ ^пол _ 72Г __ 1 ^эл — ~р — / г \ — ~~г = ~. (556) 'поли ;2 7 Jo_ , г \ д°_ + Г — + 1 \ т / т тг Воспользовавшись формулой (552), можно вывести формулу для расчета числа элементов, необходимого для питания задан- ным током / потребителя, обладающего сопротивлением г. Из формулы (552) /A + /r==f т IrQ = (£0 - /г) т. ИЛИ 43—1377 673
Откуда находим, что /п = 1гй Е0-1г ’ • (557) Пример 211. Определить количество гальванических элементов типа ЗС-Л-ЗО, необходимое для составления параллельной батареи, если электро- движущая сила одного элемента Eq = 1,5 в, внутреннее сопротивление одного элемента Го.= О,6 ом, сопротивление потребителя г = 4,85 ом, а величина тока в потребителе должна быть равна / = 0,3 а. Определить также для данного режима величину тока /о, отдаваемого каждым элементом, и элек- трический коэффициент полезного действия батареи элементов. Решение. Применяя формулу (557), находим число элементов ба- тареи /г0 0,3-0,6 щ = е \ тгг = 4 элемента. Е0 — 1г 1,5-0,3-4,85 Ток /о, отдаваемый каждым элементом в отдельности, равен /0 = — = - о,О75 а. и т 4 Электрический коэффициент полезного действия согласно формуле (556) равен 1 ^ЭЛ-------~ 'о _ тг 06*---------- 0,97, или у]эл = 97%. 4-4,85 + 1 § 199. СМЕШАННОЕ СОЕДИНЕНИЕ ХИМИЧЕСКИХ ИСТОЧНИКОВ ЭЛЕКТРИЧЕСКОЙ ЭНЕРГИИ (ЭЛЕМЕНТОВ) * Смешанное соединение элементов представляет собой сочета- ние последовательного и параллельного соединения элементов, Рис. 436. Смешанное соединение элементов в батарею На рис. 436 показана схема смешанно соединен- ных элементов. Здесь мы видим, что элементы в от- дельных группах соедине- ны последовательно, а са- ми группы соединены ме- жду собой параллельно. Смешанное соединение элементов применяется в том случае, когда ток I и напряжение U потребите- ля больше электродвижу- щей силы Eq (напряжение Uq) и разрядного тока /разр отдельного элемента. Допустим, что в бата- рее смешанно соединенных 674
элементов имеется -тп, параллельно соединенных групп, каждая из которых составлена из п последовательно со- единенных одинаковых элементов (рис. 436). Электродвижущая сила Е батареи смешанно соединенных элементов рав- на электродвижущей силе Ео одного элемента, умноженной на число эле- ментов п в одной группе Е = пЕ$. (558) Рис. 437. Схема электриче- ской цепи со смешанно со- единенными элементами и внешним сопротивлением Полное внутреннее сопротивление 7?0 батареи смешанно со- единенных элементов равно внутреннему сопротивлению г0 одного элемента, умноженному на число элементов п в группе и поде- ленному на число групп т в батарее: о _ № (559) где п — число последовательно соединенных элементов в каж- дой группе; т — число параллельно соединенных групп. Если батарея смешанно соединенных элементов питает током потребитель с сопротивлением г ..(рис. 437), то величину тока /, отдаваемого всей батареей элементов, можно определить, исходя из* формулы закона Ома <56°) т а ток /о, отдаваемый каждым элементом батареи, в этом случае равен <561) т. е. ток /0, протекающий через каждый отдельный элемент, ра- вен току /, отдаваемому всей батареей, разделенному на число параллельных групп /п всей батареи. В соответствии с этим емкость Q всей батареи смешанно со- единенных элементов равна емкости одного элемента Q, умно- женной на число параллельных групп т в батарее: Q = znQ0. (562) Напряжение на зажимах батареи смешанно соединенных эле- ментов равно и = пЕй —= Е - IR0. (563) 43* 675
Электрический коэффициент полезного действия батареи сме- шанно соединенных элементов равен Р Рг 1 4.. = -^=—!^-----(564) Пример 212. Батарея смешанно соединенных гальванических элементов ЗО-Л-ЗО состоит из двух параллельных групп (т = 2) по шесть элементов (п = 6) в каждой. Определить величину тока» отдаваемого батареей, напряжение на зажи- мах батареи, электрический коэффициент полезного действия батареи, а также время пол него если электродвижущая тивление Го = 0,5 ом, г = 28,5 ом. Решение. Ток, муле (560) равен разряда оатареи при непрерывной заданной нагрузке, сила одного элемента £0=1,5 в, внутреннее сопро- емкость Qo = 30 а-ч, а внешнее сопротивление отдаваемый батареей потребителю, согласно фор- пЕ0 6-1,5 п । 6-0,5 п0 _ -Го + Г -^ + 28,5 Напряжение на зажимах батареи U = пЕй— /2^°. = 6-1,5—0,3--^^-= 8,55 в. и т 2 • ' Емкость всей батареи Q = tnQQ = 2-30 = 60 а-ч. Время разряда батареи t = ~~ = = 200 часов. / V,O Пример 213. Требуется рассчитать аккумуляторную батарею для аварий- ного освещения на номинальную мощность Р=1,32 кет при напряжении £=110 в, если в нашем распоряжении находятся аккумуляторные батареи типа 4НКН-60М, т. е. батареи, имеющие номинальное напряжение £0 = 5 в, номинальный разрядный ток 7разр='7,5 а и номинальную емкость Qo = = 60 а-ч. Внутренним сопротивлением батареи практически можно прене- бречь. Решение. Величина тока I при полной нагрузке батареи равна / Р 1320 Ю - “ и по “12 Число параллельно соединенных групп в батарее равно 12 1R / m = ~7------ 'разр Следовательно, необходимо взять т = 2 группы. Определим число батарей 4НКН-60М в составе каждой группы: £ НО . п == -yj- = —г— = 22 батареи. С/Q о 676
Таким образом, общее число батарей типа 4НКН-60М, входящих в рас- считываемую смешанную батарею, равно ДГ = пт = 22-2 = 44. Емкость всей батареи Q = tnQ0 == 2-60 = 120 а-ч. Время работы батареи при полной нагрузке без подзарядки / Q 120 1А f = -р- = —— = ю часов. § 200. АККУМУЛЯТОРНЫЕ БАТАРЕИ В современной электротехнической практике применяются са- мые разнообразные типы аккумуляторных батарей, изготовляе- мые нашей электропромышленностью. Ниже приведены таблицы основных данных некоторых щелочных и кислотных аккумуля- торных батарей. Рис. 438. Батареи щелочных аккумуляторов В табл. 57 приведены данные некоторых щелочных аккумуля- торных батарей, широко применяемых в радиотехнике (рис. 438). Примечание к табл. 57. Условное обозначение батареи 64АКН-2.25 надо понимать так: 64 — число элементов в батарее, А—анодная, КН— кадмиево-никелевая, 2,25 — емкость батареи 2,25 ампер-часа. Условное обо- значение батареи 5НКН-45 надо понимать так: 5 — число элементов в бата- рее, Н — накальная, КН — кадмиево-никелевая, 45 — емкость батареи 45 ам- пер-часов. Если батарейный ящик имеет съемную крышку, то в конце условного обозначения батареи ставится буква М. Например, батарея типа 5НКН-100М имеет съемную крышку. 677
Таблица 57 Основные данные щелочных аккумуляторных батарей Тип батареи Количество элементов в батарее Номинальное напряжение батареи, в Номинальная емкость, а~ч Нормальный зарядный ток, а Нормальный разрядный ток, а Размеры, л/м Вес батареи, кг длина сз К Я О. я 3 высота 32АКН-2.25М 32 40 2,25 0,56 0,28 525 165 168 14,2 64АКН-2,25 64 80 2,25 0,56 0,28 525 317 168 28,6 10НКН-22М 10 12,5 22 5,5 2,75 465 148 525 21,0 17НКН-22 17 21,25 22 5,5 2,75 435 285 252 35,0 4НКН-45М 4 5 45 11,25 5,65 305 148 252 14,5 4НКН-45 4 5 45 11,25 5,65 305 148 252 14,0 5НКН-45 5 6,25 45 11,25 5,65 372 148 252 17,0 6НКН-45М 6 7,5 45 11,25 5,65 440 148 252 21,0 7НКН-45 7 8,75 45 11,25 5,65 508 148 252 24,0 8НКН-45М 8 10 45 11,25 5,65 575 148 252 26,6 10НКН-45 10 12,5 45 11,25 5,65 707 152 252 33,5 4НКН-60М 4 5 60 15,0 7,5 262 170 388 23,5 5НКН-60 5 6,25 60 15,0 7,5 315 170 388 29,0 7НКН-60М 7 8,75 60 15,0 7,5 431 170 388 39,0 ‘ 10НКН-60М 10 12,5 60 15,0 '7,5 600 170 388 56,0 4НКН-100М 4 5 100 25 12,5 374 178 388 33,0 5НКН-100М 5 6,25 100 25 12,5 459 178 388 42,5 10НКН-100М 10 12,5 100 25 12,5 884 178 388 82,0 10НКН-100 10 12,5 100 25 12,5 884 178 388 84,0 Таблица 58 Данные кислотных стартерных аккумуляторных батарей Тип батареи Количество элементов в батарее Номиналь- ное напря- жение ба- тареи, в Номиналь- ная ем- кость, а-ч Размеры, мм Вес, кг длина ши- рина вы- сота 6СТЭ-48 6 12 48 293 188 238 32 ЗСТЭ-112 3 6 112 288 187 227 23,6 ЗСТМ-112 3 6 112 288 178 238 25,0 ЗСТП-112 3 6 112 303 182 238 25,0 6СТЭ-128 6 12 128 516 236 242 55,0 6СТЭ-144 6 12 144 525 279 229 59,8 6СТЭ-144 6 12 144 525 307 258 63,4 6СТМ-80 6 12 80 243 187 224 16,0 ЗСТП-80 3 6 80 251 191 225 16,0 ЗСТМ-100 3 6 100 266 185 219 19,0 ЗСТП-100 3 6 100 267 182 218 19,0 ЗСТЭ-112 3 6 112 287 178 238 21,5 ЗСТП-112 3 6 112 303 182 238 21,5 ЗСТЭ-126 3 6 126 326 184 240 24,5 ЗСТП-126 3 6 126 323 182 238 24,5 ЗСТП-142 3 6 142 372 182 238 27,5 Примечание. Условное обозначение батареи ЗСТП-142 надо пони- мать так: 3 — число аккумуляторов в батарее, СТ — стартерная батарея, П — моноблок из пластмассы, 142 — емкость батареи в ампер-часах. 678
Рис. 439. Батареи кислотных аккумуляторов В табл. 58 приведены данные некоторых кислотных стартер- ных аккумуляторных батарей, изготовляемых нашей электропро- мышленностью (рис. 439). § 201. ЗАРЯД АККУМУЛЯТОРНЫХ БАТАРЕЙ Аккумуляторы обычно заряжают от сети постоянного тока или от специальных генераторов постоянного тока, предназна- ченных для заряда аккумуляторов. • • • На рис. 440 приведена простейшая схема заряда батареи аккумуляторов от сети постоянного тока. При подключении бата- реи к сети надо плюс-зажим батареи подключить к плюс-зажиму сети, а минус-зажим батареи — к минус-зажиму сети. Если по- лярность сети неизвестна, то прежде чем подключать батарею аккумуляторов, надо определить полярность ее при помощи вольтметра магнитоэлектрической системы или каким-либо иным способом. Для регулирования величины зарядного тока последовательно с аккумуляторной батареей подключают ламповый реоста'г или ползунковый, рычажный и т. п. Для наблюдения за режимом заряда аккумуляторной батареи служат амперметр А и вольтметр V. При расчете регулировочного реостата необходимо знать ве- личину напряжения в сети (на зажимах генератора), от которой заряжается батарея, величину зарядно- го тока, число аккумуляторов, соеди- ненных последовательно в батарее (в каждой параллельной группе). Расчет регулировочного сопротивле- ния для заряда щелочных аккумулятор- ных батарей производится по формуле Рис. 440. Простейшая схема заряда батареи аккумуля- торов где Гр — сопротивление регулировоч- ного реостата в омах; 679
U—напряжение источника электрической энергии (сети) в вольтах; п— число аккумуляторов, соединенных последовательно в батарее (в каждой параллельной группе); 1,4—величина напряжения в вольтах на зажимах каждого аккумулятора в начале его заряда. Пример 214. Пять аккумуляторных батарей типа 4НКН-60, соединен- ных последовательно, заряжаются от источника электрической энергии постоянного тока, номинальное напряжение которого U = 36 в и номиналь- ный ток I = 25 а. Определить сопротивление регулировочного реостата, соединенного последовательно с заряженными аккумуляторными батареями. Решение. Так как в каждой батарее типа 4НКН-60 имеется по че- тыре аккумулятора, соединенных последовательно, то в пяти батареях, соединенных последовательно, имеется п = 5.4 = 20 аккумуляторов. Нормальная величина зарядного тока равна емкости аккумуляторной батареи, разделенной на 4: 'зар - 4 — 4 — 15 а. Сопротивление регулировочного реостата найдем по формуле U—\,4n 36—1,4-20 Гр ~ 4ар “ 15 ~ 0,533 0М- Следовательно, для аккумуляторной батареи необходим реостат с со- противлением не менее 0, 533 ом, рассчитанный на ток не менее 15 а. Пример 215. Требуется зарядить три аккумуляторные батареи типа 64АКН-2,25 от источника электрической энергии постоянного тока с номи- нальным напряжением U = 120 в. Определить способ подключения батарей на заряд и величину сопротивления регулировочного реостата. Решение. Число аккумуляторов, которое при заряде может быть включено последовательно, равно напряжению U на зажимах источника электрической энергии, деленному на максимальное напряжение па зажи- мах каждого аккумулятора при заряде, т. е. на 1,8. Следовательно, U 120 п — pg- = y-g- = 6b аккумуляторов. В батарее 64АКН-2,25 последовательно соединено 64 аккумулятора. Следовательно, в каждой параллельной группе общей батареи можно иметь только одну батарею типа 64АКН-2,25. Поэтому предназначенные для заряда три батареи следует соединить в три параллельно соединенные группы, так что в каждой группе будет по одной батарее типа 64АКН-2,25, или, иначе сказать, но 64 последова- тельно соединенных аккумулятора. ' Нормальный зарядный ток в каждом аккумуляторе или батарее типа ,64АКН-2,25 равен , Q 2,25 Л „ 4)зар — 4 = 4 а* Зарядный ток в цепи /зар = 3/озар = 3*0,56 = 1,68 а» 680
Следовательно, ток, проходящий через регулировочный реостат, равен 1,68 а. Сопротивление реостата U— 1,4п 120—1,4-64 г₽ = ~цг ° —= 18,1 ом- Значит, для заряда аккумуляторных батарей необходимо иметь реостат с сопротивлением не менее 18,1 ом, рассчитанный на ток не менее 1,68 а. § 202. БАТАРЕИ ГАЛЬВАНИЧЕСКИХ ЭЛЕМЕНТОВ Батареи сухих элементов типа БАС » В радиотехнике для питания анодных цепей широко приме- няются батареи сухих элементов типа БАС (батарея анодная су- хая). На рис. 441 показана батарея БАС-80-Л-0,9, состоящая из 60 последовательно соединенных сухих стаканчиковои конструк- ции элементов. Она имеет начальную электродвижущую силу Рис. 441. Батарея сухих элементов типа БАС-80-Л-0,9 92 в, конечное напряжение 60 в, начальную емкость 0,85 а-ч при величине разрядного тока не более 0,015 а и емкость в конце срока хранения (после 10-месячного хранения), равную 0,65 а-ч. Батарея БАС-80 обычно имеет 4 вывода: вывод от минусового зажима батареи (—), вывод +60 в, вывод +80 в и вывод +90 в. В соответствии с этим от батареи можно брать три напряжения: 60, 80 и 90 в. Помимо батарей .БАС-80, широко применяются батареи БАС-60. Они состоят из 40 последовательно соединенных сухих гальванических элементов стаканчиковой конструкции, размеры которых примерно в 1,5 раза меньше, чем размеры элементов для батареи БАС-80. Широкое применение в практике находят также батареи типа БАС-Г. Это сухая анодная батарея галетной конструкции. 681
В табл. 59 приведены основные данные некоторых батарей типа БАС. Т а б л и ц а 59 Условное обозначение батареи элементов Начальная Э.Д.С.. в Начальное напряже- ние, в Начальная емкость, а-ч Конечное напряже- ние, в Сохран- ность, месяц Емкость в конце срока хранения, а-ч БАС-бО-Х-0,5 70 68 0,5 40 10 0,3 БАС-60-Х-0,7 74 71 0,7 57 12 0,5 БАС-80-Л-0,9 94 92 0,85 60 •10 0,65 БАС-80-Х-1.0 104 102 1,05 60 1.5 0,7 БАС-Г-бО-Л-1,3 74 71 1,3 40 12 0,95 БАС-Г-80-Л-0.8 96 95 0,8 60 12 0,6 БАС-Г-80-11-2,1 104 102 2,1 60 15 1,5 БАС-Г-120-Л-0.27 127 120 0,27 56 6 0,2 БАС-Г-160-Л-0.35 170 160 0,35 100 6 0/24 § 203. АНОДНЫЕ БАТАРЕИ ГАЛЕТНО-ПЛЕНОЧНОЙ КОНСТРУКЦИИ За последние годы в технике связи широкое применение на- шли батареи сухих элементов галетно-пленочной конструкции. На рис. 442 показан в разрезе один из элементов галетно- пленочной конструкции. Отрицательным электродом в этом элементе служит цинковая пластина 1. На ее наружную поверхность нанесен специальный Рис. 442. Элемент галетно-пленочной кон- струкции: 1 — цинковая пластина; 2 — электропроводящий слой; 3 — картонная диафрагма; 4— агломерат; 5— кольцо электропроводящий слой 2, непроницаемый для влаги и электро- лита. Назначение его — создавать надежный контакт с соседним однотипным элементом при последовательном соединении их в батарею. Так как электропроводящий слой не пропускает че- рез себя электролит, то этим самым он предотвращает переход его из одного элемента в другой, что нарушило бы нормальную работу элемента. Цинковая пластина 1 другой своей поверх- ностью плотно соприкасается с картонной диафрагмой 3, пропи- танной обычно электролитом, применяемым в марганцово-цин- ковом элементе. Затем идет слой 4, состоящий из механической 682
прессованной смеси двуокиси марганца МпО2 и графита (агло- мерат), играющий роль положительного электрода в элементе. Агломерат отделен от картонной диафрагмы 3 тонкой бумагой, предохраняющей элемент от попадания частиц агломерата на цинковую пластину. ‘ Все составные части галетно-пленочного элемента спрессо- ваны и скреплены пленочным хлорвиниловым кольцом 5. Благо- даря своей эластичности хлорвиниловое кольцо плотно облегает со всех сторон элемент, надежно изолируя его по боковой по- верхности. Торцовые поверхности каждого элемента остаются обнаженными, причем одна из них—это электропроводящий слой 2 на цинковой пластине, а другая — поверхность положи- тельного электрода 4. При составлении батареи последовательно соединенных га- летно-пленочных элементов последние накладываются друг на друга с таким расчетом, чтобы положительный электрод 4 одного элемента был плотно наложен на электропроводящий слой 2 другого элемента. В результате наложения элементов друг на друга разноименными полюсами образуется столбик элемен- тов, который обычно называется блоком галетно-пленочных эле- ментов. Собранный блок галетно-пленочных элементов стягивают бан- дажами, покрывают по боковой поверхности парафином и затем заделывают парафинированной бумагой. Изолирующая хлорвиниловая пленка элементов,, толщина ко- торой примерно равна 0,2 мм, обладает свойством пропускать через себя газы, образующиеся в нем в процессе работы. Бла- годаря этому в элементе можно обойтись без газовой камеры и газоотводной трубки, что уменьшает размеры, вес элемента и упрощает его конструкцию. Основное преимущество батареи галетно-пленочной конструк- ции— это получение относительно большой емкости (до 1,3 а-ч) при малых размерах батареи. Существенный недостаток батарей галетно-пленочной конструкции — их более высокое внутреннее сопротивление по сравнению с батареями стаканчикового типа.
ЧАСТЬ ПЯТАЯ ЭЛЕКТРИЧЕСКИЕ МАШИНЫ ГЛАВА XXXV ГЕНЕРАТОРЫ ПОСТОЯННОГО ТОКА § 204. ОБЩИЕ СВЕДЕНИЯ Генераторами электрической энергии назы- ваю т электрические машины, преобразующие механическую энергию в электрическую. Различают генераторы постоянного и переменного тока. Пер- вые предназначены для питания потребителей электрической энергии постоянным током, а вторые — переменным. Генераторы постоянного тока широко применяются в совре- менной электротехнике. Например, в технике сильных токов гене- раторы постоянного тока используются в трамвайном деле, на электрических железных дорогах и в других специальных элек- тротехнических установках, где переменный ток использовать нельзя. В технике связи генераторы постоянного тока играют исклю- чительно большую роль. Например, зарядные генераторы по- стоянного тока заряжают аккумуляторные батареи, обслуживаю- щие аппаратуру проводной и беспроводной техники связи; ра- диогенераторы постоянного тока непосредственно питают цепи накала и анодные цепи мощных радиостанций; умформеры пре- образуют постоянный ток низкого напряжения в постоянный ток высокого напряжения для питания специальных радиостанций, например автомобильного типа и др. Наша промышленность полностью освоила серийное произ- водство генераторов постоянного тока, начиная с машин очень большой мощности, применяемых в технике сильных токов, и кончая маломощными генераторами, применяемыми в специаль- ных установках техники связи. § 205. ПРИНЦИП ДЕЙСТВИЯ ГЕНЕРАТОРОВ ПОСТОЯННОГО ТОКА Работа генераторов постоянного тока основана на принципе электромагнитной индукции. 684
Допустим, что контур abed вращается с равномерной угловой скоростью в двухполюсном магнитном поле (рис. 443) и в ре- зультате этого в нем индуктируется синусоидальная электродви- жущая сила. Концы активных сторон ab и cd контура присоединены соот- ветственно к кольцам 1 и 2, которые вместе с контуром вра- щаются вокруг их общей оси ОО\. С помощью щеток 3 и 4, на- ложенных на кольца, контур соединен с потребителем электриче- ской энергии, имеющим сопротивление R. Синусоидальная электродвижущая сила, индуктированная в контуре, создает в потребителе переменный синусоидальный Рис. 443. С помощью двух металлических колец 1 и 2 и щеток 3 и 4 рамка соединена с внешней цепью ток. Этот ток, проходя через потребитель, выделяет в его сопро- тивлении R тепло за счет электрической энергии, передаваемой контуром этому потребителю. В самом же контуре электрическая энергия создается в результате преобразования механической энергии «первичного двигателя», вращающего этот контур в маг- нитном поле. Рассмотренное нами электрическое устройство, состоящее из контура, вращающегося во внешнем магнитном поле, является принципиально простейшим генератором переменного синусои- дального тока. Чтобы такой генератор мог давать во внешнюю цепь выпрям- ленное напряжение, необходимо заменить в нем систему контакт- ных колец особым выпрямляющим приспособлейием — коллекто- ром. Допустим, что контур abed (рис. 444) вращается в магнит- ном поле и в нем индуктируется синусоидальная электродвижу- щая сила. Однако активные стороны ab и cd контура теперь присоединены не к двум контактным кольцам, а к двум полу- кольцам 1 и 2. С помощью щеток 3 и 4, наложенных на эти полукольца, контур соединен с потребителем, имеющем сопро- тивление R. 685
Рис. 444. Два изолированных, полукольца 1 и 2 спрямляют переменный ток При вращении контура вместе с ним вращаются и полу- кольца вокруг их общей оси ОО1.,Так как щетки неподвижны, то они попеременно соприкасаются то с одним, то с другим полу- кольцом. Этот «обмен» полукольцами происходит в момент, когда синусоидальная электродвижущая сила в контуре перехо- дит через свое нулевое значение. В результате каждая щетка все Рис. 445. Пульсирующее напряжение время сохраняет свою полярность неизменной: в данном случае тетка 3 все время имеет положительную полярность, а щет- ка 4 — отрицательную. Следовательно, если на полукольцах / и 2 имеется некото- рое синусоидальное напряжение, то на щетках 3 и 4 оно уже становится выпрямлен- ным и в данном слу- чае пульсирующим (рис. 445). Итак, с помощью простейшего коллекто- ра, состоящего из двух полуколец (двух кол- лекторных пластин), мо- жно на зажимах (щет- ках) простейшего гене- ратора переменного си- Рис. 446. Напряжение, выпрямленное с помо- щью коллектора 686
нусоидального тока получить выпрямленное (пульсирующее) на- пряжение. Чтобы уменьшить пульсацию выпрямленного напря- жения, в генераторах постоянного тока применяют коллектор с большим числом коллекторных пластин, а одиночный контур заменяют совокупностью контуров. Чем больше имеет коллектор пластин, тем больше приближается выпрямленное пульсирующее напряжение к постоянному (рис. 446). § 206. ОСНОВНЫЕ ЧАСТИ ГЕНЕРАТОРА ПОСТОЯННОГО ТОКА На рис. 447 представлена принципиальная схема четырехпо- люсного генератора постоянного тока. Здесь показаны следую- щие основные части генератора: ярмо /, электромагниты 2 и якорь 3. Ярмо генератора 1 — это его остов. К ярму прикреплены сер- дечники электромагнитов и крышки с подшипниками, на которых покоится вал генератора. Так как ярмо входит в магнитную цепь генератора, то оно изготовляется Рис. 447. Принципиальная схема четырехполюсного генератора по- стоянного тока. из ферромагнитного материала, например из литой стали. Электромагниты генератора (рис. 448) состоят из сердечни- ков 1 с насаженными на них катушками 2 с медной изолиро- ванной проволокой, называемы- ми катушками возбуждения. Рис. 448. Электромагнит генератора Сердечники электромагнитов изготовляются обычно из листо- вой электротехнической стали. Они прикрепляются к ярму гене- ратора болтами (рис. 448). Чтобы придать магнитным линиям в воздушных зазорах не- обходимое направление, сердечники электромагнитов снаб- жаются наконечниками, называемыми полюсными башмаками. Электромагниты, питаемые постоянным током, создают в ге- нераторе магнитное поле, магнитные линии которого показаны на рис. 447 пунктирными линиями. 687
Рис. 449. Якорь генератора: Z — сердечник; 2 — обмотка; 3 — коллектор; 4 — вал; 5 — вентилятор Каждая катушка элек- тромагнита (катушка воз- буждения) состоит из вит- ков медной изолированной проволоки или медной ленты, намотанных на каркас из жести, картона или какого-нибудь иного материала. Обмотки катушек воз- буждения соединены друг с другом последователь- но и совместно образуют обмотку возбуждения ге- нератора. Постоянный ток, протекающий по обмоткам электро- магнитов, имеет в них такое направление, что любые два соседние сердечник а электр о м а гн ита им еют разноименную полярность (см. рис. 447). Якорь генератора постоянного тока (рис. 449) состоит из сердечника с на- несенной на него обмоткой. Так как якорь генератора является одним из участков магнитной цепи, то его сердечник обычно изготовляется из ферромагнитного материала, на- пример из электротехнической стали. Во избежание потерь на вихревые токи сердечник якоря собирается из отдельных листов стали, имеющих зуб- чатую форму (рис. 450). Рис. 450. Стальной лист сердечника якоря Во впадины собранного сердечника якоря укладывается якорная обмотка, изготовленная из изолированной проволоки Рис. 451. Коллектор (для маломощных генераторов) или из медных полос прямоугольной формы, сделанных на специаль- ных шаблонах (для генераторов относительно большой мощности). Якорная обмотка прочно за- крепляется на сердечнике якоря бандажами или клиньями, чтобы она не выпадала из пазов сердеч- ника под влиянием центробежной силы. Коллектор (рис. 451) представ- ляет собой совокупность коллек- торных пластин 1 из твердотяну- той меди. 688
Коллекторные пластины изолированы друг от друга прессо- ванной слюдой 2 (миканитом), имеющей практически одинако- вую изнашиваемость с медью коллекторных пластин при трении о них щеток генератора. В нижней своей части коллекторные пластины имеют очер- тание, напоминающее ласточкин хвост. Этой своей частью Пла- стины помещены в соответствующие их форме вырезы коллек- торной втулки с?. Коллекторные пластины изолированы микани- том 4 от коллекторной втулки 3, от нажимной шайбы 5 и от нажимной гайки 6. Собранный коллектор закреплен нажимной гайкой 3, навин- чиваемой на коллекторную втулку 3. Коллектор насаживается на вал генератора, где его пластины соединяются с якорной обмоткой генератора (рис. 449). а 6 Рис. 452. Щеткодержатель со щеткой Щетки, На коллектор наложены угольные или металлические щетки, при помощи которых генератор электрически соединяется с внешней цепью. Минимальное число щеток, которое может иметь генератор постоянного тока, равно двум: одна служит положительным по- люсом генератора (положительная щетка), другая — отрица- тельным (отрицательная щетка). В многополюсных генераторах постоянного тока число пар щеток обычно равно числу пар по- люсов, что обеспечивает лучшую работу генератора. Одноимен- ные щетки генераторов имеют одинаковые потенциалы и связаны электрически друг с другом соединительными проводниками. Ширину щеток обычно выбирают так, чтобы щетка одновре- менно перекрывала две — три ламели коллектора. Это необхо- димо для того, чтобы по возможности предотвратить искрение на коллекторе под щетками при работе машины. Щетки а закреплены в щеткодержателе б (рис. 452). При помощи пружин щеткодержателя щетки своей вогнутой поверх- ностью плотно прижимаются к цилиндрической поверхности 44—1377 689
Рис. 453. Генератор постоянного тока коллектора. Щеткодер- жатель своим отвер- стием посажен на стер- жень траверсы, поворо- том которой достигает- ся необходимая уста- новка щеток генератора на коллекторе. На рис. 453 показан внешний вид генера- тора постоянного тока; здесь можно видетб траверсу /, щеткодер- жатель 2, коллектор 3, станину 4, подшипник 5 и маховик 6. § 207. ОБМОТКИ ГЕНЕРАТОРОВ ПОСТОЯННОГО ТОКА Якорная обмотка генератора уложена в пазы сердечника якоря. В зависимости от мощности генератора обмотку якоря выполняют или в виде медных изолированных стержней, соеди- ненных последовательно друг с другом в том или ином порядке, или из отдельных готовых катушек, изготовленных по шаблону и уложенных в пазы сердечника якоря, или же путем намотки вручную изолированной медной проволокой. Рис. 454. Схема волновой обмотки четырех по- люсного генератора постоянного тока 690
Обмотка наносится на сердечник якоря так, что каждые два Активных проводника, соединенных непосредственно и последо- вательно друг с другом, лежат под разными магнитными полю- сами. Расстояние между этими проводниками по поверхности якоря может быть различным и в частном случае может быть равным полюсному делению, т. е. расстоянию между се- рединами двух соседних разноименных полюсов по окружности якоря. Обмотка называется волновой, если она наносится на сердеч- ник якоря так, что провод проходит поочередно под всем» полю- сами машины и возвращается к исходному полюсу. На рис. 454 схематически показан четырехполюсный генератор с волновой обмоткой, разрезанной плоскостью чертежа перпендикулярно оси якоря. Здесь мы видим 12 пазов в якоре, в которых уложено 24 активных провода якорной обмотки. Передние (обращенные к нам) концы активных проводников при помощи соединитель- ных проводов (сплошные линии на рисунке) присоединены к ла- мелям коллектора: проводник 3 — к ламели 2, проводник 5 — к ламели 3 и т. д. Другие концы (задние) активных проводни- ков соединены друг с другом попарно (пунктирные линии на рисунке): проводник 1 — с проводником 8, проводник 7 — с про- водником 14 и т. д. Если, например, начать движение по об- мотке от проводника 2, находящегося под южным полюсом, рас- положенным вве^су, то отсюда через ламель 5 коллектора попа- дем в проводник 9, находящийся под северным полюсом, распо- ложенным справа, оттуда по соединительному проводу — в про- водник 16, находящийся под нижним южным полюсом, затем через .ламель 12 коллектора к проводнику 23, находящемуся под левым северным полюсом, далее по соединительному проводу — к проводнику 6, находящемуся под верхним южным полюсом, и т. д. Обходя таким образом последовательно всю обмотку, придем к исходному проводнику 2. По числу полюсов в машине имеются щетки, под которые поочередно набегают одна за дру- гой ламели коллектора при вращении якоря. Положительные и отрицательные щетки расположены поочередно по окружности коллектора. На .рис. 455 дана схема этой же волновой обмотки в развернутом виде. Якорь показан разрезанным по одной из образующих его цилиндрических поверхностей и затем разверну- тым на плоскости. Активные проводники изображены в виде ряда параллельно расположенных линий, пронумерованных от 1 до 24. Коллектор показан на рис. 455 внизу в виде длинной узкой ленты, разделенной на 12 ламелей. Ламели 1 и 7 контактируют с положительными щетками, а ламели 4 и 10 — с отрицательными. Это значит, что в следую- щий момент времени положительные щетки будут контактиро- вать с ламелями 12 и 6, а отрицательные — с ламелями 3 и 9 и т. д. Положительные щетки устанавливаются с таким расчетом, чтобы ток из якорной обмотки все время поступал через них во 44* 691
Рис. 455. Схема волновой обмотки четырехполюс- ного генератора в развернутом виде Рис. 456. Схема петлевой обмотки четырехпо- люсного генератора постоянного тока 692
внешнюю пень и через отрицательные щетки возвращался в якорную обмотку. На рис. 455 показана схема внешней цепи, подключенной к щеткам генератора (группа параллельно соеди- ненных электрических лампочек). Обмотка называется петлевой, если она на сердечник якоря наносится так, что провод, пройдя, например, под северным по- люсом, а затем и под соседним южным полюсом, возвращается под прежний северный полюс. На рис. 456 приведена схема че- тырехполюсного генератора с петлевой обмоткой, разрезанной плоскостью чертежа, перпендикулярной оси якоря. Если, напри- мер, начать обход обмотки с проводника /, расположенного под Рис. 457. Схема петлевой обмотки четырехпо- люсного генератора в развернутом виде верхним южным полюсом, то от него по соединительному про- воду (пунктирная линия) попадем в проводник 3, находящийся под правым северным полюсом, отсюда через ламель 2 коллек- тора к проводнику 3, находящемуся под верхним южным полю- сом, а от него по соединительному проводу к проводнику 10, на- ходящемуся под правым северным полюсом, и т. д. На рис. 457 показана схема петлевой обмотки в развернутом виде. В нашу задачу не входит подробное рассмотрение всех ти- пов обмоток, их конструктивного выполнения и расчета, а по- этому мы ограничимся вышеизложенным кратким описанием наиболее типичных форм обмоток — волновой и петлевой. По- дробное изложение теории обмоток можно найти в специальных учебниках по теории электрических генераторов постоянного тока. 693
I 208. РЕАКЦИЯ ЯКОРЯ Если генератор постоянного тока не нагружен током (холо- стой ход генератора), то магнитное поле, создаваемое током электромагнитов генератора, симметрично относительно оси по- люсов О = О и геометрической нейтрали .Oi = Oj (рис. 458). Если же генератор нагружен, то через его якорную юбмотку протекает ток, который создает свое собственное магнитное поле а Рис. 458. Магнитное поле, созда- ваемое электромагнитами гене- ратора Рис. 459. Магнитное поле тока якоря (рис. 459). Направление этого поля можно определить по пра- вилу буравчика. * Магнитное поле тока электромагнитов и магнитные поля тока якорной обмотки накладываются друг на друга и совместно образуют результирующее магнитное поле (рис. 460). Нетрудно видеть, что там, где якорь при своем вращении на- бегает на полюс электромагнита, результирующее поле слабее, чем там, где якорь сбегает с полюса электромагнита. Это объ- ясняется тем, что в первом случае основное магнитное поле, создаваемое током электромагнита, и магнитное поле тока якор- ной обмотки имеют различные направления, а во втором слу- чае— одинаковые. При отсутствии ~магнитного насыщения в стали генератора можно считать, что насколько основное поле уменьшилось с одной стороны полюса, настолько оно возросло 694
с другой. Следовательно, результирующий магнитный поток не изменился по величине. Однако по конфигурации результирующий магнитный поток значительно отличается от основного, создаваемого током элек- тромагнитов. В частности, изменилось распределение плотности магнитного потока в воздушном зазоре. Вследствие этого так называемая физическая нейтраль п = п генератора расположена под углом р к геометрической нейтрали Ох = О\ в сторону вра- щения якоря генератора. Чем больше нагружен генератор и чем больше магнитное на- сыщение его стали, тем сильнее проявляется реакция якоря, так как в этом случае происходит не только искажение магнитного потока генератора, но и некоторое уменьшение его. N Рис. 460. Результирующее магнит- ное поле генератора Рис. 461. Схема генератора с допол- нительными полюсами Реакция якоря оказывает вредное влияние на режим работы генератора, вызывая уменьшение его электродвижущей силы и искрение под щетками на коллекторе. Основными мерами борьбы с реакцией якоря являются: 1) применение в генераторах дополнительных полюсов (рис. 461), компенсирующих < магнитное поле тока якорной обмотки, и 2) сдвиг щеток с геометрической нейтрали О\ = О\ на физиче- скую нейтраль п = п (рис. 460), на некоторый угол р в сторону вращения якоря генератора, что предупреждает искрение под щетками на коллекторе. 695
§ 209. ЭЛЕКТРОДВИЖУЩАЯ СИЛА ГЕНЕРАТОРА ПОСТОЯННОГО ТОКА Допустим, что в двухполюсном магнитном поле, магнитный поток которого равен Ф, вращается якорь генератора с постоян- ным числом оборотов п. Число всех активных проводников якоря, т. е. таких, которые расположены на цилиндрической поверх- ности якоря и при движении последнего пересекают магнитный поток, равно 2. Среднее значение индуктированной электродвижущей силы в каждом из активных проводников якоря численно равно маг- нитному потоку, пересекаемому этим проводником за одну се- кунду. За один оборот якоря каждый из активных проводников пробежит под северным и под южным полюсами, т. е. он пересе- чет магнитный поток 2Ф. Если якорь совершает п оборотов в минуту, то в одну секунду он совершит ~ оборотов. Следовательно, среднее значение индуктированной электро- движущей силы в каждом из активных проводников якоря , Е= — -2Ф 22 60 где п— число оборотов якоря в минуту; Ф— магнитный поток полюсов в вольт-секундах; Е— индуктированная электродвижущая сила в вольтах. Активные проводники якоря генератора соединены последо- вательно друг с другом, а поэтому индуктированная электродви- жущая сила в них складывается. В двухполюсной машине имеется пара параллельных ветвей якорной обмотки, а поэтому средняя величина индуктированной электродвижущей силы в якорной обмотке равна р = — . 2Ф — 60 z 2 ’ ИЛИ £ = (566) где г — число., всех активных проводников на якоре генератора. Из формулы (566) следует, что средняя величина индукти- рованной электродвижущей силы в генераторе прямо пропор- циональна величине магнитного потока Ф, числу оборотов якоря п в минуту и числу s активных проводников якоря. Если многополюсный генератор имеет, например, 2р полюсов и якорная обмотка его состоит из 2а параллельных ветвей, то средняя величина индуктированной электродвижущей силы гене- ратора равна 696
или, иначе, оф Л с а ~ 60 • (567) Пример 216. Определить электродвижущую силу генератора, имеющего одну пару полюсов (р = 1) и одну пару параллельных ветвей (а=1) якор- ной обмотки. Число активных проводников якоря z — 192, число оборотов якоря 750 об/мин и величина магнитного потока Ф = 0,05 в-сек. Решение. По формуле (567) находим Е = Лгф ” = ’ .192.0,05-^-= 120 в. а 60 1 60 § 210. МОЩНОСТЬ И КОЭФФИЦИЕНТ ПОЛЕЗНОГО ДЕЙСТВИЯ ГЕНЕРАТОРОВ ПОСТОЯННОГО ТОКА j Полная электрическая мощность, развиваемая генератором постоянного тока, равна произведению электродвижущей силы Е генератора на величину полного тока 1а его якорной обмотки: Р = Е1а. Если поддерживать электродвижущую силу генератора по- стоянной, то полная электрическая мощность его будет пропор- циональна току 1а. Согласно формуле (567) электродвижущая сила генератора А это значит, что при прочих равных условиях полная элек- трическая мощность генератора растет с увеличением числа оборотов его якоря и увеличением числа полюсов его. Полезная мощность, отдаваемая генератором во внешнюю цепь, равна произведению напряжения U на зажимах генератора на величину тока /, посылаемого генератором во внешнюю цепь: Р, = где Р{— полезная мощность в ваттах; 7/—напряжение в вольтах; /—ток в амперах. Отношение полезной мощности Р\ к полной мощности Р, раз- виваемой генератором, называется электрическим коэффициентом полезного действия генератора: (568) где iq3— коэффициент полезного действия; F\—полезная мощность; Р— полная мощность. 697
Электрический коэффициент полезного действия (к. п. д.) ге- нератора зависит от его режима работы. Всякий генератор обычно конструируется так, что его к. п. д. максимален при нор- мальной его нагрузке, а поэтому всегда стремятся генератор за- грузить полностью. Наименьшим к. п. д. генератор обладает при холостом ходе, т. е. когда ток во внешней цепи равен нулю. Если генератор перегрузить, то его к. п. д. будет уменьшаться из-за резко возрастающих потерь на тепловую энергию в якор- ной обмотке и ряда побочных явлений, связанных с возросшим якорным током. Мощные генераторы, как правило, обладают значительно большими к. п. д. по сравнению с маломощными. Среднее значение к. п. д. генератора при нормальной нагрузке равно примерно 90%. При уменьшении нагрузки к. п. д. генера- тора резко падает: например, если генератор заставить работать при 74 его нормальной нагрузки, то к. п. д. генератора снизится примерно до 50 %. Промышленным коэффициентом полезного действия назы- вают отношение полезной мощности, развиваемой генератором, к той механической мощности, которую развивает двигатель на своем валу, вращая якорь генератора: чпром р > где Чпром — промышленный коэффициент полезного действия; —полезная мощность, развиваемая генератором; Р— механическая мощность, развиваемая первичным двигателем на валу. Пример 217. Определить промышленный к. п. д. генератора, если пер- вичный двигатель, приводящий его в движение, развевает на своем валу механическую мощность Р=12 л. с., а генератор, имеющий на своих зажи- мах напряжение U = 120 в, питает внешнюю цепь током / = 55,2 а. Решение. Механическая мощность, развиваемая первичным двигате- лем, Р = 12 • 736 =8832 вт. Полезная электрическая мощность, развиваемая генератором, PX = U1 = 120-55,2 = 6624 вт. Промышленный к. п. д. __ А _ 6624 _Л7- ’InpoM - р - 8832 - и’/э- Промышленный к. п. д., кроме электрических потерь в гене- раторе, учитывает все механические и магнитные потери, а по- этому он меньше, чем электрический к. п. д. 698
§ 211. КЛАССИФИКАЦИЯ ГЕНЕРАТОРОВ ПОСТОЯННОГО ТОКА ПО СПОСОБУ ИХ ВОЗБУЖДЕНИЯ И ПОНЯТИЕ ОБ ИХ ОСНОВНЫХ ХАРАКТЕРИСТИКАХ Генераторы постоянного тока подразделяются по способу их возбуждения на генераторы с .независимым возбуждением и са- мовозбуждением. Характерная особенность генераторов с независи- мым возбуждением — та, что их цепь возбуждения пи- тается постоянным током от постороннего источника электриче- ской энергии. Наоборот, генераторы с самовозбужде- нием сами питают постоянным током свою цепь возбуждения. Генераторы постоянного тока с самовозбуждением в свою очередь подразделяются на генераторы с параллельным, после- довательным и смешанным возбуждением. Генератор с параллельным возбуждением имеет обмотку возбуждения, подключенную параллельно к якор- ной обмотке. Генератор с последовательным возбужде- нием имеет обмотку возбуждения, подключенную последова- тельно с якорной обмоткой. Генератор со смешанным возбуждением имеет две обмотки возбуждения, из которых одна подключена параллельно, а другая — последовательно с якорной обмоткой. Основными величинами, характеризующими работу генерато- ров постоянного тока, являются: электродвижущая сила Е, на- пряжение на зажимах нагрузка (ток, даваемый генератором потребителю) /, ток возбуждения zB и номинальное число обо- ротов п якоря генератора в минуту. Однако весьма важно бывает знать не только основные ве- личины, характеризующие работу генератора, но и зависимость их друг от друга. Зависимость между какими-либо двумя основными величи- нами, характеризующими работу генератора, называется х а- рактеристикой генератора. Характеристики генератора обычно изображаются графически в виде некоторых кривых. Важнейшие характеристики генератора постоянного тока сле- дующие: 1) характеристика холостого хода, 2) внешняя харак- теристика и 3) регулировочная характеристика. Характеристика холостого хода генератора выражает зависимость его электродвижущей силы Е от тока возбуждения zB при постоянстве числа оборотов якоря генера- тора в минуту (n= const) и нагрузке, равной нулю (/ = 0). Внешняя характеристика генератора постоян- ного тока выражает зависимость напряжения U на зажимах ге- нератора от его нагрузки / при п = const и постоянном сопро- тивлении цепи возбуждения (rB = const). 699
Регулировочная характеристика генера- тора постоянного тока выражает зависимость тока возбужде- ния 4 от нагрузки I генератора при постоянстве напряжения (t/= const) на зажимах генератора и при п = const. В последующих параграфах мы рассмотрим характеристики основных типов генераторов постоянного тока. § 212. ГЕНЕРАТОР ПОСТОЯННОГО ТОКА С НЕЗАВИСИМЫМ ВОЗБУЖДЕНИЕМ I и V Рис. 462. Принци- пиальная схема ге- нератора постоян- ного тока с неза- висимым возбуж- дением На рис. 462 показана принципиальная схема генератора по- стоянного тока с независимым возбуждением с подключенным к нему потребителем. Обмотка возбуждения генератора не связана электрически с его якорной обмоткой. Она питается током 4 от постороннего источника электрической энергии, например от аккумуляторной батареи. Последовательно с обмоткой возбуждения, имеющей со- противление /*м, подключен регулировочный реостат гв, назы- ваемый реостатом возбуждения. Он изменяет величину тока возбуждения 4 в Дели и тем самым регу- лирует электродвижущую силу Е’генератора. К щеткам (+) и (—) генератора подведе- ны соединительные провода, связывающие ге- нератор с потребителем, например с электри- ческими лампочками. Амперметр А и вольтметр V дают возмож- ность наблюдать соответственно за нагрузкой I генератора и напряжением U на его зажи- мах. Характеристика холостого хода генерато- ра. Характеристикой холостого хода генера- тора называют зависимость электродвижущей силы Е генератора от тока возбуждения 4 при постоянстве числа оборотов п якоря в, ми- нуту и нагрузке генератора, равной нулю (/ = 0). Для снятия характеристики холостого хода генератора необходимо отсоединить его от внешней цепи и затем сообщить его якорю но- минальное число оборотов пп в минуту. Затем, изменяя с помощью реостата возбуждения гв ток возбуждения 4 генератора, можно снять кривую зависимости электродвижущей силы Е от тока возбуждения 4- Вольтметр, включенный в данную цепь (рис. 462), покажет напряжение на зажимах генератора. Но так как ток I в якоре генерато- ра при холостом ходе равен нулю, то напря- 700
жение на зажимах генера- тора в этом случае дол- жно быть равно электро- движущей силе Е генера- тора, т. е. и — Е, где £7Х. х — напряжение холостого хода на зажи- мах генератора. На рис. 463 показана характеристика холостого хода генератора постоян- ного тока с независимым возбуждением. Здесь мы Рис. 463. Характеристика холостого хода генератора с независимым возбуждением видим, что при токе воз- буждения, равном нулю (4 =0), электродвижущая' сила гене- ратора не равна нулю, она составляет несколько процентов но- минального значения электродвижущей силы генератора. Эта электродвижущая сила, называемая начальной или оста- точной электродвижущей силой, обусловлена нали- чием остаточного магнетизма в магнитной цепи гене- ратора. При вращении якоря генератора в магнитном поле оста- точного магнетизма в якорной обмотке индуктируется начальная (остаточная) электродвижущая сила £*ост. Затем по мере увели- чения тока возбуждения 4 генератора электродвижущая сила растет, изменяясь согласно крийой, напоминающей кривую на- магничивания ферромагнитных материалов. Электродвижущая сила Е генератора вначале растет весьма быстро (прямолинейный участок ab характеристики), изменяясь по линейному закону. Это объясняется тем, что при малых вели- чинах тока возбуждения сталь генератора весьма слабо намаг- ничена, ее магнитное сопротивление мало из-за относительно большой магнитной проницаемости стали. Ампер-витки обмотки возбуждения генератора в этом случае расходуются главным образом в воздушном зазоре магнитной цепи генератора (при- мерно 80% всех ампер-витков). В силу этого между магнитным потоком Ф генератора и током возбуждения 4 существует ли- нейная зависимость. А так как электродвижущая сила Е генера- тора при постоянстве числа оборотов его якоря в минуту (n = const) пропорциональна величине магнитного йотока: Е = СпФ, то электродвижущая сила Е генератора при малых токах воз- буждения 4 изменяется пропорционально величине тока. 701
Однако при дальнейшем увеличении тока возбуждения /в линейная зависимость между* ним и электродвижущей силой Е генератора нарушается. Дело в том, что по мере возрастания тока возбуждения начинает сказываться явление магнитного насыщения стали. В соответствии с этим начинает умень- шаться магнитная проницаемость р. стали генератора, а магнит- ное сопротивление ее увеличиваться. Следовательно, ампер-витки обмотки возбуждения теперь уже значительно больше расхо- дуются в стали, чем до момента магнитного насыщения (при- мерно 40% общего числа ампер-витков). В силу этого зависи- мость между магнитным потоком Ф и, следовательно, между электродвижущей силой Е генератора и током возбуждения гв уже становится нелинейной (криволинейный участок Ьс характе- ристики). При дальнейшем увеличении тока возбуждения zB в стали генератора возникает сильное магнитное насыщение. В этом слу- чае магнитная проницаемость стали рь становится небольшой, а магнитное сопротивление стали, наоборот, возрастает. Теперь уже требуется, много ампер-витков, чтобы поддержать необходи- мый магнитный поток Ф и, следовательно, значительно увели- чить ток возбуждения iB генератора. В этом случае магнитный поток и соответственно электродвижущая сила генератора из- меняются в зависимости от тока возбуждения вновь по линей- ному закону (прямолинейный участок cd). Однако -быстрота роста электродвижущей силы Е теперь заметно меньше и соот- ветственно прямолинейный участок cd характеристики холостого хода получается более пологим, чем ее прямолинейный уча- сток ab. * Характеристика холостого хода генератора с независимым возбуждением имеет важное значение при изучении режима ра- боты генератора. Например, зная характеристику холостого хода, можно выяснить магнитное состояние стали генератора и, сооб- разуясь с этим, выбрать для него наиболее оптимальный режим работы. По крутизне подъема характеристики холостого хода при малых значениях тока возбуждения можно судить о том, как распределяются ампер-витки обмотки возбуждения между сталью и воздушными зазорами. Чем меньше крутизна подъема характеристики холостого хода, тем, следовательно, относительно больше ампер-витков расходуется в воздушных зазорах генера- тора. Внешняя характеристика генератора. Когда генератор нор- мально возбужден, т. е. при нормальном числе оборотов якоря в минуту имеет номинальную электродвижущую силу, его можно нагрузить током, подключив к нему потребители электрической энергии. Нагруженный генератор создает в цепи ток гя + Я> 702
где' I — нагрузка генератора в амперах; Е—электродвижущая сила генератора в вольтах; гя — сопротивление якорной обмотки в омах; R—эквивалентное сопротивление внешнего участка цепи (потребителей). Рассмотрим внешнюю характеристику генератора. Как известно, внешней характеристикой генератора назы- вается зависимость напряжения Ц на его зажимах от его на- грузки I при п = const и гв = const. Напряжение на зажимах генератора и = Е — 1гл, (569) т. е. оно равно электродвижущей силе Е генератора без падения напряжения 1гл в якорной обмотке генератора. Из формулы (569) следует, что величина напряжения U на зажимах генератора находится в сложной зависимости от на- грузки /, так как сама электродвижущая Е зависит от величины нагрузки. При токе I = 0 (режим холостого хода) напряжение на за- жимах генератора равно его электродвижущей силе: t7 = t/x<x = £. С ростом нагрузки / генератора напряжение на его зажимах начинает уменьшаться. Это объясняется тем, что с ростом тока / увеличивается падение напряжения /гя в якорной обмотке гене- ратора’и одновременно с этим уменьшается электродвижущая сила генератора из-за влияния реакции якоря, снижающей маг- нитный поток генератора. На рис. 464 приведена внешняя характеристика генератора с независимым возбуждением. Так как- сопротивление гя якорной обмотки генератора от- носительно мало (десятые или сотые доли ома), а влияние реак- ции якоря обычно слабо благодаря применению дополнительных магнитных полюсов, то напряжение U на зажимах генератора в режиме от холостого хода до номинальной нагрузки изме- няется в пределах 5—10% номинальной величины. Внешняя характеристика генератора весьма важна в том отношении, что она дает представление о том, насколько устой- чиво напряжение на зажимах генератора при изменении его на- грузки и при п = const и rB = const. Для поддержания постоянства напряжения U на зажимах генератора при изменении нагрузки / пользуются реостатом воз- буждения. Увеличивая или уменьшая ток возбуждения /в гене- ратора путем изменения сопротивления гв реостата возбужде- ния, соответственно увеличивают или уменьшают электродвижу- щую силу генератора и, следовательно, напряжение на его за- жимах. 703
U(6) Рис. 464. Внешняя характеристика генератора с независимым возбуждением Регулировочная характеристика. Для поддержания постоян- ства напряжения U на зажимах генератора при изменении на- грузки / необходимо изменять ток возбуждения 4 генератора. Зависимость тока возбуждения 4 генератора от нагрузки / при U = const и п = const называется регулировочной характе- ристикой генератора. На рис. 465 приведена регулировочная характеристика гене- ратора постоянного тока с независимым возбуждением. При хо- лостом ходе (I = 0) генератор имеет минимальный ток возбуж- дения 40. Затем по мере роста нагрузки I генератора ток воз- буждения 4 также растет. Это понятно, так как для поддер- жания постоянства напряжения U на зажимах генератора не- обходимо увеличивать электродвижущую силу Е его, что и до- стигается увеличением тока возбуждения 4- Чем больше магнитное насыщение стали генератора, тем при прочих одинаковых усло- виях круче поднимается график регулировочной ха- рактеристики генератора. Это объясняется тем, что в данном случае с ростом тока в якорной обмотке усиливается размагничи- вающее действие реакции якоря и для компенсации его необходимо затратить большее число ампер-вит- ков обмотки возбуждения. Достоинство генерато- ров постоянного тока с не- Рис. 465. Регулировочная характеристика генератора с независимым возбуждением 704
зависимым возбуждением заключается в их хорошей внешней характеристике, так как ток возбуждения 4 генератора незави- сим от напряжения на зажимах генератора. Недостаток таких генераторов — необходимость иметь посторонний источник элек- трической энергии, питающий постоянным током обмотку возбу- ждения генератора. Генераторы постоянного тока с независимым возбуждением применяются главным образом в мощных сильноточных уста- новках. Для питания обмоток возбуждения этих генераторов обычно применяются отдельные генераторы постоянного тока низкого напряжения. § 213. ГЕНЕРАТОР С ПАРАЛЛЕЛЬНЫМ ВОЗБУЖДЕНИЕМ В отличие от генератора с независимым возбуждением гене- ратор с параллельным возбуждением относится к электрическим машинам с самовозбуждением. Это значит, что генератор дан- ного типа сам питает током свою обмотку возбуждения и не нуж- дается в постороннем источнике электрической энергии. На рис. 466 показана схема генератора постоянного тока с параллельным возбуждением. Обмотка возбуждения генера- тора подключена параллельно к якорной обмотке генератора. Для регулировки напряжения на зажимах генератора приме- няются реостаты возбуждения (шунтовые реостаты), включенные последовательно с обмотками воз- буждения. Эти реостаты, как правило, снаб- —w— жаются холостыми контактами, при по- мощи которых можно осуществить короткое за- мыкание обмотки возбуждения на себя. Послед- нее необходимо при выключении обмотки возбу- ждения. Если выключить обмотку возбуждения путем разрыва ее цепи, то исчезающее магнитное поле электромагнита создаст в обмотке очень большую электродвижущую силу самоиндукции, которая сможет пробить витки обмотки и вывести генератор из строя. Если же при выключении обмотки возбуждения замкнуть ее на себя через холостой контакт, то энергия исчезающего маг- нитного поля без вреда для генератора будет израсходована в его обмотке возбуждения на тепло. В этом случае электродвижущая сила самоиндукции в обмотке возбуждения не пре- высит номинального напряжения на зажимах генератора. Самовозбуждение генератора. Процесс само- Рис-466- Сх^ма возбуждения генератора с параллельным возбу- с параллельным ждением происходит следующим образом. возбуждением 45—1377 705
Сердечники электромагнитов генератора изготовлены из ли- той стали, а поэтому в них всякий раз после прекращения ра- боты генератора сохраняется некоторый остаточный магнетизм. Поэтому при вращении якоря генератора в магнитном поле остаточного магнетизма индуктируется небольшая электродвижу- щая сила, составляющая 2-4-5% номинальной электродвижущей силы генератора. Так как обмотка возбуждения генератора подключена к его зажимам, то в ней под влиянием индуктированной электродви- жущей силы возникает небольшой ток возбуждения, который со- здает дополнительный магнитный поток в генераторе. Если об- мотка подключена правильно к зажимам генератора, то этот Рис. 467. Характеристика холостого хода гене- ратора с параллельным возбуждением поток направлен попутно с магнитным потоком остаточного маг- нетизма. В результате этого индуктированная электродвижущая сила генератора увеличивается, а с ней вместе возрастает ток возбуждения, способствующий дальнейшему увеличению элек- тродвижущей силы, и т. д. Казалось бы, процесс нарастания электродвижущей силы Е генератора (процесс самовозбуждения генератора) должен не- прерывно прогрессировать, т. е. электродвижущая сила генера- тора-должна неограниченно'возрастать. Однако этого не наблю- дается, всякий раз рост индуктированной электродвижущей силы генератора при его самовозбуждении ограничен тем или иным пределом. Чтобы уяснить это, необходимо рассмотреть характери- стику холостого хода генератора. . Характеристика холостого хода. На рис. 467 приведена ха- рактеристика холостого хода генератора с параллельным возбу- 706
ждением, т. е. кривая зависимости напряжения U на зажимах от тока возбуждения zB генератора при п = const и rB = const. Одновременно с этим на рис. 467 показан график зависи- мости падения напряжения UB в цепи возбуждения генератора от тока возбуждения iB. Эта зависимость линейна, так как 67в = iB (rM + rB), где rM + — сопротивление цепи возбуждения (rM + rB = const). Из рис. 467 видно, что при малых величинах тока возбужде- ния zB электродвижущая сила Е больше падения напряжения 67в в обмотке возбуждения: E>UB. Значит, в этом случае генератор питает током свою обмотку возбуждения. В соответствии с этим в генераторе происходит нормальный процесс самовозбуждения, т. е. с ростом тока возбуждения iB растут электродвижущая сила Е и напряже- ние £>в на обмотке возбуждения, что в свою очередь влечет за собой увеличение тока возбуждения 4 и т. Д- Однако быстрота роста электродвижущей силы Е и напряжения 67в, как это видно из рис. 467, различна. По мере увеличения тока возбуждения скорость роста электродвижущей силы Е спадает, а скорость роста напряжения С7В сохраняется неизменной. При некоторой величине тока возбуждения i'B напряжение 67' становится рав- ным электродвижущей силе Е': U',=E'. При токе возбуждения, равном ZB, графики электродвижущей силы Е и напряжения 67в пересекаются, а при дальнейшем росте тока возбуждения они расходятся. В последнем случае электро- движущая сила Е становится меньше напряжения 67в. Однако практически этого не бывает, так как напряжение 67в, как часть электродвижущей силы Е, не может быть больше нее. Следовательно, значение тока возбуждения zB — это предель- ная величина его при заданном числе оборотов якоря генератора п = const и постоянстве сопротивления цепи возбуждения rM = const. Угол наклона прямой, выражающей зависимость напряжения UB на зажимах генератора от тока возбуждения iB (рис. 467), зависит от сопротивления цепи возбуждения и, следовательно, от сопротивления гв шунтового реостата, имеющегося в цепи воз- буждения. Чем больше это сопротивление, тем круче подни- мается прямая зависимости 67в от iB и тем при меньшем токе возбуждения произойдет пересечение графиков зависимости 67в и £ от тока возбуждения zB. Процесс самовозбуждения генератора с параллельным возбу- 45* 707
Рис. 468. Внешняя характеристика генератора с параллельным возбуждением ждением длится до тех пор, пока ток возбуждения 1„ не достигает некоторой предельной величины при заданных нормальных оборо- тах якоря генератора и электродвижущая сила Е не становится равной номинальному своему значению. Обычно процесс само- возбуждения генератора длится доли минуты. Если обмотка возбуждения генератора подключена непра- вильно к якорной обмотке, то генератор не возбудится,.так как ток возбуждения создает магнитный поток, направленный навстречу остаточному магнитному потоку, и машина размагнитится. Внешняя характеристика генератора. На рис. 468 показана внешняя характеристика генератора с параллельным возбужде- нием, т. е. зависимость напряжения U на его зажимах от на- грузки I при постоянстве числа оборотов п якоря генератора и постоянном сопротивлении его цепи возбуждения. При холостом ходе генератора напряжение на его зажимах максимально (£7Х. х). Затем с ростом нагрузки / генератора напряжение U на его за- жимах начинает падать, и несколько'быстрее, чем у генератора с независимым возбуждением. Это объясняется тем, что в дан- ном случае напряжение U уменьшается не только в результате возрастающего влияния реакции якоря и падения напряжения в якорной обмотке, но и за счет того, что с уменьшением напря- жения на зажимах генератора уменьшается его ток возбужде- ния zB и в соответствии с этим снижается величина самой элек- тродвижущей силы Е. При непрерывном уменьшении сопротивления потребителя и, следовательно, увеличения нагрузки I напряжение U на зажи- мах генератора непрерывно возрастает. Однако вскоре наступает режим, при котором дальнейшее уменьшение сопротивления потребителя приводит не к увеличе- нию нагрузки / генератора, а, наоборот, к уменьшению ее, 708
Предельное значение тока генератора при непрерывном уменьшении сопротивления потребителя называется критиче- ским током генератора (рис. 468). Величина его при- мерно 2—2,5 раза больше номинального значения тока генера- тора. В пределе, при коротком замыкании генератора, т. е. когда внешнее сопротивление станет равным нулю, напряжение на за- жимах генератора также станет равным нулю, а ток генератора станет равным току короткого, замыкания генератора. Однако он большой опасности для генератора не представляет, так как его величина не превышает номинального тока генератора. Это объ- ясняется тем, что он существует за счет небольшой электродви- жущей силы генератора, индуктированной под действием оста- точного магнитного поля. Переход генератора через режим критического тока сопрово- ждается сильным искрением под щетками коллектора из-за чрез- мерной перегрузки генератора и поэтому нежелателен. У генераторов с параллельным возбуждением, не имеющих добавочных полюсов, изменение напряжения U в режиме от холостого хода до нормальной нагрузки составляет 12—20%, а у генераторов с добавочными полюсами — 8—15% номинального напряжения. Регулировочная характеристика. На рис. 469 изображена ре- гулировочная характеристика генератора с параллельным возбу- ждением, т. е. график зависимости тока возбуждения 4 от на- грузки / генератора при U = const и п = const. Эта характеристика практически мало отличается от анало- гичной характеристики генератора с независимым возбуждением. При одинаковой их нагрузке ток в якорной обмотке генератора с параллельным возбуждением больше, чем ток в якорной об- мотке генератора с независимым возбуждением, на величину тока возбуждения первого из них. Поэтому в генераторе с па- раллельным возбуждением * при всех прочих одинако- вых условиях падение на- пряжения в якорной обмот- ке генератора и реакция якоря больше, что требует большего тока возбуждения. Следовательно, регулировоч- ная характеристика должна подниматься круче, чем у ге- нератора с независимым воз- буждением. Одно из существенных достоинств генераторов по- стоянного тока с параллель- ным возбуждением — то, что Рис. 469. Регулировочная характери- стика генератора с параллельным воз- буждением 709
возрастает. В результате резко увеличивается падение напряжения в якорной обмотке, что в свою очередь сильно снижает напряжение на зажимах генератора, ток возбуждения, электродвижущую силу генератора и ток в якорной обмотке. Все эти процессы совер- шаются настолько быстро, что весьма кратковременный крити- ческий ток не успевает перегреть провода якорной обмотки. Другое важное преимущество генератора с параллельным возбуждением перед генератором с независимым возбужде- нием — отсутствие необходимости в первом из них иметь от- дельный источник постоянного тока, питающий обмотку возбу- ждения генератора. Генераторы постоянного тока с параллельным возбуждением получили широкое применение на практике, и в частности в тех- нике связи для зарядных агрегатов, для питания радиоустановок и т. д. § 214. ГЕНЕРАТОР С ПОСЛЕДОВАТЕЛЬНЫМ ВОЗБУЖДЕНИЕМ Генератор с последовательным возбуждением имеет обмотку возбуждения, включенную последовательно с якорной обмоткой (рис. 470). Ток в обмотке возбуждения ZB равен току (нагрузке) / гене- ратора: 4 = /. Рис. 470. Схема генера- тора с последовательным возбуждением Так как нагрузка I генератора с по- следовательным возбуждением при холо- стом ходе его равна нулю, то и ток воз- буждения zB его равен нулю. Следова- тельно, снять характеристику холостого хода генератора, т. е. зависимость напря- жения U на зажимах генератора от тока возбуждения при п = const, в данном генераторе невозможно. Напряжение на зажимах генератора с последовательным возбуждением при хо- лостом ходе составляет несколько про- центов номинального напряжения генера- тора, так как оно обусловлено действием магнитного поля остаточного магнетизма стали генератора. Следовательно, чтобы возбудить гене- ратор, необходимо присоединить к нему внешнюю цепь (потребители), этим са- мым создав условие для возникновения тока в обмотке возбуждения. 719
Рис. 471. Внешняя характеристика генератора с по- следовательным возбуждением Внешняя характеристика генератора с последовательным воз- буждением приведена на рис. 471. Напряжение U на зажимах генератора вначале растет вместе с нагрузкой I генератора (уча- сток кривой оа), а затем начинает уменьшаться. Это объяс- няется следующим. Вначале с ростом нагрузки / растет и ток возбуждения zB, так как 4= Л & следовательно, растут электро- движущая сила Е и напряжение U на зажимах генератора. Однако затем по мере увеличения нагрузки I генератора напря- жение U на его зажимах начинает спадать, потому что падение напряжения 1гя внутри якорной обмотки становится все более ощутимым. Кроме того, электродвижущая сила Е генератора по мере магнитного насыщения стали генератора увеличивается очень мало, а поэтому, невзирая на ее некоторый рост, напряже- ние на зажимах генератора после некоторой предельной на- грузки I начинает уменьшаться. Регулировочную характеристику генератора с последователь- ным возбуждением снять нельзя, потому что при изменении на- грузки генератора невозможно подбирать ток его возбуждения так, чтобы сохранить напряжение на зажимах генератора по- стоянным по своей величине. Основной недостаток генератора с последовательным возбу- ждением — резко выраженная зависимость напряжения U на его зажимах от нагрузки I. Из-за этого генераторы с последователь- ным возбуждением очень редко применяются на, практике, так как громаднейшее большинство потребителей электрической энергии требует для своей нормальной работы строго опреде- лённого номинального напряжения. Генераторы с последова- тельным возбуждением могут применяться только в условиях строгого постоянства нагрузки. § 215. ГЕНЕРАТОР СО СМЕШАННЫМ ВОЗБУЖДЕНИЕМ На рис. 472 приведена схема цепи, состоящей из генератора со смешанным возбуждением и подключенных к нему потреби- телей. Генератор имеет две обмотки возбуждения, из которых 711
Рис. 472. Схема генератора со смешанным возбуждением одна, состоящая из большого числа витков тон- кой проволоки, подключена параллельно к якор- ной обмотке генератора, а другая, состоящая из небольшого числа витков относительно толстой проволоки,— последовательно. В цепь обмотки параллельного возбуждения включен реостат воз- буждения, с помощью которого регулируется ток возбуждения в этой обмотке. Наличие параллельной и последовательной обмоток возбуждения в генераторе дает возмож- ность сочетать в нем в необходимой мере харак- теристики генераторов с параллельным и после- довательным возбуждением. Характеристика холостого хода. Так как гене- ратор при холостом ходе отключен от внешней цепи (от потребителей), то нагрузка его равна нулю (1 = 0) ив соответствии с этим ток возбу- ждения в последовательной обмотке возбужде- ния также равен нулю. Следовательно, характе- ристика совершенно аналогична характеристике холостого хода генератора с параллельным воз- буждением (рис. 473). Процесс самовозбуждения генератора со смешанным возбу- ждением такой же, как и в генераторе с параллельным возбу- ждением, так как последовательная обмотка возбуждения не принимает участия в самовозбуждении генератора из-за отсут- ствия в ней в это время тока возбуждения. Внешняя характеристика. Вид внешней характеристики гене- ратора со смешанным возбуждением зависит от соотношения ампер-витков обмоток параллельного и последовательного воз- Рис. 473. Характеристика холостого хода генератора со смешанным возбуждением 712
буждения и от направления магнитных потоков, создаваемых этими ампер-витками. На рис. 474 приведено несколько типичных внешних харак- теристик генератора со смешанным возбуждением. При холостом ходе генератора, т. е. при I = 0, напряжение на его зажимах равно номинальной величине £7Н. Оно получено в результате действия ампер-витков параллельной обмотки возбуждения в процессе самовозбуждения генератора. Затем, начиная с момента включения нагрузки и последующего роста ее, напряжение на зажимах генератора начинает изменяться. Это изменение напря- жения U в зависимости от нагрузки / происходит для различных генераторов по-разному, так как оно зависит от соотношения ам- пер-витков последовательной и параллельной обмоток генера- тора. Например, внешняя характеристика 1 относится к такому Рис. 474. Внешние характеристики генератора со смешанным возбуждением генератору со смешанным возбуждением, у которого решающее влияние на внешнюю характеристику имеют ампер-витки после- довательной обмотки возбуждения. Здесь при холостом ходе генератора (Z =='0) напряжение на его зажимах имеет некоторое номинальное значение с7й, а затем с ростом нагрузки I оно на- чинает несколько увеличиваться за счет преобладающего дей- ствия последовательной обмотки возбуждения и достигает мак- симума при нагрузке меньше номинальной. По мере приближе- ния нагрузки к номинальной /н напряжение U начинает спа- дать. И когда ток / достигает номинального значения, напряже- ние U все же несколько выше номинального. Генераторы с по- добного рода внешними характеристиками обычно применяются при передаче электрической энергии по длинным линиям. При- рост напряжения на зажимах генератора при номинальной его нагрузке в этом случае предусмотрен таким, чтобы скомпенсиро- 713
вать возросшее падение напряжения в проводах и этим сохра- нить постоянство напряжения у потребителей, включенных в конце длинной линии. Внешняя характеристика' 2 относится к такому генератору, у которого напряжение на зажимах при номинальной нагрузке такое же по величине, как и при холостом ходе. У генератора подобного рода ампер-витки последовательной обмотки возбу- ждения подобраны таким образом, что напряжение на зажимах генератора вначале несколько растет, а затем, достигнув макси- мума, спадает и при номинальной нагрузке становится равным номинальному напряжению. Чем меньше влияние ампер-витков последовательной обмотки возбуждения по сравнению с влиянием ампер-витков парал- лельного возбуждения, тем больше внешняя характеристика ге- нератора со смешанным возбуждением приближается к анало- гичной характеристике генератора с параллельным возбужде- нием. Внешняя характеристика 3 относится как раз к такому ге- нератору со смешанным возбуждением, у которого решающее влияние на характер внешней характеристики оказывает парал- лельная обмотка возбуждения. Если последовательная обмотка возбуждения генератора со смешанным возбуждением включена таким образом, что она со- здает магнитное поле, направленное навстречу магнитному полю, создаваемому параллельной обмоткой возбуждения, то внешняя характеристика генератора принимает вид графика 4. Здесь мы видим, что при холостом ходе генератора напряжение на его за- жимах равно номинальному напряжению £7Н, полученному в ре- зультате действия ампер-витков параллельной обмотки возбу- ждения. Если генератор постепенно нагружается, то вступают в действие ампер-витки последовательной обмотки возбужде- ния, в результате чего напряжение на зажимах генератора на- чинает спадать. Это объясняется тем, что магнитный поток, со- здаваемый ампер-витками последовательной обмотки возбужде- ния, направлен навстречу магнитному потоку, создаваемому параллельной обмоткой. По мере увеличения нагрузки генера- тора и соответственно тока возбуждения последовательной об- мотки быстрота убывания напряжения на зажимах генератора все более и более возрастает. Это в конце концов приводит к тому, что нагрузка генератора еще не успевает достигнуть но- минальной величины, как напряжение на зажимах генератора уже становится равным нулю. Генераторы с встречным включением последовательной и па- раллельной обмоток возбуждения применяются в том случае, когда от генератора желают получить постоянный по величине ток при изменении сопротивления внешней цепи ^от некоторого конечного значения до нуля (до короткого замыкания). Приме- ром подобного рода потребителя может служить электро- сварочный аппарат, в котором сопротивление при работе может 714
изменяться от некоторого конечного значения практически до нуля. Регулировочная характеристика. На рис. 475 представлена регулировочная характеристика генератора со смешанным воз- буждением. При холостом ходе генератора ток возбуждения равен неко- торой величине 4о> а затем с ростом нагрузки I генератора он начинает спадать. Это понятно, так как с появлением нагрузки I генератора вступает в дей- ствие последовательная об- мотка возбуждения, магнит- ное поле которой действует согласно с магнитным полем параллельной обмотки воз- буждения. Следовательно, для поддержания постоян- ства результирующего маг- нитного потока, а значит, и постоянства напряжения на зажимах генератора необхо- димо вначале несколько сни- жать ток возбуждения 4 в параллельной обмотке возбу- ждения. Однако по мере роста на- грузки генератора и прибли- жения ее к номинальной ток расти. Это объясняется тем, Рис. 475. Регулировочная характери- стика генератора со смешанным воз- буждением с возбуждения генератора начинает что с ростом нагрузки генератора усиливается влияние реакции якоря, увеличивается падение на- пряжения в якорной обмотке и возникает магнитное насыщение в стали генератора. А все это, как известно, снижает напряжение на зажимах генератора. Следовательно, в данном случае для поддержания постоянства напряжения на зажимах генератора необходимо увеличивать ток в параллельной обмотке возбужде- ния. В результате этого ток возбуждения 4 параллельной об- мотки возбуждения достигает при номинальной нагрузке /н гене- ратора величины, примерно равной току возбуждения при холо- стом ходе генератора. Основное преимущество генераторов со смешанным возбу- ждением перед прочими типами генераторов постоянного тока — их способность поддерживать практически постоянным напряже- ние на своих зажимах при изменении нагрузки в широких пре- делах. К недостаткам генераторов со смешанным возбуждением следует отнести их боязнь коротких замыканий и сложность конструкций из-за наличия в них последовательной и параллель- ной обмоток возбуждения. 715
Благодаря своей хорошей внешней характеристике генера- торы со смешанным возбуждением получили достаточно широ- кое применение на практике. § 216. ПАРАЛЛЕЛЬНАЯ РАБОТА ГЕНЕРАТОРОВ ПОСТОЯННОГО ТОКА Как известно, каждый генератор электрической энергии ра- ботает с наибольшим коэффициентом полезного действия при полной своей нагрузке. Следовательно, если генератор нагружен слабо, то режим работы его мало экономичен. Для экономич- ной работы генераторов в сети с сильно изменяющейся нагруз- кой ее обычно питают от нескольких генераторов, соединенных Нагрузка Рис. 476. Схема включения на параллельную ра- боту двух генераторов с параллельным возбу- ждением параллельно друг с другом. В те часы суток, когда нагрузка велика, например нагрузка осветительной сети в вечернее время, одновременно работает несколько генераторов с полной нагруз- кой, а в те часы, когда нагрузка резко снижается, например в дневное время, работает только какой-либо один из генерато- ров на полную нагрузку. Одна из основных задач электрической станции состоит в правильной регулировке 'работы генераторов в соответствии с потребностями приемников электрической энергии. На рис. 476 приведена схема параллельного соединения двух шунтовьгх генераторов, питающих сеть. В схеме генераторов есть амперметры, измеряющие нагрузку генераторов. При помощи кабеля генераторы через предохранители и рубильники присо- единены к распределительным шинам параллельно, т. е. их по- 716
ложительные зажимы присоединены, к одной шине, положитель- ной (+), а отрицательные — к другой, отрицательной (—). Чтобы можно было быстро отличить положительную шину от от- рицательной, их обычно окрашивают в разные цвета, например положительную в синий, а отрицательную в красный. Шины из- готовляются из медных полос прямоугольного сечения, размеры которого зависят от мощности всей установки. Непосредственно к шинам через рубильник и предохранители при помощи кабеля подключается внешняя цепь. Вольтметр магнитоэлектрической системы и специальный переключатель дают возможность измерять напряжение на зажимах каждого из генераторов и полярность каждого генератора. На параллельную работу генераторы включают следующим образом. Допустим, что один из генераторов, который назовем первым, работает на сеть. В результате возросшей нагрузки он оказался перегруженным и возникла необходимость включить второй гене- ратор. Для этого после предварительного внешнего осмотра ге- нератора его запускают в ход, добиваясь нормального числа обо- ротов и нормального напряжения на зажимах. Когда при по- мощи магнитоэлектрического вольтметра убедятся в том, что напряжение на зажимах второго генератора равно напряжению на зажимах первого генератора и что полярность его соблюдена, замыкают рубильник второго генератора, подключая его к рас- пределительным шинам. Так как электродвижущая сила второго генератора точно равна напряжению на распределительных шинах, то он не посы- лает ток в сеть и не потребляет его из сети. Если бы электро- движущая сила второго генератора была меньше напряжения на распределительных шинах, то первый генератор послал бы ток во второй и тот начал бы работать электродвигателем. Чтобы загрузить второй генератор, т. е. перевести на него часть нагрузки с первого генератора, необходимо несколько уве- личить электродвижущую силу второго генератора, изменив ток в его обмотке возбуждения при помощи шунтового реостата.- Тогда второй генератор пошлет некоторый ток во внешнюю цепь, и чем больше будет его электродвижущая сила по сравнению с* напряжением на распределительных шинах, тем больше будет величина этого тока. Одновременно с этим, уменьшая ток возбу- ждения первого генератора, уменьшают его электродвижущую силу, а в соответствии с этим и его нагрузку. Таким образом, изменяя токи возбуждения в генераторах, можно перераспределить между ними общую нагрузку сети как угодно. При этом обращают внима- ние на то, чтобы при переводе нагрузки с одного генератора на другой напряжение на распределительных шинах оставалось не- изменным. Это достигается одновременным регулированием то- ков возбуждения обоих генераторов. 717
^Нагрузка Рис. 477. Схема включения на параллельную работу двух генераторов со смешанным воз- буждением Если возникает не- обходимость отключить генератор, например первый, от сети, то его ток возбуждения посте- пенно уменьшают и одновременно с этим увеличивают ток воз- буждения второго гене- ратора, переводя на- грузку первого на вто- рой. Когда ток первого генератора достигнет нулевого значения, т. е. генератор разгрузится полностью, его отклю- чают от распределитель- ных шин, размыкая ру- бильник. Порядок подключе- ния на параллельную работу генераторов со смешанным возбуждением, отключения их и переключения нагрузки с одного генератора на другой такой же, как и при подключении и выключении шунтовых генерато- ров. Следует только отметить, что в схеме параллельно соеди- ненных генераторов со смешанным возбуждением (рис. 477) есть уравнительная шина (провод) между теми одноимен- ными щетками генераторов, к которым подсоединен один из кон- цов последовательной обмотки возбуждения генератора. Эта уравнительная шина (провод) необходима для устойчивой парал- лельной работы генераторов. Если у одного из генераторов, на- пример второго-, почему-нибудь уменьшится электродвижущая сила, то первый генератор пошлет ток во второй, и тот начнет работать в режиме электродвигателя, что неизбежно приведет к аварии. При наличии же уравнительной шины первый генера- тор пошлет по ней ток в последовательную обмотку второго гене- ратора, усилит ее магнитное поле и этим увеличит электродвижу- щую силу второго генератора до необходимой величины. Следо- вательно, при помощи уравнительной шины достигается автома- тическая регулировка электродвижущих сил параллельно рабо- тающих генераторов со смешанным возбуждением, что приводит к стабильности их работы. § 217. ПАРАЛЛЕЛЬНАЯ РАБОТА ГЕНЕРАТОРА ПОСТОЯННОГО ТОКА С АККУМУЛЯТОРНОЙ БАТАРЕЕЙ Одна из наиболее распространенных схем параллельной ра- боты генератора постоянного тока с аккумуляторной батареей — схема с буферной батареей. 718
Если нагрузка в сети мала, то генератор обслуживает сеть и в.то же время заряжает аккумуляторную батарею. Если же нагрузка в сети большая и генератор перегружен, то парал- лельно с ним начинает работать на сеть аккумуляторная ба- тарея. Таким образом, аккумуляторная батарея, помогая генератору, служит как бы буфером между ним и нагрузкой (отсюда назва- ние — буферная батарея): в соответствии с изменением нагрузки она или забирает от генератора излишек его энергии, давая ему возможность работать на полную нагрузку, с наиболее высоким к. п. д., или же разряжается на сеть и этим облегчает работу генератора в моменты образования пика нагрузки. В схеме с буферной батареей важную роль играет двойной элементный коммутатор, позволяющий подбирать необходимое число аккумуляторных элементов на заряд или разряд батареи. На рис. 478 изображен внешний вид двойного элементного ком- мутатора, состоящего из двух рычагов, сидящих на общей оси вращения и изолированных друг от друга. Один из этих рычагов называется зарядным, а другой разрядным, так как через пер- вый из них осуществляется включение аккумуляторных элемен- тов на заряд, а через второй — на разряд. При повороте вокруг оси рычаги скользят своими концами по одним и тем же кон- тактным пластинам, изолированным друг от друга. Каждая из этих пластин соединена проводом с соответствующим аккумуля- торным элементом. Следовательно, устанавливая зарядный или разрядный рычаг на ту или иную контактную пластину, мы под- ключаем через двойной элементный коммутатор соответствующее количество аккумуляторных элементов В схеме, изображенной на рис. 479, .разрядный и за- рядный рычаги двойного эле- ментного коммутатора обо- значены соответственно че- рез Pi и Р2. На схеме для большей наглядности пока- зано, что рычаги Р± и Р2 мо- гут скользить только по своим контактам. Фактиче- ски же оба рычага скользят по одним и тем же контак- там и в строго определенной последовательности один за другим: зарядный рычаг Р2 должен быть всегда правее разрядного рычага..Pi, иначе вместо заряда аккумулятор- на заряд или разряд. ных элементов может проис- рИСе 473. Двойной элементный комму- ходить их разряд. татор 719
Ознакомившись в общих чертах с двойным элементным ком- мутатором, перейдем к рассмотрению основных режимов работы источников электрической энергии в схеме генератора с буфер- ной батареей, снабженной двойным элементным коммутатором (рис. 479). 1. Работает генератор электрической энергии только на сеть. Замыкают рубильники Ki и Кг- Рубильник Кз остается разом- кнутым, а переключатель Р устанавливается в положение 2. Рис. 479. Схема параллельной работы генера- тора с буферной батареей аккумуляторов Тогда ток от плюс-зажима генератора идет через амперметр А к узловой точке а схемы и далее к потребителю, а затем через узловую точку b схемы и через переключатель Р — к минус-за- жиму генератора. 2. Работает батарея на сеть. Замыкают рубильники /<2 и #3. Рубильник Ki оставляют ра- зомкнутым, а переключатель Р устанавливают в среднее поло- 720
жение. Тогда трк от плюс-зажима батареи идет через ампер- метр А к узловой точке а схемы, а отсюда к потребителю и затем возвращается через узловую точку b схемы и разрядный ры- чаг Р\ к минус-зажиму батареи. Нетрудно видеть, что все акку- муляторные элементы, находящиеся правее разрядного рычага Pi (7, 2, 3), не принимают участия в разряде батареи. Чтобы ввести их в действие, необходимо рычаг Pi перевести вправо. Таким образом, перемещая разрядный рычаг Pi, можно регули- ровать напряжение, подаваемое батареей на потребитель. 3. Работают генератор и батарея совместно на сеть. Замыкают рубильники Ki, Kz и Кз, а переключатель Р уста- навливают в положение 2. Тогда к точке а схемы притекает как ток генератора, так и ток батареи, а отсюда суммарный ток идет к потребителю и затем возвращается к узловой точке b схемы, где он разветвляется. Одна часть его возвращается к минус- зажиму генератора через переключатель Р, а другая часть — к минус-зажиму аккумуляторной батареи через разрядный ры- чаг Pi. 4. Заряд батареи генератором. Замыкают рубильник Ki, а переключатель Р ставят в поло- жение 1. Рубильники Кг и К» оставляют разомкнутыми. Тогда ток от плюс-зажима генератора идет к узловой точке а схемы, а отсюда через амперметр А к плюс-зажиму батареи и затем, пройдя батарею, возвращается через зарядный рычаг Р2 и пере- ключатель Р к минус-зажиму генератора. Поворачивая заряд- ный рычаг Р2, можно изменять количество элементов, находя- щихся на заряде. 5. Одновременная работа генератора на сеть и на заряд ба- тареи. Замыкают рубильники Ki, Kz и Кз, а переключатель Р уста- навливают в положение 1. Тогда ток генератора, дойдя до точ- ки а схемы, разветвляется. Одна часть его идет через ампер- метр А и батарею, а другая, пройдя потребитель, узловую точку b схемы и разрядный рычаг Pi, также поступает в батарею. Таким образом,- в группе элементов, находящихся между рычагами Pi и Р2, протекает весь ток генератора, который, пройдя затем через зарядный рычаг Р2 и переключатель Pi, возвращается к минус- зажиму генератора. По мере того как заряжается батарея, т. е. напряжение ее растет, разрядный рычаг Pi передвигается влево. Что же касается зарядного рычага Р2, то он перемещается влево по мере того, как заканчивается заряд крайних элементов (№ 1,2...). Следует иметь в виду, что если напряжение на за- жимах батареи превысит напряжение на за- жимах генератора, то аккумулятор будет раз- ряжаться через генератор и последний на- чнет работать в режиме электродвигателя, что приведет к аварии. Для предупреждения подобных 46-1377 721
нежелательных случаев в схеме с буферной батареей служат приборы, которые или своевременно сигнализирует о возможном разряде батареи через генератор, или самостоятельно отключают генератор от сети. К последним типам приборов относятся мини- мальные автоматы, основанные на 'действии электромагнита. Если через обмотку электромагнита проходит ток не меньше установленного предела, то электромагнит притягивает якорь, служащий размыкателем цепи. Если ток в генераторе, а следо- вательно, и в обмотке электромагнита минимального автомата уменьшится ниже предела, то якорь электромагнита под влия- нием силы тяжести отпадет от сердечника электромагнита и разомкнет цепь генератора.
ГЛАВА XXXVI ЭЛЕКТРОДВИГАТЕЛИ ПОСТОЯННОГО ТОКА § 218. ПРИНЦИП ДЕЙСТВИЯ ЭЛЕКТРОДВИГАТЕЛЯ ПОСТОЯННОГО ТОКА Электродвигателем называется электрическая машина, преоб- разующая электрическую энергию в механическую. Работа электродвигателя основана на механическом взаимо- действии проводников с током и внешнего магнитного поля. Первый электродвигатель постоянного тока был изобретен русским ученым Б. С. Якоби. Этот электродвигатель был на- столько совершенным, что оказалось возможным применить его на практике. В 1838 г. электродвигатель Якоби был применен для передвижения лодки, вмещавшей от 16 до 18 человек, причем лодка плыла по реке Неве при встречном течении и ветре. Это свидетельствовало о достаточно большой мощности электродвига- теля. Но электродвигатель Якоби получал электрическую энер- гию от батареи гальванических элементов, так как в то время не было других, более мощных, источников электрической энергии. Поэтому использование электродвигателя Якоби для транспорта было менее выгодным, чем использование паровой машины. За- мечательное изобретение Б. С. Якоби — электродвигатель по- стоянного тока — начало широко применяться после того, как по- явились генераторы электрической энергии, дающие в большом количестве дешевую электрическую энергию. В настоящее время электродвигатели постоянного тока широко используются в электротехнике. Электрические железные дороги, трамвай, подъемные краны, лифты и ряд других установок обслу- живаются электродвигателями постоянного тока. Выдающийся русский ученый академик Э. X. Ленц на основе открытого им закона электромагнитной индукции (правило Ленца) впервые установил принцип обратимости элек- трических машин. Всякий генератор постоя нноготокаявляется обратимой машиной, т. е. если, якорь и элек- тромагниты генератора питать током от по- 46* 723
Стороннего источника электрической энергии, то он начнет работать электродвигателем, и, наоборот, если якорь электродвигателя вра- щать посторонним двигателем, то он будет ра- ботать в режиме генератора. На рис. 480 показана принципиальная схема электродвигателя постоянного тока. В проводниках, расположенных под северным полюсом, ток ухо- дит от нас за плоскость чертежа, что и отмечено крестиками. В проводниках, расположенных под южным полюсом, ток течет из-за плоскости к нам, что показано точками. По правилу левой руки определим направление действия силы F на проводники якоря. На проводники, находящиеся под северным полюсом, Рис. 480. Принципиаль- ная схема электродвига- теля постоянного тока Рис. 481. Пара сил, действующая на якорь электродвига- теля силы действуют влево, а на проводники, находящиеся под юж- ным полюсом,— вправо. Результирующие этих сил показаны на рис. 481 приложенными к двум диаметрально противоположным точкам якоря. Эта пара сил (Fi и F2) стремится вращать якорь электродвигателя против часовой стрелки. Если в электродвигателе одновременно* пе- ременить ток в проводниках якоря и в элек- тромагнитах, то направление вращенияякоря электродвигателя не изменится. Если же переменить направление тока только в проводниках якоря электродвигателя, оставив неизменной полярность электро- магнитов, или, наоборот, переменить полярность электромагнитов, оставив направление тока в проводниках якоря неизменным, то направление вращения электродвигателя изменится на обратное. Сила, действующая на каждый активный проводник якоря, определяется формулой F = BIl, (570) 724
где F — сила в ньютонах; I — ток в амперах; В—магнитная индукция в вольт-секундах на квадратный метр; /—активная длина проводника в метрах. Следовательно, чем силйюе внешнее магнитное поле, больше величина тока в проводниках якоря и больше активных провод- ников на якоре, тем больше тяговое усилие F, развиваемое на окружности якоря электродвигателя. § 219. ВРАЩАЮЩИЙ МОМЕНТ, МОЩНОСТЬ И КОЭФФИЦИЕНТ ПОЛЕЗНОГО ДЕЙСТВИЯ ЭЛЕКТРОДВИГАТЕЛЯ Суммируя силы, действующие на проводники якоря электро- двигателя, получим пару равных сил F\ = F2 = F, приложенных к диаметрально противоположным точкам якоря. Плечо этой пары сил равно диаметру якоря D (см. рис. 481). Вращаю- щий момент М пары сил относительно оси вра- щения равен произведению одной из' с и л этой пары на расстояние между направлениями действия этих сил, т. е. M=FD, (571) где М— вращающий момент в ньютонометрах; F— сила в ньютонах; D— длина плеча пары сил в метрах. Но результирующая сила F, как было указано ранее, сла- гается из совокупности сил, действующих на отдельные провод- ники, величины которых определяются по формуле F, = BI^ где Fx—сила, действующая на отдельный проводник якоря, в ньютонах; В — величина магнитной индукции в вольт-секундах на квадратный метр; /я—ток в проводниках якорной обмотки в амперах; I—активная длина каждого проводника якоря в метрах. Следовательно, тяговое усилие электродвигателя равно F = kFb где k— коэффициент пропорциональности, зависящий от кон- структивных особенностей электродвигателя. Вращающий момент М электродвигателя равен М = FD = kF. D = kIABlD. 1 Л 725
Ho ID = S — площадь просвета каждого отдельного контура якорной обмотки в квадратных метрах, a BID = BS = Ф — маг- нитный поток, пронизывающий площадь просвета каждого отдель- ного контура якорной обмотки в вольт-секундах. В соответствии с этим величину вращающего момента элек- тродвигателя можно выразить формулой = (572) т. е. вращающий момент электродвигателя за- висит от величины магнитного потока Ф, со- здаваемого электромагнитами электродвига- теля, от величины тока в якорной обмотке и от конструктивных особенностей электродви- гателя, учитываемых коэффициентом пропор- циональности k. Следовательно, чтобы- увеличить вращающий момент М электродвигателя, необходимо увеличить ток /я в якоре элек- тродвигателя и магнитный поток Ф, создаваемый его электро- магнитами. Мощность, поглощаемая электродвигателем из сети, больше, чем мощность, отдаваемая им на валу, так как часть энергии, поступающей в электродвигатель, теряется на тепловую энер- гию в обмотках электродвигателя, на вихревые токи, на пере- магничивание, на трение в подшипниках и т. д.: Рп = Р + S Л, где Рп— мощность, поглощаемая электродвигателем из сети; Р— полезная мощность, развиваемая электродвигателем; — сумма мощностей потерь в электродвигателе. Отношение полезной механической мощности, развиваемой электродвигателем на валу, к полной электрической мощности, потребляемой им из сети, называется коэффициентом по- лезного действия (к. п. д.) его: где т] — к. п. д. электродвигателя; Р— полезная механическая мощность электродвигателя; Рп — электрическая мощность, потребляемая электродвига- телем изсети. Коэффициент полезного действия электродвигателя зависит от степени его механической нагрузки. Обычно электродвигатели рассчитываются так, чтобы их максимальный к. п. д. приходился на нормальную нагрузку. Электродвигатели, обладающие боль- шими мощностями, как правило, имеет большие к. п. д., чем электродвигатели малых мощностей. Среднее- значение к. п. д. электродвигателей колеблется от 0,7 до 0,9. 726
Пример 218. Определить к. п. д. электродвигателя, развивающего на своем валу механическую мощность Pi = 5 л. с., если при напряжении на зажимах электродвигателя £7= 125 в он получает ток I = 36,8 а. Решение. Полезная мощность Рг = 5-736 = 3680 вт — 3,68 кет. Потребляемая мощность Р — U1 = 125-36,8 = 4600 вт = 4,6 кет. Коэффициент полезного действия Л _ 3,68 - ~Р - “4J" - °’8’ § 220. ПРОТИВОЭЛЕКТРОДВИЖУШАЯ СИЛА ЭЛЕКТРОДВИГАТЕЛЯ При работе электродвигателя его якорь вращается в магнит- ном поле с некоторой скоростью, зависящей от величины меха- нической нагрузки электродвигателя. Активные проводники якорной обмотки электродвигателя, пе-, ремещаясь вместе с якорем в магнитном поле, пересекают его магнитные линии, и в них индуктируется электродвижущая сила. Согласно правилу Ленца эта электродвижущая сила стремится противодействовать причине, вызвавшей ее, т. е. в данном слу- чае она направлена навстречу току в проводниках якоря (см. рис. 480). Но если это так, то она направлена навстречу и напряжению U, приложенному к электродвигателю. Электродвижущая сила, индуктированная в я к о р- ной обмотке электродвигателя, называется противоэлектродвижущей силой. Величина ее прямо пропорциональна скорости вращения якоря и величине магнит- ного потока электродвигателя: Е = спФ, (573) где Е — противоэлектродвижущая сила в вольтах; п — число оборотов якоря электродвигателя в минуту; Ф — магнитный поток в вольт-секундах; с — коэффициент пропорциональности, зависящий от кон- струкции электродвигателя. Следовательно, величина тока в якорной обмотке электродви- гателя зависит не только от величины приложенного к электро- двигателю напряжения, но и от противоэлектродвижущей силы, индуктированной в этой обмотке: где /я — ток в якорной обмотке электродвигателя; гя — сопротивление якорной обмотки. 727
Чем быстрее вращается якорь электродвигателя, тем больше противоэлектродвижущая сила в якорной обмотке и тем меньше ток /я, протекающий по этой обмотке. В пределе, когда число оборотов якоря электродвигателя в минуту равно нулю, ток в якорной обмотке при всех прочих равных условиях максима- лен, так как противоэлектро- движущая сила в якорной обмотке в этом случае равна нулю. Так как сопротивление якорной обмотки электродви- гателя обычно мало — по- рядка десятых или сотых долей ома, то при пуске электродвигателя в ход ток в якорной обмотке может оказаться чрезмерно боль- Рис. 482. Последовательно с электро- шим и опасным для элек- двигателем включен пусковой реостат гр тродвигателя. Чтобы предот- вратить возможность возник- новения опасной величины пускового тока, на время пуска элек- тродвигателя в ход подключают последовательно с якорной об- моткой пусковой реостат (рис. 482). Сопротивление его подби- рается таким, чтобы начальный пусковой ток электродвигателя не превышал номинального его тока больше чем в полтора — два раза. Величина пускового тока в начальный момент пуска электро- двигателя в ход при включенном пусковом реостате будет равна / =________~___ НаЧ Гя + Гр’ (575) гДе /нач— начальный пусковой ток электродвигателя.; U— напряжение, приложенное к электродвигателю; гя — сопротивление якорной обмотки; гр— сопротивление пускового реостата. По мере того как якорь электродвигателя при пуске в ход увеличивает скорость своего вращения, растет противоэлектро- движущая сила в якорной обмотке и соответственно уменьшается величина пускового тока. Необходимость в сопротивлении пуско- вого реостата постепенно отпадает. Поэтому по мере того как якорь электродвигателя увеличивает скорость своего вращения, плавно выводят сопротивление пускового реостата, уменьшая его величину до нуля. Пусковой реостат бывает под током только во время пуска электродвигателя в ход, так как на продолжительное пребывание под током он не рассчитан. Процесс пуска в ход электродвигателя обычна исчисляется 728
долями минуты, во время которых якорь электродвигателя успе- вает уже развить нормальное число оборотов. Пример 219. Сопротивление якорной обмотки электродвигателя гя = = 0,1 ом. Определить, какое сопротивление реостата должно быть введено в начальный момент пуска в ход электродвигателя, чтобы при напряжении 0 = 110 в, приложенном к электродвигателю, величина начального пуско- вого тока не превысила величины /нач = 50 а. Решени-е. Сопротивление пускового реостата г _ U ^начЛт _____ НО э0-0,1 Гр — — = z,l ом. 'нач § 221. РЕАКЦИЯ ЯКОРЯ ЭЛЕКТРОДВИГАТЕЛЯ Реакция якоря электродвигателя — это влияние магнитного поля тока якорной обмотки на режим электродвигателя. На рис. 483 схематически изображен электродвигатель. Здесь показан якорь с его обмоткой, казано условными знаками (+) нитных полюса N и S, со- здающих основное магнит- ное поле электродвигателя. Согласно правилу левой руки якорь в данном случае вращается против часовой стрелки. Результирующее магнит- ное поле электродвигателя представляет собой совокуп- ; ность двух магнитных полей: основного магнитного поля, создаваемого магнитными по- люсами, и магнитного поля, создаваемого током якорной обмотки. У краев магнит- ных полюсов, на которые набегает якорь при своем вращении, наблюдается уси- направление тока в которой по- и (•), и два разноименных маг- N Рис. 483. Реакция якоря электродвига- теля реакции якоря в электродвигателе ление магнитного поля, а у краев магнитных полюсов, с которых сбегает якорь,— ослабление полюсов. Действие сводится к искажению и уменьшению магнитного потока, что наиболее сильно сказывается при больших нагрузках двигателя. Чтобы избежать искрения под щетками на коллекторе, возни- кающего в результате реакции якоря, необходимо щетки сме- стить по окружности коллектора на угол 0 против движения якоря. 729
Реакция якоря оказывает вредное^влияние на режим работы электродвигателя. Для ослабления этого влияния в электродви- гателях применяют дополнительные полюса, магнитное поле ко- торых компенсирует магнитное поле тока якорной обмотки. § 222. ВЛИЯНИЕ МЕХАНИЧЕСКОЙ НАГРУЗКИ НА РЕЖИМ РАБОТЫ ЭЛЕКТРОДВИГАТЕЛЯ Если электродвигатель нагрузить, т. е. заставить его приво- дить в движение какие-либо станки, машины и т.ш., то он будет совершать механическую работу, преодолевая сопротивление ме- ханической нагрузки. Чем больше механическая нагрузка на электродвигатель, тем большее сопротивление оказывает она вра- щению электродвигателя и тем больше момент этой силы сопро- тивления. Вращающий момент электродвигателя и мо- мент силы сопротивления механической на- грузки взаимно компенсируют друг друга. Если вращающий момент электродвигателя больше момента сил сопротивления, то электродвигатель увеличивает число своих оборотов до тех пор, пока его момент вращения не уравновеши- вается полностью моментом сил сопротивления. Наоборот, если момент вращения электродвигателя меньше момента сил сопро- тивления, то скорость вращения электродвигателя начинает сни- жаться до тех пор, пока момент вращения не уравновешивает полностью момента сопротивления. Таким образом, число оборотов электродвигателя до некото- рой степени зависит от величины его механической нагрузки. С ростом механической нагрузки электро- двигателя скорость вращения его уменьшается, что влечет за собой уменьшение противоэлек- тродвижущей силы в якоре и, следовательно, увеличение тока, потребляемого электродви- гателем. Наоборот, с уменьшением механической нагрузки электродвигателя увеличивается число оборотов его якоря, его противоэлектродвижущая сила и уменьшается потребляемый им ток. Следовательно, величина тока, потребляемого электродви- гателем, строго согласована с его механической нагрузкой и ре- гулируется противоэлектродвижущей силой, индуктируемой в об- мотке якоря. При холостом ходе, когда механической нагрузки'нет, элек- тродвигатель развивает наибольшую скорость вращения и в соот- ветствии с этим потребляет наименьший ток и наименьшую энер- гию. Наоборот, если перегрузить электродвигатель, то его ско- рость вращения может снизиться настолько, что он в конце кон- цов может остановиться. Тогда потребляемый электродвигателем 730
ток из-за отсутствия противоэлектродвижущей силы станет мак- симальным и опасным для него. Автоматическое саморегулирование потребления тока и мощ- ности электродвигателем в связи с изменяющейся механической нагрузкой — очень ценное качество электродвигателя. Если к якорю электродвигателя приложено напряжение U, то при некоторой произвольной механической нагрузке ток в нем равен , (576) 'я где Е—протйвоэлектродвижущая сила; гя— сопротивление обмотки якоря; /я—ток в обмотке якоря. Индуктированная протйвоэлектродвижущая сила в обмотке якоря пропорциональна числу оборотов якоря п в единицу вре- мени и магнитному потоку Ф, создаваемому электромагнитами: Е=спФ, (577) где с — коэффициент пропорциональности. Подставив в формулу (576) вместо Е ее выражение из фор- мулы (577), получим т U — спФ откуда находим, что 078) Из формулы (578) следует, что скорость вращения якоря электродвигателя увеличивается с увеличением напряжения U на якоре электродвигателя, с уменьшением магнитного потока Ф и величины тока /я в электродвигателе. Регулировка скорости вращения якоря электродвигателя (числа оборотов якоря в ми- нуту) обычно осуществляется с помощью изменения магнитного потока электродвигателя, что практически достигается измене- нием тока возбуждения электродвигателя. Чем больше магнит- ный поток в электродвигателе, тем меньше при всех прочих рав- ных условиях должна быть скорость вращения якоря электродви- гателя, чтобы получить необходимую при данной механической нагрузке противоэлектродвижущую силу. Наоборот, чем меньше магнитный поток, тем больше оборотов в минуту должен совер- шать якорь для достижения той же величины протйвоэлектродви- жущей силы. Основными величинами, характеризующими работу электро- двигателя постоянного тока? являются: напряжение на зажи- 731
мах U, ток в якоре /я, ток возбуждения /в, вращающий мо- мент М и число оборотов в минуту п якоря электродвигателя. Зависимость между какими-либо двумя основными величи- нами, характеризующими работу электродвигателя, называется характеристикой электродвигателя. Наиболее важными характеристиками электродвигателей по- стоянного тока являются: внешние, или, иначе, рабочие, характе- ристики и регулировочные характеристики. К внешним характеристикам относятся: 1) зависимость вращающего момента М электродвигателя от тока /я в якоре при постоянстве приложенного к электродвига- телю напряжения (t/ = const) и постоянстве сопротивления ^его обмотки возбуждения (rB = const); 2) зависимость числа оборотов п якоря электродвигателя в минуту от тока /я в якоре при U = const и rB = const. Регулировочной характеристикой электродви- гателя постоянного тока называется зависимость тока возбужде- ния /в от тока /я в якоре при постоянстве напряжения ([/ = const), приложенного к электродвигателю, *и постоянстве числа оборотов якоря (п — const). В последующих параграфах мы рассмотрим характеристики основных типов электродвигателей постоянного тока. § 223. ЭЛЕКТРОДВИГАТЕЛЬ С ПОСЛЕДОВАТЕЛЬНЫМ ВОЗБУЖДЕНИЕМ В электродвигателе с последовательным возбуждением обмотка возбуждения подклю- чена последовательно к якорной обмотке электродвигателя. На рис. 484 приведена принципиальная схема электродвига- теля с последовательным возбуждением, подключенного к сети постоянного тока. Здесь мы видим, что последовательно с якорной обмоткой электродвигателя соединен пусковой реостат гп, с по- мощью которого осуществляется пуск в ход электродвигателя. Перед началом пуска в ход электродвигателя рычаг пускового реостата установлен на холостом контакте (х. к). При пуске же в ход электродвигателя этот рычаг переводится на ближайший рабочий контакт, а затем медленно поворачивается до тех пор, пока он не займет положение на последнем рабочем контакте рео- стата. В этом случае сопротивление пускового реостата будет полностью выведено, а якорь электродвигателя уже разовьет необходимую скорость вращения. При остановке электродвига- теля рычаг пускового реостата плавно переводится обратно на холостой контакт и затем размыкается рубильник К. Параллельно обмотке возбуждения присоединен регулировоч- ный реостат гр, с помощью которого можно регулировать скорость вращения якоря электродвигателя. Если, например, необходимо 732
увеличить число оборотов якоря электродвигателя, то следует уменьшить сопротивление регулировочного реостата. В этом слу- чае ток, проходящий через обмотку возбуждения, уменьшится, а с ним вместе уменьшится магнитный поток Ф электродвигателя, что в свою очередь приведет к росту,числа оборотов п в ми- нуту якоря электродвигателя. — Амперметр и вольтметр, включенные в цепь электро- двигателя, дают возможность ч следить за режимом работы электродвигателя. Вращающий момент элек- тродвигателя. Вращающий момент М электродвигателя постоянного тока зависит от величины тока /я, потреб- ляемого электродвигателем, и от магнитного потока Ф электродвигател я: /И=АГ7ЯФ, (579) где К — коэффициент про- порциональности. Если магнитного насы- щения в стали магнитопро- вода электродвигателя нет, то можно считать, что маг- нитный поток Ф электродви- гателя пропорционален току возбуждения ZB, т. е. Рис. 484. Принципиальная схема элек- ’ тродвигателя с последовательным воз- Ф = буждением где Ki — коэффициент пропорциональности. Но так как в электродвигателе с последовательным возбужде- нием якорная обмотка соединена последовательно с обмоткой возбуждения, то ток возбуждения г3 практически равен току 7Я, протекающему через якорную обмотку (/в = /я). В соответствии с этим можно считать, что магнитный поток Ф электродвигателя с последовательным возбуждением пропорционален величине тока /я, потребляемого электродвигателем, т. е. Ф = /С/я. (580) Подставив в формулу (579) вместо Ф его выражение из формулы (580), получим 41 = W2 = /<2Z2, (581) где = КК^ — коэффициент пропорциональности. 733
Рис. 485. График зависимости вращающего момента электродви- гателя с последовательным возбу- ждением от величины потребляе- мого тока Из формулы (581) следует, что вращающий момент электро- двигателя с последовательным возбуждением пропорционален квадрату тока, потребляемого электродвигателем. На рис. 485 показан график зависимости вращающего момен- та М электродвигателя с последо- вательным возбуждением от то- ка /, потребляемого электродвига- телем. Число оборотов якоря элек- тродвигателя в минуту. Число обо- ротов в минуту п якоря электро- двигателя постоянного тока зави- сит от величины приложенного к электродвигателю напряже- электродвигателя и от ния U, от величины магнитного потока Ф тока /я, потребляемого электродвигателем: U — 1лгл п =------- сФ (582) где U—напряжение, приложенное к якорю электродвигателя; с— коэффициент пропорциональности. В рассматриваемом электродвигателе с последовательным воз- буждением магнитный поток Ф пропорционален току /я, а паде- ние напряжения имеется не только в якорной обмотке электро- двигателя (7ягя), но и в цепи возбуждения (4гв = 4л)- В соот- ветствии с этим формула для определения скорости вращения якоря электродвигателя с последовательным возбуждением при- нимает следующий вид: п = и-1я(^ +г в) , (583) где U—напряжение, приложенное к электродвигателю; с1— коэффициент пропорциональности. Так как сопротивления обмотки якоря гя и цепи возбуждения гв малы, то падение напряжения в них мало по сравнению с на- пряжением (7, т. е. /я(гя + rB) <С U. Следовательно, формуле по определению числа оборотов якоря электродвигателя можно при- дать такой приближенный вид: <584> Из формулы (584) следует, что число оборотов якоря элек- тродвигателя ? Последовательным возбуждением практически об- 734
ратно пропорционально току /я, потребляемому электродвига- телем. На рис. 486 доказан график зависимости числа оборотов п якоря в минуту электродвигателя с последовательным возбужде- нием от тока /я, потребляемого электродвигателем. Таким образом, электродвигатель с последовательным воз- буждением приспосабливается к механической нагрузке. При большой нагрузке он потребляет большой ток и при этом разви- вает большой вращающий момент и малую скорость вращения своего якоря. Наоборот, при малой механической нагрузке он потребляет малый ток, развивает малый вращающий момент й большую скорость вращения якоря. Теперь посмотрим, каков режим холостого хода элек- тродвигателя с последовательным возбуждением. Допустим, что с нормально нагруженного электродвигателя внезапно сбрасы- вается вся механическая нагрузка. Например, электродвигатель с помощью ременной передачи вращал какую-либо машину, а за- тем в некоторый момент времени приводной ремень соскочил с маховика и электродвигатель, лишенный механической нагрузки, начал работать в режиме холостого хода. В результате сброса механической нагрузки с электродвигателя резко уменьшился момент сил сопротивления движению и) следовательно, вращаю- щий момент стал значительно больше него. Поэтому скорость вращения якоря электродвигателя начинает возрастать. Это бу- дет продолжаться до тех пор, пока вращающий момент не ста- нет равным моменту сил сопротивления движению, т. е. пока не наступит динамическое равновесие в системе электродвигателя и якорь не будет вращаться с равномерной скоростью. Однако состояние динамического равновесия в системе электродвигателя может не. наступить. Дело в том, что с уменьшением тока в электродвигателе уменьшается и магнитный поток Ф, создаваемый этим током. А так как при холостом ходе электро- двигателя ток, потребляем мый им, относительно мал, то и магнитный поток элек- тродвигателя также отно- сительно невелик. Вслед- ствие этого якорь электро- двигателя должен развить весьма большое число оборотов в минуту, чтобы установилась та мини- мальная величина тока, при которой наступает Рис. 486. График зависимости числа обо- ротов якоря электродвигателя с последо- вательным возбуждением от величины по- требляемого тока (от нагрузки) 735
динамическое равновесие. Однако обычно такой скорости вра- щения якорь электродвигателя не достигает, так как при неко- торой предельной скорости его вращения происходит разнос электродвигателя, в результате чего электродвигатель выбывает из строя. Отсюда следует сделать практический вывод, что электродвигатель с последовательным воз- буждением нельзя запускать в х о л о сту ю. и'з - з а возможности аварии. Рекомендуется такой электродви- гатель жестко сцеплять с механической нагрузкой (например, с помощью муфт), но ни в коем случае не применять гибких передач (например, ремни, канаты, тросы и т. п.), так как при обрыве их электродвигатель переходит в режим холостого хода. Для предупреждения разноса электродвигателя при холостом ходе иногда применяют специальные автоматы, отключающие электродвигатель от сети, при некотором предельном числе оборо- тов его якоря. Кривые зависимости вращающего момента М (рис. 485) и числа оборотов п якоря в минуту (рис. 486) от тока /я, потреб- ляемого электродвигателем, являются внешними (рабочими) ха- рактеристиками электродвигателя. Внешние характеристики электродвигателя с после- довательным возбуждением показывают, что он наиболее приго- ден в том случае, когда от электродвигателя .требуется большой вращающий момент при малом числе оборотов его якоря в ми- нуту, т. е. при пуске электродвигателя в ход под нагрузкой, и, наоборот, когда требуется относительно малый вращающий мо- мент при номинальной скорости вращения якоря. В силу этого электродвигатели с последовательным возбуждением нашли ши- рокое применение на электровозах, в моторных трамвайных ва- гонах, кранах, подъемниках и т. п. § 224. ЭЛЕКТРОДВИГАТЕЛЬ С ПАРАЛЛЕЛЬНЫМ ВОЗБУЖДЕНИЕМ В электродвигателе с параллельным возбуж- дением обмотка возбуждения подключена па- раллельно.якорной обмотке. На рис. 487 приведена схема электродвигателя постоянного тока с параллельным возбуждением, подключенного к сети по- стоянного тока. Чтобы пусковой вращающий момент электродвигателя был достаточной величины, необходимо при пуске, в ход электродви- гателя получить в нем большой магнитный., поток, так как вра- щающий момент электродвигателя пропорционален величине маг- нитного потока Ф. С этой целью рекомендуется обмотку возбуж- дения электродвигателя подключать непосредственно к сети, т. е. до пускового реостата. Для этого пусковой реостат гп снабжен металлической дугообразной пластиной, по которой-скользит ры- 736
чаг реостата при своем повороте. Эта пластина присоединена од- ним своим концом к началу пускового реостата, а другим — к одному из концов обмотки возбуждения. Нетрудно видеть, что независимо от положения рычага пускового реостата (кроме по- ложения на холостом контакте) вся- кий раз цепь возбуждения подклю- чена под полное напряжение сети. При пуске в ход электродвигате- ля рычаг пускового реостата нахо- дится на холостом контакте (х. к.). В результате этого электродвигатель отключен от сети. Пуск в ход элек- тродвигателя осуществляется плав- ным поворотом рычага пускового реостата до тех пор, пока он не пе- рейдет на последний рабочий кон- такт электродвигателя. Последовательно с обмоткой воз- буждения электродвигателя включен регулировочный реостат гр, с по- мощью которого можно изменять ток возбуждения iB и, следовательно, магнитный поток Ф электродвигате- ля и соответственно число оборотов в минуту п якоря электродвигателя. Вращающий момент электродви- гателя. Вращающий момент • элек- тродвигателя с параллельным воз- буждением зависит от .величины тока Рис. 487. Схема включения электродвигателя с парал- лельным возбуждением в сеть постоянного тока /, потребляемого электро- двигателем, и от величины магнитного потока Ф электродвигателя. Если цепь возбуждения электродвигателя имеет постоянное сопротивление^ rB = const и к этой цепи приложено постоянное напряжение U = cosnt, то ток возбуждения является постоян- ным по величине, т. е. 4 = — = const. гв В этом случае и магнитный поток Ф электродвигателя также является постоянным по величине, т. е. ф = *Л, где Ki — коэффициент пропорциональности. В соответствии с этим вращающий момент электродвигателя с параллельным возбуждением можно определить по формуле ^ = ^4=^4 (585) где /4 = А'хФ == const— коэффициент пропорциональности. 47—1377 737
Рис. 488. График зависимости вра- щающего момента электродвигателя с параллельным возбуждением от нагрузки (от величины тока) ристика) вращающего момента ным возбуждением от тока /я Из формулы (585) следует, что вращающий момент М электродвигателя с параллель- ным возбуждением пропорцио- нален величине тока /я, потреб- ляемого электродвигателем. При большой механической нагрузке электродвигателя, ко- гда реакция якоря сказывается более сильно, магнитный по- ток электродвигателя несколько снижается. Это приводит к тому, что линейная зависимость между вращающим моментом и током в якоре электродвига- теля нарушается. На рис. 488 показан график зависимости (внешняя характе- М электродвигателя с параллель- j якоре электродвигателя. Число оборотов якоря электродвигателя в минуту. Число обо- ротов якоря электродвигателя в минуту определяется по фор- муле U - /л п —---- сФ Если напряжение, приложенное к электродвигателю, и маг- нитный поток в нем постоянны по величине ([/ = const и Ф = const), то, как это видно из формулы, число оборотов п яко- ря электродвигателя в минуту зависит от ве- личины падения на- пряжения /ягя в якор- ной обмотке. Но так как сопротивление гя этой обмотки обычно относительно мало, то и падение напряже- ния в ней мало. Отсю- да следует, что зависи- мость числа оборотов якоря электродвигателя от нагрузки очень не- значительна. Опытом Рис. 489. График зависимости числа оборотов якоря электродвигателя с параллельным воз- буждением от нагрузки (величины тока) установлено, что число оборотов якоря элек- тродвигателя в минуту 738
уменьшается с ростом механической нагрузки от холостого хода до нормальной нагрузки в среднем примерно На 5.%. На рис. 489 приведен график изменения числа оборотов в ми- нуту п якоря электродвигателя с параллельным возбуждением в зависимости от тока /я, потребляемого якорем. Электродвигатель с параллельным возбуждением при пуске в ход развивает малый вращающий момент и, следовательно, не может сразу справиться с большой механической нагрузкой, а поэтому его вначале запускают на холостой ход. Так как маг- нитный поток электродвигателя при холостом ходе относительно велик, то якорь электродвигателя не может развить чрезмерно большое число оборотов в минуту и опасность разноса электро- двигателя отпадает. После того как якорь разовьет при холостом ходе нужное число оборотов в минуту и вращающий момент электродвигателя достигнет требуемой величины, на электродви- гатель дают механическую нагрузку. V Скорость вращения якоря электродвигателя можно регулиро- вать двояким способом: изменением напряжения и изменением тока возбуждения электродвигателя. Если в формуле и—1ягя п~~ сФ положить, что Ф = const и 1ЯГЯ <С U, то можно написать, что = 4’ (586) где с — постоянная величина. Из формулы (586) следует, что скорость вращения якоря электродвигателя с параллельным возбуждением пропорциональ- на напряжению, приложенному к электродвигателю. Регулировку числа оборотов якоря электродвигателя с по- мощью изменения напряжения на его зажимах можно осуще- ствить путем поглощения., части его реостатом, включенным по- следовательно с якорной обмоткой. Однако этот способ нельзя назвать рациональным, так как он связан с относительно боль- шой потерей мощности в реостате. Более рациональный способ регулирования скорости враще- ния якоря заключается в изменении тока возбуждения электро- двигателя с помощью регулировочного реостата, включенного в цепь возбуждения, Если при постоянстве возбуждения на зажимах электродви- гателя (U = const) и постоянном моменте сил сопротивления на- чать увеличивать ток возбуждения электродвигателя, то ско- рость вращения якоря электродвигателя начнет уменьшаться. Это объясняется тем, что при увеличении тока возбуждения уве- 48—1377 739
Личивается магнитный поток Ф электродвигателя. Следовательно, для того чтобы получить в якоре необходимую противоэлектро- движущую силу, соответствующую данной механической нагруз- ке, необходимо его якорю развивать меньшее число оборотов в минуту, так как Е = спФ и (587) Наоборот, если ток возбуждения уменьшить, то уменьшится магнитный поток и в соответствии с этим согласно формуле (587) увеличится скорость вращения якоря электродвигателя. V Итак, с увеличением тока возбуждения электродвигателя с параллельным возбуждением скорость вращения якоря элек- тродвигателя уменьшается, а с уменьшением тока возбужде- ния — увеличиваемся. Если электродвигатель с параллельным возбуждением рабо- тает на холостом ходу, то чрезмерное уменьшение тока возбуж- дения и соответственно магнитного потока в электродвигателе может привести к чрезмерно большой скорости вращения якоря. Особенно опасен тот случай, когда при холостом ходе электро- двигателя происходит обрыв обмотки возбуждения. Тогда маг- нитный поток в электродвигателе достигает минимальной воз- можной величины, так как он в этом случае создается остаточ- ным магнетизмом, а число оборотов электродвигателя в минуту достигает такой величины, при которой произойдет механическая авария с электродвигателем. Изменение направления вращения якоря электродвигателя с параллельным возбуждением достигается путем переключения концов обмотки возбуждения. В этом случае изменяется направ- ление магнитного потока в электродвигателе, в то время как на- правление тока в якорной обмотке остается неизменным. Следо- вательно, механическая сила, действующая на якорные провод- ники с током, изменит свое направление на противоположное, что заставит якорь вращаться в противоположную сторону. Рассмотренные нами внешние (рабочие) характеристики электродвигателя с параллельным возбуждением показывают, что данный тип электродвигателя удобен в эксплуатации там, где требуется постоянство числа оборотов якоря электродвигателя и не требуется большого вращающего момента электродвигателя при его пуске в ход. В силу этого электродвигатели постоянного тока с параллельным возбуждением нашли широкое применение для обслуживания станков, трансмиссий, вентиляторов и вообще механизмов, работающих фактически при постоянном числе обо- ротов в единицу времени. 740
s 225. ЭЛЕКТРОДВИГАТЕЛЬ CO СМЕШАННЫМ ВОЗБУЖДЕНИЕМ Рис. 490. Схема включения электродвигателя со смешан- ным возбуждением в сеть по- стоянного тока Электродвигатель со смешанным возбужде- нием имеет две обмотки возбуждения: одну па- раллельную, а другую последовательную. На рис. 490 показана схема включения электродвигателя со смешанным возбуждением в сеть постоянного тока. Рабочие характеристики электродвигателя со смешанным воз- буждением определяются тем, действие какой из обмоток воз- буждения, параллельной или по- следовательной, преобладает. Если основной магнитный поток создается главным образом параллельной об- моткой, то рабочие характеристики электродвигателя близки к характе- ристикам электродвигателя с парал- лельным возбуждением, и, наоборот, эти характеристики будут совпадать с характеристиками электродвигате- ля с последовательным возбужде- нием, если магнитный поток создает- ся главным образом последователь- ной обмоткой возбуждения. Сочета- ние параллельной и последователь- ной обмоток в электродвигателе сме- шанного возбуждения обычно подби- рают такое, чтобы характеристики его были наиболее удобными для данной специфической нагрузки. Обычно электродвигатели со сме- шанным возбуждением применяют- ся там, где нагрузка резко неравно- мерная, например, при работе экска- ваторов, поршневых насосов, лебе- док, кранов и т. п. В самом деле, когда ковш экскаватора заполнен грузом, то требуются большой вращающий момент и малое число оборотов двигателя (характеристика двигателя с последователь- ным возбуждением), а когда ковш экскаватора освободится от груза, необходимы нормальное постоянство числа оборотов в единицу времени и относительно небольшой вращающий мо- мент (характеристика двигателя с параллельным возбуждением). Если для экскаватора применить электродвигатель с последо- вательным возбуждением, то он будет удобен в моменты подъе- ма груза и крайне неудобен, когда экскаватор освобождается от груза, так как в этом случае электродвигатель разовьет очень большую скорость вращения (режим, приближающийся к холо- стому ходу). Если же для обслуживания экскаватора приспосо- 48* 741
бить двигатель с параллельным возбуждением, то он не сможет развить необходимого вращающего момента при поднятии груза экскаватором (при малом числе оборотов в единицу времени). Постоянство оборотов в минуту якоря электродвигателя с парал- лельным возбуждением служит препятствием для нормальной работы его по обслуживанию экскаватора. Из рассмотренного примера ясно, что для нормальной работы экскаватора необходимо применить такой электродвигатель со смешанным возбуждением, который сможет дать при малом чис- ле оборотов якоря в минуту большой вращающий момент и нор- мальное число оборотов при холостом ходе. Иногда в электродвигателях со смешанным возбуждением устраиваются приспособления, дающие возможность по желанию выключать ту или иную обмотку возбуждения, т. е. переключать его на схему электродвигателя с последовательным и с парал- лельным возбуждением.
ГЛАВА XXXVII ГЕНЕРАТОРЫ ПЕРЕМЕННОГО ТОКА § 226. УСТРОЙСТВО СИНХРОННЫХ ГЕНЕРАТОРОВ В генераторах переменного тока используется принцип элек- тромагнитной индукции. По своей конструкции они подразде- ляются: на генераторы с вращающимися полюсами и неподвиж- ным якорем и генераторы, с неподвижными магнитными полюса- ми и подвижным якорем. Наибольшее распро- странение получили гене- раторы с вращающимися магнитными полюсами и неподвижным якорем. Это понятно, так как благо- даря неподвижности якор- ной обмотки отпадают технические затруднения, связанные с использова- нием. скользящих контак- тов при больших напря- жениях, снимаемых с якор- ной обмотки генератора. На рис. 491 показана принципиальная схема од- нофазного генератора пе- ременного тока с непо- движным якорем и по- движными магнитными Рис. 491. Принципиальная схема однофаз- ного генератора переменного тока с не- подвижным якорем и подвижными магнит- ными полюсами полюсами. Подвижная часть генератора переменного тока называется ро- тором, а неподвижная — статором. Статоры собираются из отдельных изолированных друг от друга железных листов. На внутренней поверхности статора имеются продольные пазы, в которые укладываются провода якорной обмотки 3 генератора. 743
полюсов однофазного генератора переменного тока. Якорная обмотка 3 генератора состоит из медных изо- лированных стержней или проводов, уложенных в пазы статора и соединенных друг с другом. На рис. 491 показаны в разрезе плоскости чертежа проводники якорной обмотки. Направления индуктированной электродвижущей силы в них обозначены обычными условными символами (+) и (•). Развертка якорной обмотки и магнитных полюсов однофазно- го генератора показана на рис. 492. Направления электродвижу- щей силы и относительного движения магнитных полюсов здесь обозначены соответствующими стрелками. Ротор генератора изготовляется обычно из ' сплошного же- леза, так как он синхронно вращается со своим магнитным по- лем и поэтому в нем не могут индуктироваться вихревые токи, Полюсные наконечники 5 магнитных полюсов 4 ротора обыч- но собираются из листового железа, так как в воздушном зазо- ре, где они перемещаются, имеется магнитное поле, создаваемое током якорной обмотки, которое в полюсных наконечниках ин- дуктирует вихревые трки. Ротор 2, изображенный на рис. 491, имеет шесть явно выра- женных магнитных полюсов 4, из которых каждые два соседних полюса разноименные. На сердечники полюсов посажены катушки возбужде- ния, питаемые постоянным током. Этот ток подводится с по- мощью щеток к контактным кольцам 6, сидящим на валу гене- ратора. По способу возбуждения генераторы переменного тока делят- ся на три основных типа: ( „ 1) генераторы, обмотки возбуждения которых питаются по- стоянным током от постороннего источника электрической энер- гии, например от аккумуляторной батареи; 2) генераторы, обмотки возбуждения которых питаются по- стоянным током от постороннего генератора постоянного тока 744
малой мощности, сидящего на одном валу с обслуживаемым им генератором; 3) генераторы, обмотки возбуждения которых питаются то- ком самих же этих генераторов, предварительно выпрямленным с помощью сухих выпрямителей; генераторы подобного рода на- зываются генераторами с самовозбуждением. Генераторы переменного тока с явно выраженными полюса- ми рассчитаны на относительно небольшое номинальное число оборотов ротора в минуту, не превышающее 1000 об/мин. По- этому их обычно спаривают с тихоходными .первичными двига- телями, например с гидротурбинами Генераторы подобного ро- да обычно называют гидрогенераторами. Генераторы переменного тока с неявно выраженными полю- сами (рис. 493) рассчитаны на относительно большое номиналь- ное число оборотов в минуту (1500—3000 об/мин). Поэтому их обычно спаривают с быстроходными первичными двигателями, например с паровыми турбинами. Генераторы подобного рода обычно называют турбогенераторами. XX съезд партии в своих директивах по развитию народного хозяйства СССР на шестую пятилетку поставил задачу выпу- стить генераторы мощностью до 300 тысяч киловатт с новой си- стемой водородного^ охлаждения. Генераторы трехфазного тока имеют на своем статоре три фазные ’ обмотки, размещенные так, что в них индуктируются электродвижущие силы, сдвинутые по фазе относительно друг друга на угол 2/з^. Начала и концы фазных обмоток трехфазного генератора со- единены так, что они образуют трехфазную систему, соединен- ную «звездой» или «треугольником». На рис. 494 приведена схема развертки части якорной обмот- ки трехфазного генератора и электродвижущих сил в фаз- ных обмотках генератора и относительного движения магнитных полюсов здесь показаны соответствующими стрелками. Полюсным наконечникам магнитных полюсов ротора генератора обычно придают такую форму, при которой в проводниках статорной об- мотки индуктируется сину- соидальная электродвижу- щая сила. Если ротор генератора двухполюсный, то за один его полный оборот индукти- магнитных полюсов. 0- р Ротор изображение Рис. 4S3. Схематическое генератора переменного тока с неявно выраженными полюсами ротора 745
рованная электродвижущая сила генератора совершит полный цикл своих изменений. Следовательно, частота электродвижущей силы такого генератора будет равна /=-&-» (588) где f— частота в герцах; п—число оборотов ротора в минуту. Если генератор имеет число пар полюсов р, то соответствен- но этому частота электродвижущей силы такого генератора бу- дет в р раз больше частоты электродвижущей силы двухполюс- ного генератора: (589) Пример 220. Требуется определить частоту электродвижущей силы гене- ратора переменного тока, если ротор генератора развивает п = 3000 об/мин, а число пар полюсов генератора р = 1. Рис. 494. Развертка части якорной обмотки и магнитных полюсов трехфазного генератора Решение. Применяя формулу (589), найдем PJL - 1'3000 — 50 ги I 60 “ 60- г”‘ Из формулы (589) следует, что частота электродвижущей си- лы генератора находится в строго определенной зависимости от числа оборотов ротора генератора в минуту. Генераторы пере- менного тока, частота электродвижущей силы которых находится в строго определенной зависимости от скорости вращения ротора его, называются синхронными генераторами. 746
В отличие от синхронных генераторов в технике переменных токов имеются электрические машины, для которых нет строгой зависимости частоты электродвижущей силы (тока) от числа оборотов ротора машины. Электрические машины подобного ро- да называются асинхронными электрическими машинами пере- менного тока. Последние обычно применяются как электродви- гатели переменного тока. С ними мы более подробно ознако- мимся несколько ниже. § 227. РЕЖИМ РАБОТЫ СИНХРОННОГО ГЕНЕРАТОРА Основными величинами, характеризующими синхронный ге- нератор, являются: напряжение на зажимах (7, нагрузка /, пол- ная мощность Pi (ква), число оборотов ротора в минуту и, ко- эффициент мощности cos Важнейшие рабочие характеристики синхронного генератора следующие: 1) характеристика холостого хода; 2) внешняя характеристика; 3) регулировочная характеристика. Характеристика холостого хода. Электродвижущая сила гене- ратора пропорциональна величине магнитного потока Ф, созда- ваемого током возбуждения 4, и числу оборотов п ротора гене- ратора в минуту: Е = глФ, (590) где с — коэффициент пропорциональности. Хотя величина электродвижущей силы синхронного генера- тора зависит от числа оборотов п ротора, регулировать ее путем изменения скорости вращения ротора невозможно, так как с чис- лом оборотов ротора генератора связана частота электродвижу- щей сил&, которая должна быть сохранена постоянной. Следовательно, остается единственный способ регулировки величины электродвижущей силы синхронного генератора — это изменение основного магнитного потока Ф. Последнее обычно до- стигается путем регулирования тока возбуждения 4 с помощью реостата, введенного в цепь возбуждения генератора. В том слу- чае когда обмотка возбуждения питается током от генератора постоянного тока, сидящего на одном валу с данным синхронным генератором, ток возбуждения синхронного генератора регули- руется изменением напряжения на зажимах генератора постоян- ного тока. Зависимость электродвижущей силы Е синхронного генерато- ра от тока возбуждения 4 при постоянстве номинальной скоро- сти вращения ротора (n = const) и нагрузке, равной нулю (/ = 0), называется характеристикой холостого хода генератора. 747
Рис. 495. Характеристика холостого хода синхронного генератора На рис. 495 приведена характеристика холостого хода синхронного гене- ратора. Здесь восходя- щая ветвь 1 кривой сня- та при возрастании то- ка 4 от нуля до 4W а ни- w сходящая ветвь 2 кривой— при изменении 4 от 4™ ДО 4 = 0. Несовпадение вос- ходящей 1 и нисходящей 2 ветвей объясняется оста- точным магнетизмом. Чем больше площадь, ограни- ченная этими ветвями, тем больше потерь энер- гии в стали синхронного генератора на перемагничивание. Крутизна подъема кривой холостого хода на ее начальном прямолинейном участке характеризует магнитную цепь синхрон- ного генератора. Чем меньше расход ампер-витков в воздушных зазорах генератора, тем при прочих одинаковых условиях будет круче характеристика холостого хода генератора. Внешняя характеристика генератора. Напряжение на зажи- мах нагруженного синхронного генератора' зависит от электро- движущей силы Е генератора, от падения напряжения в актив- ном сопротивлении его статорной обмотки, падения напряжения, обусловленного электродвижущей силой самоиндукции рассея- ния Es, и падения напряжения, обусловленного реакцией якоря. Электродвижущая сила рассеяния £5, как из- вестно, зависит от магнитного потока рассеяния Ф5, который не проникает в магнитные полюса ротора генератора и, следова- тельно, не изменяет степени намагничивания генератора. Элек- тродвижущая сила самоиндукции рассеяния Es генератора отно- сительно мала, а поэтому практически ею можно пренебречь. В соответствии с этим ту часть электродвижущей силы генера- тора, которая компенсирует электродвижущую еилу самоиндук- ции рассеяния Es,‘можно считать практически равной нулю. Реакция якоря оказывает более заметное влияние на режим работы синхронного генератора и, в частности, на вели- чину напряжения на его зажимах. Степень этого влияния зави- сит не только от величины нагрузки генератора, но и от харак- тера нагрузки. Рассмотрим вначале влияние реакции якоря синхронного гене- ратора для случая, когда нагрузка генератора носит чисто актив- ный характер. Для этой цели возьмем часть схемы работающего синхронного генератора, изображенную на рис. 496,а. Здесь по- казаны часть статора с одним активным проводником якорной обмотки и часть ротора V несколькими его магнитными полюса- 748
ми. В рассматриваемый момент времени северный полюс одного из электромагнитов, вращающихся вместе с ротором против ча- совой стрелки, как раз проходит под активным проводником статорной обмотки. Электродвижущая сила, индуктированная в этом проводнике, направлена к нам из-за плоскости рисунка. А так как нагрузка генератора носит чисто активный характер, то ток / в якорной обмотке совпадает по фазе с электродвижу- щей силой. Следовательно, в активном проводнике статорной об- мотки ток течет к нам из-за плоскости рисунка. Магнитные ли- нии поля, создаваемого электромагнитами, показаны здесь сплошными линиями, а магнитные линии поля, создаваемого то- ком провода якорной обмотки, — пунктирной линией. Внизу на рис. 496,а показана векторная диаграмма магнитной индукции результирующего магнитного поля, находящегося над северным полюсом электромагнита. Здесь мы видим, что магнитная индук- ция В основного магнитного поля, создаваемого электромагни- том, имеет радиальное направление, а магнитная индукция Вл магнитного поля тока якорной обмотки направлена вправо и пер- пендикулярно вектору В. Результирующая магнитная индукция Врез направлена вверх и вправо. Это значит, что в результате сложения магнитных полей произошло некоторое искажение ос- новного магнитного поля. Слева от северного полюса оно не- сколько ослабилось, а справа — несколько усилилось. Йетрудно видеть, что радиальная составляющая вектора результирующей магнитной индукции, от которой по сути дела зависит величина индуктированной электродвижущей силы генератора, не измени- лась. Следовательно, реакция якоря при чисто активной нагруз- ке генератора не влияет на величину электродвижущей силы ге- нератора. Это значит, что и падение напряжения в генераторе при чисто активной нагрузке обусловлено только падением на- пряжения в активном сопротивлении генератора, если прене- бречь электродвижущей силой самоиндукции рассеяния. Рис. 496. Влияние реакции якоря для нагрузок: а — актирного; б — индуктивного; в — емкостноТо характера 749
Теперь допустим, что нагрузка синхронного генератора НоСит чисто индуктивный характер. В этом случае ток I отстает по фа- зе от электродвижущей силы Е на угол -у-. Это значит, что мак- симум тока возникает в проводе несколько позднее, чем макси- мум электродвижущей силы. Следовательно, когда в проводе якорной обмотки ток достигнет максимального значения, север- ный полюс N будет уже не под этим проводом, а сместится не- сколько дальше в направлении вращения ротора, как это пока- зано на рис. 496,6. В этом случае магнитные линии (пунктирные линии) магнитного потока якорной обмотки замыкаются через два соседних разноименных полюса N и S и направлены навст- речу магнитным линиям основного магнитного поля генератора, создаваемого магнитными полюсами. Это приводит к тому, что основное магнитное поле не только искажается, но и делается несколько слабее. На рис. 496,6 приведена векторная диаграмма магнитных индукций: основного магнитного поля В, магнитного поля, обусловленного реакцией якоря Вя, и результирующего магнитного поля Врез. Здесь мы видим, что радиальная состав- ляющая магнитной индукции результирующего магнитного поля стала меньше магнитной индукции В основного магнитного поля на величину ДВ. Следовательно, стала меньше и индуктирован- ная электродвижущая сила, так как она обусловлена радиаль- ной составляющей магнитной индукции. А это значит, что напря- жение на зажимах генератора при всех прочих равных условиях будет меньше, чем напряжение при чисто активной нагрузке ге- нератора. Если генератор имеет нагрузку чисто емкостного характера, то ток в нем опережает по фазе электродвижущую силу на угол -у-. Ток в проводниках якорной обмотки генератора теперь до- стигает максимума раньше, чем электродвижущая сила Е. Сле- довательно, когда в проводе якорной обмотки (рис. 496,в) ток достигнет максимального значения, северный полюс N еще не подойдет под этот провод. В этом случае магнитные линии (пунктирные линии) магнитного цотока якорной обмотки замы- каются через два соседних разноименных полюса N и S и на- правлены попутно с магнитными линиями основного магнитного поля генератора. Это приводит к тому, что основное магнитное поле генератора не только искажается, но и несколько усили- вается. На рис. 496,в приведена векторная диаграмма магнитной индукции: основного магнитного поля В, магнитного поля, обус- ловленного реакцией якоря Вя, и результирующего магнитного поля Врез. Мы видим, что радиальная составляющая магнитной индукции результирующего магнитного поля стала больше маг- нитной индукции В основного магнитного поля на величину ДВ. Следовательно, увеличилась и индуктированная электродвижу- 750
щая сила генератора. А это значит, что напряжение на зажимах генератора при всех прочих одинаковых условиях станет больше, чем напряжение при чисто'индуктивной нагрузке генератора. Выяснив влияние реакции якоря на электродвижущую силу синхронного генератора при различных по своему характеру на- грузках, перейдем к выяснению внешней характеристики гене- ратора. Внешней характеристикой , синхронного генератора называет- ся зависимость напряжения U на его зажимах от нагрузки I при постоянной скорости вращения ротора (n = const), постоянстве тока возбуждения (4 = const) и постоянстве коэффициента мощ- ности (cos <р = const}. На рис. 497 приведены внешние характеристики синхронного генератора для различных по своему характеру нагрузок. Кри- вая 1 выражает внешнюю характеристику при активной нагруз- ке (cos <р = 1,0). В этом случае напряжение на зажимах генера- тора падает при изменении нагрузки от холостого хода до номи- нальной в пределах 10—20% напряжения при холостом ходе ге- нератора. Кривая 2 выражает внешнюю характеристику при ак- тивно-индуктивной нагрузке (cos ср == 0,8). В этом случае напря- жение на зажимах генератора падает быстрее из-за размагничи- вающего действия реакции якоря. При изменении нагрузки гене- ратора . от холостого хода до номинальной напряжение умень- шается в пределах 20—30% напряжения при холостом ходе. Кривая 3 выражает внешнюю характеристику синхронного гене- ратора при активно-емкостной нагрузке (cos ср = 0,8). В этом случае напряжение на зажимах генератора несколько растет из- за намагничивающего действия реакции якоря. Регулировочная характеристика синхронного генератора вы- ражает зависимость то- ка возбуждения iB ге- нератора от нагрузки I при постоянстве дей- ствующего значения на- пряжения на зажимах генератора (U = const), постоянстве числа обо- ротов ротора генера- тора в минуту (л = = const) и постоянстве коэффициента мощно- сти (cos ср = const). На рис. 498 приве- дены три регулиро- вочные характеристики синхронного генерато- ра. Кривая 1 относит- ся к случаю активной U Рис. 497. Внешние характеристики генера- тора переменного тока для различных на- грузок (/ — активной, 2 — индуктивной и 5 — емкостной) 751
2 1 3 I о Рис. 498. Регулировочные характери- стики генератора переменного тока для различных нагрузок (/ — активной, 2 — индуктивной и 3 — емкостной) нагрузки (cos ? = 1). Здесь мы видим, что с ростом на- грузки I генератора ток воз- буждения растет. Это понят- но, так как с ростом на- грузки I увеличивается паде- ние напряжения/в активном сопротивлении якорной об- мотки генератора и требует- ся увеличить электродвижу- щую силу Е генератора пу- тем увеличения тока возбу- ждения JR, чтобы сохранить постоянство напряжения U, Кривая 2 относится к слу- чаю активно-индуктивной на- грузки при cos ср = 0,8. Эта кривая поднимается круче, чем кривая /, вследствие размагничивающего действия реакции якоря, снижающего величину электродвижущей силы Е, и, сле- довательно, напряжение U на зажимах генератора. Кривая 3 от- носится к случаю активно-емкостной нагрузки при cos ср = 0,8. Эта кривая показывает, что с ростом нагрузки генератора тре- буется меньший ток возбуждения 4 генератора для поддержания постоянства напряжения на его зажимах. Это понятно, так как в этом случае реакция якоря усиливает основной магнитный по- ток и, следовательно, способствует увеличению электродвижущей силы генератора и напряжения на его зажимах. § 228. ПАРАЛЛЕЛЬНАЯ РАБОТА СИНХРОННЫХ ГЕНЕРАТОРОВ Синхронные генераторы соединяются друг с другом парал- лельно для совместной работы на общую электрическую сеть. В те часы суток, когда нагрузка электрической станции мала, работает часть ее генераторов, а затем по мере роста нагрузки включаются и резервные генераторы. Такой способ эксплуата- ции генераторов выгоден в том отношении, что каждый из них работает на полную мощность и, следовательно, с наибольшим коэффициентом полезного действия. Рассмотрим, каков порядок подключения на параллельную работу однофазных синхронных генераторов и как они работают на общую электрическую сеть. Допустим, что однофазный синхронный генератор, который назовем генератором № 1, работает на полную нагрузку /ив силу этого возникла необходимость подключения к нему на па- раллельную работу генератора № 2 (рис. 499). Основное требо- вание, которое должно быть выполнено при подключении, сво- дится к следующему: в момент подключения генератора № 2 к 752
распределительным шинам его электродвижущая сила должна быть численно равна напряжению на этих шинах, находиться с ним в противофазе и иметь одинаковую с ним частоту, т. е. |£2| = |<Д ^ = -«, или, иначе, Е2т • sin = — 6^-sina)/, где е2—мгновенное значение электродвижущей силы генера- тора № 2; и — мгновенное значение напряжения на распределитель- ных шинах. Если это требование не будет выполнено, то из сети (от ге- нератора № 1) может пойти большой ток в генератор № 2 и по- следний начнет работать в режиме электродвигателя, что приве- дет к аварии. Прибор, с помощью которого можно правильно определить момент подключения генератора к распределительным шинам на параллельную работу, называется синхроноскопом. В про- стейшем случае синхроноскоп состоит из двух электрических ламп Л1 и Л2, включенных в провода, соединяющие генератор с распределительными шинами (рис. 499). Посмотрим, каким же образом, пользуясь синхроноскопом, можно подключить генератор № 2 к распределительным шинам на параллельную работу с генератором № 1. Рис. 499. Схема параллельного включения однофазных генераторов переменного тока 753
Вначале с помощью первичного двигателя приводят во вра- щение ротор генератора № 2, добиваясь того, чтобы он развил номинальное число оборотов в минуту. В этом случае частота электродвижущей силы генератора № 2 практически будет рав- на частоте электродвижущей силы генератора № 1. После этого в генераторе № 2 устанавливают ток возбужде- ния такой величины, чтобы показания вольтметров и Уз были одинаковыми. В этом случае величины электродвижущей силы генератора № 2 и напряжения U на распределительных шинах практически будут равны. Затем добиваются того, чтобы электродвижущая сила генера- тора № 2 и напряжение U на распределительных шинах были в противофазе. Для этой цели изменяют в весьма небольших пре- делах скорость вращения ротора -генератора № 2 и при этом наблюдают за лампами синхроноскопа. Если скорости вращения роторов генераторов № 1 и № 2 зна- чительно различаются, то электродвижущая сила генератора № 2 и напряжение на распределительных шинах не компенсируют пол- ностью друг друга. В этом случае напряжение на лампах син- хроноскопа не равно нулю и лампы светятся. При постепенном приближении скорости вращения ротора ге- нератора № 2 к скорости вращения ротора генератора № 1 на- ступает такой момент, когда лампы синхроноскопа начинают ми- гать, т. е. то потухать, то вновь зажигаться. Чем меньше разли- чаются частоты генераторов № 1 и № 2, тем реже мигают лампы. Так как лампы синхроноскопа тухнут при напряжении, не равном нулю, то' их потухание еще не является полным под- тверждением того, что момент подключения генератора № 2 на- ступил. Поэтому к одной из ламп синхроноскопа параллельно подключается так называемый нулевой вольтметр Уо. Когда лампы синхроноскопа начнут мигать очень редко, по- тухая примерно на 3—5 секунд, то в один из очередных момен- тов потухания их, когда нулевой вольтметр Уо покажет напряже- ние, практически равное нулю, замыкают рубильник К2 и, следо- вательно, подключают генератор № 2 к распределительным шинам. Если после подключения генератора № 2 к распределитель- ным шинам окажется, что скорость вращения его ротора не- сколько меньше номинальной, т. е. частота электродвижущей силы генератора меньше частоты напряжения на распределитель- ных шинах, то из сети в генератор № 2 пойдет небольшой актив- ный ток, который заставит якорь генератора увеличить скорость своего вращения до требуемой величины. Наоборот, если ско- рость вращения ротора генератора № 2 после подключения его к распределительным шинам окажется больше номинальной, то генератор № 2 пошлет в сеть активный ток, который снизит ско- рость вращения ротора до требуемой величины. .Таким образом, генератор № 2, включившись на параллельную работу в сеть, 754
автоматически выравняет скорость вращения своего ротора и до- стигнет синхронизма с генератором № 1. Генератор № 2, подключенный к распределительным шинам, еще не будет давать ток потребителям, так как в момент под- ключения его к сети его электродвижущая сила численно равна напряжению на распределительных шинах. Следовательно, вна- чале генератор № 2 будет работать в режиме холостого хода. Чтобы этот генератор нагрузить током, необходимо сообщить ему добавочное количество энергии от первичного двигателя, вращающего его ротор. Первичный двигатель после подачи на него большого количества топлива или пара и т. п. увеличивает свой вращающий момент и, следовательно, число оборотов ро- .тора генератора № 2. В результате электродвижущая сила гене- ратора № 2 начинает опережать на некоторый угол по фазе на- пряжение на распределительных шинах. А это в свою очередь приводит к тому, что между генераторами № 1 и № 2 устанав- ливается уравнительный активный ток, посылаемый генератором № 2 в генератор № 1. Этот уравнительный ток совпадает по фазе с электродвижущей силой генератора № 2 и находится в про- тивофазе с электродвижущей силой генератора № 1. JJ силу это- го ток (нагрузка) генератора № 1 начинает уменьшаться, а ток генератора № 2 — увеличиваться, т. е. генератор № 1 разгру- жается, а генератор № 2 нагружается. Дальнейшее увеличение вращающего момента первичного двигателя генератора № 2 при- водит к дальнейшей разгрузке генератора № 1 и увеличению на- грузки генератора № 2. Следовательно, для того чт о б ы н ео б х о д и м ы м обра- зом распределить нагрузку между параллель- но включенными синхронными генераторами, следует соответствующим образом распреде- лить подачу им энергии от их первичных дви- гателей. Если бы возникла необходимость отключить от сети генера- тор № 1, оставив работающим только генератор № 2, то надо было бы постепенно перевести всю нагрузку с генератора № 1 на генератор № 2 путем увеличения подачи последнему энергии от его первичного двигателя, а затем уже отключить генератор № 1 от сети рубильником Ki. Необходимо отметить, что при параллельной работе синхрон- ных генераторов невозможно перевести нагрузку с одного гене- ратора на другой путем изменения тока возбуждения. Дело в том, что при изменении тока возбуждения в одном из генерато- ров энергия, получаемая им от первичного двигателя, не возра- стает. Следовательно, скорость вращения ротора генератора не изменяется и не создается уравнительного активного тока между параллельно работающими генераторами. Между ними появляется только реактивный уравнительный ток, средняя мощ- ность которого за период равна нулю. 755
Рассмотренная нами схема синхроноскопа, при которой мо- мент включения генератора в сеть совпадает с потуханием ламп и нулевым показанием нулевого вольтметра, называется схе- мой на потухание. При включении на параллельную , работу трехфазных син- хронных генераторов, помимо требований, перечисленных выше для включения однофазных генераторов, необходимо выполнить Рис. 500. Схема параллельного включения трехфазных генера- торов еще одно дополнительное требование — получить одинако- вое чередование фаз генераторов. Это значит, что одноименные фазы генераторов должны быть подключены к од- ной и той же распределительной шине. На рис. 500 приведена схема включения трехфазного синхронного генератора № 1 в сеть и схема трехфазного синхронного генератора № 2 с лам- пами Л1 Л2, Л3 и вольтметром Vo, с помощью которых можно подключить генератор № 2 к распределительным шинам. В мо- мент времени, когда лампы Лъ Л2 и Л3 синхроноскопа начинают одновременно гаснуть на относительно продолжительное время 756
(3—5 секунд) и нулевой вольтметр показывает нуль, замыкается рубильник /<2 и, следовательно, генератор № 2 подключается к распределительным шинам на параллельную работу с генера- тором № 1. После подключения генератора № 2 он будет вначале рабо- тать в режиме холостого хода. Для загрузки его током необхо- димо увеличить вращающий момент его первичного двигателя, этим несколько увеличить число оборотов ротора генератора и дать ему возможность питать током потребители электрической энергии. Если при подготовке трехфазного генератора к подключению к распределительным шинам окажется, что лампы Л2 и Л» загораются и тухнут поочередно, то это значит, что в данном случае не соблюден порядок чередования фаз генераторов.
ГЛАВА XXXVIIt ЭЛЕКТРОДВИГАТЕЛИ ПЕРЕМЕННОГО ТОКА § 229. СИНХРОННЫЙ ЭЛЕКТРОДВИГАТЕЛЬ Синхронный генератор переменного тока, приспособленный Для работы в режиме электродвигателя, называется синхронным электродвигателем. Если синхронный генератор питать от сети переменного тока электрической энергией, то он при известных условиях может начать работать как электродвигатель. Рассмот- рим, какие же условия необходимо выполнить для этого. Предположим, что ротор синхронной трехфазной машины на- ходится в состоянии покоя. Подключим к обмотке ротора (об- мотке возбуждения) источник электрической энергии постоянного тока и установим нормальный ток возбуждения и нормальный магнитный поток в генераторе. Если теперь подключить статор- ную обмотку генератора к трехфазной сети, то возникнет вра- щающееся магнитное поле трехфазного тока статорной обмотки. Число оборотов вращающегося магнитного Поля зависит от ча- стоты переменного тока и для двухполюсной машины равно n = f£)f. Если, например, f — 50 гц, то вращающееся магнитное поле имеет п = 60 • 50 = 3000 об)мин. Постоянное магнитное поле ротора и вращающееся магнит- ное поле статора взаимодействуют друг с другом, но ротор не сможет вращаться, так как его полюсы за каждый оборот вра- щающегося магнитного поля испытывают силу, несколько раз меняющую направление. Эти силы попеременно стремятся вра- щать ротор в прямо противоположных направлениях. А так как ротор обладает относительно большой инерцией, то под влия- нием этих переменных сил он вообще не будет двигаться. Следовательно, привести во вращение ротор син- хронной машины одним лишь включением ее в сеть и заставить тем самым работать как элек- тродвигательневозможно. Допустим, что при помощи постороннего двигателя синхрон- ная машина приведена в движение, ее ротору сообщено необхо- 758
димое число оборотов и она при помощи синхроноскопа под- ключена к сети переменного тока. Если элёктродвижущая сила этого генератора равна напряжению сети и они совпадают друг с другом по частоте, то подключенный генератор не дает тока в сеть и не получает его из сети, т. е. работает на холостом ходу. Если увеличить мощность двигателя, вращающего ротор генера- тора, то ротор начнет вращаться с несколько повышенной скоро- стью и генератор будет посылать ток в сеть, работая в генератор- ном режиме. Чем больше мощность, подводимая от подвижного двигателя к генератору, тем большую нагрузку она берет на себя. Теперь допустим, что от синхронного генератора, подключен- ного к распределительным шинам и работающего вхолостую, отсоединяется первичный двигатель, приводящий в движение его ротор. С исчезновением механической силы, приводящей во вра- щение ротор генератора, число оборотов ротора начнет умень- шаться и в соответствии с этим электродвижущая сила генера- тора будет отставать по фазе от напряжения на распределитель- ных шинах. Благодаря этому в якорную (статорную) обмотку генератора из сети поступит активный ток, совпадающий по фазе с напряжением на распределительных шинах и находящийся в противофазе с электродвижущей силой синхронного генератора. Сила взаимодействия этого тока с вращающимся магнитным по- лем заставит ротор генератора увеличить скорость своего вра- щения. Это увеличение числа оборотов ротора будет продол- жаться до тех пор, пока он не начнет вращаться с синхронной скоростью, т. е. со скоростью, равной скорости вращающегося магнитного поля. Однако синхронная машина, потребляя теперь некоторый активный ток из сети, начала уже работать электро- двигателем, но, правда, пока еще в режиме холостого хода. Вра- щающий момент, развиваемый синхронным электродвигателем, в данном случае пока уравновешен моментом сил сопротивления самого вращающегося ротора. Теперь допустим, что синхронный электродвигатель, работаю- щий в режиме холостого хода, получил некоторую механическую нагрузку, например, его соединили с помощью ременной пере- дачи с валом какого-нибудь механического станка. Так как момент сил сопротивления стал теперь больше вращающего мо- мента электродвигателя, то скорость вращения ротора начнет снижаться. Это приведет к тому, что электродвижущая сила, ин- дуктированная вращающимся магнитным полем в обмотке ста- тора, будет отставать по фазе от напряжения на распределитель- ных шинах. Благодаря этому увеличится активный ток, посту- пающий из сети в электродвигатель, с ним вместе начнет расти вращающий момент электродвигателя, а следовательно, и ско- рость вращения ротора. Последняя будет увеличиваться до тех пор, пока ротор вновь не начнет вращаться с синхронной ско- ростью. Но теперь в отличие от режима холостого хода электро- двигатель будет потреблять из сети больший активный ток. 49* 759
Чем больше механическая нагрузка синхронного электродви- гателя, тем при всех прочих одинаковых условиях он больше по- требует энергии из с.ети, тем больше в его статоре активный ток и тем больший он развивает враи!ающий момент. Скорость же вращения ротора при стационарной работе. электродвигателя остается постоянной и равной синхронной скорости. В моменты изменения механической нагрузки электродвигатель несколько выходит из синхронизма, но это продолжается ничтожно малое время, в течение которого электродвигатель автоматически на- бирает вновь синхронную скорость, правда, уже при несколько ином электрическом режиме его работы. -Теперь допустим, что механическая нагрузка синхронного электродвигателя в некоторый момент стала чрезмерно большой, превышающей номинальную величину. В этом случае ротор электродвигателя резко снизит скорость своего вращения, так что она станет значительно отличаться от скорости вращающе- гося магнитного поля. Ротор выпадет из синхронизма и остано- вится. Объясняется это тем, что после некоторого предельного минимального числа оборотов в минуту ротора сила взаимодей- ствия вращающегося магнитного поля и магнитного поля ротора начинает периодически менять свое направление, так что сред- няя механическая сила, действующая на ротор, становится рав- ной нулю. Чтобы снова запустить электродвигатель, необходимо снять с него механическую нагрузку, сообщить его ротору с помощью первичного двигателя синхронную скорость вращения, затем под- ключить его к сети и после этого механически нагрузить. Теперь посмотрим, как влияет на режим работы синхронного электродвигателя изменение его тока возбуждения. Если механическая нагрузка электродвигателя постоянна, то он развивает на своем валу постоянную мощность (Р = const) независимо от величины тока возбуждения в цепи. Не зависит от тока возбуждение и скорость вращения ротора: она обуслов- лена только частотой напряжения сети, к которой подключен электродвигатель. Если изменяется ток возбуждения электродвигателя, то должно изменяться магнитное поле обмотки возбуждения (ро- торной обмотки), а вместе с ним — индуктированная электродви- жущая сила и ток I в статорной обмотке. Посмотрим, как в этом случае изменяется ток в статорной обмотке при условии постоян- ства мощности, развиваемой электродвигателем на своем валу (Р = const), и постоянства действующего значения напряжения, приложенного к электродвигателю ((7 = const). Если мощность синхронного электродвигателя постоянна (Р = const) и действующее значение напряжения, приложенного к электродвигателю, также постоянно (С/= const), то Р = ]/з” Ub cos <р =* const, 760
или, иначе, /•cos 9== с = const, (591) где с — некоторая постоянная величина. Из последнего соотношения следует, что произведение тока в статорной обмотке синхронного электродвигателя на коэффи- циент мощности его cos ср при Р = const и U = const — величина постоянная. Следовательно, если в результате изменения тока возбужде- ния синхронного электродвигателя ток в статорной обмотке уве- личится в т раз, то в такое же число раз уменьшится коэффи- циент мощности cos <р электродвигателя. И наоборот, если ток 1 уменьшится в т раз, то в такое же число раз увеличится коэф- фициент мощности cos ср. Мощность же, развиваемая электродви- гателем, останется при этом неизменной, если сохранятся неиз- менными механическая нагрузка электродвигателя и напряже- ние на его зажимах. Дело в том, что изменение тока возбуждения в синхронном электродвигателе приводит к изменению реактивной составляю- щей тока статорной обмотки и, следовательно, к изменению угла сдвига фаз тока и напряжения. Это значит, что, изменяя ток возбуждения электродвигателя, можно регулировать cos <р в цепи. Нормальным током возбуждения синхронного электродвига- теля называется такой ток, при котором коэффициент мощности cos ср = 1 и действующее значение тока в обмотке статора имеют минимальные значения. Если ток возбуждения меньше нормального, то режим в цепи статора носит индуктивный характер, т. е. ток там отстает по фазе от напряжения на некоторый угол ср. Наоборот, если ток возбуждения больше нормального, то режим в цепи статорной обмотки носит емкостный характер, т. е. ток опережает по фазе напряжение на некоторый угол ср. На рис. 501 приведен гра- фик зависимости величины тока I в статорной обмотке синхронного электродвигате- ля от тока возбуждения ZB. Левая ветвь этого графика относится к случаю недо- возбуждения электродвига- теля (индуктивный режим, в цепи статора), а правая — к случаю перевозбуждения (емкостный режим). Мини- мум тока I относится к слу- чаю нормального тока воз- буждения, при котором ток Рис. 501. График зависимости тока в статоре синхронного электродвига- теля от тока возбуждения 761
статорной обмотки совпадает по фазе с приложенным напряже- нием и соответственно этому cos ср =1. Этот режим наиболее благоприятен с точки зрения загрузки сети током. Свойство перевозбужденного синхронного электродвигателя работать в емкостном режиме используется для повышения коэффициента мощности cos ср в электрических сетях. Электри- ческим станциям, питающим электрической энергией большое ко- личество асинхронных электродвигателей, приходится обмени- ваться с ними большим количеством реактивной энергии, что значительно снижает коэффициент полезного действия ' станции и загружает генератор реактивным током. Включение в такие сети перевозбужденных синхронных электродвигателей, работаю- щих в емкостном режиме, значительно разгружает станцию от реактивного тока, так как потребители с индуктивным характе- ром нагрузки будут обмениваться реактивной энергией с пере- возбужденными электродвигателями. Пример 221. Генератор трехфазного тока, имеющий линейное напряже- ние Un — 220 в, питает током / = 200 а потребители с индуктивным харак- тером нагрузки и с коэффициентом мощности cos ф = 0,655. Требуется опре- делить, какую реактивную мощность и реактивный ток должен дать в сеть перевозбужденный синхронный электродвигатель, чтобы генератор был пол- ностью разгружен от реактивной мощности и реактивного тока. Решение. Активная мощность, развиваемая генератором, равна /3 67-cos? = 1,73-220-200-0,655 ж 50000 вт = 50 кет. Активная составляющая тока генератора la = /-cos ? = 200-0,655 = 131 а. Реактивная составляющая тока генератора /г = J/p_ /2 = J/2002 — 1312 ж 150 а. Перевозбужденный синхронный электродвигатель должен дать реактив- ную составляющую тока /г = 150 а и реактивную мощность Рг = уз UIr = 1,73-220-150 ж 57100 ва = 57,1 квар. Пуск в ход маломощного синхронного электродвигателя мож- но осуществить и без помощи постороннего первичного двига- теля. Для этой цели у синхронного электродвигателя размыкают обмотку возбуждения и подключают статорную обмотку к трех- фазной сети. Вращающееся магнитное поле тока статорной об- мотки при своем движении индуктирует вихревые токи в сердеч- нике ротора и, взаимодействуя с ними, заставляет ротор вра- щаться вместе с собой. Когда ротор достигнет практически син- хронной скорости вращения, замыкают обмотку возбуждения, и электродвигатель автоматически начинает работать синхронным электродвигателем. 762
Основное Достоинство синхронного электродвигателя— по- стоянство скорости вращения его ротора и независимость ее от механической нагрузки электродвигателя. Однако наряду с этим синхронный электродвигатель обла- дает рядом существенных недостатков. Например, он требует для своего пуска в ход первичного двигателя, выпадает из син- хронизма при перегрузке и останавливается, нуждается в посто- роннем источнике постоянного тока, питающем постоянным то- ком его обмотку возбуждения, и сложен по конструкции. § 230. АСИНХРОННЫЙ ЭЛЕКТРОДВИГАТЕЛЬ Большое применение в современной электротехнике получил асинхронный электродвигатель трехфазного тока. Честь изобретения асинхронного электродвигателя принадле- жит талантливому русскому инженеру-электрику М. О. Доливо- Добровольскому. Асинхронный электродвигатель основан на принципе взаимодей- ствия вращающегося магнитного поля трехфазного тока с то- ками, индуктированными этим полем, в системе подвижных про- водников. Неподвижная часть асинхрон- ного электродвигателя (рис. 502), называемая статором, состоит из ярма, на внутренней поверхно- сти которого размещена трехфаз- ная обмотка из ряда симметрично расположенных катушек. Все три обмотки выполнены одинаково и Рис. 502. Статор асинхронного имеют одинаковые параметры г электродвигателя и L, Благодаря этому асинхрон- ный электродвигатель в нормальных условиях работы представ- ляет собой равномерно нагруженную трехфазную систему. Токи в его обмотках (катушках) сдвинуты по фазе относительно друг друга на 120°, и амплитуды этих токов одинаковы: * *1 = 4г sin К—?), 4 = Ц • Sin --------g- гс — 4 = lm • sin (ш/----ГС — <p 763
Каждый из этих токов создает свой собственный магнитный поток: Ф, = Фот • sin (wt — <р), Ф2 = Фот-зт (®t--|-те —<р Ф3 = Фот-зш (a>t-g-те —<р Эти три магнитных потока создают результирующий магнит- ный поток Ф=1,5-Фт, (592) вращающийся с постоянной угловой скоростью <о.. Таким обра- зом, статорная обмотка асинхронного электродвигателя создает вращающееся магнитное поле. Число оборотов вращающегося магнитного поля зависит от числа пар его полюсов. Если в статоре только три катушки, то вращающееся магнитное поле имеет только одну пару полюсов. В этом случае за один период Т магнитный поток делает один оборот. Следовательно, при двухполюсной системе вращаю- щееся магнитное поле за одну минуту сделает число оборотов «1 = 60/1( где fi — частота переменного тока. Если статорную обмотку выполнить из шести катушек, по- люсы которых сдвинуты пространственно на 60° относительно друг друга, и соединить последовательно первую катушку с чет- вертой, вторую с пятой и третью с шестой, то при питании их трехфазным током получим внутри статора четырехполюсное вращающееся магнитное поле. Оно будет совершать вдвое мень- ше оборотов, чем двухполюсное магнитное поле. На рис. 503 схе- матически изображены статор и ротор четырехполюсного асин- хронного электродвигателя. Увеличение числа катушек статорной обмотки электродвига- теля, т. е. числа его полюсов, влечет за собой изменение числа оборотов его вращающегося магнитного поля. Если в этом поле р пар полюсов, то число оборотов вращающегося магнитного поля Пример 222. Определить число оборотов вращающегося магнитного поля в статоре асинхронного трехфазного двигателя, если это поле имеет три пары полюсов (р = 3), а частота переменного тока / = 50 гц. Решение. По формуле (593) находим п = 60Л=60 50 = 1000 об/Лкн. Р 3 764
Внутри статора асинхронного электродвигателя помещена подвижная часть его, называемая ротором. Ротор асинхронного электродвигателя представляет собой цилиндр, собранный из отдельных железных листов, изолированных друг от друга для устранения потерь на вихревые токи. На железном цилиндре Рис. 503. Схемы статора и ротора четырехпо- люсного асинхронного электродвигателя ротора размещена его обмотка. В маломощных двигателях это — ряд толстых неизолированных медных или алюминиевых прутьев, скрепленных по концам общими кольцами («беличье колесо», рис. 504), или изолированные проводники, замкнутые накоротко. Электродвигатели с такими короткозамкнутыми рото- рами обычно обладают малыми мощностями, примерно 1,5—2 л. с. В более мощных электродвигателях, где в роторе могут воз- никнуть большие пусковые токи, опасные для установки, после- довательно с обмоткой ротора включают реостат (рис. 505). У таких электродвигателей есть приспособление, дающее воз- можность обмотку ротора включать последовательно с реостатом 765
Рис. 504. Ротор „беличье колесо1* и замыкать ее накоротко. Устройство этого приспособления за- ключается в следующем (рис. 506). Концы трех фаз обмотки ротора соединены накоротко на задней стороне ротора, а начала их поодиночке присоединены к трем медным кольцам 1, сидящим на валу 2 с передней сто- роны ротора. Реостат ротора состоит из трех сопротивлений, Рис. 505. Схема асинхронного электродвигателя с пусковым реостатом в цепи ротора концы которых соединены в общую точку, а к началам их при- соединены провода, подключенные к щеткам. Щетки при помощи соответствующих приспособлений прижимаются к кольцам ро- тора и этим создают соединенную «звездой» замкнутую трехфаз- ную цепь, состоящую из последовательно соединенного ротора с реостатом. Замыкание обмотки ротора накоротко осущест- Рис. 50S. Приспособление для замыкания обмотки ротора накоротко: I — медные кольца; 2 — вал; 3 — втулка; 4 — рь^чаг 766
Ёляется особой втулкой 3, перемещение которой совершается при повороте рычага 4, Одновременно с поворотом рычага и замыка- нием обмотки ротора на- коротко поднимаются щет- ки и разрывают контакт проводов реостата с коль- цами. Если катушки статора подключить к трехфазной сети, то в системе ста- тора возникнет вращаю- щееся магнитное поле, которое будет пересекать проводники ротора и ин- дуктировать в них элек- тродвижущую силу. Если проводники ротора зам- кнуты, то в них возникнет индуктированный ток. В результате взаимодейст- вия вращающегося маг- нитного поля и индукти- рованного тока ротор элек- Рис. 507. Вращающееся магнитное поле увлекает за собой ротор электродвигателя тродвигателя придет в движение, вращаясь в ту же сторону, что и вращающееся маг- нитное поле, т. е. это поле при своем движении будет увлекать за собой ротор электродвигателя (рис. 507). Чем быстрее будет вращаться ротор, тем меньшее число раз в единицу времени его будет пересекать магнитное поле, тем меньше будет индуктиро- ванный ток в его проводниках. В пределе, если бы ротор достиг равенства скорости с вращающимся магнитным полем, т. е. на- чал бы вращаться с ним синхронно, то в проводниках ротора не мог бы индуктироваться ток, так как они не пересекались бы магнитным полем. Но если индуктированный ток в обмотке ро- тора исчезнет, то вращающий момент двигателя станет равным нулю и число оборотов ротора будет спадать. Следовательно, ротор асинхронного двигателя никогда не может вращаться синхронно с магнитным полем, не исключая холостого хода, так как и в этом случае момент сопротивления сил не равен нулю (необходимо затрачивать энергию на враще- ние самого ротора и на потери холостого хода). В силу этого рассматриваемый нами электродвигатель и назван асинхронным, или несинхронным. Если трехфазный -асинхронный электродвигатель имеет три статорные катушки, то его вращающееся магнитное поле яв- ляется двухполюсном; число оборотов этого поля в минуту равно , (594) 767
где ni — число оборотов в минуту вращающегося магнитного 'Поля тока статора; fi — частота тока статора. Вращающееся магнитное поле тока статора индуктирует в обмотке ротора ток. В результате взаимодействия этого тока с вращающимся магнитным полем тока статора ротор вращается в ту же сторону, что и поле статора, но с несколько меньшей скоростью («2 < «1) • Частота f2 тока, индуктированного в обмотке ротора вращаю- щимся магнитным полем тока статора, зависит от разности чи- сел оборотов в секунду вращающегося магнитного поля тока статора и вращающегося ротора: (595) Из формулы (595) следует, что при п2 = 0, т. е. при непо- движном роторе, в обмотке последнего индуктируется ток, ча- стота которого равна частоте тока статора: f ni — nt f •'2 60 60 Если же «а «1, т. е. ротор вращается почти синхронно с магнитным полем, то частота тока в роторе практически равна нулю: f __ ni—ns _n /2 ~ 60 ~ U’ Следовательно, частота тока ротора f2 может изменяться практически от нуля до fi’ Ток ротора создает свое магнитное поле, которое вращается в ту же сторону, что и сам ротор, а» следовательно, в ту же сто- рону, что и вращающееся магнитное поле тока статора. Скорость вращения магнитного поля тока ротора относитель- но самого ротора зависит от частоты тока ротора: «р = 60/2, (596) где лр—число оборотов в минуту магнитного поля тока ро- тора; /2— частота тока ротора. Скорость вращения магнитного поля ротора относительно статора будет несколько больше, чем относительно самого ро- тора: «с = лр + па, (597) 768
где лс — число оборотов в минуту магнитного поля ротора от- носительно статора; я2 — число оборотов ротора в минуту; лр — число оборотов в минуту магнитного поля ротора относительно ротора. Но согласно предыдущему «г = 60 (/, — /2) и лр = 60 /2, а поэтому вместо формулы (597) можем написать «с = бО(/1-/2) + бо/2=бо/1, или, иначе, nz = пь (598) т. е. магнитное поле ток ротора вращается от- носительно статора стой же скоростью, что и магнитное поле тока статора. Следовательно, вращающиеся магнитные поля токов статора и ротора неподвижны в пространстве относительно друг друга. Если механическая нагрузка электродвигателя возрастет, то должно уменьшиться число оборотов ротора в минуту п2. Это повлечет за собой увеличение частоты f2 тока в роторе и соот- ветственно с этим числа оборотов в минуту лр магнитного поля ротора относительно «самого ротора. При этом насколько убы- вает величина п2, настолько возрастает величина лр, так что их сумма все время сохраняется постоянной по величине: яс = я2 + лр = const. Это значит, что при всяких изменениях механи- ческой нагрузки асинхронного электродвига- теля скорость вращения магнитного поля тока ротора относительно статора сохраняется не- изменной и равно йскорости вращения магнит- н^рго поля тока статора. Отношение разности скоростей вращения магнитных полей токов статора и "ротора электродвигателя к скорости вращения магнитного поля тока статора называется скольжением: 5= П1~Па-100%, (599) где S — скольжение в процентах; ni — число оборотов в минуту вращающегося магнитного поля тока статора; и2 — число оборотов в минуту ротора. Скольжение S, как величина, характеризующая запаздыва- ние скорости вращения ротора относительно скорости вращения 769
магнитного поля статора, играет важную роль при рассмотрении режима работы асинхронного электродвигателя. Из формулы (599) следует, что при «2 = 0, т. е. при непо- движном роторе, скольжение S равно 5 =-21^-. 100% = 100%. Следовательно, в начальный момент пуска в ход асинхрон- ного электродвигателя скольжение максимально и равно 100%'. При n2 ni, т. е. в том случае, когда ротор стремится достиг- нуть синхронной скорости, 5 = -!^^-.100%->0, т. е. скольжение стремится к нулю. Согласно формуле (595) f _ «1 — /2 60 ’ ' или, иначе, , ni — пг 60 ft 60 nt ’ так как «i = 60 ft. Отсюда, принимая во внимание, что 5 = -П1~П-, получим /2=^Л. (6009 т. е. частота тока /г в роторе асинхронного элек- тродвигателя равна произведению частоты ft тока статора на скольжение S. Пример 223. Скорость вращения двухполюсного магнитного поля асин- хронного электродвигателя равна п == 3000 об/мин. Определить частоту тока статора, частоту тока ротора и скорость вращения ротора, если скольжение S = 3%. Решение. Частота тока статора Частота тока ротора /2 = = 0,03*50 = 1,5 Скорость вращения ротора = 60 (Л —/2) = 60*(50— 1,5) = 2910 об/мин. Вращающий момент. На рис. 508 приведен график зависи- мости вращающего момента асинхронного электродвигателя от скольжения. При малых величинах скольжения вращающий мо- мент М электродвигателя увеличивается с ростом скольжения S, 7/0
при средних значениях скольжения он дости- гает некоторого макси- мума, а с дальнейшим ростом скольжения убы- вает. * При пуске в ход асинхронного ' электро- двигателя вращающий момент его относитель- но невелик, но все же он больше, чем момент сил сопротивления дви- жению (Л4>7ИС0П?). Это приводит к тому, что ротор электродвигателя начинает вращаться, постепенно увеличивая Рис. 508. Зависимость вращающего момента асинхронного электродвигателя от скольже- ния скорость своего дви- жения, а скольжение, наоборот, уменьшается. Затем по мере уменьшения скольжения и, следовательно, уве- личения числа оборотов ротора в минуту вращающий момент электродвигателя пройдет через максимум и начнет спадать по своей величине. Наконец, когда установится такое скольжение, при котором вращающий момент станет равным моменту сил со- противления движению, ротор электродвигателя будет вращаться с постоянным числом оборотов в минуту. Если механическая нагрузка электродвигателя увеличится и соответственно возрастет мо- мент сил сопротивления движению (Л4сопр > Л4), то число оборотов ротора нач- нет уменьшаться, а скольжу ние и вращающий момент электродвигателя — увели- чиваться. В конечном итоге в электрддвигателе вновь установится динамическое равновесие (А4 =± Мсопр), но уже при меньшем числе обо- ротов ротора в минуту и со- ответственно при возросшем скольжении. Максимальное значение вращающего момента асин- хронного электродвигателя не зависит от активного со- противления роторной цепи. Рис. 509. Графики зависимости вра- щающего момента асинхронного элек- тродвигателя от скольжения при раз- личных активных сопротивлениях ро- торной цепи 771
Рис. 510. Зависимость тока в роторе от скольжения Однако при изменении со- противления ротора изме- няется величина скольжения, при которой создается макси- мальнь№ вращающий мо- мент. Следовательно, путем изменения активного сопро- тивления можно добиться того, чтобы максимум вра- щающего момента электро- двигателя получался при любом заданном скольже- нии. На рис. 509 приведены графики зависимости вра- щающего момента асинхрон- ного электродвигателя от скольжения при различных активных сопротивлениях роторной цепи. Чем правее расположен максимум вращающего момента на графике, т. е. чем ближе он к максимуму скольжения, тем больший вращающий момент развивает электродвигатель при пуске в ход. На рис. 510 показана зависимость роторного тока /р от скольжения. Здесь мы видим, что с ростом скольжения ток в роторе электродвигателя растет. При пуске в ход электродви- гателя, когда скольжение равно максимуму, роторный ток также максимален. Наоборот, при холостом ходе электродвигателя, ког- да скольжение электродвигателя мало, роторный ток также мал. С ростом тока в роторе увеличивается и ток в статоре из-за размагничивающего действия ампер-витков ротора. Ток статора имеет две составляющие: активную и реактивную. Активная со- ставляющая обеспечивает механическую работу элек- тродвигателя, а реактив- ная составляющая создает вращающее магнитное по- ле и одновременно с этим ампер-витки, ‘компенсиру- ющие размагничивающее действие ампер-витков ро- торного тока (реакция якоря). Так как реактивная со- ставляющая тока статора относительно велика, то угол сдвига фаз тока и напряжения в статорной обмотке больше, чем в роторной. Соответственно Рис. '511. Рабочие характеристики асин- хронного электродвигателя 772
этому коэффициент мощности в цепи статора меньше, чем в цепи ротора: * cos <рст < COS (рр. На рис. 511 изображены рабочие характеристики асинхрон- ного электродвигателя, выражающие зависимость основных вели- чин, характеризующих его работу, от полезной механической мощности Рг, развиваемой им на валу. 1. Скольжение S электродвигателя увеличивается с ро- стом его полезной мощности. Оно имеет наименьшую величину при холостом ходе (примерно до 3%), а затем с ростом полез- ной мощности увеличивается, достигая при номинальной полез- ной мощности в среднем 5%. 2. Скорость вращения ротора электродвигателя п с ростом полезной мощности уменьшается; при номинальной по- лезной мощности она на 3-4-5% меньше скорости вращения магнитного поля. 3. Ток в статорной обмотке /ст растет с увеличе- нием полезной мощности электродвигателя. При холостом ходе он имеет минимальное значение, а затем по мере, увеличения полезной мощности растет. 4. Коэффициент полезного действия т) электро- двигателя равен отношению полезной мощности к электрической, потребляемой им из сети. При холостом ходе к. п. д. равен нулю, а затем с увеличением полезной мощности он растет. При номи- нальной полезной мощности к. п. д. достигает максимума, а за- тем начинает медленно спадать. Мощные асинхронные электро- двигатели имеют к. п. д. в среднем около 90%. Пуск в ход асинхронных электродвигателей с короткозамк- нутым ротором производится простым включением рубильника. В начальный момент, когда скольжение равно единице, в обмот- ках ротора и статора возникают очень большие токи (в полтора — два раза больше нормальных). Но так как пусковой период длится несколько секунд, то пусковые токи не опасны для элек- тродвигателя. . / Чтобы избежать появления чрезмерно больших пусковых то- ков, в мощных электродвигателях в цепь ротора на время пуска в ход включают реостат или же обмотки статора переключают со «звезды» на «треугольник». В первом случае перед пуском электродвигателя в ход при помощи рычага опускают щетки на кольца ротора и затем замыкают рубильник в цепи статора. Когда ротор начинает увеличивать скорость вращения, выводят сопротивление реостата ротора и, когда ротор разовьет нормаль- ную скорость, поднимают щетки, замыкая обмотку ротора нако- ротко. На рис. 505 показана схема асинхронного электродвига- теля с пусковым реостатом. 60-1377 773
При пуске в ход электродвигателя переключением его ста- торной обмотки с «треугольника» на «звезду» напряжение на каждой фазе статорной обмотки понижается в раз по срав- нению с линейным напряжением. Следовательно, пусковой ток в статорной обмотке и соответственно с этим и в роторной об- мотке уменьшается. После того как ротор электродвигателя ра- зовьет нормальную скорость вращения, протйвоэлектродвижущая сила достигнет определенной величины и ток уменьшится, ста- торную обмотку переключают на «треугольник». Тогда каждая Рис. 512. Схема приспособления для пуска асин- хронного электродвигателя путем переключения со „звезды* на „треугольник4* фаза статорной обмотки будет находиться под линейным напря- жением и в соответствии с этим электродвигатель будет рабо- тать в нормальном режиме. На рис. 512 показана схема приспособления для пуска асин- хронного электродвигателя путем переключения со «звезды» на «треугольник». Число оборотов ротора асинхронного электродвигателя можно регулировать путем включения реостата в цепь ротора, например пускового реостата, который может выдерживать продолжитель- ное время нормальный ток нагрузки двигателя. Увеличивая со- противление реостата, достигают увеличения скольжения или, иначе говоря, уменьшения числа оборотов ротора электродвига- теля. Этот способ регулировки нельзя назвать рациональным, так как он сильно влияет на к. п. д. двигателя. 7?4
Иногда вместо реостата для регулировки числа оборотов ро- тора пользуются трёхфазным трансформатором, включенным в статорную цепь. Изменяя напряжение, подаваемое на статор при помощи трансформатора, можно в известной степени изме- нять число оборотов ротора электродвигателя. Вращающий момент электродвигателя зависит от квадрата приложенного напряжения. Понижение сетевого напряжения до 30% может привести к полной остановке электродвигателя из-за малой величины вращающего момента. 1 2 Рис. 513. Электродвигатель изме- Рис. 514. Внешний вид асинхронного няет направление своего враще- электродвигателя ния при переключении двух линейных проводов у зажимов ротора Для изменения направления вращения асинхронного электро- двигателя достаточно поменять местами концы двух проводов, идущих от сети и подключенных к статорной обмотке электро- двигателя. В этом случае изменяет направление своего вращения вращающееся магнитное поле, а с ним вместе и ротор электро- двигателя (рис. 513). Коэффициент мощности (cos <?) асинхронного элек- тродвигателя сильно зависит от его нагрузки. При нормальной нагрузке он достигает максимального значения, а при малой на- грузке его величина падает до 0,1. Там, где работает много асин- хронных электродвигателей, проблема cos ср для питающих элек- трических станций приобретает немаловажное значение. Асинхронные электродвигатели просты по своему устройству, относительно дешевы, не требуют за собой особого ухода, удоб- ны в эксплуатации, имеют малую зависимость числа оборотов от нагрузки и т. п. Поэтому они широко применяются на прак- тике и являются одним из лучших типов электродвигателей пере- менного тока. На рис. 514 показан внешний вид асинхронного электродвигателя. so* -----------
ПРИЛОЖЕНИЕ 1 АБСОЛЮТНАЯ ПРАКТИЧЕСКАЯ ЭЛЕКТРОМАГНИТНАЯ РАЦИОНАЛИЗИРОВАННАЯ СИСТЕМА ЕДИНИЦ МКСА Наименования и определения единиц системы МКСА Наименование Сокращенное обозначение Определение русским шрифтом латинским (греческим) шрифтом Единица элек- трического тока ампер Единица .коли- чества электриче- ства кулон или ампер-секунда Единица разно- сти электриче- ских потенциалов, электрического напряжения и электродвижущей силы вольт а К а-сек в А С A-S V Неизменяющийся электрический ток, ко- торый, протекая по каждому из двух бесконечно длинных параллельных пря- молинейных проводников ничтожно малого кругового сечения, расположенных на расстоянии 2 м друг от друга в безвоз- душном пространстве, создает между этими проводниками на каждый метр их длины силу взаимодействия, которая массе в одну десятимиллионную (0,0000001) кило- грамма сообщает ускорение в 1 метр на секунду в квадрате Количество электричества, протекаю- щего через поперечное сечение проводника в течение 1 секунды при неизменяющемся токе в 1 ампер Разность электрических потенциалов между двумя точками линейного провод- ника, по которому протекает неизменяю- щийся электрический ток в 1 ампер, когда мощность, потребляемая между этими точ- ками, равна 1 ватту 776
Продолжение Наименование Сокращенное обозначение Определение русским шрифтом латинским (греческим) шрифтом Единица элек- трического сопро- тивления Электрическое сопротивление между двумя точками линейного проводника, в котором неизменяющаяся разность элек- ом Единица элек- трической емко- сти ОМ Q трических потенциалов между этими точ- ками в 1 вольт вызывает электрический ток в 1 ампер Электрическая емкость конденсатора, между обкладками которого существует разность электрических потенциалов фарада Единица маг- нитного потока ф F 1 вольт, когда он заряжен количеством электричества 1 кулон Магнитный поток, при убывании кото- рого до нуля количество электричества, вольт-секунда Единица маг- нитной индукции в-сек V-S протекающего через поперечное сечение неразветвленной линейной электрической цепи, сцепленной с этим потоком и имею- щей сопротивление в 1 ом, равно 1 кулону Магнитная. индукция, при которой маг- нитный поток через поверхность 1 квад- вольт-секунда на квадратный ратный метр, расположенную в равномер- ном магнитном поле перпендикулярно метр Единица индук- в-С'К/М* V-S/m2 к линиям магнитной индукции, равен 1 вольт-секунде Индуктивность электрической цепи, тивности и взаим- ной индуктивно- сти с которой сцепляется магнитный поток самоиндукции в 1 вольт-секунду при не- изменяющемся токе в цепи, равном 1 амперу генри гн Н Взаимная индуктивность двух электриче- ских цепей, с одной из которых сцепляется магнитный поток взаимной индукции в 1 вольт-секунду при неизменяющемся токе в другой цепи, равном 1 амперу Магнитодвижущая сила вдоль замкну- того контура, однократно сцепленного с электрической цепью, по которой про- Единица маг- нитодвижущей силы ампер или а А текает электрический ток в 1 ампер ампер-виток Единица напря- женности магнит- ного поля (магнит- ной силы) а-з A-w Напряженность магнитного поля в одно- родной и изотропной среде на расстоянии 1 тг— метра от бесконечно длинного прямо- Zn — ампер на метр а'м А/ш линейного проводника ничтожно малого кругового сечения, по которому протекает электрический ток в 1 ампер т
Il Соотношение между единицами абсолютной практической электромагнитной рационализированной системы МКСА и единицами абсолютных нерационализированных систем СГСЭ и СГСМ Величина Наименование единицы в системе единиц МКСА Обозначен! русскими буквами ие единицы латински- ми или греческими буквами Одна едиь мы МКСА в себе ед ционализ! сис СГСЭ {ица систе- . содержит иниц не ра- зронянных :тем СГСМ Длина 1. Основны Метр е величины м | m 102 (см) I 102 (см) Масса Килограмм кг kg 103 (г) 103 (г) Время Секунда сек. sec 1 (сек.) 1 (сек.) Ток Ампер а А 3-109 ю-1 Скорость 2» Механичес Метр в секунду кие единицы м/сек | m/sec 102 102 1 Ускорение Метр в секунду за одну секунду м/сек2 m/sec2 102 102 Сила Ньютон н N 105 (дин) 10s (дин) Энергия, работа Джоуль дж J 107(эрг) 107(эрг) Мощность Ватт вт W 107 107 Электрический 1 заряд 3. Электрйчес I Кулон :кие единицы к | С 3-10» I 1 ю-1 Электродвижу- щая сила, напря- жение, разность потенциалов Вольт в V 1 300 103 Напряженность электрического ноля Вольт на метр в ~м~ V m 1 300 108 Электрическая емкость 778 Фарада ф F 9 10“ 10"»
Продолжение Величина Наименование единицы в системе единиц МКСА Обозначение единицы Одна единица систе- мы МКСА содержит в себе единиц нера- ционализированных систем русскими буквами латински- ми или греческими буквами * СГСЭ СГСМ Плотность тока Ампер на квад- ратный метр а А ш2 3-10» Тр-в z Электрическое сопротивление Ом ом Q ± 10- 109 Удельное сопро- тивление Ом-метр ом-м Q ш ±Ю-. 10“ Удельная про- водимость Единица на ом-метр 1 ом-м 1 Qm 9-109 10-“ 4. Магнитные единицы Магнитный поток 1 Вольт-секунда в/сек V sec I 1 1 300 108 (мкс) Магнитная ин- дукция Вольт-секунда на квадратный метр в-сек V sec m2 4- io-® о 104 (гс) Напряженность магнитного поля Ампер на метр а м А m 4-3-107 4.10-2(э) Индуктивность (коэффициент са- моиндукции) Генри гн Н 4"10-11 109 Взаимоиндук- тивность (коэффи- циент взаимоин- дукции) Генри гн Н ± 10- 109 Магнитодвижу- щая сила Ампер-виток а-в A-w 3-109 10—* (г<У) Примечание. Большинство единиц систем СГСЭ и СГСМ специаль- ного наименования не имеет, за исключением некоторых наиболее употре- бительных магнитных единиц системы СГСМ: максвелл (мкс, Мх) — для единицы магнитного потока, гаусс (ас, Gs) — для единицы магнитной индукции, эрстед (э, Ое) — для единицы напряженности магнитного поля, гильберт (ztf, Gb)— для единицы магнитодвижущей силы. 779
Ill Международные единицы измерения электрических и магнитных величин В свое время были воспроизведены вещественные образцы существую- щих практических единиц измерения электрических и магнитных величин, и они были приняты в качестве эталонов в так называемой международной практической системе единиц. Между абсолютными единицами МКСА и международными единицами существуют следующие соотношения: 1 1 1 1 1 1 1 1 1 международный международный международный международный международный международный международная джоуль = 1,00020 абсолютного джоуля ватт = 1,00020 абсолютного ватта ампер = 0,99985 абсолютного ампера кулон = 0,99985 абсолютного кулона волы = 1,00035 абсолютного вольта ом = 1,00050 абсолютного ома фарада = 0,99950 абсолютной фарады международная вольт- секунда = 1,00035 абсолютной вольт-секунды международный генри = 1,00050 абсолютного генри Примечания: 1. Международный ампер —- величина неизменяюще- гося электрического тока, который отлагает на катоде 0,001118 грамма се- ребра в секунду, проходя через водный раствор азотнокислого серебра. 2. Международный ом — сопротивление, при неизменяющемся электри- ческом токе и при температуре тающего льда, ртутного столба .длиной 106,30 сантиметра, имеющего сечение, одинаковое по всей длине, и массу 14,4521 грамма. IV Кратные и дольные единицы Наименование кратных и дольных единиц образуется путем прибавле- ния соответствующих приставок к наименованиям единиц измерения вели- чин. В нижеследующей таблице приведены наименования этих приставок. Наименование Отношение к главной единице Сокращенное обозначение русским алфавитом латинским (греческим) алфавитом Пико . . • Ю-12 п Р Нано . . 10-9 Н П Микро . 10-е мк Р Милли . 10-а М m Санти . . 10-3 с с Деци . . io—1 д d Дека . . 10 дк de Гекто . . 102 г h Кило . . Ю3 к k Мега ♦ • 100 М M Гига . . 109 Г Q Тера • . 1012 Т T
ОГЛАВЛЕНИЕ Стр. Предисловие....................................................... 3 Буквенные обозначения некоторых электрических величин ............ 5 Условные графические изображения, применяемые в электрических схемах............................................................ 6 Введение ......................................................... 9 ЧАСТЬ ПЕРВАЯ ФИЗИЧЕСКИЕ ОСНОВЫ ЭЛЕКТРОТЕХНИКИ Глава I. Электрические свойства вещества.................... 17 § 1. Материя и ее движение.........♦..................... — § 2. Химические элементы................................ 21 § 3. Атомы............................................. 24 § 4. Элементарные частицы вещества....................... 25 § 5. Строение атомов..................................... 30 § 6. Энергия атомного ядра............................... 37 § 7 Молекулы и физические тела.......................... 43 § 8. Электрические проводники первого рода............... 46 § 9. Электрические проводники второго рода..........*. 49 § 10. Полупроводники..................................... 51 § 11. Диэлектрики........................................ 55 § 12. Электропроводимость газов.......................... 57 Глава II. Электрический заряд и единицы его измерения ... 58 § 13. Системы единиц измерения электрических и магнитных величин ................................................... — § 14. Электрический заряд физического тела................ 61 § 15. Единицы измерения электрического заряда............ 64 Глава III. Электрическое поле электрических зарядов.......... 66 § 16. Понятие об электрическом поле электрических зарядов — § 17. Электростатическое поле................................. 68 § 18. Потенциал электростатического поля...................... 69 § 19. Напряжение в электростатическом поле........... 73 § 20. Работа силы электростатического поля.................... 74 § 21 Напряженность электростатического поля................. 75 § 22. Закон Кулона . . . .................................... 78 Глава IV. Проводники и диэлектрики в электростатическом поле........................................................... 82 § 23. Проводники первого рода в электростатическом поле . . — § 24. Диэлектрики в электростатическом поле................. 86 781
Стр. Глава V. Электрическая емкость.........................♦ * . . 90 § 25. Электрическая емкость проводника и единицы ее изме- рения ...................................................... — § 26. Плоский конденсатор.................................. 92 § 27. Конденсаторы........................................ 94 § 28. Параллельное соединение конденсаторов............... 104 § 29. Последовательное соединение конденсаторов........... 106 § 30. Смешанное соединение конденсаторов.................. 109 § 31. Энергия электростатического поля.................... 111 § 32. Электрическая прочность диэлектриков ............... 113 Глава VI. Электрический ток.................................... 116 § 33. Направление и величина электрического тока ........... — § 34. Плотность тока...................................... 119 § 35. Постоянный ток и стационарное электрическое поле . . 120 § 36. Сторонние электрические поля........................ 122 Глава VII. Электродвижущая сила и напряжение источника электрической энергии ...................................... 125 § 37. Электродвижущая сила источника электрической энергии — § 38. Напряжение на участках электрической цепи ...... 127 Глава VIII. Электрическое сопротивлений........................ 131 § 39. Электрическое сопротивление проводников .............. — § 40. Зависимость сопротивления проводников от их размеров и материала......................................... 132 § 41. Проводимость проводников..................... 135 § 42. Зависимость сопротивления проводников от температуры 137 § 43. Реостаты и магазины сопротивлений............. 138 § 44. Сопротивления постоянной величины............. 142 § 45. Сопротивление полупроводников и диэлектриков .... 144 Глава IX. Простая цепь постоянного тока . . . ................. 145 § 46. Закон Ома для участка цепи..................... — § 47. Закон Ома для всей цепи....................... 147 § 48. Цепь постоянного тока с последовательно соединенными потребителями ............................................ 148 § 49. Распределение потенциала в электрической цепи .... 151 Глава X. Работа и мощность электрического тока................. 155 § 50. Работа электрического тока........................... — § 51. Мощность электрического тока....................... 157 § 52. Исследование режима в цепи с источником электрической энергии, имеющим постоянную электродвижущую силу и постоянное внутреннее сопротивление..................... 159 Глава XI. Цепь постоянного тока с параллельно и смешанно соединенными сопротивлениями................................ 167 § 53. Первый закон Кирхгофа................................. — § 54. Напряжение на зажимах параллельно соединенных сопро- тивлений ................................................ 168 § 55. \распределение токов в параллельно соединенных сопро- тивлениях ................................................ 170 § 56 Эквивалентное сопротивление нескольких параллельно соединенных потребителей............................. 171 § 57. Потенциометр........................................ 174 § 58. Цепи постоянного тока со смешанно соединенными сопро- тивлениями и одним источником электрической энергии — 782
Стр. Глава XII. Сложные цепи постоянного тока.............................. 179 § 59. Второй закон Кирхгофа........................................ — § 60. Применение законов Кирхгофа к расчету сложных цепей 181 § 61. Метод контурных токов...................................... 190 Глава XIII. Тепловое действие электрического тока..................... 196 § 62. Термический эквивалент ...................................... — § 63. Закон Ленца — Джоуля....................................... 197 § 64. Плавкие предохранители..................................... 198 § 65. Электрические лампы.................................... . 200 § 66. Электрическая дуга......................................... 201 Глава XIV. Магнитное поле............................................. 204 § 67. Понятие о магнитном поле..................................... — § 68. Магнитная индукция.................. ..................... 207 § 69. Магнитная проницаемость..................................... 2Ю § 70. Магнитный поток............................................ 212 § 71. Напряженность магнитного поля.............................. 215 Глава XV. Магнитное поле постоянного тока в однородной среде................................................... 217 § 72. Магнитное поле постоянного тока, протекающего по пря- молинейному проводу . . . '................................ — § 73. Магнитное поле кольцевого тока............................. 220 § 74. Магнитное поле однослойной цилиндрической катушки 221 Глава XVI. Магнитная цепь со сталью................................... 224 § 75. Намагничивание парамагнитных и диамагнитных веществ — | 76. Намагничивание ферромагнитных веществ...................... 228 § 77. Циклическое перемагничивание............................... 233 § 78. Магнитные материалы........................................ 236 § 79. Законы для магнитных цепей ................................ 241 § 80. Расчет простой магнитной цепи............................ 248 § 81. Расчет разветвленной магнитной цепи ....................... 253 § 82. Электромагниты............................................. 256 Глава XVII. Механическое действие магнитного поля на провод- ники с токами ........................................ 259 § 83. Механическое действие магнитного поля на прямолиней- ный проводник с током................................ — § 84. Механическое действие магнитного поля на контур с током 263 § 85. Механическая сила взаимодействия проводников с токами 265 § 86 Работа сил магнитного поля............................ 267 Глава XVIII. Электромагнитная индукция................................ 269 § 87. Индуктированная электродвижущая сила в прямолиней- ном проводнике, перемещающемся в магнитном поле . . — § 88. Индуктированная электродвижущая сила в замкнутом контуре и катушке................................................ 274 § 89. Электродвижущая сила самоиндукции.......................... 278 § 90. Индуктивность цилиндрических катушек....................... 284 § 91. Электродвижущая сила взаимоиндукции........................ 288 § 92 Взаимная индуктивность двух катушек........................ 290 § 93. Последовательное и параллельное соединение катушек индуктивности ................................................... 293 § 94. Энергия магнитного поля ................................... 295 783
Стр. ЧАСТЬ ВТОРАЯ ПЕРЕМЕННЫЕ ТОКИ Глава XIX. Основные положения теории переменного тока . . 298 § 95. Общее понятие о переменном токе . .—................ — § 96. Некоторые основные определения переменного тока . . 299 § 97. Переменный синусоидальный ток...................... 301 § 98. Простейший способ получения синусоидальной электро- движущей силы............................................. 305 § 99. Векторная диаграмма синусоидального тока........... 308 § 100. Действующее значение переменного синусоидального тока...................................................... 313 Глава XX: Простая цепь переменного тока с индуктивностью и активным сопротивлением................................. 316 § 101. Активное сопротивление — § 102. Цепь переменного тока с активным сопротивлением . . 317 § 103. Индуктивность в цепи переменного тока.............. 320 § 104. Цепь переменного тока с индуктивностью и активным сопротивлением............................................ 330 § 105. Цепь с несколькими последовательно соединенными активными сопротивлениями и индуктивностями .... 337 Глава XXI. Простая цепь переменного тока с емкостью и актив- ным сопротивлением........................................ 340 § 106. Подключение конденсатора к источнику электрической энергии с постоянным напряжением и разряд конденса- тора через активное сопротивление .......................... — t § 107. Цепь переменного тока с емкостью...................... 344 § 108. Цепь переменного тока с емкостью и активным сопро- тивлением ................................................ 351 Глава XXII. Простая цепь переменного тока с последовательно соединенными индуктивностью, емкостью и активным сопро- тивлением ............................................... 358 § 109. Колебательный и апериодический разряд конденсатора — § 110. Вынужденные колебания в цепи с индуктивностью, емкостью и активным сопротивлением, соединенными последовательно .......................................... 372 § 111. Резонанс напряжений . . . ......................... 380 Глава XXIII. Цепи переменного тока с параллельно соединен- ными потребителями......................................... 394 § 112. Цепь переменного тока с двумя параллельно соединен- ными катушками индуктивности ............................... — § ИЗ. Активный и реактивный ток.......................... 398 § 114. Активная и реактивная проводимости................. 400 § 115. Определение сопротивления, угла сдвига фаз, мощности и величины тока в цепи посредством ее проводимостей 403 § 116. Применение метода проводимостей к расчету цепи с параллельно соединенными катушками индуктивности 404 § 117. Цепь переменного тока с параллельно соединенными индуктивностью и емкостью................................ 407 § 118. Резонанс токов....................................... 415 Глава XXIV. Применение метода комплексных чисел в теории переменных токов.......................................... 433 § 119. Общие замечания....................................... — § 120. Мнимое число . . . . ............................. 434 784
Стр. § 121. Комплексное число..................................... 436 § 122. Действия над комплексными числами ......... 439 § 123. Комплексы синусоидального тока и напряжения .... 444 § 124. Сопротивления цепей переменного тока в комплексной форме................................................... 447 § 125. Проводимости цепей переменного тока в комплексной форме................................................. 450 § 126. Мощность синусоидального тока в комплексной форме 452 § 127. Основные законы переменного тока в комплексной форме................................................... 454 § 128. Примеры расчета цепей переменного тока при помощи комплексного метода............................... 456 Глава XXV. Многофазные системы.................................. 462 § 129. Трехфазная система............................ —• § 130. Трехфазная система, соединенная „звездой”. 466 § 131. Трехфазная система, соединенная „треугольником” . . . 473 § 132. Мощность трехфазного тока.................... 477 § 133. Вращающееся магнитное поле трехфазного тока .... 478 § 134. Двухфазное вращающееся магнитное поле................. 483 Глава XXVI. Катушки и трансформаторы со стальными сердеч- никами ................................................... 486 § 135. Общие положения....................................... — § 136. Мощность потерь на пёремагничивание стали........... 487 § 137. Мощность потерь на вихревые токи.................... 488 § 138. Катушка со стальным сердечником в цепи переменного § 139. Общее понятие о трансформаторах................ 499 § 140. Работа трансформатора при холостом ходе........ 501 § 141. Работа трансформатора при нагрузке............. 504 § 142. Векторная диаграмма нагруженного трансформатора . ♦ 507 § 143. Внешняя характеристика трансформатора.......... 511 § 144. Коэффициент полезного действия трансформатора . . . 513 § 145. Типы трансформаторов........................... 514 § 146. Автотрансформатор.............................. 517 Глава XXVII. Цепь переменного тока с приложенным несинусои- дальным напряжением......................................... 519 § 147. Применение ряда Фурье к исследованию сложных перио- дических колебаний .......................................... — § 148. Режим в цепи при приложенном к ней несинусоидаль- ном напряжении............................................. 525 § 149. Резонанс напряжений в цепи при несинусоидальном приложенном напряжении.................................... 529 § 150. Понятие об электрических фильтрах................... 531 Глава XXVIII. Выпрямители переменного тока................... 535 § 151. Общие положения.................................... — § 152. Кенотрон......................................... 536 § 153. Ртутные выпрямители.............................. 544 § 154. Медно-закисный выпрямитель....................... 547 § 155. Селеновый выпрямитель............................ 548 § 156. Вибрационный преобразователь..................... 550 Глава XXIX. Электромагнитное поле............................. 553 § 157. Краткие сведения из истории развития учения об элек- тромагнитном поле . ..................................... — § 158. Электромагнитное поле ............ 555 785
Стр. § 159. Свободное электромагнитное поле...........; . . . 559 § 160. Энергия электромагнитного поля ................... 563 § 161. Плоские электромагнитные волны в однородном диэлек- трике .................................................... 565 § 162. Плоские волны в проводящей среде.................. 570 § 163. Стационарное электромагнитное поле................ 574 § 164. Поверхностный эффект............./................ 579 ЧАСТЬ третья ЭЛЕКТРИЧЕСКИЕ ИЗМЕРЕНИЯ Глава XXX. Электрические измерительные приборы................. 582 § 165. Значения электрических измерений ................... — § 166. Классификация электроизмерительных приборов по принципу их действия...................................... 583 § 167. Классификация электроизмерительных приборов по роду измеряемой величины............................•.......... 585 § 168. Классификация электроизмерительных приборов по сте- пени их точности ......................................... — J169. Чувствительность и постоянная прибора............... 586 170. Мощность потерь энергии в приборах................ 587 171. Приборы магнитоэлектрической системы................ — § 172. Приборы электродинамической системы............... 591 § 173. Приборы электромагнитной системы.................. 595 § 174. Приборы электростатической системы................ 597 § 175. Приборы термоэлектрической системы................ 598 § 176. Приборы вибрационной системы...................... 601 § 177. Электродинамические счетчики..................... 602 § 178. Индукционные счетчики....................... . . . 604 Глава XXXI. Электрические измерения............................ 606 § 179. Измерение величины тока............................... — § 180. Измерение напряжения................................ 609 § 181. Измерение сопротивлений методом амперметра и вольт- метра .................................................... 611 § 182. Измерение больших сопротивлений методом вольтметра 614 § 183. Измерение сопротивлений омметром.................... 616 § 184. Измерение сопротивлений методом мостика ............ 61$ § 185. Типы мостиков....................................... 621 § 186. Измерение мощности электрического тока............ . 627 § 187. Измерение электрической энергии.................... 629 ЧАСТЬ ЧЕТВЕРТАЯ ХИМИЧЕСКИЕ ИСТОЧНИКИ ЭЛЕКТРИЧЕСКОЙ ЭНЕРГИИ Глава XXXII. Аккумуляторы..................................... 631 § 188. Принцип действия аккумуляторов...................... — § 189. Щелочные аккумуляторы............................. 637 § 190. Эксплуатация щелочных аккумуляторов............... 645 § 191. Кислотные (свинцовые) аккумуляторы..............." 648 § 192. Эксплуатация кислотных аккумуляторов.............. 656 Глава XXXIII. Гальванические элементы........................ 660 § 193. Общие положения................................... — § 194. Гальванические элементы с марганцовой деполяризацией 661 § 195. Гальванические элементы с марганцово-воздушной де- поляризацией .......................................... 666 786
Стр. Глава XXXIV. Соединение химических источников электриче- ской энергии в батареи............................... . . 668 § 197. Последовательное соединение химических источников электрической энергии (элементов) ........................ 669 § 198. Параллельное соединение химических источников элек- трической энергии (элементов) ............................ 672 § 199. Смешанное соединение химических источников электри- ческой энергии (элементов)............................... 674 § 200. Аккумуляторные батареи...........................• . 677 § 201. Заряд аккумуляторных батарей....................... 679 § 202. Батареи гальванических элементов ................... 681 § 203. Анодные батарей галетно-пленочной конструкции . . . 682 часть пятая ЭЛЕКТРИЧЕСКИЕ МАШИНЫ Глава XXXV. Генераторы постоянного тока.......................... 684 § 204. Общие сведения ...................................... — § 205. Принцип действия генераторов постоянного тока ... — § 206. Основные части генератора постоянного тока....... 687 § 207. Обмотки генераторов постоянного тока................ 690 § 208. Реакция якоря....................................... 694 § 209. Электродвижущая сила генератора постоянного тока 696 § 210. Мощность и коэффициент полезного действия генерато- ров постоянного тока..................................... 697 § 211. Классификация генераторов постоянного тока пр спо- собу их возбуждения и понятие об их основных харак- теристиках .............................................. 699 § 212. Генератор постоянного тока с независимым возбужде- нием .................................................... 700 § 213. Генератор с параллельным возбуждением....... 705 § 214. Генератор с последовательным возбуждением ..... 710 § 215. Генератор со смешанным возбуждением......... 711 § 216. Параллельная работа генераторов постоянного тока . . 716 § 217. Параллельная работа генератора постоянного тока с ак- кумуляторной батареей . ................................. 718 Глава XXXVI. Электродвигатели постоянного тока................... 723 § 218. Принцип действия электродвигателя постоянного тока — § 219. Вращающий момент, мощность и коэффициент полез- ного действия электродвигателя . ........................ 725 § 220. Протйвоэлектродвижущая сила электродвигателя .... 727 § 221. Реакция якоря электродвигателя...................... 729 § 222. Влияние механической нагрузки на режим работы элек- тродвигателя ........................................... 730 ' § 223. Электродвигатель с последовательным возбуждением 732 § 224. Электродвигатель с параллельным возбуждением . . . 736 § 225. Электродвигатель со смешанным возбуждением .... 741 Глава XXXVII. Генераторы переменного тока . . ................... 743 § 226. Устройство синхронных генераторов . .................. — § 227. Режим работы синхронного генератора................. 747 § 228. Параллельная работа синхронных генераторов....... 752 Глава XXXVIII. Электродвигатели переменного тока............. 758 229. Синхронный электродвигатель........................... — 230. Асинхронный электродвигатель........................ 763 риложение........................................... . . 776
Дмитрий Георгиевич Максимов КУРС ЭЛЕКТРОТЕХНИКИ Редактор полковник Кириенко /7. С. Технический редактор Соломоник Р. Л, Корректор Рослова Н. /7. Сдано в набор 14.5.57 г. Подписано к печати 9.1.58 г, Формат бумаги 60X92</ie 49* U печ. л. = 49,25 усл. печ. л. 46,344 уч.-изд. л. Г-4103) Военное Издательство Министерства обороны Союза ССР Москва, Тверской бульвар, 18 >. Изд. № 5/7717 Зак. № 137? 2-я типография им. К. Е. Ворошилова Управления Военного Издательства Министерства обороны СССР Ленинград — Центр-1, Дворцовая пл., 10 Цена 14 р. 90 к.
I I1 i